You are on page 1of 529

Chapterwise Topicwise

Solved Papers
2021-1979

IITJEE
JEE Main & Advanced

Chemistry
Ranjeet Shahi

Arihant Prakashan (Series), Meerut


Arihant Prakashan (Series), Meerut
All Rights Reserved

© Author

Administrative & Production Offices


Regd. Office
‘Ramchhaya’ 4577/15, Agarwal Road, Darya Ganj, New Delhi -110002
Tele: 011- 47630600, 43518550

Head Office
Kalindi, TP Nagar, Meerut (UP) - 250002, Tel: 0121-7156203, 7156204

Sales & Support Offices


Agra, Ahmedabad, Bengaluru, Bareilly, Chennai, Delhi, Guwahati,
Hyderabad, Jaipur, Jhansi, Kolkata, Lucknow, Nagpur & Pune.

ISBN 978-93-25796-14-0
PO No : TXT-XX-XXXXXXX-X-XX
Published by Arihant Publications (India) Ltd.
For further information about the books published by Arihant, log on to
www.arihantbooks.com or e-mail at info@arihantbooks.com
Follow us on
CONTENTS
1. Some Basic Concepts of Chemistry 1-22 19. Extraction of Metals 282-293

2. Atomic Structure 23-40 20. Qualitative Analysis 294-306

3. Periodic Classification and 21. Organic Chemistry Basics 307-331


Periodic Properties 41-47
22. Hydrocarbons 332-349
4. Chemical Bonding 48-66
23. Alkyl Halides 350-363
5. States of Matter 67-83
24. Alcohols and Ethers 364-377
6. Chemical and Ionic Equilibrium 84-108
25. Aldehydes and Ketones 378-396
7. Thermodynamics and
Thermochemistry 109-129 26. Carboxylic Acids and
their Derivatives 397-412
8. Solid State 130-139
27. Aliphatic Compounds
9. Solutions and Colligative Properties 140-155 Containing Nitrogen 413-422

10. Electrochemistry 156-177 28. Benzene and Alkyl Benzene 423-440

11. Chemical Kinetics 178-195 29. Aromatic Compounds


Containing Nitrogen 441-457
12. Nuclear Chemistry 196-199
30. Aryl Halides and Phenols 458-470
13. Surface Chemistry 200-206
31. Aromatic Aldehydes, Ketones
14. s-Block Elements 207-217 and Acids 471-484
15. p-Block Elements-I 218-227 32. Biomolecules and Chemistry
16. p-Block Elements-II 228-248 in Everyday Life 485-501

17. Transition and 33. Environmental Chemistry 502-504


Inner-Transition Elements 249-258
JEE Advanced Solved Paper 2021 1-16
18. Coordination Compounds 259-281
SYLLABUS
JEE MAIN
Section A : PHYSICAL CHEMISTRY
UNIT I Some Basic Concepts in Chemistry Quantum mechanical approach to covalent bonding Valence bond
Matter and its nature, Dalton's atomic theory; Concept of atom, theory - Its important features, concept of hybridization involving s, p
molecule, element and compound; Physical quantities and their and d orbitals; Resonance.
measurements in Chemistry, precision and accuracy, significant Molecular Orbital Theory Its important features, LCAOs, types of
figures, S.I. Units, dimensional analysis; Laws of chemical molecular orbitals (bonding, antibonding), sigma and pi-bonds,
combination; Atomic and molecular masses, mole concept, molar molecular orbital electronic configurations of homonuclear diatomic
mass, percentage composition, empirical and molecular formulae; molecules, concept of bond order, bond length and bond energy.
Chemical equations and stoichiometry.
Elementary idea of metallic bonding. Hydrogen bonding and its
UNIT II States of Matter applications.
Classification of matter into solid, liquid and gaseous states. UNIT V Chemical Thermodynamics
Gaseous State Measurable properties of gases; Gas laws - Boyle's law, Fundamentals of thermodynamics System and surroundings, extensive
Charle's law, Graham's law of diffusion, Avogadro's law, Dalton's law of and intensive properties, state functions, types of processes.
partial pressure; Concept of Absolute scale of temperature; Ideal gas
First law of thermodynamics Concept of work, heat internal energy and
equation, Kinetic theory of gases (only postulates); Concept of average,
enthalpy, heat capacity, molar heat capacity, Hess's law of constant heat
root mean square and most probable velocities; Real gases, deviation
summation; Enthalpies of bond dissociation, combustion, formation,
from Ideal behaviour, compressibility factor, van der Waals' equation,
atomization, sublimation, phase transition, hydration, ionization and
liquefaction of gases, critical constants.
solution.
Liquid State Properties of liquids - vapour pressure, viscosity and
Second law of thermodynamics Spontaneity of processes; ΔS of the
surface tension and effect of temperature on them (qualitative
universe and ΔG of the system as criteria for spontaneity, ΔGo
treatment only).
(Standard Gibb's energy change) and equilibrium constant.
Solid State Classification of solids: molecular, ionic, covalent and
metallic solids, amorphous and crystalline solids (elementary idea); UNIT VI Solutions
Bragg's Law and its applications, Unit cell and lattices, packing in solids Different methods for expressing concentration of solution - molality,
(fcc, bcc and hcp lattices), voids, calculations involving unit cell molarity, mole fraction, percentage (by volume and mass both), vapour
parameters, imperfection in solids; electrical, magnetic and dielectric pressure of solutions and Raoult's Law - Ideal and non-ideal solutions,
properties. vapour pressure - composition plots for ideal and non-ideal solutions.
UNIT III Atomic Structure Colligative properties of dilute solutions - relative lowering of vapour
pressure, depression of freezing point, elevation of boiling point and
Discovery of sub-atomic particles (electron, proton and neutron);
osmotic pressure; Determination of molecular mass using colligative
Thomson and Rutherford atomic models and their limitations; Nature of
properties; Abnormal value of molar mass, van't Hoff factor and its
electromagnetic radiation, photoelectric effect; spectrum of hydrogen
significance.
atom, Bohr model of hydrogen atom - its postulates, derivation of the
relations for energy of the electron and radii of the different orbits, UNIT VII Equilibrium
limitations of Bohr's model; dual nature of matter, de-Broglie's Meaning of equilibrium, concept of dynamic equilibrium.
relationship, Heisenberg uncertainty principle.
Equilibria involving physical processes Solid -liquid, liquid - gas and
Elementary ideas of quantum mechanics, quantum mechanical model solid - gas equilibria, Henry's law, general characteristics of equilibrium
of atom, its important features, involving physical processes.
ψ and ψ2, concept of atomic orbitals as one electron wave functions;
Variation of ψ and ψ2 with r for 1s and 2s orbitals; various quantum Equilibria involving chemical processes Law of chemical equilibrium,
numbers (principal, angular momentum and magnetic quantum equilibrium constants (K and K) and their significance, significance of
numbers) and their significance; shapes of s, p and d - orbitals, electron ΔG and ΔGo in chemical equilibria, factors affecting equilibrium
spin and spin quantum number; rules for filling electrons in orbitals – concentration, pressure, temperature, effect of catalyst; Le -Chatelier's
aufbau principle, Pauli's exclusion principle and Hund's rule, principle.
electronic configuration of elements, extra stability of half-filled and Ionic equilibrium Weak and strong electrolytes, ionization of
completely filled orbitals. electrolytes, various concepts of acids and bases (Arrhenius, Bronsted -
Lowry and Lewis) and their ionization, acid-base equilibria (including
UNIT IV Chemical Bonding and Molecular Structure
multistage ionization) and ionization constants, ionization of water, pH
Kossel Lewis approach to chemical bond formation, concept of ionic scale, common ion effect, hydrolysis of salts and pH of their solutions,
and covalent bonds. solubility of sparingly soluble salts and solubility products, buffer
Ionic Bonding Formation of ionic bonds, factors affecting the formation solutions.
of ionic bonds; calculation of lattice enthalpy.
UNIT VIII Redox Reactions and Electrochemistry
Covalent Bonding Concept of electronegativity, Fajan's rule, dipole Electronic concepts of oxidation and reduction, redox reactions,
moment; Valence Shell Electron Pair Repulsion (VSEPR) theory and oxidation number, rules for assigning oxidation number, balancing of
shapes of simple molecules. redox reactions.
Eectrolytic and metallic conduction, conductance in electrolytic temperature on rate of reactions - Arrhenius theory, activation energy
solutions, specific and molar conductivities and their variation with and its calculation, collision theory of bimolecular gaseous reactions
concentration: Kohlrausch's law and its applications. (no derivation).
Electrochemical cells - Electrolytic and Galvanic cells, different types of UNIT X Surface Chemistry
electrodes, electrode potentials including standard electrode Adsorption - Physisorption and chemisorption and their
potential, half - cell and cell reactions, emf of a Galvanic cell and its characteristics, factors affecting adsorption of gases on solids-
measurement; Nernst equation and its applications; Relationship Freundlich and Langmuir adsorption isotherms, adsorption from
between cell potential and Gibbs' energy change; Dry cell and lead solutions.
accumulator; Fuel cells; Corrosion and its prevention.
Catalysis Homogeneous and heterogeneous, activity and selectivity of
UNIT IX Chemical Kinetics solid catalysts, enzyme catalysis and its mechanism.
Rate of a chemical reaction, factors affecting the rate of reactions Colloidal state distinction among true solutions, colloids and
concentration, temperature, pressure and catalyst; elementary and suspensions, classification of colloids - lyophilic, lyophobic; multi
complex reactions, order and molecularity of reactions, rate law, rate molecular, macromole-cular and associated colloids (micelles),
constant and its units, differential and integral forms of zero and first preparation and properties of colloids Tyndall effect, Brownian
order reactions, their characteristics and half - lives, effect of movement, electrophoresis, dialysis, coagulation and
flocculation; Emulsions and their characteristics.

Section B : INORGANIC CHEMISTRY


UNIT XI Classification of Elements and Group 16 Preparation, properties, structures and uses of dioxygen
Periodicity in Properties and ozone; Allotropic forms of sulphur; Preparation, properties,
Periodic Law and Present Form of the Periodic Table, s, p, d and f Block structures and uses of sulphur dioxide, sulphuric acid (including
Elements, Periodic Trends in Properties of Elementsatomic and Ionic Radii, its industrial preparation); Structures of oxoacids of sulphur.
Ionization Enthalpy, Electron Gain Enthalpy, Valence, Oxidation States and Group 17 Preparation, properties and uses of chlorine and
Chemical Reactivity. hydrochloric acid; Trends in the acidic nature of hydrogen halides;
Structures of Interhalogen compounds and oxides and oxoacids of
UNIT XII General Principles and Processes of Isolation of Metals halogens.
Modes of occurrence of elements in nature, minerals, ores; steps involved Group 18 Occurrence and uses of noble gases; Structures of
in the extraction of metals - concentration, reduction (chemical and fluorides and oxides of xenon.
electrolytic methods) and refining with special reference to the extraction
of Al, Cu, Zn and Fe; Thermodynamic and electrochemical principles UNIT XVI d–and f–Block Elements
involved in the extraction of metals. Transition Elements General introduction, electronic
configuration, occurrence and characteristics, general trends in
UNIT XIII Hydrogen properties of the first row transition elements - physical properties,
Position of hydrogen in periodic table, isotopes, preparation, properties ionization enthalpy, oxidation states, atomic radii, colour, catalytic
and uses of hydrogen; physical and chemical properties of water and behaviour, magnetic properties, complex formation, interstitial
heavy water; Structure, preparation, reactions and uses of hydrogen compounds, alloy formation; Preparation, properties and uses of
peroxide; Classification of hydrides ionic, covalent and interstitial; K2 Cr2 O7 and KMnO4.
Hydrogen as a fuel. Inner Transition Elements
UNIT XIV s - Block Elements (Alkali and Alkaline Earth Metals) Lanthanoids - Electronic configuration, oxidation states, chemical
reactivity and lanthanoid contraction. Actinoids - Electronic
Group 1 and 2 Elements configuration and oxidation states.
General introduction, electronic configuration and general trends in
UNIT XVII Coordination Compounds
physical and chemical properties of elements, anomalous properties of the
first element of each group, diagonal relationships. Introduction to coordination compounds, Werner's theory; ligands,
coordination number, denticity, chelation; IUPAC nomenclature of
Preparation and properties of some important compounds - sodium mononuclear coordination compounds, isomerism; Bonding
carbonate, sodium chloride, sodium hydroxide and sodium hydrogen Valence bond approach and basic ideas of Crystal field theory,
carbonate; Industrial uses of lime, limestone, Plaster of Paris and cement; colour and magnetic properties; importance of coordination
Biological significance of Na, K, Mg and Ca. compounds (in qualitative analysis, extraction of metals and in
biological systems).
UNIT XV p - Block Elements
Group 13 to Group 18 Elements UNIT XVIII Environmental Chemistry
General Introduction Electronic configuration and general trends in Environmental pollution Atmospheric, water and soil.
physical and chemical properties of elements across the periods and Atmospheric pollution - Tropospheric and stratospheric.
down the groups; unique behaviour of the first element in each Tropospheric pollutants Gaseous pollutants Oxides of carbon,
group.Group wise study of the p – block elements nitrogen and sulphur, hydrocarbons; their sources, harmful effects
Group 13 Preparation, properties and uses of boron and aluminium; and prevention; Green house effect and Global warming; Acid rain;
structure, properties and uses of borax, boric acid, diborane, boron Particulate pollutants Smoke, dust, smog, fumes, mist; their
trifluoride, aluminium chloride and alums. sources, harmful effects and prevention.
Group 14 Tendency for catenation; Structure, properties and uses of Stratospheric pollution Formation and breakdown of ozone,
allotropes and oxides of carbon, silicon tetrachloride, silicates, zeolites and depletion of ozone layer - its mechanism and effects.
silicones. Water pollution Major pollutants such as, pathogens, organic
wastes and chemical pollutants their harmful effects and
Group 15 Properties and uses of nitrogen and phosphorus; Allotrophic
prevention.
forms of phosphorus; Preparation, properties, structure and uses of
ammonia nitric acid, phosphine and phosphorus halides,(PCl3, PCl5); Soil pollution Major pollutants such as: Pesticides (insecticides,
Structures of oxides and oxoacids of nitrogen and phosphorus. herbicides and fungicides), their harmful effects and prevention.
Strategies to control environmental pollution.
Section C : ORGANIC CHEMISTRY
UNIT XIX Purification & Characterisation of Organic Compounds UNIT XXIV Organic Compounds Containing Nitrogen
Purification Crystallisation, sublimation, distillation, differential General methods of preparation, properties, reactions and uses.
extraction and chromatography principles and their applications. Amines Nomenclature, classification, structure basic character and
Qualitative analysis Detection of nitrogen, sulphur, phosphorus and identification of primary, secondary and tertiary amines and their basic
halogens. character.
Quantitative analysis (basic principles only) Estimation of carbon, Diazonium Salts Importance in synthetic organic chemistry.
hydrogen, nitrogen, halogens, sulphur, phosphorus.
UNIT XXV Polymers
Calculations of empirical formulae and molecular formulae;
Numerical problems in organic quantitative analysis. General introduction and classification of polymers, general methods
of polymerization-addition and condensation, copolymerization;
UNIT XX Some Basic Principles of Organic Chemistry Natural and synthetic rubber and vulcanization; some important
Tetravalency of carbon; Shapes of simple molecules hybridization polymers with emphasis on their monomers and uses - polythene,
(s and p); Classification of organic compounds based on functional nylon, polyester and bakelite.
groups: —C=C—,—C=C— and those containing halogens, oxygen,
nitrogen and sulphur, Homologous series; Isomerism - structural and UNIT XXVI Biomolecules
stereoisomerism. General introduction and importance of biomolecules.
Nomenclature (Trivial and IUPAC) Carbohydrates Classification aldoses and ketoses; monosaccharides
Covalent bond fission Homolytic and heterolytic free radicals, (glucose and fructose), constituent monosaccharides of
carbocations and carbanions; stability of carbocations and free radicals, oligosacchorides (sucrose, lactose, maltose) and polysaccharides
electrophiles and nucleophiles. (starch, cellulose, glycogen).
Electronic displacement in a covalent bond Inductive effect, Proteins Elementary Idea of α-amino acids, peptide bond, .
electromeric effect, resonance and hyperconjugation. polypeptides; proteins: primary, secondary, tertiary and quaternary
Common types of organic reactions Substitution, addition, structure (qualitative idea only), denaturation of proteins, enzymes.
elimination and rearrangement. Vitamins Classification and functions.
UNIT XXI Hydrocarbons Nucleic Acids Chemical constitution of DNA and RNA. Biological
Classification, isomerism, IUPAC nomenclature, general methods of functions of Nucleic acids.
preparation, properties and reactions. UNIT XXVII Chemistry in Everyday Life
Alkanes Conformations: Sawhorse and Newman projections (of Chemicals in medicines Analgesics, tranquilizers, antiseptics,
ethane); Mechanism of halogenation of alkanes. disinfectants, antimicrobials, antifertility drugs, antibiotics, antacids,
Alkenes Geometrical isomerism; Mechanism of electrophilic addition: antihistamins - their meaning and common examples.
addition of hydrogen, halogens, water, hydrogen halides Chemicals in food Preservatives, artificial sweetening agents - common
(Markownikoff's and peroxide effect); Ozonolysis, oxidation, and examples.
polymerization.
Cleansing agents Soaps and detergents, cleansing action.
Alkenes acidic character; addition of hydrogen, halogens, water and
hydrogen halides; polymerization. Unit XXVIII Principles Related to
Aromatic hydrocarbons Nomenclature, benzene structure and Practical Chemistry
aromaticity; Mechanism of electrophilic substitution: halogenation, — Detection of extra elements (N, S, halogens) in organic
nitration, Friedel – Craft's alkylation and acylation, directive influence of compounds; Detection of the following functional groups:
functional group in mono-substituted benzene. hydroxyl (alcoholic and phenolic), carbonyl (aldehyde and ketone),
UNIT XXII Organic Compounds Containing Halogens carboxyl and amino groups in organic compounds.
General methods of preparation, properties and reactions; Nature of Chemistry involved in the preparation of the following
C—X bond; Mechanisms of substitution reactions. — Inorganic compounds Mohr's salt, potash alum.
Uses/environmental effects of chloroform, iodoform, freons and DDT. — Organic compounds Acetanilide,
p-nitroacetan ilide, aniline yellow, iodoform.
UNIT XXIII Organic Compounds Containing Oxygen
General methods of preparation, properties, reactions and uses. — Chemistry involved in the titrimetric excercises - Acids bases and
Alcohols, Phenols and Ethers the use of indicators, oxali acid vs KMnO4, Mohr's salt vs KMnO4.
Alcohols Identification of primary, secondary and tertiary alcohols; — Chemical principles involved in the qualitative salt analysis
mechanism of dehydration. — Cations — Pb2+ , Cu2+, Al3+, Fe3+, Zn2+, Ni2+, Ca2+, Ba2+ , Mg2+ NH4+.
Phenols Acidic nature, electrophilic substitution reactions: Anions – CO32-, S2-, SO42-, NO2, NO3, Cl -, Br-, I- (Insoluble salts
halogenation, nitration and sulphonation, Reimer - Tiemann reaction. excluded).
Ethers: Structure — Chemical principles involved in the following experiments
Aldehyde and Ketones Nature of carbonyl group; 1. Enthalpy of solution of CuSO4
Nucleophilic addition to >C=O group, relative reactivities of aldehydes 2. Enthalpy of neutralization of strong acid and strong base.
and ketones; Important reactions such as - Nucleophilic addition 3. Preparation of lyophilic and lyophobic sols.
reactions (addition of HCN, NH3 and its derivatives), Grignard reagent;
4. Kinetic study of reaction of iodide ion with hydrogen peroxide
oxidation; reduction (Wolff Kishner and Clemmensen); acidity of α -
at room temperature.
hydrogen, aldol condensation, Cannizzaro reaction, Haloform reaction;
Chemical tests to distinguish between aldehydes and Ketones.
Carboxylic Acids Acidic strength & factors affecting it.
JEE ADVANCED
PHYSICAL CHEMISTRY Chemical Kinetics Rates of chemical reactions, Order of
reactions, Rate constant, First order reactions,
General Topics Concept of atoms and molecules,
Temperature dependence of rate constant (Arrhenius
Dalton's atomic theory, Mole concept, Chemical formulae,
equation).
Balanced chemical equations, Calculations (based on
mole concept) involving common oxidation-reduction, Solid State Classification of solids, crystalline state, seven
neutralisation, and displacement reactions, Concentration crystal systems (cell parameters a, b, c), close packed
in terms of mole fraction, molarity, molality and normality. structure of solids (cubic), packing in fcc, bcc and hcp
lattices, Nearest neighbours, ionic radii, simple ionic
Gaseous and Liquid States Absolute scale of
compounds, point defects.
temperature, ideal gas equation, Deviation from ideality,
van der Waals' equation, Kinetic theory of gases, average, Solutions Raoult's law, Molecular weight determination
root mean square and most probable velocities and their from lowering of vapour pressure, elevation of boiling
relation with temperature, Law of partial pressures, point and depression of freezing point.
Vapour pressure, Diffusion of gases.
Surface Chemistry Elementary concepts of adsorption
Atomic Structure and Chemical Bonding Bohr model, (excluding adsorption isotherms), Colloids, types,
spectrum of hydrogen atom, quantum numbers, methods of preparation and general properties,
Wave-particle duality, de-Broglie hypothesis, Uncertainty Elementary ideas of emulsions, surfactants and micelles
principle, Qualitative quantum mechanical picture of (only definitions and examples).
hydrogen atom, shapes of s, p and d orbitals, Electronic
Nuclear Chemistry Radioactivity, isotopes and isobars,
configurations of elements (up to atomic number 36),
Properties of rays, Kinetics of radioactive decay (decay
Aufbau principle, Pauli's exclusion principle and Hund's
series excluded), carbon dating, Stability of nuclei with
rule, Orbital overlap and covalent bond; Hybridisation
respect to proton-neutron ratio, Brief discussion on fission
involving s, p and d orbitals only, Orbital energy diagrams
and fusion reactions.
for homonuclear diatomic species, Hydrogen bond,
Polarity in molecules, dipole moment (qualitative aspects
only), VSEPR model and shapes of molecules (linear, INORGANIC CHEMISTRY
angular, triangular, square planar, pyramidal, square Isolation/Preparation and Properties of the following
pyramidal, trigonal bipyramidal, tetrahedral and Non-metals Boron, silicon, nitrogen, phosphorus, oxygen,
octahedral). sulphur and halogens, Properties of allotropes of carbon
(only diamond and graphite), phosphorus and sulphur.
Energetics First law of thermodynamics, Internal energy,
work and heat, pressure-volume work, Enthalpy, Hess's Preparation and Properties of the following
law, Heat of reaction, fusion and vaporization, Second law Compounds Oxides, peroxides, hydroxides, carbonates,
of thermodynamics, Entropy, Free energy, Criterion of bicarbonates, chlorides and sulphates of sodium,
spontaneity. potassium, magnesium and calcium, Boron, diborane,
boric acid and borax, Aluminium, alumina, aluminium
Chemical Equilibrium Law of mass action, Equilibrium chloride and alums, Carbon, oxides and oxyacid (carbonic
constant, Le-Chatelier's principle (effect of concentration, acid), Silicon, silicones, silicates and silicon carbide,
temperature and pressure), Significance of DG and DGo in Nitrogen, oxides, oxyacids and ammonia, Phosphorus,
chemical equilibrium, Solubility product, common ion oxides, oxyacids (phosphorus acid, phosphoric acid) and
effect, pH and buffer solutions, Acids and bases (Bronsted phosphine, Oxygen, ozone and hydrogen peroxide,
and Lewis concepts), Hydrolysis of salts. Sulphur, hydrogen sulphide, oxides, sulphurous acid,
Electrochemistry Electrochemical cells and cell reactions, sulphuric acid and sodium thiosulphate, Halogens,
Standard electrode potentials, Nernst equation and its hydrohalic acids, oxides and oxyacids of chlorine,
relation to DG, Electrochemical series, emf of galvanic bleaching powder, Xenon fluorides.
cells, Faraday's laws of electrolysis, Electrolytic Transition Elements (3d series) Definition, general
conductance, specific, equivalent and molar conductivity, characteristics, oxidation states and their stabilities,
Kohlrausch's law, Concentration cells. colour (excluding the details of electronic transitions) and
calculation of spin-only magnetic moment; Coordination (boiling points, density and dipole moments), Acidity of
compounds: nomenclature of mononuclear coordination alkynes, Acid catalysed hydration of alkenes and alkynes
compounds, cis-trans and ionisation isomerisms, (excluding the stereochemistry of addition and
hybridization and geometries of mononuclear elimination), Reactions of alkenes with KMnO4 and ozone,
coordination compounds (linear, tetrahedral, square Reduction of alkenes and alkynes, Preparation of alkenes
planar and octahedral). and alkynes by elimination reactions, Electrophilic
Preparation and Properties of the following addition reactions of alkenes with X2, HX, HOX and H2O
Compounds Oxides and chlorides of tin and lead, Oxides, (X=halogen), Addition reactions of alkynes, Metal
chlorides and sulphates of Fe2+, Cu2+ and Zn2+, Potassium acetylides.
permanganate, potassium dichromate, silver oxide, silver Reactions of Benzene Structure and aromaticity,
nitrate, silver thiosulphate. Electrophilic substitution reactions, halogenation,
Ores and Minerals Commonly occurring ores and nitration, sulphonation, Friedel-Crafts alkylation and
minerals of iron, copper, tin, lead, magnesium, aluminium, acylation Effect of o-, m- and p-directing groups in
zinc and silver. monosubstituted benzenes.
Extractive Metallurgy Chemical principles and reactions Phenols Acidity, electrophilic substitution reactions
only (industrial details excluded), Carbon reduction (halogenation, nitration and sulphonation), Reimer-
method (iron and tin), Self reduction method (copper and Tiemann reaction, Kolbe reaction.
lead), Electrolytic reduction method (magnesium and Characteristic Reactions of the following (including
aluminium), Cyanide process (silver and gold). those mentioned above) Alkyl halides, rearrangement
Principles of Qualitative Analysis Groups I to V (only reactions of alkyl carbocation, Grignard reactions,
Ag+, Hg2+, Cu2+, Pb2+, Bi3+, Fe3+, Cr3+, Al3+, Ca2+, Ba2+, Zn2+, Mn2+ nucleophilic substitution reactions, Alcohols,
and Mg2+), Nitrate, halides (excluding fluoride), sulphate esterification, dehydration and oxidation, reaction with
and sulphide. sodium, phosphorus halides, ZnCl2/concentrated HCl,
conversion of alcohols into aldehydes and ketones, Ethers,
Preparation by Williamson's Synthesis, Aldehydes and
ORGANIC CHEMISTRY Ketones, oxidation, reduction, oxime and hydrazone
Concepts Hybridisation of carbon, Sigma and pi-bonds, formation, aldol condensation, Perkin reaction, Cannizzaro
Shapes of simple organic molecules, Structural and reaction, haloform reaction and nucleophilic addition
geometrical isomerism, Optical isomerism of compounds reactions (Grignard addition), Carboxylic acids, formation
containing up to two asymmetric centres, (R,S and E,Z of esters, acid chlorides and amides, ester hydrolysis.
nomenclature excluded), IUPAC nomenclature of simple Amines, basicity of substituted anilines and aliphatic
organic compounds (only hydrocarbons, mono-functional amines, preparation from nitro compounds, reaction with
and bi-functional compounds), Conformations of ethane nitrous acid, azo coupling reaction of diazonium salts of
and butane (Newman projections), Resonance and aromatic amines, Sandmeyer and related reactions of
hyperconjugation, Keto-enol tautomerism, Determination diazonium salts, carbylamine reaction, Haloarenes,
of empirical and molecular formulae of simple nucleophilic aromatic substitution in haloarenes and
compounds (only combustion method), Hydrogen bonds, substituted haloarenes (excluding Benzyne mechanism
definition and their effects on physical properties of and Cine substitution).
alcohols and carboxylic acids, Inductive and resonance Carbohydrates Classification, mono and disaccharides
effects on acidity and basicity of organic acids and bases, (glucose and sucrose), Oxidation, reduction, glycoside
Polarity and inductive effects in alkyl halides, Reactive formation and hydrolysis of sucrose.
intermediates produced during homolytic and heterolytic
bond cleavage, Formation, structure and stability of Amino Acids and Peptides General structure (only
carbocations, carbanions and free radicals. primary structure for peptides) and physical properties.

Preparation, Properties and Reactions of Alkanes Properties and Uses of Some Important Polymers
Homologous series, physical properties of alkanes Natural rubber, cellulose, nylon, teflon and PVC.
(melting points, boiling points and density), Combustion Practical Organic Chemistry Detection of elements (N, S,
and halogenation of alkanes, Preparation of alkanes by halogens), Detection and identification of the following
Wurtz reaction and decarboxylation reactions. functional groups, hydroxyl (alcoholic and phenolic),
Preparation, Properties and Reactions of Alkenes and carbonyl (aldehyde and ketone), carboxyl, amino and
Alkynes Physical properties of alkenes and alkynes nitro, Chemical methods of separation of mono-functional
organic compounds from binary mixtures.
1
Some Basic Concepts
of Chemistry
6. The percentage composition of carbon by mole in methane is
Topic 1 Mole Concept (2019 Main, 8 April II)
(a) 75% (b) 20% (c) 25% (d) 80%
Objective Questions I (Only one correct option)
7. 8 g of NaOH is dissolved in 18 g of H2 O. Mole fraction of
1. 5 moles of AB2 weight 125 × 10−3 kg and 10 moles of A2 B2
NaOH in solution and molality (in mol kg− 1 ) of the solution
weight 300 × 10−3 kg. The molar mass of A ( M A ) and molar
respectively are (2019 Main, 12 Jan II)
mass of B ( M B ) in kg mol −1 are (2019 Main, 12 April I) (a) 0.2, 11.11 (b) 0.167, 22.20
(a) M A = 10 × 10−3 and M B = 5 × 10−3 (c) 0.2, 22.20 (d) 0.167, 11.11
(b) M A = 50 × 10−3 and M B = 25 × 10−3 8. The volume strength of 1 M H2 O2 is
(c) M A = 25 × 10−3 and M B = 50 × 10−3 (Molar mass of H2 O2 = 34 g mol −1 ) (2019 Main, 12 Jan II)
−3 −3
(d) M A = 5 × 10 and M B = 10 × 10 (a) 16.8 (b) 22.4 (c) 11.35 (d) 5.6
2. The minimum amount of O2 ( g ) consumed per gram of 9. The amount of sugar (C12 H22 O11 ) required to prepare 2 L of
reactant is for the reaction (Given atomic mass : Fe = 56, its 0.1 M aqueous solution is (2019 Main, 10 Jan II)
O = 16, Mg = 24, P = 31, C = 12, H = 1) (2019 Main, 10 April II) (a) 17.1 g (b) 68.4 g (c) 136.8 g (d) 34.2 g
(a) C3 H8 ( g ) + 5O2 ( g ) → 3CO2 ( g ) + 4H2 O( l ) 10. For the following reaction, the mass of water produced from
(b) P4 ( s ) + 5O2 ( g ) → P4 O10 ( s ) 445 g of C57 H110 O6 is :
(c) 4Fe( s ) + 3O2 ( g ) → 2Fe2 O3 ( s ) 2C57 H110 O6 ( s ) + 163O2 ( g ) → 114CO2 ( g ) + 110 H2 O ( l )
(d) 2Mg ( s ) + O2 ( g ) → 2MgO( s ) (2019 Main, 9 Jan II)

3. At 300 K and 1 atmospheric pressure, (a) 490 g (b) 495 g (c) 445 g (d) 890 g
10 mL of a hydrocarbon required 55 mL of O2 for complete 11. A solution of sodium sulphate contains 92 g of Na + ions per
combustion and 40 mL of CO2 is formed. The formula of the kilogram of water. The molality of Na + ions in that solution
hydrocarbon is (2019 Main, 10 April I) in mol kg−1 is (2019 Main, 9 Jan I)
(a) C4H7Cl (b) C4H 6 (c) C4H10 (d) C4H 8
(a) 16 (b) 4 (c) 132 (d) 8
4. 10 mL of 1 mM surfactant solution forms a monolayer
12. The most abundant elements by mass in the body of a healthy
covering 0.24 cm 2 on a polar substrate. If the polar head is human adult are oxygen (61.4%), carbon (22.9%), hydrogen
approximated as a cube, what is its edge length? (10.0 %), and nitrogen (2.6%). The weight which a 75 kg
(2019 Main, 9 April II)
person would gain if all 1 Hatoms are replaced by 2 Hatoms is
(a) 2.0 pm (b) 0.1 nm (c) 1.0 pm (d) 2.0 nm (2017 JEE Main)
5. For a reaction, (a) 15 kg (b) 37.5 kg
N 2 ( g ) + 3H 2 ( g ) → 2NH 3 ( g ), identify dihydrogen (H 2 ) (c) 7.5 kg (d) 10 kg
as a limiting reagent in the following reaction mixtures. 13. 1 g of a carbonate (M 2 CO3 ) on treatment with excess HCl
(2019 Main, 9 April I)
produces 0.01186 mole of CO2 . The molar mass of M 2 CO3
(a) 56 g of N 2 + 10 g of H 2 (b) 35 g of N 2 + 8 g of H 2 in g mol −1 is (2017 JEE Main)
(c) 14 g of N 2 + 4 g of H 2 (d) 28 g of N 2 + 6 g of H 2 (a) 1186 (b) 84.3 (c) 118.6 (d) 11.86
2 Some Basic Concepts of Chemistry

14. At 300 K and 1 atm, 15 mL of a gaseous hydrocarbon 24. The normality of 0.3 M phosphorus acid (H3PO3) is
requires 375 mL air containing 20% O2 by volume for (1999, 2M)
complete combustion. After combustion, the gases occupy (a) 0.1 (b) 0.9 (c) 0.3 (d) 0.6
330 mL. Assuming that the water formed is in liquid form 25. In which mode of expression, the concentration of a solution
and the volumes were measured at the same temperature and remains independent of temperature? (1988, 1M)
pressure, the formula of the hydrocarbon is (2016 Main) (a) Molarity (b) Normality (c) Formality (d) Molality
(a) C3 H8 (b) C4 H8 (c) C4 H10 (d) C3 H6 26. A molal solution is one that contains one mole of solute in
15. The molecular formula of a commercial resin used for (1986, 1M)
exchanging ions in water softening is C8 H7 SO3 Na (a) 1000 g of solvent (b) 1.0 L of solvent
(molecular weight = 206). What would be the maximum (c) 1.0 L of solution (d) 22.4 L of solution
uptake of Ca 2+ ions by the resin when expressed in mole per 27. If 0.50 mole of BaCl 2 is mixed with 0.20 mole of Na 3 PO4 ,
gram resin? (2015 Main) the maximum number of moles of Ba 3 (PO4 )2 that can be
1 1 2 1 formed is (1981, 1M)
(a) (b) (c) (d)
103 206 309 412 (a) 0.70 (b) 0.50 (c) 0.20 (d) 0.10
16. 3 g of activated charcoal was added to 50 mL of acetic acid 28. 2.76 g of silver carbonate on being strongly heated yields a
solution (0.06 N) in a flask. After an hour it was filtered and residue weighing (1979, 1M)
the strength of the filtrate was found to be 0.042 N. The (a) 2.16 g (b) 2.48 g (c) 2.32 g (d) 2.64 g
amount of acetic acid adsorbed (per gram of charcoal) is 29. When the same amount of zinc is treated separately with
(2015 Main) excess of sulphuric acid and excess of sodium hydroxide, the
(a) 18 mg (b) 36 mg (c) 42 mg (d) 54 mg ratio of volumes of hydrogen evolved is (1979, 1M)
17. The ratio mass of oxygen and nitrogen of a particular gaseous (a) 1 : 1 (b) 1 : 2 (c) 2 : 1 (d) 9 : 4
mixture is 1 : 4. The ratio of number of their molecule is 30. The largest number of molecules is in (1979, 1M)
(2014 Main)
(a) 1 : 4 (b) 7 : 32 (c) 1 : 8 (d) 3 : 16 (a) 36 g of water
(b) 28 g of CO
18. The molarity of a solution obtained by mixing 750 mL of
(c) 46 g of ethyl alcohol
0.5 M HCl with 250 mL of 2 M HCl will be (2013 Main)
(d) 54 g of nitrogen pentaoxide (N2 O5 )
(a) 0.875 M (b) 1.00 M (c) 1.75 M (d) 0.0975M
31. The total number of electrons in one molecule of carbon
19. Dissolving 120 g of urea (mol. wt. 60) in 1000 g of water dioxide is (1979, 1M)
gave a solution of density 1.15 g/mL. The molarity of the (a) 22 (b) 44 (c) 66 (d) 88
solution is (2011)
32. A gaseous mixture contains oxygen and nitrogen in the ratio
(a) 1.78 M (b) 2.00 M (c) 2.05 M (d) 2.22 M
of 1:4 by weight. Therefore, the ratio of their number of
20. Given that the abundances of isotopes 54 Fe, 56 Fe and 57 Fe molecules is (1979, 1M)
are 5%, 90% and 5%, respectively, the atomic mass of Fe is (a) 1 : 4 (b) 1 : 8 (c) 7 : 32 (d) 3 : 16
(2009)
(a) 55.85 (b) 55.95 Numerical Answer Type Questions
(c) 55.75 (d) 56.05 33. A 100 mL solution was made by adding 1.43 g of
21. Mixture X = 0.02 mole of [Co(NH3 )5 SO4 ]Br and 0.02 mole Na 2CO3 ⋅ xH 2O. The normality of the solution is 0.1 N. The
of [Co(NH3 )5 Br]SO4 was prepared in 2 L solution. value of x is ……… .
1 L of mixture X + excess of AgNO3 solution → Y (The atomic mass of Na is 23 g/mol) (2020 Main, 4 Sep II)
1 L of mixture X + excess of BaCl 2 solution → Z 34. Galena (an ore) is partially oxidised by passing air through it
Number of moles of Y and Z are (2003, 1M) at high temperature. After some time, the passage of air is
(a) 0.01, 0.01 (b) 0.02, 0.01 stopped, but the heating is continued in a closed furnace such
(c) 0.01, 0.02 (d) 0.02, 0.02 that the content undergo self-reduction. The weight (in kg) of
Pb produced per kg of O 2 consumed is ……… .
22. Which has maximum number of atoms? (2003, 1M)
(Atomic weights in g mol −1 : O = 16, S = 32, Pb = 207)
(a) 24 g of C (12) (b) 56 g of Fe (56)
(2018 Adv.)
(c) 27 g of Al (27) (d) 108 g of Ag (108) 35. To measure the quantity of MnCl 2 dissolved in an aqueous
23. How many moles of electron weighs 1 kg? solution, it was completely converted to KMnO4 using the
23 1 reaction,
(a) 6.023 × 10 (b) × 1031 (2002, 3M)
9.108 MnCl 2 + K 2 S2 O8 + H2 O → KMnO4 + H2 SO4 + HCl
6.023 1 (equation not balanced).
(c) × 1054 (d) × 108
9.108 9.108 × 6.023
Some Basic Concepts of Chemistry 3

Few drops of concentrated HCl were added to this solution K in a container of volume is 2.46 cm 3 . Density of surface sites
and gently warmed. Further, oxalic acid (225 mg) was is 6.023 × 1014 /cm 2 and surface area is 1000 cm 2 , find out the
added in portions till the colour of the permanganate ion
number of surface sites occupied per molecule of N2 . (2005, 3M)
disappeared. The quantity of MnCl 2 (in mg) present in the
initial solution is ……… . 45. In a solution of 100 mL 0.5 M acetic acid, one gram of active
(Atomic weights in g mol −1 : Mn = 55, Cl = 35.5) charcoal is added, which adsorbs acetic acid. It is found that the
concentration of acetic acid becomes 0.49 M. If surface area of
(2018 Adv.)
charcoal is 3.01 × 102 m2 , calculate the area occupied by single
36. In the following reaction sequence, the amount of D (in acetic acid molecule on surface of charcoal. (2003)
gram) formed from 10 moles of acetophenone is …….
46. Find the molarity of water. Given: ρ = 1000 kg/m3 (2003)
−1
(Atomic weights in g mol : H = 1, C = 12, N = 14,
47. A plant virus is found to consist of uniform cylindrical particles
O = 16, Br = 80. The yield (%) corresponding to the
of 150 Å in diameter and 5000 Å long. The specific volume of
product in each step is given in the parenthesis)
the virus is 0.75 cm 3 /g. If the virus is considered to be a single
O particle, find its molar mass. (1999, 3M)

NaOBr NH3, ∆ Br2/KOH


48. 8.0575 × 10−2 kg of Glauber’s salt is dissolved in water to
H3O+
A B C
obtain 1 dm 3 of solution of density 1077.2 kg m −3 . Calculate
(60%) (50%) (50%)
the molality, molarity and mole fraction of Na 2 SO4 in solution.
Br2(3 equivalent ) (1994, 3M)
D
AcOH
(100%)
(2018 Adv.) 49. A is a binary compound of a univalent metal. 1.422 g of A reacts
completely with 0.321 g of sulphur in an evacuated and sealed
tube to give 1.743 g of a white crystalline solid B, that forms a
Fill in the Blanks hydrated double salt, C with Al 2 (SO4 )3 . Identify A, B and C.
37. The weight of 1 × 1022 molecules of CuSO4 ⋅ 5H2 O is (1994, 2M)

…………. . (1991, 1M) 50. Upon mixing 45.0 mL 0.25 M lead nitrate solution with
25.0 mL of a 0.10 M chromic sulphate solution, precipitation of
38. 3.0 g of a salt of molecular weight 30 is dissolved in 250 g
lead sulphate takes place. How many moles of lead sulphate are
water. The molarity of the solution is ………. (1983, 1M)
formed? Also calculate the molar concentrations of species left
39. The total number of electrons present in 18 mL of water is behind in the final solution. Assume that lead sulphate is
…………. . (1980, 1M) completely insoluble. (1993, 3M)
40. The modern atomic mass unit is based on the mass of 51. Calculate the molality of 1.0 L solution of 93% H2 SO4 ,
…………. . (1980, 1M)
(weight/volume). The density of the solution is 1.84 g/mL.
(1990, 1M)
Integer Answer Type Questions 52. A solid mixture (5.0 g) consisting of lead nitrate and sodium
41. The mole fraction of a solute in a solution is 0.1. At 298 K, nitrate was heated below 600°C until the weight of the residue
molarity of this solution is the same as its molality. Density was constant. If the loss in weight is 28.0 per cent, find the
of this solution at 298 K is 2.0 g cm−3 . The ratio of the amount of lead nitrate and sodium nitrate in the mixture.
(1990, 4M)
m 
molecular weights of the solute and solvent,  solute  is ... 53. n-butane is produced by monobromination of ethane followed
 msolvent 
by Wurtz’s reaction.Calculate volume of ethane at NTP
. (2016 Adv.) required to produce 55 g n-butane, if the bromination takes
42. A compound H2 X with molar weight of 80 g is dissolved place with 90% yield and the Wurtz’s reaction with 85% yield.
in a solvent having density of 0.4 g mL−1 . Assuming no (1989, 3M)

change in volume upon dissolution, the molality of a 3.2 54. A sugar syrup of weight 214.2 g contains 34.2 g of sugar
molar solution is (2014 Adv.) (C12 H22 O11 ). Calculate (i) molal concentration and (ii) mole
fraction of sugar in syrup. (1988, 2M)
43. 29.2% (w/W ) HCl stock solution has density of 1.25g mL
−1
. The molecular weight of HCl is 36.5 g mol − 1 . The 55. An unknown compound of carbon, hydrogen and oxygen
volume (mL) of stock solution required to prepare a 200 contains 69.77% C and 11.63% H and has a molecular weight
of 86. It does not reduces Fehling’s solution but forms a
mL solution 0.4 M HCl is (2012)
bisulphate addition compound and gives a positive iodoform
test. What is the possible structure(s) of unknown compound?
Subjective Questions (1987, 3M)
44. 20% surface sites have adsorbed N2 . On heating N2 gas 56. The density of a 3 M sodium thiosulphate solution ( Na 2 S2 O3 )
evolved from sites and were collected at 0.001 atm and 298 is 1.25 g per mL. Calculate (i) the percentage by weight of
4 Some Basic Concepts of Chemistry

sodium thiosulphate (ii) the mole fraction of sodium oxygen. All volumes have been reduced to NTP. Calculate
thiosulphate and (iii) the molalities of Na + and S2 O2−
3 ions. the molecular formula of the hydrocarbon gas. (1979, 3M)
(1983, 5M) 59. In the analysis of 0.5 g sample of feldspar, a mixture of
57. (a) 1.0 L of a mixture of CO and CO2 is taken. This mixture chlorides of sodium and potassium is obtained, which weighs
is passed through a tube containing red hot charcoal. The 0.1180 g. Subsequent treatment of the mixed chlorides with
volume now becomes 1.6 L. The volumes are measured silver nitrate gives 0.2451 g of silver chloride. What is the
under the same conditions. Find the composition of percentage of sodium oxide and potassium oxide in the
mixture by volume. sample? (1979, 5M)
(b) A compound contains 28 per cent of nitrogen and 60. The vapour density (hydrogen = 1) of a mixture consisting of
72 per cent of a metal by weight. 3 atoms of metal NO2 and N2 O4 is 38.3 at 26.7°C. Calculate the number of
combine with 2 atoms of nitrogen. Find the atomic moles of NO2 in 100 g of the mixture. (1979, 5M)
weight of metal. (1980, 5M)
61. Accounts for the following. Limit your answer to two
58. 5.00 mL of a gas containing only carbon and hydrogen were sentences, “Atomic weights of most of the elements are
mixed with an excess of oxygen (30 mL) and the mixture fractional”. (1979, 1M)
exploded by means of electric spark. After explosion, the
62. Naturally occurring boron consists of two isotopes whose
volume of the mixed gases remaining was 25 mL.
atomic weights are 10.01 and 11.01. The atomic weight of
On adding a concentrated solution of KOH, the volume natural boron is 10.81. Calculate the percentage of each
further diminished to 15 mL, the residual gas being pure isotope in natural boron. (1978, 2M)

Topic 2 Equivalent Concept, Neutralisation and Redox Titration


Objective Questions I (Only one correct option) surface of water in a round watch glass. Hexane evaporates
1. An example of a disproportionation reaction is and a monolayer is formed. The distance from edge to centre
(2019 Main, 12 April I) of the watch glass is 10 cm. What is the height of the
(a) 2MnO−4 + 10I + 16H → 2Mn2 + +5I2 + 8H 2O
− + monolayer? [Density of fatty acid = 0.9 g cm −3 ; π = 3]
(2019 Main, 8 April II)
(b) 2NaBr + Cl2 → 2NaCl + Br2
(a) 10−6 m (b) 10−4 m
(c) 2KMnO4 → K 2MnO4 + MnO2 + O2
(c) 10−8 m (d) 10−2 m
(d) 2CuBr → CuBr2 + Cu
4. In order to oxidise a mixture of one mole of each of FeC2 O4 ,
2. In an acid-base titration, 0.1 M HCl solution was added to
Fe2 (C2 O4 )3 , FeSO4 and Fe2 (SO4 )3 in acidic medium, the
the NaOH solution of unknown strength. Which of the
number of moles of KMnO4 required is (2019 Main, 8 April I)
following correctly shows the change of pH of the titration
(a) 2 (b) 1 (c) 3 (d) 1.5
mixture in this experiment? (2019 Main, 9 April II)
5. 100 mL of a water sample contains 0.81 g of calcium
bicarbonate and 0.73 g of magnesium bicarbonate. The
hardness of this water sample expressed in terms of
pH pH equivalents of CaCO3 is (molar mass of calcium bicarbonate
is 162 g mol−1 and magnesium bicarbonate is 146 g mol−1 )
(2019 Main, 8 April I)
V(mL) V(mL)
(a) 5,000 ppm (b) 1,000 ppm
(A) (B)
(c) 100 ppm (d) 10,000 ppm
6. 50 mL of 0.5 M oxalic acid is needed to neutralise 25 mL of
sodium hydroxide solution. The amount of NaOH in 50 mL
pH pH
of the given sodium hydroxide solution is
(2019 Main, 12 Jan I)
(a) 40 g (b) 80 g (c) 20 g (d) 10 g
V(mL) V(mL)
(C) (D) 7. 25 mL of the given HCl solution requires 30 mL of 0.1 M
(2019 Main, 9 April II)
sodium carbonate solution. What is the volume of this HCl
(a) (D) (b) (A)
solution required to titrate 30 mL of 0.2 M aqueous NaOH
(c) (B) (d) (C) solution? (2019 Main, 11 Jan II)
3. 0.27 g of a long chain fatty acid was dissolved in 100 cm 3 of (a) 75 mL (b) 25 mL
hexane. 10 mL of this solution was added dropwise to the (c) 12.5 mL (d) 50 mL
Some Basic Concepts of Chemistry 5

8. In the reaction of oxalate with permanganate in acidic 17. The oxidation number of sulphur in S 8 , S 2 F 2 , H 2 S
medium, the number of electrons involved in producing one respectively, are (1999)
molecule of CO2 is (2019 Main, 10 Jan II) (a) 0, +1 and –2 (b) +2, +1 and –2
(a) 2 (b) 5 (c) 1 (d) 10 (c) 0, +1 and +2 (d) –2, +1 and –2
9. The ratio of mass per cent of C and H of an organic 18. The number of moles of KMnO 4 that will be needed to react
compound (Cx H y O z ) is 6 : 1. If one molecule of the above
completely with one mole of ferrous oxalate in acidic
compound (Cx H y O z ) contains half as much oxygen as
medium is (1997)
required to burn one molecule of compound Cx H y
2 3 4
completely to CO2 and H2 O. The empirical formula of (a) (b) (c) (d) 1
compound Cx H y O z is (2018 Main)
5 5 5
(a) C3 H6 O3 (b) C2 H4 O (c) C3 H4 O2 (d) C2 H4 O3 19. The number of moles of KMnO 4 that will be needed to react
10. An alkali is titrated against an acid with methyl orange as with one mole of sulphite ion in acidic solution is
(1997)
indicator, which of the following is a correct combination?
(2018 Main) 2 3 4
(a) (b) (c) (d) 1
5 5 5
Base Acid End point
(a) Weak Strong Colourless to pink 20. For the redox reaction
(b) Strong Strong Pinkish red to yellow MnO4− + C2 O24 − + H+ → Mn 2+ + CO2 + H2 O
(c) Weak Strong Yellow to pinkish red The correct coefficients of the reactants for the balanced
(d) Strong Strong Pink to colourless reaction are
11. From the following statements regarding H2 O2 choose the MnO−4 C2 O42 − H+ (1992)
incorrect statement. (2015 Main) (a) 2 5 16
(a) It can act only as an oxidising agent (b) 16 5 2
(b) It decomposed on exposure to light (c) 5 16 2
(c) It has to be stored in plastic or wax lined glass bottles in (d) 2 16 5
dark 21. The volume strength of 1.5 N H2 O2 is (1990, 1M)
(d) It has to be kept away from dust (a) 4.8 (b) 8.4 (c) 3.0 (d) 8.0
12. Consider a titration of potassium dichromate solution with 22. The oxidation number of phosphorus in Ba(H2 PO2 )2 is
acidified Mohr’s salt solution using diphenylamine as
(a) +3 (b) +2 (1988)
indicator. The number of moles of Mohr's salt required per
(c) +1 (d) –1
mole of dichromate is (2007, 3M)
(a) 3 (b) 4 (c) 5 (d) 6 23. The equivalent weight of MnSO 4 is half of its molecular
13. In the standardisation of Na 2 S2 O3 using K 2 Cr2 O7 by weight, when it converts to (1988, 1M)
iodometry, the equivalent weight of K 2 Cr2 O7 is (2001, 1M) (a) Mn 2 O3 (b) MnO2
(a) (molecular weight)/2 (b) (molecular weight)/6 (c) MnO−4 (d) MnO2−
4
(c) (molecular weight)/3 (d) same as molecular weight
14. The reaction, 3ClO − (aq) → ClO3– (aq) + 2Cl − (aq) is an Objective Question II (More than one correct option)
example of (2001) 24. For the reaction, I− + ClO3− + H2 SO4 → Cl − + HSO4− + I2
(a) oxidation reaction the correct statement(s) in the balanced equation is/are
(b) reduction reaction (a) stoichiometric coefficient of HSO−4 is 6 (2014 Adv.)
(c) disproportionation reaction (b) iodide is oxidised
(d) decomposition reaction (c) sulphur is reduced
15. An aqueous solution of 6.3 g oxalic acid dihydrate is made up (d) H2 O is one of the products
to 250 mL. The volume of 0.1 N NaOH required to
completely neutralise 10 mL of this solution is (2001, 1M) Numerical Answer Type Questions
(a) 40 mL (b) 20 mL (c) 10 mL (d) 4 mL 25. 5.00 mL of 0.10 M oxalic acid solution taken in a conical
16. Among the following, the species in which the oxidation flask is titrated against NaOH from a burette using
number of an element is + 6 (2000) phenolphthalein indicator. The volume of NaOH required for
(a) MnO−4 (b) Cr(CN)3−
6
the appearance of permanent faint pink color is tabulated
below for five experiments. What is the concentration, in
(c) NiF62− (d) CrO2 Cl 2
molarity, of the NaOH solution?
6 Some Basic Concepts of Chemistry

Exp. No. Vol. of NaOH (mL) Integer Answer Type Questions


1 12.5
31. The difference in the oxidation numbers of the two types of
2 10.5
sulphur atoms in Na 2 S4 O6 is (2011)
3 9.0
4 9.0
32. Among the following, the number of elements showing only
one non-zero oxidation state is O, Cl, F, N, P, Sn, Tl, Na, Ti
5 9.0 (2010)
(2020 Adv.)
33. A student performs a titration with different burettes and
26. Aluminium reacts with sulphuric acid to form aluminium finds titrate values of 25.2 mL, 25.25 mL, and 25.0 mL. The
sulphate and hydrogen. What is the volume of hydrogen gas number of significant figures in the average titrate value is
in litre (L) produced at 300 K and 1.0 atm pressure, when (2010)
5.4 g of aluminium and 50.0 mL of 5.0 M sulphuric acid are
combined for the reaction? Subjective Questions
(Use molar mass of aluminium as 27.0 g mol −1 , R = 0.082
34. Calculate the amount of calcium oxide required when it
reacts with 852 g of P4 O10 . (2005, 2M)
atm L mol −1 K −1 ) (2020 Adv.)
35. Hydrogen peroxide solution (20 mL) reacts quantitatively
27. A 20.0 mL solution containing 0.2 g impure H 2O2 reacts with a solution of KMnO4 (20 mL) acidified with dilute
completely with 0.316 g of KMnO4 in acid solution. The H2 SO4 . The same volume of the KMnO4 solution is just
purity of H 2O2 (in%) is ............. (molecular weight of decolourised by 10 mL of MnSO4 in neutral medium
H 2O2 = 34; molecular weight of KMnO4 = 158 ). simultaneously forming a dark brown precipitate of hydrated
(2020 Main, 4 Sep I) MnO2 . The brown precipitate is dissolved in 10 mL of 0.2 M
28. The ammonia prepared by treating ammonium sulphate with sodium oxalate under boiling condition in the presence of
calcium hydroxide is completely used by NiCl 2 ⋅ 6H2 O to dilute H2 SO4 . Write the balanced equations involved in the
form a stable coordination compound. Assume that both the reactions and calculate the molarity of H2 O2 . (2001)
reactions are 100% complete. If 1584 g of ammonium 36. How many millilitres of 0.5 M H2 SO4 are needed to dissolve
sulphate and 952 g of NiCl 2 ⋅ 6H2 O are used in the 0.5 g of copper (II) carbonate? (1999, 3M)
preparation, the combined weight (in grams) of gypsum
and the nickel-ammonia coordination compound thus 37. An aqueous solution containing 0.10 g KIO3
produced is____ (formula weight = 214.0) was treated with an excess of KI
solution. The solution was acidified with HCl. The liberated
(Atomic weights in g mol −1 : H = 1, N = 14, O = 16, S = 32, I2 consumed 45.0 mL of thiosulphate solution decolourise
Cl = 35.5, Ca = 40, Ni = 59) (2018 Adv.) the blue starch-iodine complex. Calculate the molarity of the
sodium thiosulphate solution. (1998, 5M)
Assertion and Reason 38. To a 25 mL H2 O2 solution, excess of acidified solution of
Read the following questions and answer as per the direction potassium iodide was added. The iodine liberated required
given below :
20 mL of 0.3 N sodium thiosulphate solution. Calculate the
(a) Statement I is true; Statement II is true; Statement II is the
volume strength of H2 O2 solution. (1997, 5M)
correct explanation of Statement I.
(b) Statement I is true; Statement II is true; Statement II is not 39. A 3.00 g sample containing Fe3 O4 , Fe2 O3 and an inert
the correct explanation of Statement I. impure substance, is treated with excess of KI solution in
(c) Statement I is true; Statement II is false. presence of dilute H2 SO4 . The entire iron is converted into
(d) Statement I is false; Statement II is true. Fe2+ along with the liberation of iodine. The resulting
solution is diluted to 100 mL . A 20 mL of the diluted
29. Statement I In the titration of Na 2 CO3 with HCl using solution requires 11.0 mL of 0.5 M Na 2 S2 O3 solution to
methyl orange indicator, the volume required at the reduce the iodine present. A 50 mL of the dilute solution,
equivalence point is twice that of the acid required using after complete extraction of the iodine required 12.80 mL of
phenolphthalein indicator. 0.25 M KMnO4 solution in dilute H2 SO4 medium for the
Statement II Two moles of HCl are required for the oxidation of Fe2+ . Calculate the percentage of Fe2 O3 and
complete neutralisation of one mole of Na 2 CO3 . (1991, 2M) Fe3 O4 in the original sample. (1996, 5M)
40. A 20.0 cm 3 mixture of CO, CH4 and He gases is exploded by
Fill in the Blanks an electric discharge at room temperature with excess of
30. The compound YBa 2 Cu 3 O7 , which shows super oxygen. The volume contraction is found to be 13.0 cm 3 .
conductivity, has copper in oxidation state ………. Assume A further contraction of 14.0 cm 3 occurs when the residual
that the rare earth element yttrium is in its usual + 3 oxidation gas is treated with KOH solution. Find out the composition
state. (1994, 1M)
of the gaseous mixture in terms of volume percentage.
(1995, 4M)
Some Basic Concepts of Chemistry 7

41. A 5.0 cm 3 solution of H2 O2 liberates 0.508 g of iodine from Calculate the amount of H2 C2 O4 and NaHC2 O4 in the
an acidified KI solution. Calculate the strength of H2 O2 mixture. (1990, 5M)
solution in terms of volume strength at STP. (1995, 3M) 47. An organic compound X on analysis gives 24.24 per cent
42. One gram of commercial AgNO3 is dissolved in 50 mL of carbon and 4.04 per cent hydrogen. Further, sodium extract
of 1.0 g of X gives 2.90 g of silver chloride with acidified
water. It is treated with 50 mL of a KI solution. The silver
silver nitrate solution. The compound X may be represented
iodide thus precipitated is filtered off. Excess of KI in the
by two isomeric structures Y and Z. Y on treatment with
filtrate is titrated with (M/10) KIO 3 solution in presence of
aqueous potassium hydroxide solution gives a dihydroxy
6 M HCl till all I− ions are converted into ICl. It requires compound while Z on similar treatment gives ethanal. Find
50 mL of (M/10) KIO 3 solution, 20 mL of the same stock out the molecular formula of X and gives the structure
solution of KI requires 30 mL of (M/10) KIO3 under similar of Y and Z. (1989, 5M)
conditions. Calculate the percentage of AgNO3 in the
sample. 48. An equal volume of a reducing agent is titrated separately
Reaction KIO3 + 2KI + 6HCl → 3ICl + 3KCl + 3H2 O with 1 M KMnO4 in acid, neutral and alkaline medium. The
(1992, 4M)
volumes of KMnO4 required are 20 mL in acid, 33.3 mL in
neutral and 100 mL in alkaline media. Find out the oxidation
43. A 2.0 g sample of a mixture containing sodium carbonate, state of manganese in each reduction product. Give the
sodium bicarbonate and sodium sulphate is gently heated till balanced equations for all the three half reaction. Find out the
the evolution of CO2 ceases. The volume of CO2 at 750 mm volume of 1M K 2 Cr2 O7 consumed, if the same volume of the
Hg pressure and at 298 K is measured to be 123.9 mL. A 1.5 g reducing agent is titrated in acid medium. (1989, 5M)
of the same sample requires 150 mL of (M/10) HCl for
49. A sample of hydrazine sulphate ( N2 H 6 SO4 ) was dissolved in
complete neutralisation. Calculate the percentage
composition of the components of the mixture. (1992, 5M) 100 mL of water, 10 mL of this solution was reacted with
excess of ferric chloride solution and warmed to complete
44. A 1.0 g sample of Fe2 O3 solid of 55.2% purity is dissolved in the reaction. Ferrous ion formed was estimated and it,
acid and reduced by heating the solution with zinc dust. The required 20 mL of M/50 potassium permanganate solution.
resultant solution is cooled and made up to 100.0 mL. An Estimate the amount of hydrazine sulphate in one litre of the
aliquot of 25.0 mL of this solution requires for titration. solution.
Calculate the number of electrons taken up by the oxidant in Reaction 4Fe3+ + N2 H4 → N2 + 4Fe2+ + 4H+
the reaction of the above titration. (1991, 4M)
MnO−4 + 5Fe2+ + 8H+ → Mn 2+ + 5Fe3+ + 4H2 O
45. A solution of 0.2 g of a compound containing Cu 2+ and
(1988, 3M)
C2 O2−
4 ions on titration with 0.02 M KMnO4 in presence of
50. 5 mL of 8 N nitric acid, 4.8 mL of 5 N hydrochloric acid and
H2 SO4 consumes 22.6 mL of the oxidant. The resultant a certain volume of 17 M sulphuric acid are mixed together
solution is neutralised with Na 2 CO3 , acidified with dilute and made up to 2 L. 30 mL of this acid mixture exactly
acetic acid and treated with excess KI. The liberated iodine neutralise 42.9 mL of sodium carbonate solution containing
requires 11.3 mL of 0.05 M Na 2 S2 O3 solution for complete one gram of Na 2 CO3 ⋅ 10H2 O in 100 mL of water. Calculate
reduction. Find out the mole ratio of Cu 2+ to C2 O2−4 in the the amount in gram of the sulphate ions in solution.
compound. Write down the balanced redox reactions (1985, 4M)
involved in the above titrations. (1991, 5M) 51. 2.68 × 10−3 moles of a solution containing an ion A n+ require
46. A mixture of H2 C2 O4 (oxalic acid) and NaHC2 O4 weighing 1.61 × 10−3 moles of MnO−4 for the oxidation of A n+ to A O−3
2.02 g was dissolved in water and the solution made up to one in acidic medium. What is the value of n ? (1984, 2M)
litre. Ten millilitres of the solution required 3.0 mL of 0.1 N 52. 4.08 g of a mixture of BaO and unknown carbonate MCO3
sodium hydroxide solution for complete neutralisation. In
was heated strongly. The residue weighed 3.64 g. This was
another experiment, 10.0 mL of the same solution, in hot
dissolved in 100 mL of 1 N HCl. The excess acid required
dilute sulphuric acid medium, required 4.0 mL of 0.1 N
16 mL of 2.5 N NaOH solution for complete neutralisation.
potassium permanganate solution for complete reaction.
Identify the metal M. (1983, 4M)

Answers
Topic 1 13. (b) 14. (*) 15. (d) 16. (d)
1. (d) 2. (c) 3. (b) 4. (a) 17. (b) 18. (a) 19. (c) 20. (b)
5. (d) 6. (b) 7. (d) 8. (c) 21. (a) 22. (a) 23. (d) 24. (d)
9. (b) 10. (b) 11. (b) 12. (c) 25. (d) 26. (a) 27. (d) 28. (a)
29. (a) 30. (a) 31. (a) 32. (c)
8 Some Basic Concepts of Chemistry

33. (10.00) 34. (6.47kg) 35. (126 mg) 36. (495 g) 5. (d) 6. (*) 7. (b) 8. (c)
37. (4.14 g) 38. (0.4) 39. (6.023×10 24 ) 40. C-12 9. (d) 10. (c) 11. (a) 12. (d)
isotope 13. (b) 14. (c) 15. (a) 16. (d)
41. (9) 42. (8) 43. (8 mL) 44. (2) 17. (a) 18. (b) 19. (a) 20. (a)
45. (5 × 10 −19 m 2 ) 46. (55.56 mol 21. (b) 22. (c) 23. (b) 24. (a,b,d)
L −1) 47. (70.91 × 10 6g) 48. (4.3 × 10 −3) 25. (0.11) 26. (6.15) 27. (85)
51. (10.42) 52. (1.7 g) 53. (55.55 L) 54. (9.9 × 10 −3) 28. (2992) 29. (b) 30. 7/3 31. (5)
56. (i) (37.92), (ii) (0.065), (iii) (7.73m) 57. (a) (0.6), (b) (24) 32. (2) 33. (3) 34. (1008 g) 36. (8.096 mL)
59. (i) (0.0179 g), (ii) (10.6 %) 60. (0.437) 62. (20 %) 37. (0.062 M) 38. (1.334 V) 42. (85%) 44. (1.04)
Topic 2 45. (1:2) 48. (16.67 mL) 49. (6.5gL −1) 50. (6.5376 g)
1. (d) 2. (b) 3. (a) 4. (a) 51. (2) 52. (Ca)

Hints & Solutions


Topic 1 Mole Concept (c) 4Fe(s) + 3O2(g ) → 2Fe2O3 (s)
244g 96g 96
Key Idea To find the mass of A and B in the given question, ⇒ 1 g of reactant = g of O2 consumed = 0.43 g
1. 224
mole concept is used.
given mass (w) (d) 2Mg(s) + O2(g ) → 2MgO(s)
Number of moles( n) = 48 g 32 g
molecular mass (M )
32
⇒ 1 g of reactant = g of O2 consumed = 0.67 g
48
Compound Mass of A (g) Mass of B (g)
So, minimum amount of O2 is consumed per gram of reactant
AB2 MA 2 MB (Fe) in reaction (c).
A 2B 2 2 MA 2 MB 3. In eudiometry,
300 K
 y y
We know that, CxH y +  x +  O2 → x CO2 + H2O
 4 1 atm 2
given mass (w)
Number of moles (n) =  y
molecular mass (M ) 1 mol  x +  mol x mol
 4
n×M = w …(A) 1 mL
 y
 x +  mL x mL
 4
Using equation (A), it can be concluded that
 y
10 mL  x +  × 10 mL 10x mL
5(M A + 2M B ) = 125 × 10−3 kg …(i)  4

10(2M A + 2M B ) = 300 × 10−3 kg …(ii) Given, (i) VCO2 = 10x = 40 mL ⇒ x = 4


 y
From equation (i) and (ii) (ii) VO2 = 10  x +  mL = 55 mL
 4
1 (M A + 2M B )  125 
=   y
2 (2M A + 2M B )  300 ⇒ 10  4 +  = 55 [Qx = 4]
 4
On solving the equation, we obtain
y × 10
M A = 5 × 10−3 ⇒ 40 + = 55
4
and M B = 10 × 10−3 10 4
⇒ y× = 15 ⇒ y = 15 × =6
So, the molar mass of A (M A ) is 4 10
5 × 10−3 kg mol −1 and B (M B ) is 10 × 10−3 kg mol −1. So, the hydrocarbon (CxH y ) is C4H6.

2. (a) C3H8 (g ) + 5O2 (g ) → 3CO2 (g ) + 4H2O(l ) 4. Given, volume = 10 mL


44g 160g Molarity = 1mM = 10−3 M
160 ∴Number of millimoles = 10 mL × 10−3 M = 10−2
⇒ 1g of reactant = g of O2 consumed = 3.64 g
44 Number of moles = 10−5
(b) P4 (s) + 5O2(g ) → P4O10 (s) Now, number of molecules
124g 160g
= Number of moles × Avogadro’s number
160
⇒1 g of reactant = g of O2 consumed = 129
. g = 10−5 × 6 × 1023 = 6 × 1018
124
Some Basic Concepts of Chemistry 9

Surface area occupied by 6 × 1018 molecules = 0.24 cm2 w Solute


∴Surface area occupied by 1 molecule n Solute Mw Solute
χ Solute = =
0.24 n Solute + nSolvent w Solute w
+ Solvent
= = 0.04 × 10−18 cm2 Mw Solute MwSolvent
6 × 1018
As it is given that polar head is approximated as cube. Thus, Given, wSolute = wNaOH = 8 g
surface area of cube = a2, where Mw Solute = MwNaOH = 40g mol − 1
a =edge length w Solvent = w H2 O = 18g
∴ a = 4 × 10−20 cm2
2
Mw Solvent = 18 g mol− 1
a = 2 × 10−10 cm = 2 pm 8 / 40 0.2 0.2
∴ χ Solute = χ NaOH = = = = 0167
.
8 18 0.2 + 1 12
.
5. Key Idea The reactant which is present in the lesser amount, +
i.e. which limits the amount of product formed is called 40 18
limiting reagent. Moles of solute
Now, molality (m) =
Mass of solvent (in kg)
When 56 g of N2 + 10 g of H2 is taken as a combination then w Solute 8
dihydrogen (H2 ) act as a limiting reagent in the reaction.
Mw Solute 40
N2 (g ) + 3H2 (g ) → 2NH3 (g ) …(I) = × 1000 = × 1000
wSolvent (in g ) 18
2 × 14 g 3 × 2g 2(14 + 3) g
0.2
28g 6g 34g = × 1000 = 11.11mol kg − 1
18
28g N2 requires 6g H2 gas.
Thus, mole fraction of NaOH in solution and molality of the
6g
56g of N2 requires × 56 g = 12g of H2 solution respectively are 0.167 and 11.11 mol kg − 1 .
28 g
8. Concentration of H2O2 is expressed in terms of volume strength,
12g of H2 gas is required for 56g of N2 gas but
i.e. “volume of O2 liberated by H2O2 at NTP”. Molarity is
only 10 g of H2 gas is present in option (a).
connected to volume strength as:
Hence, H2 gas is the limiting reagent. x
In option (b), i.e. 35g of N2 + 8 g of H2. Molarity (M) = or x = Molarity × 11.2
112
.
As 28 g N2 requires 6g of H2. where, x = volume strength
6g So, for 1 M H2O2, x = 1 × 112. = 112.
35g N2 requires × 35 g H2 ⇒ 7.5 g of H2.
28 g Among the given options, 11.35 is nearest to 11.2.
Here, H2 gas does not act as limiting reagent since 7.5 g of H2 Number of moles of solute (n)
gas is required for 35g of N2 and 8g of H2 is present in reaction 9. Molarity =
Volume of solution (in L)
mixture. Mass of H2 left unreacted = 8 − 7.5 g of H2.
wB (g)
= 0.5 gof H2. Also, n =
M B (gmol−1 )
Similarly, in option (c) and (d), H2 does not act as limiting
reagent. w / MB
∴ Molarity = B
For 14 g of N2 + 4 g of H2. V
As we know 28g of N2 reacts with 6g of H2. Given, wB = mass of solute ( B ) in g
6 M B = Gram molar mass of B (C12H22O11 ) = 342 g mol −1
14 g of N2 reacts with × 14 g of H2 ⇒ 3g of H2.
28 Molarity = 01. M
For 28g of N2 + 6 g of H2, i.e. 28g of N2 reacts with 6g of H2 Volume (V ) = 2 L
(by equation I). w / 342
⇒ . = B
01 ⇒ wB = 01
. × 342 × 2 g = 68.4 g
2
6. Key Idea The percentage composition of a compound is given 10. 2 C57 H110O6 (s) + 163 O2 (g ) → 110H2O(l ) + 114 CO2 (g )
by the formula.
% composition = [Composition of a substance in a compound / Molecular mass of C57H110O6
Total composition total of compound] ×100 = 2 × (12 × 57 + 1 × 110 + 16 × 6) g = 1780 g
Molecular mass of 110 H2O = 110 (2 + 16) = 1980 g
In CH4 , 1780 g of C57H110O6 produced = 1980 g of H2O.
mole of carbon = 1 1980
mole of hydrogen = 4 445g of C57H110O6 produced = × 445 g of H2O
1780
1 = 495 of H2O
∴ % of carbon by mole in CH4 = × 100 = 20%
1+ 4
Number of moles of solute
11. Molality (m) = × 1000
7. Mole fraction of solute Mass of solvent (in g)
number of moles of solute + number of moles solvent
=
number of moles of solute
10 Some Basic Concepts of Chemistry

Mass of solute (in g) × 1000 16. Given, initial strength of acetic acid = 0.06 N
=
Molecular weight of solute 
 × mass of solvent (in g) Final strength = 0.042 N; Volume = 50 mL
wNa + × 1000 92 × 1000 ∴Initial millimoles of CH3COOH = 0.06 × 50 = 3
= = = 4 mol kg − 1
M Na + × wH 2 O 23 × 1000 Final millimoles of CH3COOH = 0.042 × 50 = 2.1
∴ Millimoles of CH3COOH adsorbed = 3 − 2.1= 0.9 mmol
12. Given, abundance of elements by mass
= 0.9 × 60 mg = 54 mg
oxygen = 614. %, carbon = 22.9%, hydrogen = 10% and (mO 2 )
nitrogen = 2.6% nO 2 (M O 2 )
Total weight of person = 75 kg 17. =
nN 2 (mN 2 )
75 × 10
Mass due to 1 H = = 7.5 kg (M N 2 )
100
1 2
H atoms are replaced by H atoms, where, mO 2 = given mass of O2 , mN 2 = given mass of N2 ,
Mass due to 2 H = (7.5 × 2) kg M O 2 = molecular mass of O2 , M N 2 = molecular mass of N2 , nO 2 =
number of moles of O2 , nN 2 = number of moles of N2
∴ Mass gain by person = 7.5 kg
 mO  28 1 28 7
13. M 2CO3 + 2HCl → 2M Cl + H2O + CO2 = 2 = × =
1g 0.01186  mN 2  32 4 32 32
mole
M 1V1 + M 2V2
Number of moles of M 2CO3 reacted = Number of moles of 18. From the formula, M f =
CO2 evolved V1 + V2
1 Given, V1 = 750 mL, M 1 = 0.5 M
= 0.01186 [M = molar mass of M 2CO3]
M V2 = 250 mL, M 2 = 2 M
1
M = = 84.3 g mol − 1 750 × 0.5 + 250 × 2 875
0.01186 = = = 0.875 M
750 + 250 1000
14. CxH y (g ) +  x +  O2 (g ) → xCO2 (g ) + H2O(l )
y y
Moles of solute
 4 30 mL
2 19. Molarity =
75 mL Volume of solution (L)
O 2 used = 20% of 375 = 75 mL 120
Moles of urea = =2
Inert part of air = 80% of 375 = 300 mL 60
Total volume of gases = CO2 + Inert part of air Weight of solution = Weight of solvent + Weight of solute
= 30 + 300 = 330 mL = 1000 + 120 = 1120 g
x 30 1120 g 1
= ⇒x=2 ⇒ Volume = × = 0.973 L
1 15 1.15 g / mL 1000 mL / L
y 2.000
x+ ⇒ Molarity = = 2.05M
4 = 75 ⇒ x + y = 5 0.973
1 15 4
⇒ x = 2, y = 12 ⇒ C2 H12 20. From the given relative abundance, the average weight of Fe can be
calculated as
15. We know the molecular weight of C8 H7 SO3Na 54 × 5 + 56 × 90 + 57 × 5
A= = 55.95
= 12 × 8 + 1 × 7 + 32 + 16 × 3 + 23 = 206 100
we have to find, mole per gram of resin. 21. 1.0 L of mixture X contain 0.01 mole of each [Co(NH3 )5 SO4 ]Br
∴ 1g of C8 H7 SO3Na has number of mole and [Co(NH3 )5 Br]SO4. Also, with AgNO3, only
weight of given resin 1 [Co(NH3 )5 SO4 ]Br reacts to give AgBr precipitate as
= = mol
Molecular, weight of resin 206 [Co(NH3 )5 SO4 ]Br + AgNO3 → [Co(NH3 )5 SO4 ]NO3 + AgBr
1.0 mol Excess 1.0 mol
Now, reaction looks like With BaCl 2, only [Co(NH3 )5 Br]SO4 reacts giving BaSO4
2C8 H7 SO3Na + Ca 2+ → (C8 H7 SO3 )2 Ca + 2Na precipitate as
Q 2 moles of C8 H7 SO3Na combines with 1 mol Ca 2+ [Co(NH3 )5 Br]SO4 + BaCl 2 → [Co(NH3 )5 Br]Cl 2 + BaSO4
1.0 mol Excess 1 mol
1 Hence, moles of Y and Z are 0.01 each.
∴1 mole of C8 H7 SO3Na will combine with mol Ca 2+
2
22. Number of atoms = Number of moles
1
∴ mole of C8 H7 SO3 Na will combine with × Avogadro’s number (N A)
206 24
1 1 1 Number of atoms in 24 g C = × NA = 2NA
× mol Ca 2+ = mol Ca 2+ 12
2 206 412
Some Basic Concepts of Chemistry 11

56 30. Number of molecules present in 36 g of water


Number of atoms in 56 g of Fe = NA = NA
56 36
27 = × N A = 2N A
Number of atoms in 27 g of Al = NA = NA 18
27
108 28
Number of atoms in 108 g of Ag = NA = NA Number of molecules present in 28 g of CO = × NA = NA
108 28
46
Hence, 24 g of carbon has the maximum number of atoms. Number of molecules present in 46 g of C2H5OH = × NA = NA
46
23. Mass of an electron = 9.108 × 10−31 kg 54
Number of molecules present in 54 g of N2O5 = × N A = 0.5 N A
Q 9.108 × 10−31 kg = 1.0 electron 108
1 1031 1 Here, NA is Avogadro’s number. Hence, 36 g of water contain
∴ 1 kg = −31
electrons = × the largest (2NA ) number of molecules.
9.108 × 10 9.108 6.023 × 1023
1 31. In a neutral atom, atomic number represents the number of
= × 108 mole of electrons protons inside the nucleus and equal number of electrons around
9.108 × 6.023
it. Therefore, the number of total electrons in molecule of CO2
24. Phosphorus acid is a dibasic acid as : = electrons present in one carbon atom
O
+ 2 × electrons present in one oxygen atom

H—P — OH only two replaceable hydrogens = 6 + 2 × 8 = 22.

Weight of a compound in gram (w)
OH 32. = Number of moles (n)
Molar mass (M )
Therefore, normality = molarity × basicity = 0.3 × 2 = 0.60
Number of molecules (N )
25. Molality is defined in terms of weight, hence independent of =
Avogadro number (NA )
temperature. Remaining three concentration units are defined in
terms of volume of solution, they depends on temperature. w (O2 ) N (O2 )
⇒ = …(i)
26. Molality of a solution is defined as number of moles of solute 32 NA
present in 1.0 kg (1000 g) of solvent. w (N2 ) N (N2 )
And = …(ii)
27. The balanced chemical reaction is 28 NA
3BaCl 2 + 2Na 3PO4 → Ba 3 (PO4 )2 + 6NaCl Dividing Eq. (i) by Eq. (ii) gives
In this reaction, 3 moles of BaCl 2 combines with 2 moles of N (O2 ) w (O2 ) 28 1 28 7
Na 3PO4. Hence, 0.5 mole of of BaCl 2 require = × = × =
N (N2 ) w (N2 ) 32 4 32 32
2
× 0.5 = 0.33 mole of Na 3PO4. 33. Molar mass of Na 2CO3⋅ xH2O .
3
Since, available Na 3PO4 (0.2 mole) is less than required mole (Atomic mass of Na = 23, C = 12, O = 16)
(0.33), it is the limiting reactant and would determine the = 23 × 2 + 12 + 48 + 18x
amount of product Ba 3 (PO4 )2. = 46 + 12 + 48 + 18x
Q 2 moles of Na 3PO4 gives 1 mole Ba 3 (PO4 )2 = (106 + 18x )
1
∴0.2 mole of Na 3PO4 would give × 0.2 = 0.1 mole Ba 3 (PO4 )2 Equivalent weight of Na 2CO3⋅ xH2O
2
Molar mass M
28. Unlike other metal carbonates that usually decomposes into = = = (53 + 9x )
n factor 2
metal oxides liberating carbon dioxide, silver carbonate on
heating decomposes into elemental silver liberating mixture of [ Here, m = molar mass and n factor = 2]
carbon dioxide and oxygen gas as : Weight
Gram equivalent =
Heat 1 Equivalent weight
Ag2CO3 (s) → 2Ag (s) + CO2 (g ) + O (g )
2 2
[Given, weight of Na 2CO3 ⋅ xH2O = 143
. g]
MW = 276 g 2 × 108 = 216 g
Hence, 2.76 g of Ag2CO3 on heating will give Hence, gram equivalent of
216 1.43
× 2.76 = 2.16g Ag as residue. Na 2CO3 ⋅ xH2O =
276 53 + 9x
Gmeq
29. The balanced chemical reaction of zinc with sulphuric acid and Normality =
NaOH are Vlitre
Zn + H2SO4 → ZnSO4 + H2 (g ) ↑ 143
.
01. =
Zn + 2NaOH + 2H2O → Na 2[ Zn(OH)4 ] + H2 (g ) ↑ 53 + 9x
Since, one mole of H2 (g ) is produced per mole of zinc with both 01.
sulphuric acid and NaOH respectively, hydrogen gas is As, volume = 100 mL = 0.1 L
produced in the molar ratio of 1:1 in the above reactions.
12 Some Basic Concepts of Chemistry

143
.
So, 10−2 = 36. Given,
53 + 9x O
53 + 9x = 143
9x = 90 NaOBr NH3/∆ Br2/KOH Br2(3-eqiv.)
A B C D
x = 10.00 H3O+ (60%) (50%) (50%)
AcOH
(100%)
34. The equations of chemical reactions occurring during the process
The products formed are
are
In the presence of oxygen O HO O H2N O
2PbS + 3O2 → 2PbO + 2SO2 …(i)
NaOBr NH3/∆
By self reduction
H3O+
2PbO + PbS → 3Pb + SO2
Acetophenone Benzoic acid Benzamide
Thus 3 moles of O2 produces 3 moles of Pb (60%) (50%)
Br2/KOH
10 mol
i.e. 32 × 3 = 96 g of O 2 produces 3 × 207 = 621 g of Pb i.e., i.e.,
6 mol 3 mol
So 1000 g (1kg) of oxygen will produce (A) (B)
621 NH2 NH2
× 1000 = 6468.75 g
96 Br Br
= 6.4687 kg ≈ 6.47 kg Br2(3-eqiv.)
AcOH
Alternative Method
From the direct equation, Aniline
Br (50%)
PbS + O2 → Pb + SO2 2,4,6-tribromo aniline i.e.,
32 g 207 g 1.5 mol
(100%)
i.e., (C)
So, 32 g of O 2 gives 207 g of Pb 1.5 mol
207 (D)
1 g of O 2 will give g of Pb
32 NH2
207 Br Br
1000g of O 2 will give × 1000 = 6468.75 g
32
= 6.46875 kg ≈ 6.47kg So, 1.5 mol of are produced from

35. The balanced equations are


Br
(1) 2MnCl 2 + 5K 2S2O 8 + 8H2O →
2KMnO 4 + 4 K 2SO 4 + 6H2SO 4 + 4HCl 10 moles of acetophenone.
NH2
(2) 2KMnO 4 + 5H2C 2O 4 + 3H2SO 4 → Br Br
K 2SO 4 + 2MnSO 4 + 8H2O +10CO 2
Molar mass of = 240 + 14 + 4 + 72 = 330
Given, mass of oxalic acid added = 225mg
225
So, millimoles of oxalic acid added = = 2.5 Br
90
NH2
Now from equation 2
Br Br
Millimoles of KMnO 4 used to react with oxalic acid=1 and
Millimoles of MnCl 2 required initially=1 Hence, amount of produced is 330 × 1.5 = 495 g
∴ Mass of MnCl 2 required initially = 1 × (55 + 71) = 126mg
Alternative Method Br
m moles of MnCl 2 = m moles of KMnO 4 = x (let) 37. Molar mass of CuSO4 ⋅ 5H2O
and M eq of KMnO 4 = M eq of oxalic acid
= 63.5 + 32 + 4 × 16 + 5 × 18
225
So, x×5= ×2 = 249.5 g
90
Also, molar mass represents mass of Avogadro number of
Hence, x = 1 molecules in gram unit, therefore
∴ m moles of MnCl 2 = 1 Q 6.023 × 1023 molecules of CuSO4 ⋅ 5H2O weigh 249.5 g
Hence mass of MnCl 2 = (55 + 71) × 1 = 126 mg. 249.5
∴ 1022 molecules will weigh × 1022 = 4.14 g
6.023 × 1023
Some Basic Concepts of Chemistry 13

Number of moles of solute Mass of HCl required for 200 mL 0.4 M HCl
38. Molarity =
Volume of solution in litre 200
= × 0.4 × 36.5 = 0.08 × 36.5 g
Weight of solute 1000 1000
= ×
Molar mass Volume in mL ∴ 0.365 g of HCl is present in 1.0 mL stock solution.
3 1000 0.08 × 36.5
= × = 0.4 M 0.08 × 36.5 g HCl will be present in = 8.0 mL
30 250 0.365
39. Considering density of water to be 1.0 g/mL, 18 mL of water is 44. Partial pressure of N2 = 0.001 atm,
18 g (1.0 mol) of water and it contain Avogadro number of T = 298 K, V = 2.46 dm 3.
molecules. Also one molecule of water contain
2 × (one from each H-atom) + 8 × (from oxygen atom) From ideal gas law : pV = nRT
pV 0.001 × 2.46
= 10 electrons. n(N2 ) = = = 10−7
RT 0.082 × 298
⇒ 1.0 mole of H2O contain = 10 × 6.023 × 1023
⇒ Number of molecules of N2 = 6.023 × 1023 × 10−7
= 6.023 × 1024 electrons.
= 6.023 × 1016
40. Carbon-12 isotope. According to modern atomic mass unit, one Now, total surface sites available
atomic mass unit (amu) is defined as one-twelfth of mass of an
atom of C-12 isotope, i.e. = 6.023 × 1014 × 1000 = 6.023 × 1017
1 20
1 amu (u) = × weight of an atom of C-12 isotope. Surface sites used in adsorption = × 6.023 × 1017
12 100
w w = 2 × 6.023 × 1016
41. Moles of solute, n1 = 1 ; Moles of solvent, n2 = 2
m1 m2 ⇒ Sites occupied per molecules
χ 1 (solute) = 0.1and χ 2 (solvent) = 0.9 Number of sites 2 × 6.023 × 1016
= = = 2
χ 1 n1 w1 m2 1 Number of molecules 6.023 × 1016
∴ = = ⋅ =
χ 2 n2 m1 w2 9 45. Initial millimol of CH3COOH = 100 × 0.5 = 50
Solute (moles) w1 × 1000 × 2
Molarity = = millimol of CH3COOH remaining after adsorption
Volume (L) m1 (w1 + w2 )
= 100 × 0.49 = 49
Total mass of solution  w1 + w2 
Note Volume = =  mL ⇒ millimol of CH3COOH adsorbed = 50 – 49 = 1
Density  2 
⇒ number of molecules of CH3COOH adsorbed
Solute (moles) w1 × 1000 1
Molality = = = × 6.023 × 1023 = 6.023 × 1020
Solvent (kg) m1 × w2 1000
Given, molarity = molality 3.01 × 102
⇒ Area covered up by one molecule =
2000 w1 1000 w1 6.02 × 1020
hence, =
m1 (w1 + w2 ) m1 w2 = 5 × 10−19 m 2
w2 1
∴ = ⇒ w1 = w2 = 1 46. Mass of 1.0 L water = 1000 g
w1+ w2 2
1000
⇒ Molarity = = 55.56 mol L−1
w1 m2 1 m (solute) 18
∴ = ⇒ 1 =9
m1 w2 9 m2 (solvent)
47. Volume of one cylinderical plant virus = πr2l
42. PLAN This problem can be solved by using concept of conversion of = 3.14 (75 × 10−8 )2 × 5000 × 10−8 cm 3 = 8.83 × 10−17 cm 3
molarity into molality.
Volume of a virus
Molarity = 3.2 M ⇒ Mass of one virus =
Specific volume
Let volume of solution = 1000 mL = Volume of solvent
8.83 × 10−17 cm 3
Mass of solvent = 1000 × 0.4 = 400 g = = 1.1773 × 10−16 g
Since, molarity of solution is 3.2 molar 0.75 cm 3 g−1
∴ n solute = 3.2 mol ⇒ Molar mass of virus
3.2 = Mass of one virus × Avogadro’s number
Molality (m) = =8
400 / 1000 = 1.1773 × 10−16 × 6.023 × 1023 g
Hence, correct integer is (8).
= 70.91 × 106 g
43. Mass of HCl in 1.0 mL stock solution
48. Molar mass of Glauber’s salt (Na 2SO4 ⋅ 10H2O)
29.2
= 1.25 × = 0.365 g = 23 × 2 + 32 + 64 + 10 × 18 = 322g
100
14 Some Basic Concepts of Chemistry

80.575 If we consider 100 mL solution, weight of solution = 184 g


⇒ Mole of Na 2SO4 ⋅ 10H2O in 1.0 L solution = = 0.25
322 Weight of H2O in 100 mL solution = 184 – 93 = 91 g
⇒ Molarity of solution = 0.25 M Moles of solute
⇒ Molality = × 1000
Also, weight of 1.0 L solution = 1077.2 g Weight of solvent (g)
weight of Na 2SO4 in 1.0 L solution = 0.25 × 142 = 35.5 g 93 1000
⇒ Weight of water in 1.0 L solution = 1077.2 – 35.5 = 1041.7 g = × = 10.42
98 91
0.25
⇒ Molality = × 1000 = 0.24 m 52. Heating below 600°C converts Pb(NO3 )2 into PbO but to
1041.7
Mole of Na 2SO4 NaNO3 into NaNO2 as
Mole fraction of Na 2SO4 = ∆ 1
Mole of Na 2SO4 + Mole of water Pb(NO3 )2 → PbO(s) + 2NO2 ↑ + O ↑
2 2
0.25 MW : 330 222
= = 4.3 × 10−3. ∆ 1
1041.7 NaNO3 → NaNO2 (s) + O ↑
0.25 + 2 2
18 MW : 85 69
28
49. Compound B forms hydrated crystals with Al 2 (SO4 )3. Also, B is Weight loss = 5 × = 1.4 g
100
formed with univalent metal on heating with sulphur. Hence,
compound B must has the molecular formula M 2SO4 and ⇒ Weight of residue left = 5 – 1.4 = 3.6 g
compound A must be an oxide of M which reacts with sulphur to Now, let the original mixture contain x g of Pb(NO3 )2.
give metal sulphate as Q 330 g Pb(NO3 )2 gives 222 g PbO
A + S → M 2SO4 222 x
B
∴ x g Pb(NO3 )2 will give g PbO
330
Q 0.321 g sulphur gives 1.743 g of M 2SO4
Similarly, 85 g NaNO3 gives 69 g NaNO2
∴ 32.1 g S (one mole) will give 174.3 g M 2SO4
69 (5 − x )
Therefore, molar mass of M 2SO4 = 174.3 g ⇒ (5 – x) g NaNO3 will give g NaNO2
85
⇒ 174.3 = 2 × Atomic weight of M + 32.1 + 64
222 x 69 (5 − x )
⇒ Atomic weight of M = 39, metal is potassium (K) ⇒ Residue : + = 3.6 g
330 85
K2SO4 on treatment with aqueous Al 2 (SO4 )3 gives potash-alum.
Solving for x gives, x = 3.3 g Pb(NO3 )2 ⇒ NaNO3 = 1.7 g.
K2SO4 + Al 2 (SO4)3 + 24H2O → K2SO4Al 2 (SO4)3 ⋅ 24H2O
B C 53. Reactions involved are
If the metal oxide A has molecular formula MOx, two moles of it C2H6 + Br2 → C2H5Br + HBr
combine with one mole of sulphur to give one mole of metal 2C2H5Br + 2Na → C4H10 + 2NaBr
sulphate as Actual yield of C4H10 = 55 g which is 85% of theoretical yield.
2KOx + S → K2SO4
55 × 100
⇒ x = 2, i.e. A is KO2. ⇒ Theoretical yield of C4H10 = = 64.70 g
85
50. The reaction involved is Also, 2 moles (218 g) C2H5Br gives 58 g of butane.
3Pb(NO3 )2 + Cr2 (SO4 )3 → 3PbSO4 (s) ↓ + 2Cr(NO3 )3 ⇒ 64.70 g of butane would be obtained from
millimol of Pb(NO3 )2 taken = 45 × 0.25 = 11.25 2
× 64.70 = 2.23 moles C2H5Br
millimol of Cr2 (SO4 )3 taken = 2.5 58
Here, chromic sulphate is the limiting reagent, it will determine Also yield of bromination reaction is only 90%, in order to have
the amount of product. 2.23 moles of C2H5Br, theoretically
Q 1 mole Cr2 (SO4 )3 produces 3 moles PbSO4. 2.23 × 100
= 2.48 moles of C2H5Br required.
∴ 2.5 millimol Cr2 (SO4 )3 will produce 7.5 millimol PbSO4. 90
Hence, mole of PbSO4 precipitate formed = 7.5 × 10−3 Therefore, moles of C2H6 required = 2.48
⇒ Volume of C2H6 (NTP) required = 2.48 × 22.4 = 55.55 L.
Also, millimol of Pb(NO3 )2 remaining unreacted
34.2
11.25 – 7.50 = 3.75 54. Moles of sugar = = 0.1
342
⇒ Molarity of Pb(NO3 )2 in final solution
Moles of water in syrup = 214.2 – 34.2 = 180 g
millimol of Pb(NO3 )2 3.75 Moles of solute
= = = 0.054 M Therefore, (i) Molality = × 1000
Total volume 70 Weight of Solvent (g)
Also, millimol of Cr(NO3 )2 formed 0.1
= × 1000 = 0.55
= 2 × millimol of Cr2 (SO4 )3 reacted 180
5 Mole of sugar
⇒ Molarity of Cr(NO3 )2 = = 0.071 M (ii) Mole fraction of sugar =
70 Mole of sugar + Mole of water
51. 93% H2SO4 solution weight by volume indicates that there is 0.1
= = 9.9 × 10−3
93 g H2SO4 in 100 mL of solution. 0.1 + 10
Some Basic Concepts of Chemistry 15

55. From the given elemental composition, empirical formula can 58. In the present case, V ∝ n (Q all the volumes are measured
be derived as : under identical conditions of temperature and pressure) Hence,
Element C H O the reaction stoichiometry can be solved using volumes as :
Weight % 69.77 11.63 18.60  y y
CxH y (g ) +  x +  O2 (g ) → x CO2 (g ) + H2O (l )
Mole % 5.81 11.63 1.1625 (obtained by  4 2
dividing from M ) volume of CO2 (g ) + O2 (g ) (remaining unreacted) = 25
Simple ratio 5 10 1 ⇒ Volume of CO2 (g ) produced
Hence, empirical formula is C5H10O and empirical formula = 10 mL (15 mL O2 remaining)
weight is 86. Q 1 mL CxH y produces x mL of CO2
Since, empirical formula weight and molecular weight both are ∴ 5 mL CxH y will produce 5x mL of CO2 = 10 mL ⇒ x = 2
(86), empirical formula is the molecular formula also.  y
Also, 1 mL CxH y combines with  x +  mL of O2
Also, the compound does not reduce Fehling’s solution,  4
therefore it is not an aldehyde, but it forms bisulphite, it must be
 y
a ketone. 5 mL CxH y will combine with 5  x +  mL of O2
 4
Also, it gives positive iodoform test, it must be a methyl ketone.
 y
O ⇒ 5  x +  = 15 (15 mL of O2 out of 30 mL)
 4
 (remaining unreacted)
C3H7 — C — CH3
⇒ y = 4, hence hydrocarbon is C2H4.
Based on the above information, the compound may be one of
the following : 59. Oxides of sodium and potassium are converted into chlorides
according to following reactions :
O CH3 O
   Na 2O + 2HCl → 2NaCl + H2O
CH3CH2CH2— C — CH3 or CH3 — CH— C — CH3 K2O + 2HCl → 2KCl + H2O
2-pentanone 3-methyl -2-butanone
Finally all the chlorides of NaCl and KCl are converted into
56. (a) Let us consider 1.0 L solution for all the calculation. AgCl, hence
(i) Weight of 1 L solution = 1250 g moles of (NaCl + KCl) = moles of AgCl
Weight of Na 2S2O3 = 3 × 158 = 474 g (one mole of either NaCl or KCl gives one mole of AgCl)
474 Now, let the chloride mixture contain x g NaCl.
⇒ Weight percentage of Na 2S2O3= × 100 = 37.92
1250 x 0.118 − x 0.2451
⇒ + =
(ii) Weight of H2O in 1 L solution = 1250 − 474 = 776 g 58.5 74.5 143.5
3 Solving for x gives x = 0.0338 g (mass of NaCl)
Mole fraction of Na 2S2O3 = = 0.065
776 ⇒ Mass of KCl = 0.118 – 0.0338 = 0.0842 g
3+
18 1
3×2 Also, moles of Na 2O = × moles of NaCl
(iii) Molality of Na + = × 100 = 7.73 m 2
776 1 0.0338
57. (a) After passing through red-hot charcoal, following reaction ⇒ Mass of Na 2O = × × 62 = 0.0179 g
2 58.5
occurs
1 0.0842
C(s) + CO2 (g ) → 2CO(g ) Similarly, mass of K2O = × × 94 = 0.053 g
2 74.5
If the 1.0 L original mixture contain x litre of CO2, after
0.0179
passing from tube containing red-hot charcoal, the new ⇒ Mass % of Na 2O = × 100 = 3.58 %
volumes would be : 0.5
2x (volume of CO obtained from CO2) + 1 0.053
Mass % of K2O = × 100 = 10.6 %
– x(original CO) = 1 + x =1.6 (given) 0.5
⇒ x = 0.6 60. From the vapour density information
Hence, original 1.0 L mixture has 0.4 L CO and 0.6 L of CO2 , Molar mass = Vapour density × 2 (Q Molar mass of H2 = 2)
i.e. 40% CO and 60% CO2 by volume. = 38.3 × 2 = 76.6
(b) According to the given information, molecular formula of Now, let us consider 1.0 mole of mixture and it contains
the compound is M 3N2. Also, 1.0 mole of compound has 28 g x mole of NO2.
of nitrogen. If X is the molar mass of compound, then :
⇒ 46 x + 92 (1 − x ) = 76.6 ⇒ x = 0.3348
28
X × = 28 100
100 Also, in 100 g mixture, number of moles =
76.6
⇒ X = 100 = 3 × Atomic weight of M + 28 100
72 ⇒ Moles of NO2 in mixture = × 0.3348 = 0.437
⇒ Atomic weight of M = = 24 76.6
3
16 Some Basic Concepts of Chemistry

61. Most of the elements found in nature exist as a mixture of The difference in the volume of NaOH solution between the end
isotopes whose atomic weights are different. The atomic weight point and the equivalence point is not significant for most of the
of an element is the average of atomic weights of all its naturally commonly used indicators as there is a large change in the pH
occurring isotopes. value around the equivalence point. Most of them change their
colour across this pH change.
62. Average atomic weight
Σ Percentage of an isotope × Atomic weight 3. 100 mL (cm3) of hexane contains 0.27 g of fatty acid.
= In 10 mL solution, mass of the fatty acid,
100
10.01x + 11.01 (100 − x) 0.27
⇒ 10.81 = ⇒ x = 20% m= × 10 = 0.027 g
100 100
Density of fatty acid, d = 0.9 g cm−3
Therefore, natural boron contains 20% (10.01) isotope and 80%
other isotope. ∴Volume of the fatty acid over the watch glass,
m 0.027
V = = = 0.03 cm3
Topic 2 Equivalent Concept, Neutralisation d 0.9
and Redox Titration Let, height of the cylindrical monolayer = h cm
Q Volume of the cylinder = Volume of fatty acid
1. In disproportionation reactions, same element undergoes
oxidation as well as reduction.
Reduction
e.g.
+1 +2 0 h cm
2CuBr CuBr2 +Cu

Oxidation
10 cm
Here, CuBr get oxidised to CuBr2 and also it get reduced to Cu.
Other given reactions and their types are given below. ⇒ V = πr 2 × h
Reduction
V 0.03 cm3
+7 ⇒ h= =
2 MnO4
– –
+10I + 16 H
+ +2
2Mn + 5I2 + 8H2O πr2 3 × (10)2 cm2
Oxidation = 1 × 10−4cm = 1 × 10−6 m
In the given reaction, MnO−4 get oxidised to Mn 2+ and I− get 4. The oxidation of a mixture of one mole of each of
reduced to I2. It is an example of redox reaction. The reaction FeC2O4 , Fe2 (C2O4 )3 FeSO4 and
takes place in acidic medium. Fe2 (SO4 )3 in acidic medium with KMnO4 is as follows :
2KMnO4 → K 2MnO4 + MnO2 + O2 FeC2O4 + KMnO4 → Fe3 + + CO2 + Mn 2+ …(i)
The given reaction is an example of decomposition reaction. Fe2(C2O4 )3 + KMnO4 → Fe3 + + CO2 + Mn 2+ …(ii)
Here, one compound split into two or more simpler compounds,
atleast one of which must be in elemental form. FeSO4 + KMnO4 → Fe 3+
+ SO24− + Mn 2+
…(iii)
2NaBr + Cl2 → 2NaCl + Br2 Change in oxidation number of Mn is 5. Change in oxidation
The given reaction is an example of displacement reaction. In number of Fe in (i), (ii) and (iii) are +3, + 6, + 1, respectively.
this reaction, an atom (or ion) replaces the ion (or atom) of neq KMnO 4 = neq [ FeC 2O 4 + Fe 2 ( C 2O 4 ) 3 + FeSO 4 ]
another element from a compound. n × 5 = 1× 3 + 1× 6 + 1× 1
2. The graph that shows the correct change of pH of the titration ∴ n=2
mixture in the experiment is 5. Given, W Ca (HCO 3) 2 = 0.81 g
W Mg (HCO ) = 0.73 g
3 2
M Ca (HCO ) = 162 g mol −1,
pH 3 2

M Mg(HCO 3 ) 2 = 146 mol −1


V H 2 O = 100mL
V(mL)
Now, neq (CaCO3) = neq [Ca(HCO3 )2 ]+ neq [Mg(HCO3 )2 ]
In this case, both the titrants are completely ionised.
+ − W 0.81 0.73
×2= ×2+ ×2
HCl + NaOH - N a Cl + H O 2 100 162 146
⊕ W
As H is added to a basic solution, [ OHÈ ] decreases and [ H+ ] ∴ = 0.005 + 0.005
increases. Therefore, pH goes on decreasing. As the equivalence 100
point is reached,[OHÈ ] is rapidly reduced. After this point [OH È] W = 0.01 × 100 = 1
1
decreases rapidly and pH of the solution remains fairly constant. Thus, hardness of water sample = × 106 = 10,000 ppm
Thus, there is an inflexion point at the equivalence point. 100
Some Basic Concepts of Chemistry 17

6. The reaction takes place as follows, 12x 6


= (given and molar mass of C = 12, H = 1)
H2C2O4 + 2NaOH → Na 2C2O4 + 2H2O y 1

Now, 50 mL of 0.5 M H2C2O4 is needed to neutralize 25 mL of Now, after calculating this ratio look for condition 2 given in the
NaOH. question i.e. quantity of oxygen is half of the quantity required
to burn one molecule of compoundC xH y completely to CO2 and
∴ Meq of H2C2O4 = Meq of NaOH H2O. We can calculate number of oxygen atoms from this as
50 × 0.5 × 2 = 25 × M NaOH× 1 consider the equation.
M NaOH = 2M  y y
CxH y + x + O2 → xCO2 + H2O
Number of moles  4  2
Now, molarity =
Volume of solution (in L)  y  y
Number of oxygen atoms required = 2 × x + = 2x +
Weight / molecular mass  4   2 
=
Volume of solution (in L) 1 y  y
Now given, z = 2x + = x+
w 1000 2  2   4 
2 = NaOH ×
40 50 Here we consider x and y as simplest ratios for C and H so now
2 × 40 × 50 putting the values of x and y in the above equation.
wNaOH = = 4g
1000  y  2
z= x+ = 1+ = 1.5
Thus, (*) none option is correct.  4   4 
7. The reaction of HCl with Na 2CO3 is as follows: Thus, the simplest ratio figures for x , y and z are x = 1, y = 2 and
z = 15
.
2HCl + Na 2CO3 → 2NaCl + H2O + CO2 Now, put these values in the formula given i.e.
We know that, M eq of HCl = M eq of Na 2CO3 CxH yOz = C1H2O1.5
25 30 So, empirical formula will be [C1H2O1.5 ] × 2 = C2H4O3
× 1 × M HCl = × 0.1 × 2
1000 1000 10. Methyl orange show Pinkish colour towards more acidic
30 × 0.2 6 medium and yellow orange colour towards basic or less acidic
M HCl = = M media. Its working pH range is
25 25
The reaction of HCl with NaOH is as follows: 3.9 –4.5
Pinkish Yellow
NaOH + HCl → NaCl + H2O Red orange
Also, M eq of HCl = M eq of NaOH
Weak base have the pH range greater than 7. When methyl
6 V 30 orange is added to this weak base solution it shows yellow
×1× = × 0.2 × 1
25 1000 1000 orange colour.
V = 25mL Now when this solution is titrated against
strong acid the pH move towards more acidic 1
8. Reaction of oxalate with permanganate in acidic medium. range and reaches to end point near 3.9 where
NH2
5C2O24− + 2MnO−4 → 10CO2 + 2Mn 2+ + 8H2O yellow orange colour of methyl orange HN
changes to Pinkish red resulting to similar In conjugation
n-factor : (4 − 3 ) × 2 = 2 (7 − 2 ) = 5 change in colour of solution as well.
Number of mole 5 2 10

11. H2O2 acts as an oxidising as well as reducing agent, because
⇒ 5C2O42– ions transfer 10e to produce 10 molecules of CO2.
oxidation number of oxygen in H2O2 is −1. So, it can be oxidised
So, number of electrons involved in producing 10 molecules of to oxidation state 0 or reduced to oxidation state –2.
CO2 is 10. Thus, number of electrons involved in producing 1 H2O2 decomposes on exposure to light. So, it has to be stored in
molecule of CO2 is 1. plastic or wax lined glass bottles in dark for the prevention of
9. We can calculate the simplest whole number ratio of C and H exposure. It also has to be kept away from dust.
from the data given, as 12. n-factor of dichromate is 6.
Element Relative Molar Relative Simplest whole Also, n-factor of Mohr’s salt is 1 as :
mass mass mole number ratio O. A
6 0.5 FeSO4 (NH4 )2 SO4⋅ 6H2O → Fe3+
C 6 12 = 0.5 =1 Mohr’s salt
12 0.5
H 1 1 1 1 Q 1 mole of dichromate = 6 equivalent of dichromate.
=1 =2
1 0.5 ∴ 6 equivalent of Mohr’s salt would be required.
Since, n-factor of Mohr’s salt is 1, 6 equivalent of it would also
Alternatively this ratio can also be calculated directly in the
be equal to 6 moles.
terms of x and y as
Hence, 1 mole of dichromate will oxidise 6 moles of Mohr’s salt.
18 Some Basic Concepts of Chemistry

13. The following reaction occur between S2O32− and Cr2O2−


7 :
Here n-factor is the net change in oxidation number per formula
unit of oxidising or reducing agent. In the present case, n-factor
26H ++
3S2O32−+ 4Cr2O72−
→ 6SO24− + 8Cr 3+ + 13H2O is 2 because equivalent weight is half of molecular weight. Also,
Change in oxidation number of Cr2O72− per formula unit is 6 (it is 1
n-factor MnSO4 → Mn 2O3 1 (+ 2 → + 3)
always fixed for Cr2O2−
7 ).
2
Molecular weight MnSO4 → MnO2 2 (+ 2 → + 4)
Hence, equivalent weight of K2Cr2O7 = −
6 MnSO4 → MnO4 5 (+ 2 → + 7)
14. It is an example of disproportionation reaction because the same MnSO4 → MnO42− 4 (+ 2 → + 6)
species (ClO− ) is being oxidised to ClO−3 as well as reduced to Cl − . Therefore, MnSO4 converts to MnO2.
15. Oxalic acid dihydrate H2C2O4 ⋅ 2H2O : mw = 126 24. PLAN This problem includes concept of redox reaction. A redox
It is a dibasic acid, hence equivalent weight = 63 reaction consists of oxidation half-cell reaction and reduction
6.3 1000 half-cell reaction. Write both half-cell reactions, i.e. oxidation
⇒ Normality = × = 0.4 N half-cell reaction and reduction half-cell reaction.Then balance
63 250 both the equations.
⇒ N 1V1 = N 2V2 Now determine the correct value of stoichiometry of H2SO4.
⇒ 0.1 × V1 = 0.4 × 10 Oxidation half-reaction, 2 I − → I2 + 2 e − …(i)
Hence, V1 = 40 mL Here, I − is converted into I2. Oxidation number of I is increasing
16. In MnO−4 , oxidation state of Mn is +7 from –1 to 0 hence, this is a type of oxidation reaction.
Reduction half-reaction
In Cr(CN)63− , oxidation state of Cr is +3
6H+ + ClO3− + 6e− → Cl − + 3H2O …(ii)
In NiF62− , Ni is in + 4 oxidation state. l
Here, H2 O releases as a product. Hence, option (d) is
In CrO2Cl 2, oxidation state of Cr is +6. correct.
17. In S8 , oxidation number of S is 0, elemental state. Multiplying equation (i) by 3 and adding in equation (ii)
In S2F2, F is in – 1 oxidation state, hence S is in + 1 oxidation 6I− + ClO3− + 6H+ → Cl − + 3I2 + 3H2O
state.
In H2S, H is in +1 oxidation state, hence S is in – 2 oxidation 6I− + ClO−3 + 6H2SO4 → Cl − + 3I2 + 3H2O + 6HSO−4
state. l
Stoichiometric coefficient of HSO−4 is 6.
18. The balanced redox reaction is : Hence, option (a), (b) and (d) are correct.
3MnO−4 + 5FeC2O4 + 24H+ → 3Mn 2+ + 5Fe3+ 25. Oxalic acid solution titrated with NaOH solution using
+ 10CO2 + 12H2O phenolphthalein as an indicator.
Q 5 moles of FeC2O4 require 3 moles of KMnO4
H2 C2 O4 + 2NaOH → Na 2 C2 O4 + 2H2 O
3
∴ 1 mole of FeC2O4 will require mole of KMnO4. Equivalent of H2C2O4 reacted = Equivalent of NaOH reacted
5
5 × 2 × 01
. 9 × M(NaOH) × 1
19. The balanced chemical reaction is : = =
1000 1000
2MnO−4 + 5SO23− + 6H+ → 2Mn 2+ + 5SO24− + 3H2O 1
M (NaOH) = = 0.11
Q 5 moles SO2−
3 reacts with 2 moles of KMnO 4 9
2
∴ 1 mole of SO2−
3 will react with mole KMnO4. 26. Aluminium reacts with sulphuric acid to form aluminium
5 sulphate and hydrogen.
20. The balanced redox reaction is :
2Al + 3H2 SO4 → Al 2 (SO4 )3 + 3H2
2MnO4− + 5C2O24− + 16H + → 2Mn 2+ + 10CO2 + 16H2O  5. 4


= 0. 2 mol   50 × 5 
 27   = 0. 25 mol 
 1000 
Hence, the coefficients of reactants in balanced reaction are 2, 5
and 16 respectively. H2SO 4 is limiting reagent and moles of H2 (g ) produced
= 0.25 mol
21. Volume strength of H2O2 = Normality × 5.6 = 1.5 × 5.6 = 8.4 V
Using ideal gas equation,
22. In Ba(H2PO2 )2, oxidation number of Ba is +2. Therefore, pV = nRT
H2PO2− : 2 × (+1) + x + 2 × (−2) = − 1 0.25 × 0.082 × 300
⇒ V = ⇒ 6.15 L
⇒ x=+1 1 atm
23. Equivalent weight in redox system is defined as : 27. Given, volume of solution = 20.0 mL
Molar mass
E= Impure sample of H2O2 = 0.2 g
n-factor
Mass of KMnO4 = 0.316 g
Some Basic Concepts of Chemistry 19

Impure H2O2 react with KMnO4 (acidic) 30. If x is the oxidation state of Cu then :
+7 −1 +2 0
3 + 2 × 2 + 3x + 7 × (− 2) = 0 ⇒ x = 7/ 3
KMnO4 + H2O2 → Mn + O2
KMnO 4 acts as an oxidising agent, 31. Na2S4O6 is a salt of H2S4O6 which has the following structure
+7 +2 O O
Mn + 5e− → Mn (valency factor = 5) (0) (v)
−1 0

HO S S S S OH
O2 → O2 + 2e (valency factor = 2)
We have to compare both KMnO4 and H2O2. O O
(Mass equivalent) H O2 = (Mass equivalent) KMnO4 ⇒ Difference in oxidation number of two types of sulphur = 5
2

Weight 32. Only F and Na show only one non-zero oxidation state.
× 1000
molecular weight / valence factor O = O− , O2− , O2+ ;
weight Cl = − 1 to + 7
= × 1000
molecular weight / valence factor N = − 3 to + 5
(weight)H 2 O 2 0.316 P = − 3 to + 5
× 1000 = × 1000
34 / 2 158 / 5 Sn = + 2, + 4
0.316 34 Tl = + 1, + 3 (rare but does exist)
(weight)H2 O2 = ×5×
158 2 Ti = + 2, + 3, + 4
26.86
= 33. Average titrate value is 25.15, but the number of significant
158 figure cannot be greater than the same in either of them being
(weight)H2 O2 = 017
. g manipulated.

(Pure)H2 O2 34. The balanced reaction is


(Purify) H
2O2
= × 100 6CaO + P4O10 → 2Ca 3 (PO4 )2
(Impure)H2 O2
852
017
. Moles of P4O10 = =3
= × 100 = 85% 284
0.2 Moles of CaO required = 3 × 6 = 18
28. Balanced equations of reactions used in the problem are as follows Mass of CaO required = 18 × 56 = 1008 g
(i) ( NH4)2SO 4 + Ca(OH)2 → CaSO 4 ⋅ 2H2O + 2NH3 35. Meq of oxalate = 10 × 0.2 × 2 = 4
1 mol 1 mol 2 mol
132 g 172 g (2 × 17) = 34 g Meq of MnO2 formed = Meq of oxalate = 4
(ii) NiCl 2 ⋅ 6H2O + 6NH3 → [Ni(NH3)6] Cl 2 + 6H2O Meq of KMnO4 in 20 mL = 4
1 mol
238 g
6 mol
102 g
1 mol
232 g
⇒ Normality of H2O2 × 20 = 4
⇒ Normality of H2O2 = 0.20 N
Now, in Eq. (i)
0.20
if, 1584 g of ammonium sulphate is used. ⇒ Molarity of H2O2 = = 0.10 M
2
1584
i.e., 1584 g (NH4 )2 SO4 = = 12 mol The balanced reactions are
132
2KMnO4 + 5H2O2 + 3H2SO4 → 2MnSO4 + 5O2
So, according to the Eq. (i) given above 12 moles of (NH4 )2 SO4
+ K2SO4 + 8H2O
produces
MnO2 + Na 2C2O4 + 2H2SO4 → MnSO4 + Na 2SO4
(a) 12 moles of gypsum
+ 2CO2 + 2H2O
(b) 24 moles of ammonia
36. The balanced chemical reaction is
Here, 12 moles of gypsum = 12 × 172 = 2064 g
CuCO3 + H2SO4 → CuSO4 + H2O + CO2
and 24 moles of NH3 = 24 × 17 = 408g
0.5 × 1000
Further, as given in question, millimol of CuCO3 = = 4.048
123.5
24 moles of NH3 produced in reaction (i) is completly utilised
⇒ Millimol of H2SO4 required = 4.048
by 952g or 4 moles of NiCl 2 ⋅ 6H2O to produce 4 moles of
[Ni(NH3 )6 ] Cl 2. Q Millimol = Molarity × Volume (in mL)
So, 4 moles of [Ni(NH3 )6 ] Cl 2 = 4 × 232 = 928gms 4.048
⇒ Volume = = 8.096 mL
Hence, total mass of gypsum and nickel ammonia coordination 0.50
compound [Ni(NH3 )6 ] Cl 2 = 2064 + 928 = 2992 37. The redox reaction involved are
29. Both assertion and reason are factually true but the reason does IO−3 + 5I− + 6H+ → 3I2 + 3H2O
not exactly explain the assertion. The correct explanation is, I2 + 2S2O32− → 2I− + S4O62−
methyl orange and phenolphthalein changes their colour at
different pH.
20 Some Basic Concepts of Chemistry

0.1 4
millimol of KIO3 used = × 1000 = 0.467 Vol % of CH4 = × 100= 20%
214 20
⇒ millimol of I2 formed = 3 × 0.467 = 1.4 Vol % of He = 30%
⇒ millimol of Na 2S2O3 consumed = 2 × 1.4 = 2.8
2.8 41. The redox reaction involved is :
⇒ Molarity of Na 2S2O3 = = 0.062 M H2O2 + 2I− + 2H+ → 2H2O + I2
45
If M is molarity of H2O2 solution, then
38. Meq of H2O2 = Meq of I2 = Meq of Na 2S2O3
0.508 × 1000
If N is normality of H2O2, then 5M = (Q 1 mole H2O2 ≡≡ 1 mole I2)
254
N × 25 = 0.3 × 20 ⇒ N = 0.24 ⇒ M = 0.4
⇒ Volume strength = N × 5.6 = 1.334 V Also, n-factor of H2O2 is 2, therefore normality of H2O2 solution
39. Let the original sample contains x millimol of Fe3O4 and is 0.8 N.
y millimol of Fe2O3. In the first phase of reaction, ⇒ Volume strength = Normality × 5.6 = 0.8 × 5.6 = 4.48 V
Fe3O4 + I− → 3Fe2+ + I2 (n-factor of Fe3O4 = 2) 42. The reaction is

Fe2O3 + I → 2Fe 2+
+ I2 (n-factor of Fe2O3 = 2) KIO3 + 2KI + 6HCl → 3ICl + 3KCl + 3H2O
⇒ Meq of I2 formed = Meq (Fe3O4 + Fe2O3 ) KIO3 required for 20 mL original KI solution = 3 millimol.
= Meq of hypo required ⇒ 7.5 millimol KIO3 would be required for original 50 mL KI.
⇒ 2x + 2 y = 11 × 0.5 × 5 = 27.5 …(i) ⇒ Original 50 mL KI solution contain 15 millimol of KI.
Now, total millimol of Fe2+ formed = 3x + 2 y. In the reaction
After AgNO3 treatment 5 millimol of KIO3 is required, i.e. 10
Fe2+ + MnO−4 + H+ → Fe3+ + Mn 2+ millimol KI is remaining.
n-factor of Fe2+ = 1 ⇒ 5 millimol KI reacted with 5 millimol of AgNO3.
⇒ Meq of MnO−4 = Meq of Fe 2+ 5
⇒ Mass of AgNO3 = × 170 = 0.85 g
1000
⇒ 3x + 2 y = 12.8 × 0.25 × 5 × 2 = 32 …(ii)
Solving Eqs. (i) and (ii), we get ⇒ Mass percentage of AgNO3 = 85%
x = 4.5 43. CO2 is evolved due to following reaction :
and y = 9.25 2NaHCO3 → Na 2CO3 + H2O + CO2
4.5
⇒ Mass of Fe3O4 = × 232 = 1.044 g pV 750 123.9 1
1000 Moles of CO2 produced = = × ×
RT 760 1000 0.082 × 298
1.044
% mass of Fe3O4 = × 100 = 34.80%
3 = 5 × 10−3

Mass of Fe2O3 =
9.25
× 160 = 1.48 g ⇒ Moles of NaHCO3 in 2 g sample = 2 × 5 × 10−3 = 0.01
1000 ⇒ millimol of NaHCO3 in 1.5 g sample
1.48
% mass of Fe2O3 = × 100 = 49.33% 0.01
3 = × 1.5 × 1000= 7.5
2
40. The reaction involved in the explosion process is Let the 1.5 g sample contain x millimol Na 2CO3, then
1
CO(g ) + O (g ) → CO2 (g ) 2x + 7.5 = millimol of HCl = 15
2 x2
x mL x mL
mL ⇒ x = 3.75
2
CH4 (g ) + 2O2 (g ) → CO2 (g ) + 2H2O(l ) 7.5 × 84
⇒ Mass of NaHCO3 = = 0.63 g
y mL 2 y mL y mL 1000
The first step volume contraction can be calculated as : 3.75 × 106
Mass of Na 2CO3 = = 0.3975 g
 x  1000
 x + + y + 2 y − (x + y) = 13 0.63
 2  ⇒ % mass of NaHCO3 = × 100 = 42 %
⇒ x + 4 y = 26 …(i) 1.50
0.3975
The second volume contraction is due to absorption of CO2. % mass of Na 2CO3 = × 100 = 26.5%
1.5
Hence, x + y = 14 …(ii)
44. Mass of Fe2O3 = 0.552 g
Now, solving equations (i) and (ii), 0.552
millimol of Fe2O3 = × 1000 = 3.45
x = 10 mL, y = 4 mL and volume of He = 20 – 14 = 6 mL 160
⇒ Vol % of CO =
10
× 100 = 50% During treatment with Zn-dust, all Fe3+ is reduced to Fe2 + ,
20 hence
Some Basic Concepts of Chemistry 21

millimol of Fe2 + (in 100 mL) = 3.45 × 2 = 6.90 Because X can be represented by two formula of which one
6.90 gives a dihydroxy compound with KOH indicates that X has two
⇒ In 25 mL aliquot, = 1.725 millimol Fe2+ ion. chlorine atoms per molecule.
4
Finally Fe2+ is oxidised to Fe3+ , liberating one electron per Fe2+ ⇒ X = C2H4Cl 2 with two of its structural isomers.
ion. Therefore, total electrons taken up by oxidant. Cl— CH2— CH2 — Cl and CH3— CHCl 2
= 1.725 × 10−3 × 6.023 × 1023 I II
20 On treatment with KOH, I will give ethane-1, 2-diol, hence it is
= 1.04 × 10
Y. Z on treatment with KOH will give ethanal as
45. With KMnO4, oxalate ion is oxidised only as : ClCH2CH2Cl + OH− → CH2 — CH2
5C2O42− + 2MnO−4 + 16H+ → 2Mn 2+ + 10CO2 + 8H2O  
OH OH
Let, in the given mass of compound, x millimol of C2O2−
4 ion is (Y )
present, then –H2 O
CH3CHCl 2 + KOH → CH3CH(OH)2 → CH3CHO
Meq of C2O42− = Meq of MnO−4 Unstable (Z )
⇒ 2x = 0.02 × 5 × 22.6 ⇒ x = 1.13
48. Let the n-factor of KMnO4 in acid, neutral and alkaline media
At the later stage, with I − , Cu 2+ is reduced as :
are N 1, N 2 and N 3 respectively. Also, same volumes of reducing
2Cu 2+ + 4I − → 2CuI + I2 agent is used everytime, same number of equivalents of KMnO4
would be required every time.
and I2 + 2S2O32− → 2I− + S4O62−
100 5
Let there be x millimol of Cu 2+ . ⇒ 20N 1 = N 2 = 100N 3 ⇒ N 1 = N 2 = 5N 3
3 3
⇒ Meq of Cu 2+ = Meq of I2 = meq of hypo Also, n-factors are all integer and greater than or equal to one but
less than six, N 3 must be 1.
⇒ x = 11.3 × 0.05 = 0.565
2+ ⇒ N 1 = 5, N 2 = 3
⇒ Moles of Cu : moles of C2O42− = 0.565 : 1.13 = 1 : 2
∴ In acid medium MnO−4 → Mn 2+
46. Let us consider 10 mL of the stock solution contain x millimol In neutral medium MnO−4 → Mn 4+
oxalic acid H2C2O4 and y millimol of NaHC2O4. In alkaline medium MnO−4 → Mn6+
When titrated against NaOH, basicity of oxalic acid is 2 while ⇒ meq of K2Cr2O7 required = 100
that of NaHC2O4 is 1.
⇒ 100 = 1 × 6 × V (n-factor = 6)
⇒ 2x + y = 3 × 0.1 = 0.3 … (i)
⇒ V = 100/ 6 = 16.67 mL
When titrated against acidic KMnO4, n-factors of both oxalic 1
acid and NaHC2O4 would be 2. 49. Meq of MnO−4 required = 20 × ×5=2
50
⇒ 2x + 2 y = 4 × 0.1 = 0.4 … (ii) ⇒ Meq of Fe2+ present in solution = 2
Solving equations (i) and (ii) gives
⇒ millimol of Fe2+ present in solution = 2 (n-factor = 1)
y = 0.1, x = 0.1 Also,
0.1 Q 4 millimol of Fe2+ are formed from 1 millimol N2H4
⇒ In 1.0 L solution, mole of H2C2O4 = × 100 = 0.01
1000 1 1
∴ 2 millimol Fe2+ from × 2 = millimol N2H4
0.1 4 2
Mole of NaHC2O4 = × 100 = 0.01
1000 Therefore, molarity of hydrazine sulphate solution
⇒ Mass of H2C2O4 = 90 × 0.01 = 0.9 g 1 1 1
= × =
Mass of NaHC2O4 = 112 × 0.01= 1.12 g 2 10 20
35.5 1
47. Mass of chlorine in 1.0 g X = × 2.9 = 0.717 g ⇒ In 1 L solution mol N2H6SO4 is present.
143.5 20
1
Now, the empirical formula can be derived as : ⇒ Amount of N2H6SO4 = × 130 = 6.5 gL−1
20
C H Cl
50. Molecular weight of Na 2CO3 ⋅10H2O = 286
% wt : 24.24 4.04 71.72
1 1000
Mole : 2 4 2 ⇒ Molarity of carbonate solution = × = 0.035
286 100
Simple ratio : 1 2 1 ⇒ Normality of carbonate solution = 2 × 0.035 = 0.07 N
⇒ Empirical formula = CH2Cl.
22 Some Basic Concepts of Chemistry

8×5 BaO(s) BaO(s)


In acid solution : Normality of HNO3 = = 0.02
2000 4.08 g 3.64 g
5 × 4.8 From the decomposition information, it can be deduced that the
Normality of HCl = = 0.012
2000 original mixture contained 0.01 mole of MCO3 and the solid
Let normality of H2SO4 in final solution be N. residue, obtained after heating, contain 0.01 mole (10 millimol)
of MO.
⇒ (N + 0.02 + 0.012) × 30 = 0.07 × 42.9
Also, millimol of HCl taken initially = 100
⇒ N = 0.0681 millimol of NaOH used in back-titration = 16 × 2.5 = 40
⇒ Gram equivalent of SO2−
4 in 2 L solution = 2 × 0.0681 ⇒ millimol of HCl reacted with oxide residue = 60
= 0.1362 HCl reacts with oxides as :
96 MO + 2HCl → MCl 2 + H2O
⇒ Mass of SO2−
4 in solution = 0.1362 × = 6.5376 g 10 millimol 20 millimol
2
BaO + 2HCl → BaCl 2 + H2O
51. For the oxidation of A n+ as :
60 – 20 = 40 millimol
A n+ → AO−3 n-factor = 5 – n Therefore, the residue contain 20 millimol of BaO.
⇒ Gram equivalent of A n+ = 2.68 × 10−3 (5 − n) Also, molar mass of BaO = 138 + 16
Now equating the above gram equivalent with gram equivalent = 154
of KMnO4 : 154 × 20
2.68 × 10−3 (5 − n) = 1.61 × 10−3 × 5 ⇒ Mass of BaO = = 3.08 g
1000
⇒ n=+2 ⇒ Mass of MCO3 = 4.08 – 3.08 = 1.0 g
52. During heating MCO3 is converted into MO liberating CO2 Q 0.01 mole of MCO3 weight 1.0 g
while BaO is remaining unreacted : ∴ 1 mole of MCO3 = 100 g
Heat
MCO3 (s) → MO(s) + CO2 (g ) ↑ 0.44 g = 0.01 mol ⇒ 100 = (Atomic weight of metal) + (12 + 3 × 16)
⇒ Atomic weight of metal = 40, i.e. Ca
2
Atomic Structure
Topic 1 Preliminary Developments and Bohr’s Model
Objective Questions I (Only one correct option) 5. Which of the graphs shown below does not represent the
1. Which one of the following about an electron occupying relationship between incident light and the electron ejected from
the 1s-orbital in a hydrogen atom is incorrect? (The metal surface? (2019 Main, 10 Jan I)

Bohr radius is represented by a0 ) (2019 Main, 9 April II)


K.E. of K.E. of
(a) The electron can be found at a distance 2a0 from es s e ss
the nucleus.
(b) The magnitude of the potential energy is double
that of its kinetic energy on an average. (a) (b)
(c) The probability density of finding the electron is 0 0
maximum at the nucleus. Energy of light Intensity of light
(d) The total energy of the electron is maximum when
it is at a distance a0 from the nucleus. Number K.E. of
of ess ess
2. If p is the momentum of the fastest electron ejected
from a metal surface after the irradiation of light having
wavelength λ, then for 1.5 p momentum of the (c) (d)
photoelectron, the wavelength of the light should be 0 0
(Assume kinetic energy of ejected photoelectron to be Frequency of light Frequency of light
very high in comparison to work function)
(2019 Main, 8 April II)
6. A stream of electrons from a heated filament was passed between
4 3 two charged plates kept at a potential difference V esu. If e and m
(a) λ (b) λ are charge and mass of an electron, respectively, then the value of
9 4
h/ λ (where, λ is wavelength associated with electron wave) is
2 1
(c) λ (d) λ given by (2016 Main)
3 2
(a) 2 meV (b) meV (c) 2meV (d) meV
3. What is the work function of the metal, if the light of
wavelength 4000 Å generates photoelectron of velocity 7. Rutherford’s experiment, which established the nuclear model of
6 × 105 ms −1 from it? the atom, used a beam of (2002, 3M)

(Mass of electron = 9 × 10 −31


kg (a) β -particles, which impinged on a metal foil and got absorbed
Velocity of light = 3 × 10 ms −1
8 (b) γ-rays, which impinged on a metal foil and got scattered
(c) helium atoms, which impinged on a metal foil and got
Planck’s constant = 6.626 × 10−34 Js
scattered
Charge of electron = 1.6 × 10−19 JeV−1 ) (d) helium nuclei, which impinged on a metal foil and got
(2019 Main, 12 Jan I)
scattered
(a) 4.0 eV (b) 2.1 eV
8. Rutherford’s alpha particle scattering experiment eventually led
(c) 0.9 eV (d) 3.1 eV
to the conclusion that (1986, 1M)
4. The ground state energy of hydrogen atom is −13.6 eV. (a) mass and energy are related
The energy of second excited state of He+ ion in eV is (b) electrons occupy space around the nucleus
(2019 Main, 10 Jan II)
(c) neutrons are burried deep in the nucleus
(a) −54.4 (b) −3.4 (c) −6.04 (d) −27.2
(d) the point of impact with matter can be precisely determined
24 Atomic Structure

9. The radius of an atomic nucleus is of the order of (1985, 1M) List-I List-II
−10 −13 −15 −8
(a) 10 cm (b) 10 cm (c) 10 cm (d) 10 cm (I) Radius of the n th orbit (P) ∝ n −2

10. Bohr’s model can explain (1985, 1M) (II) Angular momentum of the electron (Q) ∝ n −1
in the nth orbit
(a) the spectrum of hydrogen atom only
(b) spectrum of an atom or ion containing one electron only (III) Kinetic energy of the electron in the (R) ∝ n0
nth orbit
(c) the spectrum of hydrogen molecule
(d) the solar spectrum
(IV) Potential energy of the electron in (S) ∝ n1
the nth orbit
11. The increasing order (lowest first) for the values of e/m (T) ∝ n2
(charge/mass) for electron ( e ), proton (p), neutron (n) and
(U) ∝ n 1/ 2
alpha particle (α) is (1984, 1M)
(a) e, p, n, α (b) n, p, e, α Which of the following options has the correct combination
(c) n , p , α , e (d) n , α, p , e considering List-I and List-II? (2019 Adv.)
12. Rutherford’s scattering experiment is related to the size of (a) (III), (P) (b) (III), (S)
the (1983, 1M) (c) (IV), (U) (d) (IV), (Q)
(a) nucleus (b) atom (c) electron (d) neutron 20. Consider the Bohr’s model of a one-electron atom where the
13. Rutherford’s experiment on scattering of α-particles showed electron moves around the nucleus. In the following List-I
for the first time that the atom has (1981, 1M) contains some quantities for the nth orbit of the atom and
(a) electrons (b) protons List-II contains options showing how they depend on n.
(c) nucleus (d) neutrons
List-I List-II

Objective Questions II (I) Radius of the nth orbit (P) ∝ n −2

(One or more than one correct option) (II) Angular momentum of the electron in (Q) ∝ n −1
the nth orbit
14. The energy of an electron in the first Bohr orbit of H-atom is
(III) Kinetic energy of the electron in the (R) ∝ n0
–13.6 eV. The possible energy value(s) of the excited state(s) nth orbit
for electrons in Bohr orbits of hydrogen is (are) (1988) (IV) Potential energy of the electron in the (S) ∝ n1
(a) − 3.4 eV (b) − 4.2 eV (c) − 6.8 eV (d) + 6.8 eV nth orbit
15. The atomic nucleus contains (1988, 1M) (T) ∝ n2
(a) protons (b) neutrons (c) electrons (d) photons (U) ∝ n 1/ 2
16. The sum of the number of neutrons and proton in the isotope
Which of the following options has the correct combination
of hydrogen is (1986, 1M)
considering List-I and List-II? (2019 Adv.)
(a) 6 (b) 5 (c) 4 (d) 3
(a) (II), (R) (b) (I), (P)
17. When alpha particles are sent through a thin metal foil, most (c) (I), (T) (d) (II), (Q)
of them go straight through the foil, because (1984, 1M)
21. According to Bohr’s theory,
(a) alpha particles are much heavier than electrons
En = Total energy K n = Kinetic energy
(b) alpha particles are positively charged
Vn = Potential energy rn = Radius of nth orbit
(c) most part of the atom is empty space
Match the following : (2006, 6M)
(d) alpha particles move with high velocity
18. Many elements have non-integral atomic masses, because Column I Column II
(a) they have isotopes (1984, 1M) A. V / K = ?
n n
p. 0
(b) their isotopes have non-integral masses B. q.
If radius of nth orbit ∝ E xn ,x= ? –1
(c) their isotopes have different masses
(d) the constituents, neutrons, protons and electrons, C. Angular momentum in lowest r. –2
combine to give fractional masses orbital
D. 1 s. 1
Match the Columns n
∝ Zy, y = ?
r
19. Consider the Bohr’s model of a one-electron atom where the
electron moves around the nucleus. In the following List-I
contains some quantities for the nth orbit of the atom and Fill in the Blanks
List-II contains options showing how they depend on n. 22. The light radiations with discrete quantities of energy are
called ................ . (1993, 1M)
Atomic Structure 25

23. The mass of a hydrogen is …… kg. (1982, 1M) Use Avogardo constant as 6.023 × 1023 mol −1 . (2020 Adv.)
24. Isotopes of an element differ in the number of …… in their
nuclei.
d

Potential energy
(1982, 1M) H H

(kJ mol–1)
25. Elements of the same mass number but of different atomic
numbers are known as …… . (1983, 1M) E0

Numerical Answer Type Questions


26. The figure below is the plot of potential energy versus d0
internuclear distance ( d ) of H2 molecule in the electronic Internuclear distance (d)
ground state. What is the value of the net potential energy
E0 (as indicated in the figure) in kJ mol −1 , for d = d 0 at Subjective Questions
which the electron-electron repulsion and the 27. With what velocity should an α-particle travel towards the
nucleus-nucleus repulsion energies are absent? As nucleus of a copper atom so as to arrive at a distance 10−13 m
reference, the potential energy of H atom is taken as zero from the nucleus of the copper atom ? (1997 (C), 3M)
when its electron and the nucleus are infinitely far apart.

Topic 2 Advanced Concept (Quantum Mechanical Theory)


Electronic Configuration and Quantum Number
Objective Questions I (Only one correct option) 3. Among the following, the energy of 2s-orbital is lowest in
(2019 Main, 12 April II)
1. The figure that is not a direct manifestation of the quantum
(a) K (b) H (c) Li (d) Na
nature of atoms is (2020 Main, 2 Sep I)
Increasing wavelength 4. The electrons are more likely to be found
a Ψ (x)
(a)
b x
–x
Absorption spectrum

c
Rb K
Na
(2019 Main, 12 April I)
Kinetic energy of
(b) photoelectrons (a) in the region a and c (b) in the region a and b
(c) only in the region a (d) only in the region c
Frequency of incident radiation 5. The ratio of the shortest wavelength of two spectral series of
hydrogen spectrum is found to be about 9. The spectral series
are (2019 Main, 10 April II)
Internal (a) Lyman and Paschen (b) Brackett and Pfund
(c) energy of (c) Paschen and Pfund (d) Balmer and Brackett
Ar
300 400 500 600 6. The graph between | ψ |2 and r (radial distance) is shown below.
Temperature (K) This represents (2019 Main, 10 April I)

T 2 >T 1
Intensity
2
(d) of black body |Ψ|
radiation T1

Wavelength

2. The number of orbitals associated with quantum numbers r


1
n = 5, ms = + is (a) 1s-orbital (b) 2 p-orbital
2 (2020 Main, 7 Jan I)
(c) 3s-orbital (d) 2s-orbital
(a) 25 (b) 50 (c) 15 (d) 11
26 Atomic Structure

7. For any given series of spectral lines of atomic hydrogen, 13. For emission line of atomic hydrogen from ni = 8 to n f = n,
let ∆ν = ν max − ν min be the difference in maximum and  1
minimum frequencies in cm −1 . The ratio the plot of wave number ( ν ) against  2  will be (The
n 
∆ νLyman / ∆ νBalmer is (2019 Main, 9 April I) Rydberg constant, RH is in wave number unit)
(a) 27 : 5 (b) 5 : 4 (c) 9 : 4 (d) 4 : 1 (2019 Main, 9 Jan I)
(a) non linear (b) linear with slope −RH
8. The quantum number of four electrons are given below:
1 (c) linear with slope RH (d) linear with intercept −RH
I. n = 4 , l = 2, ml = − 2, ms = −
2 14. The radius of the second Bohr orbit for hydrogen atom is
II. n = 3, l = 2, ml = 1, ms = +
1 (Planck’s constant ( h ) = 6.6262 × 10− 34 Js; mass of electron
2 = 91091
. × 10− 31 kg ; charge of electron
1 − 19
III. n = 4 , l = 1, ml = 0, ms = + ( e ) = 160210
. × 10 C; permitivity of vacuum
2
1 (∈0 ) = 8.854185 × 10− 12kg − 1 m − 3A 2 ) (2017 Main)
IV. n = 3, l = 1, ml = 1, ms = −
2 (a) 1.65 Å (b) 4.76 Å
The correct order of their increasing energies will be
(2019 Main, 8 April I)
(c) 0.529 Å (d) 2.12 Å
(a) IV < III < II < I (b) I < II < III < IV 15. P is the probability of finding the 1s electron of hydrogen
(c) IV < II < III < I (d) I < III < II < IV atom in a spherical shell of infinitesimal thickness, dr,
th at a distance r from the nucleus. The volume of this shell is
9. If the de-Broglie wavelength of the electron in n Bohr orbit
. πa0 (a0 is Bohr radius),
in a hydrogenic atom is equal to 15 4 πr2 dr. The qualitative sketch of the dependence of P on r is
(2016 Adv.)
then the value of n / Z is (2019 Main, 12 Jan II)
(a) 1.0 (b) 0.75 (c) 0.40 (d) 1.50 P P

10. The de-Broglie wavelength ( λ ) associated with a photoelectron


varies with the frequency ( ν ) of the incident radiation as, [ν 0 is (a) (b)
threshold frequency] (2019 Main, 11 Jan II)
1 1 0 0
(a) λ ∝ 1
(b) λ ∝ 3
r r

( ν − ν0 )4 ( ν − ν0 )2 P P
1 1
(c) λ ∝ (d) λ ∝
( ν − ν0 ) 1
(c) (d)
( ν − ν0 )2
11. Which of the following combination of statements is true
regarding the interpretation of the atomic orbitals? 0 r 0 r
(2019 Main, 9 Jan II)
16. Which of the following is the energy of a possible excited
I. An electron in an orbital of high angular momentum state of hydrogen? (2015 Main)
stays away from the nucleus than an electron in the (a) + 13.6 eV (b) – 6.8 eV
orbital of lower angular momentum. (c) –3.4 eV (d) + 6.8 eV
II. For a given value of the principal quantum number, the
size of the orbit is inversely proportional to the azimuthal 17. The correct set of four quantum numbers for the valence
quantum number. electrons of rubidium atom ( Z = 37 ) is (2013 Main)
1 1
III. According to wave mechanics, the ground state angular (a) 5, 0, 0, + (b) 5, 1, 0, +
h 2 2
momentum is equal to . 1 1
2π (c) 5, 1, ,1, + (d) 5, 0, 1, +
IV. The plot of ψ vs r for various azimuthal quantum 2 2
numbers, shows peak shifting towards higher r value. 18. Energy of an electron is given by
(a) I, III (b) II, III (c) I, II (d) I, IV  Z2 
E = − 2.178 × 10−18 J  2 
12. Heat treatment of muscular pain involves radiation of n  (2013 Main)
wavelength of about 900 nm. Which spectral line of H-atom
Wavelength of light required to excite an electron in an
is suitable for this purpose? [RH = 1 × 105 cm–1 , hydrogen atom from level n = 1to n = 2 will be
h = 6.6 × 10−34 Js, c = 3 × 108 ms −1 ] (2019 Main, 11 Jan I) (h = 6.62 × 10−34 Js and c = 3.0 × 108 ms −1 )
(a) Paschen, 5 →3 (b) Paschen, ∞ → 3 (a) 1.214 × 10−7 m (b) 2.816 × 10−7 m
(c) Lyman, ∞ → 1 (d) Balmer, ∞ → 2 (c) 6.500 × 10 −7
m (d) 8.500 × 10−7 m
Atomic Structure 27

19. The kinetic energy of an electron in the second Bohr orbit of a 29. For a d-electron, the orbital angular momentum is(1997, 1M)
hydrogen atom is [a0 is Bohr radius] (2012)  h  h  h  h
(a) 6   (b) 2   (c)   (d) 2  
h2 h2 h2 h2  2π   2π   2π   2π 
(a) (b) (c) (d)
4π 2 ma02 16π 2 ma02 32π 2 ma02 64π 2 ma02 30. The first use of quantum theory to explain the structure of
20. The number of radial nodes in 3s and 2p respectively are atom was made by (1997, 1M)

(a) 2 and 0 (b) 0 and 2 (2005, 1M)


(a) Heisenberg (b) Bohr
(c) 1 and 2 (d) 2 and 1 (c) Planck (d) Einstein
21. Which hydrogen like species will have same radius as that of 31. Which of the following has the maximum number of
Bohr orbit of hydrogen atom? (2004, 1M) unpaired electrons ? (1996, 1M)
(a) n = 2, Li 2+
(b) n = 2, Be 3+ (a) Mg2+ (b) Ti 3+ (c) V3+ (d) Fe2+
(c) n = 2, He+ (d) n = 3, Li 2+ 32. The orbital angular momentum of an electron in 2s-orbital is
7 (1996, 1M)
22. If the nitrogen atom had electronic configuration 1s , it 1 h h h
would have energy lower than that of the normal ground (a) + ⋅ (b) zero (c) (d) 2 ⋅
2 2π 2π 2π
state configuration 1s2 2s2 2 p 3 , because the electrons would
33. Which of the following relates to photons both as wave
be closer to the nucleus, yet 1s7 is not observed, because it
motion and as a stream of particles ?
violates (2002, 3M)
(1992, 1M)
(a) Heisenberg uncertainty principle
(b) Hund’s rule (a) Interference (b) E = mc2
(c) Pauli exclusion principle (c) Diffraction (d) E = hν
(d) Bohr postulate of stationary orbits 34. Which of the following does not characterise X-rays ?
1 1 (a) The radiation can ionise gases (1992, 1M)
23. The quantum numbers + and − for the electron spin
2 2 (b) It causes ZnS to fluoresce
represent (2001, 1M) (c) Deflected by electric and magnetic fields
(a) rotation of the electron in clockwise and anti-clockwise (d) Have wavelengths shorter than ultraviolet rays
direction respectively
35. The correct set of quantum numbers for the unpaired
(b) rotation of the electron in anti-clockwise and clockwise
direction respectively
electron of chlorine atom is (1989, 1M)

(c) magnetic moment of the electron pointing up and down n l m n l m


respectively (a) 2 1 0 (b) 2 1 1
(d) two quantum mechanical spin states which have no classical (c) 3 1 1 (d) 3 0 0
analogue 36. The correct ground state electronic configuration of
24. The wavelength associated with a golf ball weighing 200 g chromium atom is (1989, 1M)
and moving at a speed of 5 m/h is of the order (2001, 1M) (a) [ Ar ] 3d 5 4 s1 (b) [ Ar ] 3d 4 4 s2
(a) 10−10 m (b) 10−20 m (c) [ Ar ] 3d 6 4 s0 (d) [ Ar ] 4 d 5 4 s1
(c) 10−30 m (d) 10−40 m 37. The outermost electronic configuration of the most
25. The number of nodal planes in a px orbital is electronegative element is (1988, 90, 1M)
(2001, 1M)
(a) one (b) two (c) three (d) zero (a) ns2 np3 (b) ns2 np4
(c) ns2 np5 (d) ns2 np6
26. The electronic configuration of an element is
1s , 2s 2 p 6 , 3s2 3 p 6 3d 5 , 4 s1 . This represents its (2000, 1M)
2 2
38. The orbital diagram in which the Aufbau principle is
(a) excited state (b) ground state violated (1988, 1M)
(c) cationic form (d) anionic form (a) (b)
27. The electrons, identified by quantum numbers n and l,
(i) n = 4, l = 1, (ii) n = 4, l = 0, (iii) n = 3, l = 2, (iv) n = 3, l = 1 (c) (d)
can be placed in order of increasing energy, from the lowest to
highest, as (1999, 2M) 39. The wavelength of a spectral line for an electronic transition
(a) (iv) < (ii) < (iii) < (i) (b) (ii) < (iv) < (i) < (iii) is inversely related to (1988, 1M)
(c) (i) < (iii) < (ii) < (iv) (d) (iii) < (i) < (iv) < (ii) (a) the number of electrons undergoing the transition
(b) the nuclear charge of the atom
28. The energy of an electron in the first Bohr orbit of H-atom is
–13.6 eV. The possible energy value(s) of the excited state(s) (c) the difference in the energy of the energy levels involved in
the transition
for electrons in Bohr orbits of hydrogen is (are) (1998, 2M)
(d) the velocity of the electron undergoing the transition
(a) –3.4 eV (b) – 4.2 eV
(c) – 6.8 eV (d) + 6.8 eV
28 Atomic Structure

40. The ratio of the energy of a photon of 200 Å wavelength 48. The ground state electronic configuration of nitrogen atom
radiation to that of 4000 Å radiation is (1986, 1M) can be represented by (1999, 3M)
1 1
(a) (b) 4 (c) (d) 2. (a)
4 2
41. Which one of the following sets of quantum numbers (b)
represents an impossible arrangement? (1986, 1M)
n l m s (c)
1
(a) 3 2 –2
2 (d)
1
(b) 4 0 0
2 49. Which of the following statement (s) is (are) correct ?
1 (1998, 2M)
(c) 3 2 –3 (a) The electronic configuration of Cr is [Ar] 3d 5 4 s1 (atomic
2
1 number of Cr = 24)
(d) 5 3 0 −
2 (b) The magnetic quantum number may have a negative value
42. Electromagnetic radiation with maximum wavelength is (c) In silver atom, 23 electrons have a spin of one type and 24 of
(a) ultraviolet (b) radio wave (1985, 1M)
the opposite type. (atomic number of Ag = 47)
(c) X-ray (d) infrared (d) The oxidation state of nitrogen in HN3 is – 3
76
43. Which electronic level would allow the hydrogen atom to 50. An isotone of 32 Ge is (1984, 1M)
absorb a photon but not to emit a photon? (1984, 1M) (a) 77
(b) 77
32 Ge 33 As
(a) 3s (b) 2p (c) 2s (d) 1s 77 78
(c) 34 Se (d) 34 Se
44. Correct set of four quantum numbers for the valence
(outermost) electron of rubidium ( Z = 37 ) is (1984, 1M)
Assertion and Reason
1 1
(a) 5, 0, 0, + (b) 5, 1, 0 , + Read the following questions and answer as per the direction
2 2
1 1 given below :
(c) 5, 1, 1, + (d) 6, 0, 0, +
2 2 (a) Both Statement I and Statement II are correct; Statement
II is the correct explanation of Statement I
45. The principal quantum number of an atom is related to the
(a) size of the orbital (1983, 1M)
(b) Both Statement I and Statement II are correct; Statement
(b) spin angular momentum II is not the correct explanation of Statement I
(c) orientation of the orbital in space (c) Statement I is correct; Statement II is incorrect
(d) orbital angular momentum (d) Statement I is incorrect; Statement II is correct
46. Any p-orbital can accommodate upto (1983, 1M) 51. Statement I The first ionisation energy of Be is greater than
(a) four electrons that of B.
(b) six electrons
Statement II 2p-orbital is lower in energy than 2s. (2000)
(c) two electrons with parallel spins
(d) two electrons with opposite spins
Passage Based Questions
Objective Questions II The hydrogen-like species Li2+ is in a spherically symmetric
state S 1 with one radial node. Upon absorbing light the ion
(One or more than one correct option)
undergoes transition to a state S 2 . The state S 2 has one radial
47. The ground state energy of hydrogen atom is node and its energy is equal to the ground state energy of the
−13.6 eV. Consider an electronic state Ψ of He+ whose hydrogen atom.
energy, azimuthal quantum number and magnetic quantum
52. The state S 1 is
number are −3.4 eV, 2 and 0, respectively. (2010)
(a) 1s (b) 2s (c) 2p (d) 3s
Which of the following statement(s) is(are) true for the
state Ψ? (2019 Adv.) 53. Energy of the state S 1 in units of the hydrogen atom ground
(a) It is a 4 d state state energy is (2010)
(b) The nuclear charge experienced by the electron in this state (a) 0.75 (b) 1.50 (c) 2.25 (d) 4.50
is less than 2e, where e is the magnitude of the electronic 54. The orbital angular momentum quantum number of the state
charge
S 2 is (2010)
(c) It has 2 angular nodes (a) 0 (b) 1 (c) 2 (d) 3
(d) It has 3 radial nodes
Atomic Structure 29

Match the Columns


Answer Q. 55, Q. 55 and Q. 56 by appropriately matching the information given in the three columns of the following table.
The wave function, ψ n , l , ml is a mathematical function whose value depends upon spherical polar coordinates ( r, θ , φ ) of the electron
and characterised by the quantum number n , l and ml . Here r is distance from nucleus, θ is colatitude and φ is azimuth. In the
mathematical functions given in the Table, Z is atomic number and a0 is Bohr radius. (2017 Adv.)

Column 1 Column 2 Column 3


3  Zr 
(I) 1s-orbital (i) (P)
Z  2 −  a 0 
ψn, l , ml ∝   e
 a0 
ψn, I, ml (r)

0
r/a0

(II) 2s-orbital (ii) One radial node (Q) 1


Probability density at nucleus ∝
a03
5  Zr 
(III) 2 pz-orbital (iii) (R) Probability density is maximum at nucleus
 Z  2 − a 
ψn, l ml ∝   re  0  cosθ
 a0 

(IV) 3 dz2-orbital (iv) xy-plane is a nodal plane (S) Energy needed to excite electron from n = 2 state to n = 4 state
27
is times the energy needed to excite electron from n = 2
32
state to n = 6 state

55. For He+ ion, the only INCORRECT combination is


(a) (I) (i) (S) (b) (II) (ii) (Q) Fill in the Blanks
(c) (I) (iii) (R) (d) (I) (i) (R) 59. The outermost electronic configuration of Cr is
56. For the given orbital in Column 1, the Only CORRECT .......................... . (1994, 1M)
combination for any hydrogen-like species is 60. 8 g each of oxygen and hydrogen at 27°C will have the total
(a) (II) (ii) (P) (b) (I) (ii) (S) kinetic energy in the ratio of .......... . (1989, 1M)

(c) (IV) (iv) (R) (d) (III) (iii) (P) 61. The uncertainty principle and the concept of wave nature of
matter were proposed by ............ and .............respectively.
57. For hydrogen atom, the only CORRECT combination is (1988, 1M)
(a) (I) (i) (P) (b) (I) (iv) (R) 62. Wave functions of electrons in atoms and molecules are
(c) (II) (i) (Q) (d) (I) (i) (S) called .............. . (1993, 1M)
58. Match the entries in Column I with the correctly related 63. The 2 px, 2 p y and 2 p z orbitals of atom have identical shapes
quantum number(s) in Column II. (2008, 6M)
but differ in their ........... . (1993, 1M)
Column I Column II 64. When there are two electrons in the same orbital, they have
p. …… spins. (1983, 1M)
A. Orbital angular momentum Principal
of the electron in a quantum
True/False
hydrogen-like atomic orbital. number
q.
65. In a given electric field, β -particles are deflected more than
B. A hydrogen-like one-electron Azimuthal
wave function obeying quantum α-particles in spite of α-particles having larger charge.
(1993, 1M)
Pauli’s principle. number
66. The electron density in the XY-plane in 3d x 2 − y
orbital is
2
C. Shape, size and orientation r. Magnetic zero. (1986, 1M)
of hydrogen-like atomic quantum 67. The energy of the electron in the 3d-orbital is less than that in
orbitals. number the 4s-orbital in the hydrogen atom. (1983, 1M)
D. Probability density of s. Electron spin 68. Gamma rays are electromagnetic radiations of wavelengths
electron at the nucleus in quantum of 10−6 to 10−5 cm. (1983, 1M)
hydrogen-like atom. number 69. The outer electronic configuration of the ground state
chromium atom is 3d 4 4 s2 . (1982, 1M)
30 Atomic Structure

Integer Answer Type Questions work done in the above neutralisation process. Also, if the
magnitude of the average kinetic energy is half the
70. Not considering the electronic spin, the degeneracy of the magnitude of the average potential energy, find the average
second excited state ( n = 3 ) of H-atom is 9, while the potential energy. (1996, 2M)
degeneracy of the second excited state of H− is (2015 Adv.)
81. Calculate the wave number for the shortest wavelength
71. In an atom, the total number of electrons having quantum transition in the Balmer series of atomic hydrogen.(1996, 1M)
numbers (2014 Adv.)
82. Iodine molecule dissociates into atoms after absorbing light
1
n = 4, | ml | = 1 and ms = − is to 4500Å. If one quantum of radiation is absorbed by each
2 molecule, calculate the kinetic energy of iodine atoms.
72. The atomic masses of He and Ne are 4 and 20 amu,
(Bond energy of I2 = 240 kJ mol –1 ) (1995, 2M)
respectively. The value of the de-Broglie wavelength of He
gas at −73°C is ‘M’ times that of the de-Broglie wavelength 83. Find out the number of waves made by a Bohr’s electron in
of Ne at 727°C. M is (2013 Adv.) one complete revolution in its 3rd orbit. (1994, 3M)
73. The work function (φ) of some metals is listed below. The 84. What transition in the hydrogen spectrum would have the
number of metals which will show photoelectric effect when same wavelength as the Balmer transition n = 4 to n = 2 of
light of 300 nm wavelength falls on the metal is (2011) He+ spectrum? (1993, 3M)
Metal Li Na K Mg Cu Ag Fe Pt W 85. Estimate the difference in energy between 1st and 2nd
Φ (eV) 2.4 2.3 2.2 3.7 4.8 4.3 4.7 6.3 4.75 Bohr’s orbit for a hydrogen atom. At what minimum atomic
number, a transition from n = 2 to n = 1 energy level would
74. The maximum number of electrons that can have principal result in the emission of X-rays with l = 3.0 × 10–8 m ?
quantum number, n = 3 and spin quantum number,
Which hydrogen atom-like species does this atomic number
ms = − 1 / 2 , is (2011)
correspond to? (1993, 5M)

Subjective Questions 86. According to Bohr’s theory, the electronic energy of


hydrogen atom in the nth Bohr’s orbit is given by :
75. (a) Calculate velocity of electron in first Bohr orbit of hydrogen
atom (Given, r = a0 ). −21.7 × 10−19
En = J
(b) Find de-Broglie wavelength of the electron in first Bohr n2
orbit. Calculate the longest wavelength of electron from the third
(c) Find the orbital angular momentum of 2p-orbital in terms of Bohr’s orbit of the He+ ion. (1990, 3M)
h / 2π units. (2005, 2M)
87. What is the maximum number of electrons that may be
76. (a) The Schrodinger wave equation for hydrogen atom is present in all the atomic orbitals with principal quantum
3/ 2 number 3 and azimuthal quantum number 2 ?
1  1  r  − r / 2a 0 (1985, 2M)
ψ 2s =   2 −  e
4 (2π )1/ 2  a0   a0  88. Give reason why the ground state outermost electronic
configuration of silicon is (1985, 2M)
where, a0 is Bohr’s radius. Let the radial node in 2s be at r0.
3s 3p 3s 3p
Then, find r in terms of a0.
(b) A base ball having mass 100 g moves with velocity and not
100 m/s. Find out the value of wavelength of base ball.
(2004, 2M) 89. The electron energy in hydrogen atom is given by
21.7 × 10−12
77. The wavelength corresponding to maximum energy for En = − erg. Calculate the energy required to
hydrogen is 91.2 nm. Find the corresponding wavelength for n2
He+ ion. (2003, 2M) remove an electron completely from the n = 2 orbit. What is
the longest wavelength (in cm) of light that can be used to
78. Calculate the energy required to excite 1 L of hydrogen gas
cause this transition? (1984, 3M)
at 1 atm and 298 K to the first excited state of atomic
hydrogen. The energy for the dissociation of H—H bond is 90. Calculate the wavelength in Angstroms of the photon that is
436 kJ mol −1 . (2000) emitted when an electron in the Bohr’s orbit, n = 2 returns to
the orbit, n = 1in the hydrogen atom. The ionisation potential
79. An electron beam can undergo diffraction by crystals.
Through what potential should a beam of electrons be of the ground state hydrogen atom is 2.17 × 10−11 erg per
accelerated so that its wavelength becomes equal to 1.54 Å. atom. (1982, 4M)
(1997 (C), 2M) 91. The energy of the electron in the second and third Bohr’s
80. Consider the hydrogen atom to be proton embedded in a orbits of the hydrogen atom is − 5.42 × 10−12 erg and
cavity of radius a0 (Bohr’s radius) whose charge is − 2.41 × 10−12 erg respectively. Calculate the wavelength of
neutralised by the addition of an electron to the cavity in
vacuum, infinitely slowly. Estimate the average total energy the emitted light when the electron drops from the third to the
of an electron in its ground state in a hydrogen atom as the second orbit. (1981, 3M)
Answers
Topic 1 33. (a) 34. (c) 35. (c) 36. (a)
1. (d) 2. (a) 3. (b) 4. (c) 37. (c) 38. (b) 39. (c) 40. (d)
5. (d) 6. (c) 7. (d) 8. (b) 41. (c) 42. (b) 43. (d) 44. (a)
9. (b) 10. (b) 11. (d) 12. (a) 45. (a) 46. (d) 47. (a,c)
13. (c) 14. (a) 15. (a,b) 16. (d) 48. (a,d) 49. (a,b,c) 50. (b,d) 51. (c)
17. (a,c) 18. (a,c) 19. (a) 20. (c)
52. (b) 53. (c) 54. (b) 55. (c)
21. A → r; B → q; C → p, D → s
56. (a) 57. (d)
22. (photons) 23. (1 . 66 × 10 –27 kg) 24. (neutrons)
58. A → q; B → p, q, r, s C → p, q, r D → p, q, r
25. (isobars) 26. (–5242.41) 27. 6.3 × 10 6
5 1
59. Cr = [Ar] 3d , 4s 60. 1 : 16
Topic 2
61. Heisenberg, de-Broglie. 62. orbital
1. (c) 2. (a) 3. (a) 4. (a)
63. Orientation in space 64. opposite
5. (a) 6. (d) 7. (c) 8. (c)
65. True 66. False 67. True 68. False
9. (b) 10. (d) 11. (d) 12. (b)
69. False 70. (3) 71. (6) 72. (5)
13. (c) 14. (d) 15. (c) 16. (c) 73. (4.14 eV) 74. (9) 77. (22.8 nm) 78. (98.44 kJ)
17. (a) 18. (a) 19. (c) 20. (a)
79. (63.56 V) 81. (2.725× 10 6 M −1)
21. (b) 22. (c) 23. (d) 24. (c)
82. (2.16 × 10 20 J/atom) 86. (471 nm) 87. (10)
25. (a) 26. (b) 27. (a) 28. (a)
−5
29. (a) 30. (b) 31. (d) 32. (b) 89. (3.66 × 10 cm) 90. (1220 Å) 91. (660 nm)

Hints & Solutions


Topic 1 Preliminary Developments decreases. The maximum in the curve corresponds to the
and Bohr’s Model distance at which the probability of finding the electron in
maximum.
1. Statement (d) is incorrect. For 1s-orbital radial probability
density (R 2 ) against r is given as: 2. The expression of kinetic energy of photo electrons,
1
mv 2 = E − E0
KE =
2
When, KE > > E0, the equation becomes,
1
KE = mv 2 = E
R2 2
1s
1 2 hc p2 hc
⇒ mv = ⇒ =
2 λ 2m 2
λ
r 2 1 1
⇒ λ = hc × 2m × 2 ⇒ λ ∝ 2
For 1s-orbital, probability density decreases sharply as we move p p
away from the nucleus. hc
E= = energy of incident light.
The radial distribution curves obtained by plotting radial λ
probability functions vs r for 1s-orbital is E0 = threshold energy or work functions,
1 2 1 (mv )2 1 p2
mv = × = × 2
2 2 m2 2 m
Q p = momentum = mv
4πrR2

As per the given condition,


2
λ 2  p1 
= 
λ 1  p2 
2 2
λ2  p   2 4
r ⇒ =  =  =
λ  15
. × p  3 9
The graph initially increases and then decreases. It reaches a
maximum at a distance very close to the nucleus and then
32 Atomic Structure

4 Q λ 1 = λ  So, option (a) is correct.


⇒ λ2 = λ
9  p1 = p  (ii) KE of ejected electrons does not depend on the intensity of
3. Work function of metal (φ ) = hν 0 incident light.
where, ν 0 = threshold frequency
KE
1
Also, mev 2 = hν − hν 0
2
1
or mev 2 = hν − φ …(i)
2 0
Intensity of light
1 hc
mev 2 = −φ …(ii)
2 λ So, option (b) is correct.
Given : λ = 4000 Å = 4000 × 10−10m (iii) When, number of ejected electrons is plotted with frequency
v = 6 × 105 ms −1, of light, we get

me = 9 × 10−31 kg, c = 3 × 108 ms −1


Number
h = 6.626 × 10−34 Js of e–’s
Thus, on substituting all the given values in Eq. (i), we get
1
× 9 × 10−31 kg × (6 × 105 ms −1 )2
2 0
Frequency of light (n)
6.626 × 10−34 J s × 3 × 108 ms −1
= −φ So, option (c) is also correct.
4000 × 10−10 m
(iv) KE = hν − hν 0
∴ . × 10−21 kgm 2s −2 − 4.96 × 10−19 J
φ = 162
= 3.36 × 10−19 J [1 kgm2s −2 = 1J]
KE
= 21
. eV
0 n0 n
4. The ground state energy of H-atom is + 13.6 eV.
For second excited state, n = 2 + 1 = 3
Z2
∴ E3 (He+ ) = − 13.6 × 2 eV [Qfor He+ , Z = 2] Slope = + h, intercept = − hν 0. So, option (d) is not correct.
n
22
6. Plan As you can see in options, energy term is mentioned hence, we
= − 13.6 × 2 eV = − 6.04 eV have to find out relation between h / λ and energy. For this, we
3 shall use de-Broglie wavelength and kinetic energy term in eV.
h
5. For photoelectric effect, de-Broglie wavelength for an electron (λ ) =
(i) KE = E − E0 p
h
⇒ p= …(i)
λ
Kinetic energy of an electron = eV
|Ψ|2 For 1s-orbital
number of radial node = 1–0–1=0 p2
As we know that, KE =
2m
r p2
∴ eV = or p = 2 meV …(ii)
where, 2m
KE = Kinetic energy of ejected electrons. h
From equations (i) and (ii), we get = 2 meV
E = Energy of incident light = hν λ
E0 = Threshold energy = hν0
7. Rutherford used α-particle (He2+ nuclei) in his experiment.
ν = Frequency of incident light
ν0 = Threshold frequency 8. According to Rutherford’s model, there is a heavily positively
charged nucleus and negatively charged electrons occupies space
around it in order to maintain electro-neutrality.
KE 9. Radius of a nucleus is in the order of 10−13 cm, a fact.
0
E0 E
10. Bohr’s model is applicable to one-electron system only.
11. Neutron has no charge, hence e/ m is zero for neutron. Next,
α-particle (He2+ ) has very high mass compared to proton and
electron, therefore very small e/ m ratio. Proton and electron
Slope = ± 1, intercept = − E0
Atomic Structure 33

have same charge (magnitude) but former is heavier, hence has 1  Ze2 
smaller value of e/ m.
21. A. Vn = −  
4 πε 0  r 
e
: n<α < p< e 1  Ze2 
m Kn =  
8πε 0  r 
12. The negligibly small size of nucleus compared to the size of
atom was first established in Rutherford’s experiment. Vn
⇒ = − 2  (r )
13. The most important findings of Rutherford’s experiment is Kn
discovery of nucleus.
Ze2
14. Energy of electron in H-atom is determined by the expression: B. En = − ∝ r −1
8πε 0r
13.6
En = − eV where, n =1, 2, 3, .... ⇒ x = − 1  (q )
n2
13.6 h
In excited states, E2 = − = − 3.4 eV C. Angular momentum = l (l + 1) = 0 in 1s-orbital — (p)
4 2π
a0n2 1
13.6 D. rn = ⇒ ∝ Z  (s)
E3 = − = − 1.51 eV etc. Z rn
9
15. Nucleus is composed of neutrons and protons. 22. Photons have quantised energy.
16. The isotopes of hydrogen are 1 H2 and 1H3. 10−3
23. Mass of one H-atom = kg = 1.66 × 10−27 kg
6.023 × 1023
17. Alpha particles passes mostly undeflected when sent through
thin metal foil mainly, because 24. Isotopes have different number of neutrons.
(i) it is much heavier than electrons. 25. Isobars have same mass number but different atomic numbers.
(ii) most part of atom is empty space.
26. Given that, electrons and nucleus are at infinite distance, so
18. Many elements have several isotopes. For such elements, potential energy of H-atom is taken as zero.
atomic mass is average of the atomic masses of different
Therefore, according to Bohr’s model, potential energy of a
isotopes, which is usually non-integral.
H-atom with electron in its ground state = − 27.2 eV
19. (III) Kinetic energy of the electron in nth orbit, At d = d0, nucleus-nucleus and electron-electron repulsion is
Z2 absent.
K.E. = + 13.6 × Hence, potential energy will be calculated for 2H atoms
n2
= − 2 × 27.2 eV= − 54.4 eV
1
or K.E. ∝ or K.E. ∝ n−2 Potential enery of 1 mol H atoms in kJ
n2
. × 10−19
54.4 × 6.02 × 1023 × 16
From list-II, correct match is (III P). = = − 5242. 4192 kJ/mol
1000
(IV) Potential energy of the electron in the nth orbit,
Z2 27. When α-particle stop at 10−13m from nucleus, kinetic energy is
P.E. = − 2 × 13.6 × 2 zero, i.e. whole of its kinetic energy at the starting point is now
n
converted into potential energy.
1
P.E. ∝ 2 Potential energy of this α-particle can be determined as
n
P.E. ∝ n−2 Z1 × Z2e2
PE = −
From List II, correct match is (IV P). (4 πε 0 ) r
Hence, correct matching from list-I and list-II on the basis of
given option is (III, P). (Z1 = + 2, Z2 = + 29, ε 0 = 8.85 × 10−12 J−1 C2 m −1,
r = 10−13 m)
20. (I) Radius of the nth orbit, −19 2
2 × 29 × (1.6 × 10 )
n2 ⇒ | PE | = J
r = 0.529 × 4 × 3.14 × 8.85 × 10−12 × 10−13
Z
= 1.33 × 10−13 J
Here, r ∝ n2
From list-II, correct match is (I, T ) = kinetic energy of α-particle at t = 0
1
(II) Angular momentum of the electron, ⇒ KE = mv 2 = 1.33 × 10−13
nh 2
mvr = or mvr ∝ n
2π 2 × 1.33 × 10−13
⇒ v= = 6.3 × 106 ms−1
From list-II, correct match (II, S) 4 × 1.66 × 10−27
Hence, correct matching from list-I and list-II on the basis of
given option is (I, T ).
34 Atomic Structure

The graph between wavefunction (ψ ) and distance (r) from the


Topic 2 Advanced Concept (Quantum Mechanical nucleus helps in determining the shape of orbital.
Theory) Electronic Configuration and
Quantum Number 5. According to Rydberg’s equation,
1. Quantum nature of atoms are associated with following 1 RH  1 1 1  1 1
=  −  or ∝ − 
phenomena or processes : λ hc  n12 n22  λ  n12 n22 
(i) Absorption spectrum (ii) Emission spectrum For shortest wavelength, i.e. highest energy spectral line, n2 will
(iii) Black body radiation (iv) Photo-electricity
be (∞ ).
Internal energy (U ) of particles like atoms depends upon the
For the given spectral series, ratio of the shortest wavelength of
thermodynamic variables ( p, V , T ) of the system. Thus,
two spectral series can be calculated as follows :
U = f ( p,T ) or U = f1 (T ,V ) orU = f2 ( p,V )
1 1 1
So, quantum nature of atom is not associated with its internal − −0
λ L 32 ∞ 2 9 1
energy. (a) = = =
λP 1

1 1 − 0 9
Graph (c) is not a direct manifestation of the quantum nature of
atoms. 12 ∞ 2
1 1
2. According to quantum mechanical atom model, for each value − 2
λ Bk 5 2
∞ 1 16 16
of n (principal quantum number), there are ‘n’ different values of (b) = = × =
l (azimuthal quantum number), i.e. l = 0, 1, 2, …, (n − 1). And, λ Pf 1

1 25 1 25
for each value of l, there are 2l + 1 different values of ml 4 2 ∞2
(magnetic quantum number), i.e. ml = 0, ±1, ±2 … ± l. 1 1
− 2
∴Total number of possible combinations of n, l and ml , for a λP 5 2
∞ 1 9 9
(c) = = × =
given value of n is n2, and each such combination is associated λ Pf 1

1 25 1 25
with an orbital. Each orbital can occupy a maximum of two 32 ∞ 2
electrons, having a different value of spin quantum number (ms ), 1 1
1 1 − 2
which are + or − . λB 4 2
∞ 1 4 1
2 2 (d) = = × =
λ Bk 1

1 16 1 4
∴ Number of orbitals associated with n = 5 is n2 = 25. Each of 22 ∞ 2
1
those orbitals can be associated with ms = + as well as Note Lyman = L (n1 = 1), Balmer = B (n1 = 2)
2
1 Paschen = P (n1 = 3), Brackett = Bk (n1 = 4 )
ms = − .
2 Pfund = Pf (n1 = 5)
∴ Answer = 25 6. The graphs between | ψ |2 and r are radial density plots having
3. The energy of 2s-orbital is lowest in K(potassium). An orbital (n − l − 1) number of radial nodes. For 1s, 2s, 3s and 2 p-orbitals
gets larger as the principal quantum number n increases. these are respectively.
Correspondingly, the energy of the electron in such an orbital
becomes less negative, meaning that the electron is less strongly
bound and has less energy. The graph of principal quantum
number with atomic number is |Ψ|2 For 2s-orbital
number of radial node = 2–0–1=1
Principal quantum

2
r
number

1 2s |Ψ|2 For 3s-orbital


number of radial node = 3–0–1=2
1 25 50 75 100
Atomic number
r
4. The electrons are more likely to be found in the region aand c. At
b, wave function becomes zero and is called radial nodal surface
or simply node.
a |Ψ|2 For 2p-orbital
Y (x) number of radial node = 2–1–1=0

b +x →
← –x r

c Thus, the given graph between | ψ |2 and r represents 2s-orbital.


Atomic Structure 35

7. For any given series of spectral lines of atomic hydrogen. Thus, Eq. (i) becomes
Let ∆ν = ν max − ν min be the difference in maximum and n2
2πa0 = nλ ...(ii)
minimum frequencies in cm−1. Z
For Lyman series, n2
∴ 2πa0 = n (1.5 πa0) [Given, λ = 1 .5 πa0]
∆ν = ν max − ν min Z
General formula: n 15 . πa0 1.5
= = = 0.75
1 1 Z 2πa0 2
ν = 109677  2 − 2 
 ni nf  10. de-Broglie wavelength (λ) for electron is given by
For Lyman n1 = 1, n2 = 2, 3, K λ=
h
...(i)
1 1 1  2 m K.E
ν max = 109,677 −  = 109,677  − 0
 1 ∞ 1  Also, according to photoelectric effect
= 109,677 KE = hν − hν 0
1 1  On substituting the value of KE in Eq (i), we get
ν min = 109,677 − 2  h
 1 (2)  λ=
2m × (hν − hν 0 )
∆ν Lyman = ν max − ν min
109,677 × 3  109,677 1
= 109,677 −  ∴ λ∝
 4  = 4 (ν − ν 0 )1/ 2
For Balmer series, nh
11. (I) Angular momentum, mvr =
 1 1 109677 2π
ν max = 109,677 2 −  ⇒
 (2) ∞  4 ⇒ mvr ∝ n
∝ distance from the nucleus
 1 1  109677 × 5
ν min = 109,677 2 − 2  ⇒ (II) This statement is incorrect as size of an orbit
 (2) (3)  36
∝ Azimuthal quantum number (l )
∆ν = ν max − ν min
(Q n = constant)
109,677 109,677   1
∆ν Balmer = − × 5 = 109,677   (III) This statement is incorrect as at ground state,
4  36   9 n = 1, l = 0
∆ν Lyman 109,677 / 4 ⇒ Orbital angular momentum (wave mechanics)
=
∆ν Balmer 109,677 / 9 h
= l (l + 1) =0 [Q l = 0 ]
∆ν Lyman 9 2π
= (IV) The given plot is
∆ν Balmer 4
∆ν Lyman
∴The ratio of is 9 : 4. l=0 (n=1)
∆ν Balmer l=1 (n=2)

8. Smaller the value of (n + l ), smaller the energy. If two or more l=2 (n=3)
sub-orbits have same values of (n + l ), sub-orbits with lower ψ
values of n has lower energy. The (n + l ) values of the given l=3 (n=4)
options are as follows :
I. n = 4, l = 2 ; n + l = 6
II. n = 3, l = 2; n + l = 5
r
III. n = 4 , l = 1, n + l = 5
IV. n = 3, l = 1, n + l = 4 1   1 1 
12. ∆E = hc × = hc × RH  2 − 2  × Z 2 
Among II and III, n = 3 has lower value of energy. Thus, the λ   n1 n2  
correct order of their increasing energies will be
1 1 hc
IV < II < III < I ⇒ − 2= [for H, atom Z = 1]
n1 n2 RH × λ × Z 2 × hc
2
Circumference 2πr
9. Number of waves = ⇒ n= 1 1 1
Wavelength λ = = ×
RH × λ (1 × 10 7 m−1 ) (900 × 10−9 m)
∴ 2πr = nλ ...(i)
1 1 1
Also, we know that radius (r) of an atom is given by ⇒ − =
n12 n22 9
a n2
r= 0 1 1 1 1 ∴ n1 = 3,
Z So, in option (b) − = −0=
32 ∞ 2 9 9  n2 = ∞ 
36 Atomic Structure

13. According to Rydberg’s formula, 17. Given, atomic number of Rb, Z = 37


1 2 1 Thus, its electronic configuration is [ Kr ]5s1. Since, the last
wave number (ν) = RH Z  2 − 2 
 ni nf  electron or valence electron enter in 5s subshell.
Given, ni = n, nf = 8 [Q it is the case of emission] So, the quantum numbers are n = 5, l = 0, (for s-orbital) m = 0
(Q m = + l to −l), s = + 1 / 2 or − 1 / 2.
1 1
ν = RH × (1)2  2 − 2  Z2 
n 8  18. Given, in the question E = − 2.178 × 10−18 J  2 
n 
1 1 R R For hydrogen Z = 1,
ν = RH  2 −  = H2 − H
 n 64  n 64 1
So, E1 = − 2.178 × 10−18 J  2 
On comparing with equation of straight line, y = mx + c, we get 1 
−RH 1
Slope = RH , intercept = . −18
E2 = − 2.178 × 10 J  2 
64 2 
1 Now, E1 − E2
Thus, plot of wave number (ν) against 2 will be linear with
n 1 1  hc
i.e. ∆E = 2.178 × 10−18  2 − 2  =
slope (+ RH ). 1 2  λ
14. Bohr radius (rn ) = ∈0 n2h2 1 1 6.62 × 10−34 × 3.0 × 108
2.178 × 10−18  2 − 2  =
n2h2 1 2  λ
rn =
4π 2me2kZ ∴ λ ≈ 1.21 × 10−7 m
1 19. According to Bohr’s model,
k=
4 π ∈0 nh n2h2
mvr = ⇒ (mv )2 =
2 2
n h ∈0 a 2π 4 π 2r2
∴ rn = = n2 0
πme2Z Z 1 n2h2
⇒ KE = mv 2 = …(i)
where, m = mass of electron 2 8π 2r2m
e = charge of electron Also, Bohr’s radius for H-atom is, r = n2a0
h = Planck’s constant Substituting ‘r’ in Eq. (i) gives
k = Coulomb constant h2 h2
KE = when n = 2 , KE =
n2 × 0.53 8π 2n2a02m 32π 2 a02 m
rn = Å
Z 20. The number of radial nodes is given by expression (n − l − 1).
Radius of n th
Bohr orbit for H-atom For 3s, number of nodes = 3 − 0 − 1 = 2
= 0.53 n Å2
[Z = 1for H-atom] For 2p, number of nodes = 2 − 1 −1 = 0
a0n2
∴Radius of 2nd Bohr orbit for H-atom 21. Expression for Bohr’s orbit is, rn = = a0
Z
= 0.53 × (2)2 = 212
. Å when n = 2, Z = 4.
15. This graph shows the probability of finding the electron within 22. 1s7 violate Pauli exclusion principle, according to which an
shell at various distances from the nucleus (radial probability). orbital cannot have more than two electrons.
The curve shows the maximum, which means that the radial 1 1
probability is greatest for a given distance from the nucleus.
23. + and − just represents two quantum mechanical spin states
2 2
This distance is equal to Bohr’s radius = a0 which have no classical analogue.
24. Using the de-Broglie’s relationship :
P h 6.625 × 10−34
1s
λ= = = 2.3 × 10−30 m
mv 0.2 × 5
r 60 × 60

(a) It is for 2s-orbital. 25. Nodal plane is an imaginary plane on which probability of
(b) It is radial wave function for 1s. finding an electron is minimum. Every p-orbital has one nodal
plane :
(c) Correct
(d) Probability cannot be zero at a certain distance from nucleus.
13.6 px
16. ∴ En = − eV where, n = 1, 2, 3 ...
n2
−13.6 YZ-plane, a nodal plane
In excited states, E2 = = −3.4 eV
4
Atomic Structure 37

26. 1s2 2 s2 2 p6 3s2 3 p6 3d 5 4 s1 is ground state electronic configuration 38. Option (b) is wrong representation according to aufbau
of Cr. principle. A high energy atomic orbital (2p) cannot be filled
unless the low energy orbital (2s) is completely occupied.
27. (i) n = 4, l = 1 ⇒ 4 p-orbital
(ii) n = 4, l = 0 ⇒ 4s-orbital  1 1  hc
39. Transition energy (∆E ) = kZ 2  − 2 =
n = 3, l = 2 ⇒  n1 n2  λ
2
(iii) 3d-orbital
(iv) n = 3, l = 1 ⇒ 3d-orbital
1
According to Aufbau principle, energies of above mentioned i.e. ∆E ∝
λ
orbitals are in the order of
hc E λ
(iv) 3p < (ii) 4s < (iii) 3d < (i) 4 p 40. E = ⇒ 1 = 2 =2
λ E2 λ 1
28. The energy of an electron in a Bohr atom is expressed as
41. n l m s
where, k = Constant,
kZ 2 1
En = − 2 Z = Atomic number, 3 2 −3
n 2
n = Orbit number
This is the wrong set of quantum number because | m | cannot be
= − 13.6 eV for H (n = 1) greater than l.
− 13.6
when n = 2 , E2 = eV = − 3.40 eV 42. The wavelength order is
22
X-ray < ultraviolet < infrared < radio wave
(n can have only integral value 1, 2, 3,…… ∞)
43. When electron jumps to lower orbit photons are emitted while
h
29. The orbital angular momentum (L) = l (l + 1) photons are absorbed when electron jumps to higher orbit.
2π 1s-orbital is the lower most, electron in this orbital can absorb
h photons but cannot emit.
= 6 (l = 2 for d - orbital )

44. The valence shell configuration of Rubidium (Rb) is
30. Bohr first made use of quantum theory to explain the structure of 1 1
[ Kr ] 5s1 n = 5, l = 0, m = 0, s = + or −
atom and proposed that energy of electron in an atom is 2 2
quantised.
45. The principal quantum number ‘n’ represents orbit number
31. Mg2+ = 1s2 2s2 2 p6 no unpaired electron hence, determine the size of orbitals.
3+ 2 2 6 2 6 1
Ti = 1s 2 s 2 p 3s 3 p 3d one unpaired electron 46. According to Pauli exclusion principle, an atomic orbital can
3+ 2 2 6 2 6 2 accommodate at the most, two electrons, with opposite spins.
V = 1s 2 s 2 p 3s 3 p 3d two unpaired electrons
47. Given, ground state energy of hydrogen atom = − 13.6 eV
Fe2+ = 1s2 2 s2 2 p6 3s2 3 p6 3d6 four unpaired electrons
Energy of He + = − 3.4 eV, Z = 2
32. Expression for orbital angular momentum (L) is 13.6 × Z 2
h Energy of He + , E = − eV
L = l (l + 1) = 0 for 2s-electrons n2

− 13.6 × (2 )2 13.6 × 4
Q For s-orbital, l = 0. − 3.4 eV = ⇒ n= ⇒ n=4
n2 3.4
33. Diffraction is property of wave, E = mc2 determine energy of Given, azimuthal quantum number (l ) = 2 (d – subshell
particle and E = hν determine energy of photon. Interference Magnetic quantum number (m) = 0
phenomena is exhibited by both matter and waves.
∴ Angular nodes (l ) = 2
34. X-rays is electrically neutral, not deflected in electric or Radial node = n − l − 1= 4 − 2 − 1= 1
magnetic fields.
nl = 4 d state
35. Cl (17) = 1s2 2 s2 2 p6 3s2 3 p5 Hence, options (a), (c) are correct.
48. Both (a) and (d) are correct. The three electrons in the
2p-orbitals must have same spin, no matter up spin or down spin.
The last, unpaired electron has, n = 3, l = 1( p) and m can have 49. (a) Cr = [Ar] 3d 5 4 s1 , an exception to aufbau principle.
any of the three value (− 1, 0, + 1).
(b) For a given value of l, m can have any value from
36. Cr (24) = 1s2 2 s2 2 p6 3s2 3 p6 3d 5 4 s1 (−l to + l), so can have negative value.
1442443
Ar (c) Ag is in copper group with d 10s1 configuration,
i.e. 46 electrons are spin paired.
The above configuration is exception to Aufbau’s principle.
37. Fluorine, a halogen, is the most electronegative atom, has the 50. Isotones have same number of neutrons.
76 77
2 5
electronic configuration 2s 2 p (valence shell). 32 Ge , 33As and 34Se78 have same number (44) of neutrons,
hence they are isotones.
38 Atomic Structure

51. Assertion is correct Be(1s2 , 2s2 ) has stable electronic 66. 3dx2 − y2 orbital lies in XY-plane.
configuration, removing an electron require more energy than 67. Aufbau principle.
the same for B(2 p1 ). Reason is incorrect
(Aufbau principle). 68. This is the wavelength of infrared radiation.
52. S 1 is spherically symmetrical state, i.e. it correspond to a 69. Cr = 3d 5 4 s1.
s-orbital. Also, it has one radial node. 70. In an one electron (hydrogenic) system, all orbitals of a shell
Number of radial nodes = n − l − l remains degenerate, hence in second excited state, the
⇒ n − 0 − 1 = 1 ⇒ n = 2 i.e. S 1 = 2s-orbital. degeneracy of H-atom is nine
3s 3p 3d
53. Ground state energy of electron in H-atom (EH ) H(1s1) Second excited
Ground state state of H-atom
kZ 2
EH = = k (Z = 1, n = 1) All are degenerate
n2 degeneracy = 9
For S 1 state of Li 2+ , In case of many electrons system, different orbitals of a shell are
k (3)2 9 non-degenerate. Hence,
E = 2 = k = 2.25 k
2 4 1s 1s 2s 2p 1s 2s 2p

2+ H
54. In S 2 state, E (Li ) = K (given)
Second excited
Ground state First excited state
qk state only three p-orbitals
K = 2 ⇒ n=3 (2px, 2py, 2pz) are
n
degenerate.
Since, S 2 has one radial node.
3 − l −1 = 1 ⇒ l = 1 71. PLAN This problem is based on concept of quantum number. Follow
+ the following steps to solve this problem.
55. In the wave function (ψ ) expression for 1s-orbital of He , there Write all possible orbitals having combination of same principal,
should be no angular part. Hence (iii) can’t be true for ψ 1s of azimuthal, magnetic and spin quantum number.
He+ . Then count the all possible electrons having given set of
56. Correct : 2s orbital has one radial node. quantum numbers.
No of radial node = n − l − 1 = 2 − 0 − 1 = 1 For n = 4, the total number of possible orbitals are
Also, when radial part of wave function (ψ ) is plotted against 4s 4p 4d 4f
‘‘r’’, wave function changes its sign at node. 0 –1 0 +1 –2 –1 0 +1 +2 –3 –2 –1 0 +1 +2 +3

57. i is the correct expression of wave function for 1s-orbital of According to question | m l | = 1, i.e. there are two possible
hydrogenic system.
values of m l , i.e. +1 and –1 and one orbital can contain
58. A. Orbital angular momentum 1
maximum two electrons one having s = + and other having
h 2
(L) = l (l + 1)
2π s = − 1/ 2 .
i.e. L depends on azimuthal quantum number only. So, total number of orbitals having {| m l | = 1} = 6
B. To describe a one electron wave function, three quantum Total number of electrons having
numbers n, l and m are needed. Further to abide by Pauli 1
{| m l | = 1and ms = − } = 6
exclusion principle, spin quantum number(s) is also needed. 2
1 3
C. For shape, size and orientation, only n, l and m are needed. 72. PLAN KE = mv 2 = RT
2 2
D. Probability density (ψ 2 ) can be determined if n, l and m are ∴ m2v 2 = 2mKE ∴ mv = 2mKE
known.
h h h
λ (wavelength) = = ∝
59. Cr = [Ar] 3d 5 4 s1 mv 2mKE 2m(T )
60. 1 : 16 where, T = Temperature in Kelvin
h
61. Heisenberg proposed uncertainty principle and de-Broglie λ (He at −73° C = 200 K) =
proposed wave nature of electron. 2 × 4 × 200

62. orbital h
λ (Ne at 727°C = 1000 K) =
2 × 20 × 1000
63. 2 px ,2 py and 2pz have different orientation in space.
λ (He) 2 × 20 × 1000
64. Two electrons in same orbital must have opposite spin. ∴ =M = =5
λ (Ne) 2 × 4 × 200
65. Very large mass of alpha particles than beta particles is
responsible for less deflection in former case. Thus, M =5
Atomic Structure 39

73. Energy of photon pV 1×1


78. Moles of H2 = = = 0.0409
–34 8 RT 0.082 × 298
hc hc 6.625 × 10 × 3 × 10
= J= eV = = 4 . 14 eV ⇒ Bond energy = 0.0409 × 436 = 17.84 kJ
λ eλ 300 × 10–9 × 1.602 × 10–19
Number of H-atoms produced after dissociation
For photoelectric effect to occur, energy of incident photons = 2 × 0.0409 × 6.023 × 1023 = 4.93 × 1022
must be greater than work function of metal. Hence, only Li, Na,
K and Mg have work functions less than 4.14 V.  1
Transition energy/atom = 2.18 × 10−18 1 −  J
 4
74. When n = 3, l = 0, 1, 2 i.e. there are 3s, 3p and 3d-orbitals. If all
3
these orbitals are completely occupied as = × 2.18 × 10−18 J
4
⇒ Total transition energy
1
Total 18 electrons, 9 electrons with s = + and 9 with 3
2 = × 2.18 × 10−18 × 4.93 × 1022 J
4
1
s=– . = 80.60 × 103 J = 80.60 kJ
2
Therefore, total energy required
Alternatively In any nth orbit, there can be a maximum of 2n2
electrons. Hence, when n = 3, number of maximum electrons = = dissociation energy + transition energy
1 = (17.84 + 80.60) kJ = 98.44 kJ
18. Out of these 18 electrons, 9 can have spin –
1 2 79. If accelerated by potential difference of V volt, then
and remaining nine with spin + .
2 1
nh mv 2 = eV
75. (a) mvr = 2

p2
nh 6.625 × 10−34 ⇒ = eV , here p = momentum (mv )
⇒ v= = 2m
2πmr 2 × 3.14 × 9.1 × 10−31 × 0.529 × 10−10 h h
Using de-Broglie equation, λ = =
= 2.18 × 106 ms − 1 p 2meV
h 6.625 × 10−34 6.625 × 10−34
(b) λ = = = 0.33 × 10−9 m ⇒ 1.54 × 10−10 =
mv 9.1 × 10−31 × 2.18 × 106 (2 × 9.1 × 10−31 × 1.6 × 10−19 V )1/ 2
(c) Orbital angular momentum Solving for V gives : V = 63.56 V.
h  h
(L) = l (l + 1) = 2  80. The work done in the given neutralisation process is
2π  2π 
∞ e2
[Q For p-orbital, l = 1] W =−∫ F dr and F =
a0 4πε 0r2
2
76. (a) At radial node, ψ must vanishes, i.e. ∞
e2  1  e2
⇒ W = = − = Total energy (E )
4 πε 0  r  a 0
2 2 r0
 1   r0  − a 0 4 πε 0r
ψ 22s = 0 =   2 −  e
 4 2π   a0 
Now, if ‘V’ is magnitude of potential energy, then according to
r0 given information, kinetic energy (Ek ) is V / 2. Therefore,
⇒ 2− = 0 ⇒ r0 = 2a0 V
a0 E = −V + (PE is always negative)
2
h 6.625 × 10−34
(b) λ= = = 6.625 × 10−35 m V
mv 100 × 10−3 × 100 =−
2
= 6.625 × 10−25 Å (negligibly small) − e2
⇒ V = − 2E =
77. The general Rydberg’s equation is 2πε 0r
1  1 1 81. The Rydberg’s equation for H-atom is
ν= = R (Z )2  2 − 2
λ  1
n n2  1
1 1
1 = ν (wave number) = RH  2 − 2 
⇒ ∝ Z2 λ  n1 n2 
λ
For Balmer series, n1 = 2 and n2 = 3, 4 , 5, ..., ∞
λ (He+ ) Z (H)2 1
⇒ = = For shortest λ , n2 has to be maximum, i.e. infinity. Then
λ (H) Z (He+ )2 4
 1 1 R 1.09 × 107
λ (H) 91.2 ν = RH  −  = H = = 2.725 × 106 m −1
⇒ λ (He+ ) = = nm = 22.8 nm  4 ∞ 4 4
4 4
40 Atomic Structure

82. After breaking of the bond of I2 molecule, the remaining energy For longest wavelength transition from 3rd orbit, electron must
would be distributed uniformly to iodine atoms as their kinetic jump to 4th orbit and the transition energy can be determined as
energy, i.e. 1 1
∆E = + 4 × 21.7 × 10−19  −  J = 4.22 × 10−19 J
E (energy of photon) = Bond energy + 2 × kinetic energy  9 16

6.625 × 10−34 × 3 × 108 240 × 103 Also, Q ∆E =


hc
⇒ = + 2 × Ek λ
4500 × 10−10 6.023 × 1023
hc 6.625 × 10−34 × 3 × 108
⇒ Ek = 2.16 × 1020 J/atom ∴ λ= = m
∆E 4.22 × 10−19
83. The Bohr de-Broglie relationship is
= 471 × 10−9 m = 471 nm
2πr = nλ = circumference of Bohr’s orbit.
i.e. number of complete waves formed in one complete 87. Ten, the given value of n and l correspond to 3d-orbital which
revolution of electron in any Bohr orbit is equal to orbit number, has five fold degeneracy level.
hence three. 88. The 2nd configuration is against Hund’s rule of maximum
84. The expression for transition wavelength is given by Rydberg’s multiplicity which states that the singly occupied degenerate
equation : atomic orbitals must have electrons of like spins.
1  1 1 89. The required transition is n1 = 2 to n2 = ∞ and corresponding
= RH Z 2  2 − 2 
λ  n1 n2  transition energy is
Equating the transition wavelengths of H-atom and He ion, +  1 1
∆E = 21.7 × 10−12  2 − 2  erg
 1 1 4 4  n1 n2 
RH  2 − 2  = RH  2 − 2 
 n1 n2  2 4  21.7
= × 10−12 erg = 5.425 × 10−12 erg
4
Equating termwise on left to right of the above equation gives
n1 = 1 and n2 = 2 The longest wavelength that can cause above transition can be
determined as :
85. For H-atom, the energy of a stationary orbit is determined as
hc 6.625 × 10−34 × 3 × 108
k −18 λ= =
En = − 2 where, k = constant (2.18 × 10 J) ∆E 5.425 × 10−12 × 10−7
n
 1 3 = 3.66 × 10−7 m = 3.66 × 10−5 cm
⇒ ∆E (n = 2 to n = 1) = k 1 −  = k
 4 4
90. Ionisation potential of H-like species
= 1.635 × 10−18 J = E1 = 2.17 × 10−11 erg
For a H-like species, energy of stationary orbit is determined as
kZ 2  1
En = − 2 ⇒ ∆E = 2.17 × 10−11 1 − 2  × 10−7 J
 2 
n
where, Z = atomic number hc
= 1.6275 × 10−18 J ⇒ λ =
 1 1 ∆E
⇒ ∆E = kZ 2  2 − 2 
 n1 n2  6.625 × 10−34 × 3 × 108
= m
1.6275 × 10−18
1 ∆E k 2  1 1 2 3
⇒ = = Z  −  = RH Z × = 122 × 10−9 m = 1220 Å
λ hc hc 1 4  4
4 4 91. Transition energy = [ − 2.41 − (− 5.42)] × 10−12 erg
⇒ Z2 = = = 4.05
3RH λ 3 × 1.097 × 107 × 3 × 10−8 = 3.01 × 10−12 erg
= 3.01 × 10−19 J [Q 1 erg = 10−7 J ]
⇒ Z = 2 (He+ )
hc
Also, ∆E =
86. For H-like species, the energy of stationary orbit is expressed as λ
E ( X ) = Z 2 × E (H ) 6.625 × 10−34 × 3 × 108
+
⇒ For He (Z = 2) ⇒ λ= m
3.01 × 10−19
4 × 21.7 × 10−19
E=− J = 660 × 10−9 m = 660 nm
n2
3
Periodic Classification
and Periodic Properties

Topic 1 History and Periodic Classification


Objective Questions I (Only one correct option) (c) the first ionisation energies of elements along a period do not
vary in a regular manner with increase in atomic number
1. The IUPAC symbol for the element with atomic number 119
(d) for transition elements the d-subshells are filled with electrons
would be (2019 Main, 8 April II)
monotonically with increase in atomic number
(a) unh (b) uue
(c) uun (d) une Objective Question II
2. The element with Z = 120 (not yet discovered) will be an/a (One or more than one correct option)
(2019 Main, 12 Jan I)
(a) transition metal (b) inner-transition metal 4. The statements that is/are true for the long form of the
(c) alkaline earth metal (d) alkali metal periodic table is/are (1988, 1M)
(a) it reflects the sequence of filling the electrons in the order of
3. The statement that is not correct for the periodic sub-energy level s, p, d and f
classification of elements, is (1992, 1M)
(b) it helps to predict the stable valency states of the elements
(a) the properties of elements are the periodic functions of their
(c) it reflects tends in physical and chemical properties of the
atomic numbers
elements
(b) non-metallic elements are lesser in number than metallic
(d) it helps to predict the relative ionicity of the bond between any
elements
two elements

Topic 2 Periodic Properties


Objective Questions I (Only one correct option) 3. The group number, number of valence electrons and valency
2−
1. The correct order of the ionic radii of O , N , F , Mg ,3− − 2+ of an element with atomic number 15, respectively, are
(2019 Main, 12 April I)
+ 3+
Na and Al is (2020 Main, 5 Sep II) (a) 16, 5 and 2 (b) 15, 5 and 3
(a) N3 − < O2 − < F− < Na + < Mg 2+ < Al 3+ (c) 16, 6 and 3 (d) 15, 6 and 2
(b) Al 3+ < Na + < Mg 2+ < O2 − < F− < N3 − 4. The element having greatest difference between its first and
(c) Al 3+ < Mg 2+ < Na + < F− < O2 − < N3 − second ionisation energy, is (2019 Main, 9 April I)
3− − 2− 2+ + 3+ (a) Ca (b) Sc
(d) N <F <O < Mg < Na < Al
(c) Ba (d) K
2. Within each pair of elements F and Cl, S and Se, and Li and
5. The correct option with respect to the Pauling
Na, respectively, the elements that release more energy upon
electronegativity values of the elements is
an electron gain are (2020 Main, 7 Jan II) (2019 Main, 11 Jan II)
(a) F, Se and Na (b) F, S and Li (a) P > S (b) Si < Al
(c) Cl, S and Li (d) Cl, Se and Na (c) Te > Se (d) Ga < Ge
42 Periodic Classification and Periodic Properties

6. The correct order of the atomic radii of C, Cs, Al and S is 18. Which of the following has the maximum number of
(2019 Main, 11 Jan I) unpaired electrons ? (1996, 1M)
(a) C < S < Al < Cs (b) C < S < Cs < Al (a) Mg2+ (b) Ti 3+
(c) S < C < Cs < Al (d) S < C < Al < Cs (c) V 3+ (d) Fe2+
7. In general, the properties that decrease and increase down a 19. Amongst the following elements (whose electronic
group in the periodic table, respectively are configurations are given below), the one having the highest
(2019 Main, 9 Jan I)
ionisation energy is (1990, 1M)
(a) electronegativity and atomic radius
(a) [Ne] 3s2 3 p1 (b) [Ne] 3s2 3 p3
(b) electronegativity and electron gain enthalpy 2 2
(c) electron gain enthalpy and electronegativity (c) [Ne] 3s 3 p (d) [Ar] 3d 10 4s2 4 p3
(d) atomic radius and electronegativity 20. Which one of the following is the smallest in size?
3− 2− − (1989, 1M)
8. The ionic radii (in Å) of N ,O and F respectively are
(2015 Main) (a) N3− (b) O2−
(a) 1.36, 1.40 and 1.71 (b) 1.36, 1.71 and 1.40 (c) F − (d) Na +
(c) 1.71, 1.40 and 1.36 (d) 1.71, 1.36 and 1.40 21. The first ionisation potential of Na, Mg, Al and Si are in the
9. Which one of the following alkaline earth metal sulphates order (1988, 1M)
has its hydration enthalpy greater than its lattice enthalpy? (a) Na < Mg >Al < Si (b) Na > Mg > Al > Si
(a) CaSO4 (b) BeSO4 (2015 Main) (c) Na < Mg <Al >Si (d) Na > Mg > Al <Si
(c) BaSO4 (d) SrSO4
22. The electronegativity of the following elements increases in
10. Which among the following is the most reactive? the order (1987, 1M)
(a) Cl 2 (b) Br2 (2015 Main) (a) C, N, Si, P (b) N, Si, C, P
(c) I2 (d) ICl (c) Si, P, C, N (d) P, Si, N, C
11. Which one has the highest boiling point? 23. Atomic radii of fluorine and neon in Angstrom units are
(a) He (b) Ne (c) Kr (d) Xe respectively given by (1987, 1M)
12. The first ionisation potential of Na is 5.1 eV. The value of (a) 0.72, 1.60 (b) 1.60, 1.60
electron gain enthalpy of Na + will be (2013 Main) (c) 0.72, 0.72 (d) None of these
(a) − 2.55 eV (b) − 5.1 eV 24. The first ionisation potential in electron volts of nitrogen and
(c) − 10.2 eV (d) + 2.55 eV oxygen atoms are respectively given by (1987, 1M)
13. Which of the following represents the correct order of (a) 14.6, 13.6 (b) 13.6, 14.6
increasing first ionisation enthalpy for Ca, Ba, S, Se and Ar? (c) 13.6, 13.6 (d) 14.6, 14.6
(2013 Main)
(a) Ca < S < Ba < Se < Ar (b) S < Se < Ca < Ba < Ar
25. The hydration energy of Mg 2+ is larger than that of
(c) Ba < Ca < Se < S < Ar (d) Ca < Ba < S < Se < Ar (1984, 1M)
(a) Al 3+ (b) Na +
14. Identify the least stable ion amongst the following. (c) Be2+ (d) Mg3+
(a) Li + (b) Be− (2002, 3M)
(c) B− (d) C − 26. The element with the highest first ionisation potential is
(1982, 1M)
15. The set representing the correct order of first ionisation (a) boron (b) carbon
potential is (2001, 1M) (c) nitrogen (d) oxygen
(a) K > Na > Li (b) Be > Mg > Ca 27. The correct order of second ionisation potential of carbon,
(c) B > C > N (d) Ge > Si > C nitrogen, oxygen and fluorine is (1981, 1M)
(a) C > N > O > F (b) O > N > F > C
16. The correct order of radii is (2000, 1M) (c) O > F > N > C (d) F > O > N > C
(a) N < Be < B (b) F − < O2− < N3−
(c) Na < Li < K (d) Fe3+ < Fe 2+ < Fe4+ Objective Questions II
17. The incorrect statement among the following. (1997(C), 1M)
(One or more than one correct option)
(a) The first ionisation potential of Al is less than the first 28. The option(s) with only amphoteric oxides is(are)
ionisation potential of Mg (2017 Adv.)
(b) The second ionisation potential of Mg is greater than the (a) NO, B2O3 , PbO, SnO2 (b) Cr2O3 , CrO, SnO, PbO
second ionisation potential of Na (c) Cr2O3 , BeO, SnO, SnO2 (d) ZnO, Al 2O3 , PbO, PbO2
(c) The first ionisation potential of Na is less than the first 29. Ionic radii of (1999, 3M)
ionisation potential of Mg (a) Ti 4+ < Mn 7 + (b) 35
Cl − < 37
Cl −
(d) The third ionisation potential of Mg is greater than third
(c) K + > Cl − (d) P 3+ > P 5+
ionisation potential of Na
Periodic Classification and Periodic Properties 43

30. The first ionisation potential of nitrogen and oxygen atoms Atomic Ionisation enthalpy (kJ/mol)
are related as follows. (1989, 1M) number I1 I2 I3
(a) The ionisation potential of oxygen is less than the ionisation n 1681 3374 6050
potential of nitrogen n+1 2081 3952 6122
(b) The ionisation potential of nitrogen is greater than the
n+ 2 496 4562 6910
ionisation potential of oxygen
(c) The two ionisation potential values are comparable n+ 3 738 1451 7733
(d) The difference between the two ionisation potential is too large
31. Sodium sulphate is soluble in water whereas barium sulphate
Fill in the Blanks
is sparingly soluble because (1989, 1M) 38. Compounds that formally contain Pb 4+ are easily reduced to
(a) the hydration energy of sodium sulphate is more than its lattice Pb 2+ . The stability of the lower oxidation state is due to
energy …… . (1997, 1M)
(b) the lattice energy of barium sulphate is more than its hydration 39. Ca 2+ has a smaller ionic radius than K + because it has
energy
............ (1993, 1M
(c) the lattice energy has no role to play in solubility
(d) the hydration energy of sodium sulphate is less than its lattice 40. On Mulliken scale, the average of ionisation potential and
energy electron affinity is known as ................ (1985, 1M)

41. The energy released when an electron is added to a neutral


Assertion and Reason gaseous atom is called …… . (1982, 1M)
Read the following questions and answer as per the direction
given below : True/False
(a) Statement I is true; Statement II is true; Statement II is
42. The basic nature of the hydroxides of group 13 (III B)
the correct explanation of Statement I.
decreases progressively down the group. (1993, 1M)
(b) Statement I is true; Statement II is true; Statement II is
not the correct explanation of Statement I. 43. The decreasing order of electron affinity of F, Cl, Br is
(c) Statement I is true; Statement II is false. F > Cl > Br. (1993, 1M)
(d) Statement I is false; Statement II is true. 44. In group IA of alkali metals, the ionisation potential
32. Statement I Nitrogen and oxygen are the main components decreases down the group. Therefore, lithium is a poor
in the atmosphere but these do not react to form oxides of reducing agent. (1987, 1M)
nitrogen. 45. The softness of group IA metals increases down the group
Statement II The reaction between nitrogen and oxygen with increasing atomic number. (1986, 1M)
requires high temperature. (2015 Main)

33. Statement I Pb 4+ compounds are stronger oxidising agents Subjective Questions


4+
than Sn compounds. 46. Arrange the following ions in order of their increasing radii
Statement II The higher oxidation states for the group 14 Li + , Mg 2+ , K + , Al 3+. (1997, 1M)
elements are more stable for the heavier members of the
group due to ‘inert pair effect’. (2008, 3M) 47. Compare qualitatively the first and second ionisation
potentials of copper and zinc. Explain the observation.
34. Statement I Band gap in germanium is small. (1996, 2M
Statement II The energy spread of each germanium atomic
48. Arrange the following as stated :
energy level is infinitesimally small. (2007, 3M)
“Increasing order of ionic size’’ N3– ,Na + , F− , O2 − , Mg 2+
35. Statement I The first ionisation energy of Be is greater than
(1991, 1M)
that of B.
Statement II 2p-orbital is lower in energy than 2s. 49. Explain the following :
(2000, (S), 1M) “The first ionisation energy of carbon atom is greater than
36. Statement I F-atom has a less negative electron affinity than that of boron atom whereas, the reverse is true for the second
Cl-atom. ionisation energy.’’ (1989, 2M)
Statement II Additional electrons are repelled more 50. Arrange the following in the order of their increasing size:
effectively by 3 p-electrons in Cl-atom than by 2 p-electrons in Cl − , S2 − , Ca 2+ , Ar (1986, 1M)
F-atom. (1998, 2M)
51. Arrange the following in order of their
Numerical Answer Type Questions (i) decreasing ionic size Mg 2+ , O2 − , Na + ,F−
37. The 1st, 2nd and 3rd ionisation enthalpies, I1 , I 2 , and I 3 , of (ii) increasing first ionisation energy Mg, Al, Si, Na
four atoms with atomic numbers n , n + 1, n + 2, and n + 3, (iii) increasing bond length F2 , N2 , Cl 2 , O2 (1985, 3M)
where n < 10, are tabulated below. What is the value of n ?
(2020 Adv.)
44 Periodic Classification and Periodic Properties

Answers
Topic 1 21. (a) 22. (c) 23. (a) 24. (a)
1. (b) 2. (c) 3. (d) 4. (b,c,d) 25. (b) 26. (c) 27. (c) 28. (a,b)
29. (d) 30. (a,b,c) 31. (a,b) 32. (a)
Topic 2
33. (c) 34. (c) 35. (c) 36. (c)
1. (c) 2. (c) 3. (b) 4. (d)
37. (9) 38. (inert pair effect)
5. (d) 6. (a) 7. (a) 8. (c)
39. (higher effective nuclear charge)
9. (b) 10. (d) 11. (d) 12. (b)
40. (electronegativity) 41. (electron affinity)
13. (c) 14. (b) 15. (b) 16. (b)
42. F 43. F 44. F 45. T
17. (b) 18. (d) 19. (b) 20. (d)

Hints & Solutions


Topic 1 History and Periodic Classification Topic 2 Periodic Properties
1. Atomic number (119) = 1 1 9 1
un un en 1. Size of species ∝
Nuclear charge
So, symbol of the element = uue
Name of the element = ununennium Iso-electronic species are those atoms or ions which has the
It is expected to be s-block element an alkali metal and the first same number of electrons.
element in eighth period. It is the lightest element that has not Size of species decreases with increasing protons.
yet been synthesised. More is effective nuclear charge (Zeff) lesser will be ionic size.
2. The element with Z = 120 will be an alkaline earth metal. Correct order of ionic radii
Recently, oganesson (Og) with atomic number 118 is named by Al 3+ < Mg2+ < Na + < F− < O2− < N3−
IUPAC is a noble gas and placed just two place before 120. So,
the general electronic configuration is represented as [noble gas] 2. The first electron gain enthalpy is exothermic (or negative).
ns2 and element with Z = 120 exist as an alkaline earth metal.
Generally, electron gain enthalpy becomes less exothermic (or
3. (a) Correct statement According to Moseley’s law, the less negative) when comparing elements of a group from top to
properties of elements are the periodic function of their bottom.
atomic numbers. Therefore, electron gain enthalpy of S > Se and Li > Na.
(b) Correct statement The whole s-block, d-block, f -block But there are some exceptions to this.
and heavier p-block elements are metal. One of them is the case of a group 17 elements where electron
(c) Correct statement Trend is not regular, Be has higher first gain is most negative for Cl instead of F, due to extra small size
ionisation energy than B, nitrogen has higher first ionisation of fluorine.
energy than oxygen. ∴ Upon an electron gain, energy releases in the order :
(d) Inccorrect statement d-subshells are not filled Cl > F, S > Se and Li > Na
monotonically, regularity break at chromium and copper. 3. The group number, number of valence electrons and valency of
4. (a) Incorrect Electrons are not filled in sub-energy levels an element with atomic number 15 are 15, 5 and 3 respectively.
s, p, d and f in the same sequence. Modern periodic table is based on the atomic number. Number
(b) Correct Number of valence shell electrons usually of valence electrons present in an atom decides the group
determine the stable valency state of an element. number. Electronic configuration of element having atomic
number 15 = 1s2 2s2 2 p6 3s2 3 p3
(c) Correct Physical and chemical properties of elements are
periodic function of atomic number which is the basis of Valence electrons
modern, long form of periodic table. As five electrons are present in valence shell, its group number
(d) Correct Relative ionicity of the bond between any two is 15. Valency of element having atomic number 15 is +3
(8 − 5 = 3).
elements is function of electronegativity difference of the
bonded atoms which in turn has periodic trend in long form 4. The electronic configuration of given elements are as follows :
of periodic table. K(19) = 1s2 2s2 2 p6 3s2 3 p6 4 s1
Periodic Classification and Periodic Properties 45

Mg(12) = 1s2 2s2 2 p6 3s2 It is considered that, in case of isoelectronic species as the
Sr(38) = 1s2 2s2 2 p6 3s2 3 p6 4 s2 3d 10 4 p6 5s2 negative charge increases, ionic radii increases and therefore the
value of ionic radii are
Sc(21) = 1s2 2s2 2 p6 3s2 3 p6 4 s2 3d 1
N3− = 1.71 (highest among the three)
First ionisation enthalpy (IE) of K is lowest among the given O2− = 1.40 F− = 136. (lowest among the three)
options. Here, the energy required to remove an electron from
Time Saving Technique There is no need to mug up the radius
4 s1 is least as only one electron is present in the outermost shell.
values for different ions. This particular question can be solved
IE (I) is comparatively high for Mg and Sr and two electrons
through following time saving.
(fully-filled) are placed in s-orbital. Second ionisation enthalpy
of K is highest among the given options. Trick The charges on the ions indicate the size as
N3− > O2− > F− . Thus, you have to look for the option in which
Now, removal of an electron occur from p6 (fully-filled). So,
the above trend is followed. Option(c) is the only one in which
high energy is required to remove the electron. From the above this trend is followed. Hence, it is the correct answer.
discussion, it can be concluded that (IE 2 − IE 1 ) value is
maximum for K (potassium). 9. As we move down the group, size of metal increases. Be has
lower size while SO2− 4 has bigger size, that’s why BeSO 4 breaks
5. The electronegativity values of given elements on the Pauling
scale can be shown as follows: easily and lattice energy becomes smaller but due to lower size
of Be, water molecules are gathered around and hence hydration
Period No. Group 13 Group 14 Group 15 Group 16 energy increases.
3 Al (1.5) Si (1.8) P (2.1) S (2.5) On the other hand, rest of the metals, i.e Ca, Ba, Sr have bigger
4 Ga (1.6) Ge (1.8) Se (2.4) size and that’s why lattice energy is greater than hydration energy.
5 Te (2.01) Time Saving Technique In the question of finding hydration
energy only check the size of atom. Smaller sized atom has more
On moving from left to right across a period, i.e. from Ga to Se,
hydration energy. Thus, in this question Be is placed upper most
the effective nuclear charge increases and size decreases. in the group has lesser size and not comparable with the size of
As a result, the value of electronegativity increases due to sulphates. Hence, BeSO4 is the right response.
increase in the attraction between the outer electrons and the
10. Cl 2, Br2 and I2 are homonuclear diatomic molecule in which
nucleus. Whereas on moving down the group, (i.e. from Se to
Te), the atomic size increases. electronegativity of the combining atoms is same, so they are
more stable and less reactive, whereas, I and Cl have different
As a result, the force of attraction between the outer electron and electronegativities and bond between them are polarised and
the nucleus decreases. Hence, the electronegativity decreases. reactive. Therefore, interhalogen compounds are more reactive.
Time Saving Technique In this type of question of halogen,
6. Element Period Group No.
only go through the polarity of the molecules. As we know,
No.
diatomic molecule does not have polarity but molecules with
C 2nd 14
dissimilar sizes have polarity resulting in more reactivity.
Al 3rd 13 Along the period atomic radius
S 3rd 16 decreases, so, radii : Al > S. 11. As we move down the group of noble gases, molecular mass
increases by which dipole produced for a moment and hence
Cs 6th 1
London forces increases from He to Xe.
With the addition of a new shell, period number as well as atomic Therefore, more amount of energy is required to break these
radius increases. It is because of the successive addition of one forces, thus boiling point also increases from He and Xe.
extra shell of electrons. So, the order of the atomic radii of the
12. Na → Na + + e− First IE
given elements will be: C < S < Al < Cs
Na + e− → Na
+
7. The summary of variation of periodic properties is given in table
below: Electron gain enthalpy of Na + is reverse of (IE)
Because reaction is reverse so ∆H (eq) = − 5.1 eV
S.No. Periodic property Variation 13. Ionisation energy increases along a period from left to right and
Along a period Along a group decreases down a group. The position of given elements in the
1. Atomic radius Decreases Increases periodic table is as
2. Electron gain Increases Decreases Group No. 2 16 18
enthalpy Ca S Ar
3. Electronegativity Increases Decreases Ba Se
Thus, electronegativity decreases and atomic radius increases Thus, the order of increasing ∆H IE1 is Ba < Ca < Se < S < Ar
down a group in the periodic table.
14. Be− is the least stable ion, Be (1s2 2s2 ) has stable electronic
8. Number of electrons in N3−, = 7 + 3 = 10 configuration, addition of electron decreases stability.
Number of electrons in O2− = 8 + 2 = 10
15. In a group, ionisation energy decreases down the group
Number of electrons in F− = 9 + 1 = 10 Be > Mg > Ca
Since, all the three species have each 10 electrons, hence they
16. Among isoelectronic species, greater the negative charge,
are isoelectronic species.
greater the ionic size, hence F− < O2− < N3− .
46 Periodic Classification and Periodic Properties

17. (a) Correct statement In a period, element of 2nd group has In general, ionisation energy increases from left to right in a
higher first ionisation potential than element of group 13. period. However, exception occur between adjacent atoms in a
(b) Incorrect statement Mg+ require less energy for further period, greater amount energy is required for removal of
electron from completely half-filled or completely filled orbital
ionisation than Na + because of noble gas configuration of Na + .
than the same for adjacent atom with either less than completely
(c) Correct statement Ionisation energy increases from left half-filled or less than completely filled orbital. Therefore,
to right in a period. ionisation potential of O+ is greater than that of F+ . Also ionisation
18. Mg2+ = 1s2 2 s2 2 p6 = no unpaired electron potential of N+ is greater than C+ but less than both O+ and F+
(periodic trend). Hence, overall order is 2nd IP : O > F > N > C.
Ti 3+ = 1s2 2 s2 2 p6 3s2 3 p6 3d 1 = one unpaired electron
V3+ = 1s2 2 s2 2 p6 3s2 3 p6 3d 2 = two unpaired electrons 28. (c) is incorrect because NO is neutral oxide.
2+ 2 2 6 2 6 6 (d) is incorrect because CrO is basic oxide.
Fe = 1s 2 s 2 p 3s 3 p 3d = four unpaired electrons
29. (a) Ti 4+ > Mn 7+ is the correct order of size due to lower positive
19. [Ne] 3s2 3 p3 has highest ionisation energy, periodic trend.
charge on Ti 4+ .
20. Among isoelectronic species, the relation in size is (b) Cl − = 37 Cl − : Isotopes with same charge have same size
37
cation < neutral < anion
because isotopes differ in compositions of nuclei which do
Hence, Na + has smallest size. not affect the atomic/ionic radius.
21. Ionisation energy increases from left to right in a period. (c) K+ < Cl − is the correct order. Among isoelectronic species,
However, exception occur between group 2 and group 13 anion has greater size than cation.
elements on account of stability of electronic configuration of
valence shell. (d) P3+ > P5+ is the correct order. For the same elements, lower
IE the positive charge, larger the ions.
Group 2 = > Group 13 = 30. (a) and (b) are infact the same statements and both are correct. N
ns2 ns2 np1 has slightly greater ionisation energy than oxygen which is
⇒ The desired order is Na < Mg > Al < Si against periodic trend. This exception is due to completely
half-filled (2 p3 ) orbital in nitrogen that makes ionisation
22. Electronegativity increases from left to right in a period and slightly difficult than oxygen.
decreases from top to bottom in a group. Variation is more rapid (c) Also correct : Although N has greater first ionisation
in group than in a period, hence the desired order is
potential than oxygen, two values of ionisation potentials are
Electronegativity : Si < P < C < N
comparable since they are adjacent in a period, i.e. electrons
23. Atomic radius of noble gases are greater than halogens of same are removed from same orbit during ionisation.
period, hence (a) is the correct answer.
(d) Incorrect – opposite to (c). of the bonded atoms which in
24. First ionisation energy of oxygen is less than that of nitrogen on turn has periodic trend in long form of periodic table.
the ground of stability of valence shell configuration, hence (a)
31. (a) Correct For greater solubility, hydration energy must be
is the correct answer.
greater than lattice energy.
25. Hydration energy depends on charge of ion and ionic radius. (b) Correct Greater lattice energy discourage dissolution of a salt.
Higher the charge, greater the hydration energy. On the other (c) Incorrect When a salt dissolve, energy is required to break
hand, smaller the size, greater the hydration energy. Charge is the lattice, which comes from hydration process.
considered first for comparison. Hence, Mg2+ has higher
(d) Incorrect Explained in (A).
hydration energy than Na + .
32. Statement I and II are true and Statement II is the correct
26. Nitrogen has highest ionisation potential due to exceptional explanation of statement I.
stability of its valence shell configuration mentioned in
question 21. 33. Statement I is true. Stronger oxidising agent is one which itself
27. For second ionisation potential, electron will have to be can easily be reduced. Pb4+ is unstable, due to inert pair effect,
removed from valence shell of the following ions: can easily be reduced to stable Pb2+ , hence a stronger oxidising
agent than Sn 4+ .
C+ (5e–) = 1s2 2s2 Statement II is false. Due to inert pair effect, the higher
2p oxidation states of group 14 elements becomes less stable for
+ –
N (6e ) = heavier member.
1s2 2s2
2p 34. Both statements I and II are true and Statement II is the correct
explanation of statement I.
O+ (7e–) = 1s2 2s2
2p 35. Statement I is true Be has higher first ionisation energy than B
which is against periodic trend.
F+ (8e–) = 1s2 2s2 Statement II is false 2s-orbital is lower in energy than 2p,
2p Aufbau’s principle.
Periodic Classification and Periodic Properties 47

36. Statement I is true; Statement II is false. 47. Zn = 3d 10 4 s2 , Cu = 3d 10 4 s1


F atom has slightly lower affinity for the electron than chlorine. The first ionisation energy is greater for Zn but reverse is true for
It is due to the reason that additional electrons are repelled more 2nd ionisation energy.
effectively by 2p-electrons in F than by 3p-electrons in Cl-atom.
48. Ionic size Mg2+ < Na + < F− < O2− < N3−
37. By observing the values of different ionisation energies, I 1 , I 2
and I 3 for atomic number (n + 2 ), it is observed that there is very 49. The first ionisation energy of carbon is greater than the same of
large difference between the second ionisation energy and first boron as predicted from periodic trend. However, for 2nd
ionisation energy (I 2 >> I 1 ).
B+ = 1s2 ; more stable than C+ =1s2 2s2
This indicates that number of valence shell electrons is 1 and
2s2 2p1
atomic number (n + 2 ) should be an alkali metal.
Also for atomic number (n + 3 ), I 3 >> I 2. ionisation trend is reversed due to stability of completely filled
This indicates that it will be an alkaline earth metal which 2s-orbital of B+ :
suggests that atomic number (n + 1) should be a noble gas and
atomic number (n) should belong to halogen family. Since,
50. Size Ca 2+ < Ar < Cl − < S2− .
n < 10; hence, n = 9 (F atom) 51. (i) Mg2+ , O2− , Na + and F − are all isoelectronic, has
38. Inert pair effect-favours lower oxidation state. 10 electrons each. Among isoelectronic species, the order of
size is cation < neutral < anion.
39. Higher effective nuclear charge due to greater p/e ratio. Also, between cations, higher the charge, smaller the size
IP + EA and between anions, greater the negative charge, larger the
40. Electronegativity = (Mulliken formula)
2 size. Therefore, the decreasing order of ionic radii :
O2− > F− > Na + > Mg2+
41. Electron affinity–definition.
42. Basic nature of hydroxides increases down a group. (ii) First ionisation energy increases from left to right in a period.
However, exception occur between group 2 and 13 and group 15
43. Cl has maximum electron affinity, hence the correct order is and 16 where trend is reversed on the grounds of stability of
Cl > F > Br completely filled and completely half-filled orbitals. Therefore,
44. Ionisation potential decreases down the group but this is not the Ionisation energy (1st) : Na < Al < Mg < Si
only criteria of reducing power. (iii) If the atoms are from same period, bond length is inversely
45. In a group, size increases from top to bottom. proportional to bond order. In a group, bond length is related
directly to atomic radius.
46. Li + < Al 3+ < Mg2+ < K+ . Size decreases from left to right in a Therefore,
period and it increases from top to bottom in a group. Variation bond length N2 < O2 < F2 < Cl 2
is more pronounced in group than in period.
4
Chemical Bonding

Topic 1 Preliminary Concepts of Electrovalent


and Covalent Bonding
Objective Questions I (Only one correct option) 7. The number and type of bonds between two carbon atoms in
1. The isoelectronic set of ions is (2019 Main, 10 April I) CaC2 are (1996, 1M)
− + +
(a) F , Li , Na and Mg 2+ (a) one sigma (σ ) and one pi ( π ) bonds
(b) one sigma (σ ) and two pi ( π ) bonds
(b) N3 − , Li + , Mg 2+ and O2 − (c) one sigma (σ ) and one half pi ( π ) bonds
(c) Li + , Na + , O2 − and F− (d) one sigma (σ ) bond
(d) N3 − , O2 − , F− and Na +
8. The molecule which has zero dipole moment is (1989, 1M)
(a) CH2 Cl 2 (b) BF3 (c) NF3 (d) ClO2
2. Which of the following compounds contain(s) no covalent
bond(s)? 9. Element X is strongly electropositive and element Y is
KCl, PH3 , O2 , B2 H6 , H2 SO4 (2018 Main) strongly electronegative. Both are univalent. The compound
formed would be (1980, 1M)
(a) KCl, B2 H6 , PH3 (b) KCl, H2 SO4
(a) X +Y − (b) X −Y + (c) X −− Y (d) X → Y
(c) KCl (d) KCl, B2 H6
3. The intermolecular interaction that is dependent on the 10. Which of the following compound is covalent? (1980, 1M)

inverse cube of distance between the molecules is (2015 Main) (a) H2 (b) CaO
(a) ion-ion interaction (b) ion-dipole interaction (c) KCl (d) Na 2 S
(c) London force (d) hydrogen bond 11. The total number of electrons that take part in forming the
4. The nodal plane in the π-bond of ethene is located in bonds in N2 is (1980, 1M)
(a) 2 (b) 4 (c) 6 (d) 10
(a) the molecular plane (2002, 3M)
(b) a plane parallel to the molecular plane 12. The compound which contains both ionic and covalent bonds
(c) a plane perpendicular to the molecular plane which is (1979, 1M)
bisects the carbon-carbon σ-bond at right angle (a) CH4 (b) H2 (c) KCN (d) KCl
(d) a plane perpendicular to the molecular plane which
contains the carbon-carbon σ-bond Objective Questions II
5. Amongst H2 O, H2 S, H2 Se and H2 Te, the one with the highest (One or more than one correct option)
boiling point is (2000, 1M)
13. Each of the following options contains a set of four
(a) H2 O because of hydrogen bonding molecules. Identify the option(s) where all four molecules
(b) H2 Te because of higher molecular weight posses permanent dipole moment at room temperature.
(c) H2 S because of hydrogen bonding (a) SO2 , C6 H5 Cl, H2 Se, BrF5 (2019 Adv.)
(d) H2 Se because of lower molecular weight (b) BeCl 2 , CO2 , BCl 3 , CHCl 3
6. Arrange the following compounds in order of increasing (c) NO2 , NH3 , POCl 3 , CH3 Cl
dipole moment, toluene (I), m-dichlorobenzene (II), (d) BF3 , O3 , SF6 , XeF6
o-dichlorobenzene (III), p-dichlorobenzene (IV) (1996, 1M) 14. Dipole moment is shown by (1986, 1M)
(a) I < IV < II < III (b) IV < I < II < III (a) 1, 4-dichlorobenzene (b) cis-1, 2-dichloroethene
(c) IV < I < III < II (d) IV < II < I < III (c) trans-1, 2-dichloroethene (d) trans-1, 2-dichloro-2- pentene
Chemical Bonding 49

Numerical Answer Type Questions 17. Statement I LiCl is predominantly a covalent compound.
15. Consider the following compounds in the liquid form : Statement II Electronegativity difference between Li and Cl
O 2 ,HF,H 2O,NH 3 ,H 2O 2 ,CCl 4 ,CHCl 3 , C6H 6 ,C6H 5Cl is too small. (1998, 2M)

When a charged comb is brought near their flowing stream, Fill in the Blank
how many of them show deflection as per the following
figure? (2020 Adv.) 18. There are …… π -bonds in a nitrogen molecule. (1982, 1M)

True/False
19. All molecules with polar bonds have dipole moment.
(1985, 1/2 M)
20. Linear overlapping of two atomic p-orbitals leads to a sigma
bond. (1983, 1M)

Subjective Questions
21. Arrange the following ions in order of their increasing radii:
Li + , Mg 2+ , K + , Al 3+ . (1997, 1M)
+ +
22. Between Na and Ag , which is stronger Lewis acid and
why? (1997, 3M)

23. In the reaction, I + I2 → I3− , which is the Lewis acid?
(1997, 1M)
16. Among the species given below, the total number of 24. Explain the difference in the nature of bonding in LiF and LiI.
diamagnetic species is____ (1996, 2M)
H atom, NO2 monomer, O−2 (superoxide), dimeric sulphur in 25. The dipole moment of KCl is 3.336 × 10−29 C-m which
vapour phase, Mn 3 O4 ,( NH4 )2 [ FeCl 4 ], ( NH4 )2 [ NiCl 4 ], indicates that it is a highly polar molecule. The interatomic
K 2 MnO4 , K 2 CrO4 (2018 Adv.) distance between K + and Cl − in this molecule is
2.6 × 10−10 m. Calculate the dipole moment of KCl molecule
Assertion and Reason if there were opposite charges of one fundamental unit
Read the following questions and answer as per the direction located at each nucleus. Calculate the percentage ionic
given below: character of KCl. (1993, 2M)

(a) Statement I is true; Statement II is true; Statement II is the 26. Give reasons in two or three sentences only for the following :
correct explanation of Statement I “Hydrogen peroxide acts as an oxidising as well as a reducing
(b) Statement I is true; Statement II is true; Statement II is not agent.’’ (1992, 1M)
the correct explanation of Statement I 27. State four major physical properties that can be used to
(c) Statement I is correct; Statement II is incorrect distinguish between covalent and ionic compounds. Mention
(d) Statement I is incorrect; Statement II is correct the distinguishing features in each case. (1978, 2M)

Topic 2 VBT, Hybridisation and VSEPR Theory


Objective Questions I (Only one correct option) 2. The correct statement about ICl 5 and ICl −4 is
1. The correct statements among I to III are : (2019 Main, 8 April II)

I. Valence bond theory cannot explain the color exhibited (a) ICl 5 is square pyramidal and ICl −4 is tetrahedral
by transition metal complexes. (b) ICl 5 is square pyramidal and ICl −4 is square planar
II. Valence bond theory can predict quantitatively the (c) Both are isostructural
magnetic properties of transition metal complexes. (d) ICl 5 is trigonal bipyramidal and ICl −4 is tetrahedral
III. Valence bond theory cannot distinguish ligands as weak
and strong field ones. (2019 Main, 9 April II) 3. The ion that has sp 3 d 2 -hybridisation for the central atom, is
(a) II and III only (b) I, II and III (2019 Main, 8 April II)

(c) I and II only (d) I and III only (a) [ICl 2 ]− (b) [BrF2 ]− (c) [ICl 4 ]− (d) [IF6 ]−
50 Chemical Bonding

4. The size of the iso-electronic species Cl − , Ar and Ca 2+ is 17. Which of the following molecular species has unpaired
affected by (2019 Main, 8 April I) electron (s)? (2002, 3M)
(a) azimuthal quantum number of valence shell (a) N2 (b) F2 (c) O–2 (d) O2–
2
(b) electron-electron interaction in the outer orbitals
18. Specify the coordination geometry around and hybridisation
(c) principal quantum number of valence shell of N and B atoms in a 1 : 1 complex of BF3 and NH3 .
(d) nuclear charge (a) N : tetrahedral, sp 3 ; B: tetrahedral, sp 3 (2002, 3M)
5. In which of the following processes, the bond order has (b) N : pyramidal, sp 3 ; B: pyramidal, sp 3
increased and paramagnetic character has changed to (c) N: pyramidal, sp 3 ; B: planar, sp 2
diamagnetic? (2019 Main, 9 Jan II) (d) N: pyramidal, sp 3 ; B: tetrahedral, sp 3
(a) O2 → O2+ (b) N2 → N+2 19. The correct order of hybridisation of the central atom in the
(c) O2 → O22 − (d) NO → NO+ following species NH3 , [PtCl 4 ]2 − , PCl 5 and BCl 3 is
6. Total number of lone pair of electron in I−3 ion is (2018 Main)
(2001, 1M)
(a) dsp 2 , dsp 3 , sp 2 and sp 3 (b) sp 3 , dsp 2 , sp 3 d and sp 2
(a) 3 (b) 6
(c) dsp 2 , sp 2 , sp 3 and dsp 3 (d) dsp 2 , sp 3 , sp 2 and dsp 3
(c) 9 (d) 12
20. The common features among the species CN – , CO
7. The group having isoelectronic species is (2017 Main)
and NO+ are (2001, 1M)
(a) O2 − , F− , Na + , Mg 2 + (b) O− , F− , Na, Mg + (a) bond order three and isoelectronic
(c) O2 − , F− , Na, Mg 2 + (d) O− , F− , Na + , Mg 2 + (b) bond order three and weak field ligands
(c) bond order two and acceptors
8. The correct statement for the molecule, CsI3 is (2014 Main)
(d) isoelectronic and weak field ligands
(a) it is a covalent molecule
(b) it contains Cs + and I−3 ions 21. The hybridisation of atomic orbitals of nitrogen in NO+2 , NO−3
(c) it contains Cs 3+ and I− ions and NH+4 are (2000, 1M)
(d) it contains Cs + , I− and lattice I2 molecule (a) sp, sp 3 and sp 2 respectively
9. The species having pyramidal shape is (2010) (b) sp, sp 2 and sp 3 respectively
(a) SO3 (b) BrF3 (c) SiO2−
3 (d) OSF2 (c) sp 2 , sp and sp 3 respectively
(d) sp 2 , sp 3 and sp respectively
10. Assuming that Hund’s rule is violated, the bond order and
magnetic nature of the diatomic molecule B2 is (2010)
22. In the compound CH2 == CH  CH2  CH2  C ≡≡ CH, the
C2  C3 bonds is of (1999, 2M)
(a) 1 and diamagnetic (b) 0 and diamagnetic
(a) sp - sp 2 (b) sp 3 - sp 3
(c) 1 and paramagnetic (d) 0 and paramagnetic
(c) sp - sp 3 (d) sp 2 - sp 3
11. The species having bond order different from that in CO is
23. The geometry of H2 S and its dipole moment are (1999, 2M)
(a) NO− (b) NO+ (2007, 3M)
(a) angular and non-zero (b) angular and zero
(c) CN− (d) N2 (c) linear and non-zero (d) linear and zero
12. Among the following, the paramagnetic compound is 24. The geometry and the type of hybrid orbital present about the
(2007, 3M) central atom in BF3 is (1998, 2M)
(a) Na 2 O2 (b) O3 (c) N2 O (d) KO2 (a) linear, sp (b) trigonal planar, sp 2
13. Which of the following contains maximum number of lone (c) tetrahedral, sp 3 (d) pyramidal, sp 3
pairs on the central atom? (2005, 1M) 25. Which one of the following compounds has
(a) ClO −3 (b) XeF 4 (c) SF 4 (d) I −3 sp 2 - hybridisation? (1997, 1M)

14. Number of lone pair(s) in XeOF4 is/are (a) CO2 (b) SO2 (c) N2 O (d) CO
(2004, 1M)
(a) 0 (b) 1 (c) 2 (d) 3 26. Among KO2 , AlO−2 , BaO2 and NO+2 , unpaired electron is
present in (1997 C, 1M)
15. Which of the following are isoelectronic and isostructural ?
(a) NO+2 and BaO2 (b) KO2 and AlO−2
NO–3 , CO2– –
3 , ClO3 , SO3 (2003, 1M)
(c) Only KO2 (d) Only BaO2
(a) NO3 , CO2−

3 (b) SO3 , NO–3
27. The cyanide ion CN− and N2 are isoelectronic, but in contrast
(c) ClO–3 , CO2– (d) CO2–
3 , SO3
3
to CN− , N2 is chemically inert because of (1997 C, 1M)
16. Among the following, the molecule with the highest dipole (a) low bond energy
moment is (2003, 1M) (b) absence of bond polarity
(a) CH 3 Cl (b) CH 2 Cl 2 (c) unsymmetrical electron distribution
(c) CHCl 3 (d) CCl 4 (d) presence of more number of electron in bonding orbitals
Chemical Bonding 51

28. Among the following species, identify the isostructural 40. On hybridisation of one s and one p-orbital we get
pairs. (a) two mutually perpendicular orbitals (1984, 1M)
NF3 , NO3− , +
BF3 , H3 O , N3 H (b) two orbitals at 180°
(1996, 1M) (c) four orbitals directed tetrahedrally
(d) three orbitals in a plane
(a) [NF3 ,NO−3 ] and [BF3 ,H3 O+ ]
41. Carbon tetrachloride has no net dipole moment because of
(b) [NF3 , N3 H] and [NO−3 ,BF3 ]
(a) its planar structure (1983, 1M)
(c) [ NF3 , H3 O+ ] and [NO–3 , BF3 ] (b) its regular tetrahedral structure
(d) [NF3 , H3 O+ ] and [N3 H, BF3 ] (c) similar sizes of carbon and chlorine atoms
29. Which one of the following molecules is planar? (1996, 1M) (d) similar electron affinities of carbon and chlorine
(a) NF3 (b) NCl 3 (c) PH3 (d) BF3 42. The ion that is isoelectronic with CO is (1982, 1M)
(a) CN− (b) O+2 (c) O−2 (d) N+2
30. The maximum possible number of hydrogen bonds a water
molecule can form is (1992, 1M) 43. Among the following, the linear molecule is (1982, 1M)
(a) 2 (b) 4 (c) 3 (d) 1 (a) CO2 (b) NO2 (c) SO2 (d) ClO2
31. The type of hybrid orbitals used by the chlorine atom in 44. If a molecule MX 3 has zero dipole moment, the sigma
ClO−2 is (1992, 1M) bonding orbitals used by M (atomic number < 21) are
(a) sp 3
(b) sp 2 (a) pure p (b) sp-hybridised (1981, 1M)

(c) sp (d) None of these (c) sp 2 -hybridised (d) sp 3 -hybridised

32. The molecule which has pyramidal shape is (1989, 1M) Objective Questions II
(a) PCl 3 (b) SO3 (c) CO2– (d) NO–3
3 (One or more than one correct option)
33. Which of the following is paramagnetic? (1989, 1M) 45. The molecules that will have dipole moment are (1992, 1M)
(a) O–2 (b) CN– (c) CO (d) NO+ (a) 2, 2-dimethyl propane (b) trans-2-pentene
34. The Cl—C—Cl angle in 1, 1, 2, 2-tetrachloroethene and (c) cis-3-hexene (d) 2,2,3,3-tetramethyl butane
tetrachloromethane respectively will be about (1988, 1M)
46. Which of the following have identical bond order?
(a) 120° and 109.5° (b) 90° and 109.5° (a) CN– (b) O–2 (1992, 1M)
(c) 109° and 90° (d) 109.5° and 120° (c) NO+ (d) CN+
35. The molecule that has linear structure is 47. The linear structure assumed by (1991, 1M)
(1988, 1M)
(a) CO2 (b) NO2 (c) SO2 (d) SiO2 (a) SnCl 2 (b) CS2 (c) NO+2 (d) NCO–

36. The species in which the central atom uses sp 2 -hybrid 48. CO2 is isostructural with (1986, 1M)
orbitals in its bonding is (1988, 1M) (a) HgCl 2 (b) C2 H2 (c) SnCl 2 (d) NO2
(a) PH3 (b) NH3 (c) CH+3 (d) SbH3
37. Of the following compounds, which will have a zero dipole Match the Columns
moment ? (1987, 1M) 49. Match the orbital overlap figures shown in Column I with the
(a) 1, 1-dichloroethylene description given in Column II and select the correct answer
(b) cis-1, 2-dichloroethylene using the codes given below the Columns. (2014 Adv.)

(c) trans-1, 2-dichloroethylene Column I Column II


(d) None of the above A. 1. p-d π antibonding
38. The hybridisation of sulphur in sulphur dioxide is (1986, 1M)
(a) sp (b) sp 3
B. 2. d-d σ bonding
(c) sp 2 (d) dsp 2
39. The bond between two identical non-metal atoms has a pair
of electrons (1986, 1M) C. 3. p-dπ bonding
(a) unequally shared between the two
(b) transferred fully from one atom to another
D. 4. d-d σ antibonding
(c) with identical spins
(d) equally shared between them
52 Chemical Bonding

Codes 64. In benzene, carbon uses all the three p-orbitals for
A B C D A B C D hybridisation. (1987, 1M)
(a) 4 3 2 1 (b) 1 2 3 4
(c) 2 3 1 4 (d) 4 1 2 3 65. SnCl 2 is a non-linear molecule. 1
(1985, M)
2
50. Match each of the diatomic molecules in Column I with its
property/properties in Column II. (2009) Integer Answer Type Questions
Column I ColumnII 66. The sum of the number of lone pairs of electrons on each
A. B2 p. Paramagnetic central atom in the following species is
B. N2 q. Undergoes oxidation [TeBr6 ]2 − , [BrF2 ]+ , SNF3 and [XeF3 ]−
C. O−2 r. Undergoes reduction (Atomic numbers : N = 7, F = 9, S = 16, Br = 35,
Te = 52, Xe = 54) (2017 Adv.)
D. O2 s. Bond order ≥ 2
67. Among the triatomic molecules/ions BeCl 2 , N−3 , N2 O, NO+2 ,
t. Mixing of ‘s’ and ‘p’ orbitals
O3 , SCl 2 , ICl −2 , I−3 and XeF2 , the total number of linear
Codes
molecules(s)/ion(s) where the hybridisation of the central
A B C D
atom does not have contribution from the d-orbital(s) is
(a) q, r, s p, r, t, s q, r, t p, q, t
[atomic number of S = 16 , Cl = 17 , I = 53 and Xe = 54]
(b) p, q, r, t q, r, s, t p, q, r, t p, r, s, t (2015 adv.)
(c) q, r, s, t p, q, r r, s, t p, q, r, t
(d) p, q, s, t p, q, s p, t q, r, t 68. A list of species having the formula XZ4 is given below
(2014 Adv.)

Fill in the Blanks XeF4 , SF4 , SiF4 , BF4− , BrF4− , [Cu(NH3 )4 ] 2+ , [FeCl 4 ] 2− ,

51. Among N2 O, SO2 , I+3 and I–3 , the linear species are …… and [CoCl 4 ] 2− and [PtCl 4 ] 2−
…… (1997 C, 1M)
Defining shape on the basis of the location of X and Z atoms,
the total number of species having a square planar shape is
52. When N2 goes to N+2 , the N  N bond distance … , and when
69. The total number of lone-pair of electrons in melamine is
O2 goes to O+2 the O  O bond distance …… (1996, 1M) (2013 Adv.)

53. The two types of bonds present in B2 H6 are covalent and …… 70. Based on VSEPR theory, the number of 90° F—Br—F angles
(1994, 1M) in BrF5 is (2010)

54. The kind of delocalisation involving sigma bond orbitals is


called................. (1994, 1M)
Subjective Questions
55. The valence atomic orbitals on C in silver acetylide is 71. Predict whether the following molecules are isostructural or
.............hybridised. (1990, 1M) not. Justify your answer.
(i) NMe3 (ii) N(SiMe3 )3 (2005, 2M)
56. The shape of CH3 + is …… . (1990, 1M)
72. On the basis of ground state electronic configuration, arrange
57. …… hybrid orbitals of nitrogen atom are involved in the the following molecules in increasing O—O bond length
formation of ammonium ion. (1982, 1M) order. KO2 , O2 , O2 [AsF6 ] (2004, 2M)
58. Pair of molecules which forms strongest intermolecular 73. Draw the shape of XeF4 and OSF4 according to VSEPR
hydrogen bonds is ……… . (SiH4 and SiF4 , acetone and
theory. Show the lone pair of electrons on the central atom.
CHCl 3 , formic acid and acetic acid) (1981, 1M)
(2004, Main, 2M)
59. The angle between two covalent bonds is maximum in …… . 74. Using VSEPR theory, draw the shape of PCl 5 and BrF5 .
(CH4 , H2 O, CO2 ) (1981, 1M)
(2003, 2M)

True/False 75. Draw the molecular structures of XeF 2 , XeF 4 and XeO2 F2 ,
indicating the location of lone pair(s) of electrons. (2000, 3M)
60. The dipole moment of CH3 F is greater than that of CH3 Cl.
(1993, 1M) 76. Interpret the non-linear shape of H2 S molecule and
non-planar shape of PCl 3 using valence shell electron pair
61. H2 O molecule is linear. (1993, 1M) repulsion (VSEPR) theory. (Atomic number : H = 1, P = 15,
62. The presence of polar bonds in a polyatomic molecule S = 16, Cl = 17) (1998, 4M)
suggests that the molecule has non-zero dipole moment. 77. Using the VSEPR theory, identify the type of hybridisation
(1990, 1M) and draw the structure of OF2 . What are the oxidation states of
63. sp 3 hybrid orbitals have equal s and p character. (1987, 1M) O and F ? (1997, 3M)
Chemical Bonding 53

+
78. Write the Lewis dot structural formula for each of  H   H 
the following. Give also, the formula of a neutral molecule,  ••••   •••• 
which has the same geometry and the same arrangement of H • •O• • H H • •N• • H
the bonding electrons as in each of the following. An    
   
example is given below in the case of H3 O+ and NH3 .
Lewis dot. structure Neutral .molecule (1983, 4M)

(i) O2−
2 (ii) CO2−
3 (iii) CN (iv) NCS−

Topic 3 Resonance, LCAO, MOT, Other Bonding Types


Objective Questions I (Only one correct option) 7. Two pi and half sigma bonds are present in
(2019 Main, 10 Jan I)
1. The intermolecular potential energy for the molecules A, B, C
and D given below suggests that: (2020 Main, 4 Sep I) (a) O+2 (b) N2 (c) N+2 (d) O2
Interatomic distance (pm) 8. According to molecular orbital theory, which of the following
50 100 150 is true with respect to Li +2 and Li −2 ? (2019 Main, 9Jan I)
0
(a) Both are unstable
–100
(b) Li+2 is unstable and Li−2 is stable
–200 (c) Both are stable
Potential
Energy –300 (d) Li+2 is stable and Li−2 is unstable
(kJ mol–1) A–D
–400 9. According to molecular orbital theory, which of the
A–A A–C
–500 following will not be a viable molecule? (2018 Main)

–600
A–B (a) He2+
2 (b) He+2
(c) H−2 (d) H2−
2
(a) A-B has the stiffest bond 10. Which of the following species is not paramagnetic?
(b) D is more electronegative than other atoms (2017 Main)
(c) A-A has the largest bond enthalpy (a) NO (b) CO
(d) A-D has the shortest bond length (c) O2 (d) B2
2. During the change of O2 to O−2 , the incoming electron goes to 11. Assuming 2s-2p mixing is not operative, the paramagnetic
the orbital. (2019 Main, 10 April I) species among the following is (2014 Adv.)

(a) π2 px *
(b) π 2 px (c) π2 p y *
(d) σ 2 p z (a) Be2 (b) B2
(c) C2 (d) N2
3. HF has highest boiling point among hydrogen halides, 12. Stability of the species Li 2 , Li −2 and Li +2 increases in the order
because it has (2019 Main, 9 April II) of (2013 Main)
(a) lowest ionic character (a) Li 2 < Li 2+ < Li 2− (b) Li –2 < Li +2 < Li 2
(b) strongest van der Waals’ interactions (c) Li 2 < Li 2− < Li 2+ (d) Li −2 < Li 2 < Li +2
(c) strongest hydrogen bonding
13. In which of the following pairs of molecules/ions both the
(d) lowest dissociation enthalpy
species are not likely to exist? (2013 Main)
4. Among the following species, the diamagnetic molecule is (a) H+2 , He2− (b) H −2 , He22 −
2
(2019 Main, 9 April II)
(a) CO (b) B2 (c) NO (d) O2 (c) H22 + , He2 (d) H−2 , He2+
2

5. Among the following, the molecule expected to be stabilised 14. Hyperconjugation involves overlap of which of the following
by anion formation is C2 , O2 , NO, F2 . (2019 Main, 9 April I) orbitals? (2008, 3M)
(a) C2 (b) F2 (a) σ - σ (b) σ - p
(c) NO (d) O2 (c) p - p (d) π - π

6. Among the following molecules/ions, C2− 2− 2− 15. According to MO theory, (2004, 1M)
2 , N2 , O2 , O2
(a) O+2 is paramagnetic and bond order greater than O2
Which one is diamagnetic and has the shortest bond length?
(2019 Main, 8 April II)
(b) O+2 is paramagnetic and bond order less than O2
(a) C2−
2 (b) O2 (c) O2−
2 (d) N 2−
2
(c) O+2 is diamagnetic and bond order is less than O2
(d) O+2 is diamagnetic and bond order is more than O2
54 Chemical Bonding

16. Molecular shape of SF4 , CF4 and XeF4 are 24. Statement I The electronic structure of O 3 is
(2000, 1M)
(a) the same, with 2, 0 and 1 lone pair of electrons respectively +
O
(b) the same, with 1, 1 and 1 lone pair of electrons respectively •• −
• •
(c) different, with 0, 1 and 2 lone pair of electrons respectively • O • O •• .
•• ••
(d) different, with 1, 0 and 2 lone pair of electrons respectively
O
17. In compounds of type ECl 3 , where E = B, P, As or Bi, the ••

angles Cl—E—Cl is in order (1999, 2M) Statement II • O O •• structure is not allowed
•• ••
(a) B > P = As = Bi (b) B > P > As > Bi because octet around O cannot be expanded. (1998, 2M)
(c) B < P = As = Bi (d) B < P < As < Bi
Match the Columns
18. The correct order of increasing CO bond length of
CO, CO23 − , CO2 is (1999, 2M) 25. Match the reactions in Column I with nature of the
reactions/type of the products in Column II. (2007, 6M)
(a) CO23 − < CO2 < CO (b) CO2 < CO2–
3 < CO
Column I Column II
(c) CO < CO2– 3 < CO2 (d) CO < CO2 < CO2–
3
A. O−2 → O2 + O22 − 1. Redox reaction
19. Which contains both polar and non-polar bonds? (1997, 1M)
(a) NH4 Cl (b) HCN B. CrO24 − + H+ → 2. One of the products has
(c) H2 O2 (d) CH4 trigonal planar structure
20. Which one among the following does not have the hydrogen C. MnO−4 + NO−2 3. Dimeric bridged
bond? (1983, 1M) + tetrahedral metal ion
+ H →
(a) Phenol (b) Liquid NH3
(c) Water (d) HCl D. NO−3 + H2 SO4 4. Disproportionation
2+
Objective Questions II + Fe →

(One or more than one correct option)


Codes
21. According to molecular orbital theory, which of the
A B C D A B C D
following statements is(are) correct? (2016 adv.)
(a) 2 1, 4 3 4 (b) 1, 4 3 1, 2 1
(a) C2−
2 is expected to be diamagnetic (c) 2 3 1 4 (d) 3 4 2, 3 1
(b) O2+
2 is expected to have a longer bond length than O2
(c) N+2 and N−2 have the same bond order Integer Answer Type Questions
(d) He+2 has the same energy as two isolated He atoms 26. Chlorine reacts with hot and concentrated NaOH and produces
22. Hydrogen bonding plays a central role in which of the compounds ( X ) and (Y ). Compound ( X ) gives white
following phenomena? (2014 Adv.)
precipitate with silver nitrate solution. The average bond
order between Cl and O atoms in (Y ) is …… .
(a) Ice floats in water
(2020 Main, 7 Jan I)
(b) Higher Lewis basicity of primary amines than tertiary
amines in aqueous solutions 27. Among H2 ,He+2 , Li 2 , Be2 , B2 , C2 , N2 , O−2 and F2 , the
(c) Formic acid is more acidic than acetic acid number of diamagnetic species is
(d) Dimerisation of acetic acid in benzene
(Atomic numbers : H = 1, He = 2, Li = 3, Be = 4, B = 5, C = 6,
23. Which one of the following molecules is expected to exhibit N = 7, O = 8, F = 9) (2017 Adv.)
diamagnetic behaviour? (2013 Main)
(a) C 2 (b) N 2 (c) O 2 (d) S 2 Subjective Questions
28. Write the MO electron distribution of O2 . Specify its bond
Assertion and Reason
order and magnetic property. (2000, 3M)
Read the following questions and answer as per the direction given
below: 29. Arrange the following as stated.
(a) Statement I is correct; Statement II is correct; Statement II is “Increasing strength of hydrogen bonding ( X  H  X ).”
the correct explanation of Statement I. O, S, F, Cl, N (1991, 1M)
(b) Statement I is correct; Statement II is correct; Statement II is 30. What effect should the following resonance of vinyl chloride
not the correct explanation of Statement I.
have on its dipole moment? (1987, 1M)
(c) Statement I is correct; Statement II is incorrect. +
(d) Statement I is incorrect; Statement II is correct. CH2 ==CH  Cl ←→ CH2–  C HCl
Answers
Topic 1
1. (d) 2. (c) 3. (b) 4. (a) 49. (c) A → 2; B → 3; C →1; D →4
5. (a) 6. (b) 7. (b) 8. (b) 50. (b) A → p, q, r, t; B → q, r, s, t; C → p, q, r, t; D → p, r, s, t
9. (a) 10. (a) 11. (c) 12. (c) 51. N 2O, I 3− 52. increases, decreases
13. (a, c) 14. (a) 15. (6) 16. (1) 53. three centre bond-two electrons
17. (c) 18. (2) 19. F 20. T 54. hyperconjugation
25. (80.2%) 55. sp 56. Triangular planar 57. sp 3
Topic 2 58. HCOOH and CH3COOH 59. CO2 60. F
1. (d) 2. (b) 3. (c) 4. (d) 61. F 62. F 63. F 64. F
5. (d) 6. (c) 7. (a) 8. (d) 65. T 66. (6) 68. (4) 69. (6)
9. (d) 10. (a) 11. (a) 12. (d)
Topic 3
13. (d) 14. (b) 15. (a) 16. (a)
1. (a) 2. (b) 3. (c) 4. (a)
17. (c) 18. (a) 19. (b) 20. (a)
5. (a) 6. (a) 7. (c) 8. (d)
21. (b) 22. (d) 23. (a) 24. (b)
9. (d) 10. (b) 11. (c) 12. (b)
25. (b) 26. (c) 27. (b) 28. (c)
13. (c) 14. (b) 15. (a) 16. (d)
29. (d) 30. (b) 31. (a) 32. (a)
17. (b) 18. (a) 19. (c) 20. (d)
33. (a) 34. (a) 35. (a) 36. (c)
21. (a, c) 22. (a, b, d) 23. (a,b) 24. (a)
37. (c) 38. (c) 39. (d) 40. (b)
25. (b) A → 1, 4; B → 3; C →1, 2; D →1 26. (1.67)
41. (b) 42. (a) 43. (a) 44. (c)
27. (6) 28. (2)
45. (b, c) 46. (a, c) 47. (b, c, d) 48. (a, b)

Hints & Solutions


Topic 1 Preliminary Concepts of Electrovalent
Covalent
and Covalent Bonding P bond
O O
1. Key Idea Isoelectronic species contains same number of H H (O2)
H
electrons. (PH3) Covalent
H H 1.33Å Bonds
Å H
The species with its atomic number and number of electrons are 1.19
as follows : 120° B 97° B
H H H
Species (ions) At. no. ( Z ) No. of electrons
H H H
N3− 7 7 + 3 = 10 H
H H
O2− 8 8 + 2 = 10 B B B B
H H H
F− 9 9 + 1 = 10 H H (B2H6) H Banana bond
Na + 11 11 − 1 = 10 O
+
Li 3 3 −1 = 2 S
Mg 2+
12 12 − 2 = 10 HO OH O
Sulphuric acid
Thus, option (d) contains isoelectronic set of ions. (H2SO4)
2. KCl is the only ionic compound. The structure of PH3, O2, B2H6 All bond between S and O atom are covalent bonds.
and H2SO4 are given below
56 Chemical Bonding

3. Ion-ion interaction is dependent on the square of distance, i.e. 8. BF3 has triangular planar arrangement.
1
ion-ion interaction ∝ F
r2 120°
1 F—B
Similarly, ion-dipole interaction ∝ F
r3
1 1 sp2
London force ∝ and dipole-dipole interaction ∝
r6 r3 Three identical vectors acting in outward direction at equal
Superficially it seems as both ion-dipole interaction and angles in a plane cancel each other giving zero resultant, hence
hydrogen bonding vary with the inverse cube of distance between non-polar.
the molecules but when we look at the exact expressions of field 9. Strongly electropositive, univalent X will form an 1 : 1 ionic
(force) created in two situations, compound with strongly electronegative, univalent Y.
it comes as X + Y → X +Y −
2| P |
| E | or | F | = (In case of ion-dipole interaction) 10. H2 is a covalent, diatomic molecule with a sigma covalent bond
4 π ∈ r3
between two hydrogen atoms.
2q2r – 4 q2a 11. N2 has triple bond and each covalent bond is associated with
and F = (In case of dipole-dipole interaction)
4 π ∈0 r3 one pair of electrons, therefore, six electrons are involved in
forming bonds in N2.
From the above, it is clear that the ion-dipole interaction is the
better answer as compared to dipole-dipole interaction, i.e. 12. In KCN, the bonding between potassium ion and cyanide ion is
hydrogen bonding. ionic while carbon and nitrogen are covalently bonded in
cyanide ion as:
Covalent bonds
H H + –
4. [K] [ C ≡≡N]
C—–C
H H Ionic bond

Pi bond is formed by the p-orbitals whose lobes have minima in 13. Key Idea Dipole moment of a bond depends on the
the plane of molecule, hence molecular plane is the nodal plane of difference in the electronegativities of bonded atoms.
pi-bond. More is the difference in the electronegativities, greater
will be the dipole moment. Also,
5. H-bond is the strongest intermolecular force. For symmetrical molecule, µ = 0
All are different with 1, 0 and 2 lone pairs of electrons at central
For unsymmetrical molecule, µ ≠ 0
atom.
The molecules which gives permanent dipole moment are
6. p-dichlorobenzene is non-polar.
polar in nature.
Cl— —Cl Cl
S Se
p-dichlorobenzene , ,
The two dipole vectors cancelling each other giving zero O O H H
resultant dipole moment. o-dichlorobenzene has greater m¹0 m¹0 m¹0
(polar) (polar) (polar)
dipole moment than meta-isomer.
F
Cl F F
m1 m2
Cl µ1 > µ2 Br
Cl Be Cl
µ1
,
µ2 F m¹0 F mnet» 0
(non-polar)
Cl Cl (polar)

(o-dichlorobenzene) (m-dichlorobenzene) Cl
dipole vectors are at 60° angle dipole vectors are at 120° angle m1 m2
,
O C O B Cl
Toluene is less polar than both ortho and para
mnet 0 m 0
dichlorobenzene. Therefore, the increasing order of dipole Cl
(non-polar) (non-polar)
moment is
p-dichlorobenzene (IV) < toluene (I) < m-dichlorobenzene (II) Cl
N N Cl
< o-dichlorobenzene (III)
H C Cl , , , O P Cl ,
7. The carbide (C2−
2 ) ion has the following bonding pattern:
O O H H H
Cl Cl
– – m¹0 m¹0 m¹0 m¹0
: C ≡≡ C : one sigma and two pi-bonds. (polar) (polar) (polar) (polar)
Chemical Bonding 57

F F
(vi) (NH4)2FeCl 4 has Fe as central metal atom with +2 oxidation
H F F state. The electronic configuration of Fe 2+ in the complex is
B O
H C Cl , , , Xe 4s 4p
3d
F F O O
H m¹0 m=0 sp3
(polar) (non-polar) m¹0 F F
Cl– Cl– Cl– Cl– hybridisation
(polar) F 4 unpaired
m»0
(non-polar)
electrons

Thus, options (a, c) are correct. (vii) (NH4)2NiCl 4 has Ni as central metal atom with +2 oxidation
state. The electronic configuration of Ni 2+ in the complex is
14. 1,4-dichlorobenzene is non-polar, individual dipole vectors
4s 4p
cancel each other. 3d
µ≠0 sp3
Cl Cl H Cl ClCH2 H Cl– Cl– Cl– Cl– hybridisation
2 unpaired
C==C C==C C==C electrons
H H Cl H Cl C2 H 5 (viii) In K2MnO4 central metal atom Mn has +6 oxidation state
µ=0
Polar Non-polar Polar with following structure
O–
15. Only polar liquid will be attracted towards charged comb due to
the formation of electrically charged droplets in the polar liquid 2K+ Mn
stream, induced by a nearby charged object. Hence, liquid –
O
showing deflection are HF,H2O,NH3 ,H2O2 ,CHCl 3 , C6H5Cl. O O
16. Among the given species only K2CrO4 is diamagnetic as central Electronic configuration of Mn6+ is
0
metal atom Cr in it has [ Ar ]3d electronic configuration i.e., all
3d 4s
paired electrons. The structure and oxidation state of central
metal atom of this compound are as follows
O one unpaired
electron
Structure K+ Cr , Oxidation state Cr6+
O – 17. Statement I is correct but Statement II is incorrect. The
O– O covalency in LiCl is due to small size of Li + ion which brings
Rest all the compounds are paramagnetic. Reasons for their about large amount of polarisation in bond.
paramagnetism are given below 18. These are 2π-bonds in a nitrogen molecule.
(i) H-atom have 1s1 electronic configuration, i.e. 1 unpaired 19. The resultant of individual bond dipoles may or may not be
electron. non-zero.
20. Linear overlapping of p-orbitals form sigma bond while sidewise
N
(ii) NO2 , i.e. in itself is an odd electron species. overlapping of two p-orbitals forms a pi bond.
O O
(iii) O−2 (Superoxide) has one unpaired electron in π * molecular 21. Li + < Al 3+ < Mg2+ < K+
orbital. 22. Ag+ is stronger Lewis acid because it can easily accommodate
(iv) S2 in vapour phase has O2 like electronic configuration i.e., lone pair of electrons from Lewis base. On the other hand, Na +
have 2 unpaired electrons in π * molecular orbitals. has noble gas configuration, cannot accept lone pair of electron,
not at all a Lewis acid.
(v) Mn 3O4 has following structure
23. I2 is Lewis acid because I − coordinate its one lone pair to I2.
+2 O +4
O +2
Mn Mn Mn 24. Both LiF and LiI are expected to be ionic compounds. However ,
O O LiI is predominantly covalent because of small size of Li + and
large size of iodide ion. A smaller cation and a larger anion
Thus, Mn is showing +2 and +4 oxidation states. The outermost introduces covalency in ionic compound.
electronic configuration of elemental Mn is 3d 5 4 s2. Hence, in
both the above oxidation states it has unpaired electrons as 25. Dipole moment is calculated theoretically as
3d 4s µ = q⋅d
Mn2+ Here, q = 1.6 × 10−19 C and d = 2.6 × 10−10 m
5 unpaired electrons µ Theo = 1.6 × 10−19 × 2.6 × 10−10 = 4.16 × 10−29 cm
3d 4s µ obs 3.336 × 10−29
Mn4+ % ionic character = × 100 = × 100
µ Theo 4.16 × 10−29
3 unpaired electrons = 80.2%
58 Chemical Bonding

26. In hydrogen peroxide (H2O2 ), oxygen is in –1 oxidation state, The hybridisation of given species are as follows :
can be oxidised to O2 (zero oxidation state) or can be reduced to l
For [ ICl2 ]− and [ BrF2 ]−
H2O (–2 oxidation state of oxygen).
1
H = (7 + 2 − 0 + 1) = 5 (sp3d )
Hence, H2O2 can act as both oxidising agent and reducing agent. 2
With strong oxidising agent like KMnO4, H2O2 acts as a l
For [ ICl4 ]− ,
reducing agent while with strong reducing agent like H2C2O4 , it
1
acts as an oxidising agent. H = (7 + 4 − 0 + 1) = 6 (sp3d 2 )
2
27. (i) Melting points Ionic compounds have higher melting points l
For [ IF6 ]− ,
than covalent compounds.
(ii) Boiling points Ionic compounds have higher boiling points 1
(7 + 6 − 0 + 1) = 7 (sp3d 3 )
H =
than covalent compounds. 2
(iii) Solubility Ionic compounds have greater solubility in water 4. The radius of isoelectronic species is inversely proportional to
than a covalent compound. their nuclear charge or atomic number (Z). Thus, greater the
value of Z, lesser the radii of isoelectronic species.
(iv) Conductivity in aqueous solution Ionic compounds have
greater electrical conductivity in aqueous solution while 5.
covalent compounds are usually non-conducting. Species Valence MOs Bond Order Paramagnetic/
 N b − N a  Diamagnetic
  Nature
 2 
Topic 2 VBT, Hybridisation and 6−1
NO(15e− ) [ 8 e− ] π 2 px2 = π 2 py2σ 2 pz2 = 2.5 Paramagnetic
VSEPR Theory 2
1. Among the given statements, correct statements are I and III –e− π * 2 p1x = π * 2 py0σ * 2 pz 0
only. Valence bond theory (VBT) cannot explain the colour
exhibited by transition metal complexes. This theory cannot [ 8 e− ] π 2 px2 = π 2 py2σ 2 pz2 6−0 Diamagnetic
=3
distinguish ligands as weak and strong field ones. NO + (14e− ) 2
•• π * 2 px0 = π * 2 py0σ * 2 pz0
2. For ICl5
1 N 2 (14 e− ) [ 8 e− ] π 2 px2 = π 2 py2σ2 pz2 6−0 Diamagnetic
H= (7 + 5 − 0 + 0) = 6 (sp3d 2 ) −
=3
2 –e 2
N 2+ (13 e− ) π * 2 px0 = π * 2 py0 , σ 2 pz0 5−0 Paramagnetic
Cl = 2.5
2
[ 8e− ] π 2 px2 = π 2 p2y σ 2 pz 1
3 2
Cl Cl sp d hybridised
Geometry : Octahedral
I Shape / Structure : Square pyramidal
π * 2 px0 = π * 2 p0y σ * 2 pz0

Cl Cl
O2 (16 e− ) [ 8 e− ] σ 2 pz2 π 2 px2 = π 2 py2 6−2 Paramagnetic
=2
For &&ICl−4 π * 2 p1x = π * 2 p1y σ * 2 pz0
2
1 –e− [ 8 e− ] σ 2 pz2 π 2 px2 = π 2 py2
H = (7 + 4 − 0 + 1) = 6 (sp3d 2 ) O2+ (15 e− )
2 6−1
π * 2 p1x = π * 2 py0σ * 2 pz0 = 2.5 Paramagnetic
Cl Cl sp 3 d 2 hybridised +2e− 2
s Geometry : Octahedral [ 8 e− ] σ 2 pz2 π 2 px2 = π 2 py2
I Shape/Structure : Square planar O22− (18 e− )
Cl Cl π * 2 px2 = π * 2 py2σ * 2 pz0 6 − 4 = 1 Diamagnetic
2
So, ICl5 and ICl−4 are isolobal but not isostructural.
So, only in the conversion of NO → NO+, the bond order has
3. Key Idea The hybridisation for a central atom in a species increased (2.5 → 3) and paramagnetic character has changed to
can be calculated using formula diamagnetic.
1
H = (V + M − C + A ) 6. The structure of I−3 ion is
2 –
where, H = No. of hybridised orbitals used by central atoms. I
V = No. of valence electrons of the central atom.
M = No. of mono-valent atoms (bonded).
C = No. of cationic (positive) charge. I
A = No. of anionic (negative) charge.
I

Hence, 9 is the correct answer.


Chemical Bonding 59

7. Isoelectronic species are those which contains same number of SF4 : F one lone pair at S.
electrons. F
S
Species Atomic number Number of electrons F
F
O2 − 8 10
•• •• −
F− 9 10 I3− : I I •
• ← •
• I •• three lone pairs at central iodine.
•• ••
+
Na 11 10 O
2+ F  F
Mg 12 10
14. Xe At central atom (Xe), there is one lone pair.
• •
O− 8 9 F F
Na 11 11
15. NO−3 and CO2−
3 both have 32 electrons, central atom
+
Mg 12 11 sp2 hybridised, triangular planar.
∴ Option (a) is correct which contains isoelectronic species 16. CH3Cl has the highest dipole moment.
O2 − , F − , Na + , Mg 2 + . 17. O−2 has odd number(17) of electrons, therefore it must contain
8. I −3 is an ion made up of I 2 and I − which has linear shape. at least one unpaired electron.
While Cs + is an alkali metal cation.  F H 
   
9. 18. F  B− ← N+  H Both ‘B’ and ‘N’ sp3 tetrahedral.
F—S==O S    
 F H 

F O  
F F
S is sp3 hybridised Pyramidal 19. NH3 = sp3 ,[ PtCl 4 ]2− = dsp2 , PCl 5 = sp3d , BCl 3 = sp2
SO3 is planar (S is sp2 hybridised), BrF3 is T-shaped and SiO2−
3 is
20. All three have 14 electrons (iso electronic) with bond order of
planar (Si is sp2 hybridised). three.
O H
10. For molecules lighter than O2, the increasing order of energies of
+ +
molecular orbitals is –
N H
O==N==O, O—N==O,
* 21.
π 2 p y  π 2 p y ..... H
σ1s σ* 1sσ 2s σ* 2s  σ 2 p σ
*
2 p sp sp2 sp3 H
 π 2 p z
x x
*
π 2 p z 1 2 3 4 5 6
22. CH2 == CH  CH2  CH2  C ≡≡ CH
where, π2 p y and π2 p z are degenerate molecular orbitals, first
Hybridisation at C2 = sp2 and at C3 = sp3.
singly occupied and then pairing starts if Hund’s rule is obeyed.
If Hund’s rule is violated in B2 , electronic arrangement would 23. H2S has sp3 hybridised sulphur, therefore, angular in shape
be with non-zero dipole moment.
π 2 p 2y H
σ1 s2 σ* 1s2 σ 2s2 σ* 2s2  ...
π 2 p z
S
No unpaired electron-diamagnetic.
H
bonding electrons − antibonding electrons
Bond order =
2
6− 4 (Non-linear, polar molecule)
= =1 F
2
+ −
11. The bond order of CO = 3. NO , CN and N2 are isoelectronic 24. F—B
with CO, have the same bond order as CO. NO− (16e− ) has bond F
order of 2. sp2
12. O−2 in KO2 has 17 electrons, species with odd electrons are always (Trigonal planar)
paramagnetic. 25. Sulphur in SO2 is sp2-hybridised.
••
13. ClO−3 : − O  Cl == O one lone pair at Cl. S
 O O
O Electron pair = 2 (σ-bonds) + 1 (lone pair) = 3
XeF4 : F Xe two lone pairs at Xe. Hybridisation = sp 2
Carbon in CO2 is sp-hybridised, N in N2 O is sp-hybridised,
F F carbon in CO is sp-hybridised.
F
60 Chemical Bonding

26. Molecular orbital electronic configuration are 32. PCl 3 has sp3-hybridised phosphorus, with one lone pair. Therefore,
molecule has pyramidal shape like ammonia.
π 2 p 2y π* 2 p 2y
KO2 (O−2 ) : σ1s2 σ* 1s2 σ 2s2 σ* 2s2 σ 2 px2 σ* 2 px0 33. O−2 has odd number of electrons, hence it is paramagnetic.
π 2 p 2z *
π 2 p1z Cl
Cl Cl 109°
Has one unpaired electron in π* 2 p orbital.
C==C 120° C Cl
AlO−2 has both oxygen in O2− state, therefore, no unpaired 34.
Cl Cl Cl
electron is present. BaO2 (O2− Cl
2 ) sp2-hybridised
* sp3-hybridised
π 2 p 2y π 2 p 2y *
σ1s2 σ* 1s2 σ 2s2 σ* 2s2 σ 2 px2 σ 2 px0
π 2 p 2z 35. CO2 is linear because carbon is sp-hybridised.
π* 2 p 2z
Has no unpaired electron. 36. In CH+3 , there are only three electron pairs around carbon atom
+
NO+2 has [O== N==O] bonding, hence no unpaired giving sp2-hybridisation state.
electron.
+
H
27. N2 is a neutral, non-polar, inert molecule while CN − is a H—C
highly polar, highly active ion. H
28. sp2-hybridised
F
NF3 : N F BF3 : F—B
F 37. Dipole vectors in trans-1, 2-dichloroethylene are at 180° and
F directed in opposite direction, cancelling each other.
F Triangular planar
Pyramidal (B-sp2) H Cl
(N-sp3)
C==C dipole moment = 0
– +
– O + O H
NO3 : H 3O : H Cl
O==N H
O H 38. In SO2, the Lewis-dot structure is
Triangular planar Pyramidal
• •
(N-sp2) (O-sp3)
O == S == O
•• + •• Electron pairs at S = 2 (σ-bonds) + 1 (lone-pair) = 3
N3H •• • N == N == N  H

Central nitrogen is sp -hybridised sp2 hybridised.
NOTE
Therefore, NF3 , H3O+ and BF3 , NO3− pairs have same shape.
π-bonded electrons are not present in hybrid orbitals, therefore not
counted in electron pairs. Rather π bonds are formed by lateral
29. BF3 has triangular planar arrangement. overlapping of pure p-orbitals.
F
39. Bonds between identical non-metal is purely covalent due to same
F—B 120° electronegativities of the bonded atoms. Hence, the bonded atoms
F have equal holds on the shared pair of electrons.
sp2-hybridised
40. Hybridisation of one ‘s’ and one ‘p’ orbitals gives two
There identical vectors acting in outward direction, at sp hybrid orbitals oriented linearly at 180°.
equal angles in a plane, cancel each other giving zero s + p → 2 sp hybrid orbitals
resultant, hence non-polar.
41. CCl 4 has a regular tetrahedral shape.
30. A water molecule can form at the most four H-bonds.
Cl
H µ
C µ Cl Net dipole = 0
O
Cl
H Cl
Four sites of H-bonding 42. CO has a total of 14 electrons and CN− also has 14 electrons.
• • C (6e− ) + N (7e− ) + e− → CN− (14 e− )

31. O  Cl == O
• • 43. CO2 is a linear molecule because of sp-hybridisation around
electron pairs at Cl = 2 (σ-bonds) + 2 (lone-pairs) = 4 carbon atom.
3 44. For non-polar MX 3, it must have triangular planar arrangement,
Hybridisation at Cl = sp
i.e. there should be sp2-hybridisation around M.
Chemical Bonding 61

CH3
H3C H C. p-d π antibonding

45. H3C  C  CH3 C == C
 H CH2CH3
CH3 Polar
Symmetric, non-polar

CH3 CH3
CH3H2C CH2CH3   D. d-d σ antibonding
C == C H3C  C  C  CH3
H H  
CH3 CH3
Polar Symmetric, non-polar ∴ A → 2, B → 3, C → 1, D → 4
Hence, (c) is the correct option.
46. CN− and NO+ are isoelectronic, have the same bond order of 3.
+
π 2 p1y
47. S== C == S O== N == O −
O  C ≡≡ N
50. (A) B2 : σ1s2 σ* 1s2 σ 2 s2 σ* 2s2 paramagnetic.
Linear Linear Linear π 2 pz1
•• •• 6− 4
Sn S Bond order = =1
2
Cl Cl O O
Bent Bent Bond is formed by mixing of s and p orbitals.
B2 undergoes both oxidation and reduction as
48. CO2 , HgCl 2 , C2H2 are all linear. Heat
B2 + O2 → B2 O3 (Oxidation)
49. PLAN This problem includes basic concept of bonding. It B2 + H2 → B2 H6 (Reduction)
can be solved by using the concept of molecular orbital theory.
π 2 p 2y
+ ve phase (B) N2 : σ1s2 σ* 1s2 σ 2 s2 σ* 2 s2σ 2 px2 diamagnetic.
10 − 4 π 2 p 2z
Bond order = = 3> 2
2
– ve phase N2 undergoes both oxidation and reduction as

Any orbital has two phase +ve and –ve. In the following N2 + O2 → NO
diagram, +ve phase is shown by darkening the lobes and –ve by Catalyst
without darkening the lobes. N2 + 3H2 → NH3
In N2 , bonds are formed by mixing of s and p orbitals.
π 2 p2y π* 2 p2
(C) O2− : σ1s2 σ
*
1s 2 σ 2 s 2 σ
*
2 s2 σ 2 px2 y σ
*
2 px0
π 2 pz2 π* 2 pz1
Bonding MO Antibonding MO −
Paramagnetic with bond order = 1.5. O2 undergoes both
When two same phase overlap with each other, it forms bonding
oxidation and reduction and bond involves mixing of s and
molecular orbital otherwise antibonding.
p-orbitals.
σ-bond
π 2 p2y π* 2 p1y
(D) O2 : σ1s2 σ
*
1s2 σ 2 s2 σ
*
2 s2 σ 2 px2 σ
*
2 px0
π 2 pz2 π* 2 p1
z
π-bond Paramagnetic with bond order = 2.
O2 undergoes reduction and the bond involves mixing of
s and p-orbitals.
On the basis of above two concepts, correct matching can be 51. N2O and I−3 are linear species.
done as shown below:
52. Bond order in N2 is 3 while same in N+2 is 2.5, hence bond
distance increases as N2 goes to N+2 .
A. d-d σ bonding Bond order in O2 is 2 while same in O+2 is 2.5, hence bond
distance decreases as O2 goes to O+2 .
53. Three centred-2 electrons.
54. Hyperconjugation involves delocalisation of σ-electrons.
B. p-d π bonding
55. sp-hybridised.
56. Triangular planar. Carbon in CH+3 is sp2 hybridised.
62 Chemical Bonding

57. sp3-hybrid orbital holding the lone pair is involved in formation 2+ 2–


of ammonium ion. H3N NH3 Cl Cl
O O Cu Pt
  H3N NH3 Cl Cl
58. H  C  OH and CH3  C  OH . Both are capable of forming
H-bonds. SF4 (See-saw) as shown below:
59. CO2, it is 180°. F
F
60. Dipole moment (µ ) = q.d S
F
Since electronegativity of F and Cl are very close, it is the
F
internuclear distance (d) that decides dipole moment here.
Hence, C  Cl bond has greater dipole moment the C-F bond. SiF4 , BF4− , [FeCl 4 ]2− , [CoCl 4 ]2− are tetrahedral as shown below:
61. H2O is V-shaped molecule.
O
H H
V-shaped
62. False
63. In sp3-hybrid orbital, there is 25 % s-character and 75 % p-character.
64. Carbon in benzene is sp2-hybridised, i.e. uses only two of its
p-orbitals in hybridisation.
65. Sn in SnCl 2 has sp2-hybridisation.
66. S.N. Species No. of σ-bonds No. of L.P at
with central atom central atom
Hence, correct integer is 4.
(i) In [ TeBr6 ]2− 6 1
(ii) In [ BrF2 ]+ 2 2 69. PLAN Melamine is a heterocyclic compound.

(iii) In SNF3 4 0 H2N N NH2


(iv) In [ XeF3 ]− 3 3
••
N N

67. Cl  Be  Cl N ≡≡ N → N ••
↑ ↑ •• NH2
sp sp
•• + Each nitrogen atom has one pair of lone pair. Thus, in all six
N≡≡ N → O•• O == N == O lone pairs.
↑ •• ↑
sp sp 70. F
F F
All the above mentioned molecules/ions have sp-hybridised
central atom and no one pair at central atom, hence linear also. Br
Others are :
O S [Cl  I  Cl] F F
O O Cl Cl Lone pair would push the Br—F bond pairs in upward direction
sp2-bent sp3-V-shaped sp3d-linear and all Br—F bond angles will contract.
[I  I  I]– F  Xe  F
71. No, (i) NMe3 is pyramidal while (ii) N(SiMe3 )3 is planar. In the
sp3d-linear sp3d-linear latter case, pπ - dπ back bonding between N and Si makes N

[Although ICl 2 , I–3 and XeF2 all also are linear but in them sp2-hybridised.
d-orbital contribute in hybridisation.] 72. Bond order : O2− = 1.5, O2 = 2, O2+ = 2.5
68. PLAN This problem includes concept of hybridisation using VBT, Bond length : O+2 < O2 < O−2
VSEPR theory, etc.,
73.
XeF4 , BrF4− , [Cu(NH3 )4 ]2+ , [PtCl 4 ]2− are square planar as shown F F
below: F F

s Xe O S
F F F F
Xe Br F
F
F F F F
F F
Square planar Trigonal bipyramidal
Chemical Bonding 63

74. F (d) A - D has shortest bond length, it is incorrect because inter


Cl Cl
F F molecular distance in between A - D more than 150 (pm)
which is height in all.
Cl P Br 2. The change of O2 to O−2 can be as follows:
+e – –
F F O2 (16 e –) O2 (17 e–)
Cl Cl [Dioxygen] [Super-oxide]
Energy Energy
Trigonal bipyramidal Square pyramidal
(P is sp3d-hybridised) (Br is sp3d 2 -hybridised) [8e– ] σ2p2z π2px2 [8e– ] σ2p2z π2p2x = π2p2y π* 2px2
75. = π2py2 π* 2px1 = π* 2p1y = π* 2p1y
F F
F F O Half-filled
anti-bonding
π*-MOs
Xe Xe Xe
So, in the formation of O−2 from O2, the 17th electron goes to the
F F * *
O π 2px or π 2py molecular orbital (anti-bonding) which is
F F
Linear Square planar See-saw shaped half-filled in O2.
3 3. HF has highest boiling point among hydrogen halides because it
76. In H2S, S is sp -hybridised with two lone pairs of electrons on it
has strongest hydrogen bonding. Here, the hydrogen bond exists
giving V-shaped (water like) shape. In PCl 3 , P is sp3-hybridised between hydrogen of one molecule and fluorine atom of another
with one lone pair of electrons on it. Therefore, PCl 3 is molecule as shown below.
pyramidal in shape. δ+ δ− δ+ δ− δ+ δ−
+2
…H F…H F…H F
×× •• ×× O In this molecule, hydrogen bond behaves like a bridge between
× × −1
77. × F ו O •× F × F F −1 two atoms that holds one atom by covalent bond and the other by
×× •• ×× V-shaped hydrogen bond.

Electron pair = P = 2 + 2 = 4 4. Key Idea Magnetic nature can be detected by molecular orbital
Hybridisation = sp3 theory. Presence of unpaired electrons means paramagnetic and
absence of unpaired electrons means diamagnetic in nature.
•• •• 2− •• ••
•  •  • • Among the given options, CO is a diamagnetic molecule. It can
78. (i) O2−
2 :  • O •• O •  and • Cl •• Cl • (Cl 2 )
•• ••
  •• •• be proved by molecular orbital (MO) theory. The electronic
configuration of given diatomic molecules are given below.
2−
 • •  CO (Number of electrons = 14)
•O•
l

 •• 
 ••  Electronic configuration = σ1s2 , σ* 1s2 , σ 2s2 , σ* 2s2,
2−
(ii) CO3 :  C  and F • • B • • F (BF3 ) σ2 pz2 , π 2 px2 ≈ π 2 p2y
 • • • •  •

• O • • O • F Since, there is no unpaired electron in the CO molecule, so it
 • • • • • • • •  is diamagnetic.
 ••
• • − •• •• l
NO (Number of electrons = 15)
(iii) CN − :  •• C • • N  and C •• •• O •• (CO) Electronic configuration = σ1s2 , σ* 1s2 , σ 2s2,
••
 

σ* 2s2 , σ 2 pz2 , π 2 px2 ≈ π 2 p2y, π * 2 px1 ≈ π * 2 p0y
 ••  ••
•• ••
(iv) NCS− :  •• S ••C •• N and •• Cl • • C • • N (ClCN) Since, NO has one unpaired electron in π * 2 px1 orbital, so it is
•• ••
 ••  • •
paramagnetic.
l
B2 (Number of electrons = 10)
Topic 3 Resonance, LCAO, MOT, Electronic configuration = σ1s2 , σ* 1s2 , σ2s2, σ* 2s2,
Other Bonding Types π 2 px1 ≈ πp1y
1. (a) A - B has stiffest bond.
Since, two unpaired electrons are present in π 2 px1 and π 2 p1y
This is correct because the potential energy of A - B has
maximum negative value [between −500 to −600]. More orbital. So, it is paramagnetic.
negative is the potential energy means high energy is l
O2 (Number of electrons = 16)
released. Higher the released energy higher is the stability
Electronic configuration = σ1s2 , σ* 1s2 , σ 2s2 , σ* 2s2,
and hence the stiffness bond.
(b) D is more electronegative than other atoms, it is incorrect. σ 2 pz2 , π 2 px2 ≈ π 2 p2y, π * 2 px1 ≈ π * 2 p1y
(c) A - A has the largest bond enthalpy, it is incorrect because Since, two unpaired electrons are present in π * 2 px1 and π * 2 p1y
value of potential energy is more negative in A - B.
orbital. So, it is also paramagnetic.
64 Chemical Bonding

5. C2 will be stabilised after forming anion. The electronic configuration of carbon is1s2 2s2 2 p2. There are twelve electrons inC2. After forming
anion (i.e. C–2 ), the electronic configuration is
l
C−2 : (σ1s) 2 (σ *1s) 2 (σ 2 s) 2 (σ * 2 s) 2 ( π 2 px2 = π 2 p 2y ) (σ2 pz1 ) or KK (σ 2 s) 2 (σ * 2 s) 2 ( π 2 px2 = π 2 p 2y ). σ 2 pz1
1 1
Bond order = (N b − N a ) = (9 − 4 ) = 2.5
2 2
For other options such as F2− , O2− , NO− , the electronic configurations are as follows :
l
F2− : (σ1s)2 (σ* 1s)2 (σ 2s)2 (σ* 2s)2 (σ 2 pz )2 (π 2 px2 = π 2 py2 ) (π * 2 px2 = π * 2 p2y )(σ* 2 pz1 )
Bond order = 1 / 2(N b − N a ) = 1 / 2(10 − 9) = 0.5
l
O−2 : (σ1s)2. (σ* 1s)2 (σ 2s)2 (σ* 2s)2 (σ 2 pz )2(π 2 px2 = π 2 p2y ) (π * 2 px2 = π * 2 p1y )
1 1
Bond order = (N b − N a ) = (10 − 7) = 15 .
2 2

l
NO : (σ1s) (σ 1s) (σ 2s) (σ * 2s) (σ 2 pz )2 (π 2 px2 = π 2 p2y ) (π * 2 px1 = π * 2 p1y )
2 * 2 2 2

1 1
Bond order = (N b − N a ) = (10 − 6) = 2
2 2
The value of bond order of C−2 is highest among the given options. Bond order between two atoms in a molecule may be taken as an
approximate measure of the bond length.
The bond length decreases as bond order increases. As a result, stability of a molecule increases.

6. Species MO energy order Bond order (BO) n, number of unpaired e− Magnetic character
C22− (14e− ) [ 8e ] π = π σ 6−0 0 Diamagnetic
2p x2 2p y2 2p z2 =3
2
O2 (16e− ) [ 8 e ]σ π = π π* = π* 6−2 2 Paramagnetic
2p z2 2p x2 2p y2 2p1x 2p1y =2
2
O22− (18e− ) [ 8 e ]σ π = π π* *
= π 6−4 0 Diamagnetic
2p z2 2p x2 2p y2 2p x2 2p y2 =1
2
N22− (16e− ) 6−2 2 Paramagnetic
[ 8e ] π = π σ *
π = *π 2p1y =2
2p x2 2p y2 2p z2 2p1x 2

1 Nb − Na 3 − 2 1
Bond length ∝ . So order of bond length Bond order (BO) = = =
BO (Bond order) 2 2 2
C22− < O2 = N22− < O22− The electronic configuration of
(BO = 3) (BO = 2) (BO = 1) Li−2 (Z = 7) = σ1s2 , σ* 1s2 , σ2 s2 , σ ∗ 2 s1
The diamagnetic species with shortest bond length is C2−
2 N − Na 4 − 3 1
Bond order (BO) = b = =
(option-a). 2 2 2
7. The energy order of MOs of the given species are as follows: For the species having the same value of BO, the specie having
lesser number of antibonding electrons[ N a ] will be more stable.
O2 (16 e− ’s) = σ1s2 , σ* 1s2 , σ 2s2 , σ* 2s2 , σ 2 p2z ,
Here, N a of Li+2 (2) < N a of Li −2 (3) . So, their order of stability
π 2 p2x = π 2 p2y , π * 2 px1 = π * 2 p1y , will be Li+2 > Li2− .
O+2 (15e− ’s ) = σ1s2, σ* 1s2 , σ2s2, σ* 2s2 , σ2 p2z , 9. Key Idea According to M.O.T, the viability of any molecule can
be judged through the calculation of bond order.
π 2 p2x = π 2 p2y , π * 2 px1 ≈π *
2 p0y
Electronic Bond order
N2 (14 e− ’s ) = σ 1s2 σ* 1s2 , σ 2s2 , σ* 2s2 Configuration
π 2 p2x = π 2 p2y, σ 2 pz2 He+2 σ 2 σ* 1 2−1
1s 1s = 0.5
2
N+2 (13e− ’s) = σ1s σ 1s σ2 s σ* 2s2
2 * 2 2

H−2 σ σ* 2−1
π 2 p2x = π 2 pz2 σ 2 pz1 1s 2 1s1 = 0.5
2
Thus, in case of N+2 , two π-bonds and half σ-bond are present in 2−2
H2− σ σ* =0
2 1s 2 1s 2
the bonding MOs. 2
8. Considering molecular orbital theory (MOT) : He2+ σ 2−0
2 1s 2 =1
The electronic configuration of Li+2 (Z = 5) = σ1s , σ 1s , σ 2s 2 * 2 1
2
Chemical Bonding 65

The molecule having zero bond order will not be viable hence, Stability order is Li −2 < Li 2+ < Li 2 (because Li −2 has more number
H2−
2 (option d) is the correct answer. of electrons in antibonding orbitals which destabilises the
10. To identify the magnetic nature we need to check the molecular species).
orbital configuration. If all orbitals are fully occupied, species is 13. Species having zero or negative bond order do not exist.
diamagnetic while when one or more molecular orbitals is/are
H2+
2 (1 + 1 − 2 = 0) = σ1s
0
singly occupied, species is paramagnetic.
(a) NO (7 + 8 = 15) − σ1s2 , σ* 1s2 , σ 2s2 , σ* 2s2 , Bond order = 0
He2 (2 + 2 = 4 ) = σ1s2 , σ* 1s2
π 2 px2 = π 2 p2y , π 2 pz2 , π * 2 px1 = π * 2 p0y
N − Na 2 − 2
One unpaired electron is present. Hence, it is paramagnetic. Bond order = b = =0
2 2
(b) CO (6 + 8 = 14 ) − σ1s2 , σ* 1s2 , σ 2s2 , σ* 2s2 ,
So, both H2+
2 and He2 do not exist.
π 2 px2 = π 2 p2y, σ 2 pz2
H H
No unpaired electron is present. Hence, it is diamagnetic.   −
(c) O2 (8 + 8 = 16) − σ1s2 , σ* 1s2 , σ 2s2 , σ* 2s2 , σ 2 pz2 , 14. H  C  CH == CH2 ←→ H  C == CH  CH2
 +
π 2 px2 = π 2 px2 , π * 2 px1 = π * 2 p1y H H
Two unpaired electrons are present. (I) (II)
Hence, it is paramagnetic. I and II are hyperconjugation structures of propene and involves
(d) B2(5 + 5) − σ1s2 , σ* 1s2 , σ 2s2 , σ* 2s2 , π 2 px1 = π 2 p1y σ-electrons of C—H bond and p-orbitals of pi bond in
delocalisation.
Two unpaired electrons are present.
Hence, it is paramagnetic. π 2 p2y π* 2 p1y
15. O+2 (15e− ) : σ1s2 σ* 1s2 σ 2s2 σ* 2s2 σ 2 px2 σ
*
2 px0
11. PLAN This problem can be solved by using the concept involved in π 2 pz2 π* 2 p0
molecular orbital theory. Write the molecular orbital electronic 10 − 5 z
configuration keeping in mind that there is no 2s-2p mixing, Bond order = = 2.5; paramagnetic.
then if highest occupied molecular orbital contain unpaired 2
electron then molecule is paramagnetic otherwise diamagnetic. −
π 2 p2y π* 2 p1y *
O2 (16e ) : σ1s σ 1s2 σ 2s2 σ
2* *
2s2 σ 2 px2 σ 2 px
Assuming that no 2s-2p mixing takes place the molecular orbital π 2 pz2 π* 2 p1
electronic configuration can be written in the following sequence z
10 − 6
of energy levels of molecular orbitals Bond order = =2
* 1s, σ 2s, σ
* 2s, σ 2 p , π 2 p ≡ π 2 p , π* 2 p ≡ π* 2 p , σ
* 2p 2
σ1s, σ z x y x y z Hence, (a) is the correct answer.
* 1s2 , σ 2s2 , σ
(a) Be2 → σ1s , σ 2 * 2s2 (diamagnetic)
F F F F F
* 1s2 , σ 2s2 , σ
2 * 2s2 , σ 2 p2 , π 2 px0
(b) B2 → σ1s , σ z (diamagnetic)
π 2 py0 S Xe
16. C
F
* 2s2 , σ 2 p2 , π 2 px ,
1

(c) C2 → σ1s2 , σ * 1s2 , σ 2s2 , σ F F F F


π 2 p1y
z
F F
π* 2 px0 * 0 See-saw shape Tetrahedral Square planar
, σ 2 p (paramagnetic)
z molecule
π* 2 py0
17. When E = Bin BCl 3 , bond angle is 120°. When E = P, As or Bi in
* 2s2 , σ 2 p2 , π 2 px ,
2
(d) N2 → σ1s2 , σ * 1s2 , σ 2s2 , σ
z
π 2 py2 ECl 3 , hybridisation at E will be sp3. Also, if central atoms are
from same group, bond angle decreases down the group provided
π* 2 px0
, σ * 2 pz0 (diamagnetic) all other things are similar. Hence, the order of bond angles is
*
π 2p 0 BCl 3 > PCl 3 > AsCl 3 > BiCl 3
y

Hence, (c) is the correct choice. 1


18. Bond length ∝
Bond order
12. Li 2 (3 + 3 = 6) = σ1s2 , σ* 1s2 , σ 2s2
1 4
Nb − Na 4 − 2 Bond order : CO2 = 2, CO = 3 , CO23− = 1 + =
Bond order = = =1 3 3
2 2
+ 2 * 2 Therefore, order of bond length is CO23− < CO2 < CO
Li 2 (3 + 3 − 1 = 5) = σ1s , σ 1s , σ 2s1
3−2 1 19. H2O2 H
Bond order = = = 0.5
2 2
O—O
Li −2 (3 + 3 + 1 = 7) = σ1s2 , σ*1s2 , σ 2s2 σ* 2s1 polar bond
H
4−3 1 Non-polar bond
Bond order = = = 0.5
2 2
66 Chemical Bonding

20. HCl does not form hydrogen bond. For formation of hydrogen state, i.e. reduced. Hence, in the above reaction, oxygen
bond, atleast one hydrogen atom must be bonded to one of the ( O−1 / 2 ) is simultaneously oxidised and reduced
three most electronegative atom O , N and F. disproportionated.
21. (B) In acidic medium, CrO2− 2−
4 is converted into Cr2 O7 which
Species Electrons MOEC N B N A BO Magnetic is a dimeric, bridged tetrahedral.
character O– O–
C2−
2
14 σ1s2 , σ * 1s2 , 10 4 3 Diamagnetic
Cr Cr
O O
σ 2s2 , σ * 2s2 , O
O O
π 2 px2 ≈ − π 2 p2y ,
(C) MnO−4 + NO−2 + H+ → Mn 2+ + NO−3
σ 2 pz2
The above is a redox reaction and a product NO−3 has
O2+ 14 As above 10 4 3 Diamagnetic
2 trigonal planar structure.
according to
O2 16 number of 10 6 2 Paramagnetic (D) NO−3 + H2SO4 + Fe2+ → Fe+ + NO
N+2 13 electrons 9 4 2.5 Paramagnetic The above is a redox reaction.
N −2 15 10 5 2.5 Paramagnetic 26. The reaction is,
He+2 3 2 1 0.5 Paramagnetic 3Cl2 + 6 NaOH 5 NaCl + NaClO3 + 3H2O
(X) (Y)
Thus, (a) is correct.
+ AgNO3
(b) Bond order O2+ 2 > O 2 thus,
Bond length of O2+
2 < O 2 thus, incorrect. AgCl
(c) N+2 and N−2 have same bond order thus correct. (white ppt.)
+ –
(d) He+2 with bond order = 0.5 is more stable thus, less energy than Y = NaClO3 (Na and ClO3 )
isolated He atoms. Thus, (d) is incorrect. –
The structure of ClO3 (chlorate ion) is,
22. PLAN This problem can be solved by using concept of H-bonding and
applications of H-bonding. Cl Cl Cl == Cl
O O O O O O O O
23. C2 (6 + 6 = 12) = σ1s , σ* 1s , σ 2s , σ* 2s ,
2 2 2 2
π 2 px2 ≈ π 2 p2y O O O O
Since, all the electrons are paired, it is a diamagnetic species. ‘ClO’ bond order in the hybrid
N2 ( 7 + 7 = 14 ) = σ1s , σ* 1s2, σ 2s2 ,
2
=
Number of bonds between Cl and O
Total number of O (surrounding atoms)
σ* 2s2 , π 2 px2 ≈ π 2 p 2y , σ 2 p 2z
5
It is also a diamagnetic species because of the absence of = = 166
. or 1.67
3
unpaired electrons.
O2 (8 + 8 = 16) or S2 = σ1s2 , σ* 1s2 , σ 2s2 , σ∗ 2s2 , 27. H2 , Li 2 , Be2 , C2 , N2 and F2 are diamagnetic according to
molecular orbital theory.
σ 2 p 2 , π 2 p 2 ≈ π 2 p 2 π* 2 p1 ≈ π* 2 p1
z x y x y
Due to the presence of two unpaired electrons, O2 and S2 both are π 2 p2y π* 2 p1y
28. O2 : σ1s2 σ* 1s2 σ 2s2 σ* 2s2 σ 2 px2
paramagnetic molecules.
π 2 pz2 π* 2 p1
24. Statement I is correct, given structure is one of the resonance z

structure of ozone. 10 − 6
Bond order = = 2, paramagnetic.
+ + 2
O ←→ O
29. Strength of hydrogen bonding in X—H—X depends on
O O− −O O electronegativity as well as size of X . X with higher
Statement II is also correct because oxygen cannot expand its electronegativity and smaller size forms stronger H-bond.
octet. It is also the explanation for the given structure of ozone. Hence, increasing order of strength of H-bond is
25. (A) In the reaction : O−2 → O2 + O22− S < Cl < N < O < F
Oxygen on reactant side is in − 1/ 2 oxidation state. In 30. Resonance in vinyl chloride increases polar character of the
product side, one of the oxygen is in zero oxidation state, molecule.
i.e. oxidised while the other oxygen is in –1 oxidation
5
States of Matter
Topic 1 Gaseous State (a) vmp of H 2 ( 300 K ); vmp of N 2(300 K); vmp of
O2(400 K)
Objective Questions I (Only one correct option) (b) vmp of O2 ( 400 K ); vmp of N 2(300 K); vmp of
1. If the distribution of molecular speeds of a gas is as per the H 2(300 K)
figure shown below, then the ratio of the most probable, the (c) vmp of N 2(300 K); vmp of O2 ( 400 K ); vmp of
average, and the root mean square speeds, respectively, is H 2(300 K)
(2020 Adv.)
(d) vmp of N 2(300 K); vmp of H 2(300 K); vmp of
O2(400 K)
4. Consider the following table.
Fraction of
molecules

Gas a/(k Pa dm6mol −1) b/(dm 3mol −1 )


A 642.32 0.05196
B 155.21 0.04136
C 431.91 0.05196
D 155.21 0.4382
Speed
a and b are van der Waals’ constants. The correct
(a) 1 : 1 : 1 (b) 1 : 1 : 1.224
statement about the gases is (2019 Main, 10 April I)
(c) 1 : 1.128 : 1.224 (d) 1 : 1.128 : 1
(a) gas C will occupy lesser volume than gas A; gas B
2. For one mole of an ideal gas, which of these statements must be will be lesser compressible than gas D
true? (2020 Main, 4 Sep I) (b) gas C will occupy more volume than gas A; gas B
(A) U and H each depends only on temperature. will be more compressible than gas D
(B) Compressibility factor Z is not equal to 1. (c) gas C will occupy more volume than gas A; gas B
(C) C p,m − CV,m = R will be lesser compressible than gas D
(D) dU = CV dT for any process. (d) gas C will occupy more volume than gas A; gas B
(a) (B), (C) and (D) (b) (A) and (C) will be lesser compressible than gas D
(c) (A), (C) and (D) (d) (C) and (D) 5. At a given temperature T, gases Ne, Ar, Xe and Kr are
3. Points I, II and III in the following plot respectively correspond found to deviate from ideal gas behaviour. Their equation
to (vmp : most probable velocity) RT
(2019 Main, 10 April II) of state is given as, p = at T.
V −b
Here, b is the van der Waals’ constant. Which gas will
Distribution function f( v)

exhibit steepest increase in the plot of Z (compression


factor) vs p? (2019 Main, 9 April II)
(a) Xe (b) Ar (c) Kr (d) Ne
6. Consider the van der Waals’ constants, a and b, for the
following gases.
Gas Ar Ne Kr Xe
6 −2
a/(atm dm mol ) 1.3 0.2 5.1 4.1
I II III
−2 3 −1
Speed, v b/(10 dm mol ) 3.2 1.7 1.0 5.0
68 States of Matter

Which gas is expected to have the highest critical figure below. The temperature of one of the bulbs is then
temperature ? (2019 Main, 9 April I) raised to T2 . The final pressure p f is (2016 Main)
(a) Kr (b) Xe (c) Ar (d) Ne
 T1   T 
7. The combination of plots which does not represent isothermal (a) 2 pi   (b) 2 pi  2 
 T1 + T2   T1 +T2 
expansion of an ideal gas is
(2019 Main, 12 Jan II)  TT   TT 
(c) 2 pi  1 2  (d) pi  1 2 
 T1 +T2   T1 +T2 

p p
13. If Z is a compressibility factor, van der Waals’ equation at
low pressure can be written as (2014 Main)
RT a
(a) Z = 1 + (b) Z = 1 −
O 1/Vm O Vm pb VRT
(A ) (B) pb pb
(c) Z = 1 − (d) Z = 1 +
RT RT
pVm 14. For gaseous state, if most probable speed is denoted by C *,
U average speed by C and root square speed by C, then for a
large number of molecules, the ratios of these speeds are
O O (a) C * : C : C = 1.225 : 1.128 : 1 (2013 Main)
p Vm
(C) (D) (b) C * : C : C = 1.128 : 1.225 : 1
(c) C * : C : C = 1 : 1.128 : 1.225
(a) ( A ) and (C ) (b) ( B ) and (C )
(c) ( B ) and ( D ) (d) ( A ) and ( D ) (d) C * : C : C = 1 : 1.225 : 1.128

8. An open vessel at 27ºC is heated until two fifth of the air 15. For one mole of a van der Waals’ gas when b = 0 and
(assumed as an ideal gas) in it has escaped from the vessel. T = 300 K, the pV vs 1/V plot is shown below. The value of
Assuming that the volume of the vessel remains constant, the the van der Waals’ constant a (atm L mol − 2 ) (2012)
temperature at which the vessel has been heated is
(2019 Main, 12 Jan II) 24.6
(a) 750 K (b) 500 K
pV (L atm mol–1)

23.1
(c) 750ºC (d) 500ºC
21.6
9. The volume of gas A is twice than that of gas B. The
20.1
compressibility factor of gas A is thrice than that of gas B at
same temperature. The pressures of the gases for equal
number of moles are (2019 Main, 12 Jan I)
(a) p A = 2 pB (b) 2 p A = 3 pB
(c) p A = 3 pB (d) 3 p A = 2 pB 0 2.0 3.0
1/ V (mol L–1)
10. A 10 mg effervescent tablet containing sodium bicarbonate
and oxalic acid releases 0.25 mL of CO2 at T = 29815
. K and (a) 1.0 (b) 4.5 (c) 1.5 (d) 3.0
p = 1 bar. If molar volume of CO2 is 25.0 L under such 16. The term that corrects for the attractive forces present in a
condition, what is the percentage of sodium bicarbonate in real gas in the van der Waals’ equation is (2009)
each tablet? (a) nb (b) n 2 a / V 2
[Molar mass of NaHCO 3 = 84 g mol −1 ] (2019 Main, 11 Jan I) 2 2
(c) − ( n a / V ) (d) − nb
(a) 8.4 (b) 0.84 (c) 16.8 (d) 33.6 17. The given graph represent the variations of Z
11. 0.5 moles of gas A and x moles of gas B exert a pressure of 200 (compressibility factor ( Z ) = pV / nRT ) ) versus p, for three
3
Pa in a container of volume 10m at 1000 K. Given R is the real gases A, B and C. Identify the only incorrect statement.
(2006, 5M)
gas constant in JK −1 mol−1 , x is (2019 Main, 9 Jan I) C A
2R 4 −R 4 +R 2R A Ideal gas
(a) (b) (c) (d) B
4−R 2R 2R 4+R C
1
12. Two closed bulbs of equal volume (V) containing an ideal gas B
Z
initially at pressure pi and temperature T1 are connected
through a narrow tube of negligible volume as shown in the 0 p (atm)
States of Matter 69

(a) For the gas A, a = 0 and its dependence on p is linear at all 25. A gas will approach ideal behaviour at (1999, 2M)
pressure (a) low temperature and low pressure
(b) For the gas B, b = 0 and its dependence on p is linear at all (b) low temperature and high pressure
pressure (c) high temperature and low pressure
(c) For the gas C, which is typical real gas for which neither a (d) high temperature and high pressure
nor b = 0 . By knowing the minima and the point of
intersection, with Z = 1, a and b can be calculated
26. According to Graham’s law, at a given temperature the ratio
rA
(d) At high pressure, the slope is positive for all real gases of the rates of diffusion of gases A and B is given by
rB
18. If helium and methane are allowed to diffuse out of the
(where, p and M are pressures and molecular weights of gases
container under the similar conditions of temperature and
A and B respectively)
pressure, then the ratio of rate of diffusion of helium to 1 1
(1998, 2M)
methane is (2005)  p   M 2  M   p 2
(a)  A   A  (b)  A   A 
(a) 2.0 (b) 1.0  pB   M B   M B   pB 
(c) 0.5 (d) 4.0 1 1
 p   M 2  M   p 2
19. For a monatomic gas kinetic energy = E. The relation with (c)  A   B  (d)  A   B 
 pB   M A   M B   pA 
rms velocity is (2004, 1M)

 2E 
1/ 2
 3E 
1/ 2 27. The compressibility factor for an ideal gas is (1997, 1M)
(a) u =   (b) u =   (a) 1.5 (b) 1.0 (c) 2.0 (d) ∞
 m  2m
1/ 2 1/ 2 28. The ratio between the root mean square speed of H2 at 50 K
 E  E
(c) u =   (d) u =   and that of O2 at 800 K is (1996, 1M)
 2m  3m 1
(a) 4 (b) 2 (c) 1 (d)
20. Positive deviation from ideal behaviour takes place because 4
of (2003, 1M) 29. Equal weights of ethane and hydrogen are mixed in an empty
(a) molecular interaction between atom and pV / nRT > 1 container at 25° C. The fraction of the total pressure exerted
(b) molecular interaction between atom and pV / nRT < 1 by hydrogen is (1993, 1M)
(c) finite size of atoms and pV / nRT > 1 (a) 1 : 2 (b) 1 : 1 (c) 1 : 16 (d) 15 : 16
(d) finite size of atoms and pV / nRT < 1 30. At constant volume, for a fixed number of moles of a gas the
21. Which of the following volume (V ) temperature (T ) plots pressure of the gas increases with rise of temperature due to
represents the behaviour of one mole of an ideal gas at the (a) increase in average molecular speed (1992, 1M)
atmospheric pressure? (2002, 3M) (b) increase rate of collisions amongst molecules
(a) V(L) (38.8 L, (b) V(L) (c) increase in molecular attraction
373 K) (28.6 L, (d) decrease in mean free path
373 K)
(22.4 L, (20.4 L,
273 K) 273 K) 31. According to kinetic theory of gases, for a diatomic molecule
(1991, 1M)
T(K) T(K) (a) the pressure exerted by the gas is proportional to mean
(c) V(L) (d) V(L) velocity of the molecule
(30.6 L,
373 K) (b) the pressure exerted by the gas is proportional to the root
(22.4 L, (22.4 L,
273 K) (14.2 L,
mean velocity of the molecule
273 K)
373 K) (c) the root mean square velocity of the molecule is inversely
proportional to the temperature
T(K) T(K)
(d) the mean translational kinetic energy of the molecule is
22. The root mean square velocity of an ideal gas at constant proportional to the absolute temperature
pressure varies with density (d ) as (2001, S, 1M) 32. The rate of diffusion of methane at a given temperature is
(a) d 2 (b) d (c) d (d) 1 / d twice that of a gas X. The molecular weight of X is
(1990, 1M)
23. The compressibility of a gas is less than unity at STP.
Therefore, (a) 64.0 (b) 32.0 (c) 4.0 (d) 8.0
(a) Vm > 22.4 L (b) Vm < 22.4 L (2000, 1M) 33. The density of neon will be highest at (1990, 1M)
(c) Vm = 22.4 L (d) Vm = 44.8 L (a) STP (b) 0° C, 2 atm
24. The rms velocity of hydrogen is 7 times the rms velocity of (c) 273° C, 1 atm (d) 273° C, 2 atm
nitrogen. If T is the temperature of the gas (2000, 1M) 34. The value of van der Waals’ constant a for the gases O2 , N2 ,
(a) T (H2 ) = T (N2 ) (b) T (H2 ) > T (N2 ) NH3 and CH4 are 1.360, 1.390, 4.170 and 2.253L2 atm mol −2
(c) T (H2 ) < T (N2 ) (d) T (H2 ) = 7 T (N2 ) respectively. The gas which can most easily be liquefied is
(a) O2 (b) N2 (c) NH3 (d) CH4
70 States of Matter

35. A bottle of dry ammonia and a bottle of dry hydrogen Objective Questions II
chloride connected through a long tube are opened
simultaneously at both ends the white ammonium chloride
(One or more than one correct option)
ring first formed will be (1988, 1M) 45. Which of the following statement(s) is(are) correct regarding
(a) at the centre of the tube the root mean square speed (U rms ) and average translational
(b) near the hydrogen chloride bottle kinetic energy ( Eav ) of a molecule in a gas at equilibrium?
(2019 Adv.)
(c) near the ammonia bottle
(a) Urms is inversely proportional to the square root of its
(d) throughout the length of the tube molecular mass
36. In van der Waals’ equation of state for a non-ideal gas, the (b) Urms is doubled when its temperature is increased four times
term that accounts for intermolecular forces is (1988, 1M) (c) Eav is doubled when its temperature is increased four times
 a
(a) (V − b ) (b) RT (c)  p + 2  (d) ( RT )−1 (d) Eav at a given temperature does not depend on its
 V  molecular mass
37. The average velocity of an ideal gas molecule at 27° C is 46. One mole of a monoatomic real gas satisfies the equation
0.3 m/s. The average velocity at 927° C will be (1986, 1M) p(V − b)= RT where, b is a constant. The relationship of
(a) 0.6 m/s (b) 0.3 m/s (c) 0.9 m/s (d) 3.0 m/s interatomic potential V(r) and interatomic distance r for gas is
given by (2015 Adv.)
38. Rate of diffusion of a gas is (1985, 1M)
(a) directly proportional to its density V(r) V(r)
(b) directly proportional to its molecular weight (a) 0 (b) 0
r r
(c) directly proportional to the square root of its molecular weight
(d) inversely proportional to the square root of its
molecular weight V(r) V(r)
39. Equal weights of methane and hydrogen are mixed in an (c) 0 (d) 0
empty container at 25° C. The fraction of the total pressure r r
exerted by hydrogen is (1984, 1M)
1 8 1 16 47. According to kinetic theory of gases
(a) (b) (c) (d) (2011)
2 9 9 17 (a) collisions are always elastic
40. When an ideal gas undergoes unrestrained expansion, no (b) heavier molecules transfer more momentum to the wall
cooling occurs because the molecules (1984, 1M) of the container
(a) are above the inversion temperature (c) only a small number of molecules have very high velocity
(b) exert no attractive forces on each other (d) between collisions, the molecules move in straight lines
(c) do work equal to loss in kinetic energy with constant velocities
(d) collide without loss of energy 48. A gas described by van der Waals’ equation (2008, 4M)
41. Helium atom is two times heavier than a hydrogen molecule. (a) behaves similar to an ideal gas in the limit of large molar
At 298 K, the average kinetic energy of a helium atom is volumes
(a) two times that of a hydrogen molecule (1982, 1M) (b) behaves similar to an ideal gas in the limit of large pressures
(b) same as that of a hydrogen molecule (c) is characterised by van der Waals’ coefficients that are
(c) four times that of a hydrogen molecule dependent on the identity of the gas but are independent
(d) half that of a hydrogen molecule of the temperature
(d) has the pressure that is lower than the pressure exerted
42. Equal weights of methane and oxygen are mixed in an empty by the same gas behaving ideally
container at 25°C. The fraction of the total pressure exerted
49. If a gas is expanded at constant temperature (1986, 1M)
by oxygen is (1981, 1M)
1 1 2 1 273 (a) the pressure decreases
(a) (b) (c) (d) × (b) the kinetic energy of the molecules remains the same
3 2 3 3 298 (c) the kinetic energy of the molecules decreases
43. The temperature at which a real gas obeys the ideal gas laws (d) the number of molecules of the gas increases
over a wide range of pressure is (1981, 1M)
(a) critical temperature (b) Boyle temperature Numerical Answer Type Questions
(c) inversion temperature (d) reduced temperature 50. A spherical balloon of radius 3 cm containing helium gas has a
44. The ratio of root mean square velocity to average velocity of pressure of 48 × 10−3 bar. At the same temperature, the
a gas molecule at a particular temperature is (1981, 1M) pressure, of a spherical balloon of radius 12 cm containing the
(a) 1.085 : 1 (b) 1 : 1.086 (c) 2 : 1.086 (d) 1.086 : 2 same amount of gas will be ............ × 10−6 bar.
(2020 Main, 6 Sep I)
States of Matter 71

51. A closed tank has two compartments A and B, both filled with 54. The experimental value of d is found to be smaller than the
oxygen (assumed to be ideal gas). The partition separating the estimate obtained using Graham’s law. This is due to
two compartments is fixed and is a perfect heat insulator (Fig. (a) larger mean free path for X as a compared of that of Y
1). If the old partition is replaced by a new partition which can (b) larger mean free path for Y as compared to that of X
slide and conduct heat but does not allow the gas to leak (c) increased collision frequency of Y with the inert gas as
across (Fig. 2), the volume (in m3 ) of the compartment A after compared to that of X with the inert gas
the system attains equilibrium is ____. (d) increased collision frequency of X with the inert gas as
compared to that of Y with the inert gas
1 m3, 5 bar
3 m3, 1 bar, 300 K
55. The value of d in cm (shown in the figure), as estimated from
400 K
B Graham’s law, is
A
(a) 8 (b) 12 (c) 16 (d) 20
Fig. 1 Match the Column
56. Match the gases under specified conditions listed in
A B Column I with their properties/laws in Column II.
Column I Column II
Fig. 2 (2018 Adv.) A. Hydrogen gas ( p = 200 atm, p. compressibility
T = 273 K) factor ≠ 1
Assertion and Reason
B. Hydrogen gas ( p ~ 0, T = 273 K) q. attractive forces
Read the following questions and answer as per the direction given are dominant
below : C. CO2 ( p = 1 atm, T = 273 K) r. pV = nRT
(a) Statement I is correct; Statement II is correct; Statement
II is the correct explanation of Statement I D. Real gas with very large molar s. p (V − nb ) = nRT
volume
(b) Statement I is correct; Statement II is correct; Statement
II is not the correct explanation of Statement I (2007, 6M)

(c) Statement I is correct; Statement II is incorrect Fill in the Blanks


(d) Statement I is incorrect; Statement II is correct 57. The absolute temperature of an ideal gas is …… to/than the
52. Statement I The pressure of a fixed amount of an ideal gas is average kinetic energy of the gas molecules. (1997, 1M)
proportional to its temperature. 58. 8 g each of oxygen and hydrogen at 27°C will have the total
Statement II Frequency of collisions and their impact both kinetic energy in the ratio of …… . (1989, 1M)

increase in proportion to the square root of temperature. 59. The value of pV for 5.6 L of an ideal gas is ......... RT, at NTP.
(2000) (1987, 1M)

53. Statement I The value of van der Waals’ constant ‘a’ is 60. The rate of diffusion of a gas is .......... proportional to both
larger for ammonia than for nitrogen. .......... and square root of molecular mass. (1986, 1M)
61. C p − CV for an ideal gas is ……
Statement II Hydrogen bonding is present in ammonia. (1984, 1M)
(1998) 62. The total energy of one mole of an ideal monoatomic gas at
27°C is …… cal. (1984, 1M)
Passage Based Questions
X and Y are two L = 24 cm
True / False
volatile liquids with 63. A mixture of ideal gases is cooled up to liquid helium
molar weights of temperature (4.22 K) to form an ideal solution. (1996, 1M)
10 g mol −1 and 40 g  n2a
64. In the van der Waals’ equation,  p + 2  (V − nb ) = nRT
mol −1 respectively. Cotton wool d Initial formation Cotton wool  V 
of the product soaked in Y
Two cotton plugs, one soaked in X the constant ‘a’ reflects the actual volume of the gas
soaked in X and the molecules. (1993, 1M)
other soaked in Y, are simultaneously placed at the ends of a tube of 65. A gas in a closed container will exert much higher pressure
length L = 24 cm, as shown in the figure. due to gravity at the bottom than at the top. (1985, 1/2 M)
The tube is filled with an inert gas at 1 atm pressure and a 66. Kinetic energy of a molecule is zero at 0°C. (1985, 1/2 M)
temperature of 300 K. Vapours of X and Y react to form a product
which is first observed at a distance d cm from the plug soaked in X. Integer Answer Type Questions
Take X and Y to have equal molecular diameters and assume ideal
behaviour for the inert gas and the two vapours. (2014 Adv.)
67. The diffusion coefficient of an ideal gas is proportional to its
mean free path and mean speed. The absolute temperature of
72 States of Matter

an ideal gas is increased 4 times and its pressure is increased 2 77. Using van der Waals’ equation, calculate the constant a when
times. As a result, the diffusion coefficient of this gas two moles of a gas confined in a four litre flask exert a
increases x times. The value of x is ... (2016 Adv.) pressure of 11.0 atm at a temperature of 300 K. The value of b
68. A closed vessel with rigid walls contains 1 mole of 238
92 Uand 1
is 0.05 L mol –1 . (1998, 4M)

mole of air at 298 K. Considering complete decay of 238 92 U to


78. An evacuated glass vessel weighs 50.0 g when empty 148.0 g
206
Pb, the ratio of the final pressure to the initial pressure of when filled with a liquid of density 0.98 g mL–1 and 50.5 g
82
the system at 298 K is (2015 Adv.)
when filled with an ideal gas at 760 mm Hg at 300 K.
Determine the molar mass of the gas. (1998, 3M)
69. If the value of Avogadro number is 6.023 × 10 mol −1 and
23
79. A mixture of ideal gases is cooled up to liquid helium
the value of Boltzmann constant is 1.380 × 10−23 JK −1 , then
temperature (4.22 K) to form an ideal solution. Is this
the number of significant digits in the calculated value of the
universal gas constant is (2014 Adv.)
statement true or false ? Justify your answer in not more than
two lines. (1996, 1M)
70. To an evacuated vessel with movable piston under external
pressure of 1 atm, 0.1 mole of He and 1.0 mole of an unknown 80. The composition of the equilibrium mixture (Cl 2 s 2Cl)
compound (vapour pressure 0.68 atm at 0° C ) are introduced. which is attained at 1200° C, is determined by measuring the
Considering the ideal gas behaviour, the total volume (in rate of effusion through a pin-hole. It is observed that at
litre) of the gases at 0° C is close to (2011) 1.80 mm Hg pressure, the mixture effuses 1.16 times as fast
as krypton effuses under the same conditions. Calculate the
Subjective Questions fraction of chlorine molecules dissociated into atoms (atomic
weight of Kr = 84) (1995, 4M)
71. At 400 K, the root mean square (rms) speed of a gas X
(molecular weight = 40) is equal to the most probable speed of 81. A mixture of ethane (C2 H6 ) and ethene (C2 H4 ) occupies 40 L
gas Y at 60 K. The molecular weight of the gas Y is (2009) at 1.00 atm and at 400 K. The mixture reacts completely with
72. The average velocity of gas molecules is 400 m s − 1 , find the 130 g of O2 to produce CO2 and H2 O. Assuming ideal gas
rms velocity of the gas. (2003, 2M) behaviour, calculate the mole fractions of C2 H4 and C2 H6 in
73. The density of the vapour of a substance at 1 atm pressure and the mixture. (1995, 4M)

500 K is 0.36 kg m–3 . The vapour effuses through a small hole 82. An LPG (liquefied petroleum gas) cylinder weighs 14.8 kg
at a rate of 1.33 times faster than oxygen under the same when empty. When full it weighs 29.0 kg and shows a pressure
condition. of 2.5 atm. In the course of use at 27° C, the weight of the full
(i) Determine, (a) molecular weight (b) molar volume cylinder reduces to 23.2 kg. Find out the volume of the gas in
(c) compression factor (Z) of the vapour and (d) which cubic metres used up at the normal usage conditions, and the
forces among the gas molecules are dominating, the final pressure inside the cylinder. Assume LPG to the n-butane
attractive or the repulsive? with normal boiling point of 0° C. (1994, 3M)
(ii) If the vapour behaves ideally at 1000 K, determine the 83. A 4 : 1 molar mixture of He and CH4 is contained in a vessel
average translational kinetic energy of a molecule.
(2002, 5M) at 20 bar pressure. Due to a hole in the vessel, the gas mixture
leaks out. What is the composition of the mixture effusing out
74. The compression factor (compressibility factor) for one mole initially? (1994, 2M)
of a van der Waals’ gas at 0° C and 100 atm pressure is found to
be 0.5. Assuming that the volume of a gas molecule is negligible, 84. A gas bulb of 1 L capacity contains 2.0 × 1021 molecules of
calculate the van der Waals’ constant ‘a’. (2001, 5M) nitrogen exerting a pressure of 7.57 × 103 Nm–2 . Calculate
75. Calculate the pressure exerted by one mole of CO2 gas at the root mean square (rms) speed and the temperature of the
273 K if the van der Waals’ constant a = 3.592 dm 6 atm gas molecules. If the ratio of the most probable speed to root
mol −2 . Assume that the volume occupied by CO2 molecules is mean square speed is 0.82, calculate the most probable speed
negligible. (2000) for these molecules at this temperature. (1993, 4M)

76. (i) One mole of nitrogen gas at 0.8 atm takes 38 s to diffuse 85. At room temperature, the following reaction proceed nearly
through a pin-hole, whereas one mole of an unknown to completion. 2NO + O2 → 2NO2 → N2 O4
compound of xenon with fluorine at 1.6 atm takes 57 s to The dimer, N2 O4 , solidifies at 262 K. A 250 mL flask and a
diffuse through the same hole. Calculate the molecular 100 mL flask are separated by a stopcock. At 300 K, the nitric
formula of the compound. oxide in the larger flask exerts a pressure of 1.053 atm and the
(ii) The pressure exerted by 12 g of an ideal gas at smaller one contains oxygen at 0.789 atm.
temperature t ° C in a vessel of volume V litre is one atm. The gases are mixed by opening the stopcock and after the
When the temperature is increased by 10°C at the same end of the reaction the flasks are cooled to 220 K. Neglecting
volume, the pressure increases by 10%. Calculate the the vapour pressure of the dimer, find out the pressure and
temperature t and volume V. composition of the gas remaining at 220 K. (Assume the
(Molecular weight of the gas = 120) (1999, 5M) gases to behave ideally). (1992, 4M)
States of Matter 73

86. At 27° C, hydrogen is leaked through a tiny hole into a vessel pressure becomes 1.5 atm. Assuming ideal gas behaviour,
for 20 min. Another unknown gas at the same temperature calculate the ratio of the molecular weights M A : M B .
and pressure as that of hydrogen is leaked through same hole (1983, 2M)
for 20 min. After the effusion of the gases the mixture exerts a 94. At room temperature, ammonia gas at 1 atm pressure and
pressure of 6 atm. The hydrogen content of the mixture is 0.7 hydrogen chloride gas at p atm pressure are allowed to effuse
mole. If the volume of the container is 3 L. What is the through identical pin holes from opposite ends of a glass tube
molecular weight of the unknown gas? (1992, 3M) of one metre length and of uniform cross-section.
87. Calculate the volume occupied by 5.0 g of acetylene gas at Ammonium chloride is first formed at a distance of 60 cm
50° C and 740 mm pressure. (1991, 2M) from the end through which HCl gas is sent in. What is the
value of p ? (1982, 4M)
88. The average velocity at T1 K and the most probable at T2 K of
95. Calculate the average kinetic energy, in joule per molecule in
CO2 gas is 9.0 × 104 cm s –1 . Calculate the value of T1 and T2
8.0 g of methane at 27°C. (1982, 2M)
(1990, 4M)
96. The pressure in a bulb dropped from 2000 to 1500 mm of
89. A spherical balloon of 21 cm diameter is to be filled up with mercury in 47 min when the contained oxygen leaked through
hydrogen at NTP from a cylinder containing the gas at 20 atm a small hole. The bulb was then evacuated. A mixture of
at 27° C. If the cylinder can hold 2.82 L of water, calculate the oxygen and another gas of molecular weight 79 in the molar
number of balloons that can be filled up. (1987, 5M) ratio of 1 : 1 at a total pressure of 4000 mm of mercury was
90. Calculate the root mean square velocity of ozone kept in a closed introduced. Find the molar ratio of the two gases remaining in
vessel at 20° C and 82 cm mercury pressure. (1985, 2M) the bulb after a period of 74 min. (1981, 3M)

91. Give reasons for the following in one or two sentences. 97. A hydrocarbon contains 10.5 g of carbon per gram of
(i) Equal volumes of gases contain equal number of moles. hydrogen. 1 L of the vapour of the hydrocarbon at 127°C and
(1984, 1M) 1 atm pressure weighs 2.8 g. Find the molecular formula of
(ii) A bottle of liquor ammonia should be cooled before the hydrocarbon. (1980, 3M)
opening the stopper. (1983, 1M) 98. 3.7 g of a gas at 25°C occupied the same volume as 0.184 g of
92. Oxygen is present in one litre flask at a pressure of 7.6 × 10−10 hydrogen at 17°C and at the same pressure. What is the
molecular weight of the gas ? (1979, 2M)
mm Hg. Calculate the number of oxygen molecules in the
flask at 0° C. (1983, 2M)
99. 4.215 g of a metallic carbonate was heated in a hard glass
tube, the CO2 evolved was found to measure 1336 mL at
93. When 2 g of a gas A is introduced into an evacuated flask kept 27°C and 700 mm of Hg pressure. What is the equivalent
at 25° C, the pressure is found to be one atmosphere. If 3 g of weight of the metal ? (1979, 3M)
another gas B is then added to the same flask, the total
100. Calculate the density of NH3 at 30°C and 5 atm pressure.
(1978, 2M)

Topic 2 Liquid State


Objective Questions I (Only one correct option) 2. At 100°C and 1 atm if the density of the liquid water is
1. The qualitative sketches I, II and III given below show the 1.0 g cm–3 and that of water vapour is 0.0006 g cm−3 , then the
variation of surface tension with molar concentration of three volume occupied by water molecules in 1 L of steam at this
different aqueous solutions of KCl, CH3 OH and temperature is (2000, 1M)
− +
CH3 (CH2 )11 OSO3 Na at room temperature. (2016 Adv.) (a) 6 cm 3 (b) 60 cm 3
(c) 0.6 cm 3 (d) 0.06 cm 3
Surface tension

Surface tension

Surface tension

I II III
3. The critical temperature of water is higher than that of O2
because the H2O molecule has (1997)
(a) fewer electrons than O2
(b) two covalent bonds
Concentration Concentration Concentration (c) V-shape
(d) dipole moment
The correct assignment of the sketches is
I II III 4. A liquid is in equilibrium with its vapour at it’s boiling point.
(a) KCl CH3OH CH3 (CH2 )11 OSO−3 Na + On the average, the molecules in the two phases have equal
(a) inter-molecular forces
(b) CH3 (CH2 )11 OSO3− +
Na CH3OH KCl
(b) potential energy
(c) KCl CH3 (CH2 )11 OSO−3 Na + CH3OH (c) kinetic energy
(d) CH3OH KCl CH3 (CH2 )11 OSO3− Na + (d) total energy
Answers
Topic 1
1. (b) 2. (c) 3. (c) 4. (b) 56. (A → p,s; B → r; C→ p, q; D → r)
5. (a) 6. (a) 7. (c) 8. (b) 57. (less) 58. (1 : 16) 59. (0.25)
9. (b) 10. (a) 11. (b) 12. (b) 60. (inversely, time) 61. (R) 62. (900)
13. (b) 14. (c) 15. (c) 16. (b) 63. (F) 64. (F) 65. (F) 66. (F)
17. (b) 18. (a) 19. (a) 20. (a) 67. (4) 68. (9) 69. (4) 70. (7 L)
21. (c) 22. (d) 23. (b) 24. (c) 71. (4) 72. 435 ms −1 74. (1.25) 75. (0.99 atm)
25. (c) 26. (c) 27. (b) 28. (c) 77. (6.46) 78. (123 g mol −1) 80. (0.14)
29. (d) 30. (a) 31. (d) 32. (a) 82. (2.46 m 3) 83. (8:1) 84. (407 ms −1) 85. (0.221 atm)
86. (1020 g mol −1) 87. (5.23 L) 89. (10)
33. (b) 34. (c) 35. (b) 36. (c)
90. (390.2 ms −1) 92. (2.7 × 1010 g mol −1)
37. (a) 38. (d) 39. (b) 40. (b)
93. (1 : 3) 94. (2.20 atm) 95. (6.2 × 10 −21 J/molecule)
41. (b) 42. (a) 43. (b) 44. (a)
98. (41.32 g) 99. (12.15) 100. (3.42 gL −1)
45. (a, b, d) 46. (c) 47. (a) 48. (a,c)
49. (a,b) 50. (750) 51. (2.22) 52. (d) Topic 2
53. (a) 54. (d) 55. (c) 1. (d) 2. (c) 3. (d) 4. (c)

Hints & Solutions


Topic 1 Gaseous State Heat capacity at constant volume,
1. Fraction of molecules vs velocity graph is Maxwell distribution dH
nCp =
curve. dT
This curve is slightly unsymmetrical as shown below. dH = nC p dT [n = 1mol]
Most probable speed (vp) dH = 1 × C p × dT
In mathematically term,
Average speed (vavg)
Fraction of molecules

H = f (T ) [QC p = constant]
Root-mean-square speed (vrms) Hence, H depends on function of temperature
(b) Compressibility factor (Z ) describe the deviation of real gas
from ideal gas behaviour.
pV
Z= …(i)
RT
For ideal gas,
vp vavg vrms pV = nRT [n = 1mol]
Speed v
pV = RT …(ii)
The ratio of most probable, the average and the root mean square Put in the value of pV from Eq. (ii) to Eq. (i)
speeds for this graph is 1 : 1.128 : 1.224 RT
Z= =1
But the graph in question is completly symmetrical. Therefore, RT
the most probable and the average speed will be same here but Compressibility factor is 1 for ideal gas then option (b) is
root mean square speed will be greater than average speed. So, incorrect.
the correct ratio is 1 : 1: 1224
. .
(c) For ideal gas,
2. Statements (A), (C) and (D) are true whereas (B) is false. C p − CV = nR (n = 1mol)
(A) For 1 mole, U = internal energy and H = enthalpy. C p − CV = R
Heat capacity at constant volume. It is correct statement.
dU
nCV = (d) ∆U = CV dT
dt dU
nCV =
dU = nCV dT (n = 1 mol) dT
dU = 1 × CV × dT (QCV = constant) dU = nCV dT (n = 1mol)
U = f (T ) (Function of temperature) dU = CV dT
U depends on temperature. It is also correct.
States of Matter 75

3. pV
Key Idea From kinetic gas equation, As, Z=
RT
2RT
Most probable velocity (vmp ) = so, Z =1+
pb
⇒ y = c + mx
M RT
where, R = gas constant, T = temperature, The plot of z vs p is found to be
M = molecular mass
b
2RT T slope =
vmp = , i.e. vmp ∝ RT
M M Z

Gas M T ( K) T/M
H2 2 300 300 / 2 = 150 …III (Highest)
p
N2 28 300 300 / 28 = 10.71 …I (Lowest)
The gas with high value of b will be steepest as slope is directly
proportional to b. b is the van der Waals’ constant and is equal to
O2 32 400 400 / 32 = 12.5 … II
four times the actual volume of the gas molecules. Xe gas
So, possess the largest atomic volume among the given noble gases
(Ne, Kr, Ar). Hence, it gives the steepest increase in the plot of Z
I. corresponds to vmp of N2 (300 K)
(compression factor) vsp.
II. corresponds to vmp of O2 (400 K)
III. corresponds to vmp of H2 (300 K) 6. Critical temperature is the temperature of a gas above which it
cannot be liquefied what ever high the pressure may be. The
4. For 1 mole of a real gas, the van der Waals’ equation is kinetic energy of gas molecules above this temperature is
sufficient enough to overcome the attractive forces. It is
 a
 p + 2  (V − b) = RT represented as Tc.
 V  8a
Tc =
The constant ‘a’ measures the intermolecular force of attraction 27Rb
of gas molecules and the constant ‘b’ measures the volume 8 × 13
.
correction by gas molecules after a perfectly inelastic binary For Ar, Tc = = 0.0144
27 × 8.314 × 3.2
collision of gas molecules.
8 × 0.2
For gas A and gas C given value of ‘b’ is For Ne, Tc = = 0.0041
0.05196 dm3 mol −1. Here, 27 × 8.314 × 17
.
8 × 51
.
a ∝ intermolecular force of attraction For Kr, Tc = = 018
.
∝ compressibility ∝ real nature 27 × 8.314 × 1.0
8 × 4.1

1 For Xe, Tc = = 0.02
volume occupied 27 × 8.314 × 5.0
The value of Tc is highest for Kr (Krypton).
Value of a/(kPa dm6 mol −1) for gas A (642.32) > gas C (431.91)
So, gas C will occupy more volume than gas A. Similarly, for a 7. In isothermal expansion, pVm = K (constant)
given value of a say 155.21 kPa dm6 mol −1 for gas B and gas D This relation is plotted in graph ‘C’
1 K
∝ intermolecular force of attraction Likewise, p=
b Vm
∝ compressibility ∝ real nature This relation is plotted in graph “ A”.
1
∝ Thus, graph B and D are incorrect.
volume accupied For them the correct graphs are:
b/(dm 3 mol −1) for gas B (0.04136) < Gas D (0.4382)
So, gas B will be more compressible than gas D.
p for graph B and U for graph D
5. Noble gases such as Ne, Ar, Xe and Kr found to deviate from
ideal gas behaviour.
Xe gas will exhibit steepest increase in plot of Z vs p. Vm Vm
Equation of state is given as:
RT 8. Given, temperature (T1 ) = 27° C = 273 + 27 = 300 K
p= ⇒ p(V − b) = RT
(V − b) Volume of vessel = constant
pV − pb = RT ⇒ pV = RT + pb Pressure in vessel = constant
pV pb 2 3
=1+ Volume of air reduced by so the remaining volume of air is .
RT RT 5 5
76 States of Matter

Let at T1 the volume of air inside the vessel is n so at T2 the pf V


3 After mixing, number of moles in left chamber =
volume of air will be n. RT1
5 pf V
Now, as p and V are constant, so Number of moles in right chamber =
RT2
3
n ⋅T1 = n T2 ...(i) pf V pf V pf V  1 1
5 Total number of moles = + =  + 
Putting the value of T1 in equation (i) we get, RT1 RT2 R  T1 T2 
3 As total number of moles remains constant.
n × 300 = n × T2
5 2 piV pf V pf V  T2 
Hence, = + ⇒ pf = 2 pi  
5 RT1 RT1 RT2  T1 + T2 
or T2 = 300 × = 500 K
3
13. PLAN To solve this problem, the stepwise approach required, i.e.
9. Given, ZA = 3ZB (i) Write the van der Waals’ equation, then apply the condition that
pV at low pressure, volume become high,
Compressibility factor (Z) = [for real gases]
nRT i.e. V − b~−V
On substituting in equation (i), we get (ii) Now calculate the value of compressibility factor (Z ).
pAVA 3p V [ Z = pV / RT ]
= B B …(ii) According to van der Waals’ equation,
nARTA nB RTB
 a
Also, it is given that  p + 2  (V − b) = RT
 V 
VA = 2VB , nA = nB and TA = TB
∴ Eq. (ii) becomes  a
At low pressure,  p + 2  V = RT
pA × 2VB 3 pBVB  V 
= a a
nB RTB nB RTB ⇒ pV + = RT or pV = RT −
V V
⇒ 2 pA = 3 pB
pV a
10. 2NaHCO3 + H2C2O4 → 2CO2 + Na 2C4O4 + H2O Divide both side by RT, =1−
RT RTV
2 mol 1 mol 2 mol
⇒ In the reaction, number of mole of CO2 produced. 2RT
14. C * = Most probable speed =
pV 1 bar × 0.25 × 10−3 L M
n= =
RT 0.082 L atm K−1mol −1 × 298.15 K 8RT
C = Average speed =
−5
= 1.02 × 10 mol πM
Weight of NaHCO3 C = Root square speed corrected as root means square speed, i.e.
Number of mole of NaHCO3 =
Molecular mass of NaHCO3 3RT
rms = and as we know C* < C < C
∴ . × 10−5 × 84 × 103 mg
wNaHCO3 = 102 M
= 0.856 mg * 4 3
C : C : C =1 : : = 1 : 1.128 : 1.225
0.856 p 2
⇒ NaHCO 3 % = × 100 = 8.56%
10 NOTE
11. From the ideal gas equation, As no option correspond to root square speed, it is understood as
misprint. It should be root mean square speed.
pV = ΣnRT ...(i)
Given: p = 200 Pa, V = 10m3 , T = 1000 K 15. The van der Waals’ equation of state is
nA = 0.5 moles, nB = x moles  n2a
 p + 2  (V − nb) = nRT
On substituting the given values in equation (i), we get  V 
200 × 10 = (nA + nB ) × R × 1000
For one mole and when b = 0, the above equation condenses to
200 × 10
0.5 + x =  a
R × 1000  p + 2  V = RT
 V 
1 2 2 1 4−R
+x= = − = a
2 R R 2 2R ⇒ pV = RT − …(i)
V
12. Initially, 1
pV Eq. (i) is a straight equation between pV and whose slope is ‘
Number of moles of gases in each container = i V
RT1 − a’. Equating with slope of the straight line given in the graph.
piV 20.1 − 21.6
Total number of moles of gases in both containers = 2 −a= = − 1.5 ⇒ a = 1.5
RT1 3− 2
States of Matter 77

16. In the van der Waals’ equation V


27. Compressibility factor (Z ) = =1
Videal
 n a
2
 p + 2  (V − nb) = nRT Q For ideal gas V = Videal
 V 
3RT
n2a 28. Expression of rms is, urms =
The additional factor in pressure, i.e. corrects for M
V2
intermolecular force while b corrects for molecular volume. 3R × 50
urms (H2 at 50 K) 2
17. Option (b) is incorrect statement because at high pressure slope ⇒ =
urms (O2 at 800 K) 3R × 800
of the line will change from negative to positive.
32
r(He) 16
18. = =2 :1 50 32
r(CH4 ) 4 = × =1
2 800
3
19. Kinetic energy (E ) = kT 29. Let x g of each gas is mixed.
2 x
Mole of ethane =
3kT 2E 30
RMS speed (u) = ⇒ u=
m m x
Mole of hydrogen =
2
20. Positive deviation corresponds to Z > 1 x
pV pV 15
Q Z= , for positive deviation, > 1. Mole fraction of hydrogen = 2 =
nRT nRT x x 16
+
21. Option (b) and (d) are ruled out on the basis that at the initial 2 30
point of 273 K, 1 atm, for 1.0 mole volume must be 22.4 L, and it Partial pressure of H2
⇒ = Mole fraction of hydrogen
should increase with rise in temperature. Total pressure
Option (a) is ruled out on the basis that initial and final points are = 15 : 16
not connected by the ideal gas equation V ∝ T , i.e. V /T do not 8RT
have the same value at the two points. 30. Average speed =
In option (c), at the initial point, the volume is 22.4 L as required πM
by ideal gas equation and (V /T ) have the same value at both i.e. at constant volume, for a fixed mass, increasing temperature
initial and final points. increases average speeds and molecules collide more frequently
3RT to the wall of container leading to increase in gas pressure.
22. Root mean square velocity (urms ) =
M 31. The mean translational kinetic energy (∈) of an ideal gas is
3
Also, p ⋅ M = dRT ∈= k BT ; T = Absolute temperature, i.e. ∈ ∝ T
2
Substituting for RT / M in urms expression gives,
rCH 4 MX
urms =
3p
⇒ urms ∝
1 32. =2= ⇒ M X = 64
d rX 16
d
w
V 33. The ideal gas equation, pV = nRT = RT
23. Compressibility factor (Z ) = < 1(given) M
Vid  w
⇒ V < 22.4 L ⇒ pM =   RT = dRT (d = density)
V 
Q Vid (1 mol ) = 22.4 L at STP pM
⇒ d=
3RT RT
24. Root mean square speed urms =
M i.e. density will be greater at low temperature and high pressure.
urms (H2 ) T (H2 ) 28 14T (H2 ) 34. The ease of liquefication of a gas depends on their intermolecular
⇒ = 7= × ⇒ 7=
urms (N2 ) 2 T (N2 ) T (N2 ) force of attraction which in turn is measured in terms of van der
⇒ T (N2 ) = 2T (H2 ) i.e. T (H2 ) < T (N2 ) Waals’ constant a. Hence, higher the value of a, greater the
intermolecular force of attraction, easier the liquefication.
25. At high temperature and low pressure, the gas volume is
In the present case, NH3 has highest a, can most easily be
infinitely large and both intermolecular force as well as
liquefied.
molecular volume can be ignored. Under this condition
postulates of kinetic theory applies appropriately and gas 35. HCl will diffuse at slower rate than ammonia because rate of
approaches ideal behaviour. 1
effusion ∝ .
26. Rate of effusion ∝ pi ; pi = Partial pressure of ith component M
1 Therefore, ammonia will travel more distance than HCl in the
∝ same time interval and the two gas will first meet nearer to HCl
M
end.
78 States of Matter

36. In van der Waals’ equation of state 44. The two types of speeds are defined as;
 a 3RT
 p + 2  (V − b) = RT (For 1 mole) Root mean square speed (urms ) =
 V  M
The first factor ( p + a/V 2 ) correct for intermolecular force while 8RT
Average speed (uav ) =
the second term (V − b) correct for molecular volume. πM
8RT For the same gas, at a given temperature, M and T are same,
37. Expression for average velocity is uav = therefore
πM
urms 3RT 8RT
For the same gas but at different temperature = :
uav M πM
uavg (T1 ) T1 300 1
= = = 8
uavg (T2 ) T2 1200 2 = 3: = 3 : 2.54 = 1.085 : 1
π
⇒ uav (927° C) = 2 × uav (27° C) = 0.6 ms−1 45. The explanation of given statements are as follows:
1 (a) Urms is inversely proportional to the square root of its
38. Rate of effusion ∝ ,
M molecular mass.
3RT
39. Let x grams of each hydrogen and methane are mixed, Urms =
x M
Moles of H2 = Hence, option (a) is correct.
2
x (b) When temperature is increased four times then Urms become
Moles of CH4 = doubled.
16
x 3R
Urms = × 4T
2 8 M
⇒ Mole fraction of H2 = =
x x 9 3RT
+ Urms = 2 ×
2 16 M
Partial pressure of H2 8
⇒ = Mole fraction of H2 = Hence, option (b) is correct.
Total pressure 9
(c) and (d) Eav is directly proportional to temperature but
40. According to postulates of kinetic theory, there is no does not depends on its molecular mass at a given
intermolecular attractions or repulsions between the molecules 3
temperature as Eav = KT . If temperature raised four times
of ideal gases. 2
3 than Eav becomes four time multiple.
41. According to kinetic theory, average kinetic energy (E ) = k BT Thus, option (c) is incorrect and option (d) is correct.
2
where, k B is Boltzmann’s constant. Since, it is independent of 46. Equation of state p(V − b) = RT indicates absence of
molar mass, it will be same for He and H2 at a given temperature. intermolecular attraction or repulsion, hence interatomic
potential remains constant on increasing ‘π’ in the beginning. As
42. If x g of both oxygen and methane are mixed then :
the molecules come very close, their electronic and nuclear
x
Mole of oxygen = repulsion increases abruptly.
32
x 47. (a) According to a postulate of kinetic theory of gases, collision
Mole of methane = between the molecules as well as with the wall of container is
16
perfectly elastic in nature.
x
32 1 (b) If a gas molecule of mass m moving with speed u collide to
⇒ Mole fraction of oxygen = = the wall of container, the change in momentum is
x x 3
+ ∆p = – 2mu. Therefore, heavier molecule will transfer more
32 16
momentum to the wall as there will be greater change in
According to law of partial pressure
momentum of the colliding gas molecule. However, this is
Partial pressure of oxygen ( pO 2 ) = Mole fraction × Total pressure not postulated in kinetic theory.
pO 2 1 (c) According to Maxwell-Boltzmann distribution of molecular
⇒ =
p 3 speed, very few molecules have either very high or very low
43. It is the Boyle temperature TB . At Boyle temperature, the first speeds. Most of the molecules moves in a specific,
virial coefficient ( B ) vanishes and real gas approaches ideal intermediate speed range.
behaviour. (d) According to kinetic theory of gases, a gas molecule moves
a in straight line unless it collide with another molecule or to
TB =
Rb the wall of container and change in momentum is observed
only after collision.
Here, a and b are van der Waals’ constants.
States of Matter 79

48. Option (a) is correct because in the limit of large volume, both 11
or 4 (1+ x ) = 15 − 5x or 4 + 4 x = 15 − 5x or x =
intermolecular force and molecular volume becomes negligible 9
in comparison to volume of gas. 11 20
Hence, new volume of A i.e., (1+ x ) will comes as 1 + = or
Option (b) is wrong statement because in the limit of large 9 9
pressure Z > 1. 2.22.
Option (c) is correct statement. For a van der Waals’ gas, 52. Assertion is incorrect because besides amount, pressure also
van der Waals’ constants a and b are characteristic of a gas, depends on volume. However, reason is correct because both
independent of temperature. frequency of collisions and impact are directly proportional to
Option (d) is wrong statement because Z can be either less or root mean square speed which is proportional to square root of
greater than unity, hence real pressure can be less or greater than absolute temperature
ideal pressure. 53. a is the measure of intermolecular force of attraction. Greater the
intermolecular force of attraction (H-bond in the present case)
49. Pressure is inversely proportional to volume at constant higher the value of a.
temperature, hence (a) is correct.
Average kinetic energy of a gas is directly proportional to 54. X is a lighter gas than Y, hence X has greater molecular speed. Due
absolute temperature, hence (b) is correct. to greater molecular speed of X, it will have smaller mean free path
and greater collision frequency with the incrt gas molecules. As a
Expansion at constant temperature cannot change the number of
result X will take more time to travel a given distance along a
molecules, hence (d) is incorrect.
straight line. Hence X and Y will meet at a distance smaller than one
50. Initial pressure ( p1 ) = 48 × 10−3 bar calculated from Graham’s law.
Final pressure ( p2 ) = …… × 10−6 bar Hence, (d) is the correct choice.
4 55. PLAN This problem can be solved by using the concept of Graham’s law
Initial volume (V1 ) = π (3)3 of diffusion according to which rate of diffusion of non-reactive
3
gases under similar conditions of temperature and pressure are
4
Final volume (V2 ) = π (12)3 inversely proportional to square root of their density.
3 1
Rate of diffusion ∝
According to Boyle’s law p1V1 = p2V2 molar weight of gas
pV
p2 = 1 1 Let distance covered by X is d, then distance covered by Y is
V2 24 – d.
4 If rX and rY are the rate of diffusion of gases X and Y,
48 × 10−3 × π (3)3
3 48 × 10−3 × (3)3 40
p2 = = rX
=
d
= =2
4 (12)3
π (12)3 rY 24 − d 10
3
[Q Rate of diffusion ∝ distance travelled]
48 × 10−3 × 27
= = 0.0277 × 27 × 10−3 = 750 × 10−6 bar d = 48 − 2d
1728
⇒ 3d = 48 ⇒ d = 16 cm
Hence, the correct answer is 750.
Hence, (c) is the correct choice.
51. Given p1 = 5 bar, V1 = 1 m 3, T1 = 400 K
56. A. At p = 200 atm, very high pressure, Z > 1. Also, at such a high
5  n2a
So, n1 = (from pV = nRT )
400 R pressure, the pressure correction factor  2  can be ignored
V 
3
Similarly, p2 = 1 bar, V2 = 3 m 3, T2 = 300 K, n2 = in comparison to p.
300 R
B. At p ~ 0, gas will behave like an ideal gas, pV = nRT .
Let at equilibrium the new volume of A will be (1+ x )
C. CO2 (p = 1atm, T = 273 K), Z < 1.
So, the new volume of B will be (3− x )
D. At very large molar volume, real gas behaves like an
Now, from the ideal gas equation. ideal gas.
p1V1 pV
= 2 2 3
n1RT1 n2RT2 57. Less; E = RT
2
and at equilibrium (due to conduction of heat) 3
p1 p2 58. 1 : 16, KE = nRT . At same temperature, KE (total) ∝ n.
= 2
T1 T2 1
59. 0.25 RT because at NTP, 5.6 L = mole.
V1 V2 4
So, = or V1n2 = V2n1
n1 n2 60. Inversely, time.
After putting the values 61. For an ideal gas, Cp − CV = R
3 5 (3 − x ) 5 3 3
(1+ x ) × = (3 − x ) × or (1+ x ) = 62. At 27°C, E = RT = × 2 × 300 = 900 cal
300 R 400 R 4 2 2
80 States of Matter

63. An ideal gas cannot be liquefied because there exist no 71. Given, urms = umps
intermolecular attraction between the molecules of ideal gas.
3RT 2RT
64. a is the measure of intermolecular force. ⇒ =
M (X ) M (Y )
65. In a close container, gas exert uniform pressure everywhere in 3R × 400 2R × 60
the container. ⇒ = ⇒ M (Y ) = 4
40 M (Y )
3
66. KE = RT where, T is absolute temperature (in Kelvin). uav 8RT 3RT 8
2 72. = : =
urms πM M 3π
67. (DC) Diffusion coefficient ∝ λ (mean free path) ∝ U mean
3π 3 × 3.14
Thus (DC) ∝ λ Umean ⇒ urms = uav = × 400 = 434 ms−1
8 8
RT T
But, λ= ⇒ λ∝ rgas
2 N0 σp p 32
73. = 1.33 =
rO 2 Mgas
8RT
and U mean =
πM (i) (a) Mgas = 18 g mol −1
U mean ∝ T 18
(b) Vm = = 50 L mol −1
0.36
(T )3/ 2
∴ DC ∝ pV 1 × 50
p (c) Z = = = 1.22
RT 0.082 × 500
3/ 2 3/ 2
(DC)2 p  T2   p   4T  (d) Q Z > 1, repulsive force is dominating.
(x ) =  1    =  1   1
(DC)1  p2   T1   2 p1   T1 
3 3
(ii) Ek = k BT = × 1.38 × 10−23 × 1000 J = 2.07 × 10−20 J
 1 2 2
=   (8) = 4
 2
74. In case of negligible molecular volume, b = 0. For 1 mole of gas
68. 92 U
238
→ 82Pb
206
+ 8 2He4 (g ) + 6− 1β 0  a a
 p + 2  V = RT ⇒ pV + = RT
n(gas)[Initial] = 1 (air)  V  V
n(gas)[Final] = 8 (He) + 1(air) = 9 pV a  pV 
⇒ + =1 =Z
 RT 
Q
⇒ At constant temperature and volume; RT VRT
p ∝ n. a ap
⇒ Z+ =1 ⇒ Z+ =1
So,
pf nf 9
= = =9  ZRT  ZR 2T 2
ni 1   RT
pi  p 
69. PLAN This problem can be solved by using the concept involved in ZR 2T 2 (1 − Z ) 0.5 (0.082 × 273)2 (1− 0.5)
calculation of significant figure. ⇒ a= =
p 100
Universal gas constant, R = kN A
a = 1.25 atm L2 mol −2
where, k = Boltzmann constant
and N A = Avogadro’s number 75. In case of negligible molecular volume, b = 0 and
∴ −23
R = 1.380 × 10 × 6.023 × 10 J/Kmol 23 van der Waals’ equation reduces to
= 8.31174 =~ 8.312  n2a
 p + 2  V = nRT
Since, k and N A both have four significant figures, so the value of  V 
R is also rounded off upto 4 significant figures. RT a
⇒ p= − 2 (n =1 mole)
[When number is rounded off, the number of significant figure is V V
reduced, the last digit is increased by 1 if following digits ≥ 5 and 0.082 × 273 3.592
is left as such if following digits is ≤ 4.] = − = 0.99 atm
22.4 (22.4)2
Hence, correct integer is (4).
76. (i) For the same amount of gas being effused
70. Since, the external pressure is 1.0 atm, the gas pressure is also
1.0 atm as piston is movable. Out of this 1.0 atm partial pressure r1 t2 p1 M2 57 0.8 M2
= = ⇒ =
due to unknown compound is 0.68 atm. r2 t1 p2 M1 38 1.6 28
Therefore, partial pressure of He ⇒ M 2 = 252 g mol −1
= 1.00 – 0.68 = 0.32 atm.
Also, one molecule of unknown xenon-fluoride contain only
n(He)RT 0.1 × 0.082 × 273
⇒ Volume = = =7L one Xe atom [M (Xe) = 131], formula of the unknown gas can
p(He) 0.32 be considered to be XeFn.
⇒ Volume of container = Volume of He. ⇒ 131 + 19n = 252; n = 6.3, hence the unknown gas is XeF6.
States of Matter 81

(ii) For a fixed amount and volume, p ∝ T 82. Weight of butane gas in filled cylinder = 29 − 14.8 kg = 14.2 kg
1 T ⇒ During the course of use, weight of cylinder reduces to 23.2 kg
⇒ = where, T = Kelvin temperature
1.1 T + 10
⇒ Weight of butane gas remaining now = 23.2 − 14.8 = 8.4 kg
⇒ T = 100 K = t + 273 Also, during use, V (cylinder) and T remains same.
⇒ t = − 173° C p1 n1
Therefore, =
nRT  12  0.082 × 100 p2 n2
Volume = =  × = 0.82 L
p  120 1 n   8.4   n w 
⇒ p2 =  2  p1 =   × 2.5 Here, 2 = 2 
 n1   14.2  n1 w1 
77. The van der Waals’ equation is
 n2a = 1.48 atm
 p + 2  (V − nb) = nRT Also, pressure of gas outside the cylinder is 1.0 atm.
 V 
⇒ pV = nRT
V 2  nRT  (4 )2  2 × 0.082 × 300  nRT (14.2 − 8.4 ) × 103 0.082 × 30
⇒ a= 2 
− p = 2 
− 11 ⇒ V = = × L
n V − nb  (2)  4 − 2 (0.05)  p 58 1
= 6.46 atm L2 mol −2 = 2460 L = 2.46 m 3
78. Mass of liquid = 148 − 50 = 98 g rHe nHe M CH 4 4 16
98 83. = = =8
⇒ Volume of liquid = = 100 mL = volume of flask rCH4 n CH4 M He 1 4
0.98 Initial ratio of rates of effusion gives the initial composition of
mass of gas = 50.5 − 50 = 0.50 g mixture effusing out. Therefore, n (He) : n (CH4 ) = 8 : 1
 w
Now applying ideal gas equation : pV =   RT 2 × 1021
M 84. Number of moles = = 0.33 × 10−2
wRT 0.5 × 0.082 × 300 6 × 1023
⇒ M = = = 123 g mol −1 p = 7.57 × 103 Nm −2
pV 1 × 0.1
Now, pV = nRT
79. False, ideal gas cannot be liquefied as there is no intermolecular
pV 7.57 × 103 × 10−3
attraction between the molecules of ideal gas. Hence, there is no ⇒ T = = = 276 K
point of forming ideal solution by cooling ideal gas mixture. nR 0.33 × 10−2 × 8.314
80. If ‘α’ is the degree of dissociation, then at equilibrium 3RT 3 × 8.314 × 276
⇒ urms = = m s−1 = 496 ms−1
Cl 2 r 2Cl M 28 × 10−3
Moles 1 −α 2α Total = 1 + α umps
Also, = 0.82
From diffusion information urms
r(mix) 84 ⇒ umps = 0.82 × urms = 0.82 × 496 ms−1 = 407 ms−1
= 1.16 =
r(Kr) M (mix)
85. First we calculate partial pressure of NO and O2 in the combined
⇒ M (mix) = 62.4 system when no reaction taken place.
71 pV = constant ⇒ p1V1 = p2V2
⇒ M (mix) = = 62.4
1+ α 1.053 × 250
⇒ p2 (NO) = = 0.752 atm
⇒ α = 0.14 350
0.789 × 100
pV 1 × 40 p2 (O2 ) = = 0.225 atm
81. The total moles of gaseous mixture = = 350
RT 0.082 × 400
= 1.22 Now the reaction stoichiometry can be worked out using partial
pressure because in a mixture.
Let the mixture contain x mole of ethane. Therefore, pi ∝ ni
7
C2H6 + O2 → 2CO2 + 3H2O 2NO + O2 → 2NO2 → N2O4
x 2 Initial 0.752 atm 0.225 atm 0 0
C2H4 + 3O2 → 2CO2 + 2H2O Final 0.302 0 0 0.225 atm
1.22− x
7 x Now, on cooling to 220 K, N2O4 will solidify and only unreacted
Total moles of O2 required = x + 3 (1.22 − x ) = + 3.66 NO will be remaining in the flask.
2 2
130 x Q p∝T
⇒ = + 3.66 p1 T1
32 2 ∴ =
p2 T2
⇒ x = 0.805 mole ethane and 0.415 mole ethene.
0.805 0.302 300
⇒ Mole fraction of ethane = = 0.66 ⇒ =
1.22 p2 220
Mole fraction of ethene = 1 − 0.66 = 0.34 ⇒ p2 (NO) = 0.221 atm
82 States of Matter

pV 6×3 pressure inside the bottle. When the bottle is opened, there is
86. Total moles of gas in final mixture = = = 0.731
RT 0.082 × 300 chances of bumping of stopper. To avoid bumping, bottle
should be cooled that lowers the pressure inside.
Q Mole of H2 in the mixture = 0.70
(ii) According to Avogadro’s hypothesis, “Under identical
∴ Mole of unknown gas ( X ) = 0.031 conditions of pressure and temperature, equal volume of ideal
Because both gases have been diffused for same time gases contain equal number of molecules.”
r (H2 ) 0.70 M pV
= = ⇒ M = 1020 g mol −1 92. Number of moles (n) =
r ( X ) 0.031 2 RT
N (Number of molecules)
nRT and n=
87. V = N A (Avogadro number)
p
 pV 
5 ⇒ N = nNA =   N
For acetylene gas, 5 g = mol  RT  A
26
 7.6 × 10−10 1 
740 = ×  × 6.023 × 1023
p = 740 mm = atm  760 0.082 × 273
760
T = 50° C = 323 K = 2.7 × 1010 molecules

Substituting in ideal gas equation 93. From the given information, it can be easily deduced that in the
5 0.082 × 323 final mixture,
V = × × 76 = 5.23 L partial pressure of A = 1.0 atm
26 74
partial pressure of B = 0.5 atm
8RT1
88. uav (average velocity ) = Also
pV
nA = A =
V
πM RT RT
4
9 × 10 8 × 8.314 T1 pBV 0.5 V
⇒ ms−1 = ⇒ nB = =
100 3.14 × 44 × 10−3 RT RT
T1 = 1682.5 K nB 1 wB M 3 M 
⇒ = = × A = ×  A
Also, for the same gas nA 2 M B wA 2  M B 
uav 8RT1 2RT2 8T1 1 4T1 ⇒ M A :M B = 1 : 3
= : = × =
umps πM M π 2T2 πT2 p
94. Rate of effusion (r) ∝
4T1 M
⇒ 1=
πT2 r (NH3 ) 1 36.5 40 1 36.5
⇒ = × ⇒ =
4T 4 × 1682.5 r (HCl ) 17 p 60 p 17
⇒ T2 = 1 = = 2142 K
π 3.14 3 36.5
⇒ p= = 2.20 atm
Hence, T1 = 1682.5 K, T2 = 2142 K 2 17
3
4 3 4  21 3
89. Volume of balloon = πr = × 3.14 ×   cm 3 95. KE = k BT : k B = Boltzmann’s constant
3 3  2 2
= 4847 cm 3 ≈ 4.85 L 3
= × 1.38 × 10−23 × 300 J = 6.21 × 10−21 J/molecule
2
Now, when volume of H2 (g ) in cylinder is converted into NTP dp kp
volume, then 96. Rate of effusion is expressed as − =
dt M
p1V1 p2V2 20 × 2.82 1 × V2 k = constant, p = instantaneous pressure
= ⇒ =
T1 T2 300 273 dp k dt
⇒ − =
V2 = NTP volume ⇒ V2 = 51.324 L p M
Also, the cylinder will not empty completely, it will hold 2.82 L p kt
Integration of above equation gives ln  0  =
of H2 (g ) when equilibrium with balloon will be established.  p M
Hence, available volume of H2 (g ) for filling into balloon is  2000  k 47
51.324 − 2.82 = 48.504 L
Using first information : ln   =
 1500  32
48.504
⇒ Number of balloons that can be filled = = 10 32  4
4.85 ⇒ k= ln   …(i)
47  3
3RT 3 × 8.314 × 293 Now in mixture, initially gases are taken in equal mole ratio, hence
90. urms = = = 390.2 ms−1
M 48 × 10−3 they have same initial partial pressure of 2000 mm of Hg each.
After 74 min :
91. (i) NH3 (l ) is highly volatile, a closed bottle of NH3 (l ) contains
large number of molecules in vapour phase maintaining high
States of Matter 83

 2000 74 k Also, the decomposition reaction is :


For O2 ln   =
MCO3 → MO + CO2
 pO 2  32
0.05 mol 0.05 mol
Substituting k from Eq. (i) gives Q 0.05 mole MCO3 = 4.21 5 g
 2000 4.215
74 32  4 ∴ 1.0 mole MCO3 = = 84.3 g (molar mass)
ln   = × ln   0.05
 O2 
p 32 47  3
⇒ 84.3 = MW of M + 12 + 48
 2000 74  4 ⇒ Molecular weight of metal = 24.3
ln   = ln  
 O2 
p 47  3 Q Metal is bivalent, equivalent weight
Molecular weight
Solving gives p (O2 ) at 74 min = 1271.5 mm = = 12.15
2
 2000 74 k
For unknown gas : ln   = 100. The ideal gas equation :
 pg  79 w
pV = nRT = RT
Substituting k from (i) gives M
 2000 74 32  4 w
ln   = × ln   ⇒ pM = RT = d RT where, ‘d’ is density.
 g 
p 79 47  3 V
pM 5 × 17
Solving gives : pg = 1500 mm ⇒ d= = = 3.42 g L−1.
RT 0.082 × 303
⇒ After 74 min, p (O2 ) : p (g ) = 1271.5 : 1500
Also, in a mixture, partial pressure ∝ number of moles Topic 2 Liquid State
⇒ n (O2 ) : n (g ) = 1 : 1.18
1. I (CH3OH) : Surface tension decreases as concentration increases.
97. First we determine empirical formula as II (KCl) : Surface tension increases with concentration for ionic
C H salt.
III [CH3 (CH2 )11 OSO−3 Na + ] : It is an anionic detergent.
Weight 10.5 1
There is decrease in surface tension before micelle formation,
10.5 and after CMC (Critical Micelle Concentration) is attained, no
Mole = 0.875 1
12 change in surface tension.
Simple ratio 1 1/0.875 = 1.14
KCl (II)
Surface tension

Whole no. 7 8

⇒ Empirical formula = C7H8 CH3OH (I)



 w CH3(CH2)11OSO3 Na + (III)
From gas equation : pV =   RT
M
wRT 2.8 × 0.082 × 400 Concentration
M = = = 91.84 ≈ 92
pV 1× 1
2. Let us consider, 1.0 L of liquid water is converted into steam .
Q Molar mass (M ) is same as empirical formula weight. Volume of H2O (l) = 1L, mass = 1000 g
Molecular formula = Empirical formula = C7 H8 1000
1 ⇒ Volume of 1000 g steam = cm 3
98. For same p and V , n ∝ 0.0006
T 1000
Q Volume of molecules in cm 3 steam = 1000 cm 3
n (gas) T (H2 ) 0.0006
⇒ =
n (H2 ) T (gas) ∴ Volume of molecules in
0.184 1000
n(H2 ) = = 0.092 1000 cm 3 steam = × 0.0006 × 1000 = 0.60 cm 3
2 1000
290 3. Critical temperature is directly proportional to intermolecular
⇒ n(gas) = × 0.092 = 0.0895
298 force of attraction. H2O is a polar molecule, has greater
Q 0.0895 mole of gas weigh 3.7 g intermolecular force of attraction than O2, hence higher critical
3.7 temperature.
∴ 1 mole of gas will weigh = 41.32 g
0.0895 4. At liquid-vapour equilibrium at boiling point, molecules in two
phase posses the same kinetic energy.
99. Moles of CO2 can be calculated using ideal gas equation as :
pV  700  1336 1
n= =    × = 0.05
RT  760  1000 0.082 × 300
6
Chemical and Ionic Equilibrium

Topic 1 Chemical Equilibrium


Objective Questions I (Only one correct option) The equilibrium constant for the reaction,
1. Consider the following reaction: 2SO2 ( g ) + O2 ( g ) 2SO3 ( g ) is
-
(2019 Main, 8 April II)
N 2O4 ( g ) q 2NO2 ( g ); ∆H ° = + 58 kJ 25 77 154
(a) 10 (b) 10 (c) 10 (d) 10181
For each of the following cases (A , B ), the direction in which the K
equilibrium shifts is (2020 Main, 5 Sep I) 6. In a chemical reaction, A + 2B - 2C + D,
the
(A) temperature is decreased. initial concentration of B was 1.5 times of the
(B) pressure is increased by adding N 2 at constant T. concentration of A, but the equilibrium concentrations of
(a) (A) towards product, (B) towards reactant A and B were found to be equal. The equilibrium
(b) (A) towards reactant, (B) no change
constant (K ) for the aforesaid chemical reaction is
(2019 Main, 12 Jan I)
(c) (A) towards reactant, (B) towards product
1
(d) (A) towards product, (B) no change (a) (b) 16
4
2. The incorrect match in the following is (2019 Main, 12 April II)
(c) 1 (d) 4
(a) ∆G ° < 0, K > 1 (b) ∆G ° = 0, K = 1
(c) ∆G ° > 0, K < 1 (d) ∆G ° < 0, K < 1
7. Two solids dissociate as follows:
- B (g ) + C(g ); K
A (s ) p1 = x atm2
3. In which one of the following equilibria, K p ≠ K c ?
(2019 Main, 12 April II) D ( s ) - C( g ) + E ( g ); K p2
= y atm2
(a) 2C(s) + O2 (g )
c 2CO(g )
The total pressure when both the solids dissociate
(b) 2HI(g ) c H2 (g ) + I2 (g ) simultaneously is (2019 Main, 12 Jan I)
(c) NO2 (g ) + SO2 (g )
c NO(g ) + SO3 (g ) (a) x + y atm (b) x 2 + y2 atm
(d) 2NO(g ) c N2 (g ) + O2 (g )
(c) (x + y) atm (d) 2( x + y ) atm
4. For the reaction,
8. Consider the reaction,
2SO2 ( g ) + O2 ( g ) → 2SO3 ( g ), ∆H = − 57.2 kJ mol −1 and
N 2 ( g ) + 3H 2 ( g ) =2NH (g ) 3
K c = 1.7 × 1016 . Which of the following statement is incorrect?
The equilibrium constant of the above reaction is K p . If
(2019 Main, 10 April II)
pure ammonia is left to dissociate, the partial pressure
(a) The equilibrium constant decreases as the temperature increases of ammonia at equilibrium is given by (Assume that
(b) The addition of inert gas at constant volume will not affect the pNH 3 < < p total at equilibrium) (2019 Main, 11 Jan I)
equilibrium constant
33/ 2 K p1/ 2P 2 33/ 2 K p1/ 2P 2
(c) The equilibrium will shift in forward direction as the pressure (a) (b)
increases 4 16
1/ 2 2 1/ 2 2
(d) The equilibrium constant is large suggestive of reaction going to Kp P Kp P
(c) (d)
completion and so no catalyst is required 16 4
5. For the following reactions, equilibrium constants are given : 9. 5.1 g NH4 SH is introduced in 3.0 L evacuated flask at
52
- SO (g ); K = 10
S( s ) + O2 ( g ) 2 1 327°C. 30% of the solid NH4 SH decomposed to NH3
2S( s ) + 3O ( g ) - 2SO ( g ); K = 10 129 and H2 S as gases. The K p of the reaction at 327° C is
2 3 2
Chemical and Ionic Equilibrium 85

(R = 0.082 atm mol −1 K −1 , molar mass of S = 32 g mol −1 , 1


15. For the reaction, SO2 ( g ) + O2 ( g ) q SO3 ( g )
molar mass of N = 14 g mol ) −1 2
(2019 Main, 10 Jan II)
if K p = K C ( RT )x where, the symbols have usual meaning,
(a) 0. 242 × 10−4 atm 2 (b) 0. 242 atm 2
then the value of x is (assuming ideality) (2014 Main)
(c) 4 . 9 × 10−3 atm 2 (d) 1 × 10−4 atm 2 1 1
(a) − 1 (b) − (c) (d) 1
Kp 2 2
10. The values of for the following reactions at 300 K are,
KC 16. The species present in solution when CO2 is dissolved in
respectively (At 300 K, RT = 24.62 dm atm mol ) 3 −1 water are (2006 Main)
(a) CO2 , H2CO3 , HCO3− , CO32−
N 2 ( g ) + O2 ( g )=2NO(g ) (b) H2CO3 , CO2−
3
N 2O4 ( g ) =2NO (g ) 2
(c) HCO−3 , CO23−
N (g ) + 3H (g ) =2NH (g )
2 2 3
(d) CO2 , H2CO3
(2019 Main, 10 Jan I)
17. N2 + 3H2 r 2NH3
(a) 1, 24.62 dm 3 atm mol −1 , 606.0 dm 6 atm 2 mol −2 Which is correct statement if N2 is added at equilibrium
(b) 1, 24.62 dm 3 atm mol −1, 1.65 × 10−3 dm −6 atm −2 mol 2 condition? (2006, 3M)
(c) 24.62 dm 3 atm mol −1, 606.0 dm6 atm −2 mol 2, (a) The equilibrium will shift to forward direction because
1.65 × 10−3 dm −6 atm −2 mol 2 according to IInd law of thermodynamics, the entropy must
(d) 1, 4.1 × 10−2 dm −3atm −1 mol, 606 dm6 atm 2 mol −2 increases in the direction of spontaneous reaction
(b) The condition for equilibrium is G(N2) + 3G(H2)
11. Consider the following reversible chemical reactions, = 2G(NH3) where, G is Gibbs free energy per mole of the
K1
A2 ( g ) + B2 ( g ) -2 AB (g ) …(i) gaseous species measured at that partial pressure. The
K2 condition of equilibrium is unaffected by the use of
6 AB ( g ) - 3 A ( g ) + 3B ( g )
2 2 …(ii) catalyst, which increases the rate of both the forward and
backward reactions to the same extent
The relation between K 1 and K 2 is (2019 Main, 9 Jan II)
(c) The catalyst will increase the rate of forward reaction by α
(a) K 2 = K13 (b) K 1K 2 = 3
and that of backward reaction by β
1
(c) K 2 = K1− 3 (d) K 1K 2 = (d) Catalyst will not alter the rate of either of the reaction
3
18. Ag + + NH3 s [Ag(NH3 )]+ ; K 1 = 3. 5 × 10−3
12. An aqueous solution contains 0.10 M H2S and 0.20 M HCl. If
the equilibrium constants for the formation of HS− from H2S [Ag (NH3 )]+ + NH3 s [Ag (NH3 )2 ]+ ; K 2 = 1.7 ×10−3
. × 10−7 and that of S2− from HS− ions is 12
is 10 . × 10−13 then then the formation constant of [Ag(NH3 )2 ]+ is (2006, 3M)
the concentration of S2− ions in aqueous solution is : (a) 6.08 × 10 −6
(b) 6.08 × 106
(2018 Main) −9
(a) 5 × 10−8 M (b) 3 × 10−20 M (c) 6.08 × 10 (d) None of these
−21 −19
(c) 6 × 10 M (d) 5 × 10 M 19. Consider the following equilibrium in a closed container
13. The equilibrium constant at 298 K for a reaction, N2 O4 ( g ) r 2NO2 ( g )
A + B q C + D is 100. If the initial concentrations of all At a fixed temperature, the volume of the reaction container is
the four species were 1 M each, then equilibrium halved. For this change, which of the following statements
concentration of D (in mol L−1 ) will be (2016 Main) hold true regarding the equilibrium constant ( K p ) and degree
(a) 0.818 (b) 1.818 of dissociation (α ) ? (2002, 3M)
(c) 1.182 (d) 0.182 (a) Neither K p nor α changes
14. The standard Gibbs energy change at 300 K for the reaction, (b) Both K p and α change
2A a B + C is 2494. 2 J. At a given time, the composition
1 1 (c) K p changes but α does not change
of the reaction mixture is [A]= , [ B ] = 2 and [C ] = . The
2 2 (d) K p does not change but α changes
reaction proceeds in the
20. At constant temperature, the equilibrium constant ( K p ) for
(R = 8.314JK / mol, e = 2.718) (2015, Main)
the decomposition reaction, N2 O4 r 2NO2 , is expressed
(a) forward direction because Q > K c
4 x2 p
(b) reverse direction because Q > K c by K p = , where, p = pressure, x = extent of
(c) forward direction because Q < K c (1 − x2 )
(d) reverse direction because Q < K c decomposition. Which one of the following statement is true?
(2001, 1M)
86 Chemical and Ionic Equilibrium

(a) K p increases with increase of p (c) concentration of NH3 does not change with pressure
(b) K p increases with increase of x (d) concentration of hydrogen is less than that of nitrogen
(c) K p increases with decrease of x
(d) K p remains constant with change in p and x
28. For the reaction, H2 ( g ) + I2 ( g ) r 2HI( g )

21. When two reactants, A and B are mixed to give products, C the equilibrium constant K p changes with (1981, 1M)

and D, the reaction quotient, (Q ) at the initial stages of the (a) total pressure
reaction (2000) (b) catalyst
(a) is zero (b) decreases with time (c) the amount of H2 and I2 present
(c) is independent of time (d) increases with time (d) temperature

22. For the reversible reaction, Objective Questions II


N2 ( g ) + 3H2 ( g ) r 2NH3 ( g )
(One or more than one correct option)
at 500° C , the value of K p is 1.44 × 10–5 when partial
29. For a reaction, A P, the plots of [A] and [P] with time at
-
pressure is measured in atmosphere. The corresponding value
temperatures T1 and T2 are given below.
of K c with concentration in mol/L is (2000, S, 1M)

1.44 × 10−5 1.44 × 10−5 10 10


(a) (b) T1
(0.082 × 500)−2 (8.314 × 773)−2

[A]/(mol L–1)

[P]/(mol L–1)
T2
1.44 × 10−5 1.44 × 10–5 5 5
(c) (d)
(0.082 × 773)2 (0.082 × 773)−2 T2
T1
23. For the chemical reaction, Time Time
3 X ( g ) + Y ( g ) r X 3Y ( g )
If T2 > T1 , the correct statement(s) is are
the amount of X 3Y at equilibrium is affected by (1999, 2M)
(Assume ∆H s and ∆S s are independent of temperature and
(a) temperature and pressure ratio of ln K at T1 to ln K at T2 is greater than T2 / T1 . Here
(b) temperature only H , S , G and K are enthalpy, entropy, Gibbs energy and
(c) pressure only equilibrium constant, respectively.) (2018 Adv.)
(d) temperature, pressure and catalyst (a) ∆H s < 0, ∆S s < 0 (b) ∆Gs < 0, ∆H s > 0
24. For the reaction , (c) ∆Gs < 0, ∆S s < 0 (d) ∆Gs < 0, ∆S s > 0
CO( g ) + H2 O( g ) r CO2 ( g ) + H2 ( g ) ,
30. The % yield of ammonia as a function of time in the reaction,
at a given temperature, the equilibrium amount of CO2 ( g )
can be increased by (1998)
N2( g )+ 3H2( g ) w 2 NH3( g ); ∆H < 0 (2015 Adv.)

(a) adding a suitable catalyst at( p, T1 ) is given below.


(b) adding an inert gas
(c) decreasing the volume of the container
% yield

T1
(d) increasing the amount of CO(g )
25. One mole of N2 O4 ( g ) at 300 K is kept in a closed container
under one atmosphere. It is heated to 600 K when 20% by
Time
mass of N2 O4 ( g ) decomposes to NO2 (g). The resultant
pressure is (1996, 1M) If this reaction is conducted at ( p , T1 ), with T2 > T1 the %
(a) 1.2 atm (b) 2.4 atm (c) 2.0 atm (d) 1.0 atm yield by of ammonia as a function of time is represented by
26. An example of a reversible reaction is (1985, 1M)
(a) Pb(NO3 )2 (aq) + 2NaI (aq) = PbI2 (s) + 2NaNO3 (aq) T2 T1
% yield

% yield

T1
(b) AgNO3 (aq) + HCl (aq) = AgCl (s) + HNO3 (aq) (a) (b)
T2

(c) 2Na (s) + 2H2O (l ) = 2NaOH (aq) + H2 (g )


(d) KNO3 (aq) + NaCl (aq) = KCl (aq) + NaNO3 (aq) Time Time
27. Pure ammonia is placed in a vessel at a temperature where its
T2
dissociation constant (α ) is appreciable. At equilibrium, T1
% yield

% yield

T2 T1
N2 + 3H2 s 2NH3 (1984, 1M) (c) (d)

(a) K p does not change significantly with pressure


Time Time
(b) α does not change with pressure
Chemical and Ionic Equilibrium 87

31. The initial rate of hydrolysis of methyl acetate (1 M) by a 38. A ten-fold increase in pressure on the reaction,
weak acid (HA, 1M) is 1/100th of that of a strong acid N2 ( g ) + 3H2 ( g ) r 2NH3 ( g ) at equilibrium, results in
(HX, 1M), at 25°C. The K a (HA) is (2013 Adv.) .............. in K p .
(1996, 1M)
(a) 1 × 10−4 (b) 1 × 10−5
39. For a given reversible reaction at a fixed temperature,
(c) 1 × 10−6 (d) 1 × 10−3 equilibrium constant K p and K c are related by ..... (1994, 1M)
I 0 II
32. The equilibrium 2 Cu r Cu + Cu in aqueous
True/False
medium at 25° C shifts towards the left in the presence of
(2011)
40. The rate of an exothermic reaction increases with increasing
temperature. (1993, 1M)
(a) NO −3 (b) Cl −
41. Catalyst makes a reaction more exothermic.
(c) SCN− (d) CN− (1987, 1M)

33. For the reaction, PCl 5 ( g ) r PCl 3 ( g ) + Cl 2 ( g ) the 42. If equilibrium constant for the reaction, A2 + B2 r 2 AB,
forward reaction at constant temperature is favoured by is K, then for the backward reaction
(1991, 1M) 1 1 1
AB r A2 + B2 , the equilibrium constant is .
(a) introducing an inert gas at constant volume 2 2 K
(b) introducing chlorine gas at constant volume (1984, 1M)
(c) introducing an inert gas at constant pressure
43. When a liquid and its vapour are at equilibrium and the pressure
(d) increasing the volume of the container
is suddenly decreased, cooling occurs. (1984, 1M)
(e) introducing PCl 5 at constant volume

34. The equilibrium SO2 Cl 2 ( g ) r SO2 ( g ) + Cl 2 ( g ) is Subjective Questions


attained at 25° C in a closed container and an inert gas, 44. (a) In the following equilibrium N2O4 (g ) r 2NO2 (g )
helium is introduced. Which of the following statements when 5 moles of each are taken, the temperature is kept at 298
are correct? (1989, 1M) K the total pressure was found to be 20 bar. Given that
(a) Concentration of SO2 ,Cl 2 and SO2Cl 2 change ∆G °f (N2O4 ) = 100 kJ, ∆G °f (NO2 ) = 50 kJ
(b) More chlorine is formed (i) Find ∆G of the reaction.
(c) Concentration of SO2 is reduced (ii) The direction of the reaction in which the equilibrium shifts.
(d) None of the above (b) A graph is plotted for a real gas which follows van der Waals’
35. When NaNO3 is heated in a closed vessel, oxygen is liberated equation with pVm taken on Y-axis and p on X-axis. Find the
intercept of the line where Vm is molar volume. (2004, 4M)
and NaNO2 is left behind. At equilibrium, (1986, 1M)
(a) addition of NaNO2 favours reverse reaction 45. When 3.06 g of solid NH4 SH is introduced into a two litre
(b) addition of NaNO3 favours forward reaction evacuated flask at 27° C, 30% of the solid decomposes into
gaseous ammonia and hydrogen sulphide.
(c) increasing temperature favours forward reaction
(i) Calculate K c and K p for the reaction at 27°C.
(d) increasing pressure favours reverse reaction
(ii) What would happen to the equilibrium when more solid
36 For the gas phase reaction, NH4SH is introduced into the flask? (1999, 7M)
C2 H4 + H2 r C2 H6 ( ∆H = −32.7 kcal)
46. (a) The degree of dissociation is 0.4 at 400 K and 1.0 atm for the
carried out in a vessel, the equilibrium concentration of gaseous reaction PCl 5 r PCl 3 + Cl 2. Assuming ideal
C2 H4 can be increased by (1984, 1M) behaviour of all the gases, calculate the density of equilibrium
(a) increasing the temperature mixture at 400 K and 1.0 atm (relative atomic mass of P = 31.0
and Cl = 35.5).
(b) decreasing the pressure
(b) Given, [Ag(NH3 )+2 ] r Ag+ + 2NH3 ,
(c) removing some H2
K c = 6.2 × 10−8 and K sp of AgCl
(d) adding some C2H6
= 1.8 × 10−10 at 298 K.
Fill in the Blanks If ammonia is added to a water solution containing excess of
37. For a gaseous reaction 2B → A, the equilibrium AgCl(s) only. Calculate the concentration of the complex in
1.0 M aqueous ammonia. (1998, 3M+5M)
constant K p is …… to/than K c . (1997 C, 1M)
88 Chemical and Ionic Equilibrium

47. The progress of reaction, 53. The equilibrium constant of the reaction
A r nB A2 ( g ) + B2 ( g ) r 2 AB ( g ) at 100°C is 50. If a one litre
with time, is represented in fig. use given below. flask containing one mole of A2 is connected to a two litre
flask containing two moles of B2 , how many moles of AB
(Concentration/mol L 1 )

will be formed at 373 K? (1985, 4M)


0.5
54. One mole of N2 and 3 moles of PCl 5 are placed in a 100 L
vessel heated to 227°C. The equilibrium pressure is 2.05
0.3 atm. Assuming ideal behaviour, calculate the degree of
dissociation for PCl 5 and K p for the reaction,
0.1 PCl 5 ( g ) r PCl 3 ( g ) + Cl 2 ( g ) (1984, 6M)
1 3 5 7 55. One mole of nitrogen is mixed with three moles of hydrogen
Time/h
Determine : in a four litre container. If 0.25 per cent of nitrogen is
converted to ammonia by the following reaction
(i) the value of n
(ii) the equilibrium constant, K and N2 ( g ) + 3H2 ( g ) r 2NH3 ( g ), then
(iii) the initial rate of conversion of A. (1994, 3M) calculate the equilibrium constant, K c in concentration units.
What will be the value of K c for the following equilibrium?
48. 0.15 mole of CO taken in a 2.5 L flask is maintained at 750 K
1 3
along with a catalyst so that the following reaction can take place: N2 ( g ) + H2 ( g ) r NH3 ( g ) (1981, 4M)
CO ( g ) + 2H2 ( g ) r CH3 OH( g ) 2 2
Hydrogen is introduced until the total pressure of the system
is 8.5 atm at equilibrium and 0.08 mole of methanol is
Passage Based Questions
formed. Calculate (i) K p and K c and (ii) the final pressure if Thermal decomposition of gaseous X 2 to gaseous X at 298 K takes
the same amount of CO and H2 as before are used, but with no place according to the following equation:
catalyst so that the reaction does not take place. (1993, 5M) X 2 (g ) s 2X (g )
49. For the reaction, CO( g ) + 2H2 ( g ) r CH3 OH( g ) The standard reaction Gibbs energy, ∆ r G°, of this reaction is
positive. At the start of the reaction, there is one mole of X 2 and no
hydrogen gas is introduced into a five litre flask at 327° C,
X . As the reaction proceeds, the number of moles of X formed is
containing 0.2 mole of CO( g ) and a catalyst, until the
given by β. Thus, βequilibrium is the number of moles of X formed at
pressure is 4.92 atm. At this point 0.1 mole of CH3 OH( g ) is
equilibrium. The reaction is carried out at a constant total pressure
formed. Calculate the equilibrium constant, K p and K c .
of 2 bar. Consider the gases to behave ideally.
(Given, R = 0.083 L bar K −1 mol −1 )
(1990, 5M)

50. The equilibrium constant K p of the reaction,


56. The equilibrium constant K p for this reaction at 298 K, in
2SO2 ( g ) + O2 ( g ) r 2SO3 ( g ) terms of β equilibrium is (2016 Adv.)
is 900 atm at 800 K. A mixture containing SO3 and O2 having 8 β 2equilibrium 8 β 2equilibrium
initial pressure of 1 and 2 atm respectively is heated at (a) (b)
2 − β equilibrium 4 − β 2equilibrium
constant volume to equilibrate. Calculate the partial pressure
of each gas at 800 K. 4 β 2equilibrium 4 β 2equilibrium
(1989, 3M) (c) (d)
2 − β equilibrium 4 − β 2equilibrium
51. N2 O4 is 25% dissociated at 37° C and one atmosphere
pressure. Calculate (i) K p and (ii) the percentage dissociation 57. The incorrect statement among the following for this
at 0.1 atm and 37° C. (1988, 4M)
reaction, is (2016 Adv.)
(a) Decrease in the total pressure will result in the formation of
52. At a certain temperature, equilibrium constant ( K c ) is 16 for more moles of gaseous X
the reaction; (b) At the start of the reaction, dissociation of gaseous X 2 takes
SO2 ( g ) + NO2 ( g ) r SO3 ( g ) + NO( g ) place spontaneously
If we take one mole each of all the four gases in a one litre (c) β equilibrium = 0.7
container, what would be the equilibrium concentrations of (d) KC < 1
NO and NO2 ? (1987, 5M)
Chemical and Ionic Equilibrium 89

Topic 2 Ionic Equilibrium


Objective Questions I (Only one correct option)  Ksp 
1/ 6
 Ksp 
1/ 7
(a) S =   (b) S =  
1. 100 mL of 0.1M HCl is taken is a beaker and to it 100 mL of  144   6912
0.1 M NaOH is added in steps of 2 mL and the pH 1/ 9 1/ 7
 Ksp   Ksp 
continuously measured. Which of the following graphs (c) S =   (d) S =  
correctly depicts the change in pH? (2020 Main, 3 Sep II)
 929  216

7. If K sp of Ag 2 CO3 is 8 × 10− 12 , the molar solubility of


Ag 2 CO3 in 0.1 M AgNO3 is (2019 Main, 12 Jan II)
(a) pH 7 (b) pH 7 (a) 8 × 10− 12 M (b) 8 × 10− 13 M
(c) 8 × 10− 10 M (d) 8 × 10− 11 M
vol. of NaOH vol. of NaOH 8. 20 mL of 0.1 M H2 SO4 solution is added to 30 mL of 0.2 M
NH4 OH solution. The pH of the resultant mixture is [pK b of
NH4 OH = 4.7] (2019 Main, 9 Jan I)

(c) pH 7 (d) pH 7 (a) 9.3 (b) 5.0


(c) 9.0 (d) 5.2
9. An aqueous solution contains an unknown concentration of
vol. of NaOH vol. of NaOH Ba2 + . When 50 mL of a 1 M solution of Na2SO4 is added,
. × 10−5 m in water.
2. The molar solubility of Cd (OH)2 is 184 BaSO4 just begins to precipitate. The final volume is 500 mL.
The expected solubility of Cd(OH)2 in a buffer solution of The solubility product of BaSO4 is 1 × 10−10 . What is the
pH = 12 is (2019 Main, 12 April II) original concentration of Ba 2+ ? (2018 Main)
2.49 −9 −9
. × 10
(a) 184 −9
M (b) × 10−9 M (a) 5 × 10 M(b) 2 × 10 M
184
. . × 10−9 M
(c) 11 . × 10−10 M
(d) 10
(c) 6.23 × 10−11 M (d) 2.49 × 10−10 M
10. Which of the following are Lewis acids? (2018 Main)
3. What is the molar solubility of Al(OH)3 in 0.2 M NaOH
(a) PH3 and BCl 3 (b) AlCl 3 and SiCl 4
solution? Given that, solubility product of (c) PH3 and SiCl 4 (d) BCl 3 and AlCl 3
Al(OH)3 = 2.4 × 10−24 (2019 Main, 12 April II)
11. Which of the following salts is the most basic in aqueous
(a) 3 × 10−19 (b) 12 × 10−21 solution? (2018 Main)
(c) 3 × 10−22 (d) 12 × 10−23 (a) Al(CN)3 (b) CH3COOK
4. The pH of a 0.02 M NH4 Cl solution will be [Given (c) FeCl 3 (d) Pb(CH3COO)2
K b (NH4 OH) = 10−5 and log 2 = 0.301] (2019 Main, 10 April II) 12. pK a of a weak acid (HA) and pK b of a weak base (BOH) are
(a) 4.65 (b) 2.65 3.2 and 3.4, respectively. The pH of their salt (AB) solution is
(c) 5.35 (d) 4.35 (2017 Main)

5. Consider the following statements. (a) 7.2 (b) 6.9


(c) 7.0 (d) 1.0
I. The pH of a mixture containing 400 mL of 0.1 M H2SO4 and
400 mL of 0.1 M NaOH will be approximately 1.3. 13. How many litres of water must be added to 1 L of an aqueous
II. Ionic product of water is temperature dependent. solution of HCl with a pH of 1 to create an aqueous solution
III. A monobasic acid with K a = 10−5 has a pH = 5. The degree with pH of 2? (2013 Main)
of dissociation of this acid is 50%. (a) 0.1 L (b) 0.9 L
IV. The Le-Chatelier’s principle is not applicable to (c) 2.0 L (d) 9.0 L
common-ion effect. 14. Solubility product constant ( K sp ) of salts of types MX , MX 2
The correct statements are (2019 Main, 10 April I)
and M 3 X at temperature ‘T ’ are 4.0 × 10−8 , 3.2 × 10−14 and
(a) I, II and IV (b) II and III
(c) I and II (d) I, II and III 2.7 × 10−15 , respectively. Solubilities (mol dm− 3 ) of the salts
6. If solubility product of Zr3 (PO4 )4 is denoted by K sp and its at temperature ‘T ’ are in the order (2008, 3M)

molar solubility is denoted by S , then which of the following (a) MX > MX 2 > M 3 X (b) M 3 X > MX 2 > MX
relation between S and K sp is correct? (2019 Main, 8 April I) (c) MX 2 > M 3 X > MX (d) MX > M 3 X > MX 2
90 Chemical and Ionic Equilibrium

15. 2.5 mL of 2/5 M weak monoacidic base (K b = 1 × 10− 12 at 25. When equal volumes of the following solutions are mixed,
25°C) is titrated with 2/15 M HCl in water at 25°C. The precipitation of AgCl ( K sp = 1.8 × 10−10 ) will occur only
concentration of H+ at equivalence point is with (1988, 1M)
(K w = 1 × 10− 14 at 25°C) (2008, 3M) (a) 10−4 M (Ag+ ) and 10−4 M (Cl − )
− 13 −7
(a) 3.7 × 10 M (b) 3.2 × 10 M (b) 10−5 M (Ag+ ) and 10−5 M (Cl − )
−2 −2
(c) 3.2 × 10 M (d) 2.7 × 10 M (c) 10−6 M (Ag+ ) and 10−6 M (Cl − )
16. CH3 NH2 (0.1 mole, K b = 5 × 10− 4 ) is added to 0.08 mole of (d) 10−10 M (Ag+ ) and 10−10 M (Cl − )
HCl and the solution is diluted to one litre, resulting hydrogen 26. The pK a of acetyl salicylic acid (aspirin) is 3.5. The pH of
ion concentration is (2005, 1M)
gastric juice in human stomach is about 2-3 and the pH in the
(a) 1.6 × 10− 11 (b) 8 × 10− 11
small intestine is about 8. Aspirin will be (1988, 1M)
(c) 5 × 10− 5 (d) 8 × 10− 2
(a) unionised in the small intestine and in the stomach
17. HX is a weak acid ( K a = 10−5 ). It forms a salt NaX (0.1M) on (b) completely ionised in the small intestine and in the stomach
reacting with caustic soda. The degree of hydrolysis (c) ionised in the stomach and almost unionised in the small
of NaX is intestine
(2004, 1M)
(d) ionised in the small intestine and almost unionised in the
(a) 0.01% (b) 0.0001% (c) 0.1% (d) 0.5% stomach
18. A solution which is 10−3 M each in Mn 2+ , Fe2+ , Zn 2+ and 27. The compound that is not a Lewis acid is (1985, 1M)
Hg 2+ is treated with 10−16 M sulphide ion. If K sp of
MnS, FeS, ZnS and HgS are 10−15 , 10−23 , 10−20 and 10−54 (a) BF3 (b) AlCl 3 (c) BeCl 2 (d) SnCl 4
respectively, which one will precipitate first? (2003, 1M) 28. The conjugate acid of NH–2 is (1985, 1M)
(a) FeS (b) MgS (c) HgS (d) ZnS
(a) NH3 (b) NH2OH (c) NH+4 (d) N2H4
19. Identify the correct order of solubility of Na 2 S, CuS and ZnS
in aqueous medium. (2002)
29. The best indicator for detection of end point in titration of a
weak acid and a strong base is (1985, 1M)
(a) CuS > ZnS > Na 2S (b) ZnS > Na 2S > CuS
(a) methyl orange (3 to 4) (b) methyl red (5 to 6)
(c) Na 2S > CuS > ZnS (d) Na 2S > ZnS > CuS
(c) bromothymol blue (6 to 7.5) (d) phenolphthalein (8 to 9.6)
20. For a sparingly soluble salt A p Bq , the relationship of its
30. A certain weak acid has a dissociation constant of 1.0 × 10−4 .
solubility product ( Ls ) with its solubility (S) is (2001, 1M)
The equilibrium constant for its reaction with a strong base is
(a) Ls = S p + q ⋅ pp ⋅ qq (b) Ls = S p + q ⋅ pq ⋅ qp (1984, 1M)
(p + q) −4 −10 10
(c) Ls = S pq
⋅ p ⋅q
p q
(d) Ls = S pq
⋅ ( p. q) (a) 1.0 × 10 (b) 1.0 × 10 (c) 1.0 × 10 (d) 1.0 × 1014
21. The pH of 0.1 M solution of the following salts increases in 31. A certain buffer solution contains equal concentration of X −
the order (1999, 2M) and HX. The K b for X − is 10−10 . The pH of the buffer is
(1984, 1M)
(a) NaCl < NH4Cl < NaCN < HCl
(a) 4 (b) 7 (c) 10 (d) 14
(b) HCl < NH4Cl < NaCl < NaCN
(c) NaCN < NH4Cl < NaCl < HCl 32. The precipitate of CaF2 , ( K sp = 1.7 × 10−10 ) is obtained, when
(d) HCl < NaCl < NaCN < NH4Cl equal volumes of which of the following are mixed?
(1982, 1M)
22. Which of the following solutions will have pH close to 1.0 ? −4 2+ −4 − −2 2+
(a) 10 M Ca + 10 MF (b) 10 M Ca + 10−3 M F−
(1992, 1M)
(a) 100 mL of (M/10) HCl + 100 mL of (M/10) NaOH (c) 10−5 M Ca 2+ + 10−3 M F− (d) 10−3 M Ca 2+ + 10−5 M F−
(b) 55 mL of (M/10) HCl + 45 mL of (M/10) NaOH 33. An acidic buffer solution can be prepared by mixing the
(c) 10 mL of (M/10) HCl + 90 mL of (M/10) NaOH solution of (1981, 1M)
(d) 75 mL of (M/5) HCl + 25 mL of (M/5) NaOH (a) acetate and acetic acid
23. Amongst the following hydroxides, the one which has the (b) ammonium chloride and ammonium hydroxide
lowest value of K sp at ordinary temperature (about 25° C) is (c) sulphuric acid and sodium sulphate
(1990, 1M)
(d) sodium chloride and sodium hydroxide
(a) Mg(OH)2 (b) Ca(OH)2 (c) Ba(OH)2 (d) Be(OH)2 34. Of the given anions, the strongest base is (1981, 1M)
(a) ClO− (b) ClO−2 (c) ClO−3 (d) ClO−4
24. Which of the following is the strongest acid? (1989, 1M)
(a) ClO3 (OH) (b) ClO2 (OH)
35. At 90°C, pure water has [H3 O+ ] as 10−6 mol L−1 . What is the
value of K w at 90°C ? (1981, 1M)
(c) SO(OH)2 (d) SO2 (OH) 2
(a) 10−6 (b) 10−12 (c) 10−14 (d) 10−8
Chemical and Ionic Equilibrium 91

36. The pH of 10−8 M solution of HCl in water is (1981, 1M) 42. An acidified solution of 0.05 M Zn 2+ is saturated with 0.1 M
(a) 8 (b) −8 H 2S. What is the minimum molar concentration (M) of H +
(c) between 7 and 8 (d) between 6 and 7 required to prevent the precipitation of ZnS?
Use K sp ( ZnS) = 1. 25 × 10−22 and overall dissociation
Objective Questions II
constant of H2 S, K net = K 1 K 2 = 1 × 10−21 . (2020 Adv.)
(One or more than one correct option)
43. For the following reaction, the equilibrium constant K c at 298
37. The K sp of Ag 2 CrO4 is 1.1 × 10−12 at 298 K. The solubility (in
. × 1017 .
K is 16
mol/L) of Ag 2 CrO4 in a 0.1 M AgNO3 solution is
Fe2 + ( aq ) + S2 − ( aq ) -FeS ( s )
(a) 1.1 × 10−11 (b) 1.1 × 10−10
(c) 1.1 × 10−12 (d) 1.1 × 10−9 When equal volumes of0.06 M Fe2 + ( aq ) and
(2013 Adv.)
0. 2 M S2 − ( aq ) solutions are mixed, the equilibrium
38. Aqueous solutions of HNO3 KOH, CH3 COOH and
concentration of Fe2 + ( aq ) is found by Y × 10− 17 M. The
CH3 COONa of identical concentrations are provided. The
value of Y is ........... (2019 Adv.)
pair(s) of solutions which form a buffer upon mixing is(are)
(a) HNO3 and CH3COOH (2010) 44. The solubility of a salt of weak acid (AB) at pH 3 is
(b) KOH and CH3COONa Y × 10−3 mol L−1 . The value of Y is__ (Given that the value
(c) HNO3 and CH3COONa of solubility product of AB ( K sp ) = 2 × 10−10 and the value of
(d) CH3COOH and CH3COONa
ionisation constant of HB ( K a ) = 1 × 10−8 (2018 Adv.)
39. A buffer solution can be prepared from a mixture of
(a) sodium acetate and acetic acid in water (1999, 3M) Match the Column
(b) sodium acetate and HCl in water 45. Dilution processes of different aqueous solutions, with water,
(c) ammonia and ammonium chloride in water are given in List-I. The effects of dilution of the solution on
(d) ammonia and sodium hydroxide in water [H + ] are given in List-II.
40. Which of the following statement(s) is (are) correct?
Note Degree of dissociation (α ) of weak acid and weak base is
(a) The pH of 1.0 × 10−8 M solution of HCl is 8 (1998, 2M) << 1; degree of hydrolysis of salt << 1; [H+ ] represents the
(b) The conjugate base of H2PO4− is HPO2−
4 concentration of H+ ions
(c) Autoprotolysis constant of water increases with temperature List-I List-II
(d) When a solution of a weak monoprotic acid is titrated against P. (10 mL of 0.1 M NaOH + 1. the value of [H+ ] does not
 1 20 mL of 0.1 M acetic
a strong base, at half-neutralisation point pH =   pK a change on dilution
 2 acid) diluted to 60 mL
Q. (20 mL of 0.1 M NaOH + 2. the value of [H+ ] changes
Numerical Answer Type Questions 20 mL of 0.1 M acetic to half of its initial value
acid) diluted to 80 mL on dilution
41. A solution of 0.1 M weak base (B) is titrated with 0.1 M of a
strong acid (HA). The variation of pH of the solution with the R. (20 mL of 0.1M HCl + 20 3. the value of [H+ ] changes
mL of 0.1 M ammonia to two times of its initial
volume of HA added is shown in the figure below. What is the
solution) diluted to 80 mL value on dilution.
pK b of the base? The neutralisation reaction is given by
S. 10 mL saturated solution 4. the value of [H+ ] changes
B + HA → BH+ + A − . (2020 Adv.) of Ni(OH)2 in equilibrium 1
to times of its initial
with exces solid Ni(OH)2 2
is diluted to 20 mL (solid value on dilution
12 Ni(OH)2 is still present
10 after dilution).
5. the value of [H+ ] changes
8
pH to 2 times of its initial
6 value on dilution
4
Match each process given in List-I with one or more effect(s)
2 in List-II. The correct option is (2018 Adv.)
(a) P → 4; Q → 2; R → 3; S → 1
0 2 4 6 8 10 12
(b) P → 4; Q → 3; R → 2; S → 3
Volume of HA (mL)
(c) P → 1; Q → 4; R → 5; S → 3
(d) P → 1; Q → 5; R → 4; S → 1
92 Chemical and Ionic Equilibrium

Fill in the Blanks (ii) If 6 g of NaOH is added to the above solution, determine the
final pH (assuming there is no change in volume on mixing,
46. In the reaction, I− + I2 → I3− , the Lewis acid is ............. K a of acetic acid is 1.75 × 10−5 mol/L. (1984, 1M)
(1997, 1M)
59. The average concentration of SO2 in the atmosphere over a
47. Silver chloride is sparingly soluble in water because its lattice
energy is greater than .............. energy. city on a certain day is 10 ppm, when the average temperature
(1987, 1M)
is 298 K. Given that the solubility of SO2 in water at 298 K is
48. An element which can exist as a positive ion in acidic solution 1.3653 mol/L and pK a of H2 SO3 is 1.92, estimate the pH of
and also as a negative ion in basic solution is said to rain on that day. (2000, 5M)
be................... (1984, 1M)
60. The solubility of Pb(OH)2 in water is 6.7 × 10−6 M. Calculate
49. The conjugate base of HSO–4 in aqueous solution is ……
the solubility of Pb(OH)2 in a buffer solution of pH = 8.
(1982, 1M)
(1999, 4M)
True/False
61. (a) Find the solubility product of a saturated solution of
50. The following species are in increasing order of their acidic Ag2CrO4 in water at 298 K if the emf of the cell
property : ZnO, Na 2 O2 , P2 O5 , MgO. (1985, 1/2M) Ag|Ag+ (saturated. Ag2CrO4 solution.) || Ag+ (0.1 M) | Ag is
0.164 V at 298 K. (1998, 6M)
51. Solubility of sodium hydroxide increases with increase in
temperature. (1985, 1/2M) (b) What will be the resultant pH when 200 mL of an aqueous
solution of HCl (pH = 2.0) is mixed with 300 mL of an
52. Aluminium chloride ( AlCl 3 ) is a Lewis acid because it can aqueous solution of NaOH (pH = 12.0) ? (1998, 6M)
donate electrons. (1982, 1M)
62. A sample of AgCl was treated with 5.00 mL of 1.5 M
Integer Answer Type Questions Na 2 CO3 solution to give Ag 2 CO3 . The remaining solution
53. The molar conductivity of a solution of a weak acid HX (0.01 contained 0.0026 g of Cl – ions per litre. Calculate the
M) is 10 times smaller than the molar conductivity of a solubility product of AgCl. [K sp (Ag 2 CO3 ) = 8.2 × 10−12 ]
solution of a weak acid HY (0.10 M). If λ 0X − ≈ λ 0Y − , the (1997, 5M)
difference in their pK a values, pK a (HX ) − pK a (HY ), is 63. An acid type indicator, HIn differs in colour from its
(consider degree of ionisation of both acids to be <<1). conjugate base (In − ). The human eye is sensitive to colour
(2015 Adv.)
differences only when the ratio [ In – ] / [ HIn ] is greater than
54. In 1 L saturated solution of AgCl [K sp ( AgCl ) = 1.6 × 10−10 ], 10 or smaller than 0.1. What should be the minimum change
0.1 mole of CuCl [K sp ( CuCl )= 1.0 × 10−6 ] is added. The in the pH of the solution to observe a complete colour
change? ( K a = 1.0 × 10−5 )
resultant concentration of Ag + in the solution is 1.6 × 10− x . (1997, 2M)
The value of ‘x’ is (2011) 64. The ionisation constant of NH+4 in water is 5.6 × 10−10 at
55. Amongst the following, the total number of compounds 25° C. The rate constant for the reaction of NH+4 and OH− to
whose aqueous solution turns red litmus paper blue is form NH3 and H2 O at 25° C is 3.4 × 1010 L/mol/s. Calculate
KCN K 2SO4 (NH4 )2 C2O4 NaCl the rate constant per proton transfer from water to NH3 .
Zn(NO3 )2 FeCl 3 K 2CO3 NH4NO3 (1996, 3M)
LiCN (2010)
65. What is the pH of a 0.50 M aqueous NaCN solution?
Subjective Questions (pK b of CN− = 4.70). (1996, 2M)

56. The dissociation constant of a substituted benzoic acid at 66. Calculate the pH of an aqueous solution of 1.0 M ammonium
25°C is 1.0 × 10− 4 . The pH of 0.01 M solution of its sodium formate assuming complete dissociation.
salt is (2009)
(pK a of formic acid = 3.8 and pK b of ammonia = 4.8)
(1995, 2M)
57. 0.1 M of HA is titrated with 0.1 M NaOH, calculate the pH at
– + −
end point. Given, K a (HA) = 5 × 10− 6 and α << 1. 67. For the reaction, [Ag(CN)2 ] r Ag + 2CN
(2004)
The equilibrium constant, at 25° C, is 4.0 × 10−19 . Calculate
58. 500 mL of 0.2 M aqueous solution of acetic acid is mixed
with 500 mL of 0.2 M HCl at 250°C. the silver ion concentration in a solution which was originally
0.10 M in KCN and 0.03 M in AgNO3 . (1994, 3M)
(i) Calculate the degree of dissociation of acetic acid in the
resulting solution and pH of the solution.
Chemical and Ionic Equilibrium 93

68. An aqueous solution of a metal bromide MBr2 (0.05 M) is 76. What is the pH of the solution when 0.20 mole of HCl is
saturated with H2 S. What is the minimum pH at which MS added to one litre of a solution containing
will precipitate? K sp for MS = 6.0 × 10−21 , concentration of (i) 1 M each of acetic acid and acetate ion,
−7 −13 (ii) 0.1 M each of acetic acid and acetate ion?
saturated H2 S = 0.1 M, K 1 = 10 and K 2 = 1.3 × 10 , for
Assume the total volume is one litre.
H2 S.
K a for acetic acid = 1.8 × 10−5.
(1993, 3M)
(1987, 5M)
69. The pH of blood stream is maintained by a proper balance of
H2 CO3 and NaHCO3 concentrations. What volume of 5 M 77. The solubility of Mg(OH)2 in pure water is 9.57 × 10−3 g / L.
NaHCO3 solution should be mixed with a 10 mL sample of Calculate its solubility (in g/L) in 0.02 M Mg(NO3 )2 solution.
blood which is 2 M in H2 CO3 , in order to maintain a pH of (1986, 5M)
7.4? (K a for H2 CO3 in blood is 7.8 × 10−7 ) (1993, 2M) 78. The concentration of hydrogen ions in a 0.20 M solution of
70. The solubility product ( K sp ) of Ca(OH)2 at 25° C is formic acid is 6.4 × 10−3 mol/L. To this solution, sodium
formate is added so as to adjust the concentration of sodium
4.42 × 10−5 . A 500 mL of saturated solution of Ca(OH)2 is
formate to one mole per litre.
mixed with equal volume of 0.4 M NaOH. How much
Ca(OH)2 in milligrams is precipitated? What will be the pH of this solution? The dissociation
(1992, 4M) constant of formic acid is 2.4 × 10−4 and the degree of
71. A 40 mL solution of a weak base, BOH is titrated with 0.1N dissociation of sodium formate is 0.75. (1985, 3M)
HCl solution. The pH of the solution is found to be 10.04 and
9.14 after the addition of 5.0 mL and 20.0 mL of the acid 79. A solution contains a mixture of Ag + (0.10 M) and
respectively. Find out the dissociation constant of the base. Hg 2+ (0.10 M) which are to be separated by selective
(1991, 6M) precipitation. Calculate the maximum concentration of
iodide ion at which one of them gets precipitated almost
72. The solubility product of Ag 2 C2 O4 at 25° C is completely. What percentage of that metal ion is
−11 3 −3
1.29 × 10 mol L . A solution of K 2 C2 O4 containing precipitated? (1984, 4M)
0.1520 mole in 500 mL water is shaken at 25° C with excess of K sp : AgI = 8.5 × 10−17 , HgI2 = 2.5 × 10−26
Ag 2 CO3 till the following equilibrium is reached
80. The dissociation constant of a weak acid HA is 4.9 × 10−8 .
Ag 2 CO3 + K 2 C2 O4 r Ag 2 C2 O4 + K 2 CO3
After making the necessary approximations, calculate
At equilibrium, the solution contains 0.0358 mole of (i) pH
K 2 CO3 . Assuming the degree of dissociation of K 2 C2 O4 and (ii) OH− concentration in a decimolar solution of the acid.
K 2 CO3 to be equal, calculate the solubility product of (Water has a pH of 7). (1983, 2M)
Ag 2 CO3 . (1991, 4M)
81. Give reason for the statement that “the pH of an aqueous
73. What is the pH of a 1.0 M solution of acetic acid? To what solution of sodium acetate is more than seven”. (1982, 1M)
volume must one litre of this solution be diluted so that the pH
of the resulting solution will be twice the original value? 82. 20 mL of 0.2 M sodium hydroxide is added to 50 mL of 0.2 M
Given, K a = 1.8 × 10−5 (1990, 4M)
acetic acid solution to give 70 mL of the solution. What is the
pH of this solution?
74. Freshly precipitated aluminium and magnesium hydroxides Calculate the additional volume of 0.2 M NaOH required to
are stirred vigorously in a buffer solution containing make the pH of the solution 4.74.
0.25 mol/L of NH4 Cl and 0.05 M of ammonium hydroxide. (Ionisation constant of CH3 COOH = 1.8 × 10−5 ). (1982, 3M)
Calculate the concentration of aluminium and magnesium
ions in solution. 83. How many moles of sodium propionate should be added to
K b [NH4 OH] = 1.8 × 10−5 1 L of an aqueous solution containing 0.020 mole of
K sp [Mg(OH)2 ] = 8.9 × 10 −12 propionic acid to obtain a buffer solution of pH 4.75? What
will be pH if 0.010 moles of HCl are dissolved in the above
K sp [Al(OH)3 ] = 6 × 10−32 (1989, 3M)
buffer solution? Compare the last pH value with the pH of
75. How many gram-mole of HCl will be required to prepare one 0.010 M HCl solution. Dissociation constant of propionic
litre of buffer solution (containing NaCN and HCl) of pH 8.5 acid, K a at 25°C is1.34 × 10−5 . (1981, 4M)
using 0.01 g formula weight of NaCN?
K HCN = 4.1 × 10−10 (1988, 4M)
Answers
Topic 1 13. (d) 14. (d) 15. (d) 16. (b)
1. (b) 2. (d) 3. (a) 4. (d) 17. (a) 18. (c) 19. (d) 20. (a)
5. (a) 6. (d) 7. (d) 8. (b) 21. (b) 22. (d) 23. (d) 24. (a)
9. (b) 10. (b) 11. (c) 12. (b) 25. (a) 26. (d) 27. (c) 28. (a)
13. (b) 14. (b) 15. (b) 16. (a) 29. (d) 30. (c) 31. (a) 32. (b)
17. (b) 18. (a) 19. (d) 20. (d) 33. (a) 34. (a) 35. (b) 36. (d)
21. (d) 22. (d) 23. (a) 24. (d) 37. (b) 38. (c, d) 39. (a, b, c) 40. (b, c)
25. (b) 26. (d) 27. (a) 28. (d) 41. (3.00) 42. (0.20) 43. (8.9) 44. (4.47)
29. (a,c) 30. (b) 31. (a) 32. (b, c, d) 45. (d) 46. I2 47. hydration 48. amphoteric
33. (c, d, e) 34. (d) 35. (c, d) 36. (a, b, c, d) 49. SO 2−
4 50. F 51. F 52. F
37. smaller 38. no change 39. K p = Kc ( RT ) ∆n 53. (3) 54. (1.6 × 10 −7) 56. (8) 57. (9)
40. T 41. F 42. F 43. T 58. (4.75) 59. (4.86) 60. (1.2 × 10 −3 M)
47. (1.2) 53. (1.86) 54. (0.33) 56. (b) 62. (2 × 10 −8) 65. (11.5) 66. (6.50) 68. (1)
57. (c) 69. (80) 71. (1.8 × 10 −5) 72. (9.67 ×
10 −11) 73. (27.78 × 10 3)
Topic 2
75. (0.177) 77. (8.7 × 10 −4 gL −1) 78. (4.20)
1. (a) 2. (d) 3. (c) 4. (c)
79. (99.83) 81. (>7)
5. (d) 6. (b) 7. (c) 8. (a)
9. (c) 10. (d) 11. (b) 12. (b)

Hints & Solutions


Topic 1 Chemical Equilibrium
1. N 2O 4 (s) r 2NO 2 (s); ∆H ° = + 58 kJ 3.
Because in case of an endothermic reaction (∆H = + ve), the Key Idea The relationship between K p and K c is
equilibrium constant increases with rise in temperature and ∆ng
K p = K c (RT )
hence, the reaction moves in forward direction.On adding N 2,
pressure is increased at constant T, and volume would also be where, ∆ng = nproducts − nreactants
constant, so no change is observed. If ∆ng = 0 then K p = K c
2. The incorrect match is ∆G ° < 0, K < 1. If ∆ng = + ve then K p > K c
If ∆ng = − ve then K p < K c
For an ideal gas ∆G ° = − RT ln K .
∆G °
∴ ln K = − and K = e− ∆G ° / RT Consider the following equilibria reactions
RT (a) 2C(s) + O2 (g ) - 2CO(g )
The above equation is helpful in predicting the spontaneity of the
∆ng = nproduct − nreactant = 2 − (1) = 1
reaction. e.g.
∆ng ≠ 0 ⇒ So, K p ≠ K c
(i) If ∆G ° < 0, – ∆G °/ RT = + ve and e− ∆G ° / RT > 1and hence,
(b) 2HI(g ) - H2(g ) + I2 (g )
K > 1. It means that the reaction occur spontaneously in the
forward direction or products predominate over reactants. ∆ng = nproduct − nreactant = 2 − 2 = 0
∆ng = 0 ⇒ So, K p = K c
(ii) If ∆G ° > 0; − ∆G °/ RT = − ve and
(c) NO2 (g ) + SO2 (g ) - NO(g ) + SO3 (g )
e− ∆G ° / RT < 1and hence, K < 1. It means that the reaction is
∆ng = nproduct − nreactant = 2 − 2 = 0
non-spontaneous in forward direction (i.e. product side) but ∆ng = 0 ⇒ So, K p = K c
spontaneous in reverse direction (i.e. reactants predominate
(d) 2NO(g ) - N2(g ) + O2 (g )
over products or the reaction occurs rarely).
∆ng = nproduct − nreactant = 2 − 2 = 0
(iii) When K = 1, then ∆G ° = 0. This situation generally occur at
equilibrium. ∆ng = 0 ⇒ So, K p = K c
Chemical and Ionic Equilibrium 95

4. The explanation of given statements are as follows: At equilibrium


(a) For the given equilibrium, ∆H is negative, so the equilibrium K p1 = x = pB ⋅ pC = p1( p1 + p2 ) …(i)
constant will decrease with increase in temperature and the Similarly, D(s) e C (g)+ E(g)
equilibrium will shift in the backward direction. At equilibrium p1 + p2 ⋅ p2
Thus, statement (a) is correct. K p 2 = y = pC ⋅ pE =( p1 + p2) p2 …(ii)
(b) When inert gas is added at constant volume and constant On adding Eq. (i) and (ii), we get.
temperature, an equilibrium remains undisturbed. K p1 + K p 2 = x + y = p1( p1 + p2)+ p2( p1 + p2)
Thus, statement (b) is correct. = ( p1 + p2 )2
(c) For the equilibrium,
or x + y = p1 + p2 …(iii)
∆ng = 2 − (2 + 1) = − 1, i.e. (−ve)
Now, total pressure is given as
So, increase in pressure will shift the equilibrium in the
forward direction. pT = pB + pC + pE
Thus, statement (c) is correct. = p1 + ( p1 + p2)+ p2 = 2 ( p1 + p2) …(iv)
(d) The reaction takes place in the presence of a catalyst which is On substituting the value of p1 + p2 from Eq. (iii) to Eq. (iv), we
V2O5 (s) in contact process or NO(g ) in chamber process. get
Thus, statement (d) is incorrect. pT = 2 x + y atm

5. S + O2 - SO2 , K 1 8. N 2 (g ) + 3H2 (g ) = 2NH3 (g )


1 At equilibrium: pN 2 = P, pH 2 = 3P, pNH 3 = 2P
∴ SO2 - S + O2 , K 1′ =
K1 ⇒ p(total ) = pN 2 + pH 2 + pNH 3 −~p + p
N2 H2
1 [Q P(total ) >> pNH 3 ]
or, 2SO2 - 2S + 2O2, K 1′′ = (K 1′ )2 = … (i)
K 12 = p + 3p = 4 p
2
⇒ 2S + 3O2 - 2SO3 , K 2 … (ii) p2NH 3 pNH
Now, Kp = = 3

Now, [(i) + (ii)] gives pN 2 × pH2 p × (3 p)3


3

2SO2 + O2 - 2SO3 , K 3 2
pNH 2
pNH
The value of equilibrium constant, = 3
4
= 3
4
[QP = 4 p]
27 × p  P
1 27 ×  
K 3 = K 2 × K 1′′ = K 2 × 2  4
K1
2
1 pNH × 44 2 32 × 3 × P 4 × K p
= 10129 × = 10129 − 104 = 1025 Kp = 2
3
4
⇒ pNH =
(1052 )2 3 ×3×P 3
44

6. For the given chemical reaction, 3 × 31/ 2 × P 2 × K p1/ 2 33/ 2 × P 2 × K p1/ 2


⇒ pNH 3 = =
K 42 16
A + 2B w 2C + D
−1
At, t = 0 a0 1.5a 0 0 0 9. Molar mass of NH4SH = 18 + 33 = 51g mol
t = t eq a0 − x 1.5a 0 − 2x 2x x
Number of moles of NH4SH introduced in the vessel
[x = degree of dissociation] Weight 5.1
Given, at equilibrium. = = = 01
. mol
Molar mass 51
[ A] =[ B]
a0 − x = 15
. a0 − 2x NH4 SH( s ) c NH3 ( g ) + H2 S( g )
Number of 0.1 0 0
x = 0.5a0 moles at t = 0
∴ [A] = a0 − x = a0 − 0.5a0 = 0.5a0 At t = t eq . (1 − 0. 03)
01 30% of 30% of 0.1
[ B] = 15
. a0 − 2x = 15. a0 − 2 × 0.5a0 = 0.5a0 01. = 0. 03 = 0. 03
Active mass 0. 03 0. 03
[C] = 2x = 2 × 0.5a0 = a0 = 0. 01 = 0. 01
(mol L −1) 3 3
[D] = x = 0.5a0
[ NH3 ][ H2S ] 0.01 × 0.01
[C ]2[ D] KC = = = 10−4 (mol L −1 2
)
Now, K= [ NH4HS(s) ] 1
[ A][ B]2
∆ng
Now, substituting the values in above equation, we get ⇒ K p = KC (RT )
(a0 )2 × (0.5a0 ) [where, ∆ng = Σnproduct − Σnreactant ]= 2 − 0 = 2
K= =4
(0.5a0 ) × (0.5a0 )2 ∴ K p = KC (RT )2
7. The equilibrium reaction for the dissociation of two solids is = 10− 4 × [0.082 × (273 + 327)]2 atm 2
given as: = 0.242 atm 2
A(s) e B(g)+ C (g)
p1 p1 + p 2
96 Chemical and Ionic Equilibrium

10. We know that, the relationship between K p and KC of a chemical 14. Given, ∆G ° = 2494.2 J
1
equilibrium state (reaction) is 2×
[ B ][C ] 2 =4
∆ng Kp ∆ng Q= =
K p = KC (RT ) ⇒ = (RT ) [ A ]2  1
2
KC  
 2
where, ∆ng = ΣnProducts − ΣnReactants
∴ We know, ∆G = ∆G °+ RT ln Q
(i) N2(g ) + O2(g ) c 2NO(g )
2 − (1 + 1) 0
= 2494.2 + 8.314 × 300 ln 4
⇒ (RT ) = (RT ) = 1
= 28747.27 J (+ ve value)
(ii) N2O4 (g ) c 2NO2 (g ) Q
⇒ (RT ) 2− 1
= RT = 24.62 dm3 atmmol−1 Also, we have ∆G = RT ln
K
(iii) N2 (g ) + 3H2 (g ) c 2NH3 (g ) If ∆G is positive, Q > KC
⇒ (RT )2− (3 + 1) = (RT )− 2 Therefore, reaction shifts in reverse direction.
1
= 15. For the given reaction, ∆ng = nP − nR
(24.62 dm atm mol −1 )2
3
where, nP = number of moles of products
= 1.649 × 10−3 dm –6 atm − 2 mol 2
n R = number of moles of reactants
[ AB ]2 ∆ng
11. (i) A2(g )+ B2(g ) c 2AB(g ); K1 = K p = K c ( RT )
[ A2 ][ B2 ]
(ii) 6 AB(g ) c 3 A2(g ) + 3 B2 (g ); 1
∆ng = −
[ A ] [ B2 ]3
3
1 1 2
K2 = 2 = = 3, 16. When CO2 is dissolved in water, following equilibria are established:
[ AB ]6  [ AB ]2 
3
K1
  H2O + CO2 r H2CO3
 [ A2 ][ B2 ]
⇒ K 2 = K 1−3 H2CO3 r H+ + HCO−3
12. Given [H2S] = 010
. M HCO−3 r H+ + CO23−
[HCl] = 0.20 M So, [H+ ] = 0.20 M Therefore, in solution, all of the above mentioned species exist.
H2S-H +
+ HS− , K 1 = 10
. × 10−7 17. At equilibrium, ∆G = 0

HS - H +
+ S2− , K 2 = 12
. × 10−13 G(reactants) = G (products)
It means for, G (N2 ) + 3G (H2 ) = 2G (NH3 )
H2S - 2H +
+ S2−
A catalyst does not affect either equilibrium composition or
. × 10−7 × 12
K = K 1 × K 2 = 10 . × 10−13 equilibrium constant, it just increases rate of both forward and
. × 10−20
= 12 backward reaction but by the same factor.
K × [ H2S]
Now [ S2− ] = [according to the final equation] 18. Ag+ + NH3 r [ Ag(NH3 )+ ] K 1 = 3.5 × 10− 3
[ H+ ]2
[Ag(NH3 )+ ] + NH3 r [ Ag(NH3 )+2 ] K 2 = 1.7 × 10− 3
. × 10−20 × 01
12 . M
=
(0.2M)2 Adding : Ag+ + 2NH3 r [ Ag(NH3 )2+ ]
. × 10−20 × 1 × 10−1 M
12 K = K 1 × K 2 = 5.95 × 10− 6
= −2
= 3 × 10−20 M
4 × 10 M
19. N2O4 (g ) r 2NO2 (g ) Total
13. A + B q C + D 1−α 2α 1 +α
Initially at t = 0 1 1 1 1 1−α 2α 4α 2
pi : p p Kp = p
At equilibrium 1−x 1−x 1+x 1+x 1+ α 1+ α 1 − α2
2
[C ][ D ] (1 + x)(1 + x) (1 + x)
K eq = = = At constant temperature, halving the volume will change both p
[ A ][ B ] (1 − x)(1 − x) (1 − x)2
and α but K p remains constant.
2
 1 + x 1+ x 4 x2 p
or 100 =   or 10 = 20. N2O4 r 2NO2, K p = . K p is function of temperature
 1 − x 1− x 2x 1 − x2
1− x
or 10 − 10x = 1 + x only, does not change with either p or x.
10 − 1 = x + 10x [C ][ D ]
9 = 11x 21. A + B r C + D, Q =
9 [ A ][ B ]
x= = 0.818
11 As time passes, amount of products ‘C’ and ‘D’ increases, hence
∴ [ D ] = 1 + x = 1 + 0.818 = 1818
. Q increases.
Chemical and Ionic Equilibrium 97

22. N2 (g ) + 3H2 (g ) r 2NH3 (g ) ∆n = − 2 [ H+ ][ A − ]


Ka =
K p = K c ( RT )∆n [ HA ]
(Rate)HA = k [ H+ ]HA
Kp 1.44 × 10− 5
Kc = =
(RT )∆n (0.082 × 773)− 2 (Rate) HX = k [ H+ ]HX
23. Both temperature and pressure will change the equilibrium (Rate) HX = 100(Rate)HA
amount of X 3Y (g ). Temperature changes the value of equilibrium +
∴ Also in strong acid, [ H ] = [ HX ] = 1M
constant.
(Rate) HX [ H + ] HX 1
24. Adding reactant will drive the reaction in forward direction in = 100 = =
order to restore equilibrium. Therefore, addition of CO (g) will (Rate) HA [ H + ] HA [ H+ ]HA
increase the equilibrium amount of CO2. 1
∴ [ H + ] HA =
25. N2O4 r 2NO2 100
At 300 K : 1.0 atm 0 HA r H+ + A −
At 600 K : 2.0 – 0.40 0.80 Total pressure = 2.40 atm 1 0 0
(1 − x) x x
26. In reactions (a), (b) and (c), atleast one of the product is either x = 0.01
insoluble precipitate or a gas that drive the reaction continuously [ H+ ][ A − ] 0.01 × 0.01
to right and do not allow equilibrium to be established. ∴ Ka = = = 1.01 × 10−4
[ HA ] 0.99
Following is the reversible reaction.
KNO3 (aq) + NaCl (aq) r KCl (aq) + NaNO3 (aq) 32. Cl − , CN − and SCN − forms precipitate with Cu (I), remove Cu (I)
ion from equilibrium and reaction shifts in backward direction
27. Kp for a given reversible reaction depends only on temperature. according to Le-Chatelier's principle.
28. Equilibrium constant of a given reversible reaction depends only 33. If inert gas is introduced at constant pressure, volume of
on temperature. container will have to be increased and this will favour the
29. For the reaction, A =P forward reaction. Also adding PCl 5 (g ) at constant volume will
favour forward reaction because PCl 5 (g ) is a reactant.
Given, T1 < T2
ln K 1 T2 34. SO2Cl 2 (g ) r SO2 (g ) + Cl 2 (g ),
> …(i)
ln K 2 T1 Adding inert gas at constant volume will not affect partial
pressure of reactant or products, hence will not affect
It shows, On increasing the temperature, K decreases so reaction
equilibrium amount of either reactant or products.
is exothermic i.e., ∆H o < 0
1
Besides, graph shows K >1 35. NaNO3 (s) r NaNO2 (s) + O2 (g ) , ∆H > 0
2
So ∆ G º< 0
NaNO3 and NaNO2 are in solid state, changing their amount has
Now from equation (i)
no effect on equilibrium. Increasing temperature will favour
T1 ln K 1 > T2 ln K 2 forward reaction due to endothermic nature of reaction. Also,
− ∆G º1> − ∆G º2 increasing pressure will favour backward reaction in which some
Likewise (− ∆H º+ T1∆S º ) > (− ∆H º+ T2 ∆S º ) O2 (g ) will combine with NaNO2 (s) forming NaNO3.
or simply T1∆S º> T2∆S º
36. C2H4 + H2 r C2H6, ∆H = − 32.7 kcal
So, (T2 − T1 ) ∆S ° < 0
The above reaction is exothermic, increasing temperature will
∴ ∆S º< 0 favour backward reaction, will increase the amount of C2H4.
In other words, increase of ∆G with increase in temperature is Decreasing pressure will favour reaction in direction containing
possible only when ∆S ° < 0.Hence, options (a) and (c) are correct. more molecules (reactant side in the present case). Therefore,
30. Since, the reaction is exothermic, there will be less ammonia at decreasing pressure will increase amount of C2H4.
equilibrium and higher temperature. However, rate of reaction Removing H2 , which is a reactant, will favour reaction in
increases with rise in temperature, NH3 will be formed at faster backward direction, more C2H4 will be formed.
rate in the initial stage when the temperature is high. Adding C2H6 will favour backward reaction and some of the
C2H6 will be dehydrogenated to C2H4.
H+
31. PLAN RCOOR′ + H 2O → RCOOH + R′ OH K
37. Smaller : K p = c
Acid hydrolysis of ester is follows first order kinetics. RT
For same concentration of ester in each case, rate is dependent on 38. changing pressure has no effect on equilibrium constant.
[ H+ ] from acid.
39. K p = K c (RT )∆n, where, ∆n = Σn (products) − Σn (reactants)
Rate = k [ RCOOR′ ]
Also for weak acid, HA r H+ + A − 40. Rate of any reaction increases on rising temperature.
41. Catalyst has no effect on thermodynamics of reaction.
98 Chemical and Ionic Equilibrium

1 47. Observing the graph indicates that when 0.20 mole of A is


42. It is .
K reacted, 0.40 mole of product is formed.
43. Evaporation is an endothermic process. A r nB
− 0.20 + 0. 40 ⇒ n=2
44. (a) N2O4 (g ) r 2NO2 (g )
At equilibrium, [A] = 0.30 M, [B] = 0.60 M
∆G ° = 2 ∆G f° (NO2 ) − ∆G f° (N2O4 ) = 0
[ B ] 2 0.36
Kc = = = 1.2
Also ∆G ° = − RT ln K = 0 , K = 1 [ A ] 0.30
Let the reaction shifts in forward direction.
48. CO (g ) + 2H2 (g ) r CH3OH (g )
N2O4 (g ) r 2NO2 (g ) Total
0.15 – 0.08 x − 0.16 0.08
5−x 5 + 2x 10 + x
5−x 5 + 2x Total moles at equilibrium = x − 0.01
pi : × 20 × 20
10 + x 10 + x 8.5 × 2.5
x − 0.01 = = 0.34 ⇒ x = 0.35
(5 + 2x )2 10 + x 0.082 × 750
⇒ K = × × 20 = 1
(10 + x )2 5−x 0.07
(i) Partial pressures : CO = × 8.5
⇒ 2
81x + 405x + 450 = 0 0.34
0.18
x = − 1.66 and – 3.33 H2 = × 8.5
0.34
Both values of x indicates that reaction actually proceeds in
backward direction. 0.08
CH3OH = × 8.5
 a  0.34
(b)  p +  (Vm − b) = RT 0.08  0.34 
2
 Vm2  Kp = ×  = 0.056
(0.07) (0.18) 2  8.5 
 ap2   pV 
p+   − b = RT 0.08
 ( pV )2   p  (ii) Concentrations : [CH3OH] = = 0.032 M
2.5
⇒[( pV 2 ) p + ap2 ][( pV ) − b ] = p ( pV )2 RT 0.18
[H2 ] = = 0.072 M
⇒ p ⋅ [ pV 2 + ap ] ( pV − bp) = p ( pV 2 ) RT 2.5
But p = 0 0.07
[CO] = = 0.028 M
Intercept = RT ⇒ ( pV )3 = ( pV )2 RT 2.5
0.032
Kc = = 213.33
45. (i) Mole of solid NH4HS taken initially = 3.06 = 0.06 (0.028) (0.072)2
51
At equilibrium NH4HS (s) r NH3 (g ) + H2S (g ) 49. CO (g ) + 2H2 (g ) r CH3OH (g )
0.018 0.018 Mole : 0.2 – 0.10 x − 0.20 0.10 ⇒ Total moles = x
 0.018
2
4.92 × 5
Kc =   = 8.1 × 10
−5 ⇒ x= = 0.5
 2  0.082 × 600
0.018 × 0.082 × 300 ⇒ moles of H2 at equilibrium = x − 0.2 = 0.3
p (NH3 ) = = 0.22 atm
2 0.1 0.3
Partial pressures : CO = p, H2 = p,
K p = (0.22)2 = 4.84 × 10− 2 0.5 0.5
0.1
(ii) Addition of solid NH4HS will have no effect on equilibrium. CH3OH = p
0.5
46. (a) PCl 5 (g ) r PCl 3 (g ) + Cl 2 (g ) Total moles p
1−α α α 1+ α 5 25 25
Kp = 2
= 2= = 0.11 atm − 2
208.5  p  3  9p 9 (4.92)2
Average molar mass = = 148.9    p
1.4  5  5 
1 × 148.9 0.1 0.3
ρ (density) =
pM
= Concentrations : [CO] = M , [H2] = M,
RT 0.082 × 400 5 5
0.1 (0.1/ 5)
[CH3OH] = M ⇒ KC = = 277.77 M −2.
= 4.54 g/L 5 (0.1/ 5) (0.3 / 5)2
(b) AgCl (s) + 2NH3 (aq) r [ Ag(NH3 )2+ ] + Cl −
−x 50. 2SO2 (g ) + O2 (g ) r SO3 (g )
1 − 2x x x
2 Initial pi : 0 2 1
K sp  x 
K = = 2.9 × 10− 3 =   Equilibrium pi : 2p 2+ p 1− 2p
Kc  1 − 2 x
(1 − 2 p)2
Kp = 900 = [Ignoring p in comparison to 2]
x = 0.049 M (2 + p) (2 p)2
Chemical and Ionic Equilibrium 99

1 [NH3 ]2 (0.50)2
p= atm Kc = = × 16
87 [N2 ][H2 ]3 (0.75) (2.25)3
2
Partial pressure of SO2 = 2 p = atm = 0.468 L2 mol − 2
87
1 3
1 175 Also for : N2 + H2 r NH3
Partial pressure of O2 = 2 + p = 2 + = atm 2 2
87 87
K ′c = K c = 0.68
 1  85
Partial pressure of SO3 = 1 − 2 p = 1 − 2   = atm
 87 87 56. X 2 (g ) 2 2 X (g )
51. N2O4 r 2NO2 Total At t=0 1 0
1−α 2α 1+ α  x
1−α 2α At equilibrium 1 −  (where, x = β eq )
x
pi : p p  2
1+ α 1+ α  x
1 − 
 x  2
4α 2 4 (0.25)2 Total moles = 1 +  and Mole fraction, X 2 (g ) =
Kp = p = = 0.26 atm  2  x
1 − α2 1 − (0.25)2 1 + 
   2
When p = 0.10 atm  x 
4α 2 (0.1) X (g ) =   and p = 2 bar
0.26 = ⇒ α = 0.62  1+ x 
1 − α2  2  x
 1− 
Partial pressure, pX 2 =  2  . p and p = p ⋅ x
52. SO2 (g ) + NO2 (g ) r SO3 (g ) + NO (g ) X
 1+ x   x
1− x 1− x x x
  1 + 
2  2
Qc = 1 < K c , i.e. reaction proceed in forward direction to attain 2
  x 
 px / 1 + 2 
equilibrium.
= 
2
 x  ∴ K p = pX2 / pX 2
16 =   ⇒ x = 0.80 (1 − x / 2)
 1 − x p
 x
1 + 
[NO] = 0.80 M, [NO2 ] = 0.20 M  2
53. A2 (g ) + B2 (g ) r 2 AB (g ) ∆n = 0 4 px 2 8 β 2eq
= =
(4 − x 2 ) (4 − β 2eq )
[ AB ]2 (n )2 (2x )2
K = = AB =
[ A2 ][ B2 ] nA2 ⋅ nB 2 (1 − x ) (2 − x ) 4 px 2
57. (a) Kp = = px 2 (Q 4 > > > x)
(4 − x 2 )
4 x2
⇒ 50 = ⇒ 23x 2 − 75x + 50 = 0 1
x 2 − 3x + 2 ∴ x∝
p
752 − 4 × 23 × 50
75 ± If p decreases, x increases. Equilibrium is shifted in the forward
⇒ x= = 0.93, 2.32 side. Thus, statement (a) is correct.
46
2.32 is not acceptable because x cannot be greater than 1. (b) At the start of the reaction, Q = 0 where, Q is the reaction
quotient ∆G = ∆G ° + 2.303RT log Q
Mole of AB = 2x = 2 × 0.93 = 1.86
Since, ∆G ° > 0, thus ∆G is −ve.
pV 2.05 × 100 Hence, dissociation takes place spontaneously.
54. Total moles of gases at equilibrium = = = 5.0
RT 0.082 × 500 Thus, (b) is correct.
Out of this 5 moles, 1.0 mole is for N2 (g ) and remaining 4 moles 4 × 2(0.7)2
(c) If we use x = 0.7 and p = 2 bar then K p =
for PCl 5 and its dissociation products. [ 4 − (0.7)2 ]
PCl 5 r PCl 3 + Cl 2 = 1.16 > 1
3−x x x Thus, (c) is incorrect.
3+ x = 4 ⇒ x =1 (d) At equilibrium, ∆G = 0
1 ∴ ∆G ° = − 2.303RT log K p
Degree of dissociation = = 0.33
3 Since, ∆G ° = + ve
55. N2 + 3H2 r 2NH3 Hence, K p < 1
Initial : 1.0 3.0 0 Kp
KC =
Equilibrium 1 – 0.25 3 – 0.75 0.05 (RT )
= 0.75 = 2.25
0.75 2.25 0.50 Then KC < 1. Thus, (d) is correct.
[N2 ] = , [H2 ] = , [NH3 ] =
4 4 4
100 Chemical and Ionic Equilibrium

Topic 2 Ionic Equilibrium Ksp = [Cd 2 + ][ OH− ]2

1. Given 100 mL of 0.1 M HCl is taken in beaker and to it 100 mL of . × 10− 5 )3


Ksp = (S )(2S )2 = 4 S 3 = 4 (184
0.1 M NaOH is added.
Ksp = 24.9 × 10− 15
This is acid (HCl) and base (NaOH) titration. Here,
phenolphthalein act as an indicator and colour change is pink. Ksp
[Cd 2 + ] =
The correct graph that depicts the change in pH is as follows. [ OH− ]2
At first HCl (acid) is take in beaker and base (NaOH) is taken in 24.9 × 10− 15
burette. [Cd 2 + ] = = 24.9 × 10− 15 × 10 + 4
(10− 2 )2
When base is added drop wise then acid-base reaction is occurs
and following changes are observed. ⇒ = 24.9 × 10− 11 M
B [Cd 2 + ] ⇒ 2.49 × 10− 10 M
The expected solubility of Cd(OH)2 in a buffer solution of
pH = 12 is 2.49 × 10− 10 M.
pH 7
3. Key Idea Concentration of substance in a saturated solution is
A
defined as its solubility (S). Its value depends upon the nature
Volume of NaOH of solvent and temperature. For reaction,
pH → 0 to 7 is acidic. AB -A +
+ B−
pH → 7 is neutral.
K sp = [ A ][ B − ]
+
pH → 7 to 14 is basic.
Initially, the graph increases steeply but when concentration of Al(OH)3 - Al 3+ + 3OH−
acid becomes equal to base then no change is seen in graph. Initially 1 0 0
This point is called neutral point or equivalence point. At equilibrium 1− S S 3S + 0.2
Equivalence point, where pH = 7.
After the equivalence point (pH = 7), base (NaOH) is NaOH → Na + + OH −
0. 2 0. 2
continuously mixed drop by drop then pH increases by 7 and
graph also increases slowly. At the point B, solution become
Ksp of Al(OH)3 = 2.4 × 10−24 (Given)
basic, again base (NaOH) is mixed drop by drop and the solution
becomes basic at point B. Following inference can be seen in the Ksp = [ Al3+ ][ OH− ]3
graphs.
2.4 × 10−24 = [ S ][ 3S + 0.2 ]3 [Q 0.2 >> S ]
(a) According to this graph, pH = 7 is equivalence point and use
−24
of NaOH (base) drop by drop leads to basic solution as a 2.4 × 10 = [ S ][ 0.008 ]
result, value of pH increases slowly. This is correct option.
[ S ] = 3 × 10−22
(b) According to graph, it shows straight line that mean graph
increases instantly but this titration show pH increases slowly Key Idea NH4Cl is a salt of weak base (NH4OH) and strong
4.
because base (NaOH) is mix drop by drop.
acid (HCl). On hydrolysis, NH4Cl will produce an acidic
(c) After equivalence point (pH = 7) graph is constant. It is solution (pH < 7) and the expression of pH of the solution is
wrong because graph increases and the solution becomes 1
pure basic then graph show constant value. pH = 7 − (pK b + logC )
2
(d) According to graph, equivalence point (pH = 7) graph should
Given, K b (NH4OH) = 10−5
increase but it decreases.
∴ pK b = − log K b = − log(10−5 ) = 5
2. Key Idea The concentration of substance in a saturated
C = concentration of salt solution = 0.02 M
solution is defined as its solubility (S). Its value depends
= 2 × 10−2 M
upon the nature of solvent and temperature.
1
Ax By - xA y + + yB x − Ksp = [ A y + ]x [ B x − ] y Now, pH = 7 − (pK b + logC )
2
. × 10− 5 M
Solubility of Cd (OH)2 (S ) = 184 On substituting the given values in above equation, we get
Given, pH = 12 [for Cd (OH)2 in buffer solution] 1 1
= 7 − [ 5 + log(2 × 10−2 )] = 7 − [ 5 + log 2 − 2 ]
So, pOH = 2 (QpH + pOH = pK w ) 2 2
12 + pOH = 14 1
= 7 − [ 5 + 0.301 − 2 ]= 7 − 165
. = 5.35
pOH = 14 − 12 = 2 2
∴ [ OH− ] = 10− 2 in buffer solution. 5. The explanation of given statements are as follows:
For reaction Cd (OH)2 → Cd 2 + + 2 OH− In statement (I), millimoles of H+ = 400 × 01. × 2 = 80
S2
Millimoles of OH− = 400 × 01
S
. = 40 (Limiting reagent)
Chemical and Ionic Equilibrium 101

∴ Millimoles of H+ left = 80 − 40 = 40 Thus, the relation between molar solubility(S) and solubility
40 40 1 product (K sp ) will be
[ H+ ] = = M = M 1/ 7
400 + 400 800 20  K sp 
S = 
 1  6912
⇒ pH = − log[ H+ ] = − log 
 20
7. Let the solubility of Ag2CO3 is S. Now, 0.1 M of AgNO3 is added
= − log1 + log 2 + log10 = −0 + 0.301 + 1
to this solution after which let the solubility of Ag2CO3 becomes
⇒ 1.30
S′.
Hence, the option (a) is correct.
∴ [Ag+ ] = S + 0.1 and [CO2− 3 ] = S′
In statement (II), ionic product of H2O is temperature dependent.
K sp = (S + 0.1)2 (S ′ ) ...(i)
K w = [ H+ ][ OH− ] ≈ 10−14 (mol / L)2 at 25ºC
− 12
Given, K sp = 8 × 10
With increase in temperature, dissociation of H2O units into H+
and OH− ions will also increase. As a result, the value of ionic Q K sp is very small, we neglect S′ against S in Eq. (i)
product, [H+ ] × [ OH− ]will be increased. e.g. ∴ . )2 S ′
K sp = (01
or 8 × 10− 12 = 0.01 S ′
Temperature K w (mol/L2)
or S′ = 8 × 10− 12 × 102 = 8 × 10− 10 M
5ºC 0186
. × 10−14
Thus, molar solubility of Ag2CO3 in 0.1 M
25ºC 1008
. × 10−14
AgNO3 is 8 × 10− 10 M.
45ºC 4.074 × 10−14
8. The reaction takes place when H2SO4 is added to NH4OH is as
Hence, the option (b) is correct. follows :
In statement (III), for a weak monobasic acid HA H2SO4 + 2NH4OH → (NH4 )2 SO4 + 2H2O
HA H⊕ + A s
- Strong acid Weak base Salt of strong acid
+ weak base
(1 − α) C M αC M αC M
Millimoles at t = 0 20 × 0.1 = 2 30 × 0.2 = 6 0
⇒ pH of the solution is 5, i.e. Millimoles at t = t 0 2 2
[H+ ] = 10−5 M = αC So, the resulting solution is a basic buffer
αC × αC 10 × α −5 [NH4OH + (NH4 )2 SO4 ].
⇒ Ka = = According to the Henderson’s equation,
(1 − α )C 1− α
[(NH4 )2SO4 ]
10−5 × α pOH = pK b + log
⇒ 10−5 = [NH4OH]
1− α
2
⇒ α = 0.5 ⇒ α % = 50 = 4.7 + log = 4.7
2
Hence, the option (c) is correct.
⇒ pH = 14 − pOH = 14 − 4.7 = 9.3
In statement (IV), Le-Chatelier’s principle is applicable to
common ion effect. Because, in presence of common ion (given) 9. Its given that the final volume is 500 mL and this final volume
+ −
was arrived when 50 mL of 1 M Na 2SO4 was added to unknown
by strong electrolyte (say, Na A), the product of the Ba 2+ solution.
concentration terms in RHS increases. For the weaker So, we can interpret the volume of unknown Ba 2+ solution as
electrolyte, HA (say) the equilibrium shifts to the LHS, 450 mL i.e.
HA - H⊕ + A s. 450mL + 50mL → 500mL
As a result dissociation of HA gets suppressed. Hence, the option Ba 2+ Na 2SO 4 BaSO 4
solution solution solution
(d) is incorrect.
From this we can calculate the concentration of SO2−
4 ion in the

6. Key Idea The concentration of a substance in a saturated solution via


solution is defined as its solubility(S). M 1V1 = M 2V2
1 × 50 = M 2 × 500
For Ax B y - xA y+ + yB x− ; K sp = [A y+ ] x [B x− ] y
(as 1M Na 2SO4 is taken into consideration)
For, Zr3 (PO4 )4, 1
M2 = = 01
. M
Zr3 (PO4 )4 (s) -3Zr 4+ (aq) + 4 PO34− (aq) 10
3S M 4S M Now for just precipitation,
K sp = [ Zr 4+ 3
] [ PO34− ]4 Ionic product = Solubility product (K sp )
1
i.e. [Ba 2+ ][SO24− ] = K sp of BaSO4
3 4  K sp  7
7 Given K sp of BaSO4 = 1 × 10−10
K sp = (3S ) (4 S ) = 6912 S or S =  
 6912
So, [Ba 2+ ][0.1] = 1 × 10−10 or [Ba 2+ ] = 1 × 10−9 M
102 Chemical and Ionic Equilibrium

Remember This is the concentration of Ba 2+ ions in final CH 3COOK is the salt of strong base and weak acid.
solution. Hence, for calculating the [Ba 2+ ] in original solution Hence, the solution of CH3COOK will be most basic because of
we have to use the following reaction.
M 1V1 = M 2V2 CH3COOK + H2O CH3COOH + KOH
- (Strong base)
M 1 × 450 = 10−9 × 500
(Weak acid)
as
12. For a salt of weak acid and weak base,
so, M 1 = 1.1 × 10−9 M
1 1
pH = 7 + pKa − pKb
10. Key Idea Lewis acids are defined as, 2 2
‘‘Electron deficient compounds which have the ability to accept Given, pK a (HA ) = 3.2, pK a ( BOH) = 3.4
atleast one lone pair.’’
1 1
The compound given are ∴ pH = 7 + (3.2) − (3.4) = 7 + 1.6 − 1.7 = 6. 9
2 2
PH 3-Octet complete although P has vacant 3d-orbital but does
not have the tendency to accept lone pair in it. Hence, it cannot 13. pH = 1 ∴ [ H+ ] = 10−1 = 0.1 M
be considered as Lewis acid. pH = 2 ∴ [ H+ ] = 10−2 = 0.01 M
BCl 3-Incomplete octet with following orbital picture. For dilution of HCl, M 1V1 = M 2V2
1s 2s 2p Vacant 0.1 × 1 = 0.01 × V2
p- orbital
B-
V2 = 10 L
Volume of water to be added = 10 − 1= 9 L
Used in bond 14. MX : K sp = S 2 = 4 × 10− 8 ⇒ S = 2 × 10− 4
formation with
Cl having one electron
each from B and Cl MX 2 : K sp = 4 S 3 = 3.2 × 10− 14 ⇒ S = 2 × 10− 5
Hence, vacant p-orbital of B can accept one lone pair thus it can M3X : K sp = 27S 4 = 2.7 × 10− 15 ⇒ S = 10− 4
be considered as Lewis acid.
MX > M 3 X > MX 2
Order of solubility is
AlCl 3-Similar condition is visible in AlCl3 as well i.e.
2
Al ( Valence orbital only) = 15. mmol of base = 2.5 × = 1
5
Vacant p-orbital
3s 3p 3d vacant mmol of acid required to reach the end point = 1
15
Volume of acid required to reach the end point = mL
2
Used in bond 15
formation with Cl Total volume at the end point = + 2.5 = 10 mL
2
Hence this compound can also be considered as Lewis acid. 1
SiCl 4 - Although this compound does not have incomplete octet Molarity of salt at the end point = = 0.10
10
but it shows the tendency to accept lone pair of electrons in its
vacant d-orbital. This tendency of SiCl 4 is visible in following B+ + H2O r BOH + H+
C (1 − α ) Cα Cα
reaction.
Kw
Cl Cl Kh = = 10−2
H Kb
Si + H 2O Si O Cα 2 0.1 α 2
H K h = 10− 2 = =
Cl Cl Cl Cl 1−α 1−α
Cl Cl
⇒ 10α 2 + α − 1 = 0
Lone pair acceptance
in d-orbital − 1 + 1 + 40
⇒ α= = 0.27
Cl 20
+
⇒ [H ] = Cα = 0.1 × 0.27 = 0.027 M
Si OH + HCl
Cl 16. CH3NH2 + HCl → CH3NH3+ + Cl −
Cl
Initial : 0.10 0.08 0 0
Thus, option (b) and (d) both appear as correct but most suitable Final : 0.02 0 0.08 0.08
answer is (d) as the condition of a proper Lewis acid is more well [CH3NH+3 ]
defined in BCl 3 and AlCl 3. pOH = pK b + log
[CH3NH2 ]
11. Among the given salts 0.08
FeCl 3 is acidic in nature i.e., have acidic solution as it is the salt = − log (5 × 10− 4 ) + log = 3.9
0.02
of weak base and strong acid.
pH = 14 – pOH = 10.1
Al(CN) 3 and Pb(CH 3COO) 2 are the salts of weak acid and weak
[H+ ] = 8 × 10−11
base.
Chemical and Ionic Equilibrium 103

K w 10− 14 Kh 10− 9 29. When a weak acid (HX) is titrated against a strong base NaOH,
17. K h ( X − ) = = − 5 = 10− 9 ⇒α = = = 10− 4 basic salt (NaX) is present at the end point which makes end
Ka 10 C 0.10
point slightly basic with pH around 8. Hence,
% hydrolysis = 100 α = 0.01 phenolphthalein, that changes its colour in this pH range,
would be the best choice of indicator to detect the end point.
18. Minimum S2− concentration would be required for precipitation
of least soluble HgS. 30. The reaction of HA with strong base is
2−
For HgS, S required for precipitation is HA + OH− r H2O + A −
K sp 10−54 [ A− ] [H+ ] K 10− 4
[S2− ] = 2+
= −3 = 10−51 M K = × + = a = − 14 = 1010
[Hg ] 10 −
[HA ][OH ] [H ] K w 10
19. Alkali metal salts are usually more soluble than the salts of Kw
31. K a (HX ) = = 10− 4
transition metals. Also, CuS is less soluble than ZnS because of Kb
3d 9 configuration of Cu 2+ . Therefore, solubility order is [X −]
pH = pK a + log ⇒ pK a = 4 [Q [ X − ] = [HX ]]
Na 2S > ZnS > CuS [HX ]
20. Ap Bq r pA + qB 32. For precipitation reaction, QIP > K sp .
pS qS
2
 10− 2   10− 3 
K sp = ( pS )p (qS )q = S (p + q) ⋅ pp ⋅ qq QIP = [Ca 2+ ][F− ]2 =   × 
 2   2 
21. NaCN is basic salt, has highest pH while HCl has lowest pH. = 1.25 × 10− 9 > K sp , precipitate will be formed.
NaCl is neutral salt has pH = 7 while NH4Cl is acidic salt, has pH
less than 7. 33. Acidic buffer is prepared by mixing weak acid with salt of its
pH : HCl < NH4Cl < NaCl < NaCN
conjugate base. Therefore, acetic acid and sodium acetate can be
M used to prepare acidic buffer.
22. 75 mL HCl = 15 mmol HCl
5
34. The order of acidic strength of conjugate acids is
M
25 mL NaOH = 5 mmol NaOH HOCl < HClO2 < HClO3 < HClO4
5
Reverse is the order of basic strength of their conjugate base, i.e.
After neutralisation, 10 mmol HCl will be remaining in 100 mL
of solution. ClO− is the strongest base.
10 35. K w = [H3O+ ][OH− ] = 10− 6 × 10− 6 = 10− 12
Molarity of HCl in the final solution = = 0.10
100
36. No matter, what is the concentration of HCl, its pH will always
pH = − log [H+ ] = − log (0.10) = 1 be less than 7 at 25°C. In the present case, the solution is very
dilute, pH will be between 6 and 7.
23. In case of hydroxides of Group II A, solubility increases down
the group. Therefore, Be(OH)2 is least soluble, has lowest value 37. PLAN In presence of common ion (in this case Ag + ion) solubility of
of K sp . sparingly soluble salt is decreased.
Let solubility of Ag2CrO4 in presence of 0.1 M
24. HClO4 is the strongest acid among these.
AgNO 3 = x
25. For precipitation to occur, K sp < Qsp .
Ag2CrO4 a 2 Ag+ + CrO24−
 10− 4   10− 4  2x x
Qsp =     = 2.5 × 10− 9 > K sp
 2   2  AgNO3 a Ag+ + NO3−
0.1 0.1
Hence, precipitate will be formed in this case. In all other case, +
Total [ Ag ] = (2x + 0.1) M ≈ 0.1 M
Qsp < K sp and no precipitation will occur.
as x <<< 0.1 M
26. In stomach, pH is 2-3, i.e. strongly acidic and aspirin will be [CrO 2−
4 ]= xM
almost unionised here due to common ion effect. However, pH in
Thus, [Ag+ ]2 [CrO24− ] = K sp
small intestine is 8, basic, aspirin will be neutralised here.
(0.1)2 (x ) = 1.1 × 10−12
27. BeCl 2 exist in polymeric forms and has no electron deficiency,
not a Lewis acid. Q x = 1.1 × 10−10 M
Cl Cl Cl 38. In HNO3 and CH3COONa combination, if HNO3 is present in
Be Be Be
Cl Cl Cl limiting amount, it will be neutralised completely, leaving
behind some excess of CH3COONa.
28. NH2− + H2O r NH3 + OH− CH3COONa + HNO3 → CH3COOH + NaNO3
Base Conjugate
acid
Buffer combination
104 Chemical and Ionic Equilibrium

1
39. CH3COOH + CH3COONa = Buffer solution KC = [For FeS(s) = 1mol L −1]
X × 0.07
CH3COONa + HCl → CH3COOH + NaCl (Pure solid)

1
If HCl is taken in limited quantity, final solution will have both . × 1017 =
16
CH3COOH and CH3COONa needed for buffer solution. X × 0.07
1
Ammonia and ammonium chloride forms basic buffer. X = = 8.9 × 10−17
. × 1017 × 0.07
16
40. pH of 10− 8 M solution will be between 6 and 7 but never 8. The
Given, X = Y × 10−17 = 8.9 × 10−17
conjugate base of an acid is formed by removing a proton (H+ )

∴ Y = 8.9
from acid. Therefore, HPO2−
4 is a conjugate base of H 2PO 4 .
44. Key Idea Solubility of salt of weak acid (AB) in presence
H2O r H+ + OH− ∆H > 0 of H + ions from buffer solution can be calculated with the
Increasing temperature will increase equilibrium constant of the help of following formula.
above endothermic reaction. [H+ ] 
Solubility = K sp  + 1
At the mid-point of titration pH = pK a  ka 
41. From the given diagram, 6 mL volume of HA used till equivalene Given, pH = 3, so [H+ ] = 10−3
point. At half of equivalence point, solution will be basic buffer
with B and BH+ . K a = 1× 10−8 ⇒ Ksp = 2 × 10−10
[ BH+ ] after putting the values in above formula
Q pOH = pK b + log
[B]  10−3 
At half equivalence point :[ BH+ ] = [ B ] (QpH = 11) Solubility = 2 × 10−10  −8 + 1 ≈ 2 × 10−5 = 4.47 × 10−3M
 10 
Therefore, pOH = pK b = 14 − 11 = 3
Hence, the value of y= 4.47
Q pK b = 3.00
45. For P, i.e. (10 mL of 0.1 M NaOH + 20 mL of 0.1 M acetic acid)
42. ZnS(s) → Zn2+ (aq) + S2− (aq) is diluted to 60 mL
(0.05 M)
The correct match is 1, i.e. the value of[H+] does not change on
K sp (ZnS ) = [ Zn 2+ ][ S2− ] = 125
. × 10−22 dilution due to the formation of following buffer.

0.05 × [ S2− ] = 125
. × 10−22 NaOH + CH 3COOH - CH COO Na
3
+
+ H 2O
−22
. × 10
125 Initial millimol 1 2
⇒ [ S2− ]= ⇒ 25 × 10−22 M
0.05 Final millimol 1 1
For H2S , H2S → 2H+ + S −2 Final volume – 30 mL (20 + 10) in which millimoles of
(0. 1 M) (25 × 10−22 M) CH3COOH and CH3COO −Na + are counted.
[ H+ ]2 [ S2 − ] For Q, i.e. (20 mL of 0.1 M NaOH + 20 mL of 0.1 M
K net = 1 × 10−21 =
[ H2S] CH3COOH) is diluted to 80 mL
[ H+ ]2 × 25 × 10−22 The correct match is 5, i.e. the value of [H+] changes to
1 × 10−21 =
[ 01
. ] 2 times of its initial value on dilution.
1 As per the condition given in Q the resultant solution before
[ H+ ]2 =
25 dilution contain 2 millimoles of CH3COO −Na+ in 40 mL
1 solution. Hence, it is the salt of weak acid and strong base. So,
[ H+ ] = ⇒0.2 M
5 KW K a
[H + ]initial =
. × 1017
43. Given, equilibrium constant (KC ) at 298 K = 16 C
C
Fe2+ (aq) + S2− (aq) 1 FeS (s) After dilution to 80 mL, the new ‘C ’ becomes , So,
At initial concentration 0.06 M 0.2 M – 2
(Before mixing) K wK a
At initial concentration 0.03 M 0.1 M – [H ]new =
+
or [H ] initial × 2
+
(After mixing) C /2
At equilibrium 0.03- X 0.1 – 0.03 = – For R, i.e. (20 mL of 0.1 M HCl + 20 mL of 0.1 M NH 3) is
0.07
3
[Here, KC >> 10 , thus limiting reagent will be consumed almost diluted to 80 mL
completely, 0.03 − X = 0 ∴ X = 0.03] 1
The correct match is 4, i.e. the value of [H+] changes to times
From equilibrium constant, 2
[FeS] of its initial value of dilution.
KC =
[Fe2+][S2− ]
Chemical and Ionic Equilibrium 105

As per the condition given in R the resultant solution before [ Cl – ] = K sp (CuCl) = 10–3 M
dilution contains 2 millimoles of NH4Cl in 40 mL of solution.
Hence, a salt of strong acid and weak base is formed. Now, for AgCl, K sp = 1.6 × 10–10 = [Ag+] [Cl – ]
For this,
= [Ag+] × 10–3
Kw × C
[H+]initial = ⇒ [Ag ] = 1.6 × 10–7
+
Kb
Now on dilution upto 80 mL new conc. becomes C /2.
55. Basic salts solution will have pH > 7, will change colour of
litmus paper red to blue
C
Kw × KCN, K2CO3 and LiCN are the only basic salts among these.
So, [H ]new =
+ 2
Kb 56. The hydrolysis reaction is
or [H+]new = [H + ]initial ×
1 A − + H2O r AH + OH−
2 Kw
For S , i.e. 10 mL saturated solution of Ni(OH)2 in Kh = = 10− 10
Ka
equilibrium with excess solidNi(OH)2 is diluted to 20 mL and
[OH− ] = K hC = 10− 6
solid Ni(OH)2 is still present after dilution.
The correct match is 1. pOH = 6 and pH = 8
2+ − −
Ni(OH)2 (s) - Ni + 2OH 57. At the end-point, [ A ] = 0.05
as per the condition given it is a sparingly soluble salt. Hence, on K b ( A − ) = K w / K a = 2 × 10− 9
dilution the concentration of OH− ions remains constant in
saturated solution. [OH− ] = K bC = 2 × 10− 9 × 0.05 = 10− 5
So for this solution,
pOH = 5 and pH = 9
[H+]new =[H+]initial

58. (i) CH3COOH r CH3COO + H+

46. I2 : I → I2 = I3−
C (1 − α ) Cα Cα
47. Hydration energy facilitate solubility. If no HCl is present,
48. Amphoteric 0.2
[HCl] == 0.10 M
49. SO2– 2
4 Conjugate base is formed by removing a proton from acid.
[CH3COOH] = 0.10 M
50. P2O5 is strongest acid and MgO is strongest base.
The major contributor of H+ in solution is HCl.
51. NaOH + H2O → NaOH (aq); ∆H < 0 C α (0.1)
Ka = = 1.75 × 10− 5
52. Lewis acid accept lone pair of electron. C (1− α )
Λm α = 1.75 × 10− 4
53. Degree of ionisation (α) =
Λ∞ 6
(ii) mmol of NaOH added = × 1000 = 150
X 40
Let Λ m (HY ) = x ⇒ Λ m (HX ) =
10 mmol of HCl = 500 × 0.2 = 100
Λ m (HX ) 1 α (HX ) mmol of CH3COOH = 500 × 0.2 = 100
⇒ = = [QΛ∞ (HX ) = Λ∞ (HY )]
Λ m ( XY ) 10 α (HY ) After neutralisation, mmol of CH3COOH = 50
Also : K a(HX ) = (0.01) [α( HX )]2 ...(i) mmol of CH3COONa = 50
K a (HY ) = (010
. ) [α( HY )]2 pH = pK a = 4.75

= 010
. [10 α (HX )]2 = 10 [α (HX )]2 ... (ii) 59. Partial pressure of SO2 in air = 10− 5 atm
[SO2 ]aq = 1.3653 × 10− 5 mol L− 1
K a (HX ) 0.01 1
⇒ = =
K a (HY ) 10 1000 QpK a = 1.92and concentration of H2SO3 is very low, it is almost
completely ionised as
⇒ log K a (HX ) − log K a (HY ) = − 3
H2SO3 r H+ + HSO−3
⇒ − log K a (HX ) − [ − log K a (HY )] = 3
[H+ ] = 1.3653 × 10− 5 M
⇒ pK a (HX ) − pK a (HY ) = 3
pH = – log (1.3653 × 10− 5 ) = 4.86
54. It is a case of simultaneous solubility of salts with a common ion.
Here, solubility product of CuCl is much greater than that of 60. In water, K sp = 4 S 3 = 4 (6.7 × 10− 6 )3 = 1.2 × 10− 15
AgCl, it can be assumed that Cl– in solution comes mainly from In buffer of pH = 8, pOH = 6, [OH− ] = 10−6
CuCl.
106 Chemical and Ionic Equilibrium

K sp = S [OH− ] 2 64. K a (NH+4 ) = 5.6 × 10− 10


1.2 × 10− 15 10− 14
S = = 1.2 × 10− 3 M K b (NH3 ) = K w / K a = = 1.8 × 10− 5
10− 12 5.6 × 10− 10
k1
[Ag+ ] anode i.e. NH3 + H2O r NH4+ + OH−
61. (a) E = 0.164 = – 0.059 log k2
0.10
−4 k
K = 1 = 1.8 × 10− 5
+
[Ag ] anode = 1.66 × 10 M
k2
[Ag+ ]
[CrO24− ] = = 8.3 × 10− 5 M k 1 = Kk 2 = 1.8 × 10−5 × 3.4 × 1010 = 6.12 × 105
2
K sp = [Ag+ ]2 [CrO24− ]
65. CN− + H2O r HCN + OH−
K h = 2 × 10− 5
= (1.66 × 10− 4 )2 (8.3 × 10− 5 ) = 2.3 × 10− 12
[OH− ] = K hC = 2 × 10− 5 × 0.5 = 10− 5
(b) pH of HCl = 2
∴ [ H+ ] = 10−2 M pOH = 2.5 and pH = 11.5
+ −2
Moles of H ions in 200 mL of 10 M HCl solution 66. For salts of weak acid and weak base .
10−2 1 1
= × 200 = 2 × 10−3 pH = 7 + (pK a − pK b ) = 7 + (3.8 − 4.8) = 6.50
1000 2 2
Similarly, pH of NaOH = 12 67. Ag+ + 2CN− r Ag(CN)−2
∴ [ H+ ] = 10−12 M Initial : 0.03 0.10 0
− −2 + − −14 Equilibrium : x 0.10 – 0.06 0.03
or [OH ] = 10 M [Q[H ][ OH ] = 10 m]
1

Moles of OH ion in 300 mL of 10 M NaOH solution −2 K = = 2.5 × 1018
4 × 10− 19
10−2
= × 300 = 3 × 10−3 0.03
1000 ⇒ K = 2.5 × 1018 =
(0.04)2 x
Total volume of solution after mixing = 500 mL
x = 7.50 × 10− 18 M Ag+
Moles of OH− ion left in 500 mL of solution
= (3 × 10−3 ) − (2 × 10−3 ) = 10−3 68. For H2S, H2S r 2H+ + S2−
Molar concentration of OH− ions in the resulting K = K 1 × K 2 = 1.3 × 10− 20
10−3 Minimum [S2− ] required to begin precipitation of
solution = × 1000 = 2 × 10−3 M
500
6 × 10− 21
pOH = − log (2 × 10−3 ) MS = = 1.2 × 10− 19
0.05
~ 103 = 2.699
= − log 2 + 3 log10 = − 0.3 − [H+ ]2 [S2− ] (1.2 × 10− 19 )
∴ pH = 14 − 2.699 = 11.301 K = 1.3 × 10− 20 = = [H+ ] 2
[H2S] 0.10
62. 2AgCl (s) + CO23− r Ag2CO3 (s) + 2Cl − [H+ ] = 0.10 M ⇒ pH = 1
− 2 − 2 + 2 2
[Cl ] [Cl ] [Ag ] [ K sp (AgCl)]
K = = × = 69. Mixing H2CO3 with NaHCO3 results in buffer solution.
[CO32− ] [CO23− ] [Ag+ ]2 K sp (Ag2CO3 )
[NaHCO3 ] n (NaHCO3 )
0.0026 pH = pK a + log = pK a + log

[Cl ] = M = 7.3 × 10− 5 M [H2CO3 ] n (H2CO3 )
35.5
x
The above concentration of Cl − indicates that [CO2−
3 ] remains
⇒ 7.4 = – log (7.8 × 10− 7 ) + log
20
almost unchanged.
2 ⇒ x = 400 mmol
7.3 × 10− 5 [K sp (AgCl)]
= − 12 NaHCO3 = 5 × V ⇒ V = 80 mL
1.5 8.2 × 10
3 −5
K sp (AgCl) = 2 × 10− 8
70. K sp = 4 S = 4.42 × 10
S = 0.022 M
[In − ]
63. pH = pK In + log 10 = pK In + 1 When = 10 mmol of Ca(OH)2 in 500 mL saturated solution = 11
[HIn]
mmol of NaOH in 500 mL 0.40 M solution = 200
[In − ]
= pK In + log (0.1) = pK In −1 When = 0.1 Total mmol of OH− = 200 + 2 × 11 = 222
[HIn]
pH range is pK In to pK In [OH− ] = 0.222 M
−1 + 1.
Chemical and Ionic Equilibrium 107

K sp Also, desired pH = 2 × 2.37 = 4.74


Solubility in presence of NaOH =
[OH− ]2 [H+ ] = 1.8 × 10− 5 = x α
4.42 × 10− 5 1.8 × 10− 5 α
= = 9 × 10− 4 M K a = 1.8 × 10− 5 =
(0.222)2 1−α
mmol of Ca 2+ remaining in solution = 0.9 α = 0.5 and x = 3.6 × 10− 5 M
mmol of Ca(OH)2 precipitated = 10.1 Volume (final) = 1/3.6 × 10− 5 = 27.78 × 103 L.
mg of Ca(OH)2 precipitated = 10.1 × 7.4 = 747.4 mg [NH+4 ]
74. pOH of buffer solution = pK b + log
71. Let 40 mL of base contain x mmol of BOH. [NH4OH]
BOH + HCl → BCl + H2O 0.25
= − log (1.8 × 10− 5 ) + log = 5.44
x − 0.5 0.5 When 5 mL acid is added 0.05
x− 2 2.0 When 20 mL of acid is added
[OH− ] = 3.6 × 10− 6 M
When pH is 10.04, pOH = 3.96 and when pH is 9.14, pOH is
4.86. Therefore, K sp 6 × 10− 32
[Al 3+ ] = − 3
= = 1.28 × 10− 15 M
0.50 [OH ] (3.6 × 10− 6 )3
3.96 = pK b + log …(i)
x − 0.5 K sp 8.9 × 10− 12
[Mg2+ ] = − 2
= = 0.68 M
3.96 = pK b + log
2.0
…(ii) [OH ] (3.6 × 10− 6 )2
x− 2
75. HCN for buffer will be formed by the reaction
Subtracting Eq. (i) from Eq. (ii) gives
NaCN + HCl → NaCl + HCN
 2 x − 0.5 4 (x − 0.5)
0.90 = log  ×  ⇒ 28 = mmol of NaCN present initially =
0.01
× 1000 = 0.2
x − 2 0.5  x−2 49
⇒ x = 3.5, substituting in equation (i) gives Let x mmol of HCl is added so that x mmol of NaCN will be
0.5 neutralised forming x mmol of HCN.
3.96 = pK b + log
3 [NaCN]
pH = pK a + log
K b = 1.8 × 10− 5 [HCN]
0.152 0.2 − x
72. Initial concentration of K2C2O4 = = 0.304 M, 8.5 = − log (4.1 × 10− 10 ) + log
0.50 x
Also for the following equilibrium: x = 0.177 mmol
Ag2CO3 (s) + K2C2O4 (aq) r Ag2C2O4 (s) + K2CO3
0.304 − x x 76. (i) 0.20 mole HCl will neutralise 0.20 mole CH3COONa,
producing 0.20 mol CH3COOH. Therefore, in the solution
[CO23− ] [Ag+ ]2 K sp (Ag2CO3 ) moles of CH3COOH = 1.20
K = × =
[C2O24− ] [Ag+ ]2 K sp (Ag2C2O4 ) Moles of CH3COONa = 0.80
Given, 0.304 – x = 0.0358 ⇒ x = 0.2682 [Salt]
pH = pK a + log
0.2682 [Acid]
⇒ K = = 7.5 (0.80)
0.0358 = − log (1.8 × 10− 5 ) + log = 4.56
K sp (Ag2CO3 ) = K × K sp (Ag2C2O4 ) (1.20)
= 7.5 × 1.29 × 10− 11 = 9.675 × 10− 11 (ii) CH3COONa + HCl → CH3COOH + NaCl
Initial 0.10 0.20 0 0
73. CH3COOH r CH3COO− + H+ Final 0 0.10 0.10 0.10
Now, the solution has 0.2 mole acetic acid and 0.1 mole HCl.
When concentration of CH3COOH is 1.0 M, ‘α’ is negligible,
Due to presence of HCl, ionisation of CH3COOH can be
[H+ ] = K aC = 4.24 × 10− 3 M ignored (common ion effect) and H+ in solution is mainly due
pH = − log (4.24 × 10− 3 ) = 2.37 to HCl.
[H+ ] = 0.10
Now, let us assume that solution is diluted to a volume where
concentration of CH3COOH (without considering ionisation) pH = – log (0.10) = 1.0
is x. 9.57
77. In pure water, solubility = × 10− 3 M = 1.65 × 10− 4 M
CH3COOH r CH3COO− + H+ 58
x (1 − α ) xα xα
2
K sp = 4 S 3 = 4 (1.65 × 10− 4 )3 = 1.8 × 10− 11

Ka =
1−α In 0.02 M Mg(NO3 )2;
108 Chemical and Ionic Equilibrium

K sp 1 pH = 4.15
solubility of Mg(OH)2 = ×
[Mg ] 2+
2 Kw 10− 14
[OH− ] = =
[H ] 7 × 10− 5
+
= 1.5 × 10− 5 mol L− 1
= 1.5 × 10− 5 × 58 g L− 1 = 1.43 × 10− 10 M

= 8.7 × 10− 4 g L− 1 81. Sodium acetate (CH3COONa) is a basic salt (salt of strong base
and weak acid) therefore, its aqueous solution has pH > 7.
78. HCOOH r H+ + HCOO−
82. mmol of NaOH = 20 × 0.2 = 4
HCOONa r Na + + HCOO−
1 − 0.75 0.75 mmol of acetic acid = 50 × 0.2 = 10
After neutralisation, buffer solution is formed which contain 6
In the above buffer solution, the significant source of formate ion
(HCOO− ) is HCOONa. Hence, mmol CH3COOH and 4 mmol CH3COONa.
[CH3COONa]
K a = 2.4 × 10− 4 pH = pK a + log
[CH3COOH]
[H+ ] (0.75) 4
= = − log (1.8 × 10− 5 ) + log = 4.56
[HCOOH] 6
2.4 × 10− 4 × 0.20 Now, let x mmol of NaOH is further added so that pH of the
[H+ ] = = 6.4 × 10− 5 resulting buffer solution is 4.74.
0.75
pH = – log (6.4 × 10−5 ) = 4.20 Now, the buffer solution contains (4 + x ) mmol CH3COONa and
(6 − x ) mmol of CH3COOH.
79. K sp (AgI) = 8.5 × 10− 17 = [Ag+ ][I− ] 4+x
4.74 = − log (1.8 × 10− 5 ) + log
6−x
[I− ] required to start precipitation of AgI
8.5 × 10− 17 4+x
= = 8.5 × 10− 16 M ⇒ =1
0.10 6−x
K sp (HgI2 ) = 2.5 × 10− 26 = [Hg2+ ][I− ]2 ⇒ x = 1.0 mmol = 0.2 × V

[I ] required to start precipitation of HgI2 ⇒ V = 5.0 mmol NaOH.

2.5 × 10− 26 83. For acidic buffer, the Henderson’s equation is


= = 5 × 10− 13 M
0.10 (mole of salt)
pH = pK a + log
− (mole of acid)
The above calculation indicates that lower [I ] is required for
precipitation of AgI. When [I− ] reaches to 5 × 10− 13 , AgI gets x
4.75 = – log (1.34 × 10− 5 ) + log
precipitated almost completely. 0.02
When HgI2 starts precipitating, ⇒ x = 0.015 mole of sodium propionate.
8.5 × 10− 17 Addition of 0.01 mole HCl will increase moles of propionic acid
[Ag+ ] = = 1.70 × 10− 4 M
5 × 10− 13 by 0.01 and moles of sodium propionate will decrease by same
amount.
1.70 × 10− 4 × 100
% Ag+ remaining = = 0.17 New moles of acid = 0.02 + 0.01 = 0.03
0.10
% Ag+ precipitated = 100 – 0.17 = 99.83 New moles of salt = 0.015 – 0.01 = 0.005
 0.005
80. Molarity (C ) = 0.10 pH = – log (1.34 × 10− 5) + log   = 4.09
 0.030
[H+ ] = K a ⋅ C = 7 × 10− 5 M (α is negligible) pH of 0.01 HCl = 2, just half of the pH of final buffer solution.
7
Thermodynamics and
Thermochemistry
Topic 1 Thermodynamics
Objective Questions I (Only one correct option) (a) Cyclic process : q = − W
1. An ideal gas is allowed to expand from 1 L to 10 L against a (b) Adiabatic process : ∆U = − W
constant external pressure of 1 bar. The work done in kJ is (c) Isochoric process : ∆U = q
(2019 Main, 12 April I) (d) Isothermal process : q = − W
(a) − 9.0 (b) + 10.0 (c) − 0.9 (d) − 2.0 8. For silver, C p ( J K −1 mol −1 ) = 23 + 0.01 T. If the
2. The difference between ∆H and ∆U ( ∆H − ∆U ), when the temperature (T) of 3 moles of silver is raised from 300 K to
combustion of one mole of heptane (l) is carried out at a 1000 K at 1 atm pressure, the value of ∆H will be close to
temperature T, is equal to (2019 Main, 10 April II) (2019 Main, 8 April I)
(a) − 4 RT (b) 3 RT (c) 4 RT (d) − 3 RT (a) 62 kJ (b) 16 kJ
(c) 21 kJ (d) 13 kJ
3. A process will be spontaneous at all temperature if
(2019 Main, 10 April I) 9. For a diatomic ideal gas in a closed system, which of the
(a) ∆H > 0 and ∆S < 0 (b) ∆H < 0 and ∆S > 0 following plots does not correctly describe the relation
(c) ∆H < 0 and ∆S < 0 (d) ∆H > 0 and ∆S > 0 between various thermodynamic quantities?
(2019 Main, 12 Jan I)
4. During compression of a spring the work done is 10 kJ and 2
kJ escaped to the surroundings as heat. The change in
internal energy, ∆U (in kJ) is (2019 Main, 9 April II) (a)
Cp
(b)
Cv
(a) 8 (b) −12 (c) 12 (d) −8
5. Among the following the set of parameters that represents p T
path functions, is (2019 Main, 9 April I)
(A) q + W (B) q (C) W (D) H − TS (c) U (d) Cv
(a) (A) and (D) (b) (A), (B) and (C)
(c) (B), (C) and (D) (d) (B) and (C) T V
6. 5 moles of an ideal gas at 100 K are allowed to undergo
reversible compression till its temperature becomes 200 K. 10. The standard electrode potential E O− and its temperature

 dE O 
If CV = 28 JK −1 mol −1 , calculate ∆U and ∆pV for this coefficient  −4 −1
 for a cell are 2V and − 5 × 10 VK at
process. ( R = 8.0 JK −1 mol−1 )  dT 
(2019 Main, 8 April II)
(a) ∆U = 2.8 kJ; ∆ ( pV ) = 0.8 kJ 300 K respectively. The cell reaction is
(b) ∆U = 14 J; ∆ ( pV ) = 0.8 J Zn( s ) + Cu2 + ( aq ) → Zn2 + ( aq ) + Cu( s )
(c) ∆U = 14 kJ; ∆ ( pV ) = 4 kJ −
The standard reaction enthalpy (∆ r H O ) at 300 K in kJ mol −1
(d) ∆U = 14 kJ; ∆ ( pV ) = 18 kJ
is, [Use, R = 8 JK −1 mol−1 and F = 96,000 C mol−1 ]
7. Which one of the following equations does not correctly
(2019 Main, 12 Jan I)
represent the first law of thermodynamics for the given
(a) − 412.8 (b) − 384.0
processes involving an ideal gas ? (Assume non-expansion
work is zero) (2019 Main, 8 April I)
(c) 206.4 (d) 192.0
110 Thermodynamics and Thermochemistry

11. The reaction, MgO( s ) + C( s ) → Mg ( s ) + CO( g ), 18 The entropy change associated with the conversion of 1 kg
for which ∆ r H º = + 491.1 kJ mol −1
and of ice at 273 K to water vapours at 383 K is
∆ r S º = 198.0 JK − 1 mol − 1 , is not feasible at 298 K. (Specific heat of water liquid and water vapour are 4.2 kJ
K −1kg−1 and 2.0 kJK −1 kg−1; heat of liquid fusion and
Temperature above which reaction will be feasible is
(a) 2040.5 K (b) 1890.0 K vapourisation of water are 334 kJ kg−1 and 2491 kJkg−1
(c) 2380.5 K (d) 2480.3 K respectively). (log 273 = 2.436, log 373 = 2.572,
12. The standard reaction Gibbs energy for a chemical reaction log 383 = 2.583 ) (2019 Main, 9 Jan II)
at an absolute temperature T is given by, ∆ r G º = A − BT (a) 9.26 kJ kg −1 K −1 (b) 8.49 kJ kg −1 K −1
Where A and B are non-zero constants. (c) 7.90 kJ kg −1 K −1 (d) 2.64 kJ kg −1 K −1
Which of the following is true about this reaction?
19 Consider the reversible isothermal expansion of an ideal gas
(2019 Main, 11 Jan II)
in a closed system at two different temperatures T1 and T2
(a) Endothermic if, A < 0 and B > 0
(T1 < T2 ). The correct graphical depiction of the dependence
(b) Exothermic if, B < 0
of work done (W) on the final volume (V) is
(c) Exothermic if, A > 0 and B < 0 (2019 Main, 9 Jan I)
(d) Endothermic if, A > 0 T2
|W| T2 |W|
13. For the chemical reaction, X - Y , the standard reaction T1
T1
Gibbs energy depends on temperature T (in K) as
3 (a) (b)
∆ rG ° (in kJ mol–1 ) = 120 − T
8
The major component of the reaction mixture at T is O ln V O ln V
(2019 Main, 11 Jan I)
(a) Y if T = 280 K (b) X if T = 350 K |W| T2 |W| T2
(c) X if T = 315 K (d) Y if T = 300 K
T1 T1
14. Two blocks of the same metal having same mass and at
temperature T1 and T2 respectively, are brought in contact (c) (d)
with each other and allowed to attain thermal equilibrium at
constant pressure. The change in entropy, ∆S , for this O ln V O ln V
process is (2019 Main, 11 Jan I)
 (T + T )1/ 2 
 T + T2  20. The combustion of benzene ( l ) gives CO2 ( g ) and H2 O( l ).
(a) 2 C p ln 1 2
 (b) 2 C p ln 1 
 T1T2   4T1T2  Given that heat of combustion of benzene at constant volume
is −3263.9 kJ mol −1 at 25° C; heat of combustion (in kJ
 (T + T2 )2  T + T2 
(c) C p ln 1  (d) 2 C p ln 1  mol −1 ) of benzene at constant pressure will be
 4T1T2   2T1T2  (R = 8.314 JK −1 mol −1 ) (2018 Main)
15. The process with negative entropy change is (a) 4152.6 (b) −452.46
(2019 Main, 10 Jan II) (c) 3260 (d) −3267.6
(a) synthesis of ammonia from N 2 and H 2
21. ∆U is equal to (2017 Main)
(b) dissociation of CaSO4 ( s ) to CaO( s ) and SO3 ( g )
(a) isochoric work (b) isobaric work
(c) dissolution of iodine in water
(c) adiabatic work (d) isothermal work
(d) sublimation of dry ice
22. The standard state Gibbs free energies of formation of
16. An ideal gas undergoes isothermal compression from 5 m 3
3 −2 C(graphite) and C(diamond) at T = 298 K are
to 1 m against a constant external pressure of 4 Nm . Heat
released in this process is used to increase the temperature ∆ f G ° [C(graphite)] = 0 kJ mol −1
of 1 mole of Al. If molar heat capacity of Al is 24 J mol−1 K −1 ,
∆ f G ° [C(diamond)] = 2.9 kJ mol −1
the temperature of Al increases by (2019 Main, 10 Jan II)
3 2 The standard state means that the pressure should be 1 bar,
(a) K (b) 1K (c) 2 K (d) K
2 3 and substance should be pure at a given temperature. The
conversion of graphite [C(graphite)] to diamond
17 A process has ∆H = 200 J mol −1 and ∆S = 40 JK −1 mol −1 .
[C(diamond)] reduces its volume by 2 × 10−6 m 3 mol −1 . If
Out of the values given below, choose the minimum
C(graphite) is converted to C(diamond) isothermally at
temperature above which the process will be spontaneous
(2019 Main, 10 Jan I) T = 298K, the pressure at which C(graphite) is in
(a) 20 K (b) 4 K (c) 5 K (d) 12 K equilibrium with C(diamond), is (2017 Adv.)
Thermodynamics and Thermochemistry 111

[Useful information : 1 J = 1kg m 2 s −2 , Given that ∆S ( A → C ) = 50 eu


1 Pa = 1kg m −1 −2
s ; 1 bar = 10 Pa] 5 ∆S (C → D ) = 30 eu
(a) 58001 bar (b) 1450 bar ∆S (D → B ) = –20 eu where, eu is entropy unit
(c) 14501 bar (d) 29001 bar Then, ∆S ( A → B ) is (2006, 3M)

23. One mole of an ideal gas at 300 K in thermal contact with (a) + 100 eu (b) +60 eu
surroundings expands isothermally from 1.0 L to 2.0 L (c) –100 eu (d) –60 eu
against a constant pressure of 3.0 atm. 30. A monoatomic ideal gas undergoes a process in which the
In this process, the change in entropy of surroundings ratio of p to V at any instant is constant and equals to 1. What
( ∆S surr ) in JK −1 is (1 L atm = 101.3 J) (2016 Adv.) is the molar heat capacity of the gas ? (2006, 3M)
(a) 5.763 (b) 1.013 (c) − 1.013 (d) − 5.763 4R 3R 5R
(a) (b) (c) (d) 0
2 2 2
24. The following reaction is performed at 298K
31. One mole of monoatomic ideal gas expands adiabatically at
2NO( g ) + O2 ( g ) r 2NO2 ( g )
initial temperature T against a constant external pressure of
The standard free energy of formation of NO(g) is
1 atm from 1 L to 2 L. Find out the final temperature
86.6 kJ/mol at 298 K. What is the standard free energy of
(R = 0.0821 L atm K − 1 mol − 1 ) (2005, 1M)
formation of NO2 ( g ) at 298 K? ( K p = 1.6 × 1012 ) (2015 Main) T
(a) T (b)
(a) R( 298 ) ln (1. 6 × 1012 ) – 86600 ( 2 )5 / 3 − 1
(b) 86600 + R( 298 ) ln (1. 6 × 1012 ) 2 2
(c) T − (d) T +
ln (1. 6 × 1012 ) 3 × 0.082 3 × 0.082
(c) 86600 –
R ( 298 ) 32. 2 moles of an ideal gas expanded isothermally and
(d) 0. 5 [ 2 × 86600 – R ( 298 ) ln (1. 6 × 1012 )] reversibly from 1 L to 10 L at 300 K. What is the enthalpy
change? (2004, 1M)
25. For the process, H2 O ( l ) → H2 O ( g )
(a) 4.98 kJ (b) 11.47 kJ (c) –11.47 kJ (d) 0 kJ
at T = 100° C and 1 atmosphere pressure, the correct choice is
33. Spontaneous adsorption of a gas on solid surface is an
(a) ∆S system > 0 and ∆S surrounding > 0 (2014 Adv.)
exothermic process because (2004, 1M)
(b) ∆S system > 0 and ∆S surrounding < 0 (a) ∆H increases for system (b) ∆S increases for gas
(c) ∆S system < 0 and ∆S surrounding > 0 (c) ∆S decreases for gas (d) ∆G increases for gas
(d) ∆S system < 0 and ∆S surrounding < 0
34. One mole of a non-ideal gas undergoes a change of state
26. A piston filled with 0.04 mole of an ideal gas expands (2.0 atm, 3.0 L, 95 K) → (4.0 atm, 5.0 L, 245 K) with a
reversibly from 50.0 mL to 375 mL at a constant temperature change in internal energy, ∆E = 30.0 L-atm. The change in
of 37.0°C. As it does so, it absorbs 208 J of heat. The values enthalpy ( ∆H ) of the process in L-atm is (2002, 3M)
of q and W for the process will be (a) 40.0 (b) 42.0
(R = 8.314 J / mol K, ln 7.5 = 2.01) (2013 Main)
(c) 44.0
(a) q = + 208 J, W = − 208 J
(d) not defined, because pressure is not constant
(b) q = − 208 J, W = − 208 J
(c) q = − 208 J, W = + 208 J 35. Which of the following statements is false? (2001, 1M)
(d) q = + 208 J, W = + 208 J (a) Work is a state function
27. For the process H2 O( l ) (1 bar, 373 K) → H2 O( g ) (b) Temperature is a state function
(c) Change in the state is completely defined when the
(1 bar, 373 K), the correct set of thermodynamic
initial and final states are specified
parameters is (2007, 3M)
(a) ∆G = 0, ∆S = + ve (b) ∆G = 0, ∆S = − ve (d) Work appears at the boundary of the system
(c) ∆G = + ve, ∆S = 0 (d) ∆G = − ve, ∆S = + ve 36. In thermodynamics, a process is called reversible when
(2001, 1M)
28. The value of log 10 K for a reaction A r B is (a) surroundings and system change into each other
(Given : ∆ r H ° 298 K = − 54.07 kJ mol −1 , (b) there is no boundary between system and surroundings
∆ r S ° 298 K = 10 JK −1 mol −1 and R = 8.314 JK−1 mol−1 ; (c) the surroundings are always in equilibrium with the system
2. 303 × 8. 314 × 298 = 5705) (2007, 3M)
(d) the system changes into the surroundings spontaneously
(a) 5 (b) 10 (c) 95 (d) 100 37. For an endothermic reaction, where ∆H represents the
29. The direct conversion of A to B is difficult, hence it is carried enthalpy of the reaction in kJ/mol, the minimum value for
out by the following shown path the energy of activation will be (1992, 1M)
C → D (a) less than ∆H (b) zero
↑ ↓ (c) more than ∆H (d) equal to ∆H
A B
112 Thermodynamics and Thermochemistry

38. The difference between heats of reaction at constant pressure 42. An ideal gas is expanded form ( p1 , V1 , T1 ) to ( p2 , V2 , T2 )
and constant volume for the reaction under different conditions. The correct statement(s)
2C6 H6 ( l ) + 15O2 → 12CO2 ( g ) + 6H2 O( l ) among the following is (are) (2017 Adv.)
at 25° C in kJ is (1991, 1M) (a) The work done by the gas is less when it is expanded
(a) − 7.43 (b) + 3.72 reversibly from V1 to V2 under adiabatic conditions as
compared to that when expanded reversibly form V1 to V2
(c) − 3.72 (d) + 7.43
under isothermal conditions.
(b) The change in internal energy of the gas is (i) zero, if it is
Objective Questions II expanded reversibly with T1 = T2, and (ii) positive, if it is
(One or more than one correct option) expanded reversibly under adiabatic conditions with T1 ≠ T2
39. In thermodynamics, the p-V work done is given by (c) If the expansion is carried out freely, it is simultaneously
both isothermal as well as adiabatic
w = − ∫ dV pext
(d) The work done on the gas is maximum when it is
For a system undergoing a particular process, the work done is compressed irrversibly from ( p2 , V2 ) to ( p1, V1 ) against
constant pressure p1
 RT a
w = − ∫ dV  − 2
V − b V  43. For a reaction taking place in a container in equilibrium
with its surroundings, the effect of temperature on its
This equation is applicable to a (2020 Adv.)
equilibrium constant K in terms of change in entropy is
(a) system that satisfies the van der Waals’ equation of state described by (2017 Adv.)
(b) process that is reversible and isothermal (a) With increase in temperature, the value of K for
(c) process that is reversible and adiabatic endothermic reaction increases because unfavourable change
(d) process that is irreversible and at constant pressure in entropy of the surroundings decreases
(b) With increase in temperature, the value of K for exothermic
40. Choose the reaction(s) from the following options, for which
reaction decreases because favourable change in entropy of
the standard enthalpy of reaction is equal to the standard the surrounding decreases
enthalpy of formation. (2019 Adv.)
(c) With increase in temperature, the value of K for
(a) 2C( g ) + 3 H2 ( g ) → C2 H6 ( g ) endothermic reaction increases because the entropy change
(b) 2H2 ( g ) + O2 ( g ) → 2H2 O( l ) of the system is negative
(d) With increase in temperature, the value of K for exothermic
3
(c) O2 ( g ) → O3 ( g ) reaction decreases because the entropy change of the system
2 is positive
1 44. An ideal gas in thermally insulated vessel at internal
(d) S8 ( s ) + O2 ( g ) → SO2 ( g )
8 pressure = p1 , volume = V1 and absolute temperature = T1
41. A reversible cyclic process for an ideal gas is shown below. expands irreversibly against zero external pressure, as
Here, p , V and T are pressure, volume and temperature, shown in the diagram. The final internal pressure,
respectively. The thermodynamic parameters q , w , H and U volume and absolute temperature of the gas are p2 , V2
are heat, work, enthalpy and internal energy, respectively. and T2 , respectively. For this expansion
(2018 Adv.)
pext=0

A(p1, V1, T1) C ( p 2 , V 1 , T2 ) pext=0 Irreversible


Volume (V)

p1,V1,T1 p2,V2,T2
B(p2, V2, T1)

Thermal insulation
Temperature (T) (2014 Adv.)
(a) q = 0 (b) T2 = T1
The correct options is (are)
(c) p2V2 = p1V1 (d) p2V2γ = p1V1γ
(a) q AC = ∆U BC and w AB = p2 (V2 − V1 )
(b) wBC = p2 (V2 − V1 ) and qBC = ∆H AC 45. Benzene and naphthalene form an ideal solution at room
(c) ∆HCA < ∆UCA and q AC = ∆U BC temperature. For this process, the true statement(s) is (are)
(2013 Adv.)
(d) qBC = ∆H AC and ∆HCA > ∆UCA (a) ∆G is positive (b) ∆S system is positive
(c) ∆S surroundings = 0 (d) ∆H = 0
Thermodynamics and Thermochemistry 113

46. The reversible expansion ob an ideal gas under adiabatic What is the ratio of the standard Gibbs energy of the reaction
and isothermal conditions is shown in the figure. Which of at 1000 K to that at 2000 K? (2020 Adv.)
the following statement(s) is (are) correct? (2012)
115

Partial pressure of B (bar)


(p1,V1,T1) 100
Iso
the
p rma
l
Ad

ia
ba (p2,V2,T2)
tic
(p3,V3,T3)
50
V
(a) T1 = T2
(b) T3 > T1 10
(c) Wisothermal > Wadiabatic t′ Time
(d) ∆U isothermal > ∆U adiabatic
47. For an ideal gas, consider only p -V work in going from Assertion and Reason
initial state X to the final state Z. The final state Z can be Read the following questions and answer as per the direction
reached by either of the two paths shown in the figure. given below :
(a) Statement I is true; Statement II is true; Statement II is
p(atmosphere)

X Y
the correct explanation of Statement I
(b) Statement I is true; Statement II is true; Statement II is
Z not the correct explanation of Statement I
(c) Statement I is true; Statement II is false
(d) Statement I is false; Statement II is true
V(litre)
52. Statement I There is a natural asymmetry between
[Take ∆S as change in entropy and W as work done]. converting work to heat and converting heat to work.
Which of the following choice(s) is (are) correct? (2012)
Statement II No process is possible in which the sole result
(a) ∆S X → Z = ∆S X → Y + ∆S Y → Z is the absorption of heat from a reservoir and its complete
(b) W X → Z = W X → Y + WY → Z
conversion into work. (2008, 3M)
(c) W X → Y → Z = W X → Y
(d) ∆S X → Y → Z = ∆S X → Y 53. Statement I For every chemical reaction at equilibrium,
standard Gibbs energy of reaction is zero.
48. Among the following, extensive property is (properties are) Statement II At constant temperature and pressure,
(2010)
(a) molar conductivity (b) electromotive force chemical reactions are spontaneous in the direction of
(c) resistance (d) heat capacity decreasing Gibbs energy. (2008, 3M)

54. Statement I The heat absorbed during the isothermal


49. Among the following, the state function(s) is(are)
expansion of an ideal gas against vacuum is zero.
(a) internal energy (2009)
Statement II The volume occupied by the molecules of an
(b) irreversible expansion work ideal gas is zero. (2000, S, 1M)
(c) reversible expansion work
(d) molar enthalpy Passage Based Questions
50. Identify the intensive quantities from the following. A fixed mass m of a gas is K L
(1993, 1M) subjected to transformation of
(a) enthalpy (b) temperature states from K to L to M to N and Pressure
(c) volume (d) refractive index back to K as shown in the figure.
(2013 Adv.)
Numerical Answer Type Questions N M
Volume
51. Consider the reaction, A B at 1000 K. At time t’, the
q
temperature of the system was increased to 2000 K and the 55. The pair of isochoric processes among the transformation of
system was allowed to reach equilibrium. Throughout this states is
experiment the partial pressure of A was maintained at 1 bar. (a) K to L and L to M (b) L to M and N to K
Given, below is the plot of the partial pressure of B with time. (c) L to M and M to N (d) M to N and N to K
114 Thermodynamics and Thermochemistry

56. The succeeding operations that enable this transformation of Integer Answer Type Question
states are
(a) heating, cooling, heating, cooling 67. One mole of an ideal gas is taken from a to b along two paths
(b) cooling, heating, cooling, heating denoted by the solid and the dashed lines as shown in the
graph below. If the work done along the solid line path is Ws
(c) heating, cooling, cooling, heating
and that along the dotted line path is Wd , then the integer
(d) cooling, heating, heating, cooling
closest to the ratio Wd / Ws is (2010)
4.5
Match the Columns 4.0 a
57. Match the thermodynamic processes given under Column I 3.5

with the expressions given under Column II. 3.0


p 2.5
(atm)
Column I Column II 2.0
1.5
A. Freezing of water at 273 K and 1 atm p. q=0 1.0
B. Expansion of 1 mole of an ideal gas into q. W =0 0.5 b
a vacuum under isolated conditions 0.0
0.0 0.5 1.0 1.5 2.0 2.5 3.0 3.5 4.0 4.5 5.0 5.5 6.0
C. Mixing of equal volumes of two ideal r. ∆S sys < 0 V (L)
gases at constant temperature and
pressure in an isolated container Subjective Questions
D. Reversible heating of H2 ( g ) at 1 atm s. ∆U = 0 68. For the reaction, 2CO + O2 → 2CO2 ; ∆H = − 560 kJ. Two
from 300 K to 600 K, followed by moles of CO and one mole of O2 are taken in a container of
reversible cooling to 300 K at 1 atm volume 1 L. They completely form two moles of CO2 , the
t. ∆G = 0 gases deviate appreciably from ideal behaviour. If the
pressure in the vessel changes from 70 to 40 atm, find the
58. Match the transformations in Column I with appropriate
magnitude (absolute value) of ∆U at 500 K. (1 L-atm = 0.1 kJ)
options in Column II. (2011) (2006, 3M)

Column I Column II 69. 100 mL of a liquid contained in an isolated container at a


A. CO2 ( s ) → CO2 ( g ) p. Phase transition pressure of 1 bar. The pressure is steeply increased to
100 bar. The volume of the liquid is decreased by 1 mL at
B. CaCO3 ( s ) → CaO( s ) + CO2 ( g ) q. Allotropic change this constant pressure. Find the ∆H and ∆U . (2004, 2M)
C. 2H• → H2 ( g ) r. ∆H is positive 3R 3R
70. CV value of He is always but CV value of H2 is at low
D. P(white, solid) → P(red, solid) s. ∆S is positive 2 2
t. ∆S is negative 5R
temperature and at moderate temperature and more than
2
Fill in the Blanks 5R
at higher temperature. Explain in two or three lines.
2 (2003, 2M)
59. Enthalpy is an ............... property. (1997, 1M)
71. Two moles of a perfect gas undergo the following processes :
60. When Fe(s) is dissolved in aqueous hydrochloric acid in a
(a) a reversible isobaric expansion from (1.0 atm, 20.0 L)
closed vessel, the work done is ................ . (1997)
to (1.0 atm, 40.0 L)
61. The heat content of the products is more than that of the (b) a reversible isochoric change of state from (1.0 atm,
reactants in an .............. reaction. (1993, 1M) 40.0 L) to (0.5 atm, 40.0 L)
62. A system is said to be ........ if it can neither exchange matter (c) a reversible isothermal compression from (0.5 atm,
nor energy with the surroundings. (1993, 1M) 40.0 L) to (1.0 atm, 20.0 L)
63. C p − CV for an ideal gas is ................. (i) Sketch with labels each of the processes on the same
(1984, 1M)
p-V diagram.
64. The total energy of one mole of an ideal monatomic gas at (ii) Calculate the total work (W) and the total heat change
27° C is ..............cal. (1984, 1M) (Q) involved in the above processes.
(iii) What will be the values of ∆ U, ∆H and ∆S for the
True/False overall process? (2002, 5M)
65. First law of thermodynamics is not adequate in predicting 72. When 1-pentyne (A) is treated with 4 N alcoholic KOH at
the direction of a process. (1982, 1M)
175° C, it is converted slowly into an equilibrium mixture of
66. Heat capacity of a diatomic gas is higher than that of a
1.3% 1-pentyne (A), 95.2% 2-pentyne (B) and 3.5% of
monoatomic gas. (1985, 1/2 M)
Thermodynamics and Thermochemistry 115

1, 2-pentadiene (C). The equilibrium was maintained at Calculate the enthalpy change in this process CV m for argon
175° C. Calculate ∆G° for the following equilibria. is 12.49 JK −1 mol −1 . (2000, 4M)

B r A, ∆G°1 = ? 75. A gas mixture of 3.67 L of ethylene and methane on


B r C, ∆G° 2 = ? complete combustion at 25° C produces 6.11 L of CO2 . Find
out the amount of heat evolved on burning 1 L of the gas
From the calculated value of ∆G°1 and ∆G° 2 indicate the
mixture. The heat of combustion of ethylene and methane
order of stability of (A), (B) and (C). Write a reasonable
are − 1423 and − 891 kJ mol −1 at 25°C. (1991, 5M)
reaction mechanism showing all intermediates leading to
(A), (B) and (C). 76. An athlete is given 100 g of glucose (C6 H12 O6 ) of energy
(2001, 10M)
equivalent to 1560 kJ. He utilizes 50 per cent of this gained
1
73. Show that the reaction, CO( g ) + O2 ( g ) → CO2 ( g ) at energy in the event. In order to avoid storage of energy in the
2
body, calculate the weight of water he would need to
300 K, is spontaneous and exothermic, when the standard
perspire. The enthalpy of evaporation of water is 44 kJ/mol.
entropy change is − 0.094 kJ mol −1 K −1 . The standard (1989, 2M)
Gibbs’ free energies of formation for CO2 and CO are
77. Following statement is true only under some specific
–394.4 and –137.2 kJ mol −1 , respectively. (2000, 3M) conditions. Write the conditions for that in not more than
74. A sample of argon gas at 1 atm pressure and 27° C expands two sentences
reversibly and adiabatically from 1.25 dm3 to 2.50 dm3 . “The heat energy q, absorbed by a gas is ∆H.’’ (1984, 1M)

Topic 2 Thermochemistry
Objective Questions I (Only one correct option) 1
(iii) CO ( g ) + O2 ( g ) → CO2 ( g ); ∆ r H È = z kJ mol− 1
1. Lattice enthalpy and enthalpy of solution of NaCl are 788 kJ 2
mol −1 and 4 kJ mol −1 , respectively. The hydration enthalpy Based on the above thermochemical equations, find out
of NaCl is (2020 Main, 5 Sep II)
which one of the following algebraic relationships is correct?
(2019 Main, 12 Jan II)
(a) −780 kJ mol −1 (b) 780 kJ mol −1 (a) y = 2 z − x (b) x = y − z
(c) −784 kJ mol −1 (d) 784 kJ mol −1 (c) z = x + y (d) x = y + z
2. The variation of equilibrium constant with temperature is 5. Given, C(graphite) + O2 ( g ) → CO2 ( g );
given below: (2020 Main, 6 Sep I)
∆ r H ° = − 393.5 kJ mol − 1
Temperature Equilibrium constant
1
T1 = 25° C K 1 = 10 H2 ( g ) + O2 ( g ) → H2 O( l ); ∆ r H ° = − 285.8 kJ mol −1
2
T2 = 100° C K 2 = 100 CO2 ( g ) + 2 H2 O( l ) → CH4 ( g ) + 2O2 ( g );
The values of ∆H °, ∆G° at T1 and ∆G° at ∆ r H ° = + 890.3 kJ mol −1
T2 (in kJ mol −1 ) respectively, are close to
Based on the above thermochemical equations, the value of
[use R = 8.314 JK −1 mol −1 ] ∆ r H ° at 298 K for the reaction, (2017 Main)
(a) 28.4, − 714
. and − 5.71 C(graphite) + 2 H2 ( g ) → CH4 ( g ) will be
(b) 0.64, − 714
. and − 5.71
(c) 28.4, − 5.71 and − 14. 29 (a) + 78 .8 kJ mol − 1 (b) + 144.0 kJ mol − 1
(d) 0.64, − 5.71 and − 14. 29 (c) − 74 .8 kJ mol − 1 (d) − 144.0 kJ mol − 1
−1
3. Enthalpy of sublimation of iodine is 24 cal g at 200°C. If
6. The heats of combustion of carbon and carbon monoxide are
specific heat ofI2( s) andI2 (vap.) are 0.055 and 0.031 cal g −1 K −1
− 393.5 and − 283.5 kJ mol−1 , respectively. The heat of
respectively, then enthalpy of sublimation of iodine at 250°C
formation (in kJ) of carbon monoxide per mole is
in cal g −1 is (2019 Main, 12 April I) (2016 Main)
(a) 2.85 (b) 5.7 (c) 22.8 (d) 11.4 (a) 676.5 (b) −676.5
4. Given : (c) −110.5 (d) 110.5
(i) C(graphite) + O2 ( g ) → CO2 ( g ); ∆ r H È = x kJ mol− 1 7. For the complete combustion of ethanol,
C2 H 5 OH( l ) + 3O2 ( g ) → 2CO2 ( g ) + 3H2 O( l ), the
1
(ii) C(graphite) + O2 ( g ) → CO2 ( g ); amount of heat produced as measured in bomb
2
calorimeter, is 1364.47 kJ mol −1 at 25°C. Assuming
∆ r H È = y kJ mol− 1
116 Thermodynamics and Thermochemistry

ideality the enthalpy of combustion, ∆C H, for the reaction 16. The thermal dissociation of equilibrium of CaCO3 ( s ) is
will be (R = 8.314 J K–1 mol –1 ) (2014 Main) studied under different conditions. (2013 Adv.)
(a) − 1366. 95 kJ mol −1 (b) − 1361. 95 kJ mol −1 CaCO3 ( s ) r CaO( s ) + CO2 ( g )
(c) − 1460. 50 kJ mol −1 (d) − 1350. 50 kJ mol −1 For this equilibrium, the correct statement(s) is/are
(a) ∆H is dependent on T
8. The standard enthalpies of formation of CO2 ( g ), H2 O( l ) and (b) K is independent of the initial amount of CaCO3
glucose(s) at 25°C are − 400 kJ/mol, − 300 kJ/mol and (c) K is dependent on the pressure of CO2 at a given T
− 1300 kJ/mol, respectively. The standard enthalpy of (d) ∆H is independent of the catalyst, if any
combustion per gram of glucose at 25°C is (2013 Adv.)
(a) + 2900 kJ (b) − 2900 kJ Subjective Questions
(c) − 16. 11 kJ (d) + 16.11kJ 17. In a constant volume calorimeter, 3.5 g of a gas with
9. Using the data provided, calculate the multiple bond energy molecular weight 28 was burnt in excess oxygen at 298.0 K.
−1
(kJ mol ) of a C≡≡ C bond C2 H2 . That energy is (take the The temperature of the calorimeter was found to increases
from 298.0 K to 298.45 K due to the combustion process.
bond energy of a C  H bond as 350 kJ mol −1 ) (2012) Given that the heat capacity of the calorimeter is 2.5 kJ K − 1 ,
2C( s ) + H2 ( g ) → C2 H2 ( g ) ; ∆H = 225 kJ mol −1 the numerical value for the enthalpy of combustion of the
2C( s ) → 2C( g ) ; ∆H = 1410 kJ mol −1 gas in kJ mol − 1 is (2009)

H2 ( g ) → 2H( g ) ; ∆H = 330 kJ mol −1 18. Diborane is a potential rocket fuel which undergoes
combustion according to the reaction
(a) 1165 (b) 837 (c) 865 (d) 815
B2 H6 ( g ) + 3O2 ( g ) → B2 O3 ( s ) + 3H2 O ( g )
10. The species which by definition has zero standard molar
From the following data, calculate the enthalpy change for
enthalpy of formation at 298 K is (2010)
the combustion of diborane. (2000, 2M)
(a) Br2 ( g ) (b) Cl 2 ( g ) (c) H2 O( g ) (d) CH4 ( g )
3
11. The bond energy (in kcal mol −1 ) of C— C single bond is 2B ( s ) + O2 ( g ) → B2 O3 ( s ) ; ∆H = − 1273 kJ mol −1
2
approximately (2010)
1
(a) 1 (b) 10 (c) 100 (d) 1000 H2 ( g ) + O2 ( g ) → H2 O ( l ) ; ∆H = − 286 kJ mol –1
2
12. ∆H vap = 30 kJ/mol and ∆S vap = 75 Jmol –1 K –1 . Find the
H2 O ( l ) → H2 O ( g ) ; ∆H = 44 kJ mol −1
temperature of vapour, at one atmosphere (2004, 1M)
(a) 400 K (b) 350 K (c) 298 K (d) 250 K 2B ( s ) + 3H2 ( g ) → B2 H6 ( g ) ; ∆H = 36 kJ mol −1
19. Estimate the average S–F bond energy in SF6 . The values of
13. Which of the following reactions defines ∆H f° ?
standard enthalpy of formation of SF6 (g), S( g ) and F( g )
(2003, 1M)
(a) C(diamond) + O2 ( g ) → CO2 ( g ) are : – 1100, 275 and 80 kJ mol –1 respectively. (1999, 3M)
1 1 20. From the following data, calculate the enthalpy change for
(b) H2 ( g ) + F2 ( g ) → HF( g )
2 2 the combustion of cyclopropane at 298 K. The enthalpy of
(c) N2 ( g ) + 3H2 ( g ) → 2NH3 ( g ) formation of CO2 ( g ), H2 O ( l ) and propane (g) are –393.5,
1 −285.8 and 20.42 kJ mol −1 respectively. The enthalpy of
(d) CO ( g ) + O2 ( g ) → CO2 ( g ) isomerisation of cyclopropane to propene is −33.0 kJ mol −1 .
2
(1998, 5M)
14. The ∆H f° for CO2 ( g ),CO( g ) and H2 O ( g ) are 21. Compute the heat of formation of liquid methyl alcohol in
− 393.5, − 110.5 and −241.8 kJ mol −1 respectively. The kJ mol −1 , using the following data. Heat of vaporisation of
−1 liquid methyl alcohol = 38 kJ/mol. Heat of formation of
standard enthalpy change (in kJ mol ) for the reaction
CO2 ( g ) + H2 ( g ) → CO( g ) + H2 O( g ) is gaseous atoms from the elements in their standard states :
(2000, 1M)
H = 218 kJ/mol, C = 715 kJ/mol, O = 249 kJ/mol.
(a) 524.1 (b) + 41.2
Average bond energies: (1997, 5M)
(c) −262.5 (d) − 41.2
C— H = 415 kJ/mol, C— O = 356 kJ/mol,
Objective Question II O— H = 463 kJ/mol
(One or more than one correct option) 22. The standard molar enthalpies of formation of
cyclohexane (l) and benzene (l) at 25° C are −156 and
15. The following is/are endothermic reaction(s) (1999, 3M)
+ 49 kJ mol −1 respectively. The standard enthalpy of
(a) Combustion of methane
hydrogenation of cyclohexene (l) at 25° C is −119 kJ mol −1 .
(b) Decomposition of water
Use these data to estimate the magnitude of the resonance
(c) Dehydrogenation of ethane to ethylene
energy of benzene. (1996, 2M)
(d) Conversion of graphite to diamond
Thermodynamics and Thermochemistry 117

23. The polymerisation of ethylene to linear polyethylene is 29. The standard molar heat of formation of ethane, carbon
represented by the reaction, dioxide and liquid water are −21.1, −94.1 and − 68.3 kcal
n [CH2 == CH2 ] → [ CH2 — CH2
]n respectively. Calculate the standard molar heat of
combustion of ethane. (1986, 2M)
where, n has large integral value. Given that the average
enthalpies of bond dissociation for C == C and C  C at 30. The bond dissociation energies of gaseous H2 ,Cl 2 and HCl
298 K are +590 and +311 kJ/mol respectively, calculate the are 104, 58 and 103 kcal/mol respectively. Calculate the
enthalpy of polymerization per mole of ethylene at 298 K. enthalpy of formation of HCl gas. (1985, 2M)
(1994, 2M) 31. Given the following standard heats of reactions
24. In order to get maximum calorific output, a burner should (i) heat of formation of water = − 68.3 kcal
have an optimum fuel to oxygen ratio which corresponds (ii) heat of combustion of acetylene = − 310.6 kcal
to 3 times as much oxygen as is required theoretically (iii) heat of combustion of ethylene = − 337.2 kcal
for complete combustion of the fuel. A burner which has been
Calculate the heat of reaction for the hydrogenation of
adjusted for methane as fuel (with x litre/hour of CH4 and
acetylene at constant volume (25° C).
6x litre/hour of O2 ) is to be readjusted for butane, C4 H10 . (1984, 4M)

In order to get the same calorific output, what should be the 32. The molar heats of combustion of C2 H2 ( g ), C (graphite) and
rate of supply of butane and oxygen? Assume that losses due H2 ( g ) are 310.62 kcal, 94.05 kcal and 68.32 kcal
to incomplete combustion etc., are the same for both fuels respectively. Calculate the standard heat of formation of
and that the gases behave ideally. Heats of combustions: C2 H2 ( g ) . (1983, 2M)

CH4 = − 809 kJ/mol, C4 H10 = − 2878 kJ/mol (1993, 3M) 33. The standard heats of formation of CCl 4 ( g ), H2 O( g ),
25. Determine the enthalpy of the reaction, CO2 ( g ) and HCl( g ) at 298 K are −25.5, −57.8 , −94.1 and
C3 H8 ( g ) + H2 ( g ) → C2 H6 ( g ) + CH4 ( g ), at 25° C, using −22.1 kcal/mol respectively. Calculate ∆H ° (298 K) for the
reaction
the given heat of combustion values under standard conditions.
CCl 4 ( g ) + 2H2 O ( g ) → CO2 ( g ) + 4 HCl ( g ) (1982, 2M)
Compound : H2 ( g ) CH4 ( g ) C2 H6 ( g ) C(graphite)
∆H ° (kJ/mol): −285.8 − 890.0 −1560.0 −393.0 34. The enthalpy for the following reactions ( ∆H ° ) at 25°C are
The standard heat of formation of C3 H8 ( g ) is −103 kJ/mol. given below
1 1
(1992, 3M) (i) H2 ( g ) + O2 ( g ) → OH( g ) ∆H ° = − 10.06 kcal
26. Using the data (all values are in kilocalories per mol at 2 2
25° C) given below, calculate the bond energy of C  C and (ii) H2 ( g ) → 2H( g ) ∆H ° = 104.18 kcal
C  H bonds. (iii) O2 ( g ) → 2O( g ) ∆H ° = 118.32 kcal
C( s ) → C( g ); ∆H = 172
H2 ( g ) → 2H ( g ); ∆H = 104 Calculate the O — H bond energy in the hydroxyl radical.
(1981, 2M)
1
H2 ( g ) + O2 ( g ) → H2 O( l ); ∆H = − 68.0
2
Passage Based Questions
C( s ) + O2 ( g ) → CO2 ( g ); ∆H = − 94.0
When 100 mL of 1.0 M HCl was mixed with 100 mL of 1.0 M
Heat of combustion of C2 H6 = −372.0
NaOH in an insulated beaker at constant pressure, a temperature
Heat of combustion of C3 H8 = −530.0 (1990, 5M)
increase of 5.7°C was measured for the beaker and its contents
27. The standard enthalpy of combustion at 25° C of hydrogen, (Expt. 1). Because the enthalpy of neutralisation of a strong acid
cyclohexene (C6 H10 ) and cyclohexane (C6 H12 ) are − 241, with a strong base is a constant ( −57.0 kJ mol−1 ), this experiment
− 3800 and − 3920 kJ/mol respectively. Calculate the heat could be used to measure the calorimeter constant. In a second
of hydrogenation of cyclohexene. (1989, 2M) experiment (Expt. 2), 100 mL of 2.0 M acetic acid
28. An intimate mixture of ferric oxide, Fe2 O3 , and aluminium, ( K a = 2.0 × 10−5 ) was mixed with 100 mL of 1.0 M NaOH (under
Al, is used in solid fuel rockets. Calculate the fuel value per identical conditions to Expt. 1) where a temperature rise of 5.6°C
gram and fuel value per cc of the mixture. Heats of was measured.
formation and densities are as follows: 35. Enthalpy of dissociation (in kJ mol−1 ) of acetic acid
H f (Al 2 O3 ) = − 399 kcal/mol obtained from the Expt. 2 is
H f (Fe2 O3 ) = − 199 kcal/mol (a) 1.0 (b) 10.0 (c) 24.5 (d) 51.4
Density of Fe2 O3 = 5.2 g/cc, Density of Al = 2.7 g/cc 36. The pH of the solution after Expt. 2 is
(1989, 2M) (a) 2.8 (b) 4.7 (c) 5.0 (d) 7.0
Answers
Topic 1 Thermodynamics 59. (extensive)60. (zero) 61. (exothermic reaction)
1. (c) 2. (a) 3. (b) 4. (a) 62. (isolated) 63. (R) 64. (900) 65. (T)
5. (d) 6. (c) 7. (b) 8. (a) 66. (T) 67. (2) 68. (−563 kJ) 72. (12.3 kJ)
9. (a) 10. (a) 11. (d) 12. (d) 73. (−285.4 kJ) 74. (− 116.4 J) 75. (49.82 kJ) 76. (318.96 g)
13. (c) 14. (c) 15. (a) 16. (d)
Topic 2 Thermochemistry
17. (c) 18. (a) 19. (c) 20. (d)
1. (c) 2. (c) 3. (c) 4. (d)
21. (c) 22. (c) 23. (c) 24. (d)
5. (c) 6. (c) 7. (a) 8. (c)
25. (b) 26. (a) 27. (a) 28. (b)
9. (d) 10. (b) 11. (c) 12. (a)
29. (b) 30. (a) 31. (c) 32. (d)
13. (b) 14. (b) 15. (b,c,d) 16. (a, b, c, d)
33. (c) 34. (c) 35. (a) 36. (c)
17. (9 kJ) 18. (−2035 kJ) 19. (309.16 kJ) 20. (−2091.32 kJ)
37. (c) 38. (a) 39. (a, b, c) 40. (c, d)
21. (−116.4 kJ) 22. (−152 kJ/mol)
41. (b,c) 42. (a, c, d) 43. (a, b) 44. (a,b,c)
23. (−32 kJ/mol) 24. (5.46 xL/h)
45. (b,c,d) 46. (a,c,d) 47. (a,c) 48. (c,d) 25. (−55 kJ) 27. (−121kJ/mol)
49. (a,c,d) 50. (b, d) 51. (0.25) 52. (b) 29. (−372 kcal/mol) 30. (−22 kcal/mol)
53. (d) 54. (b) 55. (b) 56. (c) 31. (−41.7 kcal) 32. (54.2 kcal)
57. A → r, t; B → p, q, s; C → p, q, s; D → p,q, s, t 33. (−41.4 kcal) 34. (121.31 kcal)
58. A → p, r, s; B → r, s; C → t; D → p, q, t 35. (1 kJ/mol) 36. (4.7)

Hints & Solutions


Topic 1 Thermodynamics 3. A process will be spontaneous when its free energy (Gibb’s
energy) change will be negative, i.e. ∆G < 0.
1. Key Idea Work done during isothermal expansion of an
ideal gas is given by the equation. Spontaneity of a process is decided by the value of ∆G, which
can be predicted from the Gibb’s equation, ∆G = ∆H − T∆S for
W = − pext (V2 − V1 )
positive/negative signs of ∆H and ∆S at any/higher/lower
temperature as:
According to the given conditions, the expansion is against
constant external pressure. So, the work done is given by
∆H ∆S Comment on ∆G Comment on the
following formula;
temperature process
W = − pext (V2 − V1 ) (T)
= − 1bar (10L − 1L)= − 9 L bar (Q1Lbar =100J) <0 >0 at any temp. <0 spontaneous
= −9 × 100 J = −0.9 kJ >0 <0 at any temp. >0 non-spontaneous
2. Key Idea The relation between ∆H and ∆U is <0 <0 at lower temp. <0 spontaneous
∆H = ∆U + ∆ngRT >0 >0 at higher temp. <0 spontaneous
where, ∆ng = Σnp − ΣnR
= number of moles of gaseous products − number of 4. In the given system, during the compression of a spring the
moles of gaseous reactants. workdone is 10 kJ and 2 kJ of heat is escaped to the
surroundings. So, q = − 2 kJ and W = 10 kJ
The general combustion reaction of a hydrocarbon is as follows : According to the first law of thermodynamics,
 y y ∆U = q + W = − 2 kJ + 10 kJ
CxH y +  x +  O2 → xCO2 + H2O
 4 2 ∆U = 8 kJ
For heptane, x = 7, y = 16 The change in internal energy, ∆U (in kJ) is 8 kJ.
⇒ C7H16 (l ) + 11O2(g ) → 7CO2(g ) + 8H2O(l ) 5. q (heat) and W (work) represents path functions. These variables
∴ ∆ng = 7 − 11 = − 4 are path dependent and their values depends upon the path
Now, from the principle of thermochemistry, followed by the system in attaining that state. They are inexact
∆H = ∆U + ∆ngRT differentials whose integration gives a total quantity depending
⇒ ∆H − ∆U = ∆ngRT = − 4RT upon the path.
Thermodynamics and Thermochemistry 119

Option (a), i.e. q + W and option (d), i.e. H–TS are state 9. For diatomic ideal gases,
functions. The value of state functions is independent to the way f  f 
in which the state is attained. All the state functions are exact CV = R and C p =  + 1 R
2 2 
differentials and cyclic integration involving a state functions is
zero. where, f = degree of freedom
f = translational degree of freedom + rotational degree of
6. Given,
freedom
n = 5 mol, T2 = 200 K, T1 = 100 K
= 3 + 2 = 5 [at normal temperature]
CV = 28 JK −1mol−1
The explanation of various plots are as follows.
∆U = nCV ∆T = nCV (T2 − T1 ) (a) We know that, C p is heat capacity at constant pressure. Thus,
= 5 mol × 28 JK −1mol−1 × (200 − 100) K it does not vary with the variation in pressure. Hence, plot
= 14 ,000 J = 14 kJ given in option (a) is incorrect.
∆pV = nR∆T = nR (T2 − T1 ) (b) In this plot, CV first increases slightly with increase in
temperature and then increases sharply with temperature.
= 5 mol × 8 JK −1mol−1 × (200 − 100) K
The sharp increase is due to increase in degree of freedom.
= 4000 J = 4 kJ Thus, plot given in option (b) is correct.
7. From the 1st law of thermodynamics, (c) For ideal gases,
∆U = q + W Internal energy (U ) ∝ T
where, ∆U = change in internal energy Thus, as temperature increases internal energy also
q = heat increases. As temperature increases further degree of
W = work done freedom also increases thus, there is slight variation in the
graph. First translational degree of freedom is present
The above equation can be represented for the given processes
involving ideal gas as follows: followed by rotational and vibrational degree of freedom.
Hence, plot given in option (c) is also correct.
(a) Cyclic process For cyclic process, ∆U = 0
(d) CV is heat capacity at constant volume. Thus, it does not vary
∴ q = −W
with variation in volume. Hence, plot given in option (d) is
Thus, option (a) is correct. correct.
(b) Adiabatic process For adiabatic process,
q=0 10. Given,
∴ ∆U = W  dE ° −4 −1
E° = 2V,   = − 5 × 10 VK
 dT 
Thus, option (b) is incorrect.
(c) Isochoric process For isochoric process, ∆V = 0. T = 300K, R = 8JK −1mol −1,
Thus, W =0 (QW = p∆V ). F = 96000Cmol −1
∴ ∆V = q
According to Gibbs-Helmholtz equation,
Thus, option (c) is correct.
∆ G = ∆ H − T∆ S …(i)
(d) Isothermal process For isothermal process, ∆U = 0
Also, ∆G = − nFE °cell …(ii)
∴ q = −W
On substituting the given values in equation (ii), we get
Thus, option (d) is correct.
∆G = − 2 × 96000 C mol−1 × 2 V
8. According to Kirchoff’s relation,
T2 [Q n = 2 for the given reaction]
∆H = n ∫ C p dT …(i) = − 4 × 96000J mol −1
= − 384000J mol −1
T1

where, ∆H = Change in enthalpy.


 dE °
C p = Heat capacity at constant pressure. Now, ∆S = nF  
 dT 
Given, n = 3 moles, T1 = 300 K, T2 = 1000 K, C p = 23 + 0.01 T
On substituting the given values in Eq. (i), we get or ∆S = 2 × 96000 C mol−1 × (−5 × 10−4 VK −1 )
1000 1000 = − 96 JK −1 mol −1
∆H = 3 ∫ (23 + 0.01 T )dT = 3 ∫ 23dT + 0.01T dT Thus, on substituting the values of ∆G and ∆S in Eq. (i), we get
300 300
2  1000 − 384000Jmol−1
 0.01 T
= 3 23T + = ∆H − 300 K × (−96 JK −1mol−1 )
 2  300
∆H = − 384000 − 28800 Jmol−1
 0.01 
= 3 23 (1000 − 300) + (10002 − 3002 ) = − 412800J mol −1
 2 
= − 412.800 kJ mol −1
= 3 [16100 + 4550 ] = 61950 J ≈ 62 kJ
120 Thermodynamics and Thermochemistry

11. According to Gibbs-Helmholtz equation,  Tf Tf   T f2 


= C p ln  ×  = C p ln  
∆ rG ° = ∆ r H ° − T∆ r S °  T1 T2  T1T2 
For a reaction to be feasible (spontaneous)   T1 + T2  2 
∆ r G° < 0      (T + T2 )2 
2  
∆ r H °−T∆ r S ° <0 = C p ln  = C p ln  1 
 T1T2   4T1T2 
Given, ∆ r H ° = + 491.1kJ mol − 1 ,  
 
∆ r S ° = 198 JK − 1 mol − 1
15. The explanation of all the options are as follows :
∴ 491.1 × 103 − T × 198 < 0 (a) N2(g ) + 3H2(g ) → 2NH3 (g ),
491.1 × 10 3 ∆ng = 2 − (1 + 3) = − 2
T> = 2480.3 K So, ∆S is also negative (entropy decreases)
198
∴Above 2480.3 K reaction will become spontaneous. ∆
(b) CaSO4 (s) → CaO(s) + SO3 (g ),
12. According to Gibb’s Helmholtz equation, ∆ng = (1 + 0) − 0 = + 1
∆ rG ° = ∆ r H ° − T∆ r S ° So, ∆S = + ve
Given, ∆ rG ° = A − BT (c) In dissolution, ∆S = + ve because molecules/ions of the
On comparing above two equations, we get, solid solute (here, iodine) become free to move in
A = ∆H ° and ∆S ° = B solvated/dissolved state of the solution,
We know that, if ∆H ° is negative, reaction is exothermic and I2 (s) Water
→ I2 (aq)
(KI)
when it is positive, reaction is endothermic.
∴ If A > 0, i.e. positive, reaction is endothermic. (d) In sublimation process, molecules of solid becomes quite
free when they become gas,
13. For a given value of T , CO2(s) → CO2 (g )
(i) If ∆ rG ° becomes < 0, the forward direction will be spontaneous Dry ice
and then the major and minor components will be Y and X So, ∆S will be positive.
respectively.
(ii) If ∆ rG ° becomes > 0, the forward direction will be 16. It is an irreversible isothermal compression of an ideal gas.
non-spontaneous and then the major and minor components (i) dE = dq + p(V f − Vi )
will be X and Y respectively. where, dE = Internal energy change
3 dq = amount of heat released
(a) ∆ rG ° = 120 − × 280 = 15
8 ⇒ 0 = dq + p (V f − Vi )
i.e. ∆ rG ° > O 0, major component = X ; [Q dE = 0 for an isothermal process]
3 ⇒ dq = − 4 (1 − 5) = 16 J
(b) ∆ rG ° = 120 − × 350 = − 1125 .
8 (ii) dq = n × C × ∆T (for Al)
i.e. ∆ rG ° < 0, major component = Y ⇒ 16 J = 1mol × 24 J mol − 1 K −1 × ∆T
3 16 2
(c) ∆ rG ° = 120 − × 315 = 1875 . ⇒ ∆T = K= K
8 24 3
i.e. ∆ rG ° > 0, major component = X 17. ∆G = ∆H − T∆S
3
(d) ∆ rG ° = 120 − × 300 = 7.5 The process will be spontaneous, when
8 ∆G = − ve, i.e. |T∆S | > | ∆ H |
i.e. ∆ rG ° > 0, major component = X Given : ∆H = 200 Jmol−1 and ∆S = 40 JK −1mol−1
14. At the thermal equilibrium, | ∆ H | 200
⇒ T > = =5K
T1 + T2 | ∆S | 40
final temperature T f =
2 So, the minimum temperature for spontaneity of the process is
Tf 5 K.
⇒ for the 1st block, ∆S I = C p ln
T1 18. The conversion of 1 kg of ice at 273 K into water vapours at
Tf 383 K takes place as follows:
⇒ for the 2nd block, ∆S II = C p ln
T2 ∆S1 ∆S2
H2O(s) H2O(l) H2O(l)
When brought in contact with each other, 273K 273K 373K
Tf Tf ∆S3
∆ S = ∆S I + ∆S II = C p ln + C p ln
T1 T2
H2O(g) H2O(g)
∆S4
383K 373K
Thermodynamics and Thermochemistry 121

∆H Fusion 334 kJ kg −1 Now we have ∆ng and other values given in the question are
∆S 1 = = . kJ kg −1K −1
= 122
∆TFusion 273 K ∆U = − 3263.9 kJ/mol
T2  373 K  T = 25° C = 273 + 25 = 298 K
∆S 2 = C ln = 4.2 kJ K −1kg −1 ln   R = 8.314 JK−1mol −1
T1  273 K 
= 4.2 × 2.303 (log 373 − log 273)kJ K −1kg −1 . ) × 8.314 × 10−3 × 298
So, ∆H p = (−3263.9) + (−15
. kJ K −1kg −1
= 4.2 × 2.303 (2.572 − 2.436) = 131 = − 3267.6 kJ mol −1
∆H vap. 2491 kJ kg − 1
∆S 3 = = = 6.67 kJ kg −1K −1 21. According to first law of thermodynamics,
∆Tvap. 373 K ∆ U = q + W = q − p∆ V
T2  383 K  In isochoric process (∆V = 0), ∆U = q
∆S 4 = Cln = 2 kJ K −1kg −1 ln  
T1  373 K  In isobaric process (∆p = 0), ∆U = q
= 2 × 2.303 (log 383 − log 373) kJ K −1kg −1 In adiabatic process (q = 0), ∆U = W
In isothermal process (∆T = 0) and ∆U = 0
= 2 × 2.303 (2.583 − 2.572) kJ K −1kg −1 = 0.05 kJ K −1kg −1
∴ ∆U is equal to adiabatic work.
∆S Total = ∆S 1 + ∆S 2 + ∆S 3 + ∆S 4
= 122 . + 6.67 + 0.05 = 9.26 kJ kg −1K −1
. + 131 22. G = H − TS = U + pV − TS
⇒ dG = dU + pdV + Vdp − TdS − SdT = Vdp − SdT
19. For isothermal reversible expansion,
[Q dU + pdV = dq = TdS ]
Vf V
| W | = nRT ln = nRT ln ⇒ dG = Vdp if isothermal process (dT = 0)
Vi Vi
⇒ ∆G = V∆p
where, V = final volume, Vi = initial final. Now taking initial state as standard state
or | W | = nRT ln V − nRT ln Vi Ggr − Ggr ° = Vgr ∆p …(i)
On comparing with equation of straight line, y = mx + c, we get G d − G d ° = Vd ∆p …(ii)
slope = m = + nRT Now (ii)-(i) gives,
intercept = − nRT ln Vi ° − G°
(Vd − Vgr )∆p = Gd − Ggr + (Ggr d
Thus, plot of |W | with lnV will give straight line in which slope
of 2(T2 ) is greater than slope of 1(T1 ) which is given in all At equilibrium, Gd = Ggr
options. ⇒ (Vgr − Vd )∆p = Gd ° − Ggr ° = 2.9 × 103 J
Now, if Vi < 1then y intercept (− nRT Vi ) becomes positive and 2.9 × 103 29 29000
if it is positive for one case then it is positive for other case also. ⇒ ∆p = Pa = × 108 Pa = bar
Thus, it is not possible that one y-intercept goes above and other 2 × 10−6 2 2
y-intercept goes below. Thus, option (b) and (d) are incorrect. 29000 29000
p = p0 + =1+ = 14501bar
If we extent plot given in option (a) it seems to be merging 2 2
which is not possible because if they are merging they give same 23. By first law, ∆E = Q + W
+ve y-intercept. But they cannot give same y-intercept because
value of T is different. For isothermal expansion, ∆E = 0
Now, if we extent the line of T1 and T2 given in option (c) it ∴ Q = −W
seems to be touching the origin. If they touch the origin then − Qirrev = W irrev = p∆V = 3(2 − 1) = 3 L atm
y-intercept becomes zero which is not possible. Thus, it is not Q (− 3 × 101.3) J 303.9
Also, ∆S surr = irrev = =− = − 1.013JK−1
the exactly correct answer but among the given options it is the T 300 K 300
most appropriate one.
24. For the given reaction,
20. Key idea Calculate the heat of combustion with the help of 2NO(g ) + O2 (g ) s 2NO2 (g )
following formula
Given, ∆G ° (NO) = 86.6 kJ/mol
∆H p = ∆U + ∆ngRT f

where, ∆H p = Heat of combustion at constant pressure ∆G °f (NO2 ) = ?


∆U = Heat at constant volume (It is also called ∆E) K p = 1.6 × 1012
∆ng = Change in number of moles (In gaseous state).
Now, we have,
R = Gas constant; T = Temperature.
∆G ° = 2∆G °
f f (NO ) − [ 2∆G f°(NO) + ∆G f°(O ) ]
2 2
From the equation,
15 = − RT lnK p = 2∆G °f (NO ) − [ 2 × 86,600 + 0 ]
C6H6( l ) + O2( g ) → 6CO2( g ) + 3H2O( l ) 2
2 1
Change in the number of gaseous moles i.e. ∆G °f (NO = [ 2 × 86600 − R × 298 ln (1.6 × 1012 )]
2) 2
15 3
∆ng = 6 − = − or −15. ∆G °f (NO = 0.5 [ 2 × 86,600 − R × (298)ln (1.6 × 1012 )]
2 2 2)
122 Thermodynamics and Thermochemistry

25. PLAN This problem is based on assumption that total entropy change 32. In case of reversible thermodynamic process,
of universe is zero. ∆H = nC p ∆T
At 100°C and 1 atmosphere pressure, ∴ Process is isothermal, ∆T = 0 ⇒ ∆H = 0
H2O (l ) r H2O(g ) is at equilibrium. 33. For a spontaneous process ∆ G < 0
For equilibrium, ∆S total = 0
and ∆S system + ∆S surrounding = 0 Also; ∆G = ∆H − T∆S
As we know during conversion of liquid to gas entropy of For adsorption of gas on solid surface, ∆S < 0. Therefore, in
system increases, in a similar manner entropy of surrounding order to be ∆G < 0, ∆H must be negative.
decreases. 34. ∆H = ∆U + ∆ ( pV ) = 30 + 2(5 − 3) + 5(4 – 2) = 44 L atm.
∴ ∆S system > 0 and ∆S surrounding < 0
35. Work is not a state-function, it depends on path followed.
26. The process is isothermal expansion, hence
36. In a reversible thermodynamic process, system always remains
q= −W
in equilibrium with surroundings.
∆E = 0
V2 37.
W = – 2.303 nRT log
V1
335
= − 2.303 × 0.04 × 8.314 × 310 × log = − 208J ∆H > 0
50
E
q = + 208 J
W = − 208 J (expansion work)
Reaction coordinates
27. At transition point (373 K, 1.0 bar), liquid remains in equilibrium Minimum value of activation energy must be greater than ∆H .
with vapour phase, therefore ∆G = 0. As vaporisation occur,
degree of randomness increases, hence ∆S > 0. 38. ∆H = ∆E + ∆ngRT ⇒ ∆H − ∆E = ∆ngRT = − 3RT
3
28. ∆G ° = ∆H ° − T∆S ° = – 54.07 × 10 J – 298 × 10 J = − 3 × 8.314 × 298 = − 7433 J = – 7.43 kJ
3
= – 57.05 × 10 J 39. Given, w = − ∫ pext dV
Also, ∆G ° = − 2.303 RT log K
For 1 mole van der Waals’ gas
− ∆G ° 57.05 × 103
⇒ log K = = = 10  RT a
2.303 RT 5705 p= − 
 V − b V 2
29. Entropy is a state function hence, For reversible process, pext = pgas
∆ S A → B = ∆ S A → C + ∆ SC → D + ∆ S D → B  RT a
w=−∫ −  dV
= 50 eu + 30 eu + (– 20 eu) = 60 eu  V − b V 2
p But, it is not applicable for irreversible process which are carried
30. Given, = 1 ⇒ p = V ...(i)
V out very fast. So, work done is calculated assuming final
Also from first law : dq = CV dT + pdV pressure remains constant throughout the process.
Thus, statement (a), (b) and (c) correct while statement (d) is
For one mole of an ideal gas : pV = RT incorrect.
⇒ pdV + Vdp = RdT ...(ii)
From (i) pdV = Vdp 40. The standard enthalpy of formation is defined as standard
enthalpy change for formation of 1 mole of a substance from its
Substituting in Eq. (ii) gives elements, present in their most stable state of aggregation.
R
2pdV = RdT ⇒ pdV = dT 3
O 2 (g ) → O 3 (g ) ;
2 2
R
⇒ dq = CV dT + dT 1
S8 (s) + O 2 (g ) → SO 2 (g )
2 8
dq R 3 R
⇒ ∫ dT = CV + 2 = 2 R + 2 = 2R In the above two reactions standard enthalpy of reaction is equal
to standard enthalpy of formation.
31. For an irreversible, adiabatic process;
41. In the given curve AC represents isochoric process as volume
0 = CV (T2 − T1 ) + pe (V2 − V1 ) at both the points is same i.e., V1
Substituting the values Similarly, AB represents isothermal process (as both the points
CV (T − T2 ) = 1(2 − 1)atm L are at T1 temperature) and BC represents isobaric process as
1 2 2 both the points are at p2 pressure.
⇒ T − T2 = = ⇒ T2 = T −
CV 3R 3 × 0.082 Now (i) for option (a)
qAC = ∆U BC = nCV (T2 − T1 )
Thermodynamics and Thermochemistry 123

where, n= number of moles − ∆H


43. ∆S surr =
Cv = specific heat capacity at constant volume Tsurr
However, W AB ≠ p2 (V2 − V1 ) instead For endothermic reaction, if Tsurr increases, ∆Ssurr will increase.
V  For exothermic reaction, if Tsurr increases, ∆Ssurr will decrease.
W AB = nRT1 ln  2 
 V1  44. PLAN This problem includes concept of isothermal adiabatic
So, this option is incorrect. irreversible expansion.

(ii) For option (b) Process is adiabatic because of the use of thermal insolution
therefore, q = 0
qBC = ∆H AC = nC p (T2 − T1 )
Q pext = 0
where, C p = specific heat capacity at constant pressure w = pext ⋅ ∆V = 0 × ∆V = 0
Likewise, Internal energy can be written as
W BC = − p2 (V1 − V2 ) ∆U = q + W = 0
Hence, this option is correct. The change in internal energy of an ideal gas depends only on
(iii) For option (c) temperature and change in internal energy (∆U ) = 0 therefore,
as nC p (T2 − T1 ) < nCV (T2 − T1 ) ∆T = 0 hence, process is isothermal and
so ∆HCA < ∆UCA T2 = T1 and p2V2 = p1V1
and qAC = ∆U BC (d) p2V2γ = p1V1γ is incorrect, it is valid for adiabatic
Hence, this option is also correct. reversible process.
(iv) For option (d) Hence, only (a), (b) and (c) are correct choices.
Although qBC = ∆H AC 45. PLAN When an ideal solution is formed process is spontaneous thus
but ∆HCA >/ ∆UCA According to Raoult’s law, for an ideal solution
Hence, this option is incorrect. ∆H = 0, ∆Vmin = 0
From the relation
42. ∆ G = ∆ H − T∆ S
(a) Since, ∆H = 0
p1 p1 ∴ ∆G = − ve
–Wirr i.e. less than zero. and ∆S surroundings = 0
p2
p2 p2 p2 –Wrev Therefore, ∆S sys = + ve
i.e. more than zero.
V1 V2 V1 V2 46. (a) Since, change of state ( p1 , V1 , T1 ) to ( p2 , V2 , T2 ) is isothermal
Irreversible compression Reversible compression therefore, T1 = T2.
(b) Since, change of state ( p1 , V1 , T1 ) to ( p3 , V3 , T3 ) is an adiabatic
Maximum work is done on the system when compression expansion it brings about cooling of gas, therefore, T3 < T1.
occur irreversibly and minimum work is done is reversible (c) Work done is the area under the curve of p-V diagram. As
compression. obvious from the given diagram, magnitude of area under the
(b) isothermal curve is greater than the same under adiabatic
p1 A
curve, hence Wisothermal > Wadiabatic
(d) ∆U = nCv ∆T
B In isothermal process, ∆U = 0 as ∆T = 0
C In adiabatic process, ∆U = nC v (T3 − T1 ) < 0 as T3 < T1 .

V1 V2 ⇒ ∆U isothermal > ∆U adiabatic


NOTE Here only magnitudes of work is being considered otherwise
AB is isothermal and AC is both works have negative sign.
adiabatic path. Work done is
area under the curve. Hence, 47. (a) Entropy is a state function, change in entropy in a cyclic
process is zero.
(c) It is incorrect. In adiabatic expansion cooling is observed,
Therefore, ∆S X → Y + ∆S Y → Z + ∆S Z → =0
hence ∆U = nCv∆T < 0. X
⇒ − ∆S Z → X = ∆S X → Y + ∆SY → Z = ∆S X → Z
(d) q = 0 (adiabatic), W = 0 (Free expansion)
Hence, ∆U = 0, ∆T = 0 (Isothermal) Analysis of options (b) and (c)
Work is a non-stable function, it does depends on the path
followed. WY → Z = 0 as ∆V = 0.
124 Thermodynamics and Thermochemistry

Therefore, W X → Y → Z = W X → Y. Also, work is the area under the 1


56. PLAN By Boyle’s law at constant temperature, p ∝
V
curve on p-V diagram. By Charles’ law at constant pressure, V ∝ T
X Y X Y Process taking place at
Constant temperature — isothermal
WX → Y
= WX→Y→Z
Constant pressure — isobaric
Constant volume — isochoric
Z WX→Y Z
p p Constant heat — adiabatic

K → L At constant p, volume thus, heating


V V increases
As shown above W X → Y + WY → Z = W X → Y = W X → Y→ Z but not L → M At constant V, pressure thus, cooling
equal to W X → Z. decreases
48. Resistance and heat capacity are mass dependent properties, M → N At constant p, volume thus, cooling
hence they are extensive. decreases
49. Internal energy, molar enthalpy are state function. Also, N → K At constant V, pressure thus, heating
reversible expansion work is a state function because between increases
given initial and final states, there can be only one reversible path.
50. Intensive properties are those property which do not depends 57. (A) → r, t ; (B) → p, q, s ; (C) → p, q, s ; (D) → q, s, t
on amount of sample. Both temperature and refractive index 0° C
(A) H2O(l ) s H2O(s)
are intensive properties while enthalpy and volumes are 1 atm
extensive properties as they depends on amount of sample. q< 0, W < 0 (expansion)
51. Given : A q B (p = 1bar) ∆S sys < 0 (solid state is more ordered than liquid state)
A
∆U < 0 ; ∆G = 0 (At equilibrium)
Using ∆G = ∆G ° + RT ln K p
(B) q = 0 (isolated), W = 0 (pext = 0)
At equilibrium : ∆G ° = − RT ln K p
∆ Ssys > 0 QV2 > V1
∆G °1 = − RT1 ln K p1 … (i)
∆U = 0 Q q = W = 0
∆G2° = − RT2 ln K p 2 … (ii) ∆G < 0 Q p2 < p1
From Eqs. (i) and (ii), (C) q = 0 (isothermal mixing of ideal gases at constant p)
∆G1° T ln K p1 1000 ln(10) 1 W = 0Q ∆U = 0; q = 0, ∆S sys > 0
= 1× = × = = 0.25
∆G2° T2 ln K p 2 2000 ln(100) 4 Q V2 > V1, ∆U = 0
Q∆ T = 0
52. Statement I is true, it is statement of first law of thermodynamics. ∆ G < 0Q mixing is spontaneous.
Statement II is true, it is statement of second law of (D) q = 0 (returning to same state and by same path)
thermodynamics. However, Statement II is not the correct W =0
explanation of statement I.
∆S sys = 0 (same initial and final states)
53. Statement I is false. At equilibrium ∆U = 0
∆G = 0, G ≠ 0. Q Ti = T f , ∆ G = 0
Statement II is true, spontaneous direction of reaction is towards 58. (A) CO2 (s) 
→ CO2 (g )
lower Gibbs free energy.
It is just a phase transition (sublimation) as no chemical
54. Statement I is true. change has occurred. Sublimation is always endothermic.
dq = dE + pext dV = 0 Product is gas, more disordered, hence ∆S is positive.
∆T = 0 (B) CaCO3 (s)  → CaO(s) + CO2 (g )
∴ dE = 0 ; pext = 0 It is a chemical decomposition, not a phase change. Thermal
∴ pext dV = 0 decomposition occur at the expense of energy, hence
Statement II is true. According to kinetic theory of gases, endothermic. Product contain a gaseous species, hence, ∆S > 0.
volume occupied by molecules of ideal gas is zero. (C) 2H  → H2 (g )
However, Statement II is not the correct explanation of Statement I. A new H—H covalent bond is being formed, hence, ∆H < 0.
55. L → M At constant V — isochoric, Also, product is less disordered than reactant, ∆S < 0.

N→K
Thermodynamics and Thermochemistry 125

(D) Allotropes are considered as different phase, hence 71. (i) 1


P(white, solid) → P(red, solid) is a phase transition as well as
1
allotropic change.
Also, red phosphorus is more ordered than white 2
p
phosphorus, ∆S < 0. 0.5 3
59. Extensive : Enthalpy is an extensive property while molar
enthalpy is an intensive property.
60. Zero : − W = p∆V = 0Q ∆V = 0 20 L 40 L
V
61. Exothermic reaction. (ii) − W 1 = p ∆V = 20 L atm
62. Isolated This system neither exchange matter nor energy with W 2 = 0 Q ∆V = 0
surroundings. 40
W 3 = nRT ln = 20 ln 2
63. R : For an ideal gas, C p − CV = R 20
Total work done = W 1 + W 2 + W 3
3 3
64. 900 cal : E = RT = × 2 × 300 cal = − 20 L atm + 0 + 20 ln 2
2 2
= – 6.14 atm
65. True First law deals with conservation of energy while second From first law : q = ∆E + (− W ) = − W
law deals with direction of spontaneous change.
(Q ∆E = 0for cyclic process)
66. True Diatomic gases have more degree of freedom than a ⇒ q = 6.14 L atm = 622.53 J
monatomic gas.
(iii) All the states function, ∆U , ∆H and ∆S are zero for cyclic
67. Work done along dashed path |− W | = Σp∆V process.
2
= 4 × 1.5 + 1 × 1 + × 2.5 = 8.65 L atm 72. At equilibrium : B r A
3
95.2% 1.3%
V2 V
Work done along solid path − W = nRT ln = p1V1 ln 2 13
V1 V1 ⇒ K1 =
952
5.5
= 2 × 2.3 log = 2 × 2.3 log 11 = 4.79 B r C
0.5
95.2% 3.5%
W d 8.65
⇒ = = 1.80 ≈ 2 35
W s 4.79 ⇒ K2 =
952
68. ∆H = ∆U + ∆ ( pV ) = ∆U + V∆p ⇒ ∆G1° = − RT ln K 1
⇒ ∆U = ∆H − V∆p 13
= – 8.314 × 448 × 2.303 log = 16 kJ
= – 560 – 1 × 30 × 0.1 = – 563 kJ 952
69. ∆U = q + W ∆G2° = − RT ln K 2
For adiabatic process, q = 0, hence ∆U = W 35
= – 8.314 × 448 × 2.303 log = 12.3 kJ
952
W = − p (∆V ) = − p (V2 − V1 )
⇒ ∆U = − 100 (99 − 100) = 100 bar mL 73. ∆ rG ° = ∆ f G ° (products) – ∆ f G ° (reactants)
∆H = ∆U + ∆ ( pV ) = – 394.4 – (– 137.2) = – 257.2 kJ < 0
where, ∆pV = p2V2 − p1V1 The above negative value of ∆G indicates that the process is
70. He is monatomic, so it has only three degree of freedom spontaneous.
(translational only) at all temperature hence, CV value is Also, ∆G ° = ∆H ° − T∆S °
3
always R. ⇒ ∆ H ° = ∆ G ° + T∆ S °
2
= – 257.2 + 300 (– 0.094)
Hydrogen molecule is diatomic, has three translational, two
rotational and one vibrational degree of freedom. The energy = – 285.4 kJ < 0
spacing between adjacent levels are in the order of : 74. Given : CV = 12.49 ⇒ C p = 20.8
translational < rotational < vibrational
Cp
At lower temperature only translational degree of freedom ⇒ = γ = 1.66
contribute to heat capacity while at higher temperature CV
rotational and vibrational degree of freedom starts contributing In case of reversible adiabatic expansion :
to heat capacity.
TV γ− 1= constant
126 Thermodynamics and Thermochemistry
γ−1 0.66
T2  V1  V  2. T1 = 298 K, T2 = 373 K, K 1 = 10, K 2 = 100
⇒ =  =  1
T1  V2   V2  K  ∆H °  1 1
log 2  =  − 
0.66  K 1  2.303R  T1 T2 
V 
⇒ T2 = T1  1   100 ∆H °  1 1 
 V2  ∴ log  =  − 
 10  2.303 × 8.314  298 373
0.66
 1 ∴ ∆H ° = 28.4 kJ/mol
= 300   = 189.86 K
 2
We know that, ∆G ° = − RT ln K
⇒ ∆H = nCp ∆T ∴∆G ° at T1 = − 8.314 × 298 × 2.303 × log(10)
1 × 1.25 ⇒ −5.71 kJ/mol
= × 20.8 × (189.86 − 300) J
0.082 × 300 ∆G ° at T2 = − 8.314 × 373 × 2.303 × log(100)
= – 116.4 J ⇒ −14.29 kJ/mol
Hence, the correct option is (c).
75. Let the mixture contain x litre of CH4 and 3.67 − x litre of
ethylene.
3. Key Idea When q is the amount of heat involved
CH4 + O2 → CO2
x x in a system then at constant pressure
C2H4 + O2 → 2CO2 q = qp and C p ∆T = ∆H
3.67 − x 2 (3.67 − x)
Given reaction :
Given : x + 2 (3.67 − x ) = 6.11 L I2 (s) → I2 (g )
⇒ x = 1.23 L Specific heat of I2 (s) = 0.055 cal g −1 K −1.
Volume of ethylene = 2.44 L
Specific heat of I2 (vap) = 0.031 cal g –1K –1.
pV 1×1
Total moles of gases in l litre = = = 0.04
RT 0.082 × 298 Enthalpy (H 1 ) of sublimation of iodine = 24 cal g –1

Also, CH4 and ethylene are in 1 : 2 volume (or mole) ratio, moles If q is the amount of heat involved in a system then at constant
0.04 2 × 0.04 pressure q = qp and
of CH4 = and moles of ethylene =
3 3 ∆H = C p ∆T
0.04 H 2 − H 1 = C p (T2 − T1 )
⇒ Heat evolved due to methane = × 891= 11.88 kJ
3 H 2 = H 1 + ∆C p (T2 − T1 )
2 × 0.04 H 2 = 24 + (0.031 − 0.055) (250 − 200)
Heat evolved due to ethylene = × 1423 = 37.94 kJ
3 H 2 = 24 + (−0.024 ) (50) = 24 − 12
. = 22. 8 cal/g
⇒ Total heat evolved on combustion of 1.0L gaseous mixture Thus, the enthalpy of sublimation of iodine at 250°C is
at 25°C is 11.88 + 37.94 = 49.82 kJ 22.8 cal/g.
1560
76. Moles of H2O needs to perspire = = 17.72 4. Second equation given in this question is wrong. Hence, No
2 × 44
answer in correct.
Weight of water needs to perspire = 17.72 × 18 = 318.96 g If corrected second equation is given,
77. At constant pressure, q = ∆H . 1
i.e. C(graphite) + O2 (g ) → CO (g )
2
and if we take the above reaction in consideration then x = y + z
Topic 2 Thermochemistry will be the answer as:
1. The enthalpy of solution of an ionic solid is numericlly equal to 1
(ii) C(graphite) + O2 (g ) → CO(g ), ∆ r H ° = y kJ/mol
the sum of its hydration and lat energies, 2
°
i.e. ∆H sol °
= ∆H hydration °
+ ∆H lattice 1
(iii) CO(g ) + O2 (g ) → CO2 (g ), ∆ r H ° = z kJ/mol
2
∆H = 4 Summing up both the equation you will get equation (i):
∴ NaCl (s) NaCl (aq)
C(graphite) + O2 (g ) → CO2 (g ), ∆ r H °= x kJ/mol
∆H° =788 ∆H° Hence, x, y and z are related as:
hyd
+ –
Na (g) + Cl (g) x =y+ z

∆sol H ° = ∆ latticeH ° + ∆ hydH ° 5. Based on given ∆ rH °


4 = 788 + ∆ hydH ° ∆ f H ° = H CO
° = − 393.5 kJ mol − 1
2
…(i)
∆H hydH ° = − 784 kJ/mol ∆ f H ° = H H° 2 O = − 285.8 kJ mol − 1 …(ii)
Thermodynamics and Thermochemistry 127

∆ f H ° = H °O 2 = 0.00 (elements) …(iii) (8.314 J K−1 mol −1) in JK−1 mol −1 must be converted into kJ by
dividing the unit by 1000.
Required thermal reaction is for ∆ f H ° of CH4
9. For calculation of C ≡≡ C bond energy, we must first calculate
Thus, from III
dissociation energy of C2H2 as
890.3 = [ ∆ f H ° (CH4 ) + 2 ∆ f H ° (O2 )]
C2H2 (g ) → 2C(g ) + 2H(g ) …(i)
− [ ∆ f H ° (CO2 ) + 2 ∆ f H ° (H2O)] Using the given bond energies and enthalpies :
= ∆ f H ° (CH4 ) + 0] − [ − 393.5 − 2 × 285.5] C2H2 (g ) → 2C (g ) + 2H (g ) ; ∆H = − 225 kJ …(ii)
2C( s) → 2C(g ); ∆H = 1410 kJ …(iii)
∴ ∆ f H ° (CH4 ) = − 74.8 kJ / mol
H2 (g ) → 2H(g ) ; ∆H = 330 kJ …(iv)
6. C(s)+ O2 (g ) → CO2 (g ) ; ∆H = − 393.5 kJ mol −1 …(i) Adding Eqs. (ii), (iii) and (iv) gives Eq. (i).
1 ⇒ C2H2 (g ) → 2C(g ) + 2H(g ) ; ∆H = 1515 kJ
CO+ O2 → CO2( g ); ∆H = − 283.5kJ mol −1 …(ii)
2 ⇒ 1515 kJ = 2 × (C  H) BE + (C ≡≡ C) BE
On subtracting Eq. (ii) from Eq. (i), we get = 2 × 350 + (C ≡≡ C) BE
1 ⇒ (C ≡≡ C) BE = 1515 − 700 = 815 kJ / mol
C (s) + O2 (g ) → CO(g );
2
10. Elements in its standard state have zero enthalpy of formation.
∆H = (− 393.5 + 283.5)kJ mol −1
Cl 2 is gas at room temperature, therefore ∆H °f of Cl 2 (g ) is zero.
= − 110 kJ mol−1(approx.)
11. C—C bond energy is approximately 100 kcal.
7. C2H5OH ( l ) + 3O2 (g ) → 2CO2 (g ) + 3H2O(l ) ∆H vap 30,000
∆U = − 1364.47 kJ/mol 12. T = = = 400 K
∆S vap 75
∆H = ∆U + ∆ngRT
1 1
∆ng = − 1 13. H2 (g ) + F2 (g ) → HF(g )
− 1 × 8.314 × 298 2 2
∆H = − 1364.47 + Here ∆H ° = Standard molar enthalpy of formation of HF(g).
1000
[Here, value of R in unit of J must be converted into kJ] 14. CO2 (g ) + H2 (g ) → CO(g ) + H2O(g )
= − 1364.47 − 2. 4776
∆H = Σ∆ f H ° (products) − Σ∆ f H ° (reactants)
= − 1366.9476 kJ/mol
= – 110.5 – 241.8 – (– 393.5) = + 41.20 kJ
or = − 1366.95 kJ/mol
1
8. PLAN ∆ c H ° (Standard heat of combustion) is the standard enthalpy
15. H2O → H2 + O2, ∆H > 0
2
change when one mole of the substance is completely oxidised. It is reverse of combustion of H2 (g ), hence endothermic.
Also standard heat of formation (∆ f H ° ) can be taken as the C2H6 → C2H4 + H2; ∆H > 0
standard of that substance.
Here, more stable (saturated) hydrocarbon is being transformed
H °CO 2 = ∆ f H ° (CO2 ) = − 400 kJ mol −1
to less stable (unsaturated) hydrocarbon, hence endothermic.
H H° 2 O = ∆ f H ° (H2O ) = − 300 kJ mol −1 C(gr) → C(d) , ∆H > 0
°
Hglucose = ∆ f H ° (glucose) = − 1300 kJ mol −1 More stable allotrope is being converted to less stable allotrope.
16. PLAN Heat of reaction is dependent on temperature (Kirchhoff’s
H O° 2 = ∆ f H ° (O2 ) = 0.00 equation) in heterogeneous system, equilibrium constant is
independent on the molar concentration of solid species.
C6H12O6 (s) + 6 O2 (g ) → 6 CO2 (g ) + 6H2O(l )
Heat of reaction is not affected by catalyst. It lowers activation
∆ cH ° (glucose) = 6[ ∆ f H ° (CO2 ) + ∆ f H °(H2O )] energy.
− [ ∆ f H ° (C6H12O6 ) + 6∆ f H ° (O 2 )] CaCO3 (s) r CaO(s) + CO2 (g )
= 6[ −400 − 300 ] − [ −1300 + 6 × 0 ] By Kirchhoff’s equation,
= − 2900 kJ mol −1 ∆H °2 (at T2 ) = ∆H °1 ( at T1 ) + ∆C p (T2 − T1 )
Molar mass of C 6H12O6 = 180 g mol −1 ∆H ° varies with temperature. Thus, (a) is correct.
K = pCO2
Thus, standard heat of combustion of glucose per gram
K is dependent on pressure of CO2 but independent of molar
−2900
= = − 16.11 kJ g−1 concentration of CaCO3. Thus, (b) and (c) are correct. At a given
180 temperature, addition of catalysis lowers activation energy, ∆H
To solve such problem, students are advised to keep much remaining constant. Thus, (d) is also correct.
importance in unit conversion. As here, value of R
128 Thermodynamics and Thermochemistry

22. + H2 ; ∆H = –119
Ea

CaO + CO2
T
E'a
∆H° ⇒ + 3H2 ;

CaCO3 E'a <E a ∆H = –119 × 3 = – 357 kJ (Theoretical)

– 357 kJ = ∆H °f (cyclohexane) − ∆H °f (C6H6 )


Ea = Activation energy in absence of catalyst
⇒ ∆H °f (C6H6 )Theoretical = – 156 + 357 = 201 kJ
Ea′ = Activation energy in presence of catalyst

17. Temperature rise = T2 − T1 = 298.45 – 298 = 0.45 K ⇒ Resonance energy = ∆H °f (exp.) − ∆H °f (Theoretical)
q = heat capacity × ∆T = 2.5 × 0.45 = 1.125 kJ = 49 – 201 = – 152 kJ/mol
1.125
⇒ Heat produced per mole = × 28 = 9 kJ 23. Per mole of ethylene polymerized, one C == C bond is broken
3.5
and two C—C bonds are formed.
18. ∆H r° = ∆H °f (B2O3 ) + 3∆H °f (H2O) – ∆H °f (B2H6 )
∆H ° (Polym.) = 590 − 2 × 311 = – 32 kJ/mol
∆H °f (H2O)(g ) = ∆H °f (H2O)(l ) + 44 = – 242 kJ 24. At same temperature and pressure, equal volumes contain equal
moles of gases.
⇒ ∆H r° = – 1273 – 3 × 242 – 36
Let 1.0 L of CH4 contain ‘n’ mol
= – 2035 kJ
⇒ x L of CH4 contain nx mol
19. SF6 (g ) → S (g ) + 6F (g ) ⇒ Heat evolved in combustion by x L CH4 = 809 nx kJ
∆H = Σ ∆H °f (products) − Σ ∆H °f (reactants) 1 
Now, 2878 kJ energy is evolved from 1 mole  L C4H10.
= 275 + 6 × 80 + 1100 = 1855 kJ n 
1855 809 nx
⇒ Average S—F bond energy = = 309.16 kJ/mol ⇒ 809 nx kJ energy will be evolved from L of C4H10
6 2878 n
20. Given : Cyclopropane →Propene (C3H6 ); ∆H = − 33 kJ =0.28 x L of C4H10
9 Also, the combustion reaction of butane is
Propene (C3H6 ) + O2 → 3CO2 (g ) + 3H2O (l );
2 13
C4 H10 + O2 → 4CO2 + 5H2O
∆H = – 3 (393.5 + 285.8) – 20.42 = – 2058.32 kJ 2
Adding : 13
9 ⇒ Rate of supply of oxygen = × 0.28 x × 3 = 5.46 x L/h
Cyclopropane + O2 (g ) → 3CO2 (g ) + 3H2 (g ) ; 2
2
25. First we need to determine heat of combustion of C3H8 .
∆H = H 1 + H 2
= −33 + (−2058.32)kJ 3C(gr) + 4 H2 (g ) → C3H8 (g ) ∆H °f = − 103 kJ
∆H = − 2091.32 kJ
°
⇒ – 103 kJ = – 3 × 393 – 4 × 285.80 – ∆ H comb (C3H8 )
21. Given : CH3OH (g ) → CH3OH (l ); ∆H = − 38 kJ
°
⇒ ∆ H comb (C3H8 ) = – 2219.20 kJ
C (g ) + 4H (g ) + O (g ) → CH3OH (g );
∆H = − (3 × 415 + 356 + 463) ⇒ ∆H r° = Σ ∆ H comb
° °
(reactants) − Σ ∆ H comb (products)
Q H = H 1 + H 2 = − 2064 kJ = – 2219.20 – 285.80 + 1560 + 890
C(g ) → C(g ); ∆H = 715 kJ = – 55 kJ
2H2 (g ) → 4H (g );
∆H = 2 × 2 × 218= 872kJ
26. Let x kcal be the C—C bond energy and y kcal be the C—H bond
energy per mole.
1
O2 (g ) → O (g ); ∆H = 249 kJ ⇒ 2C(gr) + 3H2 (g ) → C2H6 (g ) ;
2
1 ∆H ° = – 2 × 94 – 3 × 68 + 372
Adding : C (gr) + 2H2 (g ) + O2 (g ) → CH3OH (l )
2 = – 20 kcal
∆H = − 266 kJ/mol ⇒ – 20 kcal = 2 × 172 + 3 × 104 − BE (C2H6 )
1 × 1.25 ⇒ BE (C2H6 ) = 676 kcal
= × 20.8 × (189.86 − 300) J = −116.4 J
0.082 × 300
Thermodynamics and Thermochemistry 129

Similarly, 3C(gr) + 4 H2 (g ) → C3H8 (g ) ; = – 310.6 – 68.3 – (– 337.2)


∆H ° = – 3× 94 – 4 × 68 + 530 = – 24 = – 41.7 kcal
kcal 32. The standard state formation reaction of C2H2 (g ) is :
⇒ – 24 kcal = 3 × 172 + 4 × 104 − BE (C3H8 )
2C(g ) + H2 (g ) → C2H2 (g ); ∆H °f
⇒ BE (C3H8 ) = 956 kcal
∆H r° = Σ ∆ H comb
° °
(reactants) − Σ ∆ H comb (products)
Also, BE (C2H6 ) = 676 kcal = x + 6 y …(i)
= – 2 × 94.05 – 68.32 – (– 310.62)
BE (C3H8 ) = 956 kcal = 2x + 8 y …(ii)
Solving Eqs. (i) and (ii) gives = 54.2 kcal = ∆H °f (C2H2 )

y = 99 kcal (C—H) BE 33. ∆H r° = Σ ∆ f H ° (products) − Σ ∆ f H ° (reactants)


x =82 kcal (C—C) BE = – 94.1 + 4 (– 22.1) – (– 25.5 – 2 × 57.8)
= – 41.4 kcal
27. + H2 34. ∆H ° = Σ BE (reactants) − Σ BE (products)
1 1
⇒ − 10.06 =
(104.18) + (118.32) − BE (O  H)
°
∆H = Σ ∆ H comb °
(reactants) − Σ ∆ H comb (products) 2 2
BE (O—H) = 121.31 kcal
= – 3800 – 241 – (– 3920) = – 121 kJ/mol
35. Let C JK−1 be the heat capacity of calorimeter.
28. Fe2O3 (s) + 2Al (s) → Al 2O3 (s) + 2Fe(s) Mass of solution = 200 mL × 1 g mL−1 = 200 g
∆H r° = ∆H °f (products) − ∆H °f (reactants) Heat evolved in Expt.1
= 57 × 1000 × 0. 1(mol ) = 5700 J
= – 399 – (– 199) = – 200 kcal
⇒ 5700 J = (200 × 4.2 + C ) × 5.7
Mass of reactants = 56 × 2 + 16 × 3 + 27 × 2 = 214 g
⇒ 1000 = 200 × 4.2 + C …(i)
200 Let x kJ/mol is heat evolved in neutralisation of acetic acid.
⇒ Fuel value/gram = = 0.93 kcal/g
214 ⇒ x × 1000 × 010 . = (200 × 4.2 + C ) × 5.6
Volume of reactants =
160 54 x × 100
cc + cc = 50.77 cc ⇒ = 200 × 4.2 + C …(ii)
5.2 2.7 5.6
200 From (i) and (ii) : x = 56 kJ/mol
⇒ Fuel value/cc = = 3.94 kcal/cc
50.77 ⇒ Enthalpy of ionisation of acetic acid
= − 56 − (− 57) = 1 kJ/mol
29. ∆H = Σ ∆H °f (products) − Σ ∆H °f (reactants)
36. CH3COOH + NaOH → CH3COONa + H2O
= – 2 × 94.1 – 3 × 68.3 – (– 21.1) = – 372 kcal/mol
200 mmol 100 mol 0 0
1 1
30. H2 (g ) + Cl 2 (g ) → HCl (g ); ∆H °f 100 mmol 0 100 mmol
2 2 A buffer is now formed.
∆H °f = Σ BE (reactants) − Σ BE (products) [H+ ][ CH3COO− ]
Ka = = [ H+ ]
[CH3COOH]
1
= (104 + 58) − 103 = – 22 kcal/mol
2 [Q [ CH3COOH] = [ CH3COO− ]]

31. C2H2 + H2 → C2H4 ⇒ pH = pK a = − log (2 × 10−5 ) = 5 − log 2 = 4.7

∆H ° = Σ ∆ H comb
° °
(reactants) − Σ ∆ H comb (products)
8
Solid State

Objective Questions I (Only one correct option) 6. The statement that is incorrect about the interstitial
1. A crystal is made up of metal ions M 1 and M 2 and oxide ions. compounds is (2019 Main, 8 April II)
(a) they are very hard
Oxide ions form a ccp lattice structure. The cation M 1
(b) they have metallic conductivity
occupies 50% of octahedral voids and the cation M 2 occupies
(c) they have high melting points
12.5% of tetrahedral voids of oxide lattice. The oxidation
(d) they are chemically reactive
numbers of M 1 and M 2 are, respectively (2020 Main, 6 Sep II)
(a) +2, + 4 (b) +1, + 3
7. Element ‘B ’ forms ccp structure and ‘A ’ occupies half of
(c) +3, + 1 (d) +4 , + 2 the octahedral voids, while oxygen atoms occupy all the
tetrahedral voids. The structure of bimetallic oxide is
2. A diatomic molecule X 2 has a body-centred cubic (bcc) (2019 Main, 8 April I)
structure with a cell edge of 300 pm. The density of the (a) A2BO4 (b) AB2O4
molecule is 6.17 g cm −3 . The number of molecules present in (c) A2B2O (d) A4B2 O
200 g of X 2 is (Avogadro constant ( N A ) = 6 × 1023 mol −1 ) 8. The radius of the largest sphere which fits properly at the
(2020 Main, 5 Sep I) centre of the edge of a body centred cubic unit cell is
(a) 40N A (b) 2N A (c) 8N A (d) 4N A (Edge length is represented by ‘a’) (2019 Main, 11 Jan II)
3. The ratio of number of atoms present in a simple cubic, body (a) 0.134 a (b) 0.027 a (c) 0.047 a (d) 0.067 a
centered cubic and face centered cubic structure are, 9. A solid having density of 9 × 103 kgm −3 forms face centred
respectively. (2019 Main, 12 April II)
cubic crystals of edge length 200 2 pm. What is the molar
(a) 8 : 1 : 6 (b) 1 : 2 : 4
mass of the solid?
(c) 4 : 2 : 1 (d) 4 : 2 : 3
[Avogadro constant = 6 × 1023 mol−1 , π = 3]
4. An element has a face-centred cubic (fcc) structure with a cell (2019 Main, 11 Jan I)
edge of a. The distance between the centres of two nearest (a) 0.03050 kg mol−1 (b) 0.4320 kg mol−1
tetrahedral voids in the lattice is (2019 Main, 12 April I)
(c) 0.0432 kg mol−1 (d) 0.0216 kg mol−1
a 3
(a) 2a (b) a (c) (d) a 10. A compound of formula A2B3 has the hcp lattice. Which
2 2
atom forms the hcp lattice and what fraction of tetrahedral
5. Consider the bcc unit cells of the solids 1 and 2 with the voids is occupied by the other atoms ? (2019 Main, 10 Jan II)
position of atoms as shown below. The radius of atom B is
2
twice that of atom A. The unit cell edge length is 50% more is (a) hcp lattice- A, tetrahedral voids-B
solid 2 than in 1. What is the approximate packing efficiency 3
in solid 2? 1
(2019 Main, 8 April II) (b) hcp lattice-A, tetrahedral voids-B
3
A A A A 1
(c) hcp lattice-B, tetrahedral voids-A
A 3
A A A
2
B (d) hcp lattice-B, tetrahedral voids-A
A 3
A A A
A 11. Which primitive unit cell has unequal edge lengths
A A A A
( a ≠ b ≠ c ) and all axial angles different from 90°?
Solid 1 Solid 2 (2019 Main, 10 Jan I)
(a) Hexagonal (b) Monoclinic
(a) 65% (b) 90% (c) 75% (d) 45% (c) Tetragonal (d) Triclinic
Solid State 131

12. At 100°C, copper (Cu) has FCC unit cell structure with cell 21. A compound M p X q has cubic close packing (ccp)
edge length of x Å. What is the approximate density of Cu (in g arrangement of X . Its unit cell structure is shown below. The
cm −3 ) at this temperature? empirical formula of the compound, is (2012)
[Atomic mass of Cu = 63.55 u] (2019 Main, 9 Jan II)
211 205
(a) 3 (b) 3
x x M
105 422 X
(c) 3 (d) 3
x x
13. The one that is extensively used as a piezoelectric material is
(2019 Main, 9 Jan I) (a) MX (b) MX 2 (c) M 2 X (d) M 5 X 14
(a) quartz (b) tridymite 22. The packing efficiency of the two-dimensional square unit
(c) amorphous silica (d) mica cell shown below is (2010)
14. Which type of ‘defect’ has the presence of cations in the
interstitial sites? (2018 Main)
(a) Schottky defect (b) Vacancy defect
(c) Frenkel defect (d) Metal deficiency defect
l
15. A metal crystallises in a face centred cubic structure. If the
edge length of its unit cell is ‘a’, the closest approach between (a) 39.27% (b) 68.02% (c) 74.05% (d) 78.54%
two atoms in metallic crystal will be (2017 Main) 23. Which of the following fcc structure contains cations in
a alternate tetrahedral voids?
(a) 2 a (b) 2 2 a (c) 2 a (d) (2005, 1M)
2 (a) NaCl (b) ZnS (c) Na 2 O (d) CaF2
16. Sodium metal crystallises in a body centred cubic lattice with 24. A substance Ax B y crystallises in a face centred cubic (fcc)
a unit cell edge of 4.29Å. The radius of sodium atom is lattice in which atoms A occupy each corner of the cube and
approximately (2015 Main) atoms B occupy the centres of each face of the cube. Identify
(a) 1.86Å (b) 3.22Å the correct composition of the substance Ax B y (2002, 1M)
(c) 5.72Å (d) 0.93Å (a) AB3 (b) A4 B3 (c) A3 B
17. CsCl crystallises in body centred cubic lattice. If ‘a’ its edge (d) composition cannot be specified
length, then which of the following expressions is correct? 25. In a solid AB having the NaCl structure, A atoms occupy the
(2014 Main) corners of the cubic unit cell. If all the face centred atoms
3a
(a) r + + rCl − = 3a (b) r + + rCl − = along one of the axes are removed, then the resultant
Cs Cs 2 stoichiometry of the solid is (2001, S, 1M)
3 (a) AB2 (b) A2 B (c) A4 B3 (d) A3 B4
(c) r + rCl − = a (d) r + rCl − = 3a
Cs + 2 Cs +
26. The coordination number of a metal crystallising in a
18. The arrangement of X − ions around A + ion in solid AX is hexagonal close-packed structure is (1999, 2M)
given in the figure (not drawn to scale). If the radius of X − is (a) 12 (b) 4 (c) 8 (d) 6
250 pm, the radius of A + is (2013 Adv.)


Objective Questions II
X
A+
(One or more than one correct option)
27. The cubic unit cell structure of a compound containing cation
M and anion X is shown below. When compared to the anion,
the cation has smaller ionic radius. Choose the correct
(a) 104 pm (b) 125 pm statement(s). (2020 Adv.)
(c) 183 pm (d) 57 pm
19. Experimentally it was found that a metal oxide has formula
M 0.98O. Metal M, present as M 2+ and M 3 + in its oxide.
Fraction of the metal which exists as M 3+ would be
(a) 7.01% (b) 4.08% (2013 Main)
(c) 6.05% (d) 5.08%
20. Which of the following exists as covalent crystals in the solid
state? (2013 Main)
(a) Iodine (b) Silicon
M X
(c) Sulphur (d) Phosphorus
132 Solid State

(a) The empirical formula of the compound is MX. Assertion and Reason
(b) The cation M and anion X have different coordination
geometries. Read the following questions and answer as per the direction
given below :
(c) The ratio of M — X bond length of the cubic unit cell edge
(a) Statement I is correct Statement II is correct Statement II is
length is 0.866.
the correct explanation of Statement I
(d) The ratio of the ionic radii of cation M to anion X is 0.414.
(b) Statement I is correct Statement II is correct Statement II is
28. The correct statement(s) for cubic close packed (ccp) three not the correct explanation of Statement I
dimensional structure is (are) (2016 Adv.) (c) Statement I is correct Statement II is incorrect
(a) The number of the nearest neighbours of an atom present (d) Statement I is incorrect Statement II is correct
in the topmost layer is 12
33. Statement I In any ionic soid (MX ) with Schottky defects,
(b) The packing efficiency of atom is 74%
(c) The number of octahedral and tetrahedral voids per atom the number of positive and negative ions are same.
are 1 and 2, respectively Statement II Equal numbers of cation and anion vacancies
(d) The unit cell edge length is 2 2 times the radius of the atom are present. (2001, 1M)

29. If the unit cell of a mineral has cubic close packed (ccp) array Passage Based Questions
of oxygen atoms with m fraction of octahedral holes
occupied by aluminium ions and n fraction of tetrahedral Passage
holes occupied by magnesium ions, m and n respectively, are In hexagonal systems of crystals, a frequently encountered
(2015 Adv.)
1 1 1 arrangement of atoms is described as a hexagonal prism. Here, the
(a) , (b) 1 , top and bottom of the cell are regular hexagons and three atoms are
2 8 4
sandwiched in between them. A space-filling model of this
1 1 1 1 structure, called hexagonal close-packed (hcp), is constituted of a
(c) , (d) ,
2 2 4 8 sphere on a flat surface surrounded in the same plane by six
30. The correct statement(s) regarding defects in solids is/are identical spheres as closely as possible. Three spheres are then
(a) Frenkel defect is usually favoured by a very small placed over the first layer so that they touch each other and
difference in the sizes of cation and anion (1999) represent the second layer. Each one of these three spheres touches
(b) Frenkel defect is a dislocation defect three spheres of the bottom layer.
(c) Trapping of an electron in the lattice leads to the formation
Finally, the second layer is covered with a third layer that is
of F-centre identical to the bottom layer in relative position. Assume radius of
every sphere to be ‘r ’.
(d) Schottky defects have no effect on the physical properties
of solids 34. The number of atoms in one of this hcp unit cell is
(2008, 3 × 4M = 12M)
31. Which of the following statement(s) is/are correct? (a) 4 (b) 6 (c) 12 (d) 17
(a) The coordination number of each type of ion in CsCl crystal
is 8 (1998, 2M)
35. The volume of this hcp unit cell is
64 r3
(b) A metal that crystallises in bcc structure has a coordination (a) 24 2 r3 (b) 16 2 r3 (c) 12 2 r3 (d)
number of 12 3 3
(c) A unit cell of an ionic crystal shares some of its ions with 36. The empty space in this hcp unit cell is
other unit cells (a) 74 % (b) 47.6 % (c) 32 % (d) 26 %
(d) The length of the unit cell in NaCl is 552 pm.
(rNa + = 95 pm; rCl − =181pm) Match the Column
37. Match the crystal system / unit cells mentioned in Column I
Numerical Answer Type Question with their characteristic features mentioned in Column II.
(2007, 6M)
32. Consider an ionic solid MX with NaCl structure. Construct a
new structure (Z) whose unit cell is constructed from the unit Column I Column II
cell of MX following the sequential instruction given below. A. Simple cubic and face p. have these cell
Neglect the charge balance. (2018 Adv.) centred cubic parameters a = b = c and
(a) Remove all the anions (X ) except the central one α = β = γ = 90°
(b) Replace all the face centered cations (M ) by anions (X ) B. Cubic and rhombohedral q. are two crystal systems
(c) Remove all the corner cations (M ) C. Cubic and tetragonal r. have only two
(d) Replace the central anion (X ) with cation (M ) crystallographic angles
 Number of anions  of 90°
The value of   in Z is ___ D. Hexagonal and s. belong to same crystal
 Number of cations 
monoclinic system
Solid State 133

Integer Answer Type Questions 44. (i) Marbles of diameter 10 mm are to be put in a square area
of side 40 mm so that their centres are within this area.
38. A crystalline solid of a pure substance has a face-centred
cubic structure with a cell edge of 400 pm. If the density of (ii) Find the maximum number of marbles per unit area and
the substance in the crystal is 8 g cm −3 , then the number of deduce an expression for calculating it. (2003, 4M)

atoms present in 256 g of the crystal is N × 1024 . The value 45. The figures given below show the location of atoms in three
of N is (2017 Adv.) crystallographic planes in a fcc lattice. Draw the unit cell for
39. The number of hexagonal faces that are present in a truncated the corresponding structures and identify these planes in
octahedron is (2011) your diagram. (2000)
−1
40. Silver (atomic weight = 108 g mol ) has a density of
10.5 g cm−3 . The number of silver atoms on a surface of area
10−12 m2 can be expressed in scientific notation as y × 10 x.
The value of x is (2010)

Subjective Questions
46. A metal crystallises into two cubic phases, face centred cubic
41. The edge length of unit cell of a metal having molecular (fcc) and body centred cubic (bcc), whose unit cell lengths
weight 75 g/mol is 5 Å which crystallises in cubic lattice. If are 3.5 and 3.0 Å, respectively. Calculate the ratio of
the density is 2 g/cc then find the radius of metal atom. densities of fcc and bcc. (1999, 3M)
(N A = 6 × 1023 ). Give the answer in pm. (2006, 3M) 47. Chromium metal crystallises with a body centred cubic
42. An element crystallises in fcc lattice having edge length lattice. The length of the unit edge is found to be 287 pm.
400 pm. Calculate the maximum diameter of atom which can Calculate the atomic radius . What would be the density of
be placed in interstitial site without distorting the structure. chromium in g/cm 3 ? (1997, 3M)
(2005, 2M) 48. A metallic element crystallises into a lattice containing a
43. The crystal AB (rock salt structure) has molecular weight sequence of layers of ABABAB…… Any packing of layers
6.023 y u. Where, y is an arbitrary number in u. If the leaves out voids in the lattice. What percentage of this lattice
minimum distance betweeen cation and anion is y1 / 3 nm and is empty space? (1996, 3M)
the observed density is 20 kg / m3 . Find the (i) density in 49. Sodium crystallises in a bcc cubic lattice with the cell edge,
kg / m3 and (ii) type of defect. (2004, 2M) a = 4.29 Å. What is the radius of sodium atom? (1994, 2M)

Answers
1. (a) 2. (d) 3. (b) 4. (c) 5. (b) 6. (d) 7. (b) 8. (d) 9. (a)
10. (c) 11. (d) 12. (d) 13. (a) 14. (c) 15. (d) 16. (a) 17. (c) 18. (a)
19. (b) 20. (b) 21. (b) 22. (d) 23. (b) 24. (a) 25. (d) 26. (a) 27. (a,c)
28. (b,c,d) 29. (a) 30. (b,c) 31. (a,c,d) 32. (c) 33. (a) 34. (b) 35. (a) 36. (d)
37. A → p, s; B → q; C → q; D → q, r 38. (2) 39. (8) 40. (7) 41. (217 pm) 42. (117 pm)
46. (1.26) 47. (7.3 g/cm 3) 48. (0.74) 49. (1.86 Å)

Hints & Solutions


1. Number of O2− ions in ccp = 4 So, formula is (M 1 )2 (M 2 ) O4.
This must be neutral. Both metals must have + 8charge on total.
Total charge on oxide = − 8
From given options :
Number of tetrahedral voids = 8
Oxidaion number of M 1 = + 2
Number of voids = 4 Oxidaion number of M = + 4
 2
Number of cation M 1 occupies 50% of octahedral voids,
∴ (2 × + 2) + (1 × + 4 ) = + 8
50
×4=2 Hence, correct option is (a).
100
Number of cation M 2 occupies 12.5% of tetrahedral voids of 2. We know that,
oxide lattice, Z×M
d=
12.5 a3 × N A
× 8=1
100 Given, density d = 6 . 17 g,
134 Solid State

edge length a = 300 pm = 3 × 10− 8 cm, [As the atoms A are present at the edges only zA =
1
× 8 = 1,
23 −1 8
N A = 6 × 10 mol
atom B is present only at the body centre zB = 1]
2×M
∴6 . 17 = (For bcc, Z = 2) 4 3  4 3 
(3 × 10− 8 )3 × 6 × 1023  πrA × 1 +  πrB × 1
3  3 
⇒ M = 50 g / mol ∴ PE 2 =
a23
Hence, number of molecules present in 200 g of X 2 is
4 3 4 4 3
w 200 πrA + π (2rA )3 πrA × 9
N = × NA = × N A = 4N A 3 3 π
M 50 = 3
= 3 =
(2 3 rA ) 8 × 3 3 rA3 2 3
3. The ratio of number of atoms present in simple cubic, body = 90.72% ≈ 90%
centred cubic and face centered cubic structure are 1 : 2 : 4
respectively. 6. Interstitial compounds are formed when a neutral atom with a
small radius occupies in an interstitial hole (tetrahedral or
4. In fcc unit cell, two tetrahedral voids are formed on each of the octahedral voids) in a transition metal’s hcp or ccp lattices
four non-parallel body diagonals of the cube at a distance of (host lattice). Examples of small atoms (guest atom) are H, B, C
3a / 4 from every corner along the body diagonal. and N.
One of the body diagonal Interstitial compounds are non-stoichiometric (Birtholide) in
of cubic unit cell
A composition. They are very hard with very high melting points.
The electrical conductivity of interstitial compounds are
1
— AB
4 comparable to that of the pure metal. These are chemically
√3 a Position of 2 TVs unreactive in nature.
— A′ 1 on one of the
4 —(AB)
2 body diagonal 7. The number of element ‘B ’ in the crystal structure = 4 N
of the unit cell
Number of tetrahedral voids = 2N
B′ 1 Number of octahedral voids = N
—(AB)
4 N 4
B ∴Number of ‘A’ in the crystal = = =2
√3 a 2 2

4 Number of oxygen (O) atoms = 2N = 2 × 4 = 8
∴The structure of bimetallic oxide = A2 B4 O8 = AB2 O4
The angle between body diagonal and an edge is cos−1 (1/ 3 ).
8. For body centred cubic bcc structure,
So, the projection of the line on an edge is a/ 4. Similarly, other
tetrahedral void also will be a/4 away. So, the distance between 3
radius (R ) = a ...(i)
 a a a 4
these two is a − − = .
 4  4 2 Where, a = edge length
According to question, the structure of cubic unit cell can be
shown as follows:
5. Key Idea Packing efficiency
R 2r R
Volume occupied by sphere
= × 100
Volume of cube

Given,
rB = 2rA
50
a2 = a1 + a1 = 15
. a1
100
For bcc lattice
4 rA = 3 a1 a

3 a1 4 rA ∴ a = 2(R + r) ...(ii)
rA = ⇒ a1 =
4 3 On substituting the value of R from Eq. (i) to Eq. (ii), we get
 4r  3  4 rA  a 3
∴ a2 = 15
.  A =   = a+ r
 3 2  3 2 4
a 3 2a − 3a
a2 = 2 3 rA r= − a=
2 4 4
4 3 4 a(2 − 3 )
πrA × zA + πrB3 × zB r=
Packing efficiency = 3 3 4
a23 r = 0.067a
Solid State 135

9. Density of a crystal 14. It is the ‘‘Frenkel defect’’ in which cations leave their original
M ×Z d × N A × a3 site and occupy interstitial site as shown below.
d= 3
⇒M =
NA × a Z + – + – + – + – Original
– + – + – + – + vacant site
3 −3
Given, d = 9 × 10 kg m + – + – + – + – of cation
– + – – + – +
M = Molar mass of the solid + – + – + – + –
Cation in
+ interstitial site
Z = 4 (for fcc crystal) – + – + – + – +
N A = Avogadro’s constant = 6 × 1023 mol −1 + – + – + – + –
– + – + – + – +
a = Edge length of the unit cell
= 200 2 pm = 200 2 × 10−12 m 15. For fcc arrangement, 4 r = 2a
On substituting all the given values, we get where, r = radius and a = edge length
(9 × 103 ) kg m −3 × (6 × 1023 ) mol −1 × (200 2 × 10−12 )3 m 3 2a
= ∴ Closest distance = 2 r = =
a
4 2 2
= 0.0305 kg mol −1
16. For bcc unit cell, 3 a = 4a
10. Total effective number of atoms in hcp unit lattice = Number of
3 3
octahedral voids in hcp = 6 r= a= × 4.29Å = 1.85Å
4 4
∴Number of tetrahedral voids (TV) in hcp
r = 1.85Å ≈ 1.86Å
= 2 × Number of atoms in hcp lattice
= 2 × 6 = 12 17. In CsCl, Cl − lies at corners of simple cube and Cs+ at the body
As, formula of the lattice is A2 B3 . centre. Hence, along the body diagonal, Cs+ and Cl − touch each
Suppose, A B other so rCs + + rCl − = 2r
1  Calculation of r
 × TV (hcp)
3  In ∆EDF,
1 G
⇒ × 12 6 B
3 H
2 r A
⇒ 1
3
2r
c a
⇒ 2 3 F
r
C
1 b
So, A = tetrahedral voids, B = hcp lattice a
3 E a D

11. Triclinic primitive unit cell has dimensions as, a ≠ b ≠ c and Body centred cubic unit cell
α ≠ β ≠ γ ≠ 90° .
FD = b = a2 + a2 = 2 a
Among the seven basic or primitive crystalline systems, the
triclinic system is most unsymmetrical. In other cases, edge In ∆AFD,
length and axial angles are given as follows : c2 = a2 + b2 = a2 + ( 2 a)2 = a2 + 2 a2
Hexagonal : a = b ≠ c and α = β = 90°, γ = 120° c2 = 3 a2 ⇒ c = 3 a
Monoclinic : a ≠ b ≠ c and α = γ = 90°, β ≠ 90° As ∆ AFD is an equilateral triangle.
Tetragonal : a ≠ b ≠ c and α = β = γ = 90° ∴ 3a = 4r [QC = 3r + r + r]
12. For fcc, rank of the unit cell (Z ) = 4 ⇒ r=
3a
Mass of one Cu-atom, M = 63.55 u 4
Avogadro’s number, N A = 6.023 × 1023 atom 3 3
Hence, rCs + + rCl− = 2 r = 2 × a= a
4 2
Edge length, a = x Å = x × 10−8 cm
Z×M 18. PLAN Given arrangement represents octahedral void and for this
density (d ) = r+ (cation)
= 0.414
N A × a3 r− (anion)
4 × 63.55 422.048 r( A + )
= = g cm−3 = 0.414
6.023 × 1023 × (x × 10−8 )3 x3 r( X − )
13. Piezoelectric materials are those materials that produce an r( A + ) = 0.414 × r( X − ) = 0.414 × 250pm
electric current when they are placed under mechanical stress. = 103.5 pm ≈ 104 pm
Crystalline solids can be used as piezoelectric material hence,
quartz is a correct answer.
136 Solid State

19. From the valency of M 2+ and M 3+ , it is clear that three Number of B is removed because it is not present on face centres.
M 2+ ions will be replaced by M 3+ causing a loss of one ⇒ A remaining = 4 − 1 = 3, B remaining = 4,
M 3+ ion. Total loss of them from one molecule of Formula = A3 B4
MO = 1 − 0.98 = 0.02
26. Three consecutive layers of atoms in hexagonal close packed
Total M 3+ present in one molecule of lattice is shown below:
MO = 2 × 0.02 = 0.04
That M 2+ and M 3+ = 0.98 A
0.04 × 100
Thus, % of M 3+ = = 4.08%
0.98
20. Silicon exists as covalent crystal in solid state. (Network like
structure, as seen in diamond). X B
21. Contribution of atom from the edge centre is 1/4. Therefore,
number of
1
M = × 4 (from edge centre) + 1(from body centre) = 2 A
4
1 1
Number of X = × 8 (from corners) + × 6
8 2
Atom X is in contact of 12 like atoms, 6 from layer B and 3 from
(from face centre) = 4
top and bottom layers A each.
⇒ Empirical formula = M 2 X 4 = MX 2
1 27. (a) The empirical formula of the compound:
22. Contribution of circle from corner of square = Contribution of M and X : M  X ⇒ MX
4 1
2× 
1
4× 
⇒ Effective number of circle per square  2  4
1 (b) Coordination number of both M and X is 8.
= × 4 + 1(at centre) = 2
4
a2 a2 3 3
⇒ Area occupied by circle = 2πr2, r = radius. (c) Distance between M and X = + = a= a
4 2 4 2
Also, diagonal of square 4 r = 2 L, where L = side of square.
⇒ 0.866a
Area occupied by circles
⇒ Packing fraction = (d) rM : rX = ( 3 − 1) : 1 ⇒ 0.732:1, thus statement (d) is
Area of square
incorrect.
2πr2 2πr2 π
= 2 = = = 0.785
L 8r2 4 28. (a) Nearest neighbour in the topmost layer of ccp structure is 9
⇒ % packing efficiency = 78.5%. thus, incorrect.
2−
(b) Packing efficiency is 74% thus, correct.
23. In ZnS, S (sulphide ions) are present at fcc positions giving
(c) Tetrahedral voids = 2
four sulphide ions per unit cell. To comply with 1 : 1
stoichiometry, four Zn 2+ ions must be present in four alternate Octahedral voids = 1 per atom thus, correct.
tetrahedral voids out of eight tetrahedral voids present.
In NaCl, Na + ions are present in octahedral voids while in Na 2O,
Na + ions are present in all its tetrahedral voids giving the desired A
2 : 1 stoichiometry. In CaF2 , Ca 2+ ions occupies fcc positions
and all the tetrahedral voids are occupied by fluoride ions. a
1
24. In cubic system, a corner contribute th part of atom to one unit
8
1 a C
B
cell and a face centre contribute part of atom to one unit cell.
2
Therefore,
4
1
Number of A per unit cell = × 8 = 1 (d) Edge length, a = r = 2 2r
8 2
1 thus, correct
Number of B per unit cell = × 6 = 3
2 Explanation Edge length = a
⇒ Formula = AB3 Radius = r

+
25. In NaCl, Na occupies body centre and edge centres while Cl AC 2 = AB 2 + BC 2
occupies corners and face centres, giving four Na + and four Cl −
(4 r)2 = a2 + a2 = 2a2
per unit cell. In the present case A represent Cl − and B represents
Na + . Two face centres lies on one axis. 4 r = 2a
1
⇒ Number of A removed = 2 × = 1
2
Solid State 137

2 a 32. The unit cell of initial structure of ionic solid MX looks like
⇒ r= a=
4 2 2 Cl– (at face centre)
⇒ a=2 2 r
Cl– (at corner)
In ccp structure, number of spheres is 4.
4  Na+ (at face)
Hence, volume of 4 spheres = 4  πr3
3 

Total volume of unit cell = a3 = (2 2r)3


Na+ (at corner edge)
% of packing efficiency
Volume of 4 spheres
= In NaCl type of solids cations (Na + ) occupy the octahedral
Volume of unit cell
voids while anions (Cl − ) occupy the face centre positions.
4 
4  πr3 However, as per the demand of problem the position of cations
3 
= × 100 and anions are swapped.
[ 2( 2r)]3 We also know that (for 1 unit cell)
= 74.05% − ~ 74%
(A) Total number of atoms at FCC = 4
29. Oxide ions are at ccp positions, hence 4O2– ions. Also, there (B) Total number of octahedral voids = 4
are four octahedral voids and eight tetrahedral voids. Since ‘m’ (as no. of atoms at FCC = No. of octahedral voids)
fraction of octahedral voids contain Al 3+ and ‘n’ fraction of Now taking the conditions one by one
tetrahedral voids contain Mg2+ ions, to maintain
(i) If we remove all the anions except the central one than
etectroneutrality 2(2Al 3+ = + 6charge) and number of left anions.
(Mg2+ = + 2charge), will make unit cell neutral = 4 −3 = 1
2 1 1
Hence: m = = , n= (ii) If we replace all the face centred cations by anions than
4 2 8 effective number of cations will be = 4 − 3 = 1
30. (a) Wrong statement. A small difference in sizes of cation and Likewise effective number of anions will be = 1+ 3 = 4
anion favour Schottky defect while Frenkel defect is (iii) If we remove all the corner cations then effective number of
favoured by large difference in sizes of cation and anion. cations will be 1− 1= 0
(b) Correct statement. In Frenkel defect the smaller atom or ion
gets dislocated from its normal lattice positions and occupies (iv) If we replace central anion with cation then effective
the interstitial space. number of cations will be 0 + 1= 1
(c) Correct Statement In F-centre defect, some anions leave Likewise effective number of anions will be 4 − 1= 3
the lattice and the vacant sites hold the electrons trapped in it Thus, as the final outcome, total number of cations present in
maintaining the overall electroneutrality of solid. Z after fulfilling all the four sequential instructions = 1
(d) Wrong statement : In Schottky defect, some of the atoms or Likewise, total number of anions = 3
ions remaining absent from their normal lattice points Number of anions 3
Hence, the value of = =3
without distorting the original unit cell dimension. This Number of cations 1
lowers the density of solid.
33. In ionic solid MX (1 : 1 solid) same number of M n+ and X n− ions
31. (a) The unit cell of CsCl has bcc arrangement of ions in which are lost in Schottky defect to maintain electroneutrality of solid.
each ion has eight oppositely charged ions around it in the
nearest neighbours as shown below :
A

Cs+ 34. B
Q
Cl–
C M
Unit cell of CsCl P

(b) In bcc, coordination number of atom is 8. R S N


(c) In an unit cell, a corner is shared in eight unit cells and a face A hcp unit cell
centre is shared between two adjacent unit cells. Contribution of atoms from corner = 1/ 6
(d) In NaCl unit cell; 2(rNa+ + rCl − ) = a Contribution from face centre = 1 / 2
⇒ a = 2 (95 + 181) = 552 pm 1 1
⇒ Total number of atoms per unit cell = 12 × + 2 × + 3= 6
Hence, a, c, d are correct. 6 2
138 Solid State

35. In close packed arrangement, side of the base = 2r ⇒ a3 (Volume of unit cell) = 6.83 × 10−23 cm 3
⇒ RS = r ⇒ a = 4 × 10−8 cm = 4 × 10−10 m
Also MNR is equilateral triangle, ∠ PRS = 30°
RS 3 ⇒ Surface area of unit cell = a2 = 1.6 × 10−19 m 2
In triangle PRS , cos 30° = =
PR 2 ⇒ Number of unit cells on 10−12 m 2 surface
2 2
⇒ PR = RS = r 10−12
3 3 = = 6.25 × 106
1.6 × 10−19
2
In right angle triangle PQR : PQ = QR 2 − PR 2 = 2 r
3 Q There are two atoms (effectively) on one face of unit cell
2 Number of atoms on 10−12 m 2 surface = 2 × number of unit cell
⇒ Height of hexagon = 2PQ = 4 r
3 = 1.25 × 107 . [Q y × 10x ]
3 2 ⇒ x = 7 ⇒ y = 125
.
⇒ Volume = Area of base × height = 6 (2r)2 × 4 r
4 3
41. From the given information, the number of atoms per unit cell
= 24 2 r3 and therefore, type of unit cell can be known as
NM
36. Packing fraction =
Volume occupied by atoms ρ=
Volume of unit cell NA a3
4
= 6 × πr3 ×
1
= 0.74 ρ N A a3 2 × 6 × 1023 × (5 × 10−8 cm )3
⇒ N = = = 2 ( bcc)
3 24 2 r3 M 75
⇒ Fraction of empty space = 1 − 0.74 = 0.26 = 26% ⇒ In bcc, 4 r = 3a
3 3
⇒ r= a= × 5 × 10−10 m
37. A. Simple cubic and face centred cubic both have cell 4 4
parameters a = b = c and α = β = γ = 90°. Also both of them = 2.17 × 10−10 m = 217 pm
belongs to same, cubic, crystal system.
B. The cubic and rhombohedral crystal system belongs to 42. In a cubic crystal system, there are two types of voids known as
different crystal system. octahedral and tetrahedral voids. If r1 is the radius of void and r2
is the radius of atom creating these voids then
C. Cubic and tetragonal are two different types of crystal
 r1 
systems having different cell parameters.   = 0.414
 r2  octa
D. Hexagonal and monoclinic are two different crystal system
and both have two of their crystallographic angles of 90°.  r1 
and   = 0.225
4×M  r2  tetra
38. Density (ρ) = 8 =
N A (4 × 10−8 cm )3 The above radius ratio values indicate that octahedral void has
−24 256 larger radius, hence for maximum diameter of atom to be present
⇒ M = 128 × 10 NA ⇒ No. of atoms = × NA
M in interstitial space :
=
256
× N A = 2 × 1024 r1 = 0.414 r2
128 × 10−24 N A Also in fcc, 4 r2 = 2a
39. The truncated octahedron is the 14-faced Archimedean solid, ⇒ Diameter required (2r1 ) = (2r2 ) × 0.414
with 14 total faces : 6 squares and 8 regular hexagons. a
= × 0.414
The truncated octahedron is formed by removing the six right 2
square pyramids one from each point of a regular octahedron as : 400 × 0.414
= = 117 pm
2
43. (i) In rock salt like crystal AB, there are four AB units per unit
cell. Therefore, density (d ) is
4 × 6.023 y
d=
6.023 × 1023 × 8 y × 10−27
[Q a = 2 y1/ 3 nm = 2 y1/ 3 × 10−9 m ]
Truncated octahedron
= 5 × 103 g/m 3 = 5 kg/m 3
Truncated octahedron unfolded in two-dimension
(ii) Since, observed density is greater than expected, theoretical
40. Ag crystallises in fcc unit cell with 4 atoms per unit cell. density, there must be some excess metal occupying
4 × 108 interstitial spaces. This type of defect is known as metal
ρ= = 10.5 g cm −3.
6.023 × 1023 × a3 excess defect.
Solid State 139

44. (i) Side of square = 40 mm N


46. Density ∝
Diameter of marble = 10 mm a3
3 3
Number of marble spheres along an edge of square with their d1 N 1  a2  4 3 
⇒ =   =   = 1.26
centres within the square = 5 (shown in diagram) d2 N 2  a1  2  3.5
10 mm
47. In bcc unit cell, 4 r = 3a
3a 3
⇒ r (Cr) = = × 287 pm = 124.3 pm
4 4
NM
Density of solid =
N A ⋅ a3
N = Number of atoms per unit cell, M = Molar mass
3
a = Volume of cubic unit cell, N A = Avogadro’s number
3
40 mm 2 × 52 g  1 
= ×  = 7.3 g / cm 3
Maximum number of marbles per unit area = 5 × 5 = 25 6.023 × 1023  2.87 × 10−8 cm 
(ii) If x mm is the side of square and d is diameter of marble then 48. The given arrangement : ABABAB...... represents hexagonal
maximum number of marbles on square area with centres close-packed unit cell in which there are six atoms per unit cell.
within square area can be known by the following general Also, volume of unit cell = 24 2r3.
formula :
2 Volume occupied by atoms
x  ⇒ Packing fraction =
N =  + 1 Volume of unit cell
d 
4 1
= 6 × πr3 × = 0.74
3 24 2 r3
⇒ Percent empty space = 100 (1 − 0.74) = 26%
45. 49. In bcc arrangement of atoms : 4 r = 3a, atoms on body
diagonal remain in contact
3a 3 × 4.29
⇒ r= = = 1.86 Å
4 4
9
Solutions and
Colligative Properties

Topic 1 Solution and Vapour Pressure of Liquid Solutions


Objective Questions I (Only one correct option) 5. For the solution of the gases w , x, y and z in water at 298 K,
1. Henry’s constant (in kbar) for four gasesα, β, γ and δ in water the Henry’s law constants ( K H ) are 0.5, 2, 35 and 40 K bar,
at 298 K is given below : respectively. The correct plot for the given data is
(2019 Main, 8 April II)
α β γ δ
Partial pressure

Partial pressure
KH 50 2 2 × 10−5 0.5 z
3 −3
(density of water = 10 kg m at 298 K) This table implies (a) z
(b) y
that (2020 Main, 3 Sep I) y x
(a) α has the highest solubility in water at a given pressure x w w
(b) solubility of γ at 308 K is lower than at 298 K (0, 0) Mole fraction (0, 0) Mole fraction
(c) The pressure of a 55.5 molal solution of γ is 1 bar of water of water
(d) The pressure of a 55.5 molal solution of δ is 250 bar z
z y
Partial pressure

Partial pressure

2. The mole fraction of a solvent in aqueous solution of a solute is


0.8. The molality (in mol kg −1 ) of the aqueous solution is y xw
(c) (d)
(2019 Main, 12 April I)
(a) 13.88 × 10−2 (b) 13.88 × 10−1 x
−3 w
(c) 13.88 (d) 13.88 × 10
(0, 0) Mole fraction (0, 0) Mole fraction
3. What would be the molality of 20% (mass/mass) aqueous of water of water
solution of KI? (Molar mass of KI = 166 g mol −1 )
6. The vapour pressures of pure liquids A and B are 400 and 600
(2019 Main, 9 April I)
mmHg, respectively at 298 K. On mixing the two liquids, the
(a) 1.48 (b) 1.51 (c) 1.35 (d) 1.08 sum of their initial volumes is equal to the volume of the final
4. Liquid M and liquid N form an ideal solution. The vapour mixture. The mole fraction of liquid B is 0.5 in the mixture.
pressures of pure liquids M and N are 450 and 700 mmHg, The vapour pressure of the final solution, the mole fractions
respectively, at the same temperature. Then correct statement of components A and B in vapour phase, respectively are
is (2019 Main, 9 April I) (2019 Main, 8 April I)
x M = mole fraction of M in solution; (a) 450 mmHg, 0.4, 0.6 (b) 500 mmHg, 0.5, 0.5
x N = mole fraction of N in solution; (c) 450 mmHg, 0.5,0.5 (d) 500 mmHg, 0.4,0.6
y M = mole fraction of M in vapour phase; 7. Liquids A and B form an ideal solution in the entire
y N = mole fraction of N in vapour phase composition range. At 350 K, the vapour pressures of pure A
x y x y
(a) M > M (b) M = M and pure B are 7 × 103 Pa and 12 × 103 Pa, respectively. The
xN yN xN yN
composition of the vapour in equilibrium with a solution
xM yM containing 40 mole percent of A at this temperature is
(c) < (d) ( x M − y M ) < ( x N − y N )
xN yN (2019 Main, 10 Jan I)
Solutions and Colligative Properties 141

(a) xA = 0.76; xB = 0.24 Objective Questions II


(b) xA = 0.28; xB = 0.72
(One or more than one correct option)
(c) xA = 0.4; xB = 0.6
(d) xA = 0.37; xB = 0.63 15. For a solution formed by mixing liquids L and M , the vapour
pressure of L plotted against the mole fraction of M in
8. Which one of the following statements regarding Henry’s law solution is shown in the following figure. Here xL and x M
is not correct? (2019 Main, 8 Jan I)
represent mole fractions of L and M , respectively, in the
(a) Different gases have different K H (Henry’s law solution. The correct statement(s) applicable to this system is
constant) values at the same temperature (are) (2017 Adv.)
(b) Higher the value of K H at a given pressure, higher is the
solubility of the gas in the liquids
(c) The value of K H increases with increase of temperature Z
and K H is function of the nature of the gas pL
(d) The partial pressure of the gas in vapour phase is
proportional to the mole fraction of the gas in the solution
9. 18 g of glucose (C6 H12 O6 ) is added to 178.2 g water. The
1 XM 0
vapour pressure of water (in torr) for this aqueous solution is
(2016 Main) (a) The point Z represents vapour pressure of pure liquid M
(a) 76.0 (b) 752.4 and Raoult’s law is obeyed from xL = 0 to xL = 1
(c) 759.0 (d) 7.6 (b) Attractive intermolecular interactions between L - L in
10. The vapour pressure of acetone at 20°C is 185 torr. pure liquid L and M - M in pure liquid M are stronger
When 1.2 g of a non-volatile substance was dissolved in 100 g than those between L - M when mixed in solution
of acetone at 20°C, its vapour pressure was 183 Torr. The (c) The point Z represents vapour pressure of pure liquid M
molar mass of the substance is (2015, 1M) and Raoult’s law is obeyed when xL → 0
(a) 32 (b) 64 (d) The point Z represents vapour pressure of pure liquid L
(a) 128 (b) 488 and Raoult’s law is obeyed when xL → 1
11. The Henry’s law constant for the solubility of N2 gas in water 16. Mixture(s) showing positive deviation from Raoult’s law at
at 298 K is 1.0 × 105 atm. The mole fraction of N2 in air is 0.8. 35°C is (are) (2016 Adv.)
The number of moles of N2 from air dissolved in 10 moles of (a) carbon tetrachloride + methanol
water of 298 K and 5 atm pressure is (2009) (b) carbon disulphide + acetone
(a) 4.0 × 10− 4 (b) 4.0 × 10− 5 (c) benzene + toluene
(c) 5.0 × 10− 4 (d) 4.0 × 10− 6 (d) phenol + aniline
12. A molal solution is one that contains one mole of a solute in Numerical Answer Type Questions
(1986, 1M)
17. Liquids A and B form ideal solution for all compositions of A
(a) 1000 g of the solvent (b) 1 L of the solvent and B at 25°C. Two such solutions with 0.25 and 0.50 mole
(c) 1 L of the solution (d) 22.4 L of the solution fractions of A have the total vapour pressure of 0.3 and 0.4
13. For a dilute solution, Raoult’s law states that (1985, 1M) bar, respectively. What is the vapour pressure of pure liquid B
in bar? (2020 Adv.)
(a) the lowering of vapour pressure is equal to the mole
fraction of solute 18. The mole fraction of urea in an aqueous urea solution
(b) the relative lowering of vapour pressure is equal to the containing 900 g of water is 0.05. If the density of the solution
mole fraction of solute is 1.2 g cm −3 , then molarity of urea solution is …… (Given
(c) the relative lowering of vapour pressure is proportional data : Molar masses of urea and water are 60 g mol −1 and 18 g
to the amount of solute in solution mol −1 , respectively) (2019 Adv.)

(d) the vapour pressure of the solution is equal to the mole 19. Liquids A and B form ideal solution over the entire range of
fraction of solvent composition. At temperature T, equimolar binary solution of
14. An azeotropic solution of two liquids has boiling point lower liquids A and B has vapour pressure 45 torr. At the same
than either of them when it (1981, 1M)
temperature, a new solution of A and B having mole fractions
x A and xB , respectively, has vapour pressure of 22.5 torr. The
(a) shows negative deviation from Raoult’s law value of x A / xB in the new solution is ____.
(b) shows no deviation from Raoult’s law
(c) shows positive deviation from Raoult’s law (Given that the vapour pressure of pure liquid A is 20 Torr at
(d) is saturated temperature T) (2018 Adv.)
142 Solutions and Colligative Properties

True/False the solution is 600 mm Hg. What is the molecular weight of


20. Following statement is true only under some specific the solid substance? (1990, 3M)

conditions. Write the condition for it. 26. The vapour pressure of a dilute aqueous solution of glucose
“Two volatile and miscible liquids can be separated by (C6 H12 O6 ) is 750 mm of mercury at 373 K.
fractional distillation into pure components.” (1994) Calculate (i) molality and (ii) mole fraction of the solute.
(1989, 3M)
Subjective Questions
21. The vapour pressure of two miscible liquids A and B are 300 27. The vapour pressure of ethanol and methanol are 44.5 and
and 500 mm of Hg respectively. In a flask 10 moles of A is 88.7 mm Hg respectively. An ideal solution is formed at the
mixed with 12 moles of B. However, as soon as B is added, A same temperature by the mixing 60 g of ethanol with 40 g of
starts polymerising into a completely insoluble solid. The methanol. Calculate the total vapour pressure of the solution
polymerisation follows first-order kinetics. After 100 min, and the mole fraction of methanol in the vapour. (1986, 4M)
0.525 mole of a solute is dissolved which arrests the 28. An organic compound (Cx H2 y O y ) was burnt with twice the
polymerisation completely. The final vapour pressure of the
solution is 400 mm of Hg. Estimate the rate constant of the amount of oxygen needed for complete combustion to CO2
polymerisation reaction. Assume negligible volume change and H2 O. The hot gases when cooled to 0°C and 1 atm
on mixing and polymerisation and ideal behaviour for the pressure, measured 2.24 L. The water collected during
final solution. (2001, 4M) cooling weight 0.9 g. The vapour pressure of pure water at
20°C is 17.5 mm Hg and is lowered by 0.104 mm when 50 g
22. The molar volume of liquid benzene (density = 0.877 g/mL) of the organic compound are dissolved in 1000 g of water.
increases by a factor of 2750 as it vaporises at 20° C and that Give the molecular formula of the organic compound.
of liquid toluene (density = 0.867 g mL−1 ) increases by a (1983, 5M)
factor of 7720 at 20° C. A solution of benzene and toluene at 29. Two liquids A and B form ideal solution. At 300 K, the
20° C has a vapour pressure of 45.0 torr. Find the mole vapour pressure of a solution containing 1 mole of A and
fraction of benzene in the vapour above the solution. 3 moles of B is 550 mm of Hg. At the same temperature, if
(1996, 3M)
one more mole of B is added to this solution, the vapour
23. What weight of the non-volatile solute urea pressure of the solution increases by 10 mm of Hg.
(NH2 — CO —NH2 ) needs to be dissolved in 100 g of water, Determine the vapour pressure of A and B in their pure
in order to decrease the vapour pressure of water by 25%? states. (1982, 4M)
What will be the molality of the solution? (1993, 3M)
30. The vapour pressure of pure benzene is 639.70 mm of Hg and
24. The degree of dissociation of Ca(NO3 )2 in a dilute aqueous the vapour pressure of solution of a solute in benzene at the
solution, containing 7.0 g of the salt per 100 g of water at same temperature is 631.9 mm of Hg. Calculate the molality
100° C is 70%. If the vapour-pressure of water at 100° C is of the solution. (1981, 3M)
760mm, calculate the vapour pressure of the solution. 31. What is the molarity and molality of a 13% solution
(1991, 4M) (by weight) of sulphuric acid with a density of 1.02 g/mL ? To
25. The vapour pressure of pure benzene at a certain temperature what volume should 100 mL of this solution be diluted in
is 640 mm Hg. A non-volatile, non-electrolyte solid weighing order to prepare a 1.5 N solution ? (1978, 2M)
2.175 g is added to 39.0 g of benzene. The vapour pressure of

Topic 2 Colligative Properties


Objective Questions I (Only one correct option) constants are in the ratio of 1 : 5. The ratio of the elevation in
∆T ( A )
1. A solution is prepared by dissolving 0.6 g of urea (molar mass their boiling points, b , is
= 60 g mol −1 ) and 1.8 g of glucose (molar mass = 180 g ∆Tb ( B ) (2019 Main, 10 April II)

mol −1 ) in 100 mL of water at 27°C. The osmotic pressure of (a) 5 : 1 (b) 10 : 1 (c) 1 : 5 (d) 1 : 0.2
the solution is ( R = 0.08206 L atm K −1 mol −1 ) 3. At room temperature, a dilute solution of urea is prepared by
(2019 Main, 12 April II) dissolving 0.60 g of urea in 360 g of water. If the vapour
(a) 8.2 atm (b) 2.46 atm pressure of pure water at this temperature is 35 mm Hg,
lowering of vapour pressure will be
(c) 4.92 atm (d) 1.64 atm
(Molar mass of urea = 60 g mol −1 ) (2019 Main, 10 April I)
2. 1 g of a non-volatile, non-electrolyte solute is dissolved in
(a) 0.027 mmHg (b) 0.031 mmHg
100 g of two different solvents A and B, whose ebullisocopic
(c) 0.017 mmHg (d) 0.028 mmHg
Solutions and Colligative Properties 143

4. Molal depression constant for a solvent is 4.0 K kg mol −1 . 12. For 1 molal aqueous solution of the following compounds,
The depression in the freezing point of the solvent for 0.03 which one will show the highest freezing point? (2018 Main)
mol kg −1 solution of K 2SO4 is (a) [Co(H2 O)6 ]Cl 3 (b) [Co(H2 O)5 Cl]Cl 2 ⋅ H2 O
(Assume complete dissociation of the electrolyte) (c) [Co(H2 O)4 Cl 2 ]Cl ⋅ 2H2 O (d) [Co(H2 O)3 Cl 3 ] ⋅ 3H2 O
(2019 Main, 9 April II)
13. The freezing point of benzene decreases by 0.45°C when 0.2 g
(a) 0.18 K (b) 0.36 K (c) 0.12 K (d) 0.24 K of acetic acid is added to 20 g of benzene. If acetic acid
5. The osmotic pressure of a dilute solution of an ionic associates to form a dimer in benzene, percentage association
of acetic acid in benzene will be
compound XY in water is four times that of a solution of
. K kg mol − 1)
(K f for benzene = 512
0.01 M BaCl2 in water. Assuming complete dissociation of (2017 Main)
the given ionic compounds in water, the concentration of (a) 64.6 % (b) 80.4 % (c) 74.6 % (d) 94.6 %
XY (in mol L−1 ) in solution is (2019 Main, 9 April I) 14. Pure water freezes at 273 K and 1 bar. The addition of 34.5 g
−2 −4
(a) 4 × 10 (b) 16 × 10 of ethanol to 500 g of water changes the freezing point of the
(c) 4 × 10 −4
(d) 6 × 10 −2 solution. Use the freezing point depression constant of water
as 2 K kg mol −1 . The figures shown below represent plots of
6. Molecules of benzoic acid (C6H5COOH) dimerise in vapour pressure (V.P.) versus temperature (T ). [Molecular
benzene. ‘w’ g of the acid dissolved in 30 g of benzene weight of ethanol is 46 g mol −1 ] (2017 Adv.)
shows a depression in freezing point equal to 2 K. If the Among the following, the option representing change in the
percentage association of the acid to form dimer in the
freezing point is
solution is 80, then w is
(Given that K f = 5 K kg mol− 1 , molar mass of benzoic V.P./bar Water Water

V.P./bar
acid = 122 g mol− 1 ) (2019 Main, 12 Jan II)
(a) 1
Ice
(b) 1
Ice
Water+Ethanol Water+Ethanol
(a) 1.8 g (b) 1.0 g (c) 2.4 g (d) 1.5 g
7. Freezing point of a 4% aqueous solution of X is equal to 270 273 T/K 271 273 T/K
freezing point of 12% aqueous solution of Y . If molecular
weight of X is A, then molecular weight of Y is Water Water
Ice Ice
(2019 Main, 12 Jan I)
V.P./bar

V.P./bar
(c) 1 (d) 1
(a) 4A (b) 2A (c) 3A (d) A
Water+Ethanol Water+Ethanol
8. K 2HgI4 is 40% ionised in aqueous solution. The value of its
270 273 T/K 271 273 T/K
van’t Hoff factor (i) is (2019 Main, 11 Jan II)
(a) 1.6 (b) 1.8 (c) 2.2 (d) 2.0 15. Consider separate solution of 0.500 M C2 H5 OH (aq),
0.100 M Mg 3 (PO4 )2 (aq), 0.250 M KBr (aq) and 0.125 M
9. The freezing point of a diluted milk sample is found to be
Na 3 PO4 (aq) at 25°C. Which statement is true about these
−0.2° C, while it should have been −0.5°C for pure milk.
solution, assuming all salts to be strong electrolytes?
How much water has been added to pure milk to make the
(a) They all have the same osmotic pressure (2014 Main)
diluted sample? (2019 Main, 11 Jan I)
(b) 0.100 M Mg 3 (PO4 )2 (aq) has the highest osmotic pressure
(a) 2 cups of water to 3 cups of pure milk
(b) 1 cup of water to 3 cups of pure milk (c) 0.125 M Na 3 PO4 (aq) has the highest osmotic pressure
(c) 3 cups of water to 2 cups of pure milk (d) 0.500 M C2 H5 OH (aq) has the highest osmotic pressure
(d) 1 cup of water to 2 cups of pure milk 16. For a dilute solution containing 2.5 g of a non-volatile
10. Elevation in the boiling point for 1 molal solution of non-electrolyte solute in 100 g of water, the elevation in
glucose is 2 K . The depression in the freezing point for 2 boiling point at 1 atm pressure is 2°C. Assuming concentration
molal solution of glucose in the same solvent is 2 K. The of solute is much lower than the concentration of solvent, the
relation between K b and K f is (2019 Main, 10 Jan II)
vapour pressure (mm of Hg) of the solution is (take K b = 0.76
K kg mol −1 ). (2012)
(a) K b = 15
. Kf (b) K b = 0.5 K f
(c) K b = K f (d) K b = 2K f (a) 724 (b) 740 (c) 736 (d) 718

11. A solution contain 62 g of ethylene glycol in 250 g of water 17. The freezing point (in° C) of solution containing 0.1 g of
is cooled upto –10º C. If K f for water is 1.86 K kg mol , −1 K 3 [Fe(CN)6 ] (mol. wt. 329) in 100 g of water
( K f = 1.86 K kg mol −1 ) is
then amount of water (in g) separated as ice is (2011)
(2019 Main, 9 Jan II) (a) − 2.3 × 10−2 (b) −5.7 × 10−2
(a) 32 (b) 48 (c) 64 (d) 16 (c) −5.7 × 10−3 (d) −1.2 × 10−2
144 Solutions and Colligative Properties

18. When 20 g of naphthoic acid (C11 H8 O2 ) is dissolved in 50 g Numerical Answer Type Questions
of benzene (K f = 1.72 K kg mol − 1 ), a freezing point
depression of 2 K is observed. The van’t Hoff factor ( i ) is
27. The osmotic pressure of a solution of NaCl is 0.10 atm and
(2007, 3M)
that of a glucose solution is 0.20 atm. The osmotic pressure of
a solution formed by mixing 1 L of the sodium chloride
(a) 0.5 (b) 1 (c) 2 (d) 3
solution with 2 L of the glucose solution is x × 10−3 atm. x is
19. The elevation in boiling point, when 13.44 g of freshly
prepared CuCl 2 are added to one kilogram of water, is. [Some ……… (nearest integer). (2020 Main, 4 Sep II)

useful data, K b = 0.52 K kg mol − 1 , molecular weight of 28. On dissolving 0.5 g of a non-volatile non-ionic solute to 39 g of
CuCl 2 =134.4 g]. (2005, 1M) benzene, its vapour pressure decreases from 650 mmHg to 640
(a) 0.05 (b) 0.1 (c) 0.16 (d) 0.21 mmHg. The depression of freezing point of benzene (in K)
20. 0.004 M Na 2 SO4 is isotonic with 0.01 M glucose. Degree of upon addition of the solute is ............ (Given data : Molar
dissociation of Na 2 SO4 is (2004, S, 1M)
mass and the molal freezing point depression constant of
(a) 75% (b) 50% (c) 25% (d) 85% benzene are 78 g mol − 1 and 512. K kg mol − 1 , respectively).
(2019 Adv.)
21. During depression of freezing point in a solution the
following are in equilibrium (2003) 29. The plot given below shows p −T curves (where p is the
(a) liquid solvent, solid solvent pressure and T is the temperature) for two solvents X and Y
(b) liquid solvent, solid solute and isomolal solution of NaCl in these solvents. NaCl
(c) liquid solute, solid solute completely dissociates in both the solvents.
(d) liquid solute, solid solvent
22. The molecular weight of benzoic acid in benzene as 1 2 3 4
determined by depression in freezing point method
corresponds to
760
(a) ionisation of benzoic acid (1996, 1M)
(b) dimerisation of benzoic acid
Pressure (mmHg)

1. Solvent X
(c) trimerisation of benzoic acid 2. Solution of NaCl in solvent X
(d) solvation of benzoic acid 3. Solvent Y
4. Solution of NaCl in solvent Y
23. The freezing point of equimolal aqueous solutions will be
highest for (1990, 1M)
(a) C6 H5 NH3 Cl (aniline hydrochloride)
(b) Ca(NO3 )2
360

362

367
368

(c) La(NO3 )3
(d) C6 H12 O6 (glucose) Temperature (K)
24. Which of the following 0.1 M aqueous solution will have the
lowest freezing point? (1989, 1M) On addition of equal number of moles of a non-volatile solute
(a) Potassium sulphate (b) Sodium chloride S in equal amount (in kg) of these solvents, the elevation of
(c) Urea (d) Glucose boiling point of solvent X is three times that of solvent Y.
25. When mercuric iodide is added to the aqueous solution of Solute S is known to undergo dimerisation in these solvents.
potassium iodide (1987, 2M) If the degree of dimerisation is 0.7 in solvent Y , the degree of
(a) freezing point is raised dimerisation in solvent X is ____. (2018 Adv.)

(b) freezing point is lowered


(c) freezing point does not change Subjective Questions
(d) boiling point does not change 30. 75.2 g of C 6 H5 OH (phenol) is dissolved in a solvent of
Objective Questions II K f = 14. If the depression in freezing point is 7 K, then find
(One or more than one correct option) the percentage of phenol that dimerises. (2006, 2M)

26. In the depression of freezing point experiment, it is found that 31. 1.22 g C6 H5 COOH is added into two solvents and data of ∆Tb
the (1999, 3M) and K b are given as :
(a) vapour pressure of the solution is less than that of pure (i) In 100 g CH3COCH3 ∆Tb = 0.17, K b = 1.7 K kg/mol
solvent (ii) In 100 g benzene, ∆Tb = 0.13 and K b = 2.6 K kg/mol
(b) vapour pressure of the solution is more than that of pure
solvent Find out the molecular weight of C6 H5 COOH in both the
(c) only solute molecules solidify at the freezing point cases and interpret the result. (2004, 2M)
(d) only solvent molecules solidify at the freezing point
Solutions and Colligative Properties 145

32. Consider the three solvents of identical molar masses. Match Freezing point depression constant of ethanol
their boiling point with their K b values ( K ethanol
f ) = 2.0 K kg mol −1

Solvents Boiling point Kb values Boiling point elevation constant of water


( K bwater ) = 0.52 K kg mol −1
X 100°C 0.92
Boiling point elevation constant of ethanol
Y 27°C 0.63
( K bethanol ) = 1.2 K kg mol −1
Z 283°C 0.53
Standard freezing point of water = 273 K
(2003)
Standard freezing point of ethanol = 155.7 K
33. To 500 cm3 of water, 3.0 × 10–3 kg of acetic acid is added. If Standard boiling point of water = 373 K
23% of acetic acid is dissociated, what will be the Standard boiling point of ethanol = 351.5 K
depression in freezing point? K f and density of water are Vapour pressure of pure water = 32.8 mm Hg
1.86 K kg –1 mol –1 and 0.997 g cm−3 , respectively. Vapour pressure of pure ethanol = 40 mm Hg
(2000, 3M) Molecular weight of water = 18 g mol −1
34. Nitrobenzene is formed as the major product along with a Molecular weight of ethanol = 46 g mol −1
minor product in the reaction of benzene with a hot mixture of
In answering the following questions, consider the solutions
nitric acid and sulphuric acid. The minor product consists of
to be ideal dilute solutions and solutes to be non-volatile and
carbon : 42.86%, hydrogen : 2.40%, nitrogen : 16.67% and
non-dissociative. (2008, 3 × 4M = 12M)
oxygen : 38.07%,
(i) Calculate the empirical formula of the minor product. 37. The freezing point of the solution M is
(ii) When 5.5 g of the minor product is dissolved in 45 g of (a) 268.7 K (b) 268.5 K
benzene, the boiling point of the solution is 1.84°C higher (c) 234.2 K (d) 150.9 K
than that of pure benzene. Calculate the molar mass of the 38. The vapour pressure of the solution M is
minor product then determine its molecular and structural (a) 39.3 mm Hg (b) 36.0 mm Hg
formula. (Molal boiling point elevation constant of benzene is (c) 29.5 mm Hg (d) 28.8 mm Hg
2.53 K kg mol − 1 ). (1999)
39. Water is added to the solution M such that the mole fraction
35. A solution of a non-volatile solute in water freezes at of water in the solution becomes 0.9. The boiling point of this
− 0.30° C.The vapour pressure of pure water at 298 K is 23.51 solution is
mm Hg and K f for water is 1.86 K kg mol −1 . Calculate the (a) 380.4 K (b) 376.2 K
vapour pressure of this solution at 298 K. (1998, 4M)
(c) 375.5 K (d) 354.7 K
36. Addition of 0.643 g of a compound to 50 mL of benzene
(density : 0.879 g/mL) lowers the freezing point from 5.51° C Fill in the Blank
to 5.03° C. If K f for benzene is 5.12, calculate the molecular 40. Given that ∆T f is the depression in freezing point of the
weight of the compound. (1992, 2M) solvent in a solution of a non-volatile solute of molality, m,
the quantity lim ( ∆T f / m ) is equal to ...... (1994, 1M)
m→ 0
Passage Based Questions
Passage 1 Integer Answer Type Question
Properties such as boiling point, freezing point and vapour pressure 41. If the freezing point of a 0.01 molal aqueous solution of a
of a pure solvent change when solute molecules are added to get cobalt (III) chloride-ammonia complex (which behaves as a
homogeneous solution. These are called colligative properties. strong electrolyte) is − 0.0558°C, the number of chloride(s)
Applications of colligative properties are very useful in day-to-day in the coordination sphere of the complex is
life. . K kg mol −1 ]
[ K f of water = 186 (2015 Adv.)
One of its examples is the use of ethylene glycol and water mixture
as anti-freezing liquid in the radiator of automobiles. 42. MX 2 dissociates into M 2+ and X − ions in an aqueous
A solution M is prepared by mixing ethanol and water. The mole solution, with a degree of dissociation (α ) of 0.5. The ratio of
fraction of ethanol in the mixture is 0.9. the observed depression of freezing point of the aqueous
Given, freezing point depression constant of water solution to the value of the depression of freezing point in the
( K water
f ) = 1.86 K kg mol −1 absence of ionic dissociation is (2014 Adv.)
Answers
Topic 1 Topic 2
1. (d) 2. (c) 3. (b) 4. (a) 1. (c) 2. (c) 3. (c) 4. (b)
5. (a) 6. (d) 7. (b) 8. (b) 5. (d) 6. (c) 7. (c) 8. (b)
9. (b) 10. (b) 11. (a) 12. (b) 9. (c) 10. (d) 11. (c) 12. (d)
13. (d) 14. (b) 15. (a) 16. (a)
13. (b) 14. (c) 15. (b, d) 16. (a, b)
17. (a) 18. (a) 19. (c) 20. (a)
17. (0.2) 18. (2.98 M) 19. (19) 20. (T)
21. (a) 22. (b) 23. (d) 24. (a)
22. (0.72) 23. (18.5) 24. (746.32 mm) 25. (a) 26. (a,d) 27. (167.00) 28. (1.02)
25. (65.25) 26. (0.75) 27. (0.657) 30. (0.158) 29. (0.05) 30. (75%) 32. (0.23°C) 35. (23.44 mm)
31. (180.40 mL) 36. (156 g/mol) 37. (d) 38. (a) 39. (b)
40. (K f ) 42. (2)

Hints & Solutions


Topic 1 Solution and Vapour Pressure 2. Mass of solute (w2 ) × 1000
Key Idea Molality ( m) =
Molar mass of solute (M 2 ) ×
of Liquid Solutions mass of solvent (w1 )
1. Henry’s equation for solubility (S ) of a gas (β) in a liquid is w2 1000
expressed in terms of mole- fraction of the gas (χ B ) at a given m= ×
M2 w1
temperature,
1000
p = KH × χ B = KH × S and also, m = n2 ×
n1 × M1
1
So, solubility of gas, S ∝ at T (K) and given pressure.
KH X solvent = 0.8 (Given) It means that nsolvent (n1 ) = 0.8 and
Order of solubility of the gases (high the value of K H , lower is nsolute (n2 ) = 0.2
the solubility) : 1000 1000
Using formula m = n2 × = 0.2 × = 13.88 mol kg
γ >δ >β >α n1 × M 1 0.8 × 18
−1
So, option (a) is not correct.
Again, K H ∝ temperature, i.e. solubility of the gas will decrease
3. Key Idea Molality is defined as number of moles of solute
with increase in temperature also.
per kg of solvent.
But this conclusion cannot be drawn from table.
w2 1000
So, option (b) is not correct. m = ×
Mw2 w1
We know, mole-fraction of a solute ( B ) in a binary aqueous
solution, w2 = mass of solute, Mw2 = molecular mass of solute
18 m w1 = mass of solvent.
χB = [Qm = molality]
1000 + 18 m The molality of 20% (mass/mass) aqueous solution of KI
can be calculated by following formula.
For γ at m = 55.5 molal
w × 1000
 18 × 55.5  m= 2
p = K Hγ × m = (2 × 10−5 ) ×   Mw2 × w1
 1000 + 18 × 55.5
20% aqueous solution of KI means that 20 gm of KI is present in
= 1. 81 × 10−5 K bar = 1. 8 × 10−2 bar 80 gm solvent.
So, option (c) is not correct. 20 1000
m= × = 1. 506 ≈ 1. 51mol/kg
For δ at m = 55.5 molal 166 80
 18 × 55.5  Key Idea For a solution of volatile liquids the partial vapour
p = K Hδ = (0.5) ×   4.
 1000 + 18 × 55.5 pressure of each component of the solution is directly
~ 250bar proportional to its mole fraction present in solution. This is
= 0.2498k bar −
known as Raoult’s law.
So, option (d) is correct.
Liquid M and N form an ideal solution. Vapour pressures of pure
liquids M and N are 450 and 700 mm Hg respectively.
Solutions and Colligative Properties 147

∴ pºN > pºM p°A = 7 × 103 Pa, p°B = 12 × 103 Pa


So, by using Raoult’s law On substituting the given values in Eq. (i),
yN > xN …(i) we get
and xM > yM …(ii) p = 0.4 × 7 × 103 + 0.6 × 12 × 103
Multiplying (i) and (ii) we get
= 10 × 103 Pa = 1 × 104 Pa
yN xM > yM xN
xM yM In vapour phase,
∴ > p x′ p° 0.4 × 7 × 103
xN yN xA = A = A A = = 0.28
p p 1 × 104
Thus, correct relation is (a).
∴ xB = 1 − 0.28 = 0.72 [Q xA + xB = 1]
5. According to Henry’s law (at constant temperature)
pgas = K H × χ gas (solute) = K H × [1 − χ H 2 O (solvent) ] 8. At constant temperature, solubility of a gas (S ) varies inversely
pgas = K H − K H χ H 2 O with Henry’s law constant (K H )
Pressure P
pgas = partial pressure of the gas above its solution with a liquid KH = =
(solvent) say water. Solubility of a gas in a liquid S
χ gas = mole fraction of the gas (solute) in the solution. Thus, higher the value of K H at a given pressure, the lower is the
χ H 2 O = mole fraction of water (solvent). solubility of the gas in the liquid.
9. Key Idea Vapour pressure of water ( p° ) = 760 torr
Mass (g)
Number of moles of glucose =
pgas pgas Molecular mass (g mol −1 )
KH KH 18 g
= = 0.1 mol
180 gmol −1
Molar mass of water = 18 g/mol
χH2O=0 χH2O=0 Mass of water (given) = 178.2g
χgas=1 χgas=0 Number of moles of water
Mass of water 178. 2g
[ i.e. pgas = K H ] Higher the value of K H, higher = = = 9.9 mol
Molar mass of water 18 g / mol
will be the partial pressure of the gas ( pgas ), at a given
temperature. The plot of pgas vs χ H2O gives a (−ve) slope. Total number of moles = (0.1 + 9.9) moles = 10 moles
pgas = K H − K H × χ H2O Now, mole fraction of glucose in solution = Change in pressure
with respect to initial pressure
Comparing the above equation with the equation of straight line
y = mx + c ∆p 01 .
i.e. = or ∆p = 0.01p° = 0.01 × 760 = 7.6 torr
Slope = − K H , intercept = K H p° 10
So, (i) Higher the value of K H, more (−ve) will be the slope and it ∴ Vapour pressure of solution = (760 − 7.6) torr = 752.4 torr
is for z (K H = 40 K bar ) 10. Given, p° = 185 Torr at 20°C
(ii) Higher the value of K H, higher with the value of intercept, i.e.
partial pressure and it is also for z. ps = 183 Torr at 20°C
Mass of non-volatile substance, m = 1.2 g
6. (d) According to Dalton’s law of partial pressure
Mass of acetone taken = 100g
ptotal = pA + pB = pA° χ A + pB° χ B …(i)
M =?
Given, pAº = 400 mm Hg, pBº = 600 mm Hg p° − ps n
As, we have =
χ B = 0.5, χ A + χ B = 1 ∴ χ A = 0.5 ps N
On substituting the given values in Eq. (i). We get, Putting the values, we get,
ptotal = 400 × 0.5 + 600 × 0.5 = 500 mm Hg 1.2
Mole fraction of A in vapour phase, 185 − 183 M 2 1.2 × 58
= ⇒ =
p pAº χ A 0.5 × 400 183 100 183 100 × M
YA = A = = = 0.4 58
ptotal ptotal 500
183 × 1. 2 × 58
Mole of B in vapour phase, ∴ M =
2 × 100
YA + YB = 1
M = 63.684 = 64 g/mol
YB = 1 − 0. 4 = 0.6
7. For ideal solution, 11. Give, K H = 1 × 105 atm, χ N 2 = 0.8
p = x′ A p°A + x′ B p°B nH 2 O = 10 moles, ptotal = 5 atm
Q x′ A = 0.4, x′ B = 0.6 pN 2 = ptotal × χ N 2 = 5 × 0.8 = 4 atm
148 Solutions and Colligative Properties

According to Henry’s law, 17. Using Raoult’s law equation for a mixture of volatile liquids.
pN 2 = K H × χ N 2 pT = p°A χ A + pB° χ B
4 = 105 × χ N 2 0.3 = 0.25 χ pA° + 0.75 χpB° … (i)
−5
χ N 2 = 4 × 10 0.4 = 0.5 χ p°A + 0.5 χpB° … (ii)
nN 2 By solving equation (i) and (ii)
= 4 × 10−5 p°A = 0.6 bar and pB° = 0.2 bar
nN 2 + nH 2 O
nN 2 Thus, the vapour pressure of pure liquid B in bar is 0.2.
= 4 × 10−5 Number of moles of solute × 1000
nN 2 + 10 18. Key Idea Molarity (M ) =
Volume of solution (in mL)
nN 2 = 4 × 10−4 Mass
Also, volume =
12. Molality = moles of solute present in 1.0 kg of solvent. Density
Given, mole fraction of urea (χ urea ) = 0.05
13. The relative lowering of vapour pressure : Mass of water = 900g
− ∆p Density = 12 . g/cm 3
= χ2 (mole fraction of solute)
p° nurea 900
χ urea = [Q Moles of water = = 50]
14. In case of positive deviation from Raoult's law, the observed nurea + 50 18
vapour pressure is greater than the ideal vapour pressure and nurea
boiling point of azeotrope becomes lower than either of pure 0.05 = ⇒ 19nurea = 50
nurea + 50
liquid.
nurea = 2.6315 moles
15. The graph shown indicates that there is positive deviation
because the observed vapour pressure of L is greater than the wurea = nurea × (M ⋅ wt )urea = (2. 6315 × 60)g
ideal pressure 2.6315 × 60 + 900  Mass of solution 
V= Q Density =
12
.  Volume of solution 
Z = 88157
. mL
pL
Real Now, molarity
1000
= Number of moles of solute ×
al Volume of solution (mL)
Ide
2.6315 × 1000
= = 2.98 M
88157
.
1 xM 0
Since, deviation is positive, the intermolecular force between L 19. Key Idea Use the formula
and M is smaller than the same in pure L and pure M. pTotal = pA° × χ A + pB° × χ B
Also as xL → 1, xM → 0, the real curve approaching ideal curve 1
and for equimolar solutions χ A = χ B =
where Raoult’s law will be obeyed. 2
16. When intermolecular attraction between two components A and Given, pTotal = 45 torr for equimolar solution
B in the mixture is same as between A and A or B and B, hence it pA° = 20 torr
is a case of ideal solution. 1 1 1
So, 45 = pA° × + pB° × = ( pA° + pB° )
When intermolecular attraction between A and B in a mixture is 2 2 2
smaller than that between A and A or B and B, then mixture is
more vaporised, bp is lowered. It is a case of positive deviation or pA° + pB° = 90 torr …(i)
from Raoult’s law. But we know pA° = 20 torr
When intermolecular attraction between A and B is higher than
that between A and A or B and B, then mixture is less vaporised, so, pB° = 90 − 20 = 70torr (From Eq. (i))
bp is increased. It is a case of negative deviation. Now, for the new solution from the same formula
(a) Methanol molecules (CH3OH) are hydrogen bonded. In a Given, pTotal = 22 . 5 torr
mixture of CCl 4 and CH3OH, extent of H-bonding is So, 22 . 5 = 20χ A + 70 (1− χ A ) (As χ A + χ B =1)
decreased. Mixture is more vaporised thus, positive deviation or 22.5 = 70 − 50χ A
from Raoult’s law. 70 − 22 . 5
(b) Acetone molecules have higher intermolecular attraction due So, χA = = 0.95
50
to dipole-dipole interaction. With CS2, this interaction is
Thus χ B = 1 − 0.95 = 0.05 (as χ A + χ B =1)
decreased thus, positive deviation.
Hence, the ratio
(c) Mixture of benzene and toluene forms ideal solution.
χ A 0.95
(d) Phenol and aniline have higher interaction due to = = 19
χ B 0.05
intermolecular H-bonding. Hence, negative deviation.
Solutions and Colligative Properties 149

20. It will be true only if boiling points of two liquids are n1


⇒ Mole fraction of solvent =
significantly different. n1 + in2
21. Let after 100 min, x moles of A are remaining unpolymerised  100
 
 18 
moles of B = 12 = = 0.982
 100 7
Moles of non-volatile solute = 0.525   + 2.4 ×
 18  164
χ
⇒ Mole fraction of A = ⇒ p = p0 χ1 = 760 × 0.982
χ + 12 + 0.525
12 (VP of H2O at 100°C = 760 mm of Hg)
Mole fraction of B = = 746.32 mm
χ + 12 + 0.525
 χ   12  25. According to Raoult’s law :
⇒ 400 =   × 300 +   × 500 p = p0 χ1
 χ + 12.525  χ + 12.525
 n1 
⇒ χ = 9.9 ⇒ 600 = 640  
 n1 + n2 
⇒ Moles of A polymerised in 100 min = 10 – 9.9 = 0.10
1 10 1 10 n2 64 1
⇒ k = ln = ln min −1 ⇒ = −1=
t 9.9 100 9.9 n1 60 15
= 1.005 × 10−4 min −1 39 1
⇒ n2 = × = 0.033
78 15
78
22. Volume of 1.0 mole liquid benzene = mL = 88.94 mL 2.175
0.877 ⇒ = 0.033
M
⇒ Molar volume of benzene vapour at 20°C
⇒ M = 65.25
88.94 × 2750
= L = 244.58 L
1000 26. At 373 K (bp) of H2O, Vapour pressure = 760 mm
0.082 × 293 VP of solution at 373 K = 750 mm
⇒ VP of pure benzene at 20°C = × 760 mm
244.58 ⇒ p = p0χ 1 or 750 =760 χ 1
= 74.65 mm 75
⇒ χ1 = = mole fraction of H2O
Similarly; molar volume of toluene vapour 76
75 1
92 7720 ⇒ χ2 = 1 − = = mole fraction of solute
= × L = 819.2L 76 76
0.867 1000
n2 1
0.082 × 293 Now =
⇒ VP of pure toluene = × 760 mm = 22.3mm n1 + n2 76
819.2
n1
Now, let mole fraction of benzene in the liquid phase = χ ⇒ = 75
n2
⇒ 4.65 χ + 22.3 (1 – χ ) = 45 n2 1000
⇒ χ = 0.43 ⇒ Molality = × 1000 = = 0.74 molal
n1M 1 75 × 18
⇒ Mole fraction of benzene in vapour phase
60
Partial vapour pressure of benzene 27. Moles of ethanol = = 1.3
= 46
Total vapour pressure 40
74.65 × 0.43 Moles of methanol = = 1.25
= = 0.72 32
45 1.3
⇒ Mole fraction of ethanol = = 0.51
23. Vapour pressure of solution = 0.75 × VP of water 1.3 + 1.25
⇒ Vapour pressure of solution = pethanol + pmethanol
⇒ 75 = 100 χ 1 : χ 1 = mole fraction of solute
3 1 = 0.51 × 44.5 + 0.49 × 88.7
⇒ χ 1 = and χ 2 = 1 − χ 1 = = 66.16 mm
4 4
χ 2 n2 1 n 100 Mole fraction of methanol in vapour phase
⇒ = = ⇒ n2 = 1 = = 1.85 pmethanol 43.463
χ 1 n1 3 3 18 × 3 = = = 0.657
Total vapour pressure 66.16
⇒ Weight of urea = 1.85 × 60 = 111 g
n 1000 1 1000 28. From lowering of vapour pressure information :
Molality = 2 × = × = 18.5
n1 M1 3 18 0.104 n2
= χ2 =
17.5 n1 + n2
24. Ca(NO3 )2 r Ca 2+ + 2NO3−
1−α α 2α n1
⇒ + 1 = 168.27
i = 1 + 2α where, α = 0.7 ⇒ i = 1 + 2 × 0.7= 2.4 n2
150 Solutions and Colligative Properties

n1 1000 M n
⇒ = 167.27 ⇒ × = 167.27 For the relation, π = CRT = RT
n2 18 50 V
⇒ M = 150 g/mol Given, mass of urea = 0.6 g
Also, the combustion reaction is : Molar mass of urea = 60 g mol− 1
CxH2 yO y + xO2 → xCO2 + yH2O Mass of glucose = 1.8 g
Q 18 y g of H2O is produced from 1.0 mole of compound. Molar mass of glucose = 180 g mol− 1
0.9 1
∴0.9 g of H2O will be produced from = mol [ n (urea ) + n2 (glucose)]
18 y 20 y π= 2 RT
V
x  0.6 18 . 
⇒ At the end, moles of O2 left =  + 
20 y  60 180
= × 1000 × 0.0821 × 300
x 100
moles of CO2 formed =
20 y = (0.01 + 0.01) × 10 × 0.0821 × 300
2x 2.24 π = 4 . 92 atm
⇒ Total moles of gases at STP = =
20 y 22.4
2. The expression of elevation of boiling point,
⇒ x= y w2 × 1000
⇒ Molar mass; 150 = 12x + 2x + 16x = 30x ∆Tb = K b × m × i = k b × ×i
M 2 × w1
150
⇒ x= =5 where, m = molality
30
⇒ Formula = C5H10O5 i = van’t Hoff factor = 1(for
non-electrolyte/non-associable)
29. When 1.0 mole of A is mixed with 3 moles of B. w2 = mass of solute in g = 1g (present in both of the solutions)
550 = 0.25 pA° + 0.75 pB° …(i) M 2 = molar mass of solute in g mol −1 (same solute in both
When 1.0 mole of A is mixed with 4 moles of B. of the solutions)
560 = 0.20 pA° + 0.80 pB° …(ii) w1 = mass of solvent in g = 100 g (for both of the
solvents A and B)
Now, solving (i) and (ii) pA° = 400 mm
K b = ebullioscopic constant
pB° = 600mm. So, the expression becomes,
∆Tb ∝ K b
30. According to Raoult’s law :
∆Tb ( A ) K b ( A ) 1  K b ( A) 1 
p = p0χ 1 ⇒631.9 = 639.7 χ 1 ⇒ = = Given K ( B ) = 5 
∆Tb ( B ) K b ( B ) 5  
⇒ χ 1 = 0.9878 ⇒ χ 2 = 0.0122 b

0.0122
⇒ Molality = × 1000 = 0.158 3. Key Idea For dilute solution, lowering of vapour pressure
0.9878 × 78 ∆p
(∆p) = p0 − p and relative lowering of vapour pressure =
p0
31. Let us consider 1.0 L of solution.
Weight of solution = 1000 × 1.02 = 1020 g which is a colligative property of solutions.
∆p
Weight of H2SO4 = 1020 ×
13
= 132.60 g = χ B × i ⇒ ∆p = χ B × i × p0
100 p0
Weight of H2O = 1020 – 132.60 = 887.40 g where, p0 = vapour pressure of pure solvent
132.60 i = van’t Hoff factor
⇒ Molarity = = 1.353 M
98 χ B = mole fraction of solute
132.60 1000
Molality = × = 1.525 m Given,
98 887.40
Normality = 2 × M = 2.706 p° = vapour pressure of pure water of 25º C = 35 mm Hg
⇒ 2.706 × 100 = 1.5 V ⇒ V = 180.40 mL χ B = mole fraction of solute (urea)
0.60
nB 60 0.01 0.01
= = = = = 0.0005
Topic 2 Colligative Properties nA + nB 360
+
0.60 20 + 0.01 20.01
18 60
1. Key Idea Osmotic pressure is proportional to the molarity
i = van’t Hoff factor = 1 (for urea)
(C) of the solution at a given temperature (T).
Now, according to Raoult’s law
Thus, π ∝ C, π = CRT (for dilute solution)
n ∆p = χ B × i × pº
π = RT On substituting the above given values, we get
V
∆p = 0.0005 × 1 × 35 = 0.0175 mm Hg
Solutions and Colligative Properties 151

4. Key Idea Depression in freezing point (∆ Tf ) is given by On substituting all the given values in Eq. (i), we get
∆Tf = iK f m 0.6 × 5 × w × 1000
2= , w = 2.44 g
i = vant Hoff factor 122 × 30
K f = molal depression constant Thus, weight of acid (w) is 2.4 g.
m = molality
7. Given, Freezing point of 4% aqueous solution of X .
K f = 4.0 K kg mol −1 (Given) = Freezing point of 12% aqueous solution of Y
m = 0.03 mol kg −1
(Given) or (∆T f )X = (∆T f )Y [Q∆T f = T f° − T f ]
∆T f = ? K f × mX = K f mY
For K 2SO4, i = 3 where, mX and mY are molality of X and Y, respectively.
It can be verified by the following equation : or mX = mY
K 2SO4 2K + + SO2−
- Number of moles of solute (n)
4 Now, molality =
Using formula Mass of solvent (in kg)
∆T f = iK f × m Weight
n=
∆T f = 3 × 4 × 0.03 = 0.36 K Molecular mass
wX wY
=
5. Key Idea Osmotic pressure is proportional to the molarity (C) M X × (wsolvent )1 MY × (wsolvent )2
of the solution at a given temperature, π = CRT
Given, wX = 4 and w(solvent )1 =96
Concentration of BaCl2 = 0.01M (Given) wY = 12and w(solvent )2 = 88
π XY = 4 π BaCl 2 (Given) MX = A
i × CRT = 4 × i × CRT …(i) 4 × 1000 12 × 1000
∴ =
For the calculation of i, M X × 96 MY × 88
XY → X + + Y − (Here, i = 2) 12 × 1000 × M X × 96
Thus, MY =
BaCl2 → Ba 2+ + 2Cl− (Here, i = 3) 4 × 1000 × 88
Putting the values of i in (i) 96 × 12
= × A = 3.27A ≈ 3A
4 × 88
2 × [ XY ] = 4 × 3 × [ BaCl2 ]
2 × [ XY ] = 12 × 0.01 8. The ionisation of K2HgI4 in aqueous solution is as follows:
12 × 0.01 K 2 [ HgI4 ] - 2K + + [ HgI4 ]2 −
[ XY ] =
2 van’t Hoff factor (i) for ionisation reaction is given as,
So, the concentration of XY = 0.06 mol L−1 i = 1 + α ( n − 1)
= 6 × 10−2 mol L−1 where,
n = number of ions,
6. Molecules of benzoic acid dimerise in benzene as:
α = degree of ionisation or dissociation
2(C6H5COOH) - (C6H5COOH)2
From above equation, it is clear that n = 3
Now, we know that depression in freezing point (∆T f ) is given
i = 1 + 0.4 (3 − 1)
by following equation:
[Given, % α = 40% or α = 0.4]
i × K f × wsolute × 1000
∆T f = i × K f × m = ...(i) = 1.8
Mwsolute × wsolvent
9. We know that,
Given, wsolute (benzoic acid) = w g
Depression in freezing points (∆T f )
wsolvent (benzene) = 30g
T ° f − Tf = K f × m × i
Mw Solute (benzoic acid) = 122 g mol− 1, ∆T f = 2 K
where, K f = molal depression constant
K f = 5 Kkg mol− 1, %α = 80 or α = 0.8 wsolute × 1000
m = molality =
2(C6H5COOH) - (C6H5COOH)2 M solute × wsolvent (in g)
Initial 1 0 i = van’t Hoff factor
Final 1−α α/2
For diluted milk
= 1 − 0. 8 = 0. 2 0. 8 / 2 = 0. 4
∆T f1 = K f × m1 × i
Total number of moles at equilibrium = 0.2 + 0.4 = 0.6 wmilk × 1000
Number of moles at equilibrium ⇒ 0 − (0.2) ⇒ 0.2 = K f × ×1
i= M milk × w1 (H2O)
Number of moles present initially
For pure milk
0.6
i= = 0.6 ∆T f2 = K f × m2 × i
1
152 Solutions and Colligative Properties

wmilk × 1000 [Co(H2O)5 Cl]Cl 2 ⋅ H2O


-[Co(H O) Cl] + 2Cl 2+ −
‘i’ is 3
⇒ 0 − (−0.5) = 0.5 = K f × ×1 2 5
M milk × w2 (H2O) + −
[Co(H O) Cl ]Cl ⋅ 2H O -[Co(H O) Cl ] + Cl
2 4 2 2 2 4 2 ‘i’ is 2
0.2 K f wmilk × 1000 M × w2 (H2O) w2 (H 2O)
So, = × × milk = and for [Co(H2O)3 Cl 3 ]⋅ 3H2O, ‘i’ is 1 as it does not show
0.5 K f M milk × w1 (H2O) wmilk × 1000 w1 (H2O) ionisation. Hence, [Co(H2O)3 Cl 3 ]⋅ 3H2O have minimum number
w2 (H2O) (in pure milk) 2 of particles in the solution.
⇒ =
w1 (H2O) (in diluted milk) 5 So, freezing point of its solution will be maximum.
i.e. 3 cups of water has to be added to 2 cups of pure milk. 13. Let the degree of association of acetic acid (CH3COOH) in
10. Elevation in boiling point (∆Tb ) = K b × m × i benzene is α, then
2CH3 COOH q (CH3 COOH)2
Depression is freezing point (∆T f ) = K f × m × i Initial moles 1 0
where, m = molality α
Moles at equilibrium 1−α
For the glucose solution (van’t Hoff factor, i = 1), 2
α α α
∆Tb1m = ∆T f2m = 2K ∴ Total moles = 1 − α + =1− or i = 1 −
2 2 2
So, Kb × 1 × 1 = K f × 2 × 1 ⇒ K b = 2K f Now, depression in freezing point (∆T f ) is given as
11. Considering the expression of the depression in freezing point of ∆T f = i K f m K(i)
a solution, where, K f = molal depression constant or
cryoscopic constant.
∆T f = K f × m × i m = molality
wB × 1000
T f° − T f = K f × ×i …(i) number of moles of solute 0.2 1000
M B × wA (in g ) Molality = = ×
weight of solvent (in kg) 60 20
Here, T f° = 0°C, T f = − 10°C
Putting the values in Eq. (i)
wB = mass of ethylene glycol = 62 g
M B = molar mass of ethylene glycol  α  0.2 1000 
∴ 0.45 = 1 − (512
. ) ×
 
 2   60 20 
 CH2 — CH2 
 
    α 0.45 × 60 × 20
  1− =
 OH

OH 

. × 0.2 × 1000
2 512
−1
= 62 g mol α α
⇒ 1− = 0.527 ⇒ = 1 − 0.527
wA = mass of water in g as liquid solvent, 2 2
i = van’t-Hoff factor = 1 (for ethylene glycol in water) ∴ α = 0.946
K f = 1.86 K kg mol −1
Thus, percentage of association = 94.6%
On substituting in Eq. (i), we get 34.5
62 × 1000 14. −∆T f = ik f m 2 = 1 × 2 × × 1000 = 3
0 − (− 10) = 186
. × ×1 46 × 500
62 × wA
Vapour pressure curves shown in (b) is in agreement with the
. × 62 × 1000
186 calculated value of −∆T f . (a) is wrong, vapour pressure
⇒ wA = = 186 g
10 × 62 decreases on cooling.
So, amount of water separated as ice (solid solvent) 15. PLAN This problem includes concept of colligative properties (osmotic
= 250 − wA = (250 − 186)g = 64 g pressure here) and van’t Hoff factor. Calculate the effective
molarity of each solution.
12. Key idea ‘‘Addition of solute particles to a pure solvent results i.e. effective molarity = van’t Hoff factor × molarity
to depression in its freezing point.’’ 0.5 M C2H5OH (aq) i=1
All the compounds given in question are ionic in nature so, Effective molarity = 0 .5
consider their van’t Hoff factor (i ) to reach at final conclusion.
0.25 M KBr (aq) i=2
The solution with maximum freezing point must have minimum Effective molarity = 0 .5 M
number of solute particles. This generalisation can be done with 0.1 M Mg3 (PO4 )2(aq) i=5
the help of van’t Hoff factor (i ) i.e. Effective molarity = 0 . 5 M
Number of solute particles ∝ van’t Hoff factor (i ) 0.125 M Na 3PO4 (aq) i=4
Thus, we can say directly Effective molarity = 0. 5 M
Solution with maximum freezing point will be the one in which
Molarity is same hence, all colligative properties are also same.
solute with minimum van’t Hoff factor is present
3+ −
Now, for Co(H2 O)6 ]Cl 3 - [Co(H2 O)6 ] + 3Cl NOTE This question is solved by assuming that the examiner has
taken Mg 3(PO 4 )2 to be completely soluble. However, in real it is
van’t Hoff factor ( i ) is 4. Similarly for, insoluble (sparingly soluble).
Solutions and Colligative Properties 153

16. The elevation in boiling point is Lower the value of i, smaller will be the depression in freezing
n point, higher will be the freezing temperature, if molalities are
∆Tb = K b ⋅ m : m =molality = 2 × 1000 equal. Hence, glucose solution will have highest freezing
w1
temperature.
[n2 = Number of moles of solute, w1 = Weight of
24. K2SO4 : i=3
solvent in gram]
n2 5 NaCl : i = 2
⇒ 2 = 0.76 × × 1000 ⇒ n2 = Urea : i = 1
100 19
Also, from Raoult’s law of lowering of vapour pressure : Glucose : i = 1
Greater the value of i, greater the lowering in freezing point,
− ∆p n2 n
= x2 = ≈ 2 [Q n1 >> n2 ] lower will be the freezing temperature, if molarity in all cases are
p° n1 + n2 n1
same. Therefore, K2SO4 solution has the lowest freezing point.
5 18
⇒ − ∆p = 760 × × = 36 mm of Hg
19 100 25. Addition of HgI2 to KI solution establishes the following
⇒ p = 760 − 36 = 724 mm of Hg equilibrium :
3– HgI2 + 2KI r K2[HgI4 ]
17. van’t Hoff factor (i) = 4 {3K++ [Fe(CN)6 ]
0.1 1000 1 The above equilibrium decreases the number of ions (4 ions on
Molality = × = left side of reactions becomes three ions on right side), hence
329 100 329
rises the freezing point.
⇒ – ∆T f = iK f . m
1 26. In depression of freezing point experiment, vapour pressure of
= 4 × 1.86 × = 2.3 × 10–2
329 solution is less than that of pure solvent as well as only solvent
⇒ T f = –2 .3 × 10–2 ° C molecules solidify at freezing point.

(As % freezing point of water is 0ºC) 27. Osmotic pressure,


π = i × C × RT
18. Molality = 
20  1000
 × = 2.325 m Here, i = van’t Hoff factor, T = temperature
 172 50
C = concentration and R = gas constant.
⇒ − ∆T f = 2 = iK f ⋅ m
For NaCl, i = 2
2
⇒ i= = 0.5 So, π NaCl = i × C NaCl × RT
1.72 × 2.325
. = 2 × C NaCl × RT
01
13.44 0.05
19. Molality = = 0.1 C NaCl =
134.1 RT
i=3 For glucose, i = 1because it cannot ionise
⇒ ∆Tb = iK b ⋅ m = 3 × 0.52 × 0.1 So, π glucose = i × Cglucose × RT
= 0.156 0.2 = 1 × Cglucose × RT
20. For isotonic solutions, they must have same concentrations of 0.2
Cglucose =
ions, Therefore, RT
0.004 i (Na 2SO4) = 0.01 (Q nNaCl = numbers of moles NaCl)
0.01 nNaCl in 1 L = C NaCl × Vlitre
⇒ i= = 2.5
0.004 0.05
= (nglucose = number of moles of glucose)
Also Na 2SO4 r 2Na + + SO24− RT
1− α 2α α 0.4
nglucose in 2 L = Cglucose × Vlitre =
⇒ i = 1 + 2α = 2.5 RT
α = 0.75 = 75% Vtotal = 1 + 2 = 3L
0.05
21. During freezing, liquid solvent solidify and solid solvent remains So, final conc. NaCl =
3RT
in equilibrium with liquid solvent. 0.4
Final conc. glucose =
22. In benzene, benzoic acid dimerises as : 3RT
1 π total = π NaCl + π glucose = [ i × C NaCl + Cglucose ] RT
C6H5COOH r (C H COOH)2
2 6 5
 2 × 0.05 0.4  0.5
= +  × RT = atm
23. C6 H5 NH3 Cl : i = 2;  3RT 3RT  3
Ca(NO3 )2 : i=3 = 01666
. atm = 166.6 × 10−3 atm = 167.00 × 10−3 atm
La(NO3 )3 : i = 4; So, x = 167.00
C6H12O6 : i=1
154 Solutions and Colligative Properties

Key Idea First calculate, molar mass of solute using the  α 


28. ∆Tb [Y] (s) = 1− 2  K b (Y)
p° − ps nsolute  2
formula, = and then calculate ∆T f
p° nsolute + nsolvent Given, ∆Tb (X)(s) = 3∆Tb (Y)(s)
by applying the formula; ∆T f = K f × m.  α1   α 
1−  K b (X) = 3 × 1− 2  × K b (Y)
 2  2
When 0.5 g of non-volatile solute dissolve into 39 gm of benzene  α   α 
then relative lowering of vapour pressure occurs. Hence, vapour or 2 1− 1  = 3 1− 2  [Q K b (X) = 2K b (Y) ]
 2  2
pressure decreases from 650 mmHg to 640 mmHg.
Given, vapour pressure of solvent ( p° ) = 650 mmHg  α   0.7
or 2  1− 1  = 3  1−  (as given, α 2 = 0.7)
 2  2
Vapour pressure of solution ( ps ) = 640 mmHg
Weight of non-volatile solute = 0.5 g or 4 − 2α 1 = 6 − 2.1 or 2α 1 = 01 .
Weight of solvent (benzene) = 39 g so, α 1 = 0.05
From relative lowering of vapour pressure, 1000 × K f × W B
30. Molar mass of solute (M B ) =
p° − ps nsolute W A × ∆T f
= xSolute =
p° nsolute + nsolvent 1000 × 14 × 75.2
0.5 ⇒ MB =
1000 × 7
650 − 640 molar mass
= M B = 150.4 g per mol
650 0.5 39
+ Actual molar mass of phenol = 94 g/mol
molar mass 78
Calculated molar mass
0.5 Now, van’t Hoff factor, i =
10 molar mass Observed molar mass
= 94
650 0.5 ∴ i= = 0.625
+ 0.5
molar mass 150.4
0.5 + 0.5 × molar mass = 65 × 0.5 Dimerisation of phenol can be shown as :
∴ Molar mass of solute = 64 g 2C6H5OH r (C6 H5OH)2
Initial 1 0
From molal depression of freezing point,
α
K f × wsolute At equilibrium 1–α
∆T f = K f × molality = 2
(MW )solute × wsolvent α
Total number of moles at equilibrium, i = 1 − α +
0.5 × 1000 2
∆T f = 512
. × ⇒ ∆T f = 102. K
64 × 39 α
i =1−
2
29. From the graph we can note α
∆Tb for solution X i.e., But i = 0.625, thus, 0.625 = 1 −
2
∆Tb (X) = 362 − 360 = 2 α /2 = 1 − 0.625
Likewise, ∆Tb for solution Y i.e., ∆Tb (Y) = 368 − 367 = 1 α = 0.75
Now by using the formula Thus, the percentage of phenol that dimerises is 75%.
∆Tb = i × molality of solution× K b
For solution X
31. (i) ∆Tb = K b ⋅ m2
1.22 1000
2 = i × mNaCl × K b (X) …(i) ⇒ 0.17 = 1.7 × × ⇒ M = 122
M 100
Similarly for solution y 1.22 1000
1= i × mNaCl × K b (Y) ……(ii) (ii) 0.13 = 2.6 × × ⇒ M = 244
M 100
from Eq. (i) and (ii) above
The above molar masses suggests thapt benzoic acid is
K b (X) 2
= or 2 or K b (X) = 2K b (Y) monomeric in acetone while dimeric in benzene.
K b (Y) 1
32. Higher the value of K b of a solvent suggest that there is larger
For solute S polarity of solvent molecules, which in turn implies higher
2 S → S 2 (given due to dimerisation) boiling point due to dipole-dipole interaction.
Initial α 0 Therefore, the correct order of K b values of the three given
α solvents is
Final (1 − α)
2
Solvents Boiling point Kb
 α
So, here i =  1− 
 2 X 100°C 0.63
 α  Y 27°C 0.53
∆Tb [ X ](s) = 1 − 1  K b (X)
 2 Z 283°C 0.92
Solutions and Colligative Properties 155

33. Mass of water = 500 × 0.997 g = 498.5 g 0.643 1000


⇒ 0.48 = 5.12 × ×
M 50 × 0.879
Also CH3COOH r CH3COO− + H+
1−α α α ⇒ M = 156 g/mol

⇒ i = 1 + α = 1.23 37. In the given solution ‘M’, H2O is solute.


3 1000 0.1
⇒ − ∆T f = iK f ⋅ m = 1.23 × 1.86 × × = 0.23°C Therefore, molality of H2O = × 1000 = 2.4
60 498.5 0.9 × 46
34. (i) Empirical formula determination ⇒ − ∆T f = K fethanol × 2.4 = 2 × 2.4 = 4.8
Elements C H N O ⇒ T f = 155.7 – 4.8 = 150.9 K
Weight % 42.86 2.40 16.67 38.07
38. Vapour pressure = p (H2O) + p(ethanol )
Moles 3.57 2.40 1.19 2.38
Simplest ratio 3 2 1 2
= 32.8 × 0.1 + 40 × 0.9
= 3.28 + 36 = 39.28 mm
⇒ Empirical formula = C3H2NO2
5.5 1000
39. Now ethanol is solute.
(ii) ∆Tb = 1.84 = 2.53 × × 0.1
M 45 Molality of solute = × 1000 = 6.17
⇒ M = 168 0.9 × 18
Q Empirical formula weight (84) is half of molar mass, ⇒ ∆Tb = 6.17 × 0.52 = 3.20
molecular formula is C6H4N2O4 a dinitrobenzene : ⇒ Tb = 373 + 3.2 = 376.2 K
NO2  ∆T f 
40. lim   = K f (Cryoscopic constant)
m→ 0 m 

41. 1+ ∆T f = iK f m
NO2 ∆T f = 0 – (–0 .0558° C) = 0.0558° C
(C6H4N2O4) 0.0558
⇒ i (vant Hoff’s factor) = =3
35. − ∆T f = K f ⋅ m2 . × 0.01
186

0.3 This indicates that complex upon ionisation produces three


⇒ m2 = = 0.1613 ions as:
1.86
[Co(NH3 )5 Cl]Cl 2 → [Co(NH3 )5 Cl]2+ (aq) + 2Cl − (aq)
n 1000
Also, m2 = 2 × = 0.1613 Thus, only one Cl is inside the coordination sphere.
n1 M1
n2 0.1613 × 18 42. MX 2 → M 2+ + 2 X −
⇒ = = 2.9 × 10−3
n1 1000 van’t Hoff factor for any salt can be calculated by using equation
n2 n + n1 i = 1 + α (n − 1)
⇒ + 1= 2 = 2.9 × 10−3 + 1
n1 n1 where, n = number of constituent ions
n1 1 ∴ i (MX 2 ) = 1 + α (3 − 1) = 1 + 2α
⇒ = χ1 = = 0.997
n1 + n2 1 + 2.9 × 10−3 (∆T f )observed
= i = 1 + 2α
(∆T f )theoretical
⇒ p = p0χ 1 = 23.51 × 0.997 = 23.44 mm
∴ i = 1 + 2 × 0.5 ⇒ i = 2
36. − ∆T f = 5.51 − 5.03 = 0.48
⇒ − ∆T f = 0.48 = K f ⋅ m
10
Electrochemistry
Topic 1 Electrochemical Cells The reducing power of the metals increases in the order
(2019 Main, 10 Jan I)
Objective Questions I (Only one correct option) (a) Zn < Mg < Ni < Ca (b) Ni < Zn < Mg < Ca
1. Given, (c) Ca < Zn < Mg < Ni (d) Ca < Mg < Zn < Ni
Co 3 + + e− → Co 2 + ; E ° = + 181V
. 6. The anodic half-cell of lead-acid battery is recharged using
4+ − 2+ electricity of 0.05 Faraday. The amount of PbSO4
Pb + 2e → Pb ; E ° = + 167 . V
electrolysed in g during the process is (Molar mass of
Ce4 + + e− → Ce3 + ; E ° = + 161
. V
PbSO4 = 303g mol −1 )
Bi 3+ + 3e− → Bi; E ° = + 0.20 V
(2019 Main, 9 Jan I)
(a) 11.4 (b) 7.6 (c) 15.2 (d) 22.8
Oxidising power of the species will increase in the order
(2019 Main, 12 April I) 7. How long (approximate) should water be electrolysed by
4+ 4+ 3+ 3+ passing through 100 amperes current so that the oxygen
(a) Ce < Pb < Bi < Co released can completely burn 27.66 g of diborane?
(b) Bi 3 + < Ce4 + < Pb 4 + < Co 3 + (Atomic weight of B = 10.8 µ) (2018 Main)
(c) Co 3 + < Ce4 + < Bi 3 + < Pb 4 + (a) 6.4 hours (b) 0.8 hours (c) 3.2 hours (d) 1.6 hours
(d) Co 3 + < Pb 4 + < Ce4 + < Bi 3 + °
8. Given, ECl −
° 3+
= 1.36 V, ECr = − 0.74 V
2 /Cl /Cr
2. A solution of Ni(NO3 )2 is electrolysed between platinum ° 2 − 3 + = 1.33 V, E ° −
ECr MnO / Mn 2 +
= 1.51 V
electrodes using 0.1 Faraday electricity. How many mole of O /Cr
2 7 4
Ni will be deposited at the cathode? (2019 Main, 9 April II) Among the following, the strongest reducing agent is
(a) 0.20 (b) 0.10 (c) 0.15 (d) 0.05 (2017 Main)

3. Calculate the standard cell potential (in V) of the cell in which (a) Cr (b) Mn 2+
(c) Cr 3+
(d) Cl −
following reaction takes place 9. For the following electrochemical cell at 298 K,
Fe2+ ( aq ) + Ag+ ( aq ) → Fe3 + ( aq ) + Ag( s ) Pt ( s ) | H2 ( g , 1bar ) | H+ ( aq , 1 M)
Given that, E ° Ag + / Ag = x V | | M 4 + ( aq ), M 2 + ( aq ) | Pt ( s )
2+
E ° Fe 2+ / Fe = y V [ M ( aq )]
Ecell = 0.092 V when = 10x
E ° Fe3 + / Fe = z V (2019 Main, 8 April II) [ M 4 + ( aq )]
(a) x + 2 y − 3 z (b) x − y RT
Given : E °M 4+ / M 2+ = 0.151 V; 2.303 = 0.059 V
(c) x + y − z (d) x − z F
4. Given, that EOs2 / H 2O = +1.23V; The value of x is (2016 Adv.)
(a) − 2 (b) − 1 (c) 1 (d) 2
E sS O 2− / SO 2− = 2.05V; EBr = +1.09V, EAu 3+ / Au = +1.4V
s s
2 8 4 2
/ Br s 10. Two Faraday of electricity is passed through a solution of
The strongest oxidising agent is (2019 Main, 8 April I)
CuSO4 . The mass of copper deposited at the cathode is
(at. mass of Cu = 63.5 u) (2015 Main)
(a) Au 3+ (b) O2 (c) S2 O2−
8 (d) Br2
(a) 0 g (b) 63.5 g (c) 2 g (d) 127 g
5. Consider the following reduction processes:
° 3+ = − 0.74 V; E ° −
11. Given, ECr = 1.51 V
Zn 2+ + 2e− 
→ Zn (s); E ° = − 0.76 V /Cr MnO /Mn 2+ 4

Ca 2+
+ 2e  −
→ Ca (s); E ° = − 2.87 V
° 2− 3+ = 1.33 V; E ° − = 1.36 V
ECr O /Cr
2 7 Cl /Cl

Mg2+ + 2e− 
→ Mg(s); E ° = − 2.36 V Based on the data given above strongest oxidising agent will be
(2013 Main)
Ni2+ + 2e− 
→ Ni(s); E ° = − 0.25 V
(a) Cl (b) Cr 3+ (c) Mn 2+ (d) MnO −4
Electrochemistry 157

12. Electrolysis of dilute aqueous NaCl solution was carried out The emf for the cell reaction,
by passing 10 mA current. The time required to liberate Fe2+ + Zn → Zn 2+ + Fe is (1989, 1M)
0.01 mole of H2 gas at the cathode is (1 F = 96500 C mol −1 )
(a) – 0.35 V (b) + 0.35 V
(a) 9.65 × 104 s (b) 19.3 × 104 s (2008, 3M) (c) + 1.17 V (d) − 1.17 V
4 4
(c) 28.95 × 10 s (d) 38.6 × 10 s
19. When a lead storage battery is discharged (1986, 1M)
13. In the electrolytic cell, flow of electrons is from (2003, 1M) (a) SO2 is evolved
(a) cathode to anode in solution (b) lead is formed
(b) cathode to anode through external supply (c) lead sulphate is consumed
(c) cathode to anode through internal supply (d) sulphuric acid is consumed
(d) anode to cathode through internal supply
20. The reaction,
14. Standard electrode potential data are useful for understanding 1
the suitability of an oxidant in a redox titration. Some H2 ( g ) + AgCl( s ) r H+ ( aq ) + Cl − ( aq ) + Ag ( s )
2
half-cell reactions and their standard potentials are given occurs in the galvanic cell (1985, 1M)
below :
(a) Ag |AgCl ( s )|KCl(soln) | AgNO3 |Ag
MnO−4 ( aq) + 8H+ ( aq ) + 5e− → Mn 2+ ( aq ) + 4H2 O( l ),
(b) Pt|H2 ( g )|HCl(soln) | AgNO3 (soln)|Ag
E° = 1.51 V
Cr2 O27 − ( aq ) + 14 H+ ( aq ) + 6e− → 2Cr 3+ ( aq ) + 7H2 O( l ) , (c) Pt |H2 ( g )HCl (soln) | AgCl ( s )|Ag
(d) Pt | H2 ( g ) | KCl (soln) | AgCl( s )|Ag
E° = 1.38 V
Fe3+ ( aq ) + e− → Fe2+ ( aq ) E° = 0.77 V 21. The electric charge for electrode deposition of one gram
− − equivalent of a substance is
Cl 2 ( g ) + 2e → 2Cl ( aq ) E° = 1.40 V (1984, 1M)
(a) one ampere per second
Identify the incorrect statement regarding the quantitative (b) 96,500 coulombs per second
estimation of aqueous Fe(NO3 )2 (2002, 3M) (c) one ampere for one hour
(a) MnO−4 can be used in aqueous HCl (d) charge on one mole of electrons
(b) Cr2 O2−
7 can be used in aqueous HCl 22. A solution containing one mole per litre of each Cu (NO3 )2 ,

(c) MnO4 can be used in aqueous H2 SO4 AgNO3 ,Hg 2 (NO3 )2 and Mg(NO3 )2 is being electrolysed by
(d) Cr2 O2−
7 can be used in aqueous H2 SO4 using inert electrodes. The values of standard electrode
potentials in volts (reduction potential) are
15. Saturated solution of KNO3 is used to make ‘salt-bridge’
Ag + / Ag = + 0.80, Hg 2+
2 / 2Hg = + 0.79
because (2001, 1M)
Cu 2+ / Cu = + 0.34, Mg 2+ / Mg = − 2.37
(a) velocity of K + is greater than that of NO–3
With increasing voltage, the sequence of deposition of metals
(b) velocity of NO−3 is greater than that of K +
on the cathode will be (1984, 1M)
(c) velocities of both K + and NO−3 are nearly the same (a) Ag, Hg, Cu, Mg (b) Mg, Cu, Hg, Ag
(d) KNO3 is highly soluble in water (c) Ag, Hg, Cu (d) Cu, Hg, Ag
16. The gas X at 1 atm is bubbled through a solution containing a 23. Faraday’s laws of electrolysis are related to the (1983, 1M)
mixture of 1 M Y − and 1 M Z − at 25° C . If the order of
(a) atomic number of the cation
reduction potential is Z > Y > X , then (1999, 2M)
(b) atomic number of the anion
(a) Y will oxidise X and not Z (c) equivalent weight of the electrolyte
(b) Y will oxidise Z and not X (d) speed of the cation
(c) Y will oxidise both X and Z
24. The standard reduction potentials at 298K for the following
(d) Y will reduce both X and Z
half cells are given :
17. The standard reduction potential values of three metallic
Zn 2 + ( aq ) + 2e− r Zn ( s ) ; E° = −0.762 V
cations, X, Y, Z are 0.52, − 3.03 and − 1.18 V respectively.
The order of reducing power of the corresponding metals is Cr 3 + ( aq ) + 3e− r Cr ( s ) ; E° = −0.740 V
(a) Y > Z > X (b) X > Y > Z (1998, 2M) 2H+ ( aq ) + 2e− r H2 ( g ); E° = 0.000 V
(c) Z > Y > X (d) Z > X > Y
Fe3 + ( aq ) + e− r Fe2 + ( aq ); E° = 0.770 V
18. The standard reduction potentials E°, for the half reactions
Which is the strongest reducing agent? (1981, 1M)
are as
(a) Zn( s ) (b) Cr( s )
Zn = Zn 2+ + 2e− , E° = + 0.76 V
(c) H2 ( g ) (d) Fe2+ ( aq )
Fe = Fe2+ + 2e− , E° = 0.41 V
158 Electrochemistry

Objective Questions II Passage Based Questions


(One or more than one correct option) Chemical reactions involve interaction of atoms and
25. In a galvanic cell, the salt-bridge (2014 Adv.) molecules. A large number of atoms/molecules
(a) does not participate chemically in the cell reaction (approximately 6.023 × 1023 ) are present in a few grams of
(b) stops the diffusion of ions from one electrode to another any chemical compound varying with their atomic/molecular
(c) is necessary for the occurrence of the cell reaction masses. To handle such large numbers conveniently, the
(d) ensures mixing of the two electrolytic solutions mole concept was introduced. This concept has implications
in diverse areas such as analytical chemistry, biochemistry,
26. For the reduction of NO–3 ion in an aqueous solution E° is electrochemistry and radiochemistry. The following example
+ 0.96 V. Values of E° for some metal ions are given below illustrates a typical case, involving chemical/electrochemical
V2+ ( aq ) + 2e– → V; E° = − 1.19 V reaction, which requires a clear understanding of the mole
Fe3+ ( aq ) + 3e– → Fe; E° = − 0.04V concept. A 4.0 M aqueous solution of NaCl is prepared and
Au 3+ ( aq ) + 3e– → Au; E° = + 1.40 V 500 mL of this solution is electrolysed. This leads to the
Hg 2+ ( aq ) + 2e– → Hg ; E° = + 0.86V evolution of chlorine gas at one of the electrodes
The pair(s) of metals that is/are oxidised by NO–3 in aqueous (atomic mass : Na = 23, Hg = 200 , 1F = 96500 C).
solution is (are) (2009) (2007, 3 × 4M = 12M)
(a) V and Hg (b) Hg and Fe (c) Fe and Au (d) Fe and V 30. The total number of moles of chlorine gas evolved is
(a) 0.5 (b) 1.0
Numerical Answer Type Questions (c) 2.0 (d) 3.0
27. Consider a 70% efficient hydrogen-oxygen fuel cell working 31. If the cathode is a Hg electrode, the maximum weight
under standard conditions at 1 bar and 298 K. Its cell reaction is (in gram) of amalgam formed from this solution is
1 (a) 200 (b) 225
H 2( g )+ O 2( g ) → H 2O( l )
2 (c) 400 (d) 446
The work derived from the cell on the consumption of 32. The total charge (coulombs) required for complete
. × 10−3 mole of H2 ( g ) is used to compress 1.00 mole of a
10 electrolysis is
monoatomic ideal gas in a thermally insulated container. (a) 24125 (b) 48250
What is the change in the temperature (in K) of the ideal gas? (c) 96500 (d) 193000
The standard reduction potentials for the two half-cells are
Subjective Questions
given below:
O 2 (g) + 4H+ (aq) + 4 e− → 2H 2O(),
l E° = 1.23 V,
33. The following electrochemical cell has been set-up :
Pt (1) | Fe3+ , Fe2+ ( a = 1) | Ce4+ , Ce3+ ( a = 1) | Pt (2)
2H+ ( aq ) + 2e− → H 2 ( g ), E° = 0.00 V
E° (Fe3+ , Fe2+ ) = 0.77 V
−1 −1 −1
Use, F = 96500 C mol , R = 8.314 J mol K . (2020 Adv.) and E° (Ce4+ , Ce3+ ) = 1.61 V
28. For the electrochemical cell, If an ammeter is connected between the two platinum
Mg( s )|Mg 2 + ( aq, 1 M) || Cu 2 + ( aq , 1 M) | Cu( s ) electrodes, predict the direction of flow of current, will the
current increases or decreases with time? (2000, 2M)
The standard emf of the cell is 2.70 V at 300 K. When the
concentration of Mg 2+ is changed to x M, the cell potential 34. Copper sulphate solution (250 mL) was electrolysed using a
platinum anode and a copper cathode. A constant current of
changes to 2.67 V at 300 K. The value of x is _____.
2 mA was passed for 16 min. It was found that after
F
(Given, = 11500 K V−1 , where F is the Faraday constant electrolysis the absorbance of the solution was reduced to
R 50% of its original value. Calculate the concentration of
and R is the gas contant, In (10) = 2.30) (2018 Adv.) copper sulphate in the solution to begin with. (2000, 3M)
29. Consider an electrochemical cell : 35. A cell, Ag | Ag + || Cu 2+ | Cu , initially contains 1 M Ag + and 1
n+
A( s ) | A ( aq , 2M )|| B 2 n + ( aq ,1M)| B( s ). The value of ∆H s M Cu 2+ ions. Calculate the change in the cell potential after
for the cell reaction is twice of ∆Gs at 300 K. If the emf of the the passage of 9.65 A of current for 1 h. (1999, 6M)
cell is zero, the ∆S s (in J K −1 mol −1 ) of the cell reaction per
36. How many grams of silver could be plated out on a serving
mole of B formed at 300 K is …… . tray by electrolysis of a solution containing silver in +1
(Given : ln( 2 ) = 0.7, R (universal gas constant) = 8.3 J K −1 oxidation state for a period of 8.0 h at a current of 8.46 A?
mol −1 . H , S and G are enthalpy, entropy and Gibbs energy, What is the area of the tray, if the thickness of the silver
plating is 0.00254 cm? Density of silver is 10.5 g/cm3 .
respectively.) (2018 Adv.)
(1997, 3M)
Electrochemistry 159

37. The Edison storage cell is represented as: 44. An acidic solution of Cu 2+ salt containing 0.4 g of Cu 2+ is
Fe ( s ) / FeO( s) / KOH( aq ) /Ni 2 O3 ( s ) / Ni( s ) electrolysed until all the copper is deposited. The electrolysis is
The half-cell reactions are : continued for seven more minutes with the volume of solution
Ni 2 O3 ( s ) + H2 O ( l ) + 2e− r 2NiO( s ) + 2OH− , kept at 100 mL and the current at 1.2 A. Calculate the volume
E° = + 0.40 V of gases evolved at NTP during the entire electrolysis.
FeO ( s ) + H2 O( l ) + 2e− r Fe( s ) + 2OH− , (1989, 5M)

E° = −0.87 V 45. In a fuel cell hydrogen and oxygen react to produces


(i) What is the cell reaction?
electricity. In the process hydrogen gas is oxidised at the anode
and oxygen at the cathode. If 67.2 L of H2 at STP react in 15
(ii) What is the cell emf ? How does it depend on the
concentration of KOH? min, what is the average current produced? If the entire current
(iii) What is the maximum amount of electrical energy that is used for electro deposition of copper from copper (II)
can be obtained from one mole of Ni 2O3? (1994, 4M) solution, how many grams of copper will be deposited?
Anode reaction : H2 + 2OH− → 2H2 O + 2e−
38. The standard reduction-potential for the half-cell
NO−3 ( aq ) + 2H+ + e− → NO2 ( g ) + H2 O is 0.78 V Cathode reaction : O2 + 2H2 O + 2e− → 4OH− (1988, 4M)

(i) Calculate the reduction-potential in 8M H+ . 46. A cell contains two hydrogen electrodes. The negative electrode
is in contact with a solution of 10−6 M hydrogen ions. The emf
(ii) What will be the reduction-potential of the half-cell in a
neutral solution? Assume all the other species to be at unit of the cell is 0.118 V at 25° C . Calculate the concentration of
concentration. (1993, 2M) hydrogen ions at the positive electrode. (1988, 2M)

39. Chromium metal can be plated out from an acidic solution 47. A 100 watt, 110 V incandecent lamp is connected in series with
containing Cr O3 according to the following equation. an electrolyte cell containing cadmium sulphate solution.
What weight of cadmium will be deposited by the current
Cr O3 ( aq ) + 6H+ ( aq ) + 6e− → Cr ( s ) + 3H2 O flowing for 10 h? (1987, 5M)
Calculate (i) How many grams of chromium will be plated 48. During the discharge of a lead storage battery, the density of
out by 24,000 C and (ii) How long will it take to plate out sulphuric acid fell from 1.294 to 1.139 g/mL. Sulphuric acid of
1.5 g of chromium by using 12.5 A current? (1993, 2M) density 1.294 g/mL is 39% H2 SO4 by weight and that of
40. An aqueous solution of NaCl on electrolysis gives density 1.139 g/mL is 20% H2 SO4 by weight. The battery
H2 ( g ), Cl 2 ( g ) and NaOH according to the reaction. holds 3.5 L of the acid and the volume remained practically
constant during the discharge.
2Cl – ( aq ) + 2H2 O r 2OH− ( aq ) + H2 ( g ) + Cl 2 ( g )
Calculate the number of ampere-hours for which the battery
A direct current of 25 A with a current efficiency of 62% is must have been used. The charging and discharging reactions
passed through 20 L of NaCl solution (20% by weight). are
Write down the reactions taking place at the anode and Pb + SO24 − = PbSO4 + 2e− (charging)
cathode. How long will it take to produce 1kg of Cl 2 ? What −
will be the molarity of the solution with respect to hydroxide PbO2 + 4H+ + SO2−
4 + 2e

ion? (Assume no loss due to evaporation) (1992, 3M) = PbSO4 + 2H2 O (discharging) (1986, 5M)

41. For the galvanic cell, 49. How long a current of 3 A has to be passed through a solution
Ag | AgCl( s ) , KCl (0.2 M) || KBr (0.001 M), AgBr (s) | Ag of silver nitrate to coat a metal surface of 80 cm2 with a
0.005 mm thick layer?
Calculate the emf generated and assign correct polarity to
each electrode for a spontaneous process after taking into Density of silver is 10.5 g/cm3 . (1985, 3M)
account the cell reaction at 25° C. 50. In an electrolysis experiment current was passed for 5 h
[ K sp (AgCl) = 2.8 × 10−10 , K sp (AgBr) = 3.3 × 10−13] (1992, 4M) through two cells connected in series. The first cell contains a
42. A current of 1.70 A is passed through 300.0 mL of 0.160M solution of gold and the second contains copper sulphate
solution. 9.85 g of gold was deposited in the first cell. If the
solution of a ZnSO4 for 230 s with a current efficiency of
oxidation number of gold is +3, find the amount of copper
90%. Find out the molarity of Zn 2+ after the deposition Zn.
deposited on the cathode of the second cell. Also calculate the
Assume the volume of the solution to remain constant
magnitude of the current in ampere.
during the electrolysis. (1991, 4M)
(Atomic weight of Au = 197 and atomic weight of
43. Calculate the quantity of electricity that would be required Cu = 63.5)
to reduce 12.3 g of nitrobenzene to aniline, if the current (1983, 3M)
efficiency for the process is 50%. If the potential drop across 51. A current of 3.7 A is passed for 6 h between nickel electrodes
the cell is 3.0 V, how much energy will be consumed? in 0.5 L of a 2.0 M solution of Ni(NO3 )2 . What will be the
(1990, 3M) molarity of solution at the end of electrolysis? (1978, 2M)
Topic 2 Conductivity of Electrolytic Solutions and their
Measurement and Nernst Equation
Objective Questions I (Only one correct option) 4. Consider the statements S1 and S 2 :
1. Molar conductivity ( Λ m ) of aqueous solution of sodium S 1 : Conductivity always increases with decrease in the
stearate, which behaves as a strong electrolyte, is recorded at concentration of electrolyte.
varying concentrations (C ) of sodium stearate. Which one of S 2 : Molar conductivity always increases with decrease in the
the following plots provides the correct representation of concentration of electrolyte.
micelle formation in the solution? The correct option among the following is
(critical micelle concentration (CMC) is marked with an (2019 Main, 10 April I)
arrow in the figures) (2019 Adv.) (a) S1 is correct and S2 is wrong
(b) S1 is wrong and S2 is correct
(c) Both S1 and S2 are wrong
Λm
Λm CMC (d) Both S1 and S2 are correct
(a) (b) 5. The standard Gibbs energy for the given cell reaction in kJ
CMC mol−1 at 298 K is
Zn( s ) + Cu2 + ( aq ) → Zn2 + ( aq ) + Cu( s ),
√C √C
E° = 2Vat 298 K
(Faraday’s constant, F = 96000 C mol−1 )
(2019 Main, 9 April I)
CMC (a) 384 (b) 192 (c) −384 (d) −192
Λm Λm CMC
(c) (d) 6. Λ°m for NaCl, HCl and NaA are 126.4, 425.9 and
100.5 S cm 2 mol− 1 , respectively. If the conductivity of 0.001
M HA is 5 × 10− 5 S cm − 1 , degree of dissociation of HA is
√C √C (2019 Main, 12 Jan II)
(a) 0.25 (b) 0.50 (c) 0.75 (d) 0.125
2. The decreasing order of electrical conductivity of the 7. Given the equilibrium constant ( K C ) of the reaction :
following aqueous solution is
0.1 M formic acid (A), Cu( s ) + 2Ag+ ( aq ) → Cu2 + ( aq ) + 2Ag( s )
0.1 M acetic acid (B), °
is 10 × 1015 , calculate the Ecell of this reaction at 298 K.
0.1 M benzoic acid (C). (2019 Main, 12 April II)
(a) A > C > B (b) C > B > A  RT 
2.303 F at 298 K = 0.059 V
(c) A > B > C (d) C > A > B (2019 Main, 11 Jan II)
3. Which one of the following graphs between molar (a) 0.4736 V (b) 0.04736 mV
conductivity (Λ m ) versus C is correct? (c) 0.4736 mV (d) 0.04736 V
(2019 Main, 10 April II) 8. For the cell, Zn( s)|Zn2+ ( aq )||M x+ ( aq )|M ( s), different half
cells and their standard electrode potentials are given below.
Na
Cl

KC
l Au 3+ (aq)/ Ag + (aq)/ Fe 3+ (aq)/ Fe 2+ (aq)/
(a) Λ m (b) Λ m M x+ (aq)/M(s)
Au(s) Ag(s) Fe 2+ (aq) Fe(s)
Na
KC

Cl

E ° M x + / M /V − 0.44
l

1.40 0.80 0.77

√C √C If E° Zn 2+ / Zn = − 0.76 V, which cathode will give a maximum


value of E° cell per electron transferred? (2019 Main, 11 Jan I)
KC

Ag+ Fe2+ Au3+ Fe3 +


l

(a) (b) (c) (d)


(c) Λ m (d) Λ m Ag Fe Au Fe2+
Na 9. In the cell,
Na

KC

Cl
Cl

Pt(s) H 2 ( g , 1 bar) HCl( aq )|AgCl( s ) Ag( s ) Pt( s ) the cell


√C √C potential is 0.92 V when a 10−6 molal HCl solution is used.
Electrochemistry 161

The standard electrode potential of ( AgCl/ Ag,Cl− ) Match E° of the rebox pair in Column I with the values given
 2.303RT  in Column II and select the correct answer using the code
electrode is Given, = 0.06 V at 298 K given below the lists. (2013 Adv.)
 F 
(a) 0.40 V (b) 0.20 V (c) 0.94 V (d) 0.76 V Column I Column II
10. If the standard electrode potential for a cell is 2V at 300 K, the 3+
P. E° (Fe / Fe) 1. − 0.18 V
equilibrium constant (K) for the reaction,
Q. E° ( 4H2 O r 4H+ + 4OH− ) 2. – 0.4 V
Zn( s ) + Cu2+ ( aq ) - Zn 2+
( aq ) + Cu( s )
2+ +
at 300 K is approximately R. E° ( Cu + Cu → 2 Cu ) 3. – 0.04 V

( R = 8 JK −1 mol−1 , F = 96000 C mol−1 ) S. E° (Cr 3 + , Cr 2 + ) 4. – 0.83 V


(2019 Main, 9 Jan II)
Codes
(a) e−160 (b) e160 (c) e−80 (d) e320
P Q R S P Q R S
11. For the following cell, (a) 4 1 2 3 (b) 2 3 4 1
Zn ( s )| ZnSO4 ( aq )||CuSO4 ( aq )|Cu( s ) (c) 1 2 3 4 (d) 3 4 1 2
when the concentration of Zn2+ is 10 times the concentration 17. An aqueous solution of X is added slowly to an aqueous
of Cu2+ , the expression for ∆G (in J mol −1 ) is solution of Y as shown in Column I. The variation in
conductivity of these reactions is given in Column II. Match
[F is Faraday constant; R is gas constant;
Column I with Column II and select the correct answer using
T is temperature; E° (cell) = 11
. V] (2017 Adv.)
the codes given below the Columns. (2013 Adv.)
(a) 2.303 RT +11
. F (b) 1.1 F
(c) 2.303 RT − 2.2 F (d) −2.2 F Column I Column II
12. Galvanisation is applying a coating of (2016 Main) P. (C2 H5 )3 N + CH3 COOH 1. Conductivity decreases
(a) Cr (b) Cu (c) Zn (d) Pb X Y and then increases
13. Given below are the half-cell reactions (2014 Main) Q. KI(0.1 M) + AgNO3 (0.01 M) 2. Conductivity decreases
Mn 2 + + 2e− → Mn ; E° = − 1.18 eV X Y and then does not
2 (Mn 3+ + e− → Mn 2+ ) ; E° = + 1.51eV change much
The E ° for 3Mn 2 + → Mn + 2Mn 3+ will be R. CH3 COOH + KOH 3. Conductivity increases
(a) − 2. 69 V; the reaction will not occur X Y and then does not
(b) − 2. 69 V; the reaction will occur change much
(c) − 0. 33 V; the reaction will not occur S. NaOH + HI 4. Conductivity does not
(d) − 0. 33 V; the reaction will occur X Y change much and then
14. The equivalent conductance of NaCl at concentration C and increases
at infinite dilution are λ C and λ ∞ , respectively. The correct Codes
relationship between λ C and λ ∞ is given as (where, the P Q R S P Q R S
constant B is positive) (2014 Main) (a) 3 4 2 1 (b) 4 3 2 1
(a) λ C = λ ∞ + ( B ) C (b) λ C = λ ∞ − ( B ) C (c) 2 3 4 1 (d) 1 4 3 2
(c) λ C = λ ∞ − ( B ) C (d) λ C = λ ∞ + ( B ) C
18. Consider the following cell reaction,
15. Resistance of 0.2 M solution of an electrolyte is 50 Ω . The 2Fe( s ) + O2 ( g ) + 4H+ ( aq ) → 2Fe2+ ( aq ) + 2H2 O ( l ),
specific conductance of the solution of 0.5 M solution of same E ° = 1.67 V
electrolyte is 1.4 S m−1 and resistance of same solution of the
At [Fe2+ ] = 10−3 M, P(O 2 ) = 0.1 atm and pH = 3, the cell
same electrolyte is 280 Ω . The molar conductivity of 0.5 M
solution of the electrolyte in Sm 2mol−1 is potential at 25° C is (2011)
(2014 Main)
(a) 1.47 V (b) 1.77 V (c) 1.87 V (d) 1.57 V
(a) 5 × 10−4 (b) 5 × 10−3 (c) 5 × 103 (d) 5 × 102
19. AgNO3 (aqueous) was added to an aqueous KCl solution
16. The standard reduction potential data at 25°C is given below. gradually and the conductivity of the solution was measured.
E° (Fe3 + / Fe2 + ) = + 0.77 V; E° (Fe2 / Fe) = −0.44 V; The plot of conductance (Λ) versus the volume of AgNO3 is
E° (Cu 2 + / Cu) = + 0.34 V; E° (Cu + / Cu) = + 0.52 V; × ×
Λ ×Λ Λ Λ ×
××

+ − × ×
E ° ( O2 ( g ) + 4H + 4 e ) → 2H2 O) = + 1.23 V; × × ×
× ××× ×
×

×× × ××
×

×
×

×
×


E ° ( O2 ( g ) + 2H2 O + 4 e ) → 4OH) = + 0.40 V
Volume Volume Volume Volume
E° (Cr 3 + / Cr) = −0.74 V; E° (Cr 2 + / Cr) = +0.91 V (P) (Q) (R) (S) (2011)
(a) (P) (b) (Q) (c) (R) (d) (S)
162 Electrochemistry

20. The half cell reactions for rusting of iron are : At 1250 K : 2Cu( s)+ 1 / 2 O 2 ( g ) → Cu 2O( s);
+ −1 ∆Gs = − 78,000 J mol −1
2H + 2e + O2 → H2 O( l ); E° = + 1.23V
2 1
Fe2 + + 2e− → Fe( s ); E° = − 0.44 V H 2( g )+ O2 ( g ) → H 2O ( g );
2
∆G° (in kJ) for the reaction is (2005, 1M)
(a) – 76 (b) – 322 (c) – 122 (d) – 176 ∆Gs = − 1,78,000 J mol −1 ; G is the Gibbs energy (2018 Adv.)

21. Zn | Zn 2+ ( a = 0.1M ) || Fe2+ ( a = 0.01M) | Fe.


Passage Based Questions
The emf of the above cell is 0.2905 V. Equilibrium constant
for the cell reaction is (2004, 1M) Passage I
(a) 100.32/ 0.059 (b) 100.32/ 0.0295 The electrochemical cell shown below is a concentration cell.
(c) 100.26/ 0.0295 (d) 100.32/ 0.295 M | M 2+ (saturated solution of a sparingly soluble salt,
22. The correct order of equivalent conductance at infinite MX 2 )||M 2+ (0.001 mol dm −3 )|M . The emf of the cell depends
dilution of LiCl, NaCl and KCl is (2001, 1M) on the difference in concetration of M 2+ ions at the two
(a) LiCl > NaCl > KCl (b) KCl > NaCl > LiCl electrodes. The emf of the cell at 298K is 0.059 V.
(c) NaCl > KCl > LiCl (d) LiCl > KCl > NaCl (2012)

23. For +
the electrochemical cell, ( M |M )||( X | X ), − 28. The solubility product (K sp : mol 3 dm −9 ) of MX 2 at 298 based
E° ( M + / M ) = 0.44 Vand E° ( X / X − ) = 0.33 V. on the information available the given concentration cell is
(take 2.303 × R × 298/ F = 0.059 V)
From this data one can deduce that (2000, 1M)
(a) 1 × 10−15 (b) 4 × 10−15 (c) 1 × 10−12 (d) 4 × 10−12
(a) M + X → M + + X − is the spontaneous reaction
(b) M + + X – → M + X is the spontaneous reaction 29. The value of ∆G (kJ mol −1 ) for the given cell is
(c) Ecell = 0.77 V (take 1 F = 96500 C mol −1 )
(d) Ecell = − 0.77 V (a) − 5.7 (b) 5.7 (c) 11.4 (d) –11.4

24. The standard reduction potentials of Cu 2+ /Cu and Cu 2+ / Cu + Passage II


are 0.337 V and 0.153 V respectively. The standard electrode The concentration of potassium ions inside a biological cell is
potential of Cu + /Cu half-cell is (1997, 1M)
at least twenty times higher than the outside. The resulting
(a) 0.184 V (b) 0.827 V (c) 0. 521 V (d) 0.490 V potential difference across the cell is important in several
processes such as transmission of nerve impulses and
Numerical Answer Type Questions maintaining the ion balance. A simple model for such a
concentration cell involving a metal M is :
25. For the disproportionation reaction
M ( s ) | M + ( aq; 0.05 molar) || M + ( aq; 1 molar) | M ( s )
2Cu+ ( aq ) c Cu ( s ) + Cu 2 + ( aq ) at 298 K, In K (where K
is the equilibrium constant) is ………× 10−1 . For the above electrolytic cell the magnitude of the cell
potential | Ecell | = 70 mV. (2010)
° 2+
Given : ( ECu = 016
. V,
/ Cu + 30. For the above cell
° +
RT (a) Ecell < 0; ∆G > 0 (b) Ecell > 0; ∆G < 0
ECu / Cu
= 0.52 V and = 0.025)
F (2020 Main, 2 Sep II) (c) Ecell < 0; ∆G° > 0 (d) Ecell > 0; ∆G° < 0
26. An oxidation-reduction reaction in which 3 electrons are 31. If the 0.05 molar solution of M + is replaced by a 0.0025
transferred has a ∆G° of 17.37 kJ mol −1 at 25° C. The value of molar M + solution, then the magnitude of the cell potential
°
Ecell (in V) is …… ×10−2 . would be
(1 F = 96,500 C mol −1 ) (2020 Main, 5 Sep I) (a) 35 mV (b) 70 mV
27. The surface of copper gets tarnished by the formation of (c) 140 mV (d) 700 mV
copper oxide. N 2 gas was passed to prevent the oxide
formation during heating of copper at 1250 K. However, the Passage III
N 2 gas contains 1 mole % of water vapour as impurity. The Redox reaction play a pivotal role in chemistry and biology.
water vapour oxidises copper as per the reaction given below The values of standard redox potential ( E° ) of two half-cell
2Cu( g )+ H 2O( g ) → Cu 2O( s)+ H 2( g ) reactions decide which way the reaction is expected to
proceed. A simple example is a Daniell cell in which zinc
pH 2 is the minimum partial pressure of H 2 (in bar) needed to goes into solution and copper gets deposited. Given below are
prevent the oxidation at 1250 K. The value of ln( pH 2 ) is …… . a set of half-cell reactions (acidic medium) along with their
(Given : total pressure = 1 bar, R (universal gas constant) E ° (V with respect to normal hydrogen electrode) values.
= 8 J K −1 mol −1 , ln(10 ) = 2.30 Cu ( s ) and Cu 2O ( s ) are Using this data obtain the correct explanations to Questions
mutually immiscible.) 17–19. (2007, 4 × 3M = 12M)
Electrochemistry 163

I2 + 2e− → 2I− E° = 0.54 Subjective Questions


− −
Cl 2 + 2e → 2Cl E° = 1.36
38. We have taken a saturated solution of AgBr, K sp is
Mn 3+ + e− → Mn 2+ E° = 1.50 12 × 10− 14 . If 10− 7 M of AgNO3 are added to 1 L of this
3+ − 2+
Fe + e → Fe E° = 0.77 solution, find conductivity (specific conductance) of this
O2 + 4H+ + 4e− → 2H2O E° = 1.23 solution in terms of 10 − 7 Sm− 1 units. (2006, 6M)
° −3 −1
Given, λ (Ag + ) = 6 × 10 Sm mol ,
2
32. Among the following, identify the correct statement.
(a) Chloride ion is oxidised by O2 λ °(Br − ) = 8 × 10− 3 Sm2 mol −1 ,
(b) Fe2+ is oxidised by iodine
λ °(NO – ) = 7 × 10− 3 Sm2 mol −1 .
(c) Iodide ion is oxidised by chlorine 3

(d) Mn 2+ is oxidised by chlorine 39. Calculate ∆Gr° of the following reaction:


33. While Fe3+ is stable, Mn 3+ is not stable in acid solution (a) Ag (aq) + Cl − (aq) → AgCl (s)
+

because
Given
(a) O2 oxidises Mn 2+ to Mn 3+
(b) O2 oxidises both Mn 2+ to Mn 3+ and Fe2+ to Fe3+ ∆G °f (AgCl ) −109 kJ /mol
(c) Fe3+ oxidises H2 O to O2 ∆G °f (Cl )− −129kJ /mol
(d) Mn 3+ oxidises H2 O to O2
∆G ° (Ag+ )
f
77kJ /mol
34. Sodium fusion extract, obtained from aniline, on treatment
with iron (II) sulphate and H2 SO4 in the presence of air gives Represent the above reaction in form of a cell. Calculate E°
a Prussian blue precipitate. The blue colour is due to the of the cell. Find log10 K sp of AgCl. (2005, 6M)
formation of (b) 6.539 × 10−2 g of metallic Zn(u = 65.39) was added to
(a) Fe4 [Fe (CN)6 ] 3 (b) Fe3 [Fe(CN)6 ] 2 100 mL of saturated solution of AgCl. Calculate
(c) Fe4 [Fe(CN) 6 ] 2 (d) Fe3 [Fe(CN)6 ] 3 [ Zn 2+ ]
log10 . Given that
[ Ag+ ]2
Passage IV
Ag+ + e− → Ag; E° = 0.80V
Tollen’s reagent is used for the detection of aldehydes. When 2+ −
a solution of AgNO3 is added to glucose with NH4 OH, then Zn + 2e → Zn; E° = −76V
gluconic acid is formed. Also find how many moles of Ag will be formed?
Ag + + e− → Ag; Ered °
= 0.80 V 40. Find the equilibrium constant for the reaction
C 6 H12 O6 + H2 O → C 6 H12 O7 + 2H+ + 2e− ; Cu 2+ + In 2 + r Cu + + In 3+
Gluconic acid °
Eoxi = − 0.05 V Given that E°Cu 2+ /Cu + = 0.15V,
Ag(NH 3 )+2 + e− → Ag ( s) + 2NH 3; °
Ered = 0.337 V E° In 2+ / In + = − 0.4V,
[Use 2.303 ×
RT
= 0.0592 and
F
= 38.92 at 298 K] E° = −0.42 V
In 3+ / In + (2004, 4M)
F RT
(2006, 3 × 4M = 12M) 41. (a) Will pH value of water be same at temperature 25°C and
35. 2Ag + + C 6 H12 O6 + H2 O → 2Ag ( s) + C 6 H12 O7 + 2H+ 4°C? Justify in not more than 2 or 3 sentences.
Find ln K of this reaction. (b) Two students use same stock solution of ZnSO4 and a
(a) 66.13 (b) 58.38 (c) 28.30 (d) 46.29 solution of CuSO4. The emf of one cell is 0.03 V higher
than the other. The concentration of CuSO4 in the cell with
36. When ammonia is added to the solution, pH is raised to 11.
higher emf value is 0.5 M. Find out the concentration of
Which half-cell reaction is affected by pH and by how much? CuSO4 in the other cell. Given : 2.303 RT /F = 0.06V .
°
(a) Eoxi will increase by a factor of 0.65 from Eoxi
° (2003, 2M)
(b) Eoxi will decrease by a factor of 0.65 from Eoxi
°
(c) E red will increase by a factor of 0.65 from E red 42. The standard potential of the following cell is 0.23 V at 15° C
(d) E red will decrease by a factor of 0.65 from E °red and 0.21 V at 35° C.
37. Ammonia is always added in this reaction. Which of the Pt | H2 ( g ) | HCl( aq ) | AgCl ( s ) | Ag ( s )
following must be incorrect? (i) Write the cell reaction.
(a) NH3 combines with Ag + to form a complex (ii) Calculate ∆H ° and ∆S ° for the cell reaction by assuming
(b) Ag(NH3 ) 2+ is a stronger oxidising reagent than Ag + that these quantities remain unchanged in the range 15° C to
(c) In the absence of NH3 silver salt of gluconic acid is 35° C.
formed (iii) Calculate the solubility of AgCl in water at 25° C.
(d) NH3 has affected the standard reduction potential of Given, the standard reduction potential of the
glucose/gluconic acid electrode (Ag+ (aq)/Ag (s) is 0.80 V at 25° C. (2001, 10M)
164 Electrochemistry

43. Find the solubility product of a saturated solution of 51. The standard reduction potential at 25° C of the reaction,
Ag 2 CrO4 in water at 298 K, if the emf of the cell Ag | Ag + 2H2 O + 2e− r H2 + 2OH− , is −0.8277 V. Calculate the
(Saturated Ag 2 CrO4 solution. ) ||Ag + (0.1 M) | Ag is 0.164 V equilibrium constant for the reaction,
at 298 K. (1998, 6M) 2H2 O r H3 O+ + OH− at 25° C. (1989, 3M)

44. Calculate the equilibrium constant for the reaction, 52. The emf of a cell corresponding to the reaction.
2Fe3+ + 3I− r 2Fe2+ + I3− . The standard reduction
Zn( s ) + 2H+ ( aq ) → Zn 2+ (0.1M) + H2 , ( g ,1atm)
potentials in acidic conditions are 0.77 V and 0.54 V
is 0.28 V at 25° C.
respectively for Fe3+ / Fe2+ and I−3 / I− couples. (1998, 3M)
Write the half-cell reactions and calculate the pH of the
45. Calculate the equilibrium constant for the reaction solution at the hydrogen electrode.
Fe2+ + Ce4+ r Fe3+ + Ce3+ E° ( Zn 2+ / Zn ) = −0.76 V E ° + =0
H /H2 (1986, 4M)
4+ 3+ 3+ 2+
Given, E° ( Ce /Ce ) = 1.44 V, E° ( Fe /Fe ) =0.68 V 53. Give reasons in one or two sentences.
(1997, 2M)
“Anhydrous HCl is a bad conductor of electricity but aqueous
46. The standard reduction potential for Cu 2+ /Cu is +0.34 V. HCl is a good conductor.” (1985, 1M)
Calculate the reduction potential at pH = 14 for the above
54. Consider the cell,
couple. K sp of Cu (OH)2 is 1.0 × 10−19 . (1996, 3M)
Zn | Zn 2 + ( aq ) (1.0 M ) || Cu 2 + ( aq ) (1.0 M ) | Cu
47. An excess of liquid mercury is added to an acidified solution The standard reduction potentials are 0.350 V for
of 1.0 × 10−3 M Fe3+ . It is found that 5 % of Fe3+ remains at
Cu 2 + ( aq ) + 2e− → Cu
equilibrium at 25° C. Calculate E° (Hg 2+ / Hg ) assuming that
and −0.763 V for Zn 2 + ( aq ) + 2e− → Zn
the only reaction that occurs is
(i) Write down the cell reaction.
2Hg + 2Fe3+ → Hg 2+ 2 + 2Fe
2+
(ii) Calculate the emf of the cell.
Given, E° (Fe3+ / Fe2+ ) = 0.77 V (1995, 4M) (iii) Is the cell reaction spontaneous or not? (1982, 2M)
+
48. The standard reduction potential of the Ag /Ag electrode at
Integer Answer Type Questions
298 K is 0.799 V. Given that for AgI, K sp = 8.7 × 10−17 ,
55. The conductance of a 0.0015 M aqueous solution of a weak
evaluate the potential of the Ag + / Ag electrode in a saturated monobasic acid was determined by using a conductivity cell
solution of AgI. Also calculate the standard reduction consisting of platinised Pt electrodes. The distance between
potential of the I− / AgI/Ag electrode. (1994, 3M) the electrodes is 120 cm with an area of cross section of 1
49. Zinc granules are added in excess to a 500 mL of 1.0 M nickel cm 2 . The conductance of this solution was found to be
nitrate solution at 25° C until the equilibrium is reached. If the 5 × 10−7 S. The pH of the solution is 4. The value of limiting
standard reduction potential of Zn 2+ / Zn and Ni 2+ / Ni are molar conductivity ( Λ°m ) of this weak monobasic acid in
– 0.75 V and – 0.24 V respectively. Find out the aqueous solution is Z × 102 S cm −1 mol −1 . The value of Z is
concentration of Ni 2+ in solution at equilibrium. (1991, 2M) (2017 Adv.)
2+ +
50. The standard reduction potential of Cu / Cu and Ag / Ag 56. All the energy released from the reaction
electrodes are 0.337 and 0.799 V respectively. Construct a X → Y , ∆ r G ° = − 193 kJmol −1 is used for oxidising M + as
galvanic cell using these electrodes so that its standard emf is M + → M 3 + + 2e− , E ° = − 0.25 V.
positive. For what concentration of Ag + will the emf of the
Under standard conditions, the number of moles of M +
cell, at 25° C, be zero if the concentration of Cu 2+ is 0.01 M?
oxidised when one mole of X is converted to Y is
(1990, 3M)
[F = 96500 C mol − ] (2015 Adv.)

Answers
Topic 1 25. (a, b) 26. (a,b,d) 27. (13.32) 28. (10)
1. (b) 2. (d) 3. (a) 4. (c) 29. (–11.62 JK mol −1) 30. (b) 31. (d)
5. (b) 6. (b) 7. (c) 8. (a)
32. (d) 34. (8 × 10 −5 M) 35. (0.01 V) 36. (300 cm 2)
9. (d) 10. (b) 11. (d) 12. (b)
37. (−245.11 kJ) 40. (1.4085 M) 41. (−0.037 V) 42. (0.154 M)
13. (c) 14. (a) 15. (c) 16. (a)
43. (347.40 kJ) 45. (190.50 g) 46. (10 −4 M) 47. (19.1 g)
17. (a) 18. (b) 19. (d) 20. (c)
21. (d) 22. (c) 23. (c) 24. (a) 48. (265 Ah) 49. (125 s) 50. (0.80 A) 51. (1.172 M)
Electrochemistry 165

Topic 2 29. (d) 30. (b) 31. (c) 32. (c)


1. (b) 2. (a) 3. (c) 4. (b) 33. (d) 34. (a) 35. (b) 36. (c)
5. (c) 6. (d) 7. (a) 8. (a) 37. (d) 38. (55) 40. (1010) 41. (0.05 M)
9. (b) 10. (b) 11. (c) 12. (c) 43. (2.45 × 10 −12) 44. (5.89 × 10 7)
13. (a) 14. (c) 15. (a) 16. (d) 45. (6.88 × 1012) 46. (− 0.222 V) 47. (0.7926 V) 49. (1.7 × 1017)
17. (a) 18. (d) 19. (d) 20. (b) 50. (1.57 × 10 −9) 51. (1.04 × 10 −14 ) 52. (8.6)
−1 −1
21. (b) 22. (b) 23. (b) 24. (c) 55. (6 × 10 S cm
2
mol ) 56. (4 mol)
25. (144) 26. (–6) 27. (–14.16) 28. (b)

Hints & Solutions


Topic 1 Electrochemical Cells ⇒ Eº
Fe3+ / Fe2 +
= (3z − 2 y) V
1. Key Idea Negative E ° means that redox couple is weaker So, from equation (i)
oxidising agent than H+ /H2 couple. Positive E° means that º
Ecell = xV − (3z − 2 y) V = (x − 3z + 2 y) V
redox couple is a stronger oxidising agent than H+ / H2 4. Higher the standard reduction potential (E ºMn + / M ), better is
couple
oxidising agent. Among the given, E °S2O 82− / SO 24− is highest, hence
Given, Co3+ + e− → Co2+ ; E ° = + 1.81 V S2O82− is the strongest oxidising agent.
4+ − 2+
Pb + 2e → Pb ; E ° = + 1.67 V The decreasing order of oxidising agent among the given option
Ce4+ + e− → Ce3+ ; E ° = + 1.61 V is as follows:
S2O82− > Au 3+ > O2 > Br2
Bi3+ + 3e− → Bi; E ° = + 0.20 V
Oxidising power of the species increases in the 5. Reducing power of an element
order of Bi3 + < Ce4+ < Pb4+ < Co3+ . 1

Higher the emf value, stronger the oxidising power. The Standard reduction potential
maximum value of emf is possessed by Co3+ . Hence, it has Here, E ° values of the given metals are as,
M2 + / M
maximum oxidising power. Whereas Bi3+ possess the lowest
Metals Ni Zn Mg Ca
emf value. Hence, it has minimum oxidising power.
E°( V ) − 0.25 − 0.76 − 2.36 − 2.87
2. A solution of Ni(NO3 )2 is electrolysed between platinum
→
electrodes using 0.1 Faraday electricity. It means that 0.1 Reducing power
equivalent of Ni2+ will be discharged. Thus, the correct order of increasing reducing power of the given
Electrolysis of Ni(NO3 )2 gives metal is,
Ni2+ + 2e− → Ni (Atomic mass of Ni = 58.7) Ni < Zn < Mg < Ca.
Number of equivalents = Number of moles × number of electrons.
6. Key Idea This question is based upon Faraday’s first law
. = Number of moles × 2
01 which states that “Mass of any substance deposited or
01
. liberated at any electrode is directly proportional to the
∴Number of moles of Ni = = 0.05
2 quantity of electricity passed.”
3. Fe2+ (aq) + Ag + (aq) → Fe3 + (aq) + Ag(s) During charging:
º
∴ Ecell = Eº + − Eº 3+ 2+ = xV − E º 3+ 2+ … (i) Pb + SO24 − → PbSO4 + 2e−
Ag /Ag Fe / Fe Fe / Fe

Now, for two half-cells ⇒ 1 F ≡ 1g-equiv. of PbSO4


1 303
(i) Fe2+ + 2e− → Fe; EFe
º
2+
/ Fe
= E1º = yV ∆G2º = − 2FE1º = mol of PbSO 4 ⇒ g PbSO4
2 2
(ii) Fe3 + + 3e− → Fe ; EFe
º
= E2º = zV ∆G2º = − 3FE2º 303
3+
/ Fe ∴ 0.05 F ≡ × 0.05 g of PbSO4 = 7.575 g of PbSO4
2
So, Fe3 + + e– → Fe2+ ; E º = E3º = ?
Fe3+ / Fe2 +
7. Given that, i = 100 amp. also, 27.66 g of diborane (B2H6 )
; ∆G3 = − 1 × FE3º
º
Molecular mass of B2 H6 = 10.8 × 2 + 6 = 27.6
Again, ∆G3 = ∆G2 − ∆G1º
º º
Given mass 27.66
Number of moles of B2 H6 in 27.66 g = = ≈1
⇒ − FE3º = − 3FE2º − (− 2FE1º ) Molar mass 27.6
⇒ − E3º = 2E1º − 3E2º ⇒ E3º = 3E2º − 2E1º Now consider the equation
B2H6 + 3O2 → B2O3 + 3H2O
166 Electrochemistry

From the equation we can interpret that 3 moles of oxygen is 12. 0.01 mol of H2 = 0.02 g equivalent
required to burn 1 mole (i.e. 27.6 g) B2 H6 completely.
⇒ Coulombs required = 0.02 × 96500 = 1930 C
Also consider the electrolysis reaction of water i.e.
⇒ Q = It = 1930 C
H2O s 2H+ + O−− 1930
+2e −
⇒ t= = 19.3 × 104 s
2H+ → 2H → H2 ↑ 10 × 10− 3
Cathode
2 such 13. In electrolytic cell electrolysis occur at the cost of electricity :
O−− →
Anode
O → O2 ↑

−2e atoms At cathode : M n+ + ne → M (electron gone in solution)
From the above equation it can be easily interpreted that in At anode : X n−
→ X + ne−
electrolysis of water for the production of 1 mole of oxygen from (electron supplied to anode)
1 mole of H2O at anode 4 moles electrons are required. Therefore, electron is moving from cathode to anode via internal
Likewise for the production of 3 moles of O2 12(3 × 4 ) moles of circuit.
electrons will be needed.
So, the total amount of charge required to produce 3 moles of
14. MnO−4 cannot be used for oxidation of Fe2+ in HCl medium
oxygen will be 12 × F or 12 × 96500 because the following reaction is spontaneous :
We know Q = it MnO−4 + Cl − → Mn 2+ + Cl 2; E° = 1.51 − 1.40 = 0.11 V
So, 12 × 96500 = 100 × tin seconds In all other cases, the redox process between oxidising agent and
12 × 96500 medium (HCl or H2SO4) are non-spontaneous, would not
or = tin hours = 3.2 hours
100 × 3600 interfere oxidation of Fe2+ .
8. The substances which have lower reduction potentials are 15. One of the requirement for electrolyte used in salt-bridge is, both
cation and anion must have comparable size so that they migrate
° 3 + / Cr = − 0.74 V) is
stronger reducing agents. Therefore, Cr (ECr
towards electrodes of opposite polarity at comparable speeds.
the strongest reducing agent among all the other given options.
16. Higher the value of reduction potential, stronger the oxidising
9. Oxidation at anode agent.
° = 0.00 V
H2 (g ) → 2H+ (aq)+ 2e−; ESHE Q E° : Z > Y > X
Reduction at cathode ⇒ Y will oxidise X but not Z.
M 4+ (aq)+ 2e− → M 2+ (aq); EM° 4+ / M 2+ = 0.151 V 17. Lower the value of E° , stronger the reducing agent.
Net: M 4+ (aq) + H2 (g ) → M 2+ (aq) + 2H+ (aq); Reducing power:
[ M 2+ ] [ H+ ] 2 [ M 2+ ] Y (E ° = − 3.03 V) > Z (E ° = − 1.18 V) > X (E ° = 0.52 V).
K= ° = 0.151 V )=
(Ecell
4+
[ M ] pH 2 [ M 4+ ] 18. Fe2+ + 2e− → Fe ; E° = − 0.41V
2+ −
° −
Ecell = Ecell
0.059
log K Zn → Zn + 2e ; E° = + 0.76V
2 ⇒ Fe 2+
+ Zn → Zn 2+
+ Fe ; E° = + 0.35V
0.059 [ M 2+ ]
0.092 = 0151. − log 4+ 19. In a lead storage battery, sulphuric acid is consumed as :
2 [M ]
0.059 Pb + PbO2 + 2H2SO4 → 2PbSO4 + 2H2O
0.059 = log10 x
2 20. In a galvanic cell, oxidation occur in the left hand electrode
∴ log10 = 2
x
chamber and reduction in right hand electrode chamber. In the
∴ x=2 following cell.
10. Given, Q = 2F Pt | H2 (g ) | HCl (l )||AgCl (s) | Ag(s)
Atomic mass of Cu = 63.5u The cell reactions are :
1
Valency of the metal Z = 2 H (g ) → H+ + e− At anode
2 2
We have, CuSO4 → Cu 2+ + SO24− − −
AgCl(s) + e → Ag + Cl At cathode
Cu 2+ + 2 e− → Cu 1
1mol 2mol 1mol =63.5g Net : H2 (g ) + AgCl (s) → H+ + Ag(s) + Cl −
2F 2
Alternatively.
E 2 × 63.5 21. One gram equivalent of an electrolyte required 1.0 mole of
W = ZQ = ⋅ 2F = 2E = = 63.5 electronic charge for discharging.
F 2
11. Higher the standard reduction potential, better is oxidising agent. 22. In aqueous solution, only those ions who are less electropositive
°
Among the given EMnO −
/ Mn 2+
is highest, hence MnO −4 is the than hydrogen (E° > 0) would be deposited.
4
Therefore, in the present case, only Ag, Hg and Cu would be
strongest oxidising agent.
deposited on passing electricity through aqueous solution of
these ions, Mg will not be deposited.
Electrochemistry 167

Also, higher the value of E°, easier will be their reduction, and [Mg 2+ (aq)]= xM
therefore, the sequence in which ions will be deposited on F
increasing voltage across the electrodes is : and = 11500 KV −1
R
Ag, Hg, Cu. where F = Faraday constant, R = gas constant
23. Faraday’s law of electrolysis is related to equivalent weight of From the formula,
2+
electrolytes as “the number of Faraday’s passed is equal to the RT [Mg (aq)]
° −
Ecell = Ecell ln
number of gram equivalent of electrolytes discharged.” nF [Cu 2+ (aq)]
24. Lower the value of E°, stronger the reducing agent. After putting the given values
25. RT x
PLAN This problem is based on characteristics of salt-bridge. 2.67 = 2.70 − ln
Functions of salt-bridge are 2F 1
R × 300
(i) It connects the two half-cells and completes the cell circuit. or 2.67 = 2.70 − × ln x
(ii) It keeps the solutions of two half-cells and complete the cell 2F
− R × 300
circuit but does not participate chemically in the cell reaction. −0.03 = × ln x
2F
(iii) It maintains the diffusion of ions from one electrode to
0.03 × 2 F 0.03 × 2 × 11500
another electrode. or ln x = × = = 2.30
300 R 300
(iv) A cell reaction may also occur in the absence of salt-bridge.
Sometimes, both the electrodes dip in the same electrolyte So, ln x = 2.30
solution and in such cases we do not require a salt-bridge.” or x =10 (as given ln (10) = 2.30)
So, option (c) is incorrect. 29. Given,
(v) This prevent mixing of two electrolytic solutions hence, A (s)| A n+ (aq, 2 M ) || B 2n+ (aq, 1 M ) | B (s)
option (d) is incorrect choice.
So, reactions at respective electrode will be
Hence, correct choices are (a), (b).
Anode A (s)→ A n+ + ne− × 2
26. Metals with E° value less than 0.96 V will be able to reduce NO−3 Cathode B 2n+ + 2 ne− → B (s)
in aqueous solution.Therefore, metals V (E° = − 1.19 V),
Overall reaction
Fe (E° = – 0.04 V), Hg (E° = 0.86 V) will all reduce NO−3 but Au
2 A (s) + B 2n+ (aq) → 2 A n+ (aq) + B (s)
(E° = 1.40 V) cannot reduce NO−3 in aqueous solution.
Further,
27. Vessel is insulated, thus q = 0 ∆H ° = 2∆G ° and Ecell = 0 is also given
For the given reaction : Now by using the Nernst equation
H2 (g ) + 1/ 2 O2 (g ) → H2O(l ); E° = 123. − 0.00 = 123
. V RT [Product]
Ecell = E°cell − ln
∆G ° = − nFE ° = − 2 × 96500 × 123 . J / mol nF [Reactant]
Therefore, work derived from this fuel cell using 70% efficiency After putting the values
and on consumption of 10 . × 10− 3 mol of H2 (g ) RT [ A n+ ]2
0 = Ecell
° − ln
. × 0.7 × 1 × 10− 3
= 2 × 96500 × 123 2nF [ B 2n+ ]
= 16617
. J RT [ 2 ]2 RT
or E° = ln = ln 4 …(i)
This work done = change in internal energy (for monoatomic 2 nF [1] 2nF
gas, CV , m = 3R / 2 ) , Further from the formula,
. ×2
16617 ∆G ° = − nFE ° ⇒ ∆G ° = − 2nFE °
166.17 = nCV , m∆T ⇒ ∆T = ⇒ 13.32 K
1 × 3 × 8.314 Now putting the value of E° from eq. (i)
RT
28. Equation of cell reaction according to the cell notation given, is ∆G ° = − 2nF × ln 4 …(ii)
Reduction 2nF
∆G ° = − RT ln 4
Mg(s) + Cu2+(aq) Mg2+(aq) + Cu(s) Finally, using the formula
∆ G ° = ∆ H ° − T∆ S °
Oxidation ∆G ° = 2∆G ° − T∆S ° (as ∆H ° = 2∆G °, given)
° = 2.70 V, T = 300 K
Given, Ecell ∆ G ° = T∆ S °
∆G ° − RT ln 4
with [Mg 2+ (aq)]=1 M and [Cu 2+ (aq)]=1 M or ∆S ° = =
T T
and n= 2 (from eq. (ii), ∆G ° = − RT ln 4)
Further, Ecell = 2 . 67 V = − R ln 4 = − 8.3 × 2 × 0.7
with [Cu 2+(aq) ]=1M (as all values given)
= − 1162
. J/K-mol
168 Electrochemistry

500 37. Given, FeO (s)/Fe (s) E° = – 0.87 V


30. Moles of NaCl electrolysed = 4 × = 2.0
1000 and Ni 2O3 /NiO (s) E° = + 0.40 V
⇒ moles of Cl 2 produced = 1.0 Electrode at lower reduction potential act as anode and that at
2Cl − → Cl 2 + 2e− higher reduction potential act as cathode.
Hg (i) Electrodes reaction :
31. At cathode Na + + e− → Na(Hg) Fe(s) + 2OH− → FeO (s) + H2O (l )
Amalgam
E° = + 0.87 V
Two moles of Na formed during electrolysis would produce two
moles of Na(Hg) amalgam. Ni 2O3 (s) + H2O (l ) + 2e− → 2NiO (s) + 2OH− E° = 0.40 V
⇒ mass of amalgam = 2 × (23 + 200) = 446 g Net : Fe (s) + Ni 2O3 (s) → 2NiO (s) + FeO (s)
E° = 1.27 V
32. Two Faraday of electric charge would be required for
electrolysis of 2.0 moles of NaCl. (ii) Emf is independent of concentration of KOH.
⇒ total coulombs = 2 × 96500 = 193000 C (iii) Maximum amount of energy that can be obtained = ∆G °
⇒ ∆G ° = − nE ° F = – 2 × 1.27 × 96500 J = – 245.11 kJ
° . Also, left
33. Since, activities of all the ions are unity, Ecell = Ecell
i.e. 245.11 kJ is the maximum amount of obtainable energy.
hand electrode is at lower reduction potential, it act as anode and
1
E ° = E ° (Ce4+ , Ce3+ ) − E ° (Fe3+ , Fe2+ ) = 0.84 38. (i) E = 0.78 – 0.0592 log 2 = 0.887 V
8
i.e. electrons will flow from left to right hand electrode and 1
current from right hand electrode [Pt (2)] to left hand electrode (ii) E = 0.78 – 0.0592 log = – 0.0488 V
(10− 7 )2
[Pt(1)].
[Fe3+ ][Ce3+ ] 39. Molar mass of Cr = 52 g
Also, E = E ° − 0.0592 log 52
[Fe2+ ][Ce4+ ] Equivalent mass of Cr = g
6
As electrolysis proceeds, E will decrease and therefore, current.
(i) Mass of Cr deposited on passing 24000 Coulombs
2 × 10− 3 × 16 × 60
34. The number of Faraday’s passed = 24000 52
96500 = × g = 2.15 g
96500 6
=1.99 × 10− 5 1.5 9
(ii) Number of gram equivalent of Cr = ×6=
⇒ number of gram equivalent of Cu 2+ deposited = 1.99 × 10− 5 52 52
1.99 9
⇒ number of moles of Cu 2+ deposited = × 10− 5 ≈ 10− 5 ⇒ Coulombs required for 1.5 g Cr = × 96500 = It
2 52
Absorbance is directly proportional to [Cu 2+ ]. Therefore, 9 × 96500
⇒ t= s = 22.27 min
if ‘C’ be the initial molarity, 0.5 C will be the final molarity. 52 × 12.5
⇒ 0.5 C × 0.25 = 10− 5 ⇒ C = 8 × 10− 5 M 40. At anode 2Cl − → Cl 2 + 2e−
9.65 × 60 × 60 At cathode 2H2O + 2e− → H2 + 2OH−
35. The number of Faraday’s passed = = 0.36 F
96500 1000
1 kg Cl 2 = equivalent of Cl 2 = 28.17 equivalent
After electrolysis : [Ag+ ] = 1.36 M 35.5
0.36 ⇒ Theoretical electricity requirement = 28.17 F
[Cu 2+ ] = 1 – = 0.82 M
2 Q Efficiency is only 62%
E1 (before electrolysis) = E ° ∴ Electricity requirement (experimental)
0.0592 [Ag+ ]2 28.17 × 100
E2 (after electrolysis) = E ° − log = F = 45.44 F
2 [Cu 2+ ] 62
0.0592 (1.36)2 ⇒ 45.44 × 96500 = 25 t (in second)
⇒ E1 − E2 = log = 0.01 V (decreased)
2 0.82 ⇒ t = 48.72 h
36. Coulombs passed = 8.46 × 8 × 60 × 60 = 243648 C Also, gram equivalent of HO− produced = 28.17
28.17
243648 ⇒ Molarity of HO− = = 1.4085 M
Number of Faraday’s passed = = 2.52 20
96500
63.5 2.8 × 10− 10
Weight of Cu plated = 2.52 × g = 80.01 g 41. [Ag+ ] in left hand electrode chamber =
2 0.2
80.01 = 1.4 × 10− 9 M
Volume of Cu plated = = 7.62 cm 3
10.5 3.3 × 10− 13
[Ag+ ] in right hand electrode chamber =
7.62 0.001
⇒ Area plated out = = 3000 cm 2
0.00254 = 3.3 × 10− 10 M
Electrochemistry 169

[Ag+ ] anode 46. Q Emf = 0.118 V > 0, it is galvanic cell and anode is negative
emf = 0 – 0.0592 log
[Ag+ ] cathode electrode :
At anode : H2 (g ) → 2H+ (10− 6 M ) + 2e−
1.4 × 10− 9
= – 0.0592 log = – 0.037 V At cathode : 2H+ (x ) + 2e− → H2
3.3 × 10− 10
Cell reaction : H+ (x ) → H+ (10− 6 M )
Therefore, the cell as written is non-spontaneous and its reverse 10− 6
will be spontaneous with emf = 0.037 V. Emf = 0.118 V = 0 – 0.0592 log ⇒ x = 10− 4 M
x
1.7 × 230
42. Faraday’s passed = = 4.052 × 10− 3 F 47. 100 W lamp will produce 100 Js − 1.
96500 10
Faradays used for reduction of Zn 2+ = 4.052 × 10− 3 × 0.9 ⇒ 100 J = 110 × C ⇒ C= Coulombs
11
= 3.65 × 10− 3 Therefore, total Coulomb passed in 10 h
⇒ Meq. of Zn 2+ reduced = 3.65 10
= × 10 × 60 × 60 = 32727.27 C
Initial meq. of Zn 2+ = 300 × 0.16 × 2 = 96 11
⇒ Meq. of Zn 2+ remaining = 96 − 3.65 = 92.35 Number of gram equivalent of Cd 2+ deposited
32727.27
92.35 1 = = 0.34
⇒ Molarity of Zn 2+ = × = 0.154 M 96500
2 300
112.4
43. Weight of Cd deposited = 0.34 × g = 19.1 g
NO2 NH2 2
48. For 1.0 L H2SO4 :
39
Initial mass of H2SO4 = 1294 × = 504.66 g
100
Change in oxidation number at nitrogen = 4 − (− 2) = 6 20
Final mass of H2SO4 = 1139 × = 227.80 g
123 100
Equivalent weight of nitrobenzene = g
6 ⇒ H2SO4 consumed/litre = 504.66 – 227.80 = 276.86 g
12.3 × 6 ⇒ Total H2SO4 used up = 276.86 × 3.5 = 969.01 g
⇒ gram equivalent of nitrobenzene = = 0.60
123 969.01
= mol = 9.888 mol
⇒ Theoretical requirement = 0.60 × 96500 C = 57900 C 98
⇒ Actual requirement of electricity = 2 × 57900 = 115800 C Q 1 mole of H2SO4 is associated with transfer of 1.0 mole of
electrons, total of 9.888 moles of electron transfer has occurred.
Q V ×C = J
Coulomb produced = 9.888 × 96500
⇒ Energy consumed = 115800 × 3 J = 347.40 kJ
9.888 × 96500
Ampere-hour = = 265 Ah
44. If the salt is CuSO4 3600
During deposition of Cu at cathode, O2 (g ) will evolve at anode 0.005
49. Volume of Ag coating = 80 cm 2 × cm = 0.04 cm 3
0.4 × 2 10
gram-equivalent of Cu deposited = = 0.0126
63.5 ⇒ mass of Ag coating = 0.04 × 10.5 g = 0.42 g
Volume of O2 liberated at NTP at anode 0.42
⇒ gram equivalent of Ag = = number of Faraday’s
= 0.0126 × 5600 mL = 70.56 mL 108
0.42
In the next 7 min, H2 at cathode and O2 at anode would be ⇒ × 96500 C = 3 × t ⇒ t = 125 s
produced. 108
1.2 × 7 × 60 9.85
Faraday’s passed = = 5.22 × 10− 3 50. Moles of Au deposited = = 0.05
96500 197
⇒ Volume of H2 (at NTP) = 5.22 × 10− 3 × 11200 mL ⇒ gram equivalent of Au deposited = 0.05 × 3 = 0.15
Now, according to Faraday’s law of electrolysis, if same
= 58.46 mL
quantity of electricity is passed through different cells
Volume of O2 (at NTP) = 5.22 × 10− 3 × 5600 mL = 29.23 mL connected in series, same number of gram equivalents of
Therefore, O2 (g) at NTP = 70.56 + 29.23 = 99.79 mL electrolytes are discharged at respective electrodes.
H2 (g ) at NTP = 58.46 mL ⇒ gram equivalent of Cu deposited = 0.15
67.2 63.5
45. Total number of gram equivalent of H2 used = =6 ⇒ amount of Cu deposited = 0.15 × = 4.7625 g
11.2 2
⇒ 6 × 96500 = 15 × 60 × I ⇒ I = 643.33A Also, Coulombs passed = 0.15 × 96500 = I × 5 × 60 × 60
63.5 0.15 × 96500
Mass of Cu deposited = 6 × g = 190.50 g ⇒ I= = 0.80 A
2 5 × 3600
170 Electrochemistry

51. During electrolysis, Ni 2+ will be reduced at cathode and H2O 3. NaCl and KCl are strong electrolytes. So, the study of their molar
will be oxidised at anode. conductances (λ m ) can be experimentally verified by
3.7 × 6 × 60 × 60 Debye-Huckel Onsagar equation,
Number of Faraday’s passed = = 0.828 Λcm = Λ0m − B C
96500
⇒ 0.828 g equivalent of Ni 2+ will be deposited at cathode. Λcm = molar conductance at concentration.

Initial moles of Ni 2+ ion = 2 × 0.5 = 1.0 Λ0m = molar conductance at infinite dilution. i.e. C → 0

0.828 B = Debye-Huckel Onsagar constant.


Moles of Ni 2+ ion remaining after electrolysis = 1.0 – For (both NaCl and KCl) a strong binary electrolyte like AB, the
2
nature of the plot of Λ m vs C will be
= 0.586
2+ 0.586
⇒ Molarity of Ni in final solution = = 1.172 M
0.50

Λm
Topic 2 Conductivity of Electrolytic
Solutions and their Measurement
and Nernst Equation √C

1. Key Idea The aqueous solution of ionic surfactant, i.e. ⊕


Size of Na is being smaller than K ⊕ and Na ⊕ will remain in
– +
sodium stearate (C17H35COONa) acts as a strong univalent more hydrated state, i.e. larger sized in aqueous solution. As a
result, ionic mobility as well as ionic conductance of Na ⊕
type of electrolyte in the concentration range below the CMC
and the linear function of dependence of Λ m on C has a (or NaCl as ClÈ is common to NaCl and KCl) will be lower than
K ⊕ (or KCl). Thus, the plot of Λ m vs C for NaCl and KCl is as
small negative slope.
follows :

CMC of Lm on ÖC has a
Lm small negative slope. KC
l
Na
Cl
Λm
¾
√C
√C
At normal or low concentration, sodium stearate
[CH3(CH2)16COO −Na+ ] behaves as strong electrolyte and for
4. The explanation of statements (S 1 and S 2) are as follows :
strong electrolyte, molar conductance (Λm ) decreases with
increase in concentration. In conductivity cell, conductivity (κ ) is equal to the sum of ionic
Above particular concentration, sodium stearate forms aggregates conductances (c), of an electrolytic solution present is unit
known as micelles. The concentration is called as CMC. Since, volume of the solution enclosed by two electrodes of unit area
number of ions decreases and hence Λm also decreases. (a ≠ 1) separated by a unit length (l = 1).
l
Hence, option (b) is correct. κ = c×
a
2. Electrical conductivity of the given aqueous solutions depends ⇒ κ = cwhen l = 1, a = 1
on the degree of ionisation. Degree of ionisation is directly So, with decrease in the concentration of electrolyte, number of
proportional to the acidic strength. Electron withdrawing groups ions in the given unit volume also decreases, i.e. κ [conductivity]
(EWGs) increases the stability of the carboxylate ion by also decreases.
dispersing the negative charge through resonance effect on the Thus, statement S 1 is wrong. S 2 : Molar conductivity (λ m ) is
conjugate while electron donating groups (EDGs) decreases the defined as the conducting power of all the ions present in a
stability of the carboxylate ion by intensifying the negative solution containing 1 mole of an electrolyte.
charge. 1000
O O λ m = κ × VmL = κ ×
M
EWG C s EDG C s
where, VmL = volume in mL containing 1 mole of electrolyte
O O
Acidity of carboxylic Acidity of carboxylic m = molar concentration (mol/L)
acids increases due acids decreases due So, in a conductivity cell
to the presence of electron to the presence of
1
withdrawing groups (EWGs) electron donating groups λm ∝
M
The correct order of acidic strength and electrical conductivity is
i.e. molar conductivity increases with decrease in the
as follows:
concentration (M) of electrolyte.
HCOOH > PhCOOH > CH3COOH
A C B Thus, statement S 2 is correct.
Electrochemistry 171

5. Key Idea Gibbs energy of the reaction is related to E°cell by 1000 × κ


Λm =
the following formula M
∆Gº = − nFE °cell Given, κ = conductivity ⇒ 5 × 10− 5 S cm − 1
∆G º = Gibbs energy of cell M = Molarity ⇒ 0.001 M
nF = amount of charge passed 1000 × 5 × 10− 5 S cm− 1
E = EMF of a cell ∴ Λm = = 50 S cm 2 mol − 1
10− 3 M
Given reaction is, Therefore, degree of dissociation (α), of HA is,
Zn + Cu 2+ → Zn 2+ + Cu Λ 50 S cm2mol− 1
α= m = = 0.125
Eºcell = 2.0 V Λ m 400 S cm2mol− 1
°

F = 96000 C
7. According to Nernst equation,
n=2
2.303 RT
To find the value of ∆G º (kJ mol), we use the formula Ecell = E °cell − log Q
nF
∆G º = − nFE ºcell
2.303 RT
∆G º = −2 × 96000 × 2 = −384000 J/mol Given, = 0.059 V
F
−384000
In terms of kJ/mol, ∆G º = = −384 kJ/mol 0.059
1000 ∴ Ecell = E °cell − log Q
n
6. According to Kohlrausch’s law, the molar conductivity of HA at At equilibrium, Ecell = 0
infinite dilution is given as, 0.059
E °cell = log KC
Λ°m (HA ) = [ Λ°m (H+ ) + Λ°m (Cl− ) ] + [ Λ°m (Na + ) + Λ°m ( A − ) ] n
− [ Λ°m (Na + ) + Λ°m (Cl− ) ] For the given reaction, n = 2
Also, KC = 10 × 1015 [given]
= 425.9 + 100.5 − 126.4 = 400 S cm 2 mol − 1
0.059
Also, molar conductivity at given concentration is given as, ∴ E°cell = log (10 × 1015 ) = 0.472V ≈ 0.473V
2

8. Cell E °cell (SRP) = E °C − E ° A E°cell free e transfer


Anode (A) Cathode (C)
Zn Ag
156
.
1. [Zn + 2Ag+ → Zn2+ + 2Ag] . V for 2e−
0.80 − (−0.76) = + 156 + = + 0.78 V
2

Zn Fe 0.32
2. [Zn + Fe 2+ → Zn2+ + Fe] − 0.44 − (− 0.76) = + 0.32 V for 2e− + = + 016
. V
2

Zn Au
216
.
3. [ 3Zn + 2Au 3+ → 3Zn 2+ + 2Au] 1.40 − (−0.76) = + 2.4 V for 6e− + = + 0.36 V
6

Zn Fe
153
.
4. [ 3Zn + 2Fe3+ → Zn 2+ + 2Fe2+ ] . V for 2e−
0.77 − (−0.76) = + 153 + = + 0.765 V
2

9. It is an electrochemical cell. The overall cell reaction can be (10−6 )2 × 12


⇒ 0.92 = ( Ec° − 0 ) − 0.06 × log
written, as 1 × 12
H 2 ( g ) + 2AgCl( s ) → 2HCl( aq ) + 2Ag(s)
= Ec° + 0.06 × 6 × 2
(1 bar) (10−6 M)
⇒ Ec° = 0.92 − 0.72 = 0.20 V
(i) According to Nernst equation,
° ° 2.303 × RT [HCl]2 [ Ag]2 Note 10 −6 molal HCl is a very dilute solution.
Ecell = ( Ecathode − Eanode )− log So, 10 −6 m ~
− 10 −6 M
n×F pH [ AgCl]2 2
10. The relationship between standard electrode potential (E° ) and
Here, (i) Ec° = EAgCl/
°
Ag, Cl −
°
= Ecathode equilibrium constant (K ) of the cell reaction,
° ° Zn(s)+ Cu 2+(aq) c Zn 2+(aq)+ Cu(s)
(ii)Eanode = E2H +
/H
= 0.00 V
2 can be expressed as,
(Standard hydrogen electrode) RT
E° = ln K ⇒ K = enFE ° / RT
nF
172 Electrochemistry

Given, n = 2, F = 96000 C mol −1 Now, molar conductivity


−1 −1 κ
E° = 2 V, R = 8 JK mol λm=
1000 × m
T = 300 K
1/ 4 1
2 × 96000 × 2 = =
∴ K =e 8 × 300
= e160 1000 × 0 .5 2000
= 5 × 10−4 S m 2 mol −1
11. The redox reaction is : Zn(s) + Cu 2+ → Zn 2+ + Cu
2.303 RT
16. PLAN When different number of electrons are involved in a redox
The Nernst equation is E = E ° − log10 reaction
2F ° = ∆G °1 + ∆G °2
∆Gnet
2.303RT
= 11
. − − n3FE °3 = − n1FE °1 − n2FE °2
2F n E ° + n2E °2
∴ E °3 = 1 1
 2.303 RT  n3
Also, ∆G = − nEF = −2F 11
. − 
 2F 
(P) E3° Fe3+ / Fe
= − 2.2F + 2.303RT
Net reaction Fe3+ → Fe
= 2.303RT − 2. 2F
is obtained from n E°
12. Zinc metal is the most stable metal to cover iron surfaces. The 3+ − 2+ °
Fe + e → Fe n1 = 1 E1 = 0.77 V
d
process of coating the iron surface by zinc is called
2+ −
galvanisation. Fe + 2e → Fe n2 = 2 E2° = − 0.44 V
3+ −
13. Standard electrode potential of reaction [ E° ]can be calculated as Q Fe + 3e → Fe n3 = 3 E3° = ?
o =E − E
Ecell R P n1E °1 + n2E °2 0.77 + 2(−0.44) −0.11
E °3 = = = = −0.04 V
where, ER = SRP of reactant , EP = SRP of product n3 3 3
If Ecell
o = +ve, then reaction is spontaneous otherwise Thus, P — (3)
non-spontaneous.
Net reaction
E ° = 1.51 V
Mn 3+  
1 → Mn 2+ 4 H2O r 4 H+ + 4 OH−
2+ E 2° = − 1.18 V
Mn  
→ Mn is obtained from n E°
∴ For Mn 2+
disproportionation, 2H2O → O 2 + 4 H+ + 4 e− n1 = 4 −1.23 V
E° = − 1. 51 V − 1.18 V = − 2. 69 V < 0 2H2O + O 2 + 4 e− → 4OH− n2 = 4 + 0.40 V
Thus, all reaction will not occur. 4H2O → 4H + 4 e + −
n3 = 4 ?
14. According to Debye Huckel Onsager equation,
n1E °1 + n2E °2
λC = λ ∞ − B C E °3 = = E °1 + E °2
n3
where, λ C = limiting equivalent conductivity at concentration C
= − 1.23 + 0.40 = −0.83 V
λ ∞ = limiting equivalent conductivity at infinite dilution
Thus, Q — (4)
C = concentration (R) Cu 2+ + Cu → 2Cu +
15. In order to solve the problem, calculate the value of cell constant For thus E° of Cu 2+ → Cu +
of the first solution and then use this value of cell constant to is also required. n E°
calculate the value of k of second solution. Afterwards, finally Cu 2+ + 2e− → Cu 2 0.34 V
calculate molar conductivity using value of k and m. Cu → Cu + + e− 1 − 0.52 V
For first solution, 2+ − +
Cu + e → Cu E°3 ?
k =1.4 Sm −1, R = 50 Ω, M = 0.2
1 l n1E1 ° + n2E2 ° 2 × 0.34 + 1 × (−0.52)
Specific conductance (κ ) = × E3 ° = = = 0.16 V
R A n3 1
1 l Also, n E°
1.4 Sm −1 = ×
50 A Cu → Cu + + e− n1 = 1, − 0.52 V

l
= 50 × 1.4 m −1 Cu 2+ + e− → Cu + n2 = 1 0.10 V
A 2+ +
l Cu + Cu → 2Cu
For second solution, R = 280, = 50 × 1.4 m −1
A E° = − 0.52 + 0.16 = − 0.36 V
1 1 Thus, (R) — (1)
κ= × 1.4 × 50 =
280 4
Electrochemistry 173

(S) Cr 3+ → Cr 2+ In burette
In flask base Curve
is obtained from n E° acid
Cr 3+ + 3e− → Cr 3 − 0.74 V IV. Weak Weak Conductivity increases due
Cr → Cr 2+ + 2e− 2 + 0.91V [(C 2H 5) 3N] (CH 3COOH) to formation of ions and
then remains constant due
Cr 3+ + e− → Cr 2+ 1 ? to addition of weak base.
− 0.74 × 3 + 2 × 0.91 V. KX AgNO3 Insoluble salt AgX is
E3 ° = = −0.4 V formed, hence
1
Thus, S = (2) conductance remains
P — (3), Q — (4), R — (1), S — (2) constant. It increases due
to addition of KX.
17. The variation is conductivities in general can be seen as :
In burette 18. The half reactions are Fe( s ) → Fe2+ ( aq ) + 2e– × 2
In flask base Curve
acid O2 (g ) + 4H+ + 4 e– → 2H2O
I. Strong Strong Conductance first 2Fe(s) + O2 (g ) + 4H+ → 2Fe2+ (aq) + 2H2O(l ) ;
(HI) (NaOH) decreases due to formation
of H2O and then increases 0.059 (10–3 )2
E = E º– log = 1.57V
due to addition of strong 4 (10–3 )4 (0.1)
electrolyte.
19. As AgNO3 is added to solution, KCl will be displaced according
to following reaction.
AgNO3 (aq) + KCl(aq) 
→ AgCl(s) + KNO3 (aq)
Conductance

For every mole of KCl displaced from solution, one mole of


KNO3 comes in solution resulting in almost constant
conductivity. As the end point is reached, added AgNO3 remain
Volume of acid added in solution increasing ionic concentration, hence conductivity
increases.
II. Strong Weak Conductance increases
20. The net reaction is
(CH 3COOH) (KOH) slightly as NH+4 (salt) is
1
hydrolysed forming HCl. 2H+ + O2 + Fe → H2O + Fe2+ ; E° = 1.67 V
After neutral point, it acid 2
increases rapidly due to 2 × 1.67 × 96500
∆G ° = − nE ° F = − kJ = – 322.31 kJ
addition of strong 1000
21. The cell reaction is :
Zn + Fe2+ r Zn 2+ + Fe ; Ecell = 0.2905 V
Conductance

0.059 [Zn 2+ ]
⇒ E = E° − log
2 [Fe2+ ]
0.059 0.1
⇒ E° = 0.2905 + log = 0.32 V
2 0.01
Volume of acid added 0.059
Also E° = log K
n
III. Weak Strong Conductivity decreases 2E ° 0.32
(CH 3COOH) (KOH) due to neutralisation of ⇒ log K = =
0.059 0.0295
conducting strong base and
then remains constant due ⇒ K = (10)0.32/ 0.0295
to addition of weak acid. 22. In LiCl, NaCl and KCl, anions are same.
Cations have same charge but different size. Smaller cations are
Conductance

more heavily hydrated in aqueous solution giving larger


hydrated radius and thus smaller ionic speeds and equivalent
conductance.
⇒ Equivalent conductance : KCl > NaCl > LiCl
Weak acid added 23. The spontaneous cell reaction is
to strong base
X − + M + → M + X ; E° = 0.11V
174 Electrochemistry

24. E° is an intensive property :  pH 


or 105 + 104 ln  2  ≥ 0
E° ∆G ° = − nE ° F
 pH 2O 
(i) Cu 2+ + 2e− → Cu 0.337 V – 0.674 F
2+ − + or 10 (ln pH 2 − ln pH 2O) ≥ −105
4
(ii) Cu + e → Cu 0.153 V – 0.153 F
Subtracting (ii) from (i) gives : or ln pH 2 ≥ −10 + ln pH 2O
Cu + + e− → Cu ∆G ° = − 0.521 F = − nE ° F or ln pH 2 ≥ −10 + 2.3 log (0.01) (as pH 2O = 1%)
⇒ E° = 0.521 V ≥ −10 − 4.6
Q n=1 so ln pH 2 ≥ −14.6

25. 2Cu+ (aq) c Cu(s)+ Cu 2+(aq) 28. For the given concentration cell, the cell reaction are
M → M 2 + at left hand electrode.
For the cell reaction,
°
ECell °
= ECu +
°
− ECu 2+ = (0.52 − 016
. ) V = 0.36 V M 2+ → M at right hand electrode
/ Cu / Cu+

⇒ °
∆G ° = − nF ECell ⇒ M 2+ (RHS electrode) → M 2+ (LHS electrode)
⇒ °
− RT ln K = − nF ECell E° = 0
E 1 Applying Nernst equation
°
⇒ ln K = n × × ECell =1× × 0.36 0.059 [ M 2 + ] at LHS electrode
RT 0.025 Ecell = 0.059 = 0 − log
= 14.4 = 144 × 10−1 2 0.001
[ M 2 + ] at LHS electrode
26. ∴ ∆G º = − nFEcell
º ⇒ log =−2
0.001
Here, ∆G º = 17.37 kJ mol − 1 ⇒ [ M 2+ ] at LHS electrode = 10−2 × 0.001 = 10−5 M
n = number of electrons The solubility equilibrium for MX 2 is
F = Faraday constant = 96500 C /mol MX 2 (s) r M 2+ (aq) + 2 X − (aq)
= 17.37 × 1000 J mol − 1 Solubility product, K sp = ][ M 2+ ][ X − ]2
º
17.37 × 1000 = − 3 × 96500 × Ecell = 10−5 × (2 × 10−5 )2 = 4 × 10−15
17. 37 × 1000
º
Ecell = [Q In saturated solution of MX 2 , [ X − ] = 2 [ M 2+ ]]
3 × 96500
2 × 0.059 × 96500
Ecell = − 0.06 = − 6 × 10− 2
º 29. ∆G = − nEF = − kJ = − 11.4 kJ
1000
27. Given 30. M (s) + M + (aq, 1 M ) → M + (aq, 0.05 M ) + M (s)
1 2.303 RT 0.05
(i) 2Cu(s)+ O 2 (g ) → Cu 2O(s); ∆G º= −78000 J mol −1 Ecell = 0 − log >0
2 F 1
=−78 kJ mol −1 Hence, | Ecell | = Ecell = 0.70 V and ∆G < 0 for spontaneity of
1 reaction.
(ii) H2 (g ) + O 2 (g ) → H2O(g ); ∆G º= −178000 J mol −1
2 0.0538
31. Ecell = E ° − log 0.0025 = 0.139 V ≈ 140 mV
= −178 kJ mol −1 1
So, net reaction is (By (i)-(ii)) ° >0
32. For spontaneous redox reaction : Ecell
2Cu(s)+H2O(g) → Cu 2O(s)+H2(g); For 2I− + Cl 2 → 2Cl − + I2
∆G =100000 J/ mol or 105 J/mol = 100 kJ mol −1 E° = 1.36 – 0.54 = 0.82 V > 0
Now , for the above reaction i.e. Cl 2 will spontaneously oxidise I− .
 pH  ° < 0, they are non-spontaneous.
∆G = ∆G º+ RT ln  2  In other cases Ecell
 pH 2O  33. For the reaction :
and to prevent above reaction,
(i) 4Fe3+ + 2H2O → 4Fe2+ + 4H+ + O2; E° = − 0.46 V
∆G ≥ 0
(ii) 4Mn 3+ + 2H2O → 4Mn 2+ + 4H+ + O2;E° = + 0.27 V
So,
 pH  As evidenced above, reaction (i) is non-spontaneous, therefore,
∆G º + RT ln  2  ≥ 0 Fe3+ is stable in acid solution. However, reaction (ii) is
 pH 2O  spontaneous Mn 3+ oxidises H2O to O2 and itself reduced to
After putting the values, Mn 2+ in acidic medium.
 pH  34. Sodium fusion extract from aniline produces NaCN which reacts
105 + 8 × 1250 ln  2  ≥ 0 with Fe2+ to form [Fe(CN)6 ] 4− . The complex ion then reacts with
 pH 2O  Fe3+ to give blue precipitate of prussian blue.
Fe3+ + [Fe(CN)6 ]4− r Fe4[Fe(CN)6 ]3
Prussian blue
Electrochemistry 175

35. E° for 2Ag+ + C6H12O6 + H2O r 2Ag(s) 40. Given,


+ C6H12O7 + 2H+ is 0.75 V In 2+ + e− → In + E° = – 0.40 …(i)
0.0592 2E °
Also E ° = log K ⇒ log K = = 25.33 ⇒ ∆G ° = 0.40 F
2 0.0592
⇒ ln K = 2.303 log K = 58.35 In 3+ + 2e− → In + E° = – 0.42 …(ii)
⇒ ∆G ° = 0.84 F
36. On increasing concentration of NH3 , the concentration of H+ ion
Subtracting (i) from (ii)
decreases, therefore,
In 3+ + e → In 2+ ; ∆G ° = 0.44 F = – E ° F
° − 0.0592 log [ H+ ]2 = 0 − 0.0592 × 2 log 10− 11
Ered = Ered
2 2 ⇒ E° = − 0.44 V
= 0.65 V Now, for : Cu 2+ + In 2+ → Cu + + In 3+
i.e. Ered increases by 0.65 V.
E ° = E ° (Cu 2+ / Cu + ) − E ° (In 3+ / In 2+ )
37. NH3 has no effect on the E° of glucose/gluconic acid electrode. = 0.15 – (– 0.44) = 0.59 V
−7
38. The solubility of AgBr in10 M AgNO3 solution is determined as Also E° = 0.0590 log K

AgBr r Ag+ + Br − ⇒ log K = = 10 ⇒ K = 1010
S + 10− 7 S 0.059
AgNO3 → Ag+ + NO−3 41. (a) pH = – log [H+ ]
S + 10− 7 10−7 In pure water, [H+ ] depends on value of K w which is
−14 −7
K sp = 14 × 10 = S (S + 10 ) K w = [H+ ][OH− ]
Solving for S gives : S = 3 × 10−7 M Q K w is a function of temperature, [H+ ] will change with
⇒ [Br − ] = 3 × 10−7 M, temperature.
(b) Let the emf of first cell be X volt.
[Ag+ ] = 4 × 10−7 M,
⇒ emf of 2nd cell = (X + 0.03) volt
[NO−3 ] = 10− 7 M [Cu 2+ ] in 2nd cell = 0.50 M
⇒ κ (sp. conductance) = κ Br − + κ Ag + + κ NO − [Cu 2+ ] in 1st cell = ?
3
−3 −7 −3 −7
= [8 × 10 × 3 × 10 + 6 × 10 × 4 × 10
2.303 RT [Zn 2+ ]
−3 −7 E1 = E1° − log
+ 7 × 10 × 10 ] 1000 2F [Cu 2+ ]
= 24 × 10−7 + 24 × 10−7 + 7 × 10−7
2.303 RT [Zn 2+ ]
−7
= 55 × 10 S m −1
= 55 (in terms of 10 −7
Sm ) −1 E2 = E1° − log
2F [Cu 2+ ]2
39. (a) ∆G ° = Σ ∆G °f (products) − Σ ∆G °f (reactants) 2.303 RT  [Cu 2+ ]2 
⇒ E2 − E1 =  log 
= – 109 – (– 129 + 77) kJ = – 57 kJ 2F  [Cu 2+ ]1 
Cell : Ag | AgCl, Cl − || Ag+ | Ag 0.50
⇒ 0.03 = 0.03 log
For K sp ; reaction is AgCl (s) r Ag+ + Cl − [Cu 2+ ]1

∆G ° = + 57 kJ ⇒ ∆G ° = − RT ln K sp 0.50
⇒ = 10 ⇒ [Cu 2+ ]1 = 0.05 M
∆G ° 57 × 1000 [Cu 2+ ]1
⇒ log K sp = − =− = − 10
2.3 RT 2.3 × 8.314 × 298 1
42. At anode H2 → H+ + e− ; E° = 0
Now, E° of Ag+ + Cl − r AgCl 2
∆G ° 57000 At cathode AgCl (s) + e− → Ag + Cl − ; E° = ?
E° = − = = 0.59 V
nF 96500 1
(i) Cell reaction : H2 + AgCl (s) → Ag + H+ + Cl −
(b) The cell reaction is : 2
Zn + 2Ag+ r Zn 2+ + 2Ag; E ° = 1.56 V (ii) ∆G ° = − nE ° F = ∆H ° − T ∆S °
0.059 [Zn 2+ ] At 15°C : – 0.23 × 96500 = ∆H ° − 288 ∆S ° …(i)
⇒ 0 = E° − log
2 [Ag+ ] 2 At 35°C : – 0.21 × 96500 = ∆H ° − 308 ∆S ° …(ii)
[Zn 2+ ] 2E ° 2 × 1.56 ⇒ 96500 (0.23 − 0.21) = − 20 ∆S °
⇒ log = = = 52.88 96500 × 0.02
[Ag+ ] 2 0.059 0.059 ⇒ ∆S ° = − = − 96.5 J
20
6.539 × 10− 2
Moles of Zn added = = 10− 3 Substituting value of ∆S ° in (i)
65.39
∆H ° = 288 × (− 96.5) − 0.23 × 96500 = – 49.987 kJ
⇒ Moles of Ag formed = 2 × 10− 3.
176 Electrochemistry

(iii) At 25°C 47. For reaction,


− E ° × 96500 = − 49987 − 298 (− 96.5) 2Hg + 2Fe3+ r Hg2+
2 + 2Fe
2+

⇒ E° = 0.22 V Initial : 10 − 3 M 0 0
⇒ AgCl (s) + e → Ag + Cl − ; E° = 0.22 V

Equilibrium : 5 × 10 − 5 4.75 × 10 − 4 9.5 × 10 − 4
Ag → Ag+ + e− ; E° = – 0.80 V [Fe2+ ]2 [Hg2+
2 ]
K =
Adding : AgCl (s) → Ag+ + Cl − ; E° = – 0.58 V [Fe3+ ]2
⇒ E° = 0.0592 log K sp (9.5 × 10− 4 )2 (4.75 × 10− 4 )
= = 0.17
− 0.58 (5 × 10− 5 )2
⇒ log K sp = = − 9.79
0.0592 0.0592
⇒ K sp = 1.6 × 10−10 Q E° = log K
2
0.0592 [Ag + ]anode = − 0.0226 V
43. E = 0 − log
1 [Ag + ]cathode = E ° (Fe3+ / Fe2+ ) − E ° (Hg22+ / Hg)
[Ag+ ]anode ⇒ E° (Hg2+
2 / Hg) = 0.77 + 0.0226
⇒ 0.164 = − 0.0592 log
0.10 = 0.7926 V
⇒ [Ag+ ]anode = 1.7 × 10− 4 M
48. In a saturated AgI solution;
In saturated Ag2CrO4 solution present in anode chamber :
[Ag+ ] = 8.7 × 10− 17 M
Ag2CrO4 (s) r 2Ag+ + CrO24−
1.7 × 10 M 1.7 × 10− 4 M
−4 = 9.32 × 10− 9 M
1
2 ⇒ EAg +/ Ag = E ° − 0.0592 log
K sp = [Ag+ ]2 [CrO24− ] [Ag+ ]
1
 1.7  = 0.799 – 0.0592 log
= (1.7 × 10− 4 )2  × 10− 4 = 2.45 × 10− 12 9.32 × 10− 9
 2 
= 0.324 V
44. For 2 Fe3+ + 3I− r 2Fe2+ + I−3
E ° = E ° (Fe3+ / Fe2+ ) − E ° (I−3 / I− ) Also, for AgI r Ag+ + I− ; E° = 0.0592 log K sp
= 0.77 – 0.54 = 0.23 V = – 0.95 V
0.0592 Ag → Ag+ + e− ; E° = – 0.799 V
Q E° = log K (n = 2)
2
2E ° 2 × 0.23 AgI + e− → Ag + I− E° = x
log K = = = 7.77
0.0592 0.0592 Adding : AgI → Ag+ + I− ; E° = – 0.95 V
⇒ K = 5.89 × 107
= x – 0.799
45. Fe2+ + Ce4+ r Fe3+ + Ce3+ ⇒ x = − 0.151 V
4+ 3+ 3+ 2+
E° = E° (Ce / Ce ) – E° (Fe / Fe ) 49. The redox reaction is
= 1.44 – 0.68 = 0.76 V Zn + Ni 2+ r Zn 2+ + Ni E° = + 0.51 V
E° = 0.0592 log K 0.0592
Q ⇒ E° = log K
E° 0.76 2
⇒ log K = = = 12.83 0.51 × 2
0.0592 0.0592 ⇒ log K = = 17.23
⇒ K = 6.88 × 10 12 0.0592
17
⇒ K = 1.7 × 10
46. pH = 14 Such a high value of equilibrium constant indicates that the
⇒ pOH = 0 ⇒ [OH− ] = 1.0 M reaction is almost complete. Therefore, concentration of
K sp = 10− 19 = [Cu 2+ ][OH− ]2 Zn 2+ in solution will be equal to initial concentration of Ni 2+
ion, i.e. 1.0 M.
10− 19
⇒ [Cu 2+ ] = = 10− 19 50. The galvanic cell is : Cu | Cu 2+ || Ag + | Ag
[OH− ]2
Cell reaction : Cu + 2Ag+ r Cu 2+ + 2Ag; E° = 0.462 V
For reaction : Cu 2+ + 2e− → Cu; E° = 0.34 V 0.0592 (0.01)
E = 0 = 0.462 − log
0.0592 1 2 [Ag+ ]2
E = E° − log
2 [Cu 2+ ] ⇒ [Ag+ ] = 1.57 × 10− 9 M
0.0592
= 0.34 – log 1019 = – 0.222 V
2
Electrochemistry 177

1 (iii) Q Ecell = 1.113 V > 0


51. H2 O + e− r H2 + HO− ; E° = – 0.8277 V
2 ∆G = − nEF < 0
1
H2 + H2O r H3O+ + e− ; E° = 0 V Therefore, the cell reaction is spontaneous.
2
2H2O r H3O+ + HO− E° = – 0.8277 V 55. pH = Cα = 10−4
10−4
E° = 0.0592 log K ⇒ α=
− 0.8277 0.0015
⇒ log K = = − 13.98
0.0592  A
Also, conductance (G ) = κ  
⇒ − 14
K = 1.04 × 10 .  l
 l 120
52. At anode Zn → Zn 2+ + 2e− E° = 0.76 V ⇒ κ = G   = 5 × 10−7 ×
 A 1
At cathode 2H+ + 2e− → H2 (g ) E° = 0.00 V
+ 2+
= 6 × 10−5
⇒ For Zn + 2H → Zn + H2 (g ) E° = 0.76 V
κ × 1000
⇒ Λc =
0.0592 [Zn 2+ ] C
E = E° − log
2 [H+ ]2 6 × 10−5 × 1000
=
2 (E − E ° ) 1 0.0015
⇒ = − log [Zn 2+ ] − 2 log +
0.0592 [H ] ∞ Λc 6 × 10−5 × 1000 0.0015
⇒ Λ = = ×
⇒ − 16.2 = – log (0.1) – 2 pH α 0.0015 10−4
−1 −1
1 + 16.2 2
= 600 = 6 × 10 S cm mol
⇒ pH = = 8.6
2
56. Energy obtained as one mole X is converted into Y is 193 kJ.
53. For conductivity, the charge carriers are required. In
3s 3p 3d
anhydrous state, HCl is not ionised and no charge carrier H(1s1) Second excited
state of H-atom
ions are available, hence bad conductor. However, in aqueous Ground state
solution, HCl is fully ionised producing H+ and Cl − and
All are degenerate
degeneracy = 9
conducts electricity.
Energy consumed in converting one mole of M + to M 3+
54. (i) The cell reaction is 96500
= − nE ° F = 2 × 96500 × 0.25 J = J
Zn + Cu 2+ → Zn 2+ + Cu 2
 96500
° = Ecathode
(ii) Ecell ° ° = 0.350 – (– 0.763) = 1.113 V
− Eanode ⇒ 193 × 103 = n   ⇒ n = 4 mol
 2 
Q Both Zn 2+ and Cu 2+ are at unit concentrations,
E = E ° = 1.113 V
11
Chemical Kinetics
Objective Questions I (Only one correct option) 4. For the reaction of H 2 with I2 , the rate constant is
1. Consider the following reactions 2.5 × 10− 4 dm3 mol− 1 s− 1 at 327ºC and 10. dm 3 mol− 1 s− 1
A → P1; B → P2; C → P3; D → P4, at 527ºC. The activation energy for the reaction, in
The order of the above reactions are a , b , c and d, kJ mol− 1 is (R = 8.314 JK − 1 mol− 1 ) (2019 Main, 10 April II)
respectively. The following graph is obtained when log[rate] (a) 59 (b) 72
vs log[conc.] are plotted: (c) 150 (d) 166
5. A bacterial infection in an internal wound grows as
[D]
[B] N ′ ( t ) = N0 exp ( t ), where the time t is in hours. A dose of
[A] antibiotic, taken orally, needs 1 hour to reach the wound.
Once it reaches there, the bacterial population goes down as
log [rate]

[C] dN N
= − 5N 2 . What will be the plot of 0 vs t after 1 hour ?
dt N
(2019 Main, 10 April I)

log [conc.] N0
N0 (b)
(a) N
N
Among the following, the correct sequence for the order of
the reactions is (2020 Main, 6 Sep I) t(h) t(h)
(a) D > A > B > C (b) A > B > C > D
(c) C > A > B > D (d) D > B > A > C N0 N0
(c) (d)
N N
2. NO2 required for a reaction is produced by the
decomposition of N2 O5 in CCl 4 as per the equation, t(h) t(h)
2N2 O5 ( g ) → 4NO2 ( g ) + O2 ( g )
6. Consider the given plot of enthalpy of the following reaction
The initial concentration of N2 O5 is 3.00 mol L−1 and it is between A and B. A + B → C+D
−1
2.75 mol L after 30 minutes. The rate of formation of NO2 Identify the incorrect statement. (2019 Main, 9 April II)
is (2019 Main, 12 April II)
(a) 4167
. × 10−3 mol L −1 min −1 20
−2 −1 −1
(b) 1667
. × 10 mol L min Enthalpy 15
(c) 8.333 × 10−3 mol L −1 min −1 (kJ mol–1)10
D
(d) 2.083 × 10−3 mol L −1 min −1
5
A +B
3. In the following reaction; xA → yB C
Reaction
 d [ A ]  d [ B ] coordinate
log 10 −  = log 10  + 0.3010
 dt   dt 
(a) D is kinetically stable product.
A and B respectively can be (2019 Main, 12 April I) (b) Formation of A and B from C has highest enthalpy of activation.
(a) n-butane and iso-butane (c) C is the thermodynamically stable product.
(b) C2H2 and C6 H6
(c) C2H4 and C4 H8 (d) Activation enthalpy to form C is 5 kJ mol −1 less than that to
(d) N2O4 and NO2 form D.
Chemical Kinetics 179

7. The given plots represent the variation of the concentration of 13. If a reaction follows the Arrhenius equation, the plot lnk vs
a reaction R with time for two different reactions (i) and (ii). 1/(RT) gives straight line with a gradient (− y) unit.
The respective orders of the reactions are The energy required to activate the reactant is
(2019 Main, 9 April I) (2019 Main, 11 Jan I)
y
(i) (ii) (a) unit (b) − y unit (c) yR unit (d) y unit
R
In [R] [R] 14. For an elementary chemical reaction,
k1

time time
A2 = 2A , the expression for d[dtA ] is
k −1
(2019 Main, 10 Shift II)
(a) 1, 1 (b) 0, 2 (a) 2k1[ A2 ] − k−1[ A ]2 (b) k1[ A2 ] − k−1 [ A ]2
(c) 0, 1 (d) 1, 0
(c) 2k1[ A2 ] − 2k−1 [ A ]2 (d) k1[ A2 ] + k−1[ A ]2
k1 k2
8. For a reaction scheme, A → B → C , if the rate of 15. Consider the given plots for a reaction obeying Arrhenius
formation of B is set to be zero then the concentration of B is equation (0°C < T < 300°C) : ( k and E a are rate constant and
given by (2019 Main, 8 April II) activation energy, respectively) (2019 Main, 10 Jan I)
k 
(a) k1k2[ A ] (b)  1 [ A ]
 k2 
(c) (k1 − k2 )[ A ] (d) (k1 + k2 )[ A ] k k
9. For the reaction, 2 A + B → C , the values of initial rate at
different reactant concentrations are given in the table below.
Ea T(°C)
The rate law for the reaction is (2019 Main, 8 April I)
I II

[A](mol L−1 ) [B](mol L−1 ) Initial rate Choose the correct option.
(mol L−1s−1 ) (a) Both I and II are wrong
0.05 0.05 0.045
(b) Both I and II are correct
(c) I is wrong but II is right
0.10 0.05 0.090
(d) I is right but II is wrong
0.20 0.10 0.72
16. For the reaction, 2A + B → products
(a) rate = k [A ][B ] 2
(b) rate = k [A ] [B ] 2 2
When concentration of both (A and B) becomes double, then
(c) rate = k [A ][B ] (d) rate = k [A ]2[B ]
rate of reaction increases from 0.3 mol L −1 s −1 to
10. For a reaction, consider the plot of ln k versus1/ T given in the 2.4 mol L −1 s −1 .
figure. If the rate constant of this reaction at 400 K is When concentration of only A is doubled, the rate of reaction
10− 5 s − 1 , then the rate constant at 500 K is increases from 0.3 mol L −1 s −1 to 0.6 mol L −1 s −1 .
(2019 Main, 12 Jan II)
Which of the following is true? (2019 Main, 9 Jan II)
(a) The whole reaction is of 4th order
(b) The order of reaction w.r.t. B is one
Slope = –4606 (c) The order of reaction w.r.t. B is 2
ln k
(d) The order of reaction w.r.t. A is 2
17. The following results were obtained during kinetic studies of
1/ T the reaction; (2019 Main, 9 Jan I)

(a) 4 × 10− 4 s− 1 (b) 10− 6 s− 1 2A + B → Products


−4 −1 −4 −1
(c) 10 s (d) 2 × 10 s Initial rate of
[A] [B]
Experiment reaction
11. Decomposition of X exhibits a rate constant of 0.05 µg/year. (in mol L−1 ) (in mol L−1)
(in mol L−1 min −1)
How many years are required for the decomposition of 5 µg
I. 0.10 0.20 6.93 × 10−3
of X into 2.5 µg? (2019 Main, 12 Jan I)
(a) 20 (b) 25 (c) 40 (d) 50 II. 0.10 0.25 6.93 × 10−3
12. The reaction, 2 X → B is a zeroth order reaction. If the initial III. 0.20 0.30 1386
. × 10−2
concentration of X is 0.2 M, the half-life is 6 h. When the
initial concentration of X is 0.5 M, the time required to reach The time (in minutes) required to consume half of A is
(a) 5 (b) 10
its final concentration of 0.2 M will be (2019 Main, 11 Jan II)
(c) 100 (d) 1
(a) 7.2 h (b) 18.0 h (c) 12.0 h (d) 9.0 h
180 Chemical Kinetics

18. Which of the following lines correctly show the temperature The rate law for the formation of C is
dependence of equilibrium constant, K , for an exothermic dC dC
(a) = k[ A ][ B ] (b) = k[ A ] 2[ B ]
reaction? (2018 Main) dt dt
dC dC
In K A (c) = k[ A ][ B ] 2 (d) = k[ A ]
B dt dt
1
(0, 0) T(K) 25. In the reaction, P + Q → R + S , the time taken for
××
××
75% reaction of P is twice the time taken for 50% reaction
××
××
C of P. The concentration of Q varies with reaction time as
××
××
D shown in the figure. The overall order of the reaction is
(2013 Adv.)
(a) A and B (b) B and C
(c) C and D (d) A and D [Q]0
19. At 518°C, the rate of decomposition of a sample of gaseous
acetaldehyde, initially at a pressure of 363 Torr, was 1.00 Torr [Q]
s −1 when 5% had reacted and 0.5 Torr s −1 when 33% had
Time
reacted. The order of the reaction is : (2018 Main)
(a) 2 (b) 3 (a) 2 (b) 3 (c) 0 (d) 1
(c) 1 (d) 0 26. The rate of a reaction doubles when its temperature
20. Two reactions R1 and R2 have identical pre- exponential changes from 300 K to 310 K. Activation energy of such a
factors. Activation energy of R1 exceeds that of R2 by 10 kJ reaction will be ( R = 8.314 JK −1 mol −1 and log 2 = 0.301)
mol− 1 . If k1 and k2 are rate constants for reactions R1 and R2 , (a) 53.6 kJ mol −1 (b) 48.6 kJ mol −1 (2013 Main)
k  (c) 58.5 kJ mol −1 (d) 60.5 kJ mol −1
respectively at 300 K, then ln  2  is equal to
 k1  27. Plots showing the variation of the rate constant ( k ) with
( R = 8.314 J mol − 1K − 1 ) (2017 Main)
temperature (T ) are given below. The plot that follows
(a) 8 (b) 12 (c) 6 (d) 4
Arrhenius equation is (2010)
21. Decomposition of H2 O2 follows a first order reaction. In 50
min, the concentration of H2 O2 decreases from 0.5 to 0.125 M
k k
in one such decomposition. When the concentration of H2 O2 (a) (b)
reaches 0.05 M, the rate of formation of O2 will be (2016 Main)
(a) 6.93 × 10−4 mol min −1 (b) 2.66 L min −1 at STP
(c) 1.34 × 10−2 mol min −1 (d) 6.93 × 10−2 mol min −1 T T

22. Higher order (>3) reactions are rare due to (2015 Main)
(a) low probability of simultaneous collision of all the reacting k k
(c) (d)
species
(b) increase in entropy and activation energy as more molecules are
involved
T T
(c) shifting of equilibrium towards reactants due to elastic collisions
(d) loss of active species on collision 28. For a first order reaction, A → P, the temperature (T )
23. For the elementary reaction, M → N , the rate of dependent rate constant ( k ) was found to follow the
disappearance of M increases by a factor of 8 upon doubling the equation :
concentration of M . The order of the reaction with respect to M 2000
log k = + 6.0
is (2014 Adv.) T
(a) 4 (b) 3
the pre-exponential factor A and the activation energy Ea ,
(c) 2 (d) 1
respectively, are (2009)
24. For the non-stoichiometric reaction, 2 A + B → C + D, the
(a) 1.0 × 106 s− 1 and 9.2 kJ mol − 1
following kinetic data were obtained in three separate
experiments, all at 298 K. (2014 Main) (b) 6.0 s− 1 and 16.6 kJ mol − 1
(c) 1.0 × 106 s− 1 and 16.6 kJ mol − 1
Initial Initial Initial rate of
concentration concentration formation of C (d) 1.0 × 106 s− 1 and 38.3 kJ mol − 1
[A] [B] (mol L −1s −1)
29. Under the same reaction conditions, initial concentration
(i) 0.1 M 0.1 M 1. 2 × 10−3
of 1.386 mol dm −3 of a substance becomes half in 40 s and
−3
(ii) 0.1 M 0.2 M 1. 2 × 10 20 s through first order and zero order kinetics
(iii) 0.2 M 0.1 M 2. 4 × 10−3
Chemical Kinetics 181

k  36. The rate constant for the reaction, 2N2 O5 → 4NO2 + O2 is


respectively. Ratio  1  of the rate constants for first order
 k0  3.0 × 10−5 s −1 . If the rate is 2.40 × 10−5 mol L − 1 s − 1 , then the
( k1 ) and zero order ( k0 ) of the reaction is concentration of N2 O5 (in mol L − 1 ) is (2000, 1M)
(2008, 3M)
−1 3 −3 (a) 1.4 (b) 1.2
(a) 0.5 mol dm (b) 1.0 mol dm
(c) 0.04 (d) 0.8
(c) 1.5 mol dm −3 (d) 2.0 mol −1 dm 3
37. The half-life period of a radioactive element is 140 days.
30. Consider a aG + bH → products. When
reaction,
After 650 days, one gram of the element will reduce to
concentration of both the reactants G and H is doubled, the (1986)
rate increases by eight times. However, when concentration 1 1 1 1
(a) g (b) g (c) g (d) g
of G is doubled keeping the concentration of H fixed, the rate 2 4 8 16
is doubled. The overall order of the reaction is (2007, 3M) 38. A catalyst is a substance which (1983, 1M)
(a) 0 (b) 1 (a) increases the equilibrium concentration of the product
(c) 2 (d) 3 (b) changes the equilibrium constant of the reaction
31. Which one of the following statement(s) is incorrect about (c) shortens the time to reach equilibrium
order of reaction? (2005, 1M)
(d) supplies energy to the reaction
(a) Order of reaction is determined experimentally 39. The specific rate constant of a first order reaction depends on
(b) Order of reaction is equal to sum of the power of concentration the (1983, 1M)
terms in differential rate law (a) concentration of the reactant
(c) It is not affected with stoichiometric coefficient of the reactants (b) concentration of the product
(c) time
(d) Order cannot be fractional
(d) temperature
32. (A) follows first order reaction, ( A ) → product. Concentration 40. The rate constant of a reaction depends on (1981, 1M)
of A, changes from 0.1 M to 0.025 M in 40 min. Find the rate (a) temperature
of reaction of A when concentration of A is 0.01 M. (b) initial concentration of the reactants
(2004, 1M) (c) time of reaction
(a) 3.47 × 10–4 M min –1 (b) 3.47 × 10−5 M min –1 (d) extent of reaction
(c) 1.73 × 10−4 M min –1 (d) 1.73 × 10−5 M min –1
Objective Questions II
33. In a first order reaction the concentration of reactant (One or more than one correct option)
decreases from 800 mol/dm3 to 50 mol/dm3 in 2 × 104 s. The
41. Which of the following plots is(are) correct for the given
rate constant of reaction ins −1 is (2003, 1M)
reaction?
(a) 2 × 104 (b) 3.45 × 10−5
([ P ]0 is the initial concentration of P) (2020 Adv.)
(c) 1.386 × 10−4 (d) 2 × 10−4
CH3 CH3
34. Consider the chemical reaction, H3C Br + NaOH H3C OH + NaBr
N2 ( g ) + 3H2 ( g ) → 2NH3 ( g )
CH3 CH3
The rate of this reaction can be expressed in terms of time
(P) ( Q)
derivatives of concentration of N2 ( g ), H2 ( g ) or NH3 ( g ).
Identify the correct relationship amongst the rate expressions
d [N2 ] 1 d [H2 ] 1 d[NH3 ]
Initial rate

(a) Rate = − =− =
dt 3 dt 2 dt (2002, 3M) (a) t (b)
1/2
d [N2 ] d [H2 ] d [NH3 ]
(b) Rate = − = −3 =2
dt dt dt
d [N2 ] 1 d [H2 ] 1 d [NH3 ] [P]0 [P]0
(c) Rate = = =
dt 3 dt 2 dt
d [N2 ] d [H2 ] d [NH3 ]
(d) Rate = − =− =
dt dt dt (c) [Q] (d) In [P]
[P]0 [P]0
35. If I is the intensity of absorbed light and C is the concentration
of AB for the photochemical process.
Time
AB + hν → AB* , the rate of formation of AB* is directly Time

proportional to (2001, 1M) 42. For a first order reaction, A( g ) → 2B( g )+ C ( g ) at


(a) C (b) I constant volume and 300 K, the total pressure at the beginning
(c) I 2 (d) C ⋅ I ( t = 0 ) and at time t are p0 and pt , respectively.
182 Chemical Kinetics

Initially, only A is present with concentration [ A ]0 , and t1 / 3 is 47. For the first order reaction, (1998, 2M)
the time required for the partial pressure of A to reach 1/3 rd of (a) the degree of dissociation is equal to (1 − e− kt )
its initial value. The correct option(s) is (are) (Assume that all (b) a plot of reciprocal concentration of the reactant vs time gives a
these gases behave as ideal gases) (2018 Adv.) straight line
(c) the time taken for the completion of 75% reaction is thrice the
1
of the reaction
In(3p0–pt)

2
(a) (b) t1/3 (d) the pre-exponential factor in the Arrhenius equation has the
dimension of time, T −1

Time [A ] 0
48. A catalyst (1984, 1M)
(a) increases the average kinetic energy of reacting molecules
(b) decreases the activation energy
Rate constant
In(p0–pt)

(c) alters the reaction mechanism


(c) (d)
(d) increases the frequency of collisions of reacting species

Numerical Answer Type Questions


Time [A ] 0 49 If 75% of a first order reaction was completed in 90 minutes,
60% of the same reaction would be completed in
43. In a bimolecular reaction, the steric factor P was approximately (in minutes) ........... .
experimentally determined to be 4.5. the correct option(s)
among the following is(are) (2017 Adv.)
(Take : log 2 = 0.30; log 2.5 = 0.40) (2020 Main, 4 Sep I)

(a) The activation energy of the reaction is unaffected by the value 50. Consider the kinetic data given in the following table for the
of the steric factor reaction A + B + C → Product
(b) Experimentally determined value of frequency factor is higher
than that predicted by Arrhenius equation Experiment [A] [B] [C] Rate of
(c) The value of frequency factor predicted by Arrhenius equation No. (mol dm − 3 ) (mol dm − 3 ) (mol dm − 3 ) reaction
is higher than that determined experimentally (mol dm − 3s− 1 )
(d) Since P = 4.5, the reaction will not proceed unless an effective 1 0.2 0.1 0.1 6.0 × 10− 5
catalyst is used
2 0.2 0.2 0.1 6.0 × 10− 5
44. According to the Arrhenius equation, (2016 Adv.)
3 0.2 0.1 0.2 . × 10− 4
12
(a) a high activation energy usually implies a fast reaction
(b) rate constant increases with increase in temperature. This is due 4 0.3 0.1 0.1 9.0 × 10− 5
to a greater number of collisions whose energy exceeds the
activation energy . mol dm − 3 ,
The rate of the reaction for [ A ] = 015
(c) higher the magnitude of activation energy, stronger is the [ B ] = 0. 25 mol dm − 3 and [C ] = 015
. mol dm − 3 is found to
temperature dependence of the rate constant
be Y × 10− 5 mol dm − 3s − 1 . The value of Y is ............
(d) the pre-exponential factor is a measure of the rate at which
(2019 Adv.)
collisions occur, irrespective of their energy
51. The decomposition reaction
45. For the first order reaction,

2N2 O5 ( g ) → 4NO2 ( g ) + O2 ( g ) (2011) 2N 2O 5 ( g ) → 2N 2O 4 ( g ) + O 2 ( g ) is started in a closed
(a) the concentration of the reactant decreases exponentially with time cylinder under isothermal isochoric condition at an initial
(b) the half-life of the reaction decreases with increasing temperature pressure of 1 atm. After Y × 103 s, the pressure inside the
(c) the half-life of the reaction depends on the initial concentration cylinder is found to be 1.45 atm. If the rate constant of the
of the reactant reaction is 5 × 10−4 s −1 , assuming ideal gas behaviour, the
(d) the reaction proceeds of 99.6% completion in eight half-life
value of Y is ……… (2019 Adv.)
duration
52. Consider the following reversible reaction,
46. The following statement (s) is are correct (1999, 3M)
(a) A plot of log K p vs
1
is linear
A( g )+ B( g ) - AB( g )
T The activation energy of the backward reaction exceeds that
(b) A plot of log [X] vs time is linear for a first order reaction, of the forward reaction by 2RT (in J mol −1 ). If the
x→ p pre-exponential factor of the forward reaction is 4 times that
1
(c) A plot of log p vs is linear at constant volume of the reverse reaction, the absolute value of ∆Gs (in J mol −1 )
T for the reaction at 300 K is ……… .
1
(d) A plot of p vs is linear at constant temperature (Given ; ln( 2 ) = 0.7 RT = 2500 J mol −1 at 300 K and G is the
V
Gibbs energy) (2018 Adv.)
Chemical Kinetics 183

Passage Based Questions 57. For the reaction : N2 ( g ) + 3H2 ( g ) → 2NH3 ( g )


Carbon-14 is used to determine the age of organic material. The Under certain conditions of temperature and partial pressure of
procedure is based on the formation of 14 Cby neutron capture in the the reactants, the rate of formation of NH3 is 0.001 kg/h−1. The
upper atmosphere. rate of conversion of H2 under the same condition is ....
14
7 N + 0 n1 → 14
6 C + 1p
1
kg /h −1 . (1994, 1M)
14
C is absorbed by living organisms during photosynthesis. The 14 C 58. The hydrolysis of ethyl acetate in ........... medium is a ..........
content is constant in living organism once the plant or animal dies, order reaction. (1986, 1M)
the uptake of carbon dioxide by it ceases and the level of 14 C in the
59. The rate of chemical change is directly proportional to
dead being, falls due to the decay which C-14 underoges
14 14 − ............. (1985, 1M)
6 C → 7 N + β

The half-life period of 14 C is 5770 yr. True/False


The decay constant ( λ ) can be calculated by using the following
0.693 60. For a first order reaction, the rate of the reaction doubles as
formula λ = . the concentration of the reaction (s) doubles. (1986, 1M)
t1 / 2
The comparison of the β − activity of the dead matter with that of the Integer Answer Type Questions
carbon still in circulation enables measurement of the period of the
isolation of the material from the living cycle. The method however, 61. An organic compound undergoes first order decomposition.
ceases to be accurate over periods longer than 30,000 yr. The The time taken for its decomposition to 1/8 and 1/10 of its
proportion of 14 C to 12 C in living matter is 1 : 1012. initial concentration are t1 / 8 and t1 / 10 respectively. What is the
[t ]
(2006, 3 × 4M = 12M) value of 1 / 8 × 10 ? (log 10 2 = 0.3)
[ t1 / 10 ] (2012)
53. Which of the following option is correct?
(a) In living organisms, circulation of 14 C from atmosphere is high 62. The concentration of R in the reaction R → P was
so the carbon content is constant in organism measured as a function of time and the following data is
(b) Carbon dating can be used to find out the age of earth crust and obtained :
rocks [ R ] (molar) 1.0 0.75 0.40 0.10
(c) Radioactive absorption due to cosmic radiation is equal to the
t (min) 0.0 0.05 0.12 0.18
rate of radioactive decay, hence the carbons content remains
constant in living organisms The order of the reaction is (2010)
(d) Carbon dating cannot be used to determine concentration of
14
C in dead beings Subjective Questions
54. What should be the age of fossil for meaningful 63. 2 X ( g ) → 3Y ( g ) + 2 Z ( g )
determination of its age? 0 100 200
Time
(a) 6 yr
(in min)
(b) 6000 yr
(c) 60,000 yr Partial pressure of 800 400 200
(d) It can be used to calculate any age X (in mm of Hg)
55. A nuclear explosion has taken place leading to increase in Assuming ideal gas condition. Calculate
concentration of C14 in nearby areas. C14 concentration is C1 (a) order of reaction
in nearby areas and C 2 in areas far away. If the age of the fossil (b) rate constant
is determined to be T1 and T2 at the places respectively then (c) time taken for 75% completion of reaction
(a) the age of fossil will increase at the place where explosion has (d) total pressure when px = 700 mm (2005, 4M)
1 C
taken place and T1 − T2 = ln 1
λ C2 64. For the given reaction, A + B → Products
(b) the age of fossil will decrease at the place where explosion has Following data are given
1 C
taken place and T1 − T2 = ln 1 Initial conc. Initial conc. Initial rate
λ C2
(m/L) (m/L) [mL–1s –1 ]
(c) the age of fossil will be determined to be the same [ A ]0 [ B ]0
T C
(d) 1 = 1 0.1 0.1 0.05
T2 C 2
0.2 0.1 0.1
Fill in the Blanks 0.1 0.2 0.05
(− E a / RT )
56. In Arrhenius equation, k = A exp . A may be termed
(a) Write the rate equation.
as the rate constant at ......... (1997, 1M)
(b) Calculate the rate constant. (2004, 2M)
184 Chemical Kinetics

65. Cu (half-life = 12.8 h) decays by β emission (38%), β +


64
(i) the order of the reaction with respect to A and with respect
emission (19%) and electron capture (43%). Write the decay to B.
products and calculate partial half-lives for each of the decay (ii) the rate constant at 300 K.
processes. (2002) (iii) the pre-exponential factor. (1994, 5M)
66. The rate of first order reaction is 0.04 mol L–1s –1 at10 min and 74. The gas phase decomposition of dimethyl ether follows first
0.03 mol L–1s –1 at 20 min after initiation. Find the half-life of the order kinetics
reaction. (2001, 5M) CH3 — O— CH3 ( g ) → CH4 ( g ) + H2 ( g ) + CO ( g )
67. A hydrogenation reaction is carried out at 500 K .If the same The reaction is carried out in a constant volume container at
reaction is carried out in the presence of a catalyst at the same 500° C and has a half-life of 14.5 min. Initially only dimethyl
rate, the temperature required is 400 K. Calculate the ether is present at a pressure of 0.40 atm. What is the total
activation energy of the reaction if the catalyst lowers the pressure of the system after 12 min? Assume ideal gas
activation barrier by 20 kJ mol −1 . behaviour. (1993, 4M)
(2000, 3M)
75. A first order reaction, A → B, requires activation energy of
68. The rate constant for an isomerisation reaction, A → B is 70 kJ mol −1 . When a 20% solution of A was kept at 25° C for
4.5 × 10−3 min. If the initial concentration of A is 1 M, 20 min, 25% decomposition took place. What will be the
calculate the rate of the reaction after 1 h. (1999, 4M) percentage decomposition in the same time in a 30% solution
maintained at 40°C ? Assume that activation energy remains
69. (i) The rate constant of a reaction is 1.5 × 107 s−1 at 50° C and
constant in this range of temperature. (1993, 4M)
4.5 × 107 s−1 at 100° C. Evaluate the Arrhenius parameters
76. Two reactions (i) A → products (ii) B → products, follow
A and Ea . (1998, 5M)
1 first order kinetics.The rate of the reaction (i) is doubled when
(ii) For the reaction, N2O5 (g ) → 2NO2 (g ) + O2 (g ), the temperature is raised from 300 K to 310 K. The half-life
2
calculate the mole fraction N2O5 (g ) decomposed at a for this reaction at 310 K is 30 min. At the same temperature B
constant volume and temperature, if the initial pressure is decomposes twice as fast as A. If the energy of activation for
600 mm Hg and the pressure at any time is 960 mm Hg. the reaction (ii) is half that of reaction (i), calculate the rate
Assume ideal gas behaviour. constant of the reaction (ii) at 300 K. (1992, 3M)
3 1
70. The rate constant for the first order decomposition of a certain 77. The nucleidic ratio, 1H to 1H in a sample of water is
− 18
reaction is described by the equation 8.0 × 10 : 1. Tritium undergoes decay with a half-life
1.25 × 104 K period of 12.3 yr. How many tritium atoms would 10.0 g of
log k (s −1 ) = 14.34 −
T such a sample contain 40 yr after the original sample is
(i) What is the energy of activation for the reaction? collected. (1992, 4M)
(ii) At what temperature will its half-life period be 256 min? 78. The decomposition of N2 O5 according to the equation,
(1997, 5M)
2N2 O5 ( g ) → 4NO2 ( g ) + O2 ( g )
71. One of the hazards of nuclear explosion is the generation of is a first order reaction. After 30 min from the start of the
Sr 90 and its subsequent incorporation in bones. This nucleide decomposition in a closed vessel, the total pressure developed
has a half-life of 28.1 yr. Suppose one microgram was is found to be 284.5 mm of Hg. On complete decomposition,
absorbed by a new-born child, how much Sr 90 will remain in the total pressure is 584.5 mm of Hg. Calculate the rate
his bones after 20 yr. (1995, 2M) constant of the reaction. (1991, 6M)
72. At 380° C, the half-life period for the first order 79. In Arrhenius equation for a certain reaction, the value of A and
decomposition of H2 O2 is 360 min. The energy of activation Ea (activation energy) are 4 × 1013 s −1 and 98.6 kJ mol –1
of the reaction is 200 kJ mol −1 . Calculate the time required for respectively. If the reaction is of first order, at what
75% decomposition at 450° C. (1995, 4M) temperature will its half-life period be 10 min? (1990, 3M)

73. From the following data for the reaction between A and B 80. An experiment requires minimum beta activity produced at
the rate of 346 beta particles per minute. The half-life period
[A], (mol/L) [B], (mol/L) Initial rate (mol L–1s–1) at of 42 Mo 99 , which is a beta emitter, is 66.6 h. Find the
300 K 320 K minimum amount of 42 Mo 99 required to carry out the
experiment in 6.909 h. (1989, 5M)
2. 5 × 10−4 3. 0 × 10−5 5. 0 × 10−4 2.0 × 10−3 −6
81. A first order gas reaction has k = 1.5 × 10 per second at
5.0 × 10−4 6.0 × 10−5 4.0 × 10−3 — 200° C. If the reaction is allowed to run for 10 h, what
percentage of the initial concentration would have change in
1. 0 × 10−3 6. 0 × 10−5 1. 6 × 10−2 — the product? What is the half-life of this reaction? (1987, 5M)

Calculate
Chemical Kinetics 185

82. While studying the decomposition of gaseous N2 O5 , it is 85. Rate of reaction, A + B → products is given below as a
observed that a plot of logarithm of its partial pressure versus function of different initial concentrations of A and B
time is linear. What kinetic parameters can be obtained from
[A] mol/L [B] (mol/L) Initial rate
this observation? (1985, 2M)
(mol L–1 min –1 )
83. Radioactive decay is a first order process. Radioactive carbon
0.01 0.01 0.005
in wood sample decays with a half-life of 5770 yr. What is the
rate constant (in yr − 1 ) for the decay? What fraction would 0.02 0.01 0.010
remain after 11540 yr? (1984, 3M) 0.01 0.02 0.005
84. A first order reaction is 20% complete in 10 min. Calculate (i) Determine the order of the reaction with respect to A and B.
the specific rate constant of the reaction, and What is the half-life of A in the reaction ? (1982, 4M)
(ii) the time taken for the reaction to go to 75% completion.
(1983, 2M)

Answers
1. (d) 2. (b) 3. (c) 4. (d) 49. (60) 50. (6.75) 51. (2.3) 52. (+8500J/ mol)
5. (a) 6. (d) 7. (d) 8. (b) 53. (c) 54. (b) 55. (a) 56. T = ∞
9. (a) 10. (c) 11. (d) 12. (b) 57. 0.0015 60. T 61. (9) 62. (0)
13. (d) 14. (c) 15. (b) 16. (c) 63. (950 mm Hg) 66. (25 min) 67. (100 kJ mol −1)
17. (b) 18. (a) 19. (a) 20. (d) 68. (3.26 × 10 −3 mol L −1 min −1) 71. (6.1 × 10 −7g) 72.
21. (a) 22. (a) 23. (b) 24. (d) (20.74 min)
25. (d) 26. (a) 27. (a) 28. (d) 74. (0.75 atm) 75. (67 %) 76. (3.26 × 10 −2 min −1
)
−3 −1
29. (a) 30. (d) 31. (d) 32. (a) 77. (5.6 × 10 ) 5
78. (5.2 × 10 min )
33. (c) 34. (a) 35. (d) 36. (d) 79. (311.34 K) 80. (3.56 × 10 −16 g) 81.
37. (d) 38. (c) 39. (d) 40. (a) (128.33 h)
41. (a) 42. (a,d) 43. (a,c) 44. (b,c,d) 82. (128.33 h) 83. (0.25) 85. (1.386 min)
45. (a,b,d) 46. (a,b,d) 47. (a,d) 48. (b,c)

Hints & Solutions


1. We know that, Given, [N2O5 ]initial = 3.00 mol L− 1
Rate = k (concentration) n
After 30 min, [N2O5 ] = 2 .75 mol L− 1
[n = order of reaction and k = rate constant]
2N2O5 (g ) → 4NO2 (g ) + O2 (g )
Taking log both side, t= 0 3.0 M
log (Rate) = log k + n log [concentration] t = 30 2.75 M
Slope of graph is the order of reaction greater the slope, greater is From the equation, it can be concluded that
the order of reaction.
1 − ∆[N2O5 ] 1 ∆[NO2 ]
∴Correct sequence for the order of reaction is D > B > A > C. × = ×
2 ∆t 4 ∆t
Hence, the correct option is (d).
− ∆[N2O5 ] − (2.75 − 3.00) mol L− 1 0.25
2. = = ⇒
Key Idea The rate of a chemical reaction means the speed ∆t 30 30
with which the reaction takes place. ∆[NO2 ] ∆ (N2O5 ) ∆[NO2 ] 0.25
and =−2 ⇒ =− 2 ×
For R → P ∆t ∆t ∆t 30
Rate of disappearance of R = − 1667
. × 10− 2 mol L− 1 min − 1
Decrease in conc.of R ∆[ R]
= =− 3. In the given reaction; x A → y B
Time taken ∆t
Rate of appearance of P  − d[ A ]  d[ B ]
log10 = log10 + 0.3010
Increase in conc. of P ∆[ P]  dt   dt 
= =+
Time taken ∆t Value of log 2 = 0.3010
186 Chemical Kinetics

Substituting 0.3010 by log2 On comparing the above equation with equation of straight line,
 d[ A ]  d[ B ] y = mx + c
log10 − = log10 + log 2 1
 dt   dt  We get m = 5N 0, c = − 5N 0
e
Using logarithm rules,
N0
∴ Plot of vs t is shown aside.
 − d[ A ]  d[ B ] 1  d [ A ]  d[ B ] N
= 2× ⇒− = …(i)
 dt   dt  2  dt   dt 
Using the rate equation (i) to determine the reaction involved is
2A → B
Option that fits correct in the above reaction is (c). N0

2C2H4 → C4H8 . N

4. t(h)
Key Idea The Arrhenius equation for rate constants at two
different temperatures is 6. Only statement (d) is incorrect. Corrected statement is
k Ea T2 − T1  ‘‘Activation enthalpy to form C is 15 kg mol −1 more than 5 kg
log 2 = [where, T2 > T1]
k 1 2.303R  T1T2  mol −1 that is required to form D.’’ It can be easily explained by
where, k 1 and k 2 are rate constants at temperatures T1 and T2, following graph.
respectively.
R = Gas constant, Ea = Activation energy 20
15
For the reaction, H2 + I2 → 2HI Enthalpy Activation
Given k 1 = 2.5 × 10−4 dm3mol−1s −1 (kJ mol–1) 10 D enthalpy
5
T1 = (273 + 327) K = 600 K A+B
k 2 = 1 dm 3mol −1 s −1 at T2 = (273 + 527) K = 800 K Reaction C
k2 Ea  T2 − T1  coordinate
Now, log =  
k 1 2.303R  T1T2  Activation enthalpy (or energy) is the extra energy required by
the reactant molecules that result into effective collision between
1 Ea  800 − 600
⇒ log =   them to form the products.
2.5 × 10−4 2.303 × 8.314 × 10−3  600 × 800
7. In first order reaction, the rate expression depends on the
(10 × 103 ) E 200 concentration of one species only having power equal to unity.
⇒ log = a ×
2.5 0.019 48 × 104 nR → products
⇒ log 4 + 3 log10 −~ E × 0.022 − d[ R ]
= k [R ]
a
2 × log 2 + 3 3.6 ~
⇒ Ea = = – 163.6kJ mol −1 dt
0.022 0.022 On integration, − ln[ R ] = kt − ln[ R0 ]
5. The expression for bacterial growth is or ln(R ) = ln (R0 ) − kt
N = N 0et y = c + mx
N0 m = slope = −k (negative)
= e− t
N c = intercept = ln (r0 )
From 0 to 1 hour N ′ (t ) = N 0et The graph for first order reactions is
dN
From 1 hour onwards, = −5N 2
dt
On differentiating the above equation from N ′ to N we get. ln (R)
N t
∫eN N −2dN = −5 ∫ dt [QAt 1 hour, N ′ = eN 0]
0 1

1 1 
 N − eN  = 5(t − 1) t
 0
In zero order reaction,
Multiply both sides by N 0, we get [ R ] → product
N0 1 N 1
− = 5N 0 (t − 1) or, 0 = 5N 0 (t − 1) + − d[ R ]t
N e N e ∴ = k or − d[ R ]t = kdt
dt
N0 1 
= 5N 0t + − 5N 0 On integrating, −[ R ]t = kt + c
N  e 
If t = 0,[ R ]t = [ R ]0
Chemical Kinetics 187

∴ −[ R ]t = kt − [ R ]0 10. The temperature dependence of a chemical reaction is expressed


[ R ]t = [ R ]0 − kt by Arrhenius equation,
Thus, the graph plotted between [ r ]t and t gives a straight line k = Ae− E a$ / RT …(i)
with negative slope (−k ) and intercept equal to [ R ]0. Taking natural logarithm on both sides, the Arrhenius equation
The graph for zero order reaction is becomes,
E
ln k = ln A − a
RT
E
[R]
where, − a is the slope of the plot and ln A gives the intercept.
R
Eq. (i) at two different temperatures for a reaction becomes,
k E 1 1
ln 2 = a  −  …(ii)
t k1 R  T1 T2 
K1 K2 ⇒ In the given problem,
8. A → B → C
T1 = 400K, T2 = 500 K
Rate of formation of B is
k 1 = 10− 5 s − 1, k 2 = ?
d[ B ]
= k 1[ A ] − k 2[ B ] E
dt − a (Slope) = − 4606
R
 d[ B ] 
⇒ 0 = k 1[ A ] − k 2[ B ] Q Given, =0 On substituting all the given values in Eq. (ii), we get
 dt 
k  1 1 
⇒ k 2[ B ] = k 1[ A ] ln −2 5 = 4606  − 
10  400 500
k1
⇒ Concentration of B, [ B ] = [A] k
k2 ln −2 5 = 2.303
10
9. Let the rate equation be k [ A ]x [ B ]y k2
= 10 ⇒ k 2 = 10− 4 s − 1
From Ist values, 10− 5
0.045 = k [ 0.05 ]x [ 0.05 ]y …(i) Therefore, rate constant for the reaction at
500 K is 10− 4s − 1.
From 2nd values,
0.090 = k [ 010
. ]x [ 0.05 ]y …(ii) 11. Given, rate constant (k) = 0.05 µg/year
From 3rd values, Thus, from the unit of k, it is clear that the reaction is zero order.
Now, we know that
0.72 = k [ 0.20 ] [ 010
. ] x y
…(iii)
a
On dividing equations (i) by (ii), we get half-life (t1/ 2 ) for zero order reaction = o
2k
x
0.045  0.05  where, ao = initial concentration,
=
. 
0.09  010 k = rate constant
1 x 5 µg
 0.05   0.05  t1/ 2 = = 50 years
= 2 × 0.05 µg/year
 010
.   010
. 
∴ x =1 Thus, 50 years are required for the decomposition of 5 µg of X
into 2.5 µg.
Similarly on dividing Eq. (ii) by (iii) we get
0.09  01
x
.   0.05 
y 12. For zero order reaction,
= [ A0 ] − [ At ] = kt ...(i)
. 
0.72  0.2   010
y
where, [ A0 ] = initial concentration
.  0.05 
0.01 01 [ At ] = final concentration at time ‘t’
=
. 
0.08 0.2  01 k = rate constant
 0.05 
y [A ]
0.25 = Also, for zero order reaction, t1/ 2 = 0
. 
 010 2k
Given, t1/ 2 = 6 h and [ A0 ] = 0.2 M
0.25 = [ 0.5 ]y
0.2
[ 0.5 ]2 = [ 0.5 ]y ∴ 6=
2k
∴ y=2 0.2 1
or, k= =
Hence, the rate law for the reaction 2 × 6 60
Rate = k [ A ][ B ]2 Now, from Eq. (i)
[ A0 ] − [ At ] = kt
188 Chemical Kinetics

Given, [ A0 ] = 0.5 M, [ At ] = 0.2 M a


r2  2 A   B 
b
Expt 2 =   
1  1 r1  A   B 
∴ 0.5 − 0.2 = ×t Qk =
60  60  0.6
1 ⇒ = 2a × 1 ⇒ 21 = 2 a ⇒ a = 1 …(ii)
0.3 = × t ⇒ t = 0.3 × 60 = 18h 0.3
60 ∴ From Eq. (i), 1 + b = 3 ⇒ b = 2
13. The temperature dependence of rate of a chemical reaction is ⇒ Order of the reaction (n) = a + b = 1 + 2 = 3
expressed by Arrhenius equation as, k = Ae− E α / RT …(i) ⇒ Order of the reaction wrt. A = 1
where, A = Arrhenius factor or frequency factor or ⇒ Order of the reaction wrt. B = 2
pre-exponential factor 17. Let, the rate expression is r ∝ [ A ]a [ B ] b .
R = Gas constant, Ea = Activation energy
From experiment I,
Taking log on both sides of the
6.93 × 10− 3
a b b
Eq. (i), the equation becomes r2  0.1  0.25  5
=  ×  ⇒ =1 ×  
E r1  0.1  0.20 6.93 × 10− 3  4
ln k = ln A − a ln/k
RT b 0 b
 5  5  5
On comparing with equation of ⇒ 1=   ⇒   =  ⇒ b =0
 4  4  4
straight line 1/RT
a b
( y = mx + c), the nature of the r3  0.2  0.30
1 From experiment II, =  × 
plot of lnk vs will be: r1  0.1  0.20
RT
1.386 × 10− 2
(i) Intercept = C = ln A ⇒ = (2)a × (1.5)0
0.693 × 10− 2
(ii) Slope/gradient = m = − Ea = − y ⇒ Ea = y
⇒ 2 = 2a × 1 ⇒ 21 = 2a ⇒ a =1
So, the energy required to activate the reactant, (activation
energy of the reaction, Ea is = y ) ⇒ So, r ∝ [ A ]1[ B ]0 ⇒ r ∝ [ A ]
k1
Order of the reaction (n) = 1
14. The elementary reaction, A2 c 2A
k −1 ⇒ Now, let for the 1st experiment,
follows opposing or reversible kinetics, r1 = k ⋅ [ A ]
(i) Rate of the reaction, r1 6.93 × 10− 3
⇒ k= = = 6.93 × 10− 2 s− 1
r = rforward − rbackward = k 1[ A2 ] − k − 1[ A ]2 … (i) [A] 0.1
(ii) Again, rate of the reaction can be expressed as, 0.693 0.693
⇒ t50 = = = 10 s
d [ A2 ] 1 d[ A ] k 6.93 × 10− 2
r= − =+
dt 2 dt 18. From thermodynamics,
So, the rate of appearance of A, i.e. −∆H ° ∆S °
ln k = + …(i)
d[ A ] RT R
= 2r = 2k 1[ A2 ] − 2 k − 1 [ A ]2 [from Eq. (i)]
dt Mathematically, the equation of straight line is
15. The Arrhenius equation is, y = c + mx …(ii)
k = A. e− E a / RT After comparing Eq. (ii) with (i) we get,
− ∆H ° ∆S °
where, k = rate constant, slope = and intercept =
R R
A = Arrhenius constant, Ea = activation energy,
Now, we know for exothermic reaction ∆H is negative (−)ve.
and T = temperature in K But here,
From the equation, it is clear that k decreases exponentially with −∆H °
Ea . So, the plot-I is correct. Slope = is positive
R
In the plot-II, k is plotted with temperature (in °C but not in K). So, lines A and B in the graph represent temperature dependence
So, at 0°C, k ≠ 0 and k will increase exponentially with of equilibrium constant K for an exothermic reaction as shown
temperature upto 300°C. Therefore, the plot-II is also correct. below
16. For the reaction, 2A + B → products.
Let, the rate expression is ln K A
r ∝ [ A ]a [ B ]b
a b B
r2  2 A   2 B 
Expt 1 =    1
r1  A   B  (0, 0) T(K)
2.4
⇒ = 2a × 2b ⇒ 23 = 2a + `b ⇒ 3=a+ b … (i)
0.3
Chemical Kinetics 189

19. For the reaction, Alternative Method


Decomposes In fifty minutes, the concentration of H2O2 decreases from 0.5 to
CH3CHO( g ) → CH4 + CO 0.125 M or in one half-life, concentration of H2O2 decreases
Let order of reaction with respect to CH3CHO is m. from 0.5 to 0.25 M. In two half-lives, concentration of H2O2
decreases from 0.5 to 0.125 M or 2 t1/ 2 = 50 min
Its given, r1 = 1 torr/sec. when CH3CHO is 5% reacted i.e. 95%
unreacted. Similarly, r2 = 0.5 torr/sec when CH3CHO is 33% t1/ 2 = 25 min
reacted i.e., 67% unreacted.  0.693 −1
∴ k =  min
Use the formula, r ∝ (a − x )m  25 
where (a − x ) = amount unreacted d [O2 ] 1 d [H2O2 ] k [H2O2 ]
or =− = = 6.93 × 10−4 mol min −1
m dt 2 dt 2
r1 (a − x1 ) m
r1  a − x1 
so, = or =
r2 (a − x2 )m r2  a − x2  22. The main conditions for the occurrence of a reaction is proper
orientation and effective collision of the reactants.
Now putting the given values Since, the chances of simultaneous collision with proper
m
1  0.95 orientation between two species in high order reactions are very
=  ⇒ 2 = (141
. )m or m = 2
0.5  0.67 rare, so reaction with order greater than 3 are rare.

20. According to Arrhenius equation 23. For the elementary reaction, M → N


k = Ae− E a / RT Rate law can be written as
where, A = collision number or pre-exponential factor.
Rate ∝ [ M ] n
R = gas constant, T = absolute temperature Rate = k [ M ] n …(i)
Ea = energy of activation When we double the concentration of [ M ],
− E a1 / RT rate becomes 8 times, hence new rate law can be written as
For reaction R1, k 1 = Ae …(i)
− E a2 / RT 8 × Rate = k [ 2M ] n …(ii)
For reaction R2, k 2 = Ae …(ii)
Rate k [M ]n 1 1
On dividing Eq. (ii) by Eq. (i), we get = ⇒ =
8 × Rate k [ 2M ] n 8 [2]n
(E a − Ea )
− 2 1
k2
=e RT …(iii) ⇒ [2]n = 8 = [2]3 ⇒ n = 3
k1
24. This problem can be solved by determining the order of reaction
[Q Pre-exponential factor ‘A’ is same for both reactions] w.r.t. each reactant and then writing rate law equation of the
Taking ln on both the sides of Eq. (iii), we get given equation accordingly as
 k  E a1− E a 2 R=
dC
= k [ A ]x [ B ] y
ln  2  = dt
 k1 RT
where, x = order of reaction w.r.t A
Given, E a 1 = E a 2 + 10 kJ mol − 1 = Ea 2 + 10,000 J mol − 1 y = order of reaction w.r.t B
k2 10,000 J mol − 1 1. 2 × 10−3 = k (0 .1)x (0 .1) y
∴ ln = =4 1. 2 × 10−3 = k (01
. )x (0.2) y
k 1 8.314 J mol − 1K− 1 × 300 K
−3
2.4 × 10 = k (0.2)x (01 . )y
2.303 a
21. For first order reaction, k = log 1
R = k [ A] [ B] 0
t a−x
As shown above, rate of reaction remains constant as the
Given, t = 50 min, a = 0.5 M, a − x = 0125
. M
concentration of reactant (B) changes from 0.1 M to 0.2 M and
2.303 0.5 −1
∴ k= log = 0.0277 min becomes double when concentration of A change from 0.1 to 0.2,
50 0125
. (i.e. doubled).
Now, as per reaction
25. PLAN Time of 75% reaction is twice the time taken for 50% reaction if it
2H2O2 → 2H2O + O2 is first order reaction w.r.t. P. From graph, [Q ] decreases linearly
1 d [H2O2 ] 1 d [H2O] d [O2 ] with time, thus it is zeroth order reaction w.r.t. Q
− = = dx
2 dt 2 dt dt = bk[ P ]a[Q ]b
dt
d [H2O2 ]
Rate of reaction, − = k [H2O2 ] Order w.r.t P = a=1
dt Order w.r.t Q =b=0
d [O2 ] 1 d [H2O2 ] 1 Thus, overall order of the reaction = 1 + 0 = 1
∴ =− = k [H2O2 ] …(i)
dt 2 dt 2 k2 − Ea  1 1
When the concentration of H2O2 reaches 0.05 M, 26. From Arrhenius equation, log =  − 
k1 2.303 R  T2 T1 
d [O2 ] 1
= × 0.0277 × 0.05 [from Eq. (i)] k2
dt 2 Given, = 2 T2 = 310 K
d [O2 ] k1
or = 6.93 × 10−4 mol min −1
dt T1 = 300 K
190 Chemical Kinetics

On putting values, ⇒ For N2 + 3H2 → 2NH3


− Ea  1 1  d [ N2 ] 1 d [ H2 ] 1 d [ NH3 ]
⇒ log 2 =  −  Rate = − =− =
2.303 × 8.314  310 300 dt 3 dt 2 dt
⇒ Ea = 53.603 kJ/mol 35. Rate will be directly proportional to both concentration and
27. According to Arrehnius equation, rate constant increases intensity, i.e. rate of formation of AB * ∝ C ⋅ I .
exponentially with temperature : 36. The unit of rate constant (t − 1) indicating that the decomposition
k = Ae− E a / RT reaction following first order kinetics.
⇒ Rate = k[ N2O5 ]
28. The logarithmic form of Arrhenius equation is
Rate 2.40 × 10−5
Ea [ N2O5 ] = = = 0. 8 M
log k = log A − k 3 × 10−5
2.303 RT
560
2000 37. 560 days = = 4 half-lives.
Given : log k = 6 − 140
T
Amount of reactant remaining after n-half-lives
Comparing the above two equations : n 4
log A = 6 ⇒ A = 106  1  1 1
=   × initial amount =   × 1.0 g = g
 2  2 16
Ea
and = 2000
2.303 R 38. A catalyst increases the rate of reaction but by the same factor to
⇒ Ea = 2000 × 2.303 × 8.314 J= 38.3 kJ mol −1 both forward and backward directions. Hence, a catalyst shorten
the time required to reach the equilibrium.
ln 2
29. For first order reaction t1/ 2 = = 40 s …(i) 39. Specific rate constant of reaction depends on temperature.
k1
[ A ]0 40. The rate constant (k ) of all chemical reactions depends on
For zero order reaction t1/ 2 = = 20 s …(ii) temperature.
2k 0
1 [ A ]0 k k = Ae− E a / RT
⇒ Eq. (ii)/(i) = = × 1
2 2k 0 ln 2 where, A = pre-exponential factor, Ea = activation energy.
k 1 ln 2 0.693 SN 1
⇒ = = = 0.5 41. (CH3 )3 C  Br + NaOH →
k 0 [ A ]0 1.386 ( First order reaction)
(CH3 )3C OH+ NaBr
30. Rate ∝ [G ] m [ H ] n
0.693
This is first order reaction and for first order reaction t1/ 2 =
Q Rate is double on doubling the concentration of G and k
maintaining H constant, m = 1, i.e. R ∝ [G ]. So, half-life is independent of initial concentration.
Also, when both concentration of G and H are doubled, rate Therefore, the plot (a) correct.
increases by a factor of 8. Here rate is increasing by a factor of 2 For first order reaction, (r) = k [ (CH3 )3 C  Br ];
due to G (first order in G), therefore, factor due to H is 4.  P
P 
⇒ R ∝ [ H ]2 ⇒ Overall order = m + n = 1 + 2 = 3 ln  0  = k x t or ln   = − k x t
 P  P0 
31. Order of a reaction can take any real value, i.e. negative, integer, Hence, plot (b) and (d) are incorrect.
fraction etc. For first order reaction,
[Q ]
32. For first order reaction, Q = [ P0 ](1 − ekt ) or = (1 − ekt )
[ P0 ]
2.303 a 2.303 0.1
k= log = log = 3.46 × 10−2 Hence, plot (c) is incorrect.
t a−x 40 0.025
Rate = [ k ] A = 3.46 × 10−2 × 0.01 = 3.46 × 10−4 42. Given for the reaction (at T= 300 K and constant volume = V)
A ( g ) → 2B ( g ) + C ( g )
[ A ]0 at t = 0 – –
33. For a first order reaction, kt = ln p0
[A] at t = t p0 − x 2x x
1 [ A ]0 1 800 4 ln 2 − 1 at t = t1/ 3  p − 2 p0  = p0 4 p0 2 p0
⇒ k = ln = ln = s  0 3  3 3 3
4
t [ A ] 2 × 10 50 2 × 104
We can calculate,
= 1.386 × 10− 4 s− 1 pt = p0 − x + 2x + x = p0 + 2x
p − p0
34. For any general reaction, or 2x = pt − p0 or x = t
aA + bB → cC + dD 2
1 d [A] 1 d [ B ] 1 d [C ] 1 d [ D ] Now for first order reaction,
Rate = − =− = = 1 p0
a dt b dt c dt d dt t = ln
k ( p0 − x )
Chemical Kinetics 191

Putting the value of x in the equation, 46. Equilibrium constant is related to temperature
1 p0 1 2 p0 ∆H
t = ln = ln log K p = constant −
k  p − p0  k 2 p 0 − pt + p 0 2.3 RT
p0 −  t  ⇒ Plot of log K p vs 1/ T will be a straight line.
 2 
For the first order reaction X → P
2 p0
or kt = ln or kt = ln 2 p0 − ln ( 3 p0 − pt ) log
[ X ]0
=
kt
⇒ log [ X ] = log [ X 0 ] −
kt
,
( 3 p 0 − pt ) [ X ] 2.3 2.3
or ln ( 3 p0 − pt ) = − kt + ln 2 p0 i.e. log [ X ] vs ‘t’ will give a straight line.
It indicates graph between ln (3p0 − pt ) vs ‘t’ will be a straight Also at constant temperature, pV = constant
line with negative slope , so option (a) is correct 1
⇒ Plot of p vs will give a straight line.
1 p0 1 V
t1/3 = ln = ln 3
k p0 / 3 k 47. For a first order reaction :
It indicates t1/ 3 is independent of initial concentration so, option 1
kt = ln where, α = degree of dissociation.
(b) is incorrect. 1−α
Likewise, rate constant also does not show its dependence over ⇒ 1 − α = e− kt ⇒ α = 1 − e− kt
initial concentration. Thus, graph between rate constant and [ A ]0
will be a straight line parallel to X-axis. 1 ekt
Also = , i.e. plot of reciprocal of concentration of
43. If steric factor is considered, the corrected Arrhenius equation [ A ] [ A ]0
will be reactant vs time will be exponential.
−Ea
1 100 2 ln 2
k= pAe RT where A = frequency factor by Arrhenius. Time for 75% = ln = = 2 (t1/ 2 )
k 100 − 75 k
Q p > 1, pA > A hence, (a) is correct.
The Arrhenius equation is :
Activation energy is not related to steric factor. Ea
ln k = ln A −
44. Rate constant, k = Ae− E a / RT RT
The dimensions of k and A must be same. For first order reaction,
where, Ea = activation energy and A = pre-exponential factor
dimensions of k is t − 1.
(a) If Ea is high, it means lower value of k hence, slow reaction.
Thus, incorrect. 48. A catalyst lowers the activation energy by enabling the reaction
to continue through an alternative path, i.e. catalyst changes the
(b ) On increasing temperature, molecules are raised to higher reaction mechanism. However, catalyst does not affect either
energy (greater than Ea ), hence number of collisions average kinetic energies of reactants or the collision frequency.
increases. Thus, correct.
E d (log k ) E 49. Given, first order reaction,
(c) log k = log A − a ⇒ = a2 t75 = 90 minute
RT dT RT
t60 = ?
Thus, when Ea is high, stronger is the temperature
dependence of the rate constant. Thus, correct. Consider the following reaction,
(d) Pre-exponential factor (A) is a measure of rate at which A → B
(Reactant) (Product)
collisions occur. Thus, correct. Time, t = 0 100
t15% = 90 min (100 − 75) = 25
45. (a) For a first order reaction, the t 60% = ? (100 − 60) = 40
concentration of reactant remaining
after time t is given by [A] For first order reaction,
[ A ] = [ A ] 0 e– kt 2 . 303  a0  a0 = Initial concentration 
t= log 
Therefore, concentration of reactant k  a   a = Concentration 
t
decreases exponentially with time. For 75%,
2 . 303  100
 …(i) Conc. = 100 − 75 = 25
(b) Rise in temperature increases rate constant (k) and therefore Initial conc.= 100
t75% = log
decreases half-life (t1/ 2) as k  25   
ln 2 For 60%,
t 1/ 2 =
k 2 . 303  100
 …(ii) Conc. = 100 − 60 = 40
t60% = log Initial conc.= 100
(c) Half-life of first order reaction is independent of initial k  40   
concentration.
On equating Eqs. (i) and (ii),
(d) For a first order reaction, if 100 moles of reactant is taken
2 . 303  100
initially, after n half-lives, reactant remaining is given by log 
t60% k  40 
 1
n 8
 1 =
% A = 100   = 100   = 0.3906 t75% 2 . 303  100
 2  2 log 
k  25 
⇒ A reacted = 100 – 0.3906 = 99.6%
192 Chemical Kinetics

 0.3979 52. For the reaction,


t60% = t75% ×  
 0.602  A (g ) + B (g ) - AB(g )
0.3979 Given Ea b = Ea f + 2RT or Ea b − Ea f = 2RT
t60% = 90 ×
0.602 Further
t60° = 59.48 minute ≈ 60 min Af
A f = 4 Ab or =4
50. Rate = k [ A ]x[ B ]y[C ]z Ab
Now, rate constant for forward reaction,
(Rate)1 [ 0.2 ]x [ 01 . ]z 6 × 10−5
. ]y [ 01 − E a f / RT
= = k f = Af e
x y
(Rate)2 [ 0.2 ] [ 0.2 ] [ 01. ]z 6 × 10−5
⇒ y=0 Likewise, rate constant for backward reaction,
− E ab / RT
(Rate)1 [ 0.2 ]x [ 01
. ]y [ 01
. ]z 6 × 10−5 k b = Ab e
= =
x
(Rate)3 [ 0.2 ] [ 01 y z
. ] [ 0.2 ] 12 . × 10−4 At equilibrium,
Rate of forward reaction = Rate of backward reaction
⇒ z =1
kf
(Rate)1 [ 0.2 ]x [ 01 . ]z 6 × 10−5
. ]y [ 01 i.e., k f = k b or = k eq
= = kb
(Rate)4 [ 0.3 ]x [ 01 . ]2 9 × 10−5
. ]y [ 01 − E a f / RT
⇒ x =1 Af e Af − (E a f − E ab ) / RT
so k eq = − E ab / RT
= e
So, rate = k [ A ]1[C ]1 Ab e Ab
From exp-Ist, After putting the given values
Af
Rate = 6.0 × 10−5 mol dm −3 s −1 k eq = 4 e2 (as Ea b − Ea f = 2RT and = 4)
Ab
6.0 × 10−5 = k [ 0.2 ]1[ 01
. ]1
Now, ∆G ° = − RT ln K eq = − 2500 ln(4 e2 )
k = 3 × 10−3
= − 2500 (ln 4 + ln e2 ) = − 2500 (1.4 + 2)
Given, . mol dm −3
[ A ] = 015
= − 2500 × 3.4 = − 8500J/mol
[ B ] = 0.25 mol dm −3
Absolute value = 8500 J/mol
. mol dm −3
[C ] = 015
53. Living plants maintain an equilibrium between the absorption of
∴Rate = (3 × 10−3 ) × [ 015
. ]1[ 0.25 ]0[ 015
. ]1 C14 (produced due to cosmic radiation) and the rate of decay of
= 3 × 10−3 × 015
. × 015
. C14 present inside the plant. This gives a constant amount of C14
per gram of carbon in a living plant.
−5
Rate = 6.75 × 10 mol dm −3 s −1 54. Fossil whose age is closest to half-life of C-14 (5770 yr) will
Thus, Y = 6.75 yield the most accurate age by C-14 dating.
N
51. At constant V , T 55. λT = ln 0
N

2N 2O 5 (g ) → 2N 2O 4 (g ) + O 2 (g ) where N 0 = Number of C14 in the living matter and
At initial t = 0 1 0 0 N = Number of C14 in fossil. Due to nuclear explosion,
t = Y × 103 sec 1 − 2 p 2p p amount of C14 in the near by area increases. This will increase
N 0 because living plants are still taking C-14 from atmosphere,
pTotal = 1 − 2 p + 2 p + p
during photosynthesis, but N will not change because fossil will
1. 4 = 1 + p not be doing photosynthesis.
p = 0.45 atm ⇒ T (age) determined in the area where nuclear explosion has
According to first order reaction, occurred will be greater than the same determined in normal
2.303 pi area.
k= log
t pi − 2 p C C
Also, λT1 = ln 1 ⇒ λT2 = ln 2 ⇒ T1 − T2 = = ln 1
1 C
pi = 1atm (given) C C λ C2
2 p = 2 × 0.45 = 0.9 atm C = Concentration of C-14 in fossil.
On substituting the values in above equation, 56. k = A e− Ea / RT : At T = ∞ , k = A
1
2k ⋅ t = 2.303 log 1 d [ H2 ] 1 d [ NH3 ]
1 − 0.9 57. − =
3 dt 2 dt
−4 3 1 − d [ H2 ] 3 d [ NH3 ] 3
2 × 5 × 10 × y × 10 = 2.303 log ⇒ = = × 0.001 = 0.0015 kg h − 1 .
01
. dt 2 dt 2
y = 2.303 = 2.3
58. acidic, first or basic, second.
Note Unit of rate constant (k ), i.e. s−1 represents that it is a first
order reaction. 59. Rate is directly proportional to concentration of reactants.
Chemical Kinetics 193

60. R ∝ [Reactant] 100T 100


⇒ T1 = = × 12.8 = 33.68 h
38 38
On doubling the concentration of reactant, rate would be double.
T2 = 2T1 = 67.36 h
[ A ]0
61. For a first order process kt = ln 38T1 38 × 33.68
[A] T3 = = = 29.76 h
43 43
where, [ A ]0 = initial concentration.
66. R = k [ A ] ⇒ R1 = k [ A ]1 and R2 = k [ A ]2
[ A ] = concentration of reactant remaining at time “t”.
[ A ]0 R1 4 [ A ]1
⇒ kt1/ 8 = ln = ln 8 …(i) ⇒ = =
[ A ]0 / 8 R2 3 [ A ]2
[ A ]0 [ A ]1 4 ln 2 4
and kt1/ 10 = ln = ln 10 …(ii) Also k (t2 − t1 ) = ln = ln ⇒ × 10 = ln
[ A ]0 /10 [ A ]2 3 t1/ 2 3
t1/ 8 ln 8 10 log 3 3
Therefore, = = log 8 = 3 log 2 = 3 × 0.3 = 0.9 ⇒ t1/ 2 = = = 25 min
t1/ 10 ln 10 log 4 − log 3 0.6 − 0.48


t1/ 8
× 10 = 0.9 × 10 = 9 67. k 500 = A e− E1 / RT1
t1/ 10 k 400 = A e− E 2 / RT2
62. Rate of reaction is constant with time. Q k 500 = k 400
E1 E E2 T2 400 4
63. (a) Partial pressure becomes half of initial in every 100 min, ⇒ = 2 ⇒ = = =
RT1 RT2 E1 T1 500 5
therefore, order = 1.
800 Also E1 = E2 + 20000 J
(b) k × 100 = ln = ln 2 ⇒ k = 6.93 × 10−3 min − 1
400 E1 − 20,000 4
⇒ = ⇒ E1 = 100,000 J = 100 kJ mol −1
(c) For 75% reaction; time required = 2 × half-life = 200 min E1 5
(d) 2 X (g ) → 3Y (g ) + 2 Z (g ) [ A ]0
68. kt = ln
800 − x (3 / 2 ) x x [A]
3
Total pressure = 800 + x ⇒ 4.5 × 10− 3 × 60 = ln
1
⇒ [ A ] = 0.76 M
2 [A]
Also 800 − x = 700 ⇒ x = 100
⇒ Rate = k [ A ] = 4.5 × 10− 3 × 0.76
3
⇒ Total pressure = 800 + × 100 = 950 mm Hg = 3.42 × 10−3 mol L−1 min −1
2
a k 2 Ea  T2 − T1 
(Rate)1 0.05 1  1 69. (i) ln =  
64. = = =   ⇒ a = 1; order w.r.t A. k1 R  T1T2 
(Rate)2 0.10 2  2
 4.5 × 107  Ea  50 
Order w.r.t B = 0 ⇒ ln   =   ⇒ Ea = 22 kJ
(a) Rate = k [ A ]  1.5 × 107  8.314  323 × 373
Rate 0.05 Ea
(b) k = = = 0.5 s− 1 Also ln k = ln A −
[ A ] 0.10 RT
7 22 × 1000
64 38% 0 64 At 50°C : ln A = ln (1.5 × 10 ) − = 8.33
65. 29 Cu → − 1β + 30Zn 8.314 × 323
k1
64 19% 0 64
⇒ A = 4.15 × 103 s− 1
29 Cu → + 1β + 28 Ni
k2 1
(ii) N2O5 (g ) → 2NO2 (g ) + O2 (g )
64 0 43% 64 2
29 Cu + − 1e → 28 Ni 600 − p 2p p/ 2
k3
Above are the parallel reactions occurring from Cu . 64 3
Total pressure = 960 = 600 + p ⇒ p = 240 mm
k 1 38 T k 1 38 T3 2
= = 2 = 2 and = =
k 2 19 T1 k 3 43 T1 ⇒ Partial pressure of N2O5 (g ) remaining = 600 – 240
= 360 mm
T1 , T2 and T3 are the corresponding partial half-lives.
360
Also k = k1 + k2 + k3 ⇒ Mole fraction = = 0.375
960
ln 2 ln 2 ln 2 ln 3
⇒ = + + 70. (i) The Arrhenius equation is
T T1 T2 T3 Ea
1 1 1 1 1 1 43 log k = log A −
⇒ = + + = + + 2.303 RT
T T1 T2 T3 T1 2T1 38T1
Comparing with the given equation :
1  1 43 1  38 + 19 + 43 100 Ea
= 1 + +  =   = 1.25 × 104 = ⇒ Ea = 239.33 kJ mol −1
T1  2 38 T1  38  38T1 2.303 R
194 Chemical Kinetics

(ii) When half-life = 256 min, 74. CH3  O  CH3 (g ) → CH4 (g ) + H2 (g ) + CO (g )


ln 2 0.693 − 1
k= = s = 4.5 × 10− 5 s− 1 At 12 min : 0.40 − p p p p
t1/ 2 256 × 60
Total pressure = 0.4 + 2p
1.25 × 104 0.40 ln 2
⇒ = 14.34 – log 4.5 × 10− 5 = 16.68 Also k × 12 = ln = × 12 = 1.77 ⇒ p = 0.175
T 0.40 − p 14.5
1.25 × 104
⇒ T = = 669 K ⇒ Total pressure = 0.4 + 2p = 0.4 + 2 × 0.175 = 0.75 atm
16.68
 k (40° C)  Ea  15  70 × 1000 15
w0 ln 2 10− 6 g 75. ln  =  = × = 1.35
71. k t = ln ⇒ × 20 = ln ⇒ w = 6.1 × 10− 7 g  k (25° C)  R  298 × 313 8.314 298 × 313
w 28.1 w
k (40° C)
⇒ = 3.87
72. For 1st order reaction : k (25° C)
1
k∝ 1 100 1 4
t1/ 2 Also k (25°C) = ln = ln
20 75 20 3
k (450° C)   t1/ 2 (380° C)  Ea  450 − 380 ⇒ k (40° C) = 3.87 × k (25°C)
⇒ ln   = ln  =  
k (380° C)   t1/ 2 (450° C)  R  727 × 653 1 4
= 3.87 × ln = 55.66 × 10− 3 min − 1
 360  200 × 103 70 20 3
⇒ ln  = × = 3.54 100
 t1/ 2 (450° C)  8.314 727 × 653 Now k (40°C) × 20 = ln
100 − x
⇒ t1/ 2 (450°C) =10.37 min
100
⇒ Time for 75% reaction at 450°C ⇒ 55.66 × 10− 3 × 20 = ln ⇒ x = 67%
100 − x
= 2 × t1/ 2 = 2 × 10.37 = 20.74 min
k
73. Comparing the data of experiment number 2 and 3 : 76. (i) A →
A
Product
m
R3 1.6 × 10− 2  1.0 × 10− 3  kB
(ii) B → Product
= = 
R2 4 × 10− 3  5 × 10− 4 
Ea  10 
⇒ m = 2, order w.r.t. A For (i)   = ln 2
R  300 × 310
Now comparing the data of experiment number 1 and 2 :
2 n ⇒ Ea (i) = 9300 R ln 2 = 53.6 kJ
R2 4 × 10−3  5 × 10− 4   6.0 × 10− 5  E (i )
= =    ⇒ Ea (ii) = a = 26.8 kJ
R1 5 × 10− 4  2.5 × 10− 4   3.0 × 10− 5  2
⇒ 8 = (2)2 (2)n ⇒ n = 1, order w.r.t. B. At 310 K t1/ 2 (i) = 30 min
(i) Order with respect to A = 2, order with respect to B = 1. Q Rate of (ii) = 2 rate of (i)
(ii) At 300 K, R = k [ A ]2 [ B ] ⇒ t1/ 2 (ii) = 15 min
Now for reaction (ii) :
R 5.0 × 10− 4
⇒ k= =  k (310)   t1/ 2 (300)  Ea (ii )  10 
[ A ] [ B ] (2.5 × 10− 4 )2 (3.0 × 10− 5 )
2 ln  B  = ln  =  
 k B (300)   t1/ 2 (310)  R  300 × 310
= 2.66 × 108 s− 1 L2 mol −2
 t (300)  ln 2
(iii) From first experiment : ⇒ ln  1/ 2 = ⇒ t1/ 2 (300) = 21.2 min
 15  2
Rate (320 K) = k (320 K) (2.5 × 10− 4 )2 (3.0 × 10− 5 ) ln 2 0.693
⇒ k B (300) = = = 3.26 × 10− 2 min − 1
2 × 10−3 t1/ 2 21.2
⇒ k (320 K) =
(2.5 × 10− 4 )2 (3.0 × 10− 5 ) 77. Initially :
= 1.066 × 109 s− 1 L2 mol − 2. 10 20
N (1 H3 + 1H1 ) = × 2 × 6 × 1023 = × 1023
8 3
 k (320 K)  Ea  T2 − T1 
⇒ ln  =   N (1 H1 ) 20 × 1023
 k (300 K)  R  T1T2  ⇒ 1+ =
N (1 H3 ) 3N (1 H3 )
 1.066 × 109  Ea  20  1 20 × 1023
⇒ ln   =   ⇒ 1+ = ≈ 1.25 × 1017
 2.66 × 108  8.314  300 × 320 8 × 10 − 18
3N (1 H3 )
⇒ Ea = 55.42 kJ mol −1 20 × 1023
E ⇒ N (1 H 3 ) = = 5.33 × 106
Now ln k = ln A − a 3 × 1.25 × 1017
RT
N0 ln 2 5.33 × 106
55.42 × 103 ⇒ kt = ln ⇒ × 40 = ln
At 300 K : ln (2.66 × 108 ) = ln A − N 12.3 N
8.314 × 300
⇒ N = 5.6 × 105
Solving : ln A = 41.62 ⇒ A = 1.2 × 1018
Chemical Kinetics 195

78. For the reaction : 100


⇒ = 1.055
2N2O5 → 4NO2 + O2 100 − x
If p0 is the initial pressure, the total pressure after completion of ⇒ x = 5.25% reactant is converted into product.
5 ln 2 0.693
reaction would be p0. Half-life = = = 462000 s = 128.33 h
2 k 1.5 × 10− 6
5
⇒ 584.5 = p0 ⇒ p0 = 233.8 mm [ A ]0
2 82. For a first order process : ln = kt ⇒ ln [ A ] = ln [ A ]0 − kt
[A]
Let the pressure of N2O5 decreases by ‘p’ amount after 30 min.
Therefore, If the reactant is in gaseous state
2N2O5 → 4NO2 + O2 [ A ]0
p
and kt1/ 10 − ln = ln 10 …(ii)
At 30 min : p0 − p 2p [ A ]0 / 10
2
3 t ln 8
Total pressure = p0 + p = 284.5 Therefore, 1/ 8 = = log 8 = 3 log 2 = 3 × 0.3 = 0.9
2 t1/ 10 ln 10
2 t1/ 8
⇒ p= (284.5 − 233.8) = 33.8 ⇒ × 10 = 0.9 × 10 = 9
3 t1/ 10
p0
Now, kt = ln ln p = ln p0 − kt …(i)
p0 − p
1 233.8 where p is the partial pressure of reactant remaining unreacted at
⇒ k= ln min − 1 = 5.2 × 10− 3 min − 1 instant ‘t’ and p0 is its initial partial pressure.
30 233.8 − 33.8
Also, from equation (i), ln p vs t would give a straight line.
79. Arrhenius equation is : Therefore, decomposition of N2O5 following first order kinetics.
Ea ln 2 0.693 − 1
log k = log A − 83. k = = yr = 1.2 × 10− 4 yr − 1
2.303 RT t1/ 2 5770
ln 2 0.693 1 ln 2 1
when t1/ 2 = 10 min, k = = = 1.115 × 10− 3 s− 1 Also kt = ln = × 11540 = ln 4 ⇒ f = = 0.25
t1/ 2 10 × 60 f 5770 4
Ea A 4 × 1013
⇒ = log A − log k = log = log 84. For a first order reaction,
2.303 RT k 1.115 × 10− 3 [ A ]0
kt = ln
= 16.54 [A]
Ea 98.6 × 1000
⇒T = = = 311.34 K where [ A ]0 = Initial concentration of reactant
2.303 R × 16.54 2.303 × 16.54 × 8.314
[ A ] = Concentration of reactant remaining
80. The minimum rate of decay required after 6.909 h is unreacted at time t.
346 particles min − 1. 1 [ A ]0 1 100 1 5
(i) ⇒ k = ln = ln = ln
⇒ Rate = kN t [ A ] 10 100 − 20 10 4
Rate 346 × 66.6 × 60 2.303 (log 5 − 2 log 2)
⇒ N = = = 1.995 × 106 atoms = min − 1 = 0.023 min –1
k 0.693 10
N ln 2 N 1 100 2 ln 2 2 × 0.693
⇒ kt = ln 0 ⇒ × 6.909 = ln 0 = 0.0715 (ii) t = ln = = = 60 min
N 66.6 N k 25 k 0.023
N0
⇒ = 1.074 85. Looking at the rate data of experiment number 1 and 2 indicates
N that rate is doubled on doubling concentration of A while
⇒ N 0 = 1.074 × N = 1.074 × 1.995 × 106 concentration of B is constant. Therefore, order with respect to A
= 2.14 × 106atoms of Mo is 1. Similarly, comparing data of experiment number 1 and 3,
doubling concentration of B, while concentration of A is
2.14 × 106
⇒ Mass of Mo required = × 99 = 3.56 × 10− 16 g constant, has no effect on rate.
6.023 × 1023 Therefore, order with respect to B is zero.
81. k = 1.5 × 10− 6 s− 1 ⇒ Rate = k [ A ]
0.005 0.693
kt = ln
100 ⇒ k= = 0.5 min − 1 =
100 − x 0.010 t1/ 2
100 0.693
⇒ ln = 1.5 × 10− 6 s− 1 × 10 × 60 × 60 s = 0.0054 ⇒ t1/ 2 = = 1.386 min
100 − x 0.5
12
Nuclear Chemistry
Objective Questions I (Only one correct option) Objective Questions II
1. Bombardment of aluminium by α-particle leads to its artificial (One or more than one correct option)
disintegration in two ways, (i) and (ii) as shown. Products X , Y 9. In the decay sequence.
and Z respectively, are (2011) 238 − x1 234 − x2 234 − x3
92 U 
→ 90 Th 
→ 91 Pa  →
27 (ii) 30 − x4
234 230
13Al 15P +Y Z 
→ 90 Th

(i) x1 , x2 , x3 and x4 are particles/radiation emitted by the


respective isotopes. The correct option(s) is(are) (2019 Adv.)
30 30 (a) Z is an isotope of uranium
14Si +X 14Si +Z
(b) x2 is β −
(a) proton, neutron, positron (b) neutron, positron, proton
(c) proton, positron, neutron (d) positron, proton, neutron (c) x1 will deflect towards negatively charged plate
23
(d) x3 is γ-ray
2. A positron is emitted from The ratio of the atomic mass
11 Na.
10. A plot of the number of neutrons (n) against the number of
and atomic number of the resulting nuclide is (2007, 3M)
protons (p) of stable nuclei exhibits upward deviation from
(a) 22/10 (b) 22/11
(c) 23/10 (d) 23/12 linearity for atomic number, Z > 20. For an unstable nucleus
23
having n/p ratio less than 1, the possible mode(s) of decay is
3. Na is the more stable isotope of Na. Find out the process by (are) (2016 Adv.)
which 24
11 Na can undergo radioactive decay. (2003, 1M) (a) β − - decay (β - emission) (b) orbital or K-electron capture

(a) β -emission (b) α-emission (c) neutron emission (d) β + -decay (positron emission)
(c) β + -emission (d) K-electron capture 9
11. In the nuclear transmutation, 4 Be + X → 84 Be + Y
139
4. The number of neutrons accompanying the formation of 54 Xe X and Y are (2013 Adv.)
and 94
38 Sr from the absorption of a slow neutron by
235
92 U, (a) (γ , n) (b) ( p, D) (c) (n, D) (d) (γ , p)
followed by nuclear fission is (1999, 2M) 12. Decrease in atomic number is observed during (1998, 2M)
(a) 0 (b) 2 (c) 1 (d) 3 (a) alpha emission (b) beta emission
27 29
5. 13 Al is a stable isotope. 13 Al is expected to decay by (c) positron emission (d) electron capture
(a) α-emission (b) β-emission (1996, 1M) 13. The nuclear reactions accompanied with emission of neutron(s)
(c) positron emission (d) proton emission are (1988, 1 M)
27 4 30 12 1 13
6. The radiation from a naturally occurring radioactive substance, (a) 13 Al + 2He → 15P (b) 6 C + 1H → 7N
as seen after deflection by a magnet in one direction,(1984,
are 1M) 30 30 0 241 4 244
(c) 15 P → 14Si + 1e (d) 96 Cm + 2He → 97Bk + 01e
(a) definitely alpha rays (b) definitely beta rays
(c) both alpha and beta rays (d) either alpha rays or beta rays
Numerical Answer Type Questions
7. An isotope of Ge76
32 is (1984, 1M) 238
14. 92 U is known to undergo radioactive decay to form 20682 Pb
(a) Ge77
32 (b) As77
33
by emitting alpha and beta particles. A rock initially
(c) Se77 (d) Se78
contained 68 × 10−6 g of 238
34 34
92 U. If the number of alpha
8. If uranium (mass number 238 and atomic number 92) emits an particles that it would emit during its radioactive decay of
α-particle, the product has mass number and atomic number 18
92 U to 82 Pb in three half-lives is Z × 10 , then what is the
238 206
(1981, 1 M)
(a) 236 and 92 (b) 234 and 90 value of Z? (2020 Adv.)
(c) 238 and 90 (d) 236 and 90
Nuclear Chemistry 197

15. During the nuclear explosion, one of the products is 90Sr with 21. Elements of the same mass number but different atomic number
half-life of 6.93 years. If 1 µg of 90Sr was absorbed in the bones are known as ……… (1983, 1M)
of a newly born baby in place of Ca, how much time, in years, is
required to reduce it by 90% if it is not lost metabolically 22. An element Z M A undergoes an α-emission followed by two
……… . (2020 Main, 7 Jan I) successive β -emissions. The element formed is ……
(1982, 1M)
Assertion and Reason
Read the following questions and answer as per the direction Integer Answer Type Questions
given below: 23. The periodic table consists of 18 groups. An isotope of copper,
(a) Statement I is true; Statement II is true; Statement II is on bombardment with protons, undergoes a nuclear reaction
the correct explanation of Statement I yielding element X as shown below. To which group, element
(b) Statement I is true; Statement II is true; Statement II is X belongs in the periodic table? (2012)
not the correct explanation of Statement I 63
+ 11 H → 610 n + 42α + 211 H + X
29 Cu
(c) Statement I is true; Statement II is false 235
24. The number of neutrons emitted when 92 U undergoes
(d) Statement I is false; Statement II is true
142 90
16. Statement I The plot of atomic number ( y-axis) versus number controlled nuclear fission to 54 Xe and 38 Sr is (2010)
of neutrons (x-axis) for stable nuclei shows a curvature towards
x-axis from the line of 45° slope as the atomic number is Subjective Questions
increased. 25. The total number of α and β particles emitted in the nuclear
Statement II Proton-proton electrostatic repulsions begin to reaction 238
→ 214
is ..... .
92 U 82Pb (2009)
overcome attractive forces involving protons and neutrons and
neutrons in heavier nuclides. (2008) 234 − 7α
26. 92 X → Y. Find out atomic number, mass number of Y
30 −6β
17. Statement I Nuclide is less stable than 40
13 Al 20 Ca.
Statement II Nuclides having odd number of protons and and identify it. (2004)
238
neutrons are generally unstable. (1998) 27. 92 U is radioactive and it emits α and β particles to form
206
82 Pb . Calculate the number of α and β particles emitted in this
Fill in the Blanks conversion.
18. (a) 235 1
92 U + 0n →
137
52 A + 97
40 B + ...... An ore of 92 U
238
is found to contain 92 U
238
and 82 Pb
206
in the
238
(b) 82
34 Se → 2 0
− 1e + ...... (2005, 1M × 2 = 2M) weight ratio of 1 : 0.1. The half-life period of 92 U is 4.5 × 109
19. A radioactive nucleus decays by emitting one alpha and two yr. Calculate the age of the ore. (2000)
14
beta particles, the daughter nucleus is.... of the parent. 28. Write a balanced equation for the reaction of N with
(1989, 1M) α-particle.
234
14 29. 90 Th disintegrates to give 82 Pb206 as the final product. How
20. The number of neutrons in the parent nucleus which gives N
many alpha and beta particles are emitted during this process?
on beta emission is ........... (1985, 1M)
(1986, 2M)

Answers
A−4
1. (a) 2. (c) 3. (a) 4. (b) 20. eight 21. isobars 12. ZM 23. (8)
5. (b) 6. (c) 7. (a) 8. (b) 24. 3 25. (8) 26. 206
84 Po
9. (a, b, c) 10. (b, d) 11. (a, b) 12. (a, c, d)
27. (7.12 × 10 yr) 8
29. (13)
13. (a, d) 14. (1.2) 15. (23.03) 16. (a)
82
17. (b) 18. 2 0n1, 36 Kr 19. isotope
Hints & Solutions
1. (i) 27
13 Al + 42He (α) → 30
14Si + 11X (a) By β − - decay, 10 n → 11 p + −01e neutron changes to
X is proton 11H. proton. Thus, (n/ p) ratio further decreases below 1.
27
Thus, this decay is not allowed.
(ii) 13 Al + 42He (α ) → 30
15P + 10Y
(b) By orbital or K- electron capture, 11 p + 0
−1e → 10n
1
Y is neutron, 0 n.
proton changes to neutron, hence, (n/ p) ratio increases.
30 30
15 P → 14Si + 10Z Thus stability increases. Thus correct.
0
Z is positron , +1e. (c) Neutron emission further decreases n/p ratio.
2. The required nuclear reaction is (d) By β + -emission, 11 p → 10n + +01e
23
→ 0
+ 23 proton changes to neutron.
11 Na +1e 10Na
Hence, n/ p ratio increases. Thus correct.
3. In stable isotope of Na, there are 11 protons and 12 neutrons. In
the given radioactive isotope of sodium (Na 24 ), there are 13
120
neutrons, one neutron more than that required for stability. A Neutron-rich
neutron rich isotope always decay by β-emission as 100 nuclei 1.5

Number of neutrons
1 0
0n → −1β + 1H1
80 1.4
4. The balanced nuclear reaction is
20 n1 + 92U
235
→ 54Xe
139
+ 38 Sr
94 60 1.2
29
5. 13 Al is neutron rich isotope, will decay by β-emission 40
converting some of its neutron into proton as 1.1 ratio
1 0 1 20
0 n → −1β + 1H Neutron poor nuclei
6. Both α-rays and β - rays are deflected by magnetic field. 0
20 40 60 80
7. Isotopes have same atomic number (Z ) but different mass Number of protons
76 77
number ( A ). Therefore, 32 Ge and 32 Ge are isotopes. Plot of the number of neutrons against the number of protons in
8. The nuclear reaction is stable nuclei (shown by dots).
238
92 U → 2He4 (α ) + 90Th
234

11. PLAN 94 Be + ba X → 84 Be + dc Y
9. Key Idea The lose of one α-particle will decrease the mass
number by 4 and atomic number by 2. On the other hand, loss Atomic number same
of β-particle will increase the atomic number by 1.
4 + a=4 + c
In decay sequence, 9+ b=8+ d
a=0
92U
238
90Th
234
+ 2He4 (or a) If X = 00 γ
X1 particle b=0
c=0
234 Y = 10 n
92U
234
+ (b– or –1e0) 91Pa + (b– or –1e0) d =1
234 X3 particle X2 particle
Z is isotope
a=1
of uranium
If X = 11 p
b=1
90Th
230
+ 2He4 (or a) c=1
Y = 12 D
X4 particle d=2
9
X 1 particle will deflect towards negatively charged plate due to 4 Be + 00 γ → 84 Be + 10n
presence of positive charge on α- particles. 9
4 Be + 11 p → 84 Be + 21D
Hence, options (a, b, c) are correct.
10. For the elements with atomic number (Z ) larger than 20, 12. In the following nuclear reactions, there occur decrease in
atomic number (Z )
Neutrons (n) > Protons (p); Thus, n/ p > 1 A
→ 2He4 + A− 4
, α - emission
ZX Z − 2Y
Thus, there is upward deviation from linearity. 0
If n < p, Thus n/ p < 1, then ZX
A
→ +1e + A
Z − 1Y , positron emission
0
ZX
A
+ −1e → A
Z − 1Y , electron capture
Nuclear Chemistry 199

In beta emission, increase in atomic number is observed. n 17


(13 Al 30 ) = = 1.3 > 1, unstable, β-emitters.
A
→ 0
+ , β -emission
A p 13
ZX −1e Z + 1Y
n 20
13. If sum of mass number of product nuclides is less than the sum ( Ca 40 ) = = 1, stable.
p 20 20
of parent nuclides, then neutron emission will occur. In both (a)
and (d), sum of mass number of product nuclides is one unit less Also, nuclei with both neutrons and protons odd are usually
than the sum of parent nuclides, neutron emission will balance unstable but it does not explain the assertion appropriately.
the mass number.
238
→ 206
+ 842 He + 6 0− 1β = 23.03 (yr)
14. 92 U 82 Pb
235 1 137 97
238 18. (a) 92 U + 0n → 52 A + 40 B + 2 0n1
Number of moles of 92 U present initially
82
−6 (b) 34 Se → 2 −1e0 + 36Kr
82
68 × 10
=
238 19. Isotope : Z X A → 2He4 + 2 −1e0 + ZY A − 4
238
After three half-lifes, moles of 92 U decayed 20. 8 : 6 C14 → 7 N14 + 0
−1e
−6 −6
68 × 10  1  68 × 10 7
= × 1 − 3  = × 21. Isobars have same mass number but different atomic number.
238  2  238 8
A−4
22. ZM : Z M A → 2He4 + 2 −1e0 + ZM A − 4
Therefore, number of α-particles emitted
68 × 10− 6 7 23. Balancing the given nuclear reaction in terms of atomic number
= × × 8 × 6.023 × 1023
238 8 (charge) and mass number:
63
= 1204
. × 1018 ≈ 12
. × 1018 29 Cu + 1 H1 → 60 n1 + 2 He4 (α ) + 21 H1 + 26 X 52
Thus, the correct answer is 1.2. The atomic number 26 corresponds to transition metal Fe which
belongs to 8th group of modern periodic table.
15. Radioactive decay follows first order kinetics. 235 142 90
∴Time taken for decay from N 0 to N t is 24. 92 U → 54Xe + 38 Sr + 30 n1
(N = number of nuclei) 25. 92 U
238
→ 82Pb
214
+ 6 2He4 + 2 −1e
0

1 N
t = ln 0 ⇒ Number of (α + β ) = 6 + 2 = 8
λ Nt 234
26. 92 X → 7 2He4 + 6 −1e0 + 84Y
206
1 N
t = × 2.303 log 0 …(i) Y is 84 Po
206
.
λ Nt
238 206
0.693 27. 92 U → 82Pb + 8 2He4 + 6−1 e0
Also, we know λ = (decay constant) =
t1/ 2 Present : N 0 − N N
where, t1/ 2 = 6.93 yr (given) w (U) 1
Given, = = 10
w (Pb) 0.1
N (U) 10 206 N 0 − N
⇒ = × =
Also, we know 90% nuclei are decayed N (Pb) 238 1 N
N 0 100 N 238 N0 238 2298
∴ = = 10 ⇒ = ⇒ =1+ =
Nt 10 N 0 − N 2060 N0 − N 2060 2060
N Now, applying first order rate law
Put the values of λ and 0 in Eq. (i), we get
Nt  ln 2  N0  (t )  N0 
  t = ln   ⇒ t = 1/ 2 log  
6.93  t1/ 2  N0 − N  log 2 N0 − N 
∴ t= × 2.303 × log10
0.693 4.5 × 109 2298
= log = 7.12 × 108 yr
16. After atomic number 20, proton-proton repulsion increases 0.3 2060
immensely, more neutrons are required to shield this 14
28. 7N + 2He4 → 9F
18
electrostatic repulsion, curve of stability incline towards
234 206
neutron axis. 29. 90 Th → 82Pb + 7 2He4 + 6 −1e0
17. Upto atomic number of 20, stable nuclei possess neutron to
proton ratio (n/ p) = 1.
13
Surface Chemistry
Objective Questions I (Only one correct option) (b) brownian motion in colloidal solution is faster if the viscosity
of the solution is very high.
1. Simplified absorption spectra of three complexes [(i), (ii) and
(c) addition of alum to water makes it unfit for drinking.
(iii)] of M n+ ion are provided below; their λ max values are
(d) colloidal particles in lyophobic sols can be precipitated by
marked as A, B and C respectively. The correct match electrophoresis.
between the complexes and their λ max values is
(2020 Main, 2 Sep II)
5. A gas undergoes physical adsorption on a surface and follows
x
the given Freundlich adsorption isotherm equation = Kp 0. 5
m
Adsorption of the gas increases with (2019 Main, 10 April I)
C (a) increase in p and increase in T
A
(b) increase in p and decrease in T
Absorption

B
(c) decrease in p and decrease in T
(d) decrease in p and increase in T
6. Match the catalysts Column I with products Column II.
(2019 Main, 9 April I)

λmax λmax λmax Column I (Catalyst) Column II (Product)


Wavelength (nm) (A) V2O5 (i) Polyethlyene
(B) TiCl 4 / Al(Me)3 (ii) Ethanal
(i) [ M (NCS)6 ](− 6 + n)
(C) PbCl 2 (iii) H2SO4
(ii) [ M F6 ](− 6 + n)
(D) Iron oxide (iv) NH3
(iii) [ M (NH3 )6 ]n +
(a) A-(iii), B-(i), C-(ii) (b) A-(ii), B-(i), C-(iii) (a) (A)-(ii), (B)-(iii), (C)-(i), (D)-(iv)
(c) A-(ii), B-(iii), C-(i) (d) A-(i), B-(ii), C-(iii) (b) (A)-(iv), (B)-(iii), (C)-(ii), (D)-(i)
2. Among the following, the incorrect statement about colloids (c) (A)-(iii), (B)-(i), (C)-(ii), (D)-(iv)
is (2019 Main, 12 April II) (d) (A)-(iii), (B)-(iv), (C)-(i), (D)-(ii)
(a) They can scatter light 7. The number of water molecule(s) not coordinated to copper
(b) They are larger than small molecules and have high molar mass ion directly in CuSO4 ⋅ 5H 2O, is (2019 Main, 9 April I)
(c) The osmotic pressure of a colloidal solution is of higher order (a) 2 (b) 3
than the true solution at the same concentration (c) 1 (d) 4
(d) The range of diameters of colloidal particles is between 1 and
8. The aerosol is a kind of colloid in which (2019 Main, 9 April I)
1000 nm
(a) gas is dispersed in liquid
3. Peptisation is a (2019 Main, 12 April I) (b) gas is dispersed in solid
(a) process of bringing colloidal molecule into solution (c) liquid is dispersed in water
(b) process of converting precipitate into colloidal solution (d) solid is dispersed in gas
(c) process of converting a colloidal solution into precipitate 9. Adsorption of a gas follows Freundlich adsorption isotherm.
(d) process of converting soluble particles to form colloidal solution x is the mass of the gas adsorbed on mass m of the adsorbent.
4. The correct option among the following is x x
(2019 Main, 10 April II)
The plot of log versus log p is shown in the given graph.
m m
(a) colloidal medicines are more effective, because they have small is proportional to (2019 Main, 8 April I)
surface area.
Surface Chemistry 201

17. Which of the salt-solution is most effective for coagulation of


arsenious sulphide? (2019 Main, 9 Jan II)
x (a) BaCl 2 (b) AlCl 3
log — 2
m (c) Na 3PO4 (d) NaCl
3
18. Adsorption of a gas follows Freundlich adsorption isotherm.
In the given plot,x is the mass of the gas adsorbed on mass m
log p x
of the adsorbent at pressure p ⋅ is proportional to
(a) p2 / 3 (b) p3/ 2 (c) p3 (d) p2 m
(2019 Main, 9 Jan I)
10. Among the following, the false statement is
(2019 Main, 12 Jan II)
(a) Tyndall effect can be used to distinguish between a colloidal
2 unit
solution and a true solution x
log m
(b) It is possible to cause artificial rain by throwing electrified sand 4 unit
carrying charge opposite to the one on clouds from an aeroplane
(c) Lyophilic sol can be coagulated by adding an electrolyte
(d) Latex is a colloidal solution of rubber particles which are log p
positively charged
(a) p2 (b) p1/ 4
11. Given, Gas : H2 , CH4 , CO2 , SO2 1/ 2
(c) p (d) p
Critical temperature/K 33 190 304 630
On the basis of data given above, predict which of the 19. The Tyndall effect is observed only when following
following gases shows least adsorption on a definite amount conditions are satisfied (2017 Main)
of charcoal? (2019 Main, 12 Jan I) 1. The diameter of the dispersed particles is much smaller
(a) CH4 (b) SO2 (c) CO2 (d) H2 than the wavelength of the light used.
12. Among the colloids cheese (C), milk (M) and smoke (S), the 2. The diameter of the dispersed particle is not much smaller
correct combination of the dispersed phase and dispersion than the wavelength of the light used.
medium, respectively is (2019 Main, 11 Jan II) 3. The refractive indices of the dispersed phase and
(a) C : liquid in solid; M : liquid in liquid; S : solid in gas dispersion medium are almost similar in magnitude.
(b) C : solid in liquid; M : liquid in liquid; S : gas in solid 4. The refractive indices of the dispersed phase and
(c) C : liquid in solid; M : liquid in solid; S : solid in gas dispersion medium differ greatly in magnitude.
(d) C : solid in liquid; M : solid in liquid; S : solid in gas (a) 1 and 4 (b) 2 and 4
13. An example of solid sol is (2019 Main, 11 Jan I) (c) 1 and 3 (d) 2 and 3
(a) gem stones (b) hair cream 20. For a linear plot of log ( x / m ) versus log p in a Freundlich
(c) butter (d) paint
adsorption isotherm, which of the following statements is
14. Haemoglobin and gold sol are examples of correct? (k and n are constants) (2016 Main)
(2019 Main, 10 Jan II)
(a) 1/n appears as the intercept
(a) negatively and positively charged sols, respectively
(b) Only 1/n appears as the slope
(b) negatively charged sols
(c) log   appears as the intercept
(c) positively charged sols 1
(d) positively and negatively charged sols, respectively  n
(d) Both k and 1/n appear in the slope term
15. Which of the following is not an example of heterogeneous
catalytic reaction? (2019 Main, 10 Jan I) 21. Methylene blue, from its aqueous solution, is adsorbed on
(a) Haber’s process activated charcoal at 25°C. For this process, the correct
(b) Combustion of coal statement is (2013 Adv.)
(c) Hydrogenation of vegetable oils (a) the adsorption requires activation at 25°C
(d) Ostwald’s process (b) the adsorption is accompanied by a decreases in enthalpy
(c) the adsorption increases with increase of temperature
16. The correct match between item-I and Item-II is
(d) the adsorption is irreversible
(2019 Main, 9 Jan II)
A. Benzaldehyde P. Dynamic phase 22. The coagulating power of electrolytes having ions Na + , Al 3+
and Ba 2+ for arsenic sulphide sol increases in the order
B. Alumina Q. Adsorbent (2013 Main)
C. Acetonitrile R. Adsorbate (a) Al 3+ < Ba 2+ < Na +
(a) (A) → (R) ; (B) → (Q); (C) → (P) (b) Na + < Ba 2+ < Al 3+
(b) (A) → (P); (B) → (R); (C) → (Q)
(c) Ba 2+ < Na 2+ < Al 3+
(c) (A) → (Q); (B) → (P); (C) → (R)
(d) (A) → (Q); (B) → (R); (C) → (P) (d) Al 3+ < Na + < Ba 2+
202 Surface Chemistry

23. Among the electrolytes Na 2 SO4 , CaCl 2 , Al 2 (SO4 )3 and 31. The given graph/data I, II, III and IV represent general
NH4 Cl, the most effective coagulating agent for Sb 2 S3 sol is trends observed for different physisorption and
(2009, 1M) chemisorption processes under mild conditions of
(a) Na 2SO4 (b) CaCl 2 temperature and pressure. Which of the following
(c) Al 2 (SO4 )3 (d) NH4Cl choice(s) about I, II, III and IV is (are) correct? (2012)
24. Among the following, the surfactant that will form micelles in p constant
aqueous solution at the lowest molar concentration at ambient p constant

Amount of gas

Amount of gas
conditions, is

absorbed
(2008, 3M)

absorbed
(a) CH3 (CH2 )15 N+ (CH3 )3 Br − (I) (II)
(b) CH3 (CH2 )11 OSO3− Na +
(c) CH3 (CH2 )6 COO− Na + T T
(d) CH3 (CH2 )11 N+ (CH3 )3 Br −
25. Lyophilic sols are

Potential energy
(2005, 1M)

Amount of gas
200 K
(a) irreversible sols

absorbed
250 K Eads
(b) prepared from inorganic compounds (III) (IV) 0
Distance of molecule
(c) coagulated by adding electrolytes from the surface
(d) self-stabilising Dhads = 150 kJ mol–1
p
26. Spontaneous adsorption of a gas on solid surface is an
exothermic process, because (2004, 1M) (a) I is physisorption and II is chemisorption
(a) ∆H increases for system (b) I is physisorption and III is chemisorption
(b) ∆S increases for gas (c) IV is chemisorption and II is chemisorption
(c) ∆S decreases for gas (d) IV is chemisorption and III is chemisorption
(d) ∆G increases for gas 32. Choose the correct reason(s) for the stability of the
27. Rate of physisorption increases with (2003, 1M) lyophobic colloidal particles. (2012)
(a) decrease in temperature (b) increase in temperature (a) Preferential adsorption of ions on their surface from the
(c) decrease in pressure (d) decrease in surface area solution
28. When the temperature is increased, surface tension of water (b) Preferential adsorption of solvent on their surface from the
(2002, 1M) solution
(a) increases (b) decreases (c) Attraction between different particles having opposite
(c) remains constant (d) shows irregular behaviour charges on their surface
Objective Questions II (d) Potential difference between the fixed layer and the
diffused layer of opposite charges around the colloidal
(One or more than one correct option) particles
29. The correct statement(s) about surface properties is(are)
(2017 Adv.)
33. The correct statement(s) pertaining to the adsorption of a
gas on a solid surface is (are) (2011)
(a) The critical temperatures of ethane and nitrogen are 563 K and
126 K, respectively. The adsorption of ethane will be more than (a) Adsorption is always exothermic
that of nitrogen of same amount of activated charcoal at a given (b) Physisorption may transform into chemisorption at high
temperature temperature
(b) Cloud is an emulsion type of colloid in which liquid is dispersed (c) Physisorption increases with increasing temperature but
phase and gas is dispersion medium chemisorption decreases with increasing temperature
(c) Adsorption is accompanied by decrease in enthalpy and decrease (d) Chemisorption is more exothermic than physisorption,
in entropy of the system however it is very slow due to higher energy of activation
(d) Brownian motion of colloidal particles does not depend on the
size of the particles but depends on viscosity of the solution Numerical Answer Type Questions
30. When O2 is adsorbed on a metallic surface, electron transfer 34 The mass of gas adsorbed, x per unit mass of adsorbate, m
occurs from the metal to O 2 . The true statement(s) regarding was measured at various pressures, p. A graph between
x
this adsorption is (are) (2015 Adv.) log and log p gives a straight line with slope equal to 2
(a) O2 is physisorbed
m
x
(b) heat is released and the intercept equal to 0.4771. The value of at a
m
(c) occupancy of π ∗ 2 p of O2 is increased
pressure of 4 atm is (Given, log 3 = 0.4771)
(d) bond length of O2 is increased (2020 Main, 2 Sep I)
Surface Chemistry 203

Assertion and Reason (c) Statement I is true; Statement II is false.


(d) Statement I is false; Statement II is true.
Read the following questions and answer as per the
direction given below: 35. Statement I Micelles are formed by surfactant molecules
(a) Statement I is true; Statement II is true; Statement II is a above the critical micelle concentration (CMC).
correct explanation of Statement I. Statement II The conductivity of a solution having surfactant
(b) Statement I is true; Statement II is true; Statement II is molecules decreases sharply at the CMC. (2007)
not the correct explanation of Statement I.

Answers
1. (a) 2. (c) 3. (b) 4. (d) 21. (b) 22. (b) 23. (c) 24. (a)
5. (b) 6. (c) 7. (c) 8. (d) 25. (d) 26. (c) 27. (a) 28. (b)
9. (a) 10. (d) 11. (d) 12. (a) 29. (a, c) 30. (b, c, d) 31. (a, c) 32. (a, d)
13. (a) 14. (d) 15. (b) 16. (a) 33. (a, b, d) 34. (48) 35. (b)
17. (b) 18. (c) 19. (b) 20. (b)

Hints & Solutions


1. Here, same metal ion, M n+ form three homoleptic octahedral 4. The explanation of the given statements are as follows :
complexes (i), (ii) and (iii) on separate combination with (a) Colloidal medicines are more effective because they (dispersed
È È
three mono-dentate ligands–N CS,F and NH3 respectively. phase) have larger surface area.
Thus, option (a) is incorrect.
So, we have to compare their CFSE (∆ 0 ) as well as
wavelength (λ) values, where (b) Brownian motion of dispersed phase particles in colloidal
hc 1 solution is faster if the viscosity of the solution is very low.
∆E(CFSE) = or, ∆ 0 ∝ Thus, option (b) is incorrect.
λ λ
(c) Addition of alum(K 2SO4 ⋅ Al2(SO4 )3 ⋅ 24H2O), an electrolyte
Again, ∆ 0 value will depend on power of ligand as placed in to water makes it fit for drinking purposes because alum
spectrochemical series.
coagulates mud particles from water.
1 Thus, option (c) is incorrect.
Power of ligand ∝ ∆ 0 ∝ ,
λ (d) Precipitation of lyophobic solution particles by electrophoresis
− − is called cottrell precipitation.
F − < NCS− < NH3, F >NC S>NH3
Thus, option (d) is correct.
→ Power of Ligand ← λ
5. For physisorption or physical adsorption,
→ ∆ 0
Adsorption isotherm (Temperature, T = constant) is shown below:
So, for the given complexes, it is evidented from the plot
(Absorption vs λ)
λ (max
ii )
> λ (max
i)
> λ (max
iii )
x
— x
(C ) (B ) (A ) m Moderate pressure zone, — ∝ p1/n
m
[F − ] [NCS− ] [NH 3 ]

2. Statement (c) is incorrect about colloids. Colligative p


properties such as relative lowering of vapour pressure,
elevation in boiling point, depression in freezing point and where, x = amount of adsorbate, m = amount of adsorbent,
osmotic pressure of a colloidal solution is of low order than x
the true solution at the same concentration. = degree of adsorption
m
3. Peptisation is a process of converting precipitate into 1 1
colloidal solution. This process involves the shaking of = order of the reaction, where, 0 < < 1 and so, 1< n < ∞
precipitate with the dispersion medium in the presence of n n
small amount of electrolyte. The electrolyte added is called x
Here, = Kp1/ 2,
peptising agent. m
During peptisation, the precipitate adsorbs one of the ions of x
i.e. ∝ p1/ 2
the electrolyte on its surface. This causes the development of m
positive or negative charge on precipitates, which ultimately
breakup into smaller particles of the size of a colloid.
204 Surface Chemistry

Adsorption isobar (Pressure, p = constant) The logarithm equation of Freundlich adsorption isotherm is
x 1
log = log K + log p
m n
x
— On comparing the above equation with straight line equation,
m ( y = mx + c)
we get
1
T m =slope = and c = log K
n
So, the rate of physical adsorption of the gas, increases with p From the given plot,
(when,T is constant) and decreases withT (when pis constant). y2 − y1 1 2
m= = =
x2 − x1 n 3
6. Correct match is
(A) → (iii); (B) → (i); (C) → (ii); (D) → (iv) x
∴ = Kp2 / 3
(i) TiCl4 + AlCl3 (Ziegler- Natta catalyst) is used to prepare m
polyethylene from ethene. 10. Statement given as statement (d) is incorrect. Latex is a stable
Zieglar-Natta catalyst dispersion, i.e. emulsion of polymer microparticles in an
nCH2 CH2 CH2 CH2 n aqueous medium.
Ethene Polyethylene
These microparticles belong to rubber and are negatively
(ii) V2O5 (Vanadium pentoxide) is used as catalyst to prepare charged in nature. Natural latex contains some amount of sugar,
H2SO4 from contact process. Reaction involved is resin, protein and ash as well.
V 2O 5
2SO2 ( g ) + O2 ( g ) → 2SO3 ( g ) The closest synthetic latex that can be associated with the
properties of natural latex is SBR, i.e. Styro Butane Rubber.
It is the key step in the manufacture of H2SO4.
(iii) Fe (Iron) is used as catalyst in Haber’s process for the Rest of all the statements are correct.
manufacture of ammonia. 11. Same adsorbant (charcoal in this case) at same temperature will
Fe( s ) adsorb different gases to different extent. The extent to which
N2 (g ) + 3H2 (g ) → 2NH3 (g ) gases are adsorbed is proportional to the critical temperature of
(iv) Pd (Palladium) is used to prepare ethanal. Reaction involved is gas.
PdCl 2/CuCl 2 8a
H 2C == CH 2 + O2 → CH 3CHO Q Tc =
H 2O 27Rb
This reaction is also known as Wacker’s process. where, a is the magnitude of intermolecular forces between
gaseous molecules.
7. In CuSO4 ⋅ 5H2O, one molecule of water is indirectly connected
Thus, higher the cirtical temperature more is the gas adsorbed.
to Cu. In this molecule, four water molecules form coordinate
Among the given gases, H2 has the minimum critical
bond with Cu 2+ ion while one water molecule is associated with temperature, i.e. 33K thus, it shows least adsorption on a definite
H-bond with SO2− 4 . amount of charcoal.
Structure of CuSO4 ⋅ 5H2 O
12. Dispersed Dispersion Type of Examples
H H phase medium colloid
δ– δ+ Liquid Solid Gel Cheese (C), butter,
O O H H O– O
H jellies
Cu2+ O S
H δ– δ+ Liquid Liquid Emulsion Milk (M), hair cream
O O H H O– O
Solid Gas Aerosol Smoke (S), dust
H H Thus, C : liquid in solid, M : liquid in liquid and S : solid in gas.
[Cu(H 2O)4 ] SO4 ⋅ H 2O 13. Solid sol consists of solid as both dispersed phase and dispersion
medium. In gemstones, metal crystals (salt and oxides of metals)
8. The aerosol is a kind of colloid in which solid is dispersed in gas. are dispersed in solid (stone) medium. Hair cream is an emulsion
e.g. smoke, dust.
(liquid in liquid). Butter is a colloidal solution of liquid in solid.
9. Paint is also sol (solid in liquid).
Key Idea According to Freundlich,
x 14. Haemoglobin and gold sol both are colloids and always carry an
= Kp1/n [ n > 1]
m electric charge. Haemoglobin is a positively charged sol,
where, m = mass of adsorbent, x = mass of the gas because in haemoglobin, Fe2+ ion is the central metal ion of the
x octahedral complex.
adsorbed, = amount of gas adsorbed per unit mass of
m All metal sols like, Au-sol, Ag-sol etc; are negatively charged
solid adsorbent, p = pressure, K and n = constants. sols.
Surface Chemistry 205

15. In heterogeneous catalytic reactions, physical state of reactants (i) The diameter of the colloids should not be much smaller than
and that of catalyst(s) used are different. the wavelength of light used.
Haber’s process, hydrogenation of vegetable oils and Ostwald’s (ii) The refractive indices of the dispersed phase and dispersion
process all are heterogeneous process. Combustion of coal is not medium should differ greatly in magnitude.
a heterogeneous catalytic reaction. x
l
In Haber’s process 20. According to Freundlich adsorption isotherm, = kp1/ n
m
Fe( s), Mo( s)
N 2 ( g ) + 3H 2( g ) → 2NH 3 ( g )
Slope = 1
l
Hydrogenation of vegetable oils, θ n

log x/m
[(Ph P) Rh] Cl
Vegetable oil (l ) 
3 3
→ Vanaspati (s)
(Unsaturated) or Ni ( s)
log k
l
Ostwald’s process,
Pt( s) log p
4NH3 (g ) + 5O 2 (g ) → 4NO(g ) + 6H2O(g )
V 2 O5 (s)
On taking logarithm of both sides, we get
l
No catalyst is used in combustion of coal. The reaction is x
log = log k + log p1 / n
highly spontaneous in nature. m
C + O2 → CO2 x 1
(Coal) or log = log k + log p
16. Using the principle of adsorption chromatography, qualitative m n
and quantitative analysis of benzaldehyde can be done from its y = c + mx
mixture with acetonitrile. Here, a mobile phase moves over a x
y = log ,
stationary phase (adsorbent). Adsorbents used are alumina m
(Al 2O 3 ) and silica gel. The sample solution of benzaldehyde and c = intercept = log k
acetonitrile when comes in contact with the adsorbent, 1
benzaldehyde gets adsorbed on the surface of the adsorbent. So, m = slope = and x = log p
n
benzaldehyde acts as absorbate whereas acetonitrile starts
moving as mobile phase over the stationary phase of the 21. Physical adsorption takes place with decrease in enthalpy thus
adsorbate. Hence, act as dynamic phase. exothermic change. It is physical adsorption and does not require
2− activation. Thus, (a) is incorrect.
17. Arsenious sulphide sol is a negative colloid, As2S3.(S ). So, it
Being physical adsorption ∆H < 0 thus, (b) is correct. Exothermic
will be coagulated by the cation of an electrolyte. reaction is favoured at low temperature thus (c) is incorrect.
According to the Hardy-Schulze rule, the higher the charge of the Physical adsorption is always reversible, thus (d) is incorrect.
ion, the more effective it is in bringing about coagulation. Here,
22. According to Hardy Schulze rule, greater the charge on
the cations available are Al 3+ (from AlCl 3), Ba 2+ (from BaCl 2)
oppositely charged ion, greater is its coagulating power. Since
and Na + (from Na 3PO 4 and NaCl). So, their power to coagulate arsenic sulphide is a negatively charged sol, thus, the order of
As2S3. (S2− ) will follow the order as coagulating power is Na + < Ba 2+ < Al 3+ .
Al 3+ > Ba 2+ > Na+ 23. Sb2S3 is a negative (anionic) sol. According to Hardy Schulze
rule, greater the valency of cationic coagulating agent, higher its
18. According to Freundlich adsorption isotherm,
coagulating power. Therefore, Al 2 (SO4 )3 will be the most
x x
∝ p1/ n ⇒ = Kp1/ n effective coagulating agent in the present case.
m m
24. Larger the hydrophobic fragment of surfactant, easier will be the
On taking log on both sides, we get micellisation, smaller the crticial micelle concentration.
 x 1 Therefore, CH3 (CH2 )15 N+ (CH3 )3 Br − will have the lowest
log   = log K + log p
 m n crticial micelle concentration.
On comparing with equation of straight line, y = mx + c, plot of 25. Lyophilic sols are reversible, not easily coagulated because it is
x self-stabilising.
log vs log p gives,
m 26. ∆G = ∆H − T∆S
( y2 − y1 ) 1 2 1
Slope = ⇒ = As gas is adsorbed on surface of solid, entropy decreases, i.e.
( x2 − x1 ) n 4 2 ∆S < 0. Therefore, for ∆G < 0, ∆H must be negative.
x 1/ 2
∴ ∝p 27. It is an exothermic process, according to Le-Chatelier’s principle,
m lowering temperature drive the process in forward direction.
19. Colloidal solutions show Tyndall effect due to scattering of light 28. As temperature increases surface tension of liquid decreases.
by colloidal particles in all directions in space. It is observed 29. (a) Higher the critical temperature, greater the extent of
only under the following conditions. adsorption.
(c) P (s) + Q (g ) → PQ (s)
Adsorbent Adsorbate
206 Surface Chemistry

(c) It is wrong statement as at higher temperature, physically


As gaseous adsorbate is adsorbed on solid surface, entropy adsorbed substance starts desorbing.
decreases, ∆S < 0. Also formation of bond between P and Q
(d) In physical adsorption, van der Waals' force hold the
results in release of energy, hence ∆H < 0.
adsorbate and adsorbent together which is a weak
30. Since, adsorption involves electron transfer from metal to O2, it electrostatic attraction. In chemisorption, strong chemical
is chemical adsorption not physical adsorption, hence (a) is bond binds the adsorbate to the adsorbent. Therefore,
incorrect. Adsorption is spontaneous which involves some chemisorption is more exothermic than physical adsorption.
bonding between adsorbent and adsorbate, hence exothermic.
The last occupied molecular orbital in O2 is π * 2p. Hence, 34. From Freundlich adsorption isotherm equation,
electron transfer from metal to oxygen will increase occupancy x 1
log = log K + log p
of π * 2p molecular orbitals. Also increase in occupancy of π * 2p m n
orbitals will decrease bond order and hence increase bond length
of O2.
31. Graph-I represents physisorption as in physisorption, absorbents x θ
are bonded to adsorbate through weak van der Waals’ force. log m
Increasing temperature increases kinetic energy of adsorbed
particles increasing the rate of desorption, hence amount of
adsorption decreases.
log p
Graph-II represents chemisorption as it is simple activation
energy diagram of a chemical reaction. When we plot log x / m vs log p, we get a straight line of
Graph-III also represents physical adsorption as extent of 1 1
adsorption increasing with pressure. (i) slope = = 2 ⇒ n =
n 2
Graph-IV represents chemisorption as it represents the potential
energy diagram for the formation of a typical covalent bond. (ii) intercept = log K = 0.4771
32. Lyophobic sol, which is otherwise unstable, gets stabilised by ⇒ log K = log 3 ⇒ K = 3
preferential adsorption of ions on their surface, thus developing a x
potential difference between the fixed layer and the diffused So, = Kp1/ n = 3 × 4 2 = 48.00 (Q p = 4 atm)
m
layer. Thus, option (a) and (d) are correct.
35. Both statements are independently correct but Statement II does
33. (a) In the process of adsorption, a bond is formed between not explain Statement I. Critical micelle concentration is the
adsorbate and adsorbent, hence always exothermic. minimum concentration of surfactant at which micelle formation
(b) Physisorption require very low activation energy while commences first. At critical micelle concentration, several
chemisorption require high activation energy. Therefore a molecules of surfactant coalesce together to form one single
physisorption may transform into chemisorption but only at micelle molecule. This decreases the apparent number of
high temperature. molecule suddenly lowering conductivity sharply.
14
s-Block Elements
4. HF and CH4 are called as molecular hydrides.
Topic 1 Group I Elements (a) (1), (2), (3) and (4) (b) (1), (2) and (3) only
(c) (3) and (4) only (d) (1), (3) and (4) only
Objective Questions I (Only one correct option)
8. The strength of 11.2 volume solution of H 2O2 is [Given that
1. Which of the following liberates O 2 upon hydrolysis? molar mass of
(2020 Adv.)
H = 1g mol −1 and O = 16 g mol −1 ] (2019 Main, 8 April II)
(a) Pb3O4 (b) KO2
(c) Na 2O2 (d) Li 2O2 (a) 1.7% (b) 34% (c) 13.6% (d) 3.4%

2. Dihydrogen of high purity (> 99.95%) is obtained through 9. The correct order of hydration enthalpies of alkali metal ions
(2020 Main, 6 Sep II) is (2019 Main, 8 April I)
(a) the reaction of Zn with dilute HCl (a) Li + > Na + > K + > Cs+ > Rb+
(b) the electrolysis of acidified water using Pt electrodes (b) Na + > Li + > K + > Rb+ > Cs+
(c) the electrolysis of brine solution (c) Na + > Li + > K + > Cs+ > Rb+
(d) the electrolysis of warm Ba(OH)2 solution using Ni electrodes. (d) Li + > Na + > K + > Rb+ > Cs+
3. In the following reactions, products (A ) and ( B ), 10. The correct statement(s) among I to III with respect to
respectively, are (2020 Main, 7 Jan II) potassium ions that are abundant within the cell fluids is/are
NaOH + Cl2 → ( A ) + side products (hot and conc.) (2019 Main, 12 Jan II)

Ca(OH)2 + Cl2 → ( B ) + side products (dry) I. They activate many enzymes.


(a) NaClO3 and Ca(OCl)2 (b) NaClO3 and Ca(ClO3 )2 II. They participate in the oxidation of glucose to produce
(c) NaOCl and Ca(OCl)2 (d) NaOCl and Ca(ClO3 )2 ATP.
4. The temporary hardness of a water sample is due to III. Along with sodium ions, they are responsible for the
compound X . Boiling this sample converts X to compound transmission of nerve signals.
Y . X and Y , respectively, are (2019 Main, 12 April II) (a) I, and III only (b) I, II and III
(a) Mg(HCO3 )2 and Mg(OH)2 (c) I and II only (d) III only
(b) Ca(HCO3 )2 and Ca(OH)2 11. A metal on combustion in excess air forms X . X upon
(c) Mg(HCO3 )2 and MgCO3
hydrolysis with water yields H2 O2 and O2 along with another
(d) Ca(HCO3 )2 and CaO
product. The metal is (2019 Main, 12 Jan I)
5. The incorrect statement is (2019 Main, 12 April II) (a) Li (b) Mg (c) Rb (d) Na
(a) lithium is the strongest reducing agent among the alkali metals.
12. The hardness of a water sample (in terms of equivalents of
(b) lithium is least reactive with water among the alkali metals.
CaCO3 ) containing is
(c) LiNO3 decomposes on heating to give LiNO2 and O2.
(d) LiCl crystallise from aqueous solution as LiCl ⋅ 2H2O.
(Molar mass of CaSO4 = 136 g mol −1 ) (2019 Main, 12 Jan I)
(a) 100 ppm (b) 10 ppm (c) 50 ppm (d) 90 ppm
6. The metal that gives hydrogen gas upon treatment with both
acid as well as base is (2019 Main, 12 April I) 13. The hydride that is not electron deficient is
(a) magnesium (b) mercury (2019 Main, 11 Jan II)
(c) zinc (d) iron (a) AlH3 (b) B2H6 (c) SiH4 (d) GaH3

7. The correct statements among (a) to (d) are: 14. The correct statements among (a) to (d) regarding H2 as a
(2019 Main, 8 April II) fuel are : (2019 Main, 11 Jan I)
1. Saline hydrides produce H2 gas when reacted with H2O. I. It produces less pollutants than petrol.
2. Reaction of LiAlH4 with BF3 leads to B2H6.
II. A cylinder of compressed dihydrogen weights ~ 30times
3. PH3 and CH4 are electron rich and electron precise more than a petrol tank producing the same amount of
hydrides, respectively. energy.
208 s-Block Elements

III. Dihydrogen is stored in tanks of metal alloys like NaNi 5 . 24. The hottest region of Bunsen flame shown in the figure given
IV. On combustion, values of energy released per gram of below is (2016 Main)
liquid dihydrogen and LPG are 50 and 142 kJ, Region 4
respectively. Region 3
(a) I, II and III only (b) II, III and IV only Region 2
Region 1
(c) II and IV only (d) I and III only
15. NaH is an example of (2019 Main, 11 Jan I)
(a) metallic hydride (b) electron-rich hydride
(c) saline hydride (d) molecular hydride (a) region 2 (b) region 3 (c) region 4 (d) region 1
16. Sodium metal on dissolution in liquid ammonia gives a deep 25. Which one of the following statements about water is false?
blue solution due to the formation of (2019 Main, 10 Jan II) (2016 Main)
(a) sodium ammonia complex (a) Water can act both as an acid and as a base
(b) sodium ion-ammonia complex (b) There is extensive intramolecular hydrogen bonding in the
(c) sodamide condensed phase
(d) ammoniated electrons (c) Ice formed by heavy water sinks in normal water
17. The total number of isotopes of hydrogen and number of (d) Water is oxidised to oxygen during photosynthesis
radioactive isotopes among them, respectively, are 26. The main oxides formed on combustion of Li, Na and K in
(2019 Main, 10 Jan I) excess of air respectively are (2016 Main)
(a) 2 and 1 (b) 3 and 2 (a) LiO2, Na 2O2 and K 2O (b) Li 2O2, Na 2O2 and KO2
(c) 2 and 0 (d) 3 and 1 (c) Li 2O, Na 2O2 and KO2 (d) Li 2O , Na 2O and KO2
18. The chemical nature of hydrogen peroxide is 27. Which of the following atoms has the highest first ionisation
(2019 Main, 10 Jan I)
energy? (2016 Main)
(a) oxidising and reducing agent in both acidic and basic medium (a) Na (b) K (c) Sc (d) Rb
(b) oxidising and reducing agent in acidic medium, but not in
basic medium 28. Hydrogen peroxide in its reaction with KIO4 and NH2 OH
(c) reducing agent in basic medium, but not in acidic medium respectively, is acting as a (2014 Adv.)
(d) oxidising agent in acidic medium, but not in basic medium (a) reducing agent, oxidising agent
(b) reducing agent, reducing agent
19. The metal that forms nitride by reacting directly with N 2 of (c) oxidising agent, oxidising agent
air, is (2019 Main, 9 Jan II) (d) oxidising agent, reducing agent
(a) Rb (b) K 29. In which of the following reactions H2 O2 acts as a reducing
(c) Cs (d) Li agent? (2014 Main)
20. What is reason of temporary hardness of water? I. H2O2 + 2H+ + 2e− → 2H2O
(2019 Main, 9 Jan II)
II. H2O2 − 2e− → O2 + 2H+
(a) Na 2SO 4 (b) CaCl 2
(c) NaCl (d) Ca(HCO 3) 2 III. H2O2 + 2e− → 2OH−
21. The isotopes of hydrogen are (2019 Main, 9 Jan I) IV. H2O2 + 2OH− − 2e− → O2 + 2H2O
(a) deuterium and tritium only (a) I and II (b) III and IV (c) I and III (d) II and IV
(b) protium and deuterium only
(c) protium, deuterium and tritium 30. A sodium salt of an unknown anion when treated with MgCl 2
(d) tritium and protium only gives white precipitate only on boiling. The anion is
(2004, 1M)
22. Hydrogen peroxide oxidises [Fe(CN)6 ]4− to [Fe(CN)6 ]3− in
(a) SO2−
4 (b) HCO−3 (c) CO2−
3 (d) NO−3
acidic medium but reduces [Fe(CN)6 ]3− to [Fe(CN)6 ]4− in
31. A dilute aqueous solution of Na 2 SO4 is electrolysed using
alkaline medium. The other products formed are, platinum electrodes. The products at the anode and cathode
respectively. (2018 Main) are respectively (1996, 1M)
(a) (H2O + O2 ) and H2O (a) O2 , H2 (b) S2O2– (c) O2 , Na (d) S2O82– , H2
8 , Na
(b) (H2O + O2 ) and (H2O + OH− )
(c) H2O and (H2O + O2 ) 32. Hydrolysis of one mole of peroxodisulphuric acid produces
(d) H2O and (H2O + OH− ) (a) two moles of sulphuric acid (1996, 1M)
(b) two moles of peroxomono sulphuric acid
23. Both lithium and magnesium display several similar (c) one mole of sulphuric acid and one mole of peroxomono
properties due to the diagonal relationship; however, the one sulphuric acid
which is incorrect is (2017 Main) (d) one mole of sulphuric acid, one mole of peroxomono sulphuric
(a) Both form basic carbonates acid and one mole of hydrogen peroxide
(b) Both form soluble bicarbonates
(c) Both form nitrides 33. The species that do not contain peroxide ions, is (1992, 1M)
(d) nitrates of both Li and Mg yield NO2 and O2 on heating (a) PbO2 (b) H2O2 (c) SrO2 (d) BaO2
s-Block Elements 209

34. The metallic lustre exhibited by sodium metal is explained (b) the lattice energy of barium sulphate is more than its
by (1987, 1M) hydration energy
(a) diffusion of sodium ions (c) the lattice energy has no role to play in solubility
(b) oscillation of loose electron (d) the hydration energy of sodium sulphate is less than its
(c) excitation of free protons lattice energy
(d) existence of body centred cubic lattice
35. A solution of sodium sulphate in water is electrolysed using Assertion and Reason
inert electrodes. The products at cathode and anode are Read the following questions and answer as per the direction given
respectively (1987, 1M) below :
(a) H2 , O2 (b) O2 , H2 (c) O2 , Na (d) O2 , SO2 (a) Statement I is correct; Statement II is correct; Statement
36. Nitrogen dioxide cannot be obtained by heating (1985, 1M) II is the correct explanation of Statement I
(a) KNO3 (b) Pb(NO3 )2 (c) Cu(NO3 )2 (d)AgNO3 (b) Statement I is correct; Statement II is correct; Statement
II is not the correct explanation of Statement I
37. The oxide that gives H2 O2 on treatment with a dilute acid is
(c) Statement I is correct; Statement II is incorrect
(a) PbO2 (b) Na 2O2 (1985, 1M)
(d) Statement I is incorrect; Statement II is correct
(c) MnO2 (d) TiO2
48. Statement I Alkali metals dissolve in liquid ammonia to give
38. Molecular formula of Glauber’s salt is (1985, 1M)
blue solution.
(a) MgSO4 ⋅ 7H2O (b) CuSO4 ⋅ 5H2O
(c) FeSO4 ⋅ 7H2O (d) Na 2SO4 ⋅ 10H2O Statement II Alkali metals in liquid ammonia give solvated
species of the type [ M ( NH 3 )n ]+ (M = alkali metals).
39. Heavy water is (1983, 1M)
(2007, 3M)
(a) H2O18
(b) water obtained by repeated distillation 49. Statement I LiCl is predominantly a covalent compound.
(c) D2O Statement II Electronegativity difference between Li and
(d) water at 4°C Cl is too small. (1998, 2M)
40. A solution of sodium metal in liquid ammonia is strongly 50. Statement I The alkali metals can form ionic hydrides
reducing due to the presence of (1981, 1M) which contain the hydride ion, H− .
(a) sodium atoms (b) sodium hydride
Statement II The alkali metals have low electronegativity,
(c) sodium amide (d) solvated electrons
their hydrides conduct electricity when fused and liberate
41. The temporary hardness of water is due to calcium hydrogen gas at the anode. (1994, 2M)
bicarbonate can be removed by adding (1979, 1M)
(a) CaCO3 (b) Ca(OH)2 (c) CaCl 2 (d) HCl Fill in the Blanks
51. Hydrogen gas is liberated by the action of aluminium with
Objective Questions II concentrated solution of ……… . (1987, 1M)
(One or more than one correct option) 52. Sodium dissolved in liquid ammonia conducts electricity
42. The pair(s) of reagents that yield paramagnetic species is/are because of …… (1985, 1M)
(a) Na and excess of NH3 (b) K and excess of O2
(c) Cu and dilute HNO3 (d) O2 and 2-ethylanthraquinol 53. The adsorption of hydrogen by palladium is commonly
known as …… (1983, 1M)
43. The compound(s) formed upon combustion of sodium metal
54. Iodine reacts with hot NaOH solution. The products are NaI
in excess air is (are) (2007, 2M)
and …… (1980, 1M)
(a) Na 2O2 (b) Na 2O (c) NaO2 (d) NaOH
44. Sodium nitrate decomposes above ≈ 800°C to give True/False
(a) N2 (b) O2 (1998, 2M)
55. Sodium when burnt in excess of oxygen gives sodium oxide.
(c) NO2 (d) Na 2 O
(1987, 1M)
45. Highly pure dilute solution of sodium in liquid ammonia
(a) shows blue colour (1998, 2M)
Subjective Questions
(b) exhibits electrical conductivity 56. A white solid is either Na 2 O or Na 2 O2 . A piece of red litmus
(c) produces sodium amide paper turns white when it is dipped into a freshly made
(d) produces hydrogen gas aqueous solution of the white solid.
46. When zeolite, which is hydrated sodium aluminium silicate, (i) Identify the substance and explain with balanced
is treated with hard water, the sodium ions are exchanged equation.
with (1990, 1M) (ii) Explain what would happen to the red litmus if the white
(a) H+ ions (b) SO2–
4 ions (c) Mg2+ ions (d) OH− ions solid were the other compound. (1999, 4M)

47. Sodium sulphate is soluble in water, whereas barium 57. Hydrogen peroxide acts both as an oxidising and as a
sulphate is sparingly soluble because (1989, 1M) reducing agent in alkaline solution towards certain first row
(a) the hydration energy of sodium sulphate is more than its transition metal ions. Illustrate both these properties of H2 O2
lattice energy using chemical equations. (1998, 4M)
210 s-Block Elements

58. Element A burns in nitrogen to give an ionic compound B. 60. Give reasons in one or two sentences for the following:
Compound B reacts with water to give C and D. A solution of “H2 O2 is a better oxidising agent than H2 O.” (1986, 1M)
C becomes ‘milky’ on bubbling carbon dioxide gas. Identify
A, B, C and D. (1997, 3M)
61. Sodium carbonate is prepared by Solvay process but the
same process is not extended to the manufacture of
59. Complete and balance the following chemical reaction. potassium carbonate, explain. (1981, 1M)
Anhydrous potassium nitrate is heated with excess of
metallic potassium
62. Water is a liquid, while H2 S is a gas at ordinary temperature.
KNO3 ( s ) + K ( s ) → K + K (1992, 1M) Explain . (1978, 1M)

Topic 2 Group II Elements


Objective Questions I (Only one correct option) 7. Magnesium powder burns in air to give (2019 Main, 9 April I)
1. Match the following compounds (Column -I) with their uses (a) MgO and Mg 3N2 (b) Mg (NO3 )2 and Mg 3N2
(Columns -II). (2020 Main, 6 Sep II) (c) MgO only (d) MgO and Mg (NO3 )2

Column -I Column - II
8. The covalent alkaline earth metal halide ( X = Cl, Br, I) is
(2019 Main, 8 April II)
(I) Ca(OH)2 (A) Casts of statues (a) SrX 2 (b) CaX 2 (c) MgX 2 (d) BeX 2
(II) NaCl (B) White wash 9. Match the following items in Column I with the corresponding
(III) 1 (C) Antacid items in Column II. (2019 Main, 11 Jan II)
CaSO 4 ⋅ H2O
2
Column I Column II
(IV) CaCO 3 (D) Washing soda preparation
(i) Na 2CO3 ⋅ 10H2O A. Portland cement
(a) (I)-(D), (II)-(A), (III)-(C), (IV)-(B) ingredient
(b) (I)-(B), (II)-(D), (III)-(A), (IV)-(C) (ii) Mg(HCO3 )2 B. Castner-Kellner process
(c) (I)-(B), (II)-(C), (III)-(D), (IV)-(A)
(d) (I)-(C), (II)-(D), (III)-(B), (IV)-(A)
(iii) NaOH C. Solvay process
(iv) Ca 3Al 2O6 D. Temporary hardness
2. In comparison to boron, berylium has (2019 Main, 12 April II)
(a) lesser nuclear charge and lesser first ionisation enthalpy (a) (i) - (D); (ii) - (A); (iii) - (B); (iv) - (C)
(b) greater nuclear charge and lesser first ionisation enthalpy (b) (i) - (B); (ii) - (C); (iii) - (A); (iv) - (D)
(c) greater nuclear charge and greater first ionisation enthalpy (c) (i) - (C); (ii) - (B); (iii) - (D); (iv) - (A)
(d) lesser nuclear charge and greater first ionisation enthalpy (d) (i) - (C); (ii) - (D); (iii) - (B); (iv) - (A)

3. The correct sequence of thermal stability of the following 10. The amphoteric hydroxide is (2019 Main, 11 Jan I)
carbonates is (2019 Main, 12 April I) (a) Be(OH)2 (b) Ca(OH)2 (c) Sr(OH)2 (d) Mg(OH)2
(a) BaCO3 < CaCO3 < SrCO3 < MgCO3 11. The metal used for making X-ray tube window is
(b) MgCO3 < CaCO3 < SrCO3 < BaCO3 (2019 Main, 10 Jan I)
(c) MgCO3 < SrCO3 < CaCO3 < BaCO3 (a) Na (b) Be (c) Mg (d) Ca
(d) BaCO3 < SrCO3 < CaCO3 < MgCO3
12. The alkaline earth metal nitrate that does not crystallise with
4. A hydrated solid X on heating initially gives a water molecules, is (2019 Main, 9 Jan I)
monohydrated compound Y . (a) Ca(NO3 )2 (b) Sr(NO3 )2
Y upon heating above 373 K leads to an anhydrous white (c) Ba(NO3 )2 (d) Mg(NO3 )2
powder Z. X and Z, respectively, are (2019 Main, 10 April II) 13. Which one of the following alkaline earth metal sulphates
(a) baking soda and soda ash
has its hydration enthalpy greater than its lattice enthalpy?
(b) washing soda and soda ash
(a) CaSO 4 (b) BeSO 4 (2015 Main)
(c) baking soda and dead burnt plaster
(c) BaSO 4 (d) SrSO 4
(d) washing soda and dead burnt plaster
5. The alloy used in the construction of aircrafts is 14. The following compounds have been arranged in order of their
(2019 Main, 10 April I) increasing thermal stabilities. Identify the correct order.
(a) Mg-Zn (b) Mg-Mn K 2 CO3 (I), MgCO3 (II), CaCO3 (III), BeCO3 (IV)
(c) Mg-Sn (d) Mg-Al (a) I < II < III < IV (b) IV < II < III < I (1996, 1M)
(c) IV < II < I < III (d) II < IV < III < I
6. The structures of beryllium chloride in the solid state and
vapour phase, respectively are (2019 Main, 9 April II) 15. The oxidation state of the most electronegative element in
(a) dimeric and dimeric (b) chain and chain the products of the reaction, BaO2 with dil. H2 SO4 are
(c) dimeric and chain (d) chain and dimeric (a) 0 and –1 (b) –1 and –2 (1991, 1M)
(c) –2 and 0 (d) –2 and –1
s-Block Elements 211

16. Calcium is obtained by (1980, 1M) True/False


(a) electrolysis of molten CaCl 2
(b) electrolysis of solution of CaCl 2 in water 21. MgCl 2 ⋅ 6H2 O on heating gives anhydrous MgCl 2 . (1982, 1M)
(c) reduction of CaCl 2 with carbon
(d) roasting of limestone Subjective Questions
22. Identify (X) in the following synthetic scheme and write their
Objective Questions II structures.
(One or more than one correct option) *
Ba CO3 + H2 SO4 → X (gas) (C* denotes C14 ) (2001, 1M)
17. The reagent(s) used for softening the temporary hardness of
water is(are) (2010) 23. Give reasons for the following in one or two sentences only :
(a) Ca 3 (PO4 )2 (b) Ca(OH)2 “BeCl 2 can be easily hydrolysed.” (1999, 2M)
(c) Na 2CO3 (d) NaOCl
24. The crystalline salts of alkaline earth metals contain more
18. MgSO 4 on reaction with NH4 OH and Na 2 HPO4 forms a water of crystallisation than the corresponding alkali metal
white crystalline precipitate. What is its formula? (2006, 3M) salts. Why? (1997, 2M)
(a) Mg(NH4 )PO4 (b) Mg3 (PO4 )3
(c) MgCl 2 ⋅ MgSO4 (d) MgSO4 25. Calcium burns in nitrogen to produce a white powder which
dissolves in sufficient water to produce a gas A and an
19. The material used in solar cells contains (1993, 1M)
alkaline solution. The solution on exposure to air produces a
(a) Cs (b) Si (c) Sn (d) Ti
thin solid layer of B on the surface. Identify the compounds A
Fill in the Blank and B. (1996, 3M)

20. Anhydrous MgCl 2 is obtained by heating the hydrated salt 26. Arrange the following in increasing order of basic strength :
with …… (1980 , 1M)
MgO, SrO, K 2 O, NiO, Cs2 O (1991, 1M)

Answers
Topic 1 45. (a, b) 46. (a, d) 47. (a, b) 48. (b)
1. (b) 2. (d) 3. (a) 4. (a) 49. (c) 50. (a) 51. NaOH
5. (c) 6. (c) 7. (a) 8. (d) 52. (solvated electrons) 53. (occlusion) 54. (NaIO 3)
9. (d) 10. (b) 11. (c) 12. (a) 55. (F)
13. (c) 14. (a) 15. (c) 16. (d)
Topic 2
17. (d) 18. (a) 19. (d) 20. (d)
1. (b) 2. (d) 3. (b) 4. (b)
21. (c) 22. (c) 23. (a) 24. (a)
5. (d) 6. (c) 7. (a) 8. (d)
25. (b) 26. (c) 27. (c) 28. (a)
9. (d) 10. (a) 11. (b) 12. (c)
29. (d) 30. (b) 31. (a) 32. (c)
13. (b) 14. (b) 15. (d) 16. (a)
33. (a) 34. (b) 35. (a) 36. (a)
17. (b, c, d) 18. (a) 19. (b) 20. dry HCl
37. (b) 38. (d) 39. (c) 40. (d)
21. F
41. (b) 42. (a, b, c) 43. (a, b) 44. (a, b, d)

Hints & Solutions


Topic 1 Group I Elements
1. (a) Pb3O4 + H2O → No reaction (d) Li 2O2 + 2H2O → 2LiOH + H2O2
Pb3O4 is insoluble in water or do not react with water. The reactivity of Li 2O2 toward water differs from LiO2, in
Li 2O2 results in H2O2 as a product.
(b) KO2 + 2H2O → KOH + H2O2 + 1/2 O2
Hence, the correct option is (b).
Potassium superoxide is a strong oxidant, able to convert
oxides into peroxides or molecular oxygen. Hydrolysis gives 2. Electrolysis of warm aqueous Ba(OH)2 solution between nickel
oxygen gas, hydrogen peroxide and potassium hydroxide. electrodes is a commercial method to obtain highly pure
(> 99.95%) dihydrogen.
(c) Na 2O2 + 2H2O → 2NaOH + H2O2
In the presence of Ba(OH)2, water dissociates into ions easily
When sodium peroxide dissolves in water, it is hydrolysed
and quickly and H+ ions are produced, which go on cathode,
and forms sodium hydroxide and hydrogen peroxide. The
gets discharged there and liberate hydrogen gas.
reaction is highly exothermic.
212 s-Block Elements

3. 6 NaOH + 3Cl2 → NaClO3 + 5NaCl +3H2O ∴34 g of H2O2 is present in 1000 g of solution
(Hot and conc.) ( A) 34
∴% w/w = × 100 = 3.4%
2Ca(OH)2 + 2Cl2 → Ca(OCl)2 +CaCl2 +2H2O 1000
(Dry) ( B)

Thus, A : NaClO3 9. Key Idea The amount of energy released when one mole of
gaseous ions combine with water to form hydrated ions is
B : Ca (OCl)2
called hydration enthalpy.
4. The temporary hardness of a water sample is due to compound
X [i.e. Mg(HCO3 )2]. Boiling of this sample converts X [i.e. The correct order of hydration enthalpies of alkali metal ions is
Mg(HCO3 )2] to compound Y[i.e. Mg(OH)2 ]. Generally, Li + > Na + > K+ > Rb+ > Cs+
temporary hardness is due to presence of magnesium and Li+ possesses the maximum degree of hydration due to its small
calcium hydrogen carbonates. It can be removed by boiling. size. As a consequence of hydration enthalpy, their mobility
During boiling, the soluble Mg(HCO3 )2 is converted into also get affected. Cs+ has highest and Li + has lowest mobility in
insoluble Mg(OH)2 and Ca(HCO3 )2 changed to insoluble aqueous solution.
CaCO3 . These precipitates can be removed by filteration.
Heating
10. All the statements are correct. K+ being metallic unipositive
Mg(HCO3 )2  
→ Mg(OH)2 ↓ + 2CO2 ↑ ions work as enzyme activators. These also participate in many
Heating reactions of glycolysis and Kreb’s cycle to produce ATP from
Ca(HCO3 )2 → CaCO3 ↓ + H2O + CO2 ↑
glucose.
5. Statement (c) is incorrect. LiNO3(Lithium nitrate) on heating Being unipositive these are also equally responsible for nerve
gives a mixture of Li 2O, NO2 and O2. signal transmission along with Na + . (Na + ion-pump theory)

4LiNO3 → 2Li 2O + 4NO2 ↑ + O2 ↑ 11. Metal (A) is rubidium (Rb). In excess of air, it forms RbO2( X ).
Among the alkali metals, lithium is the strongest reducing agent. X is a superoxide that have O−2 ion. It is due to the stabilisation
6. Metal that gives hydrogen gas upon treatment with both acid as of large anion by large cations through lattice energy effects.
well as base is zinc. Hence, it is amphoteric in nature. RbO2 (X ) gets easily hydrolysed by water to form the
Reactions involved are as follows: hydroxide, H2O2 and O2.
Zn + Dil. NaOH → Na 2 ZnO2 + H2 ↑ The reaction involved are as follows:
Zn + 2HCl(dil.) → ZnCl 2 + H2 ↑ Rb + O2 → RbO2 (superoxide)
(X)
7. The explanation of given statements are as follows : 2RbO2 + 2H2O → 2RbOH + H2O2 + O2
1. Saline or ionic hydrides produce H2 with H2O. (X)
⊕ È
M H + H2O → H2 ↑ + MOH 12. Hardness of water sample can be calculated in terms of ppm
Thus, statement (1) is correct. concentration of CaCO3.
Ether
2. 3LiAlH4 + 4BF3 → 2B2H6 + 3LiF + 3AlF3 Given, molarity = 10−3M
(Diborane)
i.e. 1000 mL of solution contains 10−3 mole of CaCO3.
Thus, statement (2) is correct.
3. PH3 and CH4 are covalent hydrides and in both of the ∴Hardness of water = ppm of CaCO3
hydrides, octet of P and C have been satisfied. But P in PH3 10−3 × 1000
= × 106 = 100ppm
has one lone pair of electrons and C in CH4 does not have so 1000
PH3 (group 15) and CH4 (group 14) are electron rich and 13. GaH3, AlH3 and B2H6 are the hydrides of group-13 (ns2np1 ),
electron precise hydrides, respectively.
whereas SiH4 is an hydride of group 14.
Thus, statement (3) is correct.
H H
4. HF and CH4 are called as molecular hydrides because of
sp2 sp2
their discrete and sterically symmetrical structure. Ga Al
H H H H
Thus, statement (4) is also correct.
6e – around 6e – around
8. 11.2 volume of H2O2 means that 1 mL of this H2O2 will give Ga 3+ <8e – (octet) Al 3+ <8e – (octet)
(GaH3) (AlH3)
11.2 mL of oxygen at STP.
3c–2e – bridge bond
2H2O2 (l ) → O2 (g ) + 2H2O(l ) H
H
sp3 H
2 × 34 g 22.4 L at STP B B
H H
22.4 L of O2 at STP is produced from H2O2 = 68 g H
B is sp3, but cannot satisfy octet due
∴ 11.2 L of O2 at STP is produced from to the attachment of two 3c-2e –
68 bridge bonds (B–H–B)
H2O2 = × 112
. = 34 g
22.4 So, B2H6, AlH3 and GaH3 are electron deficient hydrides.
s-Block Elements 213

But, SiH4 is an electron precise hydride of group-14 (ns2np2 ), i.e. Only tritium (T) is radioactive, because of its very high
n
value,
these hydrides can have the required number of electrons to write p
their conventional Lewis structures. n 
 = 2 .
H p 
8e– around Si (Octet gets satisfied)
Si 18. H2O2 can act as both oxidising and reducing agents in both
H
H H acidic and basic medium.
H2 O2 as oxidising agent
14. (I) H2 is a 100% pollution free fuel. So, statement (I) is correct. l
In acidic medium: H2 O2 + 2H+ + 2e− → 2H2 O
(II) Molecular weight of H2(2u).

=
1
× molecular weight of butane,
l
In basic medium : H2 O2 + 2O H + 2e− → 2H2 O + 2O2 −
29
H2 O2 as reducing agent
C 4H10 (LPG) [58u].
So, compressed H2 weighs ~30 times more than a petrol tank
l
In acidic medium : H2 O2 → O2 + 2H+ + 2e−
and statement (II) is correct. In basic medium :
(III) NaNi 5, Ti - TiH2 etc. are used for storage of H2 in small H2O2 + 2OH− → O2 + 2H2O + 2e−
quantities. Thus, statement (III) is correct. 19. Among the group-1 metals, only Li is able to form its nitride,
(IV) On combustion values of energy released per gram of liquid Li 3N. [All alkaline earth metals of group-2 form their nitride,
dihydrogen (H2 ) : 142 kJ g −1, and for LPG : 50 kJ g −1. So, M 3N2]
staement (IV) is incorrect. 6Li + N 2 → 2Li 3N (Ruby red solid)
r s (Air)
15. Na H is an example of ionic or saline hydride. These hydrides are I II
formed when hydrogen combines with metals having less [ 3M + N 2 → M 3N 2 ]
electronegativity and more electropositive character with Li ⊕ is the smallest metal ion of group-1. Smaller size of Li ⊕ and
respect to hydrogen. larger size of nitride ion, N 3− , enable Li ⊕ to polarise the
Except Be and Mg, all s-block metals form saline hydrides. spherical electron cloud of N 3− and it gives higher stability to
Hydrides of p-block elements are covalent in nature, viz, electron Li 3N.
deficient hydrides (by group-13 elements), electron-precise 20. Temporary hardness of water is due to presence of soluble
hydrides (by group-14 elements), and electron-rich hydrides (by Ca(HCO3 )2 or Mg(HCO3 )2.
group 15-17 elements). Hydrides of d , f -block metals are called Permanent hardness of water is due to the presence of CaCl 2 or
interstitial or metallic hydrides. CaSO4 or MgCl 2 or MgSO4.
16. Sodium metal on dissolution in liquid ammonia gives a deep Temporary hardness of water is also called carbonate hardness
blue solution due to the formation of ammoniated electrons. The which can be easily removed by boiling or by treatment with
reaction is represented as follows: Ca(OH)2 (Clark’s method).
Na (s) + (x + y) NH3 (l ) → [Na(NH)x ]+ + [ e(NH 3 ) y ] 21. There are three known isotopes of hydrogen, each possessing an
[Ammoniated Na + [Ammoniated
atomic number 1 and atomic masses 1, 2 and 3 respectively.
or expanded Na] electrons] These are named as protium (1 H), deuterium
Ammoniated (solvated) electrons show electronic transition in (2 H or D) and tritium (3 H or T)
visible region and the solution becomes deep blue coloured. The most common isotope is the ordinary hydrogen usually
This deep blue solution also shows the following properties due called protium. It consists of one proton in the nucleus and an
to the presence of ammoniated electrons. electron revolving around it.
(i) It is strongly reducing in nature. The second isotope of hydrogen is called heavy hydrogen or
(ii) It is paramagnetic. deuterium. It consists of one proton and one neutron in the
(iii) It is a good conductor of electricity. nucleus and an electron revolving around it. The third isotope of
hydrogen is called tritium. It consists of one proton and two
17. Hydrogen has three isotopes: neutrons in the nucleus and an electron revolving around it.
1
1H
2
1H
3
1 H
22. Both reactions in their complete format are written below
(i) In acidic medium,
Protium (P) Deuterium (D) Tritium (T) −1 −2
[Fe2+ (CN)6 ]4− + H2 O2 + 2H+ → [Fe3+ (CN)6 ]3− + 2H2 O
p 1 1 1
(ii) In alkaline medium,
−1
n 0 1 2
[Fe3+ (CN)6 ]3− + H2 O2 + 2OH− →
n 0 1 2 [Fe2+ (CN)6 ]4− + O2 + 2H2O
p Hence, H2O (for reaction (i)) and O2 + H2O
(for reaction (ii)) are produced as by product.
214 s-Block Elements

23. Mg can form basic carbonate while Li cannot. O O


2+  
5 Mg +6 CO2–
3 + 7H2O → 4MgCO3 ⋅ Mg(OH)2 HO  S − O  O − S  OH → 2H2SO4 + H2O2
⋅5H2O + 2 HCO−3  HO  H H  OH 
O O
24. Region 1 (Pre-heating zone)
Region 2 (Primary combustion zone, hottest zone) On partial hydrolysis, it gives one mole of H2SO4 and one mole
Region 3 (Internal zone) of peroxomonosulphuric acid as
Region 4 (Secondary reaction zone) O O
 
25. There is extensive intermolecular H-bonding in the condensed HO  S  O  O  S  OH → H2SO4
phase.  H  OH 
O O
1 O
26. 2Li + O2 (g ) → Li 2O
2 
(Excess) + HO  S  O  O  H
2Na + O2 (g ) → Na 2O2; K + O2 (g ) → KO2 
( Excess) (Excess) O
Peroxomonosulphuric acid
27. Order of first ionisation energy is Sc > Na > K > Rb.
Due to poor shielding effect, removal of one electron from 4s 33. In PbO2 , Pb is in +4 oxidation state and oxygen is in –2 oxidation
orbital is difficult as compared to 3s-orbital. state. In all other case, peroxide ion (O2−
2 ) is present.

28. PLAN This problem can be solved by using concept of oxidant and 34. Metallic lustre of any metal is due to oscillation of free electrons
reductant. present in the metal.
Oxidant Oxidant increases the oxidation number of the 35. H2O is reduced as well as oxidised giving H2 (g ) at cathode and
species with which it is reacted. O2 (g ) at anode.
Reductant Reductant decreases the oxidation number of the 36. KNO3 and other nitrates of alkli metals (except LiNO3) are
species with which it is reacted. thermally stable.
H2O2 reacts with KIO4 in the following manner: 37. Sodium peroxide on treatment with dilute acid gives H2O2
+7 +5 Na 2O2 + H2SO4 → Na 2SO4 + H2O2
KIO4 + H2O2 → KIO3 + H2O + O2 38. Glauber’s salt is Na 2SO4 ⋅ 10H2O.
On reaction of KIO4 with H2O2, oxidation state of I varies from 39. D2O is commonly known as heavy water.
+7 to +5, i.e. decreases. Thus, KIO4 gets reduced hence, H2O2 is 40. Presence of solvated electrons makes solution of alkali metal in
a reducing agent here. liquid ammonia makes them strongly reducing agent.
With NH2OH, it given following reaction: 41. Lime treatment remove bicarbonate hardness by forming
−1 +3 insoluble CaCO3 as
N H2OH + H2O2 → N 2 O3 + H2O Ca(HCO3 )2 + Ca(OH)2 → 2CaCO3 ↓ + 2H2O
In the above reaction, oxidation state of N varies from –1 to +3. 42. PLAN Paramagnetic character of species can be easily explained on
Here, oxidation number increases, hence H2O2 is acting as an the basis of presence of unpaired electrons, i.e. compounds
oxidising agent here. containing unpaired electron(s) is/are paramagnetic.
Hence, (a) is the correct choice. Reaction of alkali metals with ammonia depends upon the
physical state of ammonia whether it is in gaseous state or liquid
29. Release of electron is known as reduction. So, H2O2 acts as state. If ammonia is considered as a gas then reaction will be
reducing agent when it releases electrons. 1
Here, in reaction (II) and (IV), H2O2 releases two electron, (a) Na + NH3 → NaNH2 + H2
(Excess) 2
hence reaction (II) and (IV) is known as reduction.
(NaNH2 + 1/2 H2 are diamagnetic)
In reaction (I) and (III), two electrons are being added, so (I) and
If ammonia is considered as a liquid then reaction will be
(III) represents oxidation.
M + (x + y)NH3 → [ M (NH3 )x ]+ + [ e(NH 3 ) y ]−
30. Mg(HCO3 )2 on boiling decomposes to give white precipitate of • Ammoniated electron
• Blue colour
MgCO3 as: • Paramagnetic
Heat • Very strong reducing agent
Mg(HCO3 )2 (aq) → MgCO3 ↓ + H2O + CO2 ↑
(b) K + O2 → KO2 (K+ , O−2 )
31. Electrolysis of aqueous Na 2SO4 gives H2 (g ) at cathode and (Excess) Potassium superoxide
paramagnetic
O2 (g ) at anode.
(c) 3Cu + 8HNO3 → 3Cu(NO3 )2 + 2NO + 4H2O
Paramagnetic Paramagnetic
32. Peroxodisulphuric acid (H2S2O8 ) on complete hydrolysis gives
two moles of H2SO4 and one mole of H2O2 as
s-Block Elements 215

OH O H2O2 → H2O + [O]


Bleaches colour
Et Et of red litmus
(d) O2
+ H 2O 2 (ii) If the compound is Na 2O, it will hydrolyse to form NaOH.
Na 2O + H2O → 2NaOH
OH O NaOH solution formed above will change colour of red
2-ethylanthraquinol 2-ethylanthraquinone
litmus paper into blue.
Hence, option (a), (b) and (c) are correct choices.
57. KMnO4 + H2O2 → MnO2 + KOH + O2
43. When sodium metal is burnt in excess of air, mainly sodium OA RA
peroxide (Na 2O2 ) with little sodium oxide (Na 2O) are formed. FeSO4 + H2O2 → Fe3+ + H2O
RA OA
44. NaNO3 when heated, it decomposes in two stages as:
1 58. 3M + N2 → M 3 N2
T < 500° C
NaNO3 → NaNO2 + O2 A B
2
NaNO2
T > 800° C
→ Na 2O + N2 + O2 M 3N2 + 6H2O → 3M (OH)2 + 2NH3
B C D
45. In dilute solution of Na in liquid ammonia, solvated electrons M (OH)2 + CO2 → MCO3
are present whose emission spectrum gives blue colouration to C Milkyness
solution.
M can be either Ca or Ba but essentially not Mg because
Na + NH3 → Na + + NH3 (e− ) Mg(OH)2 is very sparingly soluble in water.
Solvated electron

Also, presence of solvated electrons and solvated Na + ion 59. 2KNO3 (s) + 10K(s) → 6K2O (s) + N2 (g )
makes solution highly conducting. 60. In H2O2 , the peroxide ion (O2−
2 ) is unstable, has tendency to pass

46. Zeolite acts as ion exchange resin and its Na + is exchanged with into stable oxide state (O2− ). Hence, H2O2 is a good oxidising
agent while H2O is stable.
H+ and Mg2+ ions present in hard water.
61. In Solvay process, NaHCO3 is extracted from the solution by
47. Solubility of a salt is influenced by two major factors, lattice fractional crystallisation, which is then heated to convert it into
energy and hydration energy. For greater solubility, there Na 2CO3 ⋅ KHCO3 , being more soluble than NaHCO3 , cannot be
should be smaller lattice energy and greater hydration energy. extracted by fractional crystallisation. Hence, Solvay process
48. Both statements are correct but blue colour is due to presence of fails in production of K2CO3.
solvated electron NH3 (e− ). 62. Water forms stronger intermolecular H-bonds, therefore it is
liquid at room temperature while H2S cannot form such strong
49. Statement I is correct. Small size of Li + makes it highly intermolecular bonds, gas at room temperature.
polarising, introduces predominant covalency in LiCl.
Statement II is incorrect, there is very large difference in Topic 2 Group II Elements
electronegativity of Li and Cl.
1. Correct match is I → (B), II → (D), III → (A), IV → (C)
50. Alkali metal forms MH in which hydrogen is in –1 oxidation
(I) Ca(OH)2 used in white wash.
state. Both statements are correct and statement –2 is correct
explanation of statement I. (II) NaCl is used in the preparation of washing soda.
2NH3 + H2O + CO2 → (NH4 )2 CO3
51. Al + conc. NaOH → NaAlO2 + H2 ↑
(NH4 )2CO 3 + H2O + CO 2 → 2NH4HCO 3
52. Na in liquid ammonia contain NH3 (e− ) which possesses charge NH4HCO 3 + NaCl → NH4Cl + NaHCO 3 (s)
and conduct electricity. ∆
2NaHCO 3 → Na 2CO 3 + CO 2 + H2O
53. Occlusion is a phenomena in which particles are physically
trapped in voids. 1
(III) CaSO4 ⋅ H2O (plaster of Paris) is used for making casts of
2
54. I2 disproportionate in alkali giving NaI and NaIO3.
statues.
55. Sodium when burnt in excess of oxygen, gives sodium peroxide (IV) CaCO 3 is used as an antacid.
as major product. 2. In comparison to boron, beryllium has lesser nuclear charge and

Na + O2 → Na 2O2 + Na 2O greater first ionisation enthalpy.
Major Minor
Electronic configuration of Be(4 ) = 1s2 , 2s2.
56. The substance is Na 2O2. When Na 2O2 is dissolved in water, it It possess completely filled s-orbitals. Hence, high amount of
forms NaOH and H2O2. In this case, NaOH is a strong base energy is required to pull the electron from the gaseous atom.
while H2O2 is a weak acid. Beryllium (4) lies left to the boron (5) and on moving from left
(i) Na 2O2 + 2H2O → 2NaOH + H2O2 to right an electron is added due to which nuclear charge
H2O2 decolourises red litmus paper due to its bleaching increases from Be to B.
action which is due to its oxidising character.
216 s-Block Elements

3. The correct sequence of thermal stability of carbonates is So, Be2+ readily forms covalent compounds like BeX 2, because
MgCO3 < CaCO3 < SrCO3 < BaCO3 of very high positive charge density over its small size, so that it
On moving down the group, i.e. from Mg to Ba, atomic radius readily polarises anionic spherical electron cloud.
generally increases. It is due to the addition of shell. As a result, 9. (i) Washing soda (Na 2CO3 ⋅ 10H2O) is manufactured in Solvay
the atomic size increases. CO2− 3 is a large anion. Hence, more process. In this method, CO2 gas is passed through a
stabilised by Ba 2+ (large cation) and less stabilised by Mg2+ . conc. solution of NaCl saturated with NH3. It gives ammonium
Therefore, BaCO3 has highest thermal stability followed by carbonate followed by ammonium hydrogen carbonate.
SrCO3, CaCO3 and MgCO3. The obtained NH4HCO3 is treated with solution of NaCl which
4. Baking soda (NaHCO3 ) is not a hydrated solid. Thus, ( X ) is not result in the formation of NaHCO3. The crystal obtained are
baking soda. Thus, option (a) and (c) are incorrect. Dead burnt heated to obtain Na 2CO3.
plaster (CaSO4 ) is obtained from gypsum via the formation of NaCl + NH3 + CO 2 + H2O → NaHCO3 + NH4Cl
plaster of Paris. ∆ Crystallisation
1 2NaHCO3 →Na 2CO3 → Na 2CO3 ⋅ 10H2O
380-393 K > 393K
CaSO4 ⋅2H2O → CaSO4 ⋅ H2O → CaSO4 − H2O, CO2
−1/ 2 H 2 O 1
Gypsum 2 − H 2 O Dead burnt plaster
Plaster of Paris 2
(anhydrous) (ii) Mg(HCO3 )2 and Ca(HCO3 )2 cause temporary hardness to
Therefore, the reaction takes place as follows : water that can be easily removed by boiling.
< 373K > 373K (iii) NaOH is manufactured by Castner-Kellner process.
Na 2CO3 ⋅ 10H2O → Na 2CO3 ⋅ H2O → Na 2CO3
−9H 2 O −H 2 O
Washing soda Monohydrate Anhydrous white powder In this reaction, Na amalgam flows out and treated with
(X ) (Y ) (soda ash) (Z ) water to give NaOH and H2 gas. During electrolysis,
5. Names of magnesium alloys are given by two letters followed by hydrogen is evolved at cathode and chlorine is evolved at
two numbers. The common alloying elements are A (Aluminium), anode, which are the by product of this process.

Z (zinc), T (tin), M (manganese) etc. Numbers indicate Electrolysis
Carbon anode 2Cl –2e Cl2
respective nominal compositions of main alloying elements, 2NaCl(aq) +
Hg cathode 2Na +2e
e.g. ‘AZ 91’ implies the composition of the alloy as : Al = 9%,
Zn = 1% and Mg = 100 – (9 + 1) = 90%
2Na (Na/Hg)
Among the alloys given, Mg – Al (Magnalium ; Mg = 5%,
H2O
Al = 95%) is being light, tough and strong, hence it is used in
aircrafts. 2NaOH(aq) +H2
6. The structures of beryllium chloride in the solid state and vapour (iv) Portland cement constitutes, tricalcium aluminosilicate,
phase, respectively are dimeric and chain. In vapour phase at 3CaO ⋅Al 2O3. SiO2, i.e. Ca 3Al 2O6 ⋅ SiO2.
above 900°C, BeCl2 is monomeric having a linear structure Cl 10. For group-2 metal hydroxides, basicity increases down the
BeCl. The bonding in BeCl2 is covalent and Be atom group, as:
accommodates 2 + 2 = 4 electrons in the two sp-hybrid orbitals. Be(OH)2 < Mg(OH)2 < Ca(OH)2 < Sr(OH)2 < Ba(OH)2
Below 900°C, beryllium chloride in vapour phase exists as a
This is because as the size of metal atom increases, M—OH
mixture of monomer BeCl2 and dimer Be2Cl4.
bond length increases or M—OH bond become weaker thus
7. Magnesium powder burns in air to give MgO and Mg 3N2. MgO readily breaks to release OH − ions which are responsible for the
does not combine with excess oxygen to give any superoxide. basicity of these solutions.
Mg reacts with nitrogen to form magnesium nitride (Mg 3N2 ). But Be(OH)2 shows amphoteric (basic as well as acidic)
Mg + O2 → MgO character as it reacts with acid and alkali both which is shown in
the following reactions. Be(OH)2 as a base :
3Mg + N2 → Mg3N2
Be(OH)2 + 2HCl → BeCl 2 + 2H2O
8. Key Idea According to Fajan’s rule, degree of covalency Be(OH)2 as an acid :
(ionic potential), φ ∝-polarisation power of the Be(OH)2 + 2NaOH → Na 2[Be(OH)4 ]
cation ∝ charge on the cation
1 11. Among the four elements given, Na, Be, Mg and Ca, Be has
∝ . highest IE value because of its smallest size and 2s2 valence shell
size of the cation
configuration.
Alkaline earth metals contains bipositive (H2+ ) ions in their So, X-ray cannot cause ionisation from the material used, i.e. Be
compounds. in the tube window, which may cause interference in the study.
So, here I
(i) Charge on cation, i.e. + 2 is constant. 12. A saturated aqueous solution of M (NO3 )2 on crystallisation
II
(ii) Halide present (X − ) is also constant. will produce hydrated crystal, M (NO3 )2 ⋅ nH2O only when
So, the covalent character depends on the size of alkaline earth hydration enthalpy (∆H °hyd ) of M 2 + ion will be appreciably
metal. As we move down the group, size of metal ion increases.
more negative.
Be2+ < Mg2+ < Ca 2+ < Sr 2+ < Ba 2+
s-Block Elements 217

Hydration of an ion depends on its size. Smaller the size of an NaOCl + H2 → NaOH + HOCl
ion, higher will be its charge density and as a result it will remain
HO− + HCO3− → CO23− + H2O
more solvated (hydrated) through ion dipole interaction.
Size of group-2 metal ions increases on going down the group. Ca(HCO3 )2 + Na 2CO3 → CaCO3 ↓ + 2NaHCO3
So, their ability to form hydrated crystals follows the order:
18. Magnesium ammonium phosphate is precipitated out.
Be2 + >> Mg 2 + >> Ca 2 + >> Sr 2 + >> Ba 2 +
MgSO4 + NH4OH + Na 2HPO4 → Mg(NH4 )PO4 ↓ + Na 2SO4
Thus, Ba(NO3 )2 is slightly or almost insoluble in water.
19. Si is used in solar cells, because of its semi-conductor properties.
13. As we move down the group, size of metal increases. Be has
lower size while SO2−
4 has bigger size, that’s why BeSO 4 breaks 20. Anhydrous MgCl 2 is obtained by heating hydrated salt in stream
easily and lattice energy becomes smaller but due to lower size of dry HCl.
of Be, water molecules are gathered around and hence hydration 21. Heating MgCl 2 ⋅ 6H2O brings about partial dehydration as
energy increases. On the other hand, rest of the metals, i.e. Ca,
Ba, Sr have bigger size and that’s why lattice energy is greater ∆
MgCl 2 ⋅ 6H2O → Mg(OHCl) + HCl + 5H2O
than hydration energy.
* * *
Time saving technique In the question of finding hydration 22. BaCO 3 + H 2SO 4 → BaSO 4 + H 2O + CO 2 (C = C14)
energy only check the size of atom. Smaller sized atom has more 23. Be in BeCl 2 is electron deficient, short of two lone pair of
hydration energy. Thus, in this question Be is placed upper most electrons from stable octet. H2O has lone pair of electrons, reacts
in the group has lesser size and not comparable with the size of
with BeCl 2.
sulphates. Hence, BeSO 4 is the right response.
24. Alkaline earth metal salts have M 2+ ions which has very high
14. Thermal stability of salts with common anion depends on
polarising power compared to polarising power of monovalent
polarising power of cation. Greater the polarising power, lower
be their thermal stability. Hence, metal ion (M + ) of alkali metal. Due to high polarising power of
BeCO3 (IV) < MgCO3 (II) < CaCO3 (III) < K2CO3 (I) M 2+ , it associate more water than M + .

15. The reaction involved is 25. A = NH3 , B = CaCO3.


BaO2 + H2SO4 → BaSO4 + H2O2 Reactions involved are :
The most electronegative atom, oxygen, in BaSO4 and H2O2 has Heat
3Ca + N2 → Ca 3N2
−2 and −1oxidation state respectively.
Ca 3N2 + 6H2O → 3Ca(OH)2 + 2NH3
16. Electrolysis of molten CaCl 2 gives calcium at cathode A
2+ −
Ca + 2e → Ca (at cathode) Ca(OH)2 + CO2 → CaCO3 + H2O
(From air) B
In case of electrolysis in aqueous medium, less electropositive
26. Basic strength (i) decreases from left to right in period and
H+ is reduced at cathode rather than Ca 2+.
(ii) increases from top to bottom in group. Therefore,
17. Ca(OH)2 + Ca(HCO3 )2 → 2CaCO3 ↓ + 2H2O NiO < MgO < SrO < K2O < Cs2O Basic strength
(Clark’s method)
15
p-Block Elements-I
Topic 1 Group 13 Elements
Objective Questions I (Only one correct option) 8. Aluminium is usually found in +3 oxidation state. In contrast,
1. The reaction of H3N3 B3Cl3 (A) with LiBH 4 in thallium exists in +1and +3 oxidation states. This is due to
(2019 Main, 9 Jan I)
tetrahydrofuran gives inorganic benzene (B). Further, the (a) lattice effect (b) lanthanoid contraction
reaction of (A) with (C) leads to H3N3 B3 (Me)3 . Compounds (c) inert pair effect (d) diagonal relationship
(B) and (C) respectively, are (2020 Main, 9 Jan II)
(a) diborane and MeMgBr 9. The increasing order of atomic radii of the following Group
(b) boron nitride and MeBr 13 elements is (2016 Adv.)
(c) borazine and MeBr (a) Al < Ga < In < Tl (b) Ga < Al < In < Tl
(d) borazine and MeMgBr (c) Al < In < Ga < Tl (d) Al < Ga < Tl < In
2. The green colour produced in the borax bead test of a 10. B(OH) 3 + NaOH w NaBO2 + Na[B(OH) 4 ] + H2 O
chromium (III) salt is due to (2019 Adv.)
How can this reaction is made to proceed in forward
(a) Cr 2O 3 (b) CrB
direction? (2006, 3M)
(c) Cr(BO2 )3 (d) Cr2 (B4O7 )3
(a) Addition of cis 1, 2-diol
3. The correct statements among I to III regarding group 13 (b) Addition of borax
element oxides are: (2019 Main, 9 April II) (c) Addition of trans 1, 2-diol
I. Boron trioxide is acidic. (d) Addition of Na 2HPO4
II. Oxides of aluminium and gallium are amphoteric. 11. H3 BO3 is (2003, 1M)
III. Oxides of indium and thallium are basic. (a) monobasic acid and weak Lewis acid
(a) I, II and III (b) I and III only (b) monobasic and weak Bronsted acid
(c) I and II only (d) II and III only (c) monobasic and strong Lewis acid
4. Diborane (B2 H6 ) reacts independently with O2 and H2 O to (d) tribasic and weak Bronsted acid
produce, respectively. (2019 Main, 8 April I) 12. In compounds of type ECl 3 , where E = B , P, As or Bi, the
(a) B2O3 and H3BO3 (b) B2O3 and [BH4 ]− angles Cl  E  Cl for different E are in the order
(c) H3BO3 and B2O3 (d) HBO2 and H3BO3 (1999, 2M)
5. The relative stability of + 1 oxidation state of group 13 (a) B > P = As = Bi (b) B > P > As > Bi
elements follows the order (2019 Main, 11 Jan II) (c) B < P = As = Bi (d) B < P < As < Bi
(a) Al < Ga < Tl < In
13. Moderate electrical conductivity is shown by (1982, 1M)
(b) Al < Ga < In < Tl
(a) silica (b) graphite
(c) Tl < In < Ga < Al
(d) Ga < Al < In < Tl (c) diamond (d) None of these

6. The number of 2-centre-2-electron and 3-centre-2-electron


bonds in B2 H6 , respectively, are (2019 Main, 10 Jan II)
Objective Questions II
(a) 4 and 2 (b) 2 and 4 (One or more than one correct option)
(c) 2 and 2 (d) 2 and 1 14. Among the following, the correct statement(s) is(are)
7. The electronegativity of aluminium is similar to (2017 Adv.)
(2019 Main, 10 Jan I) (a) Al (CH3 )3 has the three-centre two-electron bonds in its dimeric
(a) lithium (b) carbon structure
(c) beryllium (d) boron (b) The Lewis acidity of BCl 3 is greater than that of AlCl 3
p-Block Elements-I 219

(c) AlCl 3 has the three-centre two-electron bonds in its dimeric


structure
Match the Column
(d) BH3 has the three-centre two-electron bonds in its dimeric 22. Match the following. (2006, 6M)
structure
Column I Column II
15. The crystalline form of borax has (2016 Adv.) A. Bi 3 + → (BiO)+ p. Heat
(a) tetranuclear [ B4O5 (OH)4 ]2 − unit
(b) all boron atoms in the same plane
B. [AlO2 ]− → Al(OH)3 q. Hydrolysis
(c) equal number of sp2 and sp3 hybridised boron atoms C. SiO44 − → Si 2O67 − r. Acidification
(d) one terminal hydroxide per boron atom
D. (B4O72 − ) → [B(OH)3 ] s. Dilution by water
16. The correct statement(s) for orthoboric acid is/are
(2014 Adv.)
(a) It behaves as a weak acid in water due to self ionisation Fill in the Blank
(b) Acidity of its aqueous solution increases upon addition of 23. The two types of bonds present in B2 H6 are covalent and
ethylene glycol ……… (1994, 1M)
(c) It has a three-dimensional structure due to hydrogen
bonding True/False
(d) It is a weak electrolyte in water 24. The basic nature of hydroxide of group 13 (group IIIA)
17. In the reaction, 2 X + B2 H6 → [BH2 ( X )2 ] [BH4 ] + − decreases progressively down the group. (1993, 1M)

the amine(s) X is/are (2009) 25. All the Al  Cl bonds in Al 2 Cl 6 are equivalent. (1989, 1M)

(a) NH3
(b) CH3NH2 Integer Answer Type Questions
(c) (CH3 )2 NH 26. Three moles of B2 H6 are completely reacted with methanol. The
(d) (CH3 )3 N number of moles of boron containing product formed is
(2015 Adv.)
Numerical Answer Type Question 27. The value of n in the molecular formula Ben Al2 Si6 O18 is
18. Among B 2H 6, B 3N 3H 6, N 2O, N 2O 4, H 2S2O 3 and H 2S2O 8, (2010)
the total number of molecules containing covalent bond Subjective Questions
between two atoms of the same kind is .............(2019 Adv.)
28. AlF3 is insoluble in anhydrous HF but when little KF is added to
Assertion and Reason the compound it becomes soluble. On addition of BF3, AlF3 is
precipitated. Write the balanced chemical equations. (2004, 2M)
Read the following questions and answer as per the
direction given below : 29. (i) How is boron obtained from borax? Give chemical equations
(a) Statement I is correct; Statement II is correct Statement II is with reaction conditions.
the correct explanation of Statement I (ii) Write the structure of B2H6 and its reaction with HCl. (2002)
(b) Statement I is correct; Statement II is correct Statement II is
not the correct explanation of Statement I
30. Compound X on reduction with LiAlH4 gives a hydride Y
(c) Statement I is correct; Statement II is incorrect containing 21.72% hydrogen alongwith other products. The
(d) Statement I is incorrect; Statement II is correct compound Y reacts with air explosively resulting in boron
trioxide. Identify X and Y. Give balanced reactions involved in
19. Statement I Boron always forms covalent bond. the formation of Y and its reaction with air.
Statement II The small size of B3+ favours formation of Draw the structure of Y. (2001, 5M)
covalent bond. (2007, 3M)
31. Aluminium sulphide gives a foul odour when it becomes damp.
20. Statement I In water, orthoboric acid behaves as a weak Write a balanced chemical equation for the reaction.
monobasic acid. (1997, 2M)
Statement II In water, orthoboric acid acts as a proton 32. Anhydrous AlCl 3 is covalent. From the data given below,
donor. (2007, 3M)
predict whether it would remain covalent or become ionic in
21. Statement I Al(OH)3 is amphoteric in nature. aqueous solution.
Statement II Al  O and O  H bonds can be broken (Ionisation energy for Al = 5137 kJ mol –1
with equal ease in Al (OH)3 . (1998 , 2M)
∆H hydration for Al 3+ = – 4665 kJ mol –1
∆H hydration for Cl − = − 381 kJ mol −1
(1997, 2M)
220 p-Block Elements-I

Topic 2 Group 14 Elements


Objective Questions I (Only one correct option) 12. Identify the correct order of acidic strength of
1. The C C bond length is maximum in (2019 Main, 12 April II)
CO2 , CuO, CaO, H2 O. (2002, 3M)

(a) graphite (b) C70 (a) CaO < CuO < H2O < CO2
(c) C60 (d) diamond (b) H2O < CuO < CaO < CO2
(c) CaO < H2O < CuO < CO2
2. The basic structural unit of feldspar, zeolites, mica and asbestos (d) H2O < CO2 < CaO < CuO
is (2019 Main, 12 April I)
(a) (SiO3 )2− (b) SiO2 (c) (SiO4 )4− 13. Which one of the following oxides is neutral ? (1996, 1M)

R (a) CO (b) SnO2 (c) ZnO (d) SiO2


 14. Which of the following halides is least stable and has
(d) ( Si  O )
n (R = Me) doubtful existence? (1996, 1M)

R (a) CCl 4 (b) GeI4 (c) SnI4 (d) PbI4

Objective Question II
3. The correct order of catenation is (2019 Main, 10 April I) (One or more than one correct option)
(a) C > Sn > Si ≈ Ge (b) Si > Sn > C > Ge
15. Choose the correct statement(s) among the following:
(c) C > Si > Ge ≈ Sn (d) Ge > Sn > Si > C
(2020 Adv.)
4. The amorphous form of silica is (2019 Main, 9 April II) (a) SnCl 2 ⋅ 2H2O is a reducing agent.
(a) tridymite (b) kieselguhr (b) SnO2 reacts with KOH to form K 2[Sn(OH)6 ].
(c) cristobalite (d) quartz (c) A solution of PbCl 2 in HCl contains Pb2+ and Cl − ions.
5. C60 an allotrope of carbon contains (2019 Main, 9 April I) (d) The reaction of Pb3O4 with hot dilute nitric acid to give PbO2
(a) 16 hexagons and 16 pentagons is a redox reaction.
(b) 20 hexagons and 12 pentagons 16. With respect to graphite and diamond, which of the
(c) 12 hexagons and 20 pentagons statement(s) given below is/are correct? (2012)
(d) 18 hexagons and 14 pentagons (a) Graphite is harder than diamond
6. The element that does not show catenation is (b) Graphite has higher electrical conductivity than diamond.
(2019 Main, 12 Jan II) (c) Graphite has higher thermal conductivity than diamond.
(a) Ge (b) Sn (c) Si (d) Pb (d) Graphite has higher C C bond order than diamond
7. The element that shows greater ability to form pπ- pπ multiple
bonds, is (2019 Main, 12 Jan II) Assertion and Reason
(a) Ge (b) Si (c) Sn (d) C (a) Statement I is correc;t Statement II is correct Statement II is
the correct explanation of Statement I
8. The chloride that cannot get hydrolysed is (2019 Main, 11 Jan I)
(a) SnCl 4 (b) CCl 4 (b) Statement I is correct; Statement II is correct Statement II is
(c) PbCl 4 (d) SiCl 4 not the correct explanation of Statement I
(c) Statement I is correct; Statement II is incorrect
9. Correct statements among (I) to (IV) regarding silicones are:
(2019 Main, 9 Jan I) (d) Statement I is incorrect; Statement II is correct
I. They are polymers with hydrophobic character. 17. Statement I Pb 4+ compounds are stronger oxidising
II. They are biocompatible. agents than Sn 2+ compounds.
III. In general, they have high thermal stability and low Statement II The higher oxidation states for the group
dielectric strength.
14 elements are more stable for the heavier members of the
IV. Usually, they are resistant to oxidation and used as greases.
group due to ‘inert pair effect’. (2008, 3M)
(a) I and II only (b) I, II, III only
(c) I, II, III and IV (d) I, II and IV only 18. Statement I Between SiCl 4 and CCl 4 , only SiCl 4 reacts
10. Name the structure of silicates in which three oxygen atoms of with water.
4−
[ SiO4 ] are shared is (2005, 1M) Statement II SiCl 4 is ionic and CCl 4 is covalent.
(2001, S, 1M)
(a) pyrosilicate (b) sheet silicate
(c) linear chain silicate (d) three-dimensional silicate
Fill in the Blanks
11. Me2 SiCl 2 on hydrolysis will produce (2003, 1M)
19. A liquid which is permanently supercooled is frequently
(a) (Me)2 Si(OH)2 (b) (Me)2 Si ==O
called ……… . (1997, 1M)
(c) [  O  (Me)2 Si  O ]n (d) Me2SiCl(OH)
p-Block Elements-I 221

20. The recently discovered allotrope of carbon (e.g. C60 ) is 28. Draw the structure of a cyclic silicate, (Si 3 O9 )6– with proper
commonly known as ……… . (1994, 1M) labelling. (1998, 4M)
21. The hydrolysis of trialkyl chlorosilane R3 SiCl, yields ……… 29. Write the balanced equation for the preparation of crystalline
. (1994, 1M) silicon from SiCl 4 . (1990, 1M)

22. The hydrolysis of alkyl substituted chlorosilanes gives 30. Each entry in column X is in some way related to the entries in
……… . (1991, 1M) columns Y and Z. Match the appropriate entries.
X Y Z
True/False
Yeast Fermentation Ethanol
23. The tendency for catenation is much higher for C than for Si.
(1993, 1M) Mica Graphite Abrasive
24. Diamond is harder than graphite. (1993, 1M) Superphosphate Crystalline cubic Insulator
25. Graphite is a better lubricant on the moon than on the earth. Carbon fibres Layer structure Fertiliser
(1987, 1M)
Rock salt Diamond structure Reinforced
26. Carbon tetrachloride burns in air when lighted to give plastics
phosgene gas. (1983, 1M)
Carborundum Bone ash Preservative
Subjective Questions (1989, 3M)

27. Starting from SiCl 4 , prepare the following in steps not 31. Give reasons for the following in one or two sentences :
exceeding the number given in parenthesis (give reactions “Graphite is used as a solid lubricant.” (1985, 1M)
only). 32. Give reason for the following in one or two sentences :
(i) Silicon “Solid carbon dioxide is known as dry ice.” (1983, 1M)
(ii) Linear silicon containing methyl group only 33. Carbon acts as an abrasive and also as a lubricant, explain.
(iii) Na 2SiO3 (2001, 5M) (1981, 1M)

Answers
Topic 1 Topic 2
1. (d) 2. (c) 3. (a) 4. (a) 1. (d) 2. (c) 3. (c) 4. (b)
5. (b) 6. (a) 7. (c) 8. (c) 5. (b) 6. (d) 7. (d) 8. (b)
9. (b) 10. (a) 11. (a) 12. (b) 9. (d) 10. (a) 11. (c) 12. (a)
13. (b) 14. (a, b, c) 15. (a, c, d) 16. (b, d) 13. (a) 14. (d) 15. (a,c) 16. (b, d)
17. (a, b, c) 18. (4.00) 19. (a) 20. (a) 17. (a) 18. (c) 19. (glass)
21. (a) 22. (A → q; B → r; C → p ; D → q, r) 20. (Buckminster) 21. ( R3SiO) 2 22. (silicones)
23. (Three centre two electron bond or banana bond) 23. (T) 24. (T) 25. (T) 26. (F)
24. (F) 25. (F) 27. (3)
Hints & Solutions
Topic 1 Group 13 Elements inflammable nature, it catches fire spontaneously when exposed
1. Road map of the given reactions is as follows: to air and burns in oxygen releasing an enormous amount of
energy as:
+LiBH4
H3N3B3Cl3 (B), B2H6 + 3O2 → B2O3 + 3H2O + 1976 kJ/mol
(THF)
(A) Inorganic It gets hydrolysed readily to give boric acid.
benzene
B2H6 + 6H2O → 2H3BO3 + 6H2 ↑
+ (C) Borane Orthoboric acid Dihydrogen
H3N3B3(Me)3
Complete reactions are as follows : 5. The stability order of + 3 and + 1 oxidation states of group 13
elements will be:
Cl H H H
B3 + > Al 3 + > Ga 3 + > In 3 + >>Tl 3+
B N (order of + 3oxidation state)
MeMgBr (C) H H +LiBH4
B3N3H3(Me)3 N B B+ <<Al + < Ga + < In + < Tl +
H (THF)
Cl (order of + 1 oxidation state)
B N
The presence of two oxidation states in p-block elements is due
Cl H H H H
H to the inert pair effect.
B B– Because of the presence of poor shielding d and f -orbitals, as we
H H H + +
N N N N move from Ga to Tl, effective nuclear charge of these elements
– + increases so as to hold the valence ns2 electrons tightly. It causes
B B B B
H H H H difficulty to the ionisation of ns2-electrons and it remains inert,
N N+
only np1-electron ionises to give + 1oxidation state.
H (B3N3H6) H
(Borazine) 6. The structure of B2H6 can be shown as :
H H H
2. Borax bead test is performed only for coloured salt. Borax H
(sodium pyroborate), Na2B 4O 7 ⋅10H2O on heating gets fused and H e H
–2
3c–3e 2c 3
lose water of crystallisation. It swells up into fluffy white porous B B Þ B B sp
mass which melts into a colourless liquid which later form a clear H H
transparent glassy bead consisting of boric anhydride and H H H
H
sodium metaborate. In B2H6, four 2-centre-2-electron (2c − 2e) bonds are present in

Na2B 4O 7 ⋅10H2O → Na2B 4O 7 +10H2O ↑ the same plane and two 3-centre-2-electron (3c − 2e) bonds are
present in another plane.

Na2B 4O 7 → B 2O 3 + 2NaBO 2 7. Let, us consider the electronegativity values of the given
Boric Sodium
anhydride metaborate elements,
14442444
3
glassy bead Group-1 Group-2 Group-13 Group-19
Boric anhydride is non-volalite. When it react with Cr(III) salt Period 2 ⇒ Li Be B C
(1 . 0) (1 . 5) (2 . 0) (2 . 5)
then deep green complex is formed.
Period 3 ⇒ Al
2Cr 3+ + 3B 2O 3 → 2Cr(BO 2)3 (1. 5)
Deep green
Be and Al show diagonal relationship which is based on their
Hence, option (c) is correct. Z∗
same value (Z* is effective nuclear charge, r = atomic
3. All the given statements are correct. For group 13 elements, the r
acidic nature of oxides decreases and the basic nature of oxides radius). So, they have similar electronegativity.
increases on moving from B to Tl. This is because as we move 8. Due to inert pair effect, group-13 elements (ns2np1 ) show + 3and
down the group, the atomic size of elements goes on increasing, + 1 oxidation states in their compounds. Stability order of these
whereas the ionisation energy decreases, due to which the oxidation states will be as,
strength of metal oxide (MO) bond goes on decreasing. Thus, l
+ 3 oxidation states
boron trioxide or boron oxide is acidic and reacts with basic
oxides to give metal borates. Aluminium and gallium oxides are B3+ > Al3+ > Ga 3+ > In3+ > Tl3+
amphoteric while oxides of indium and thallium are basic in B3+ does not exist in free states. All B(III) compounds are
nature. covalent.
4. Diborane (B2H6 ) reacts independently with O2 and H2O to l
+ 1 oxidation states
produce B2O3 and H3BO3 respectively. Diborane is a colourless, B+ < Al+ < Ga + < In + < Tl+
highly toxic gas, having boiling point 180 K. Because of its B+ does not exist in ionic as well covalent compounds.
p-Block Elements-I 223

9. Due to poor shielding of d-orbital in Ga, atomic radius of Ga is 15. Na 2B4O7 ⋅ 10H2O (borax) is actually made of two tetrahedral
smaller than that of Al. Thus, Ga < Al < In < Tl. and two triangular units, and is actually written as
10. Orthoboric acid is a very weak acid, direct neutralisation does Na 2[B4O5 (OH)4 ]⋅ 5H2O.
not complete. However, addition of cis-diol allow the reaction OH
to go to completion by forming a stable complex with s sp3
[B(OH)4 ]− as: O—B O sp2
CH2  OH HO—B O B—OH
HO OH
B + 2 → 2 O—B O
HO OH CH2  OH sp s
sp3 OH
H2C  O O  CH2
 B  + 2H2O (a) Thus, correct.
H2C  O O  CH2 (b) Boron atoms are in different planes thus, incorrect.
(c) Two sp2 and two sp3-hybridised B atoms thus, correct.
11. Orthoboric acid is a weak, monobasic, Lewis acid.
(d) Each boron has one OH group thus, correct.
OH has deficiency of a lone-pair
16. (a) It does not undergo self ionisation in water but accepts an
(Lewis acid)
HO—B electron pair from water, so it behaves as weak monobasic
acid.
OH H3BO3 + H2O r B(OH)4− + H+
pπ - pπ backbonding between ‘B’ and ‘O’ decreases acid Hence, (a) is incorrect.
strength greatly :
pπ–p (b) When treated with 1, 2-dihydroxy or polyhydroxy
π compounds, they form chelate (ring complex) which
effectively remove [ B(OH)4 ]− species from solution and
HO—B—OH
thereby produce maximum number of H3O+ or H+ ions, i.e.
OH results in increased acidity.
(c) Boric acid crystallises in a layer structure in which planar
12. In BCl 3, bond angle = 120°. triangular BO3−3 ` ions are bonded together through hydrogen
In PCl 3 , AsCl 3 and BiCl 3 , central atom is sp3 hybridised. Since bonds.
P, As and Bi are from the same group, bond angle decreases One trigonal
down the group. Hence, overall order of bond angle is : planar B(OH)3 unit
H H
B > P > As > Bi
H H
13. Graphite has layered structure and conducted electricity H H
moderately. Silica and diamond have 3-dimensional network B
structures and non-conducting. H
H B H
Three centre two B H
14. (a) H H
H H H electron bond H H H H
CH3 CH3 H H H
C B B
Al Al H H
H
CH3 C CH3 B
H H
H H H H H H

(b) BCl 3 is stronger Lewis acid than AlCl 3 due to greater extent H H
of pπ − pπ back bonding in AlCl 3. (d) In water the pK a value of H3BO3 is 9.25.
(c) Three centre four H3BO3 + H2O r B(OH)−4 + H+ ; pKa = 9.25
electron bond
So, it is a weak electrolyte in water.
Cl Cl Cl 17. Diborane (B2H6 ) undergoes unsymmetric cleavage with NH3 ,
Al Al primary and secondary amine while tertiary amine brings about
Cl Cl Cl symmetrical cleavage of B2H6 as :
(d) Three centre two H H H
electron bond
B B + NH3 or 1° amine
H H or 2° amine
H H H H
B B Unsymmetric X
H H H
cleavage
+ –
[BH2(X)2] [BH4]
224 p-Block Elements-I

H H H 23. Three centred two electron bonds.


B B + 2R3N 2H3B(R3N) H H H
H H H B B Covalent bonds
Symmetrical cleavage
H H H
18. N 2O, N 2O 4 , H2S2O 3 and H2S2O 8 molecules are containing
covalent bond between two atoms. Three centred two electron
S bridged B—H—B bonds

O O 24. The basic nature of hydroxide of group-13 increases from top to


N ºº N ¾® O , S bottom due to increase in electropositive character.
(N2O) N¾N ,
HO OH O O 25. In Al 2Cl6 , Al–Cl bonds are not equivalent :
O (N2O4)
(H2S2O3) Cl Cl Cl
O O Al Al The bridged Al—Cl bonds
H H are different from terminal
H Cl Cl Cl Al—Cl bonds.
S S , B B
26. B2H6 + 6 CH3OH → 2[B(OCH3 )3 ] + 6H2
HO O¾O OH H H
O O H Therefore, from 3 moles of B2H6 , 6 moles of B(OCH3 )3 will be
(H2S2O8) (B2H6)

H 27. BenAl 2Si6O18 , 2n + 6 + 24 − 36 = 0 ⇒ n = 3


|
H B H 28. 3KF + AlF3 → B2H5Cl + H2
N N K3AlF6 + 3BF3 → AlF3 ↓ + 3KBF4
ppt
B B 29. (i) Na 2B4O7 + HCl → NaCl + H3BO3
H N H

H3BO3 + HCl → BCl 3 + H2O
(B3N3H6)

B 2H6 and B 3N 6H6 have polar bond, but do not have same kind of ∆
BCl 3 + Al → B + AlCl 3
atom.
H H H
19. Small size and high charge on B3+ makes it highly polarising. (ii) B2H6 : B B
Therefore, in most of its compounds, boron forms covalent bonds. H H H
Hence, both statement I and statement II are correct and It has 4 terminal B—H bonds. There are two B—H—B, three
statement II is a correct explanation of statement I. centred two electron bridged bonds.
20. Orthoboric acid is a weak, monobasic, Lewis acid and the poor B2H6 + HCl → B2H5Cl + H2
acidic character is due to pπ − pπ backbondings as:
LiAlH4
pπ–pπ 30. Compound X → Y a hydride + other compound. Hydride
Backbonding decreases electron Y contains 21.72% hydrogen.
HO—B—OH deficiency at boron, decreases ∆
its Lewis acid strength. Y + O2 → B2O3 + H2O
OH Therefore, Y is a hydride of boron and it is obtained by reduction
21. Due to small size and high charge on Al in Al(OH)3 the fission of X with LiAlH4. So, X is either BCl 3 or BF3.
ability of Al—O and O—H bonds become comparable and 4BCl 3 + LiAlH4 → B2H6 + 3AlCl 3 + 3LiCl
compound can give both H+ and HO− under appropriate
1442443
X Y Other products
reaction conditions as: Molar mass of B2H6 = 2 × 11 + 6 = 28
Al(OH)3 + 3HCl → AlCl 3 + 3H2O 6
Base % of H in B2H6 = × 100 = 21.5 ≈ 21.72
28
Al(OH)3 + NaOH → Na[Al(OH)4 ]
Acid B2H6 + 3O2 → B2O3 + 3H2O + Heat
Y
Therefore, both statements are correct and statement II is a
correct explanation of statement I. Structure of Y (B2H6 )
3+ + H
22. (A) Bi hydrolysis to (BiO)  q. Å
3
(B) [AlO2 ]− exist in basic medium, on acidification gives H 1.3 9ÅH
97° 1.1
Al(OH)3  r. B B 122°
(C) Orthosilicate (SiO4−
4 ) on heating changes into pyrosilicate
H H
Si 2O6−
7  p.
H
(D) Tetraborate ion [B4O72− ] on treatment with dil. acid
1.77 Å
hydrolysis gradually to orthoboric acid  q, r.
p-Block Elements-I 225

(a) There are 4 terminal B—H bonds. 6. The property of self-linking of atoms of an element through
(b) There are two 3-centre-2-electron B—H—B bridged bonds. covalent bonds to form straight or branched chains and rings of
(c) Terminal H—B—H planes are perpendicular to bridged different sizes is called catenation. Down the group, catenation
B—H—B bonds. tendency decreases due to decrease in element bond strength.
31. Al 2S3 + 6H2O → 2Al(OH)3 ↓ + 3H2S(g ) ↑ Carbon (C), silicon (Si), germanium (Ge), tin (Sn), lead (Pb) are
Foul odour group-14 elements.
Foul odour on damping of Al 2S3 is due to the formation of H2S Catenation tendency is highest in carbon while silicon has second
gas as shown above. highest tendency of catenation among all elements of family due
32. The total hydration energy of AlCl 3 to higher bond energy. The decreasing tendency of catenation
among group 14 elements is as follows:
= Hydration energy of Al 3+ + 3 × Hydration energy of Cl −
C >> Si > Ge ≈ Sn
= − 4665 + 3 (− 381) kJ/mol
However, Pb does not show catenation.
= − 5808 kJ/mol
The above hydration energy is more than the energy required for
7. Carbon (C) has greatest ability to form stable pπ-pπ multiple
ionisation of AlCl 3 into Al 3+ and 3Cl − . bonds. 2p-orbitals of this element participate in the process. The
stability of multiple bonds of C is attributed to their closeness
Due to this reason, AlCl 3 becomes ionic in aqueous solution. In
with C-nucleus. Thus, the smaller size of C plays a significant
aqueous solution, it is ionised completely as
role in the process.
AlCl 3 + 6H2O → [Al(H2O)6 ]3+ + 3Cl −
8. The compounds given are the tetrahalides
(MCl 4) of group 14 elements. For the hydrolysis, (nucleophilic
Topic 2 Groups 14 Elements substitution) of MCl 4 the nature of the M—Cl bond should be as:
1. The C C bond length is maximum in diamond having value δ+ δ–
154 pm. Here, each carbon atom undergoes sp3 hybridisation and M——Cl
linked to four other carbon atoms by using hybridised orbitals in
tetrahedral fashion. It has a rigid three-dimensional network of It must expand its covalency beyond 4 by the use
of its vacant d-orbital which will accommodate the
carbon atoms. lone pair of electrons of H2O (the–nucleophile).
C C bond length within the layers of graphite is 141.5 pm. In
C60, C C distances between single and double bonds are 143.5 sp3d
pm and 138.3 pm respectively. sp3
Cl3 M——Cl Cl3 M——Cl
2. The basic structural unit of feldspar, d + d – OH2
d0
O
zeolites, mica and asbestos is (SiO4 )4− . These all are silicates. H H
–HCl
All silicates involve two types of Si  O bonds. Transition state
(i) Terminal Si O bonds in which oxygen is bonded to a
3H2O sp3
silicon and not other atom.
M(OH)4 Cl3 M—(OH)
(ii) Bridging Si  O Si bonds in which oxygen is bonded to –3HCl
two silicon atoms.
Here, M can be Si, Sn and Pb because they have vacant
O– s nd-orbital. But, carbon is a member of second period (n = 2,
l = 0, 1),
Si O– it does not have d-orbital (l = 2). So, CCl 4 will not be hydrolysed
and correct option is (b).
O– O– s s
9. Silicones are polysiloxanes with general chemical formula,
In SiO4−
4 ion, each Si atom is bonded to four oxygen atoms [R2SiO]n, where R is an organic group such as:
tetrahedrally. CH3, C2H5, C6H5 etc.
3. Catenation property is an unique property of group 14 elements. Silicones have many useful properties:
Down the group 14, catenation power decreases as: (i) They repel water and form watertight seals.
C > Si > Ge ≈ Sn (ii) They are heat resistant because of constancy of properties
Pb does not show catenation. over a wide range of temperature (− 100° to 250° C).
4. Silica occurs in nature in several amorphous and crystalline (iii) Silicones are non-toxic.
forms. Kieselguhr is the amorphous form of silica. Quartz, (iv) Silicones are biocompatible because these do not support
tridymite and cristobalite are crystalline forms of silica. microbiological growth and these have high gas permeability
at room temperature.
5. C60 is aromatic allotrope of carbon containing 12 pentagons and
(v) They are resistant to O2, O3 and UV-radiation.
20 hexagons. It is a fullerene having a shape like soccer ball and
called Buckminster fullerene. (vi) Silicones are formulated to be electrically insulative.
226 p-Block Elements-I

(vii) Silicone grease is typically used as a lubricant for brake (c) In conc. HCl, PbCl 2 exists as chloroplumbous acid,
components in automobiles, since it is stable at high H2[PbCl 4 ]
temperature, is not water soluble and is a odourless viscous II II
liquid. PbCl 2 + 2HCl → H2[ PbCl 4 ]
10. In sheet silicates, three out of four oxygen of SiO4−
4 unit are
Hence, statement (c) is incorrect.
+2 +4
shared as shown below :
(d) Pb3O4 is a mixture of (2 PbO + PbO2 )
– – +2 +4
– Pb3O4 + 4 HNO3 → 2Pb(NO3 )2 + PbO2 + 2H2O

– It is not a redox reaction. Thus, the statement (d) is incorrect.

– – – 16. Diamond has a three-dimensional network structure, a hard



substance where graphite is soft due to layered structure.
In graphite, only three valence electrons are involved in bonding
Three oxygens of every and one electron remain free giving electrical conductivity. In
– – – – tetrahedra are diamond, all the four valence electrons are covalently bonded
shared with others
hence, insulator.
– – – Diamond is better thermal conductor than graphite. Electrical
– conductivity is due to availability of free electrons, thermal
conduction is due to transfer of thermal vibrational energy from

– one atom to another atom. A compact and precisely aligned
– crystals like diamond thus facilitate better movement of heat.
– – In graphite C  C bond acquire some double bond character,
In pyrosilicates, there is only one shared oxygen, in linear chain hence, higher bond order than in diamond.
silicates, two oxygen per tetrahedra are shared while in 17. In group 13, 14, 15 as we descend down in group, the higher
three-dimensional silicates, all four oxygens are shared.
oxidation state becomes less tenable due to inert pair effect.
11. Me2SiCl 2 on hydrolysis yields a linear chain silicone as : Therefore, lead show +2 as stable oxidation state. Hence, Pb4+
CH3 CH3 act as a strong oxidising agent, itself reduced to Pb2+ very easily.
  Both statement I and statement II are correct and statement II is a
Cl  Si  Cl + 2H2O → HO  Si  OH + 2HCl correct explanation of statement I.
 
CH3 CH3 18. SiCl 4 reacts with water due to vacant d-orbitals available with
Si as:
CH3 CH3
 Polymerisation 
nHO  Si  OH → 
[ O  Si  O ]n H2O SiCl4
 
CH3 CH3
12. CO2 is acidic oxide, H2O is neutral, CaO is strongly basic and No such vacant d-orbitals are available with carbon, hence CCl 4
CuO is weakly basic. Therefore, order of acid strength is : does not react wtih water. Otherwise, both SiCl 4 and CCl 4 are
CaO < CuO < H2O < CO2 covalent.
13. Carbon monoxide is a neutral oxide, all others are amphoteric : Statement I is correct but statement II is incorrect.
SnO2 + 4HCl → SnCl 4 + 2H2O 19. Glass is commonly known as supercooled liquid.
ZnO + 2HCl → ZnCl 2 + H2O 20. Buckminster fullerene is the name of recently discovered
SiO2 + 2NaOH → Na 2SiO3 + H2O allotrope of carbon.
SnO2 and ZnO also react with NaOH. SiO2 is also attacked by 21. After dimerisation, no reactive function group remains.
H3PO4.
R
14. PbI4 is least stable, has doubtful existence. It is due to inert pair 
effect, the stable oxidation state of lead is + 2. R2SiCl + H2O → R  Si  OH
− HCl

15. (a) Sn 2+ of stannous chloride dihydrate (SnCl 2 ⋅ 2H2O) tends to R
convert into Sn 4+ . R R
Hence, statement (a) is correct.  
→ R  Si  O  Si  R
(b) SnO2 reacts with KOH and gives K2SnO3 ⋅ 3H2O or
 
K2[ Sn(OH)6 ]because it is amphoteric in nature. R R
SnO2 + KOH → K2SnO3 + H2O or K2[ Sn(OH)6 ] Dimeric silicone
Hence, statement (b) is correct.
p-Block Elements-I 227

22. Silicones are organosilicon polymers, obtained by hydrolysis of heat


Si(OH)4 → SiO2 + 2H2O
alkyl substituted chlorosilanes.
23. Due to smaller size of carbon than silicon, C—C bond is stronger ∆
SiO2 + Na 2CO3 → Na 2SiO3 + CO2
than Si—Si bond, hence former is more likely to extend than
later. O–
28. O = Oxygen
24. Graphite has a layered structure of hexagonal carbon rings = Silicon

stacked one over other which makes it slippery.
On the other hand, in diamond, each carbon is tetrahedrally bond O O
6–
to other four carbons extended in three dimensional space, giving Si3O9
a giant, network structure. Due to this reason, diamond is harder
than graphite. O
O O
25. Graphite is better lubricant on moon than on earth because of ∆
absence of gravitational pull on the moon. 29. 3SiCl 4 + 4Al → 4AlCl 3 + 3Si
Vapour Molten Volatilizes Crystalline
26. Phosgene gas is obtained by treatment of CCl 4 with superheated
30. X Y Z
steam :
CCl 4 + H2O (vapour) → COCl 2 + 2HCl Yeast Fermentation Ethanol
∆ 3Si + 4AlCl ; Mica Layered structure Insulator
27. (i) 3SiCl 4 + 4Al → 3
Mg or Zn can also be used. Superphosphate Bone ash Fertiliser
(ii) SiCl 4 + 2CH3MgCl → (CH3 )2 SiCl 2 + 2MgCl 2 Carbon fibres Graphite Reinforced plastics

OH Rock salt Crystalline cubic Preservative


− HCl  Carborundum Diamond structure Abrasive
(CH3 )2 SiCl 2 + H2O → CH3  Si  CH3

OH 31. Graphite has layered structure and the adjacent layers are weakly
CH3 CH3 CH3 associated giving slippery nature, used as solid lubricant.
   32. Carbon dioxide solidifies at very low temperature, hence solid
→  O  Si  O  Si  O  Si  O CO2 is very cold, commonly known as dry ice. Also solid carbon
   dioxide sublime, without passing through liquid state.
CH3 CH3 CH3
33. The two common allotropes of carbon are diamond and graphite.
(iii) SiCl 4 + 4H2O → Si(OH)4 + 4HCl
Unstable
Diamond is the hardest, natural, substance, used as an abrasive
while graphite is soft, used as a lubricant.
16
p-Block Elements-II

Topic 1 Elements and Compounds of Group 15 and 16


Objective Questions I (Only one correct option)
1. The correct statement among the following is 8. The compound that does not produce nitrogen gas by the
(2019 Main, 12 April I) thermal decomposition is (2018 Main)
(a) (SiH3 )3 N is planar and less basic than (CH3 )3 N. (a) Ba(N3 )2 (b) (NH4 )2 Cr2O7
(b) (SiH3 )3 N is pyramidal and more basic than (CH3 )3N. (c) NH4NO2 (d) (NH4 )2 SO4
(c) (SiH3 )3 N is pyramidal and less basic than (CH3 )3 N. 9. The order of the oxidation state of the phosphorus atom in
(d) (SiH3 )3 N is planar and more basic than (CH3 )3 N. H3 PO2 , H3 PO4 , H3 PO3 and H4 P2 O6 is (2017 Adv.)

2. The number of pentagons in C60 and trigons (triangles) in (a) H3PO4 > H3PO2 > H3PO3 > H4P2O6
white phosphorus, respectively, are (2019 Main, 10 April II) (b) H3PO4 > H4P2O6 > H3PO3 > H3PO2
(a) 20 and 3 (b) 12 and 4 (c) H3PO2 > H3PO3 > H4P2O6 > H3PO4
(c) 20 and 4 (d) 12 and 3
(d) H3PO3 > H3PO2 > H3PO4 > H4P2O6
3. The oxoacid of sulphur that does not contain bond between
10. The species in which the N-atom is in a state of
sulphur atoms is (2019 Main 10 April I)
sp hybridisation is (2016 Main)
(a) H2S2O3 (b) H2S2O4
(a) NO−2 (b) NO−3
(c) H2S2O7 (d) H2S4O6
(c) NO2 (d) NO+2
4. The correct order of the oxidation states of nitrogen in NO,
NO2 , NO2 and N2 O3 is (2019 Main, 9 April I) 11. The pair in which phosphorus atoms have a formal oxidation
state of +3 is (2016 Main)
(a) NO2 < NO < N2O3 < N2O (b) N2O < NO < N2O3 < NO2
(a) pyrophosphorous and hypophosphoric acids
(c) O2 < N2O3 < NO < N2O (d) N2O < N2O3 < NO < NO2
(b) orthophosphorous and hypophosphoric acids
5. The pair that contains two PH bonds in each of the (c) pyrophosphorous and pyrophosphoric acids
oxoacids is (2019 Main, 10 Jan II) (d) orthophosphorous and pyrophosphorous acids
(a) H4P2O5 and H4P2O6 (b) H3PO3 and H3PO2
(c) H4P2O5 and H3PO3 (d) H3PO2 and H4P2O5 12. The product formed in the reaction of SOCl 2 with white
phosphorus is (2014 Adv.)
6. When the first electron gain enthalpy (∆ e g H ) of oxygen is (a) PCl 3 (b) SO2Cl 2
− 141 kJ/ mol, its second electron gain enthalpy is (c) SCl 2 (d) POCl 3
(2019 Main, 9 Jan II)
(a) a positive value
13. Which of the following properties is not shown by NO?
(a) It is paramagnetic in liquid state (2014 Main)
(b) a more negative value than the first
(b) It is a neutral oxide
(c) almost the same as that of the first
(d) negative, but less negative than the first (c) It combines with oxygen to form nitrogen dioxide
(d) Its bond order is 2.5
7. Good reducing nature of H3 PO2 is attributed to the presence
of (2019 Main, 9 Jan II) 14. Concentrated nitric acid upon long standing, turns
(a) two P H bonds (b) one P H bond yellow-brown due to the formation of (2013 Main)
(c) two P OH bonds (d) one P  OH bond (a) NO (b) NO2
(c) N 2O (d) N 2O4
p-Block Elements-II 229

15. Which of the following is the wrong statement? (2013 Main) 27. Polyphosphates are used as water softening agents because
(a) ONCl and ONO − are not isoelectronic they (2002, 3M)
(b) O3 molecule is bent (a) form soluble complexes with anionic species
(c) Ozone is violet-black in solid state (b) precipitate anionic species
(d) Ozone is diamagnetic gas (c) form soluble complexes with cationic species
(d) precipitate cationic species
16. The reaction of white phosphorus with aqueous NaOH gives
phosphine alongwith another phosphorus containing 28. The number of S  S bonds in sulphur trioxide trimer,
compound. The reaction type, the oxidation states of (S3 O9 ) is (2001, 1M)
phosphorus in phosphine and the other product respectively (a) three (b) two
are (2012) (c) one (d) zero
(a) redox reaction, − 3 and − 5 29. Ammonia can be dried by (2000, 1M)
(b) redox reaction, 3 and + 5 (a) conc. H2SO4 (b) P4O10
(c) disproportionation reaction, − 3 and + 5 (c) CaO (d) anhydrous CaCl 2
(d) disproportionation reaction, − 3 and + 3
30. Amongst H2 O, H2 S, H2 Se and H2 Te, the one with the
17. Which ordering of compounds is according to the highest boiling point is (2000, 1M)
decreasing order of the oxidation state of nitrogen? (2012)
(a) H2O because of hydrogen bonding
(a) HNO3 , NO, NH4Cl, N2 (b) HNO3 , NO, N2 , NH4Cl
(b) H2Te because of higher molecular weight
(c) HNO3 , NH4Cl, NO, N2 (d) NO, HNO3 , NH4Cl, N2
(c) H2S because of hydrogen bonding
18. Extra pure N 2 can be obtained by heating (2011) (d) H2Se because of lower molecular weight
(a) NH3 with CuO (b) NH4NO3
31. The correct order of acidic strength is (2000, 1M)
(c) (NH4 )2 Cr2O7 (d) Ba(N3 )2 (a) Cl 2O7 > SO2 > P4O10 (b) CO2 > N2O5 > SO3
19. The reaction of P4 with X leads selectively to P4 O6 . The X, is (c) Na 2O > MgO > Al 2O3 (d) K 2O > CaO > MgO
(a) dry O2 (2009) 32. The number of P  O  P bonds in cyclic metaphosphoric
(b) a mixture of O2 and N2 acid is (2000, 1M)
(c) moist O2 (a) zero (b) two (c) three (d) four
(d) O2 in the presence of aqueous NaOH 33. One mole of calcium phosphide on reaction with excess
water gives (1999, 2M)
20. The percentage of p-character in the orbitals forming
(a) one mole of phosphine
P—P bonds in P4 is (2007, 3M)
(b) two moles of phosphoric acid
(a) 25 (b) 33 (c) 50 (d) 75
(c) two moles of phosphine
21. Which of the following is not oxidised by O3 ? (2005, 1M) (d) one mole of phosphorus pentaoxide
(a) KI (b) FeSO4
34. Sodium thiosulphate is prepared by (1996, 1M)
(c) KMnO4 (d) K 2MnO4
(a) reducing Na 2SO4 solution with H2S
22. Which gas is evolved when PbO2 is treated with (b) boiling Na 2SO3 solution with S in alkaline medium
concentrated HNO3? (2005) (c) neutralising H2S2O3 solution with NaOH
(a) NO2 (b) O2 (d) boiling Na 2SO3 solution with S in acidic medium
(c) N2 (d) N2O
35. There is no S  S bond in (1991, 1M)
23. A pale blue liquid obtained by equimolar mixture of two (a) S 2O42– (b) S 2O52– (c) S2O32– (d) S2O72–
gases at – 30°C is (2005, 1M)
(a) N2O (b) N2O3 36. Which one of the following is the strongest base? (1989, 2M)
(c) N2O4 (d) N2O5 (a) AsH3 (b) NH3 (c) PH3 (d) SbH3
24. Which of the following isomers of phosphorus is 37. Amongst the trihalides of nitrogen, which one is least basic?
thermodynamically most stable? (2005, 1M) (a) NF3 (b) NCl 3 (1987, 1M)
(a) Red (b) White (c) NBr3 (d) NI3
(c) Black (d) Yellow 38. Which of the following oxides of nitrogen is a coloured gas ?
25. Which of the following has —O—O— linkage? (2004, 3M) (a) N2O (b) NO (1987,1M)
(a) H2S2O6 (b) H2S2O8 (c) N2O4 (d) NO2
(c) H2S2O3 (d) H2S4O6
39. The bonds present in N2 O5 are (1986, 1M)
26. For H3PO3 and H3PO4, the correct choice is (2003, 1M) (a) only ionic (b) covalent and coordinate
(a) H3PO3 is dibasic and reducing (c) only covalent (d) covalent and ionic
(b) H3PO3 is dibasic and non-reducing
(c) H3PO4 is tribasic and reducing 40. A gas that cannot be collected over water is (1985, 1M)
(a) N2 (b) O2 (c) SO2 (d) PH3
(d) H3PO3 is tribasic and non-reducing
230 p-Block Elements-II

41. Ammonia gas can be dried by (1978, 1M) Integer Answer Type Question
(a) conc H2SO4 (b) P2O5
(c) CaCl 2 (d) quicklime 51. The total number of compounds having at least one bridging
oxo group among the molecules given below is ……… .
42. Which of the following is incorrect statement? (1978, 1M)
(a) NO is heavier than O2 N 2O 3, N 2O 5, P4O 6, P4O7 , H 4 P2O 5 , H 5P3O10, H 2S 2O 3,
(b) The formula of heavy water is D2O H 2S 2O 5
(c) N2 diffuses faster than oxygen through an orifice (2018 Adv.)
(d) NH3 can be used as a refrigerant
Assertion and Reason
Objective Questions II Read the following questions and answer as per the direction
(One or more than one correct option) given below:
43. The compound(s) which generate (s) N2 gas upon thermal (a) Statement I is correct, Statement II is correct, Statement II
is the correct explanation of Statement I
decomposition below 300°C is (are) (2018 Adv.)
(b) Statement I is correct, Statement II is correct, Statement II
(a) NH4NO3 (b) (NH4 )2 Cr2O7
is not the correct explanation of Statement I
(c) Ba(N3 )2 (d) Mg3N2
(c) Statement I is correct, Statement II is incorrect
44. Based on the compounds of group 15 elements, the correct (d) Statement I is incorrect, Statement II is correct
statement(s) is (are) (2018 Adv.)
52. Statement I Nitrogen and oxygen are the main components
(a) Bi 2O5 is more basic than N2O5
in the atmosphere but these do not react to form oxides of
(b) NF3 is more covalent than BiF3
nitrogen.
(c) PH3 boils at lower temperature than NH3
Statement II The reaction between nitrogen and oxygen
(d) The N—N single bond is stronger than the P—P single bond requires high temperature. (1998, 2M)
45. The nitrogen containing compound produced in the reaction 53. Statement I The electronic structure of O3 is
of HNO 3 with P4 O10 (2016 Adv.) ⊕
(a) can also be prepared by reaction of P4 and HNO 3 O
(b) is diamagnetic O Os
(c) contains one NN bond
Statement II The following structure is not allowed
(d) reacts with Na metal producing a brown gas
because octet around O cannot be expanded.
46. The correct statement(s) about O 3 is/are (2013 Adv.) O
(a) O — O bond lengths are equal O O (1998, 2M)
(b) thermal decomposition of O3 is endothermic
54. Statement I HNO3 is a stronger acid than HNO2 .
(c) O3 is diamagnetic in nature
(d) O3 has a bent structure Statement II In HNO3 , there are two nitrogen to oxygen
47. The nitrogen oxide(s) that contain(s) N—N bond(s) is/are bonds whereas in HNO2 there is only one. (1998, 2M)
(2009) 55. Statement I Although PF5 , PCl 5 and PBr5 are known, the
(a) N2O (b) N2O3
pentahalides of nitrogen have not been observed.
(c) N2 O4 (d) Ν 2Ο 5
Statement II Phosphorus has lower electronegativity than
48. Ammonia, on reaction with hypochlorite anion, can form nitrogen. (1994, 2M)
(1999, 3M)
(a) NO (b) NH4Cl Passage Based Questions
(c) N2H4 (d) HNO2
Passage 1
49. White phosphorus (P4 ) has (1998, 2M) Upon heating KClO 3 in presence of catalytic amount of
(a) six P  P single bonds MnO 2 , a gas W is formed. Excess amount of W reacts with
(b) four P  P single bonds white phosphorus to give X . The reaction of X with pure
(c) four lone pairs of electrons HNO 3 gives Y and Z. (2017 Adv.)
(d) PPP angle of 60°
56. Y and Z are, respectively
50. Nitrogen (I) oxide is produced by (1989, 1M) (a) N2O4 and HPO3 (b) N2O4 and H3PO3
(a) thermal decomposition of NH4NO3 (c) N2O3 and H3PO4 (d) N2O5 and HPO3
(b) disproportionation of N2O4
(c) thermal decomposition of NH4NO2
57. W and X are, respectively
(a) O2 and P4O10 (b) O2 and P4O6
(d) interaction of hydroxylamine and nitrous acid
(c) O3 and P4O6 (d) O3 and P4O10
p-Block Elements-II 231

Passage 2 Codes
P Q R S P Q R S
There are some deposits of nitrates and phosphates in earth’s
(a) 4 2 3 1 (b) 3 2 1 4
crust. Nitrates are more soluble in water. Nitrates are
(c) 1 4 2 3 (d) 3 4 2 1
difficult to reduce under the laboratory conditions but
microbes do it easily. Ammonia forms large number of Fill in the Blanks
complexes with transition metal ions. Hybridisation easily
explains the ease of sigma donation capability of NH3 and 62. The lead chamber process involves oxidation of SO2 by
PH3 . Phosphine is a flammable gas and is prepared from atomic oxygen under the influence of ………as catalyst.
white phosphorus. (2008, 3 × 4M = 12M) (1992, 1M)

58. Among the following, the correct statement is 63. In P4 O10 , the number of oxygen atoms bonded to each
(a) Phosphates have no biological significance in humans phosphorus atom is ……… . (1992, 1M)
(b) Between nitrates and phosphates, phosphates are less abundant 64. The basicity of phosphorus acid (H3 PO3 ) is ………
in earth’s crust
(1990, 1M)
(c) Between nitrates and phosphates, nitrates are less abundant in
earth’s crust 65. ……… phosphorus is reactive because of its highly strained
(d) Oxidation of nitrates is possible in soil tetrahedral structure. (1987, 1M)

59. Among the following, the correct statement is True/False


(a) Between NH3 and PH3, NH3 is a better electron donor because
the lone pair of electrons occupies spherical ‘s’ orbital and is 66. Nitric oxide, though an odd electron molecule, is
less directional diamagnetic in liquid state. (1991, 1M)
(b) Between NH3 and PH3, PH3 is a better electron donor because 67. The H  N  H bond angle in NH3 is greater than the
the lone pair of electrons occupies sp3-orbital and is more H  As  H bond angle in AsH3 . (1984, 1M)
directional
68. In aqueous solution, chlorine is a stronger oxidising agent
(c) Between NH3 and PH3, NH3 is a better electron donor because
than fluorine. (1984, 1M)
the lone pair of electrons occupies sp3-orbital and is more
2+
directional 69. Dilute HCl oxidises metallic Fe to Fe . (1983, 1M)

(d) Between NH3 and PH3, PH3 is a better electron donor because
the lone pair of electrons occupies spherical ‘s’ orbital and is Numerical Answer Type Questions
less directional 70. The amount of water produced (in g) in the oxidation of
60. White phosphorus on reaction with NaOH gives PH3 as one 1 mole of rhombic sulphur by conc. HNO 3 to a compound
with the highest oxidation state of sulphur is …… (Given
of the products. This is a
(a) dimerisation reaction (b) disproportionation reaction data : Molar mass of water = 18 g mol −1 ) (2019 Adv.)
(c) condensation reaction (d) precipitation reaction 71. The total number of lone pair of electrons in N2O3 is
(2015 Adv.)
Match the Column
72. Among the following, the number of compounds that can
61. The unbalanced chemical reactions given in Column I show react with PCl 5 to give POCl 3 is O2 , CO2 , SO2 , H2 O,
missing reagent or condition (?) which are provided in H2 SO4 , P4 O10 . (2011)
Column II. Match Column I with Column II and select the
correct answer using the codes given below the Columns. 73. The total number of diprotic acids among the following is
(2013 Adv.) H3PO4 H2SO4 H3PO3
H2CO3 H2S2O7 H3BO3
Column I Column II H3PO2 H2CrO4 H2SO3 (2010)
P. ? 1. NO
PbO2 + H 2SO4 → PbSO4 + O2 + Subjective Questions
other product
74. Draw the structure of P4O10. (2005)
? NaHSO + 2. I2
Q. Na2S2O3 + H 2O → 4 75. Arrange the following oxides in the increasing order of
other product
Bronsted basicity.
Cl2O7, BaO, SO3, CO2, B2O3 (2004)
? N + other product 3. Warm
R. N 2H 4 → 2 76. Identify the compounds A, B, C, D
SO Na 2CO3 Elemental S I2
? Xe + other product 4. Cl 2 Na 2 CO3 →
2
A → B → C → D
S. XeF2 →

and give oxidation state of sulphur in each compounds.
(2003, 4M)
232 p-Block Elements-II

77. Write the balanced equations for the reactions of the (iii) Manufacture of phosphoric acid from phosphorus.
following compounds with water: (iv) Reaction of aluminium with aqueous sodium hydroxide.
(i) Al 4C3 (ii) CaNCN (iii) BF3 (iv) NCl 3 (v) XeF4 (2002, 5M) (1997, 1M × 4 = 4M)
78. Give reason(s), why elemental nitrogen exists as a diatomic 85. Draw the structure of P4 O10 and identify the number of
molecule whereas elemental phosphorus is a tetra atomic single and double P  O bonds. (1996, 3M)
molecule? (2000, 2M)
86. Account for the following. Write the answers in four or five
79. The Haber’s process can be represented by the following sentences only.
scheme. (i) The experimentally determined N  F bond lengths in NF3
CaCO3 is greater than the sum of the single bond covalent radii of N
and F.
CaO + CO2 (ii) Mg 3 N2 when reacted with water gives of NH3 but HCl is
H2O NH3.H2O not obtained from MgCl 2 on reaction with water at room
temperature.
B (iii) (SiH3 )3 N is a weaker base than (CH3 )3 N.
NaHCO3 (1995, 2M × 3 = 6M)
NaCl
87. Complete and balance the following reactions. (1994, 1M)
+D
Heat K + 5CaSO ⋅ 2H O + K
Ca 5 (PO4 )3 F + H2SO4 + H2O →
C + H2O 4 2
NH3 + H2O + E 88. In the following reaction, identify the compounds A and B
A
Identify A, B, C, D and E . (1999, 5M)
PCl 5 + SO2 → A + B (1994, 1M)

80 (a) In the following equation 89. Complete and balance the following reaction.
A + 2B + H2 O → C + 2D Red phosphorus is reacted with iodine in the presence of
water.
(A = HNO2 , B = H2SO3 , C = NH2OH).
P + I2 + H2 O → K + K (1992, 1M)
Identify D. Draw the structures of A, B, C and D.
90. Give reasons in two or three sentences only.Sulphur dioxide
(b) In the contact process for industrial manufacture of
is a more powerful reducing agent in the alkaline medium
sulphuric acid, some amount of sulphuric acid is used as a
than in acidic medium. (1992, 2M)
starting material. Explain briefly. What is the catalyst used
in the oxidation of SO2? (1999, 10M) 91. Draw the two resonance structures of ozone which satisfy
the octet rule. (1991, 1M)
81. Complete and balance the following chemical equations.
(i) P4O10 + PCl 5 → 92. Give reasons in one or two sentences.
Ammonium chloride is acidic in liquid ammonia solvent.
(ii) SnCl 4 + C2H5Cl + Na → (1998, 1 M × 2 = 2M)
(1991, 1M)
82. (a) Thionyl chloride can be synthesised by chlorinating SO2 93. Write the balanced chemical equations for the following.
using PCl5. Thionyl chloride is used to prepare
(i) Sodium nitrite is produced by absorbing the oxides of
anhydrous ferric chloride starting from its hexahydrated
nitrogen in aqueous solution of washing soda.
salt. Alternatively, the anhydrous ferric chloride can also
be prepared from its hexahydrated salt by treating with (ii) Nitrogen is obtained in the reaction of aqueous ammonia
2,2-dimethoxypropane. Discuss all this using balanced with potassium permanganate.
chemical equations. (iii) Elemental phosphorus reacts with concentrated HNO3 to
(b) Reaction of phosphoric acid with Ca3(PO4)2 yields a give phosphoric acid.
fertiliser “triple superphosphate” represent the same (iv) Sulphur is precipitated in the reaction of hydrogen
through balanced chemical equation. (1998, 5M) sulphide with sodium bisulphite solution.
83. A soluble compound of a poisonous element M, when (v) Carbon dioxide is passed through a suspension of
heated with Zn / H2 SO4 , gives a colourless and extremely limestone in water. (1991, 1 × 5 = 5M)
poisonous gaseous compound N, which on passing through 94. Write the balanced chemical equation for the following
a heated tube gives a silvery mirror of element M. Identify reactions.
M and N. (1997, 2M)
(i) Aqueous solution of sodium nitrate is heated with zinc
84. Write balanced equations for the following. dust and caustic soda solution.
(i) Phosphorus is treated with concentrated nitric acid. (ii) Sodium iodate is added to a solution of sodium bisulphite
(1990, 2M)
(ii) Oxidation of hydrogen peroxide with potassium
permanganate in acidic medium. 95. Write the two resonance structures of N2 O that satisfy the
octet rule. (1990, 2M)
p-Block Elements-II 233

96. Draw balanced equations for 105. Write down the balanced equation for the reactions when
(i) the preparation of phosphine from CaO and white (i) calcium phosphate is heated with a mixture of sand and
phosphorus. carbon.
(ii) the preparation of ammonium sulphate from gypsum, (ii) ammonium sulphate is heated with a mixture of nitric
ammonia and carbon dioxide. (1990, 2M)
oxide and nitrogen dioxide. (1985, 2M)
97. Explain the following (1989, 2M)
106. Draw the resonance structures of nitrous oxide.
(i) H3 PO3 is a dibasic acid. (1985, 90, 2M)
(ii) Phosphine has lower boiling point than ammonia.
107. Show with balanced chemical reaction what happens when
98. Write the balanced chemical equations for the following. following are mixed?
(i) Hypophosphorous acid is heated. Aqueous solution of ferric sulphate and potassium iodide.
(ii) Sodium chlorate reacts with sulphur dioxide in dilute (1984, 1M)
sulphuric acid medium.
108. Write the matched set (of three) for each entry in Column A
99. Arrange the following as indicated. CO2 , N2 O5 , SiO2 , SO3
A B C
in the order of increasing acidic character.
Asbestos Paramagnetic Air pollutant
100. Give balanced equations for the following.
(i) Phosphorus reacts with nitric acid to give equimolar ratio Lithium metal Silicates of Ca and Mg Electron donor
of nitric oxide and nitrogen dioxide. Nitric oxide Reducing agent
(ii) Carbon dioxide is passed through a concentrated aqueous
(1984, 2M)
solution of sodium chloride saturated with ammonia.
(1988, 3M) 109. Complete and balance the following reactions.
101. Give reason for “valency of oxygen is generally two, whereas (i) HNO3 + HCl → NO + Cl 2
sulphur shows valency of two, four and six.” (1988, 1M)
(ii) Ce3+ +S2O82– → SO2– 4+
4 + Ce
102. Explain the following in one or two sentences.
(iii) Cl 2 + OH– → Cl – + ClO– (1983, 3M)
(i) Magnesium oxide is used for the lining of steel making
furnace. 110. Explain, “orthophosphoric acid, H3 PO4 is tribasic but
(ii) The mixture of hydrazine and hydrogen peroxide with a phosphorous acid, H3 PO3 is dibasic”. (1982, 1M)
copper (II) catalyst is used as a rocket fuel.
(iii) Orthophosphorous acid is not tribasic acid. 111. Give structural formula for the following.
(iv) The molecule of magnesium chloride is linear, whereas (i) Phosphorous acid, H3 PO3
that of stannous chloride is angular. (1987, 4M) (ii) Pyrophosphoric acid, H4 P2 O7 (1981, 2M)
103. Write balanced equations for the following. (1987, 2M) 112. Sulphur melts to a clear mobile liquid at 119°C, but on
(i) Phosphorus is reacted with boiling aqueous solution of further heating above 160° C, it becomes viscous, explain.
sodium hydroxide in an inert atmosphere. (1981, 1M)
(ii) Dilute nitric acid is slowly reacted with metallic tin. 113. Explain the following in not more than two sentences .
104. Complete and balance the following reactions. (i) Conc. HNO3 turns yellow in sunlight.
(i) S + OH– → S2– + S2 O2–
3 + ...... (ii) Bleaching powder loses its bleaching properties when it
(ii) ClO–3 + I– + H2 SO4 → Cl – + HSO–4 + ...... + ...... is kept in an open bottle for a long time. (1980, 2M)
(1986, 2M)

Topic 2 Elements and Compounds of Group 17 and 18


Objective Questions I (Only one correct option) 2. The number of bonds between sulphur and oxygen atoms in
1. The electron gain enthalpy (in kJ/mol) of fluorine, chlorine, S2O2−
8 and the number of bonds between sulphur and
bromine and iodine, respectively, are (2020 Main, 7 Jan I) sulphur atoms in rhombic sulphur, respectively, are
(a) −333, −325, −349 and −296 (2020 Main, 8 Jan I)
(a) 4 and 6 (b) 8 and 8 (c) 4 and 8 (d) 8 and 6
(b) −296, −325, −333 and −349
(c) −333, −349, −325 and −296 3. The noble gas that does not occur in the atmosphere is
(2019 Main, 10 April II)
(d) −349, −333, −325 and −296
(a) Ra (b) Kr (c) He (d) Ne
234 p-Block Elements-II

4. Chlorine on reaction with hot and concentrated sodium 14. The shape of XeO2 F2 molecule is (2012)
hydroxide gives (2019 Main, 12 Jan II) (a) trigonal bipyramidal (b) square planar
(a) Cl − and ClO− (b) Cl − and ClO3− (c) tetrahedral (d) see-saw
(c) ClO3− and ClO−2 (d) Cl − and ClO−2 15. Aqueous solution of Na 2 S2 O3 on reaction with Cl 2 gives
5. Iodine reacts with concentrated HNO3 to yield Y along with (2008, 3M)
(a) Na 2S4O6 (b) NaHSO4
other products. The oxidation state of iodine in Y , is
(2019 Main, 12 Jan I) (c) NaCl (d) NaOH
(a) 1 (b) 3 (c) 7 (d) 5 16. When I− is oxidised by KMnO4 in alkaline medium, I−
6. Among the following reactions of hydrogen with halogens, converts into (2004, 1M)
the one that requires a catalyst is (2019 Main, 10 Jan II) (a) IO−3 (b) I2
(a) H2 + Cl 2 → 2HCl (b) H2 + I2 → 2HI (c) IO−4 (d) IO−
(c) H2 + F2 → 2HF (d) H2 + Br2 → 2HBr
7. The type of hybridisation and number of lone pair(s) of 17. The set with correct order of acidic strength is (2001, 1M)
(a) HClO < HClO2 < HClO3 < HClO4
electrons of Xe in XeOF 4 , respectively, are
(2019 Main, 10 Jan I) (b) HClO4 < HClO3 < HClO2 < HClO
(a) sp3d 2 and 1 (b) sp3d and 2 (c) HClO < HClO4 < HClO3 < HClO2
(d) HClO4 < HClO2 < HClO3 < HClO
(c) sp3d and 1 (d) sp3d 2 and 2
18. Which one of the following species is not a pseudo halide?
8. Which of the following reactions is an example of a redox (1997, 1M)
reaction? (2017 Main) (a) CNO– (b) RCOO− (c) OCN− (d) NNN−
(a) XeF4 + O2F2 → XeF6 + O2
19. The following acids have been arranged in the order of
(b) XeF2 + PF5 → [XeF] + PF6−
decreasing acidic strength. Identify the correct order.
(c) XeF6 + H2O → XeOF4 + 2HF ClOH (I), BrOH (II), IOH (III) (1996, 1M)
(d) XeF6 + 2H2O → XeO2F2 + 4HF (a) I > II > III (b) II > I > III
(c) III > II > I (d) I > III > II
9. The products obtained when chlorine gas reacts with cold and 20. KF combines with HF to form KHF2 . The compound
dilute aqueous NaOH are (2017 Main)
contains the species (1996, 1M)
(a) ClO− and ClO−3 (b) ClO−2 and ClO−3
(a) K + , F − and H+ (b) K + , F – and HF
(c) Cl − and ClO− (d) Cl − and ClO−2
(c) K + and [HF2 ] – (d) [KHF]+ and F –
10. Which among the following is the most reactive?
(2015 Main)
21. Bromine can be liberated from potassium bromide solution
(a) Cl 2 (b) Br2 (c) I2 (d) ICl by the action of (1987, 1M)
(a) iodine solution (b) chlorine water
11. Which one has highest boiling point? (2015 Main) (c) sodium chloride (d) potassium iodide
(a) He (b) Ne (c) Kr (d) Xe
22. Chlorine acts as a bleaching agent only in the presence of
12. Under ambient conditions, the total number of gases released (1983, 1M)
as products in the final step of the reaction scheme shown (a) dry air (b) moisture
below is (2014 Adv.) (c) sunlight (d) pure oxygen
Complete
hydrolysis 23. HBr and HI reduce sulphuric acid, HCl can reduce KMnO4
XeF6 P + Other product and HF can reduce (1981, 1M)
HO–/H2O (a) H2SO4 (b) KMnO4
Q (c) K 2Cr2O7 (d) None of these
Slow disproportionation
in HO–/H2O Objective Questions II
Products (One or more than one correct option)
24. With respect to hypochlorite, chlorate and perchlorate ions,
(a) 0 (b) 1 (c) 2 (d) 3 choose the correct statement(s). (2020 Adv.)
13. Among the following oxoacids, the correct decreasing order (a) The hypochlorite ion is the strongest conjugate base.
of acidic strength is (2014 Main) (b) The molecular shape of only chlorate ion is influenced by the
(a) HOCl > HClO2 > HClO3 > HClO4 lone pair of electrons of Cl.
(b) HClO4 > HOCl > HClO2 > HClO3 (c) The hypochlorite and chlorate ions disproportionate to give
(c) HClO4 > HClO3 > HClO2 > HOCl rise to identical set of ions.
(d) HClO2 > HClO4 > HClO3 > HOCl (d) The hypochlorite ion oxidises the sulphite ion.
p-Block Elements-II 235

25. The correct statement(s) about the oxoacids, HClO4 and 32. Bleaching powder contains a salt of an oxoacid as one of its
HClO, is (are) (2017 Adv.) components. The anhydride of that oxoacid is
(a) The central atom in both HCl O4 and HClO is sp3-hybridised (a) Cl 2O (b) Cl 2O7 (c) ClO2 (d) Cl 2O6
(b) HCl O4 is formed in the reaction between Cl 2 and H2O
(c) The conjugate base of HCl O4 is weaker base than H2O Passage 3
(d) HCl O4 is more acidic than HClO because of the resonance The noble gases have closed-shell electronic configuration
stabilisation of its anion and are monoatomic gases under normal conditions. The
26. The colour of the X 2 molecules of group 17 elements low boiling points of the lighter noble gases are due to weak
changes gradually from yellow to violet down the group. dispersion forces between the atoms and the absence of
other interatomic interactions.
This is due to (2017 Adv.)
(a) decrease in π * − σ * gap down the group The direct reaction of xenon with fluorine leads to a series of
(b) decrease in ionisation energy down the group compounds with oxidation numbers + 2, + 4 and + 6. XeF4
(c) the physical state of X 2 at room temperature changes from gas reacts violently with water to give XeO3 . The compounds of
to solid down the group xenon exhibit rich stereochemistry and their geometries can
(d) decreases in HOMO-LUMO gap down the group be deduced considering the total number of electron pairs in
the valence shell. (2007, 3 × 4M = 12 M)
27. The compound(s) with two lone pairs of electrons on the
central atom is (are) (2016 Adv.) 33. Argon is used in arc welding because of its
(a) BrF5 (b) ClF3 (c) XeF4 (d) SF4 (a) low reactivity with metal
(b) ability to lower the melting point of metal
28. The correct statement(s) regarding,
(c) flammability
(i) HClO, (ii) HClO2 , (iii) HClO3 and (iv) HClO4 is (are)
(d) high calorific value
(a) the number of Cl == O bonds in (ii) and (iii) together is two
(b) the number of lone pair of electrons on Cl in (ii) and (iii) 34. The structure of XeO3 is
together is three (a) linear (b) planar
(c) the hybridisation of Cl in (iv) is sp3 (c) pyramidal (d) T-shaped
(d) amongst (i) to (iv), the strongest acid is (i) 35. XeF4 and XeF6 are expected to be
(a) oxidising (b) reducing
Passage Based Questions (c) unreactive (d) strongly basic
Passage 1
The reactions of Cl 2 gas with cold-dilute and hot-concentrated Match the Column
NaOH in water give sodium salts of two (different) oxoacids 36. All the compounds listed in Column I react with water.
of chlorine, P and Q, respectively. The Cl 2 gas reacts with Match the result of the respective reactions with the
SO2 gas in the presence of charcoal, to give a product R. R appropriate options listed in Column II. (2010)
reacts with white phosphorus to give a compound S. On
hydrolysis, S gives an oxoacid of phosphorus T. (2013 Adv.) Column I Column II
29. P and Q respectively, are the sodium salts of A. (CH3 )2 SiCl 2 p. Hydrogen halide formation
(a) hypochlorous and chloric acids B. XeF4 q. Redox reaction
(b) hypochlorous and chlorous acids
C. Cl 2 r. Reacts with glass
(c) chloric and perchloric acids
(d) chloric and hypochlorous acids D. VCl5 s. Polymerisation
t. O2 formation
30. R, S and T, respectively, are
(a) SO2Cl 2 , PCl 5 and H3PO4 (b) SO2Cl 2 , PCl 3 and H3PO3
(c) SOCl 2 , PCl 3 and H3PO2 (d) SOCl 2 , PCl 5 and H3PO4 Fill in the Blank
37. The increase in solubility of iodine in aqueous solution of KI
Passage 2 is due to the formation of ….. (1982, 94, 1M)
Bleaching powder and bleach solution are produced on a
large scale and used in several household products. The True/False
effectiveness of bleach solution is often measured by
38. HBr is a stronger acid than HI because of hydrogen
iodometry. (2012)
bonding. (1993, 1M)
31. 25 mL of household bleach solution was mixed with 30 mL
of 0.50 M KI and 10 mL of 4 N acetic acid. In the titration of Numerical Answer Type Questions
the liberated iodine, 48 mL of 0.25 N Na 2 S2 O3 was used to
reach the end point. The molarity of the household bleach 39. At 143 K, the reaction of XeF4 with O 2F2 produces a xenon
solution is compound Y . The total number of lone pair(s) of electrons
(a) 0.48 M (b) 0.96 M (c) 0.24 M (d) 0.024 M present on the whole molecule of Y is ............ (2019 Adv.)
236 p-Block Elements-II

40. Reaction of Br2 with Na 2 CO3 in aqueous solution gives 50. Mention the products formed in the following
sodium bromide and sodium bromate with evolution of CO2 “Chlorine gas is bubbled through a solution of ferrous
gas. The number of sodium bromide molecules involved in bromide.” (1986, 2M)
the balanced chemical equation is (2011) 51. Complete and balance the following reaction:
ClO–3 + I– + H2SO4 → Cl – + HSO–4 + ......+ ...... (1986, 2M)
Subjective Questions
52. Arrange the following in the order of
41. Write the balanced equation for the reaction of the following
compound with water. (i) increasing bond strength HCl, HBr, HF, HI
(ii) increasing oxidation number of iodine
XeF4 (2002, 5M)
I2 , HI, HIO4 , ICl (1986, 2M)
42. Draw molecular structures of XeF2 , XeF4 and XeO2 F2 ,
53. Give reason in one or two sentences.
indicating the locations of lone pair(s) of electrons. Fluorine cannot be prepared from fluorides by chemical
(2000, 3M)
reduction method. (1985, 1M)
43. Give an example of oxidation of one halide by another
halogen. Explain the feasibility of the reaction. (2000, 2M) 54. Complete and balance the following reaction.
Cl 2 + OH– → Cl – + ClO– (1983, 3M)
44. Work out the following using chemical equations
“Chlorination of calcium hydroxide produces bleaching 55. Explain the following in not more than two sentences.
powder.” (1998, 2M) Bleaching powder loses its bleaching properties when it is
45. Complete the following chemical equations: kept in an open bottle for a long time. (1980, 2M)
(i) KI + Cl 2 → (ii) KClO3 + I2 → (1996, 2M) 56. Give reasons for the following in one or two sentences.
46. Give reasons in two or three sentences only for (i) Hydrogen bromide cannot be prepared by the action of
(i) Bond dissociation energy of F2 is less than that of Cl 2 . conc. sulphuric acid on sodium bromide.
(ii) Sulphur dioxide is a more powerful reducing agent in the (ii) When a blue litmus paper is dipped into a solution of
alkaline medium than in acidic medium. (1992, 2M) hypochlorous acid, it first turns red and then later gets
47. Write the balanced chemical equation for the following: decolourised. (1979, 2M)
Sodium bromate reacts with fluorine in the presence of alkali.
57. Write the balanced equations involved in the preparation of
48. Arrange the following as indicated. HOCl, HOClO2 , (i) bleaching powder from slaked lime (1979, 10M)
HOClO3 , HOClO in increasing order of thermal stability (ii) nitric oxide from nitric acid
(1988, 2M) (iii) chlorine from sodium chloride
49. Give balanced equation for the following: (iv) anhydrous aluminium chloride from alumina
Iodate ion reacts with bisulphite ion to liberate iodine. (1988, 3M)

Answers
Topic 1
1. (a) 2. (b) 3. (c) 4. (b) 65. (white) 66. (T) 67. (T) 68. (F)
5. (d) 6. (a) 7. (a) 8. (d) 69. (T) 70. (288) 71. (8) 72. (4)
9. (b) 10. (d) 11. (d) 12. (a) 73. (6)
13. (a) 14. (b) 15. (c) 16. (c)
Topic 2
17. (b) 18. (d) 19. (b) 20. (d)
1. (c) 2. (b) 3. (a) 4. (b)
21. (c) 22. (b) 23. (b) 24. (c)
5. (d) 6. (b) 7. (a) 8. (a)
25. (b) 26. (a) 27. (c) 28. (d)
9. (c) 10. (d) 11. (d) 12. (c)
29. (c) 30. (a) 31. (a) 32. (c)
13. (c) 14. (a) 15. (a) 16. (a)
33. (c) 34. (b) 35. (d) 36. (b)
17. (a) 18. (b) 19. (a) 20. (c)
37. (a) 38. (d) 39. (b) 40. (c)
21. (b) 22. (b) 23. (d) 24. (a,b,d)
41. (d) 42. (a) 43. (b,c) 44. (a,b,c)
25. (a, c, d) 26. (b, c) 27. (b, c) 28. (b, c)
45. (b, d) 46. (a, c, d) 47. (a, b, c) 48. (c)
29. (a) 30. (a) 31. (c) 32. (a)
49. (a, c, d) 50. (a, d) 51. (6) 52. (a)
33. (a) 34. (c) 35. (a)
53. (a) 54. (a) 55. (b) 56. (a)
36. (A → p, s B → p, q, r, t C → p, q, t D → p)
57. (b) 58. (c) 59. (c) 60. (b)
37. (KI3 ) 38. (F) 39. (19) 40. (5)
61. (d) 62. (NO2 ) 63. (Four) 64. (Two)
Hints & Solutions
Topic 1 Elements and Compounds of H2 S 2 O3 (thiosulphuric acid), H2 S 2 O4 (hyposulphurous or
Group 15 and 16 dithionous acid) and H2 S 4 O6 (tetrathionic acid) contains SS
bonds.
1. The correct statement is that (SiH3 )3 N is planar and less basic O
than (CH3 )3 N. The compounds trimethylamine (CH3 )3 N and
trisilylamine (SiH3 )3 N have similar formulae, but have totally H2S2O3 ⇒ HO—S—OH
different structures. In trimethylamine the arrangement of
S
electrons is as follows :
O OH
1s 2s 2p H2S2O4 ⇒ S——S
Electronic structure of
HO O
nitrogen atom
(ground state) O O
Three unpaired electrons
form bonds with CH3 groups H2S4O6 ⇒ HO—S—S—S—S—OH
tetrahedral arrangements of
three bond pairs and one lone pair
O O
In trisilylamine, three sp2 orbitals are used for
4. The correct increasing order of oxidation state of nitrogen for
σ -bonding, giving a plane triangular structure. nitrogen oxides is
+1 +2 +3 +4
H3Si SiH3 N 2 O< N O < N 2 O3 < N O2
N N l
Oxidation state of N in N2O is
H 3C CH3 2(x ) − 2 = 0
CH3 SiH3 2
N(SiH3)3 x = + = +1
N(CH3)3 2
molecule molecule
l
Oxidation state of N in NO is
x−2= 0
2. In C60 (Buckminster fullerene) twenty hexagons and twelve
x = +2
pentagons are present which are interlocked resulting a shape
of soccer ball. Every ring in this structure is aromatic.
l
Oxidation state of N in N2O3 is
2x + 3(−2) = 0
6
x= =3
2
l
Oxidation state of N in NO2 is
x + 2(−2) = 0
x−4 = 0
x = +4
5. Let us consider the structure of the phosphorus oxyacids,
Phosphorus has large atomic size and less electronegativity, so
it forms single bond instead of pπ-pπ multiple bond. So, it P P
consists of discrete tetrahedral P4 molecule as shown below : H OH HO H
H OH
P H3PO2 H3PO3
Hypophosphorus Orthophosphorus
P P acid acid
(P—H bonds=2) (P—H bond=1)
P

∴ Number of trigons (triangles) = 4 P P P P


H O H O OH
3. S  S bond is not present in H2 S 2 O7 (pyrosulphuric acid or HO OH OH OH
H 4P 2O 5 H4P2O6
oleum).
Pyrophosphorus Hypophousphoric
O O acid acid
(P—H bonds=2) (P—H bond=0)
H2S2O7 ⇒ HO—S—O—S—OH
6. As given, the first electron gain enthalpy of oxygen can be shown
O O as,
While the other given oxoacids of sulphur, i.e. O(g )+ e− → O − (g ), ∆egH 1 = − 141kJ/mol
238 p-Block Elements-II

The expression of second electron gain enthalpy of oxygen will be, 12. PLAN This problem is based on chemical properties of
O − (g )+ e− → O 2− (g,) ∆egH 2 = + ve phosphorus.

∆egH 2 of oxygen is positive, i.e. endothermic, because a strong White phosphorus on reaction with thionyl chloride (SOCl 2 )
electrostatic repulsion will be observed between highy negative O − produces phosphorus trichloride.
and the incoming electron (e− ). A very high amount of energy will P4 (s) + 8SOCl 2 (l ) → 4PCl 3 (l ) + 4SO2 (g ) + 2S2Cl 2 (g )
be consumed (endothermic) by the system to overcome the But if amount of thionyl chloride (SOCl 2 ) is in excess then it
electrostatic repulsion. produces phosphorus pentachloride.
7. The structure of H3PO2 (hypophosphorous) acid is P4 + 10SOCl 2 (l ) → 4PCl 5 + 10SO2
O 13. NO is paramagnetic in gaseous state because in gaseous state,
P it has one unpaired electron.

HO H
H
Total number of electrons present = 7 + 8 = 15 e −
Due to the presence of two P  H bonds, H3 PO 2 acts a strong Hence, there must be the presence of unpaired electron in
reducing agent. e.g. gaseous state while in liquid state, it dimerises due to
+1 +1 0 +5 unpaired electron.
4 Ag NO 3 + H3PO 2 + 2H2O → 4 Ag ↓ + H3 PO 4 + 4 HNO 3
8. The thermal decomposition of given compounds is shown below 14. NO 2 is a brown coloured gas and imparts this colour to
concentrated HNO 3 during long standing.

(NH4 )2 Cr2O7 → N2 + 4H2O + Cr2O3 4 HNO 3 → 2H2O + 2NO 2 + 3O 2


NH4NO2 → N2 + 2H2O 15. (a) ONCl = 8 + 7 + 17 = 32 e


ONO− = 8 + 7 + 8 + 1 = 24 e− (correct)
(NH4 )2 SO4 → 2NH3 +H2SO4
O
(b)
Ba(N3 )2 → Ba + 3N2
O O
Thus, only (NH4 )2 SO4 does not gives N2 on heating (It give NH3 ).
While rest of the given compounds gives N2 on their thermal Central O-atom is sp 2 -hybridised with 1 lone pair, so
decomposition. bent shape (correct).
+5 +4 +3 +1
9. H3 P O 4 > H4 P2 O6 > H3 P O 3 > H3 P O 2 (c) In solid state, ozone is violet-black. Ozone does not
exist in solid state, thus incorrect.
10. Species Hybridisation (d) O 3 has no unpaired electrons, so diamagnetic (correct).
Hence, (c) is the correct.
N sp2
– 16. The reaction of white phosphorus with aqueous alkali is
O O
O P4 + 3NaOH + 3H2O → PH3 + NaH2PO2
N In the above reaction, phosphorus is simultaneously
– sp2 +1
O O oxidised [P4 (0) → NaH2 P O2 ] as well as reduced
−3
[P4 (0) → P H3 ]. Therefore, this is an example of
N sp2
O O disproportionation reaction. Oxidation number of
+
O N O sp phosphorus in PH3 is − 3 and in NaH2PO2 is + 1. However,
+ 1 oxidation number is not given in any option, one might
O
think that NaH2PO2 has gone to further decomposition on

11. Orthophosphorous acid, H3PO3 : HO  P  OH heating.
 ∆ +5
H 2NaH2PO2 → Na 2H P O4 + PH3
x
H3 PO3 = 3 + x + 3( −2 ) = 0 or x = + 3 17. Let oxidation number of N be x.
Pyrophosphorous acid, H 4 P2 O 5 : In HNO3, + 1 + x + 3 (− 2) = 0 ⇒ x = + 5
O O In NO, x−2=0 ⇒ x=+ 2
  In N2, x=0
HO  P  O  P  OH In NH4Cl, x + 4 −1= 0 ⇒ x = − 3
 
H H 18. Ba(N3 )2 Heat
→ Ba(s) + 3N2 (g )
x
Azide salt of barium can be obtained in purest form as well as
H4 P2 O5 = 4 + 2x + 5 ( − 2) = 0
the decomposition product contain solid Ba as by product
4 + 2x − 10 = 0, x = + 3
p-Block Elements-II 239

alongwith gaseous nitrogen, hence no additional step of 28. The structure of S3O9 is
separation is required. O O
Other reactions are S
Heat
NH4NO3 → N2O + 2H2O O O
O O
Heat
2NH3 + 3CuO → 3Cu + 3H2O + N2 S S
O O O
Heat
(NH4 )2 Cr2O7 → Cr2O3 + 4H2O + N2 It has no S—S linkage.
19. In limited supply of oxygen, phosphorus is oxidised to its 29. CaO, a basic oxide, is most suitable for drying of basic ammonia.
lower oxide P4O6 while excess of oxygen gives P4O10. A mixture 30. H2O, due to its ability to form intermolecular H-bonds.
of O2 and N2 is used for controlled oxidation of phosphorus into
P4O6. 31. Corresponding acids are HClO4, H2SO3 and H3PO4. Hence, the
3 order of acidic strength is
20. In P4, all phosphorus are sp -hybridised and has 75%
Cl 2 O7 > SO2 > P4 O10
p-character.
P 32. The structure of cyclic metaphosphate is
sp3 –
O O
P P P
O O
O O
P P P
– –
O O
O
21. In KMnO4, Mn is already in its highest oxidation state (+7),
cannot be oxidised by any oxidising agent. There is three P—O—P bonds.

22. PbO2 + HNO3 → Pb(NO3 )2 + H2O + O2 33. Ca 3P2 + 6H2O → 3Ca(OH)2 + 2PH3
OH −
23. Equimolar amounts of NO and NO2 at –30°C gives N2O3 (l ) 34. Na 2SO3 + S → Na 2S2O3

which is a blue liquid.
35. S2O27 − has no S—S linkage.
−30 ° C
NO( g ) + NO2 ( g ) → N2 O3( l ) O O
( Blue)

 
O— S — O— S — O−
24. Black phosphorus is thermodynamically most stable allotrope  
of phosphorus. O O
It is due to three dimensional, network structure of polymeric All others have atleast one S—S linkage.
black phosphorus.
36. Amongst XH3 where ‘X ’ is group-15 elements, basic strength
25. H2S2O8 is a peroxy acid, has—O—O—linkage decreases from top to bottom. Hence, NH3 is strongest base.
O O 37. The electron withdrawing inductive effect of halogen decreases
  electron density on nitrogen, lowers basic strength. Since,
HO— S — O— O— S — OH fluorine is most electronegative, NF3 is least basic.
 
O O 38. NO2 (g ) is deep brown coloured.
Peroxodisulphuric acid
39. In N2O5, there are σ (sigma) covalent bonds, π (pi) bonds and
26. H3PO3 is a dibasic, reducing acid. H3PO4 is tribasic, coordinate covalent bonds as
non-reducing acid. O O
O O N N
  O
H— P— OH HO— P— OH O O
 
OH OH 40. SO2 cannot be collected over water because it reacts with water
Dibasic, reducing Tribasic, non -reducing
forming H2SO3.
27. Polyphosphates are used as water softening agents because SO2 + H2 O → H2 SO3
they form soluble complexes with cationic species of hard
water.
41. Quicklime (CaO) is used for drying NH3 gas because both are
basic, do not react. On the other hand, H2SO4 and P2O5 are
Na 2[Na 4 (PO3 )6 ]+ CaSO4 → Na 2[(Ca 2 (PO3 )6 ] + Na 2SO4
acidic, reacts with ammonia forming salts. CaCl 2 forms
Soluble complex
complex with ammonia.
240 p-Block Elements-II

42. NO is lighter than O2. (b) N2O5 has no unpaired electron and is thus, diamagnetic
thus, (b) is correct.
D2O is commonly known as heavy water.
(c)
N2 is lighter than O2, effuse at faster rate under identical O O
experimental conditions. NH3 liquefies at very low temperature.
N O N
Therefore, liquid NH3 is used as a refrigerant.
O O
43. Among the given compounds, those which generate N2 on thermal There is no N—N bond, thus, (c) is incorrect.
decomposition below 300°C are ammonium dichromate i.e., (d) N2O5 + Na → NaNO3 + NO2
(NH4 )2Cr2 O7 and barium azide or nitride i.e., Ba(N3 )2 . Reactions N2O5 vapours are of brownish colour. Thus, (d) is
of their thermal decomposition are given below correct.

(i) (NH4 )2 Cr2O7 → N2 ↑ + Cr2O3 + 4H2O 46. Plan Due to resonance, bond lengths between two atoms are
Below 300 °C equal. Species is said to be diamagnetic if all electrons are
It is an exothemic reaction with paired.
∆H = − 429.1 ± 3 kcal/ mol. Process is endothermic if it takes place with absorption of
∆ heat.
(ii) Ba(N3 )2 → Ba + 3N2 ↑
Around 160 ° and above
O 218 O
r
O
r
Ammonium nitrate (NH4NO 3) on heating below 300°C gives pm s
116.80°
O →→ s
N 2O as O O O
O O

NH4NO3 → N2O + 2H2O bent molecule all electrons paired thus, diamagnetic
below 300 °C
2 O 3 → 3 O 2 ∆H ° = − 142 kJ mol −1
However, on rapid heating or explosion
Exothermic
(i.e. above 300°C) it gives off nitrogen as
Rapid heating Thus, (b) is incorrect. (a, c, d) are correct.
2NH4NO3  → 2N2 + O2 + 4H2O
or explosion 47. The structures of these oxides are
Magnesium nitride (Mg3N2 ) does not decompose at lower O O
temperatures being comparatively more stable. Its thermal
N N → O N N
decomposition requires a minimum temperature of 700°C and
proceeds as

→
− 1500° C
Mg3N2 700
 → 3Mg + N2 ↑
(a) (b) O
44. Statement wise explanation is O O O O
(i) Statement (a) Bi 2O5 is a metallic oxide while N2O5 is a
non-metallic oxide. N N N N
O


Metallic oxides being ionic are basic in nature while non


O O O O
metallic oxides being covalent are acidic in nature. This
confirms more basic nature of Bi 2O5 in comparison to N2O5. (c) (d)
Hence, this is a correct statement.
(a), (b), (c) have N—N bonds.
(ii) Statement (b) The electronegativity difference between N(3) and
F(4) is less as compared to the electronegativity difference 48. 2NH3 + OCl − → H2N— NH2 + H2O + Cl −
between Bi (1.7) and F(4). More electronegativity difference
leads to ionic compounds. Thus, NF3 must be more covalent in 49. The structure of P4 is
nature as compared to BiF3. Hence, this statement is also correct. P
(iii) Statement (c) In NH3 intermolecular hydrogen bonding is
present, which is altogether absent in PH3. Thus, PH3 boils at
P P
lower temperature than NH3.
Hence, this is also a correct statement.
(iv) Statement (d) Due to smaller size of N the lone pair-lone pair P
repulsion is more in N—N single bond as compared to O—P
single bond. This results to weaker N—N single bond as It has six P—P single bonds.
compared to P—P single bond. Hence, this statement is incorrect. There are four lone pairs on four phosphorus. P—P—P
45. P4O10 is a dehydrating agent and converts HNO3 into N2O5 bond angles are of 60°.
Heat
2HNO3 → N2O5 + H2O 50. NH4NO3 → N2O + 2H2O
P4O10 + 6H2O → 4H3PO4 NH2OH ⋅ HCl + NaNO2 → NaCl + 2H2O + N2O
(a) P4 + 20HNO3 → 4H3PO4 + 20NO2 + 4H2O
However, NH4 NO2 on heating gives N2 .
Thus, (a) is incorrect.
p-Block Elements-II 241

51. The structures of various molecules given in problem are 8. H2S 2 O5


discussed below— O O
1. N 2 O3 It is the tautomeric mixture of following two
S S
structures— HO OH
O N N O
O N N O O O
O Conclusion This compound also does not contain any
Bridging bridging oxo group.
oxo group
Conclusion 1 bridging oxo group is present in the compound.
52. Both Statement I and Statement II are true and Statement II is
correct explanation of Statement I.
2. N 2 O5 It has following structure.
O O O 53. Both Statement I and Statement II are true and Statement II is
N N correct explanation of Statement I.
O O
Bridging oxo 54. Both Statement I and Statement II are true and Statement II
group
explains the Statement I appropriately. Nitrate ion (NO−3 ) is
Conclusion 1 bridging oxo group is present in the compound. more stable than nitrite ion :
3. P4 O6 −
O O
P  
O −
O O— N→ O ←→ O == N→ O (Resonance structure)
O •• ••
P P
O N ←→ N (Resonance structure)

O O
O O O O−
P
55. Both Statement I and Statement II are independently correct
Conclusion 6 bridging oxo groups are present in the but reason is not the correct explanation of Statement I.
compound. Nitrogen does not has any vacant d-orbitals, it cannot expand
4. P4 O7 its valence shell beyond eight electrons, i.e. it cannot violate
octet. Therefore, nitrogen forms only trihalides
P
O O (NX 3 with eight electrons in valence shell of N).
Phosphorus has vacant 3d-orbitals, it can expand its valence
O
P P O shell beyond eight electrons, its both trihalides and
O pentahalides exist.
O O Passage 1
P
MnO 2
Conclusion 6 bridging oxo groups are present in the KClO3 → KCl + O2

compound. W

5. H4 P2 O5 ∆ HNO 3
O O O2 + P4 → P4O10 → N2O5 + HPO3
X Y Z
P P
H O H
56. (a) 57. (b)
OH OH Passage 2
Conclusion 1 bridging oxo group is present in the compound.
58. Due to greater solubility in water and prone to microbial
attack, nitrates are less abundant in earth’s crust.
6. H 5P3O 10
59. NH3 is stronger Lewis base than PH3. In a group of hydrides,
O O O
basic strength decreases down the group.
P P P 60. White phosphorus undergo disproportionation in alkaline
HO O O OH medium.
OH OH OH
P4 + NaOH → PH3 + NaH2 PO2
Conclusion 2 bridging oxo groups are present in the
Warm (3)
compound. 61. (P) 2PbO2 + 2H2SO4 → 2PbSO4 + O2 + 2H2O
7. H2S 2 O3
Cl 2 ( 4 )
S (Q) Na 2S2O3 + H2O → NaHSO4 + HCl
S I2 (2 )
HO OH (R) N2H4 → N2 + Hl
O NO(1)
Conclusion This compound does not contain any bridging (S) XeF2 → Xe + NOF
oxo group. Thus, P—(3), Q—(4), R—(2), S—(1)
242 p-Block Elements-II

Oxides of N2
62. NO2 : 2SO2 (g ) + O2 (g ) → 2SO3 (g )
72. PCl5 produces POCl3 with the following reagents
(NO2 ) PCl 5 + SO2 → POCl 3 + SOCl 2
63. O PCl 5 + H2O → POCl 3 + 2HCl
2PCl 5 + H2SO4 → SO2Cl 2 + 2POCl 3 + 2HCl
O P O
6PCl 5 + P4O10 → 10POCl 3

O 73. Diprotic acids = 6


O
O P P O O O O

— —


HO—S—OH P——OH HO—C—OH
P O H OH
O O
O O O O O


— —
— —
— —
Here four oxygen atoms are bonded to each phosphorus HO—S—O—S—OH HO—Cr—OH HO—S—OH
atom.
O O O
64. H3PO3 [O == PH(OH)2 ] is a dibasic acid.
Others are
65. White phosphorus has highly strained, tetrahedral O O
structure, therefore highly reactive.



66. In liquid state, nitric oxide (NO) dimerises into (NO)2 P HO—B—OH P
and odd electrons disappear giving diamagnetic property. HO OH OH OH H H OH
2NO → O == N— N== O ( l ) Triprotic Monobasic Monoprotic
Paramagnetic Diamagnetic Lewis acid

67. Both ‘N’ and ‘As’ in corresponding hydrides are 74. O


sp3-hybridised. If central atoms are from same group, P
bond angle decreases from top to bottom if all other O O
things are similar. Hence, H—N—H bond angle in NH3
O O
is greater than H—As—H bond angle in AsH3. O
O P P
68. Halogens are all good oxidising agent and their oxidising
power decreases from top to bottom (F2 to I2) in group. P
Any halogen above in group oxidises halides down in O O
group from their aqueous solution. Hence, Cl 2 can O
(P4O10)
oxidise Br − to Br2, I− to I2 but cannot oxidise F− to F2
rather F2 can oxidise Cl − to Cl 2. 75. Cl 2O7 < SO3 < CO2 < B2O3 < BaO
+4 +4 +2
69. Fe is more electropositive than hydrogen, displaces H+
+2.5
76. A = NaH SO3 ; B = Na 2 SO3 ; C = Na 2 S2O3; D = Na 2 S4 O6
ions from acid solution as :
Fe + 2HCl → FeCl 2 + H2 ↑ 77. (i) Al 4C3 + 12H2O → 4Al(OH)3 + 3CH4
(ii) CaNCN + 5H2O → CaCO3 + 2NH4OH
70. Key Idea Rhombic sulphur (S8 ) gets oxidised into
(iii) 4BF3 + 3H2O → H3BO3 + 3HBF4
sulphuric acid and water, NO2 gas is released on (iv) NCl 3 + 3H2O → NH3 + 3HOCl
reaction with conc. HNO3.
(v) 2XeF4 + 3H2O → Xe + XeO3 + F2 + 6HF
When rhombic sulphur (S8 ) is oxidised by conc. HNO 3
78. Nitrogen in N2 are bonded by one sigma and two pi bonds. Phosphorus
then H2SO 4 is obtained and NO 2 gas is released.
and other elements of this period, due to larger size, are very less likely
S8 + 48HNO 3 → 8H2SO 4 + 48NO 2 + 16 H2O
to form pi bonds, hence P4 is formed in which there is no pi bonds.
1 mole of rhombic sulphur produces = 16 moles of H2O
∴ Mass of water = 16 × 18 (molar mass of H2O) = 288 g 79. In given scheme : A = Ca (OH)2
71. N2O3 has two proposed structures. B = NH4HCO3, C = Na 2CO3
•• D = NH4Cl and E = CaCl 2
O•
• ••
•• •• •• •• •• •• 80. (a) HNO2 + 2H2SO3 + H2O → NH2OH + 2H2SO4
O == N  O  N == O and •• N N A B C D
•• •• •• O
•• •• (b) In SO3 + H2O → H2SO4, sulphuric acid is obtained in misty
O
•• form and the reaction is explosive. By adding H2SO4, above
In both cases, number of lone pair of electrons are eight. reaction is prevented :
p-Block Elements-II 243

H2SO4 + SO3 → H2S2O7 (oleum) Heat


87. Ca 5 (PO4 )3 F + 5H2SO4 + 10H2O → 3H3PO4
H2S2O7 + H2O → 2H2SO4
In the contact process, V2O5 is used as catalyst. + 5CaSO4 ⋅ 2H2 O + HF
88. PCl 5 + SO2 → POCl 3 + SOCl 2
81. (i) P4O10 + 6PCl 5 → 10 POCl 3 A B
(ii) SnCl 4 + 2C2H5Cl + 2Na → Na 2SnCl 4 + C4H10 89. Red phosphorus reacts with iodine in the presence of water to
82. (a) PCl 5 + SO2 → POCl 3 + SOCl 2 form H3PO3 and HI as–
2P + 3I2 + 6H2 O → 2H3PO3 + 6HI
(b) Ca 3 (PO4 )2 + 4H3PO4 → 3Ca(H2PO4 )2
triple superphosphate 90. SO2 acts as reducing agent on account of following reaction :
83. The poisonous element M may be As. On the basis of given SO2 + 2OH − → SO42 − + 2H + + 2e−
information
Hence, the above reaction proceeds in forward direction on
Zn /HCl
AsCl 3 + 6H → AsH3 + 3HCl increasing concentration of HO− ion. H+ is on product side,
N adding H+ retards the reaction by sending it in backward

2AsH3 → 2As + 3H2 direction.
M
91. + +
84. (i) P4 + 20HNO3 → 4H3PO4 + 20NO2 + 4H2O O O
(ii) 3KMnO4 + 5H2O2 + 3H2SO4 → K2SO4 – –
O O O O
+ 2MnSO4 + 5O2 + 8H2O
(iii) P4 + 20HNO3 → 4H3PO4 + 20NO2 + 4H2O 92. Ammonia, in liquid state undergo self-ionisation as :
(iv) 2Al + 2NaOH + 2H2O → 2NaAlO2 + 3H2 2NH3 q NH4+ + NH2−
Thus, addition of NH4 Cl to liquid ammonia increases
85. O
concentration of NH4+ in solution and NH4 Cl act as acid.
O P O
93. (i) Na 2CO3 + NO + NO2 → 2NaNO2 + CO2
4 P==O bonds (ii) 2KMnO4 + 2NH3 → 2MnO2 + 2KOH + 2H2 O + N2
O
O P O P O (iii) P4 + 20HNO3 → 4H3PO4 + 20NO2 + 4H2 O
(iv) 2H2 S + NaHSO3 + H+ → 3S + 3H2 O + Na +
P (v) CaCO3 + CO2 + H2 O → Ca(HCO3 )2
O O
O 94. (i) NaNO2 + Zn + NaOH → 3Na 2ZnO2 + NH3 + H2O
(ii) 2NaIO3 + 5NaHSO3 → 3NaHSO4 + 2Na 2 SO4
86. (i) The size of both nitrogen and fluorine are very small as well + I2 + H 2 O
as they have very high electron density. Thus in NF3, N and + − − +
F repel each other stretching the N—F bond. Hence, in NF3, 95. N ≡≡ N— O ←→ N == N == O
N—F bond lengths are greater than the sum of their single ∆
bond covalent radii. 96. (i) 15CaO + 4P4 → 5Ca 3P2 + 3P2O5 ↑
(ii) Mg 3N2 + 6H2 O → 3Mg(OH)2 + 2NH3 [Ca 3P2 + 6H2 O → 3Ca(OH)2 + 2PH3 ↑ ] × 5
MgCl 2 is a salt of strong acid HCl and strong base
15CaO + 4P4 + 30H2 O → 15Ca(OH)2
Mg(OH)2 and therefore, not hydrolysed in aqueous
solution. + 3P2 O5 ↑ + 10PH3 ↑
(iii) In (SiH3 )3 N, the lone pair of nitrogen is involved in pπ - dπ (ii) 2NH3 + CO2 + H2 O → (NH4 )2 CO3
bonding, less available on nitrogen for donation to a Lewis CaSO4 + (NH4 )2 CO3 → CaCO3 ↓ + (NH4 )2 SO4
acid, a weaker Lewis base gypsum
pπ-dπ CaSO4 + 2NH3 + CO2 + H2O → CaCO3 ↓ + (NH4 )2 SO4
H3Si—N——SiH3
97. (i) In H3PO3, there is only two replaceable H, hence dibasic
SiH3 O

Carbon does not have any vacant d-orbitals, no such pπ -dπ H— P — OH H—of OH are acidic, dibasic.
bonding occur in trimethyl amine, lone pair of nitrogen is 
available for donation to Lewis acid, hence a stronger Lewis OH
base. (ii) NH3 molecules are associated by intermolecular
H—bonds.
244 p-Block Elements-II

98. (i) 2H3PO2



→ PH3 + H3PO4 (Disproportionation) 109. (i) 2HNO3 + 6HCl → 2NO + 3Cl 2 + 4H2O
hypophosphorus (ii) 2Ce3 + + S2 O28 − → 2SO24 − + 2Ce4 +
acid
(iii) Cl 2 + 2OH− → Cl − + ClO− + H2 O
10 H +
(ii) NaClO3 + SO2  → NaCl + S + 5H2O
110. Orthophosphoric acid (H3PO4 ) has three replaceable (acidic)
99. SiO2 < CO2 < N2O5 < SO3 hydrogen while orthophosphorus acid (H3PO3 ) has only two
replaceable hydrogen.
100. (i) 4P + 10HNO3 + H2O → 5NO + 5NO2 + 4H3PO4
O O
(ii) NaCl + NH4OH + CO2 → NH4Cl + NaHCO3  
HO— P — OH O— P — OH
101. Oxygen lacks empty d-orbitals in its valence shell, cannot
 
violate octet rule, hence in most of its compound it show only OH OH
divalency. On the other hand, sulphur has vacant 3d-orbitals in orthophosphoric acid orthophosphorus acid
its valence shell, can violate octet rule, show di, tetra and hexa (it has three acidic H) (only two acidic H, H directly
bonded to P is not acidic)
valency.
111. O O O
102. (i) MgO is used for the lining of steel making furnace because (i) (ii)
it forms slag with impurities, and thus helps in removing P P P
H OH HO O OH
them from iron.
OH HO OH
(ii) The mixture of N2 H4 and H2 O2 (in presence of Cu(II) phosphorus acid (pyrosphosphoric acid)
catalyst) is used as a rocket propellant because the reaction (P is sp3-hybridised)
is highly exothermic and large volumes of gases is evolved.
112. Rhombic sulphur has a eight membered puckered ring structure.
N2H4 (l ) + 2H2O2 (l ) → N2 (g ) + 4H2O (g )
On heating ring tends to break and linear chain sulphur is
(iii) In orthophosphorus acid (H3PO3 ) only two of the three H formed. When sulphur melts, the S8 rings slip and roll over
are replaceable as one another very easily. It gives rise to a clear mobile liquid.
O
When liquid sulphur is further heated to higher temperature,

H— P — OH
rings are broken giving long chain sulphur molecules. This
 long chain molecules of sulphur gets entangled into one
OH another increasing viscosity of melt.
(Only H of —OH are acidic) 113. (i) In the presence of sunlight, concentrated nitric acid
(iv) In MgCl 2, Mg is sp-hybridised while in SnCl 2, Sn is decomposes partially as
sp2-hybridised with a lone pair at Sn. Hence, MgCl 2 is hν
Conc. HNO3 → NO2 + H+ + O2
linear while SnCl 2 is angular.
Inert atm.
It is the NO2 which impart yellow colouration to nitric acid.
103. (i) P4 + 3NaOH + 3H2O → 3NaH2PO2 + PH3 (ii) The bleaching action of bleaching powder is due to
(phosphine) presence of available chlorine, but in contact of moisture,
(ii) 4Sn + 10HNO3 → 4Sn(NO3 )2 + NH4 NO3 + 3H2 O it releases chlorine decreasing the amount of available
dil chlorine. Hence, bleaching property decreases gradually as
104. (i) 4S + 6OH− → 2S2 − + S2O32− + 3H2O bleaching powder is kept in open container for long time.
(ii) ClO3– + 6I– + 6H2SO4 → Cl – + 6HSO–4 +3I2 + 3H2O
Topic 2 Elements and Compounds of

105. (i) 2Ca 3 (PO4 )2 + 6SiO2 + 10C → 6CaSiO3 Group 17 and 18
+ 10CO + P4 1. Electron gain enthalpy (∆ eg H ) is the enthalpy change for
(white)
(ii) (NH4 )2 SO4 + NO + NO2 → 2N2 + 3H2 O + H2 SO4 converting 1 mol of isolated atoms to anions by adding
electrons. All halogens have negative ∆ eg H (exothermic)
+ − − +
106. N ≡≡ N— O ←→ N== N ==O values. Generally, ∆ eg H becomes less negative when
comparing elements of the same group from top to bottom.
107. Fe2 (SO4 )3 + 2KI → 2FeSO4 + K2SO4 + I2 But among fluorine and chlorine there is an anomaly because
In the above reaction, strong reducing agent, iodide, reducing inter-electron repulsion is stronger in fluorine due to its extra
ferric salt into ferrous salt. small size.
∴ ∆ e g H is less exothermic than expected for F-atom.
108. A B C Thus, the correct values of electron gain enthalpies
Asbestos Silicates of Ca and Mg Donar F < Cl > Br > I
Lithium metal Reducing agent Electron donor ( −333) ( −349) ( −325) ( −296 )
kJ mol −1
Nitric oxide Paramagnetic Air pollutant
p-Block Elements-II 245

2. S 2O2−
8 is 8. The reaction in which oxidation and reduction occur
simultaneously are termed as redox reaction.
+4 + 1 + 6 0
XeF4 + O2 ( F2 ) → X eF6 + O 2
O—S—O—O—S—O
Since, Xe undergoes oxidation while O undergoes reduction.
So, it is an example of redox reaction.
Total number of S  O or S==O bond = 8
9. Cl 2, Br2 and I2 form a mixture of halide and hypohalites when
S S S
S S react with cold dilute alkalies while a mixture of halides and
Rhombic sulphur is S haloate when react with concentrated cold alkalies.
S S
Cl 2 + 2NaOH → NaCl + NaClO + H2O
Total number of S S bond = 8
Cold and dilute
Thus, the correct answer is (b).
∴ Cl − and ClO− are obtained as products when chlorine gas
3. Radium (Ra) is a radioactive element. Ra belongs to group 2 reacts with cold and dilute aqueous NaOH.
(alkaline earth metals), it is not a noble gas.
Note In question noble gas which does not exist in the 10. Interhalogen compounds are generally more reactive than
atmosphere is asked and answer is Ra. But Ra (radium) is an halogens (except fluorine).
alkaline earth metal and not noble gas. It can be Rn (radon) and 11. Xe has highest boiling point.
is misprint in JEE Main Paper.
12. PLAN This problem can be solved by using concept involved in
4. Halogens form halates and halides with hot and concentrated chemical properties of xenon oxide and xenon fluoride.
solution of NaOH as : XeF6 on complete hydrolysis produces XeO3.
3 X 2 + 6NaOH → 5NaX + NaXO3 + 3H2 O
XeO3 on reaction with OH− produces HXeO−4 which on further
So, Cl2 will also give Cl− (as NaCl) and ClO3−
treatment with OH− undergo slow disproportionation reaction
(as NaClO3 ) in the above reaction. and produces XeO64− along with Xe(g ), H2O(l ) and O2 (g ) as a
Thus, option (b) is correct. by-product.
Note When halogens react with cold and dilute solution of
Oxidation half-cell in basic aqueous solution
NaOH, hypohalites and halides are produced as:
X 2 + 2NaOH → NaX + NaXO + H 2O HXeO− + 5OH− → XeO4− + 3H O + 2e–
4 6 2

5. Iodine reacts with concentrated HNO 3to yield HIO 3 along Reduction half-cell in basic aqueous solution
with NO 2 and H2 O. The reaction involved in as follows : HXeO− + 3H O + 6e− → Xe + 7OH−
4 2
I2 + 10HNO3 → 2HIO3 + 10NO2 + 4H2O Balanced overall disproportionation reaction is
The oxidation state of ‘I’ in HIO 3 is + 5 as calculated below : 4HXeO−4 + 8OH− → 3XeO64− + Xe + 6H2O
14 4244 3
1 + x + 3(−2) = 0 2 products
x − 5 = 0, x = + 5 Complete sequence of reaction can be shown as
6. Chemical reactivity of halogens decreases down the group. XeF6+3H2O XeO3+ 3H2F2
The chemical reactivity follows the order.

OH
∴ F2 > Cl2 > Br2 > I2.
The highest reactivity of fluorine is attributed to two factors: HXeO–4
(i) The low dissociation energy of F  F bond (which results in
OH–/H2O (disproportionation)
low attraction energy for the reaction).
(ii) Very strong bonds which are formed. Both properties arise XeO64-(s) + Xe(g) + H2O(l ) + O2(g)
from, small size of fluorine. I2 is being the least reactive
halogen, it requires a catalyst for the reaction. Thus, (c) is the correct answer.
H2 + I2 → 2HI
13. Decreasing order of strength of oxoacids
7. In XeOF4 , Xe is sp3d 2-hybridised. Geometry of the molecule is HClO4 > HClO3 > HClO2 > HOCl
octahedral, but shape of the molecule is square pyramidal.
Reason Consider the structures of conjugate bases of each
According to VSEPR, theory it has one π bond. Remaining six oxyacids of chlorine.
electron pairs form an octahedron with one position occupied by a
lone pair. O
O Cl Cl
Cl –
F F
O O
– O O O Cl—O –
Number of bp = 5 O O–
Xe O
Number of lp = 1
Negative charge is more delocalised on ClO4− due to
F F
resonance, hence, ClO−4 is more stable (and less basic).
Here, Xe contains one lone pair of electrons.
246 p-Block Elements-II

Hence, we can say as the number of oxygen atom(s) around Weak acid have strong conjugate base thus hypochlorite ion
Cl-atom increases as oxidation number of Cl-atom increases and has strongest conjugate base. Therefore, statement (a) is
thus, the ability of loose the H+ increases. correct.
14. In XeO2F2, the bonding arrangement around the central atom Xe is (b) Hypochlorite ion is linear and perchlorate ion is tetrahedral
and there is no effect of lone pair on hypochlorite ion. Thus
O Xe O statement (b) is correct.
F F
4σ bonds + 1.0 l p = 5
Cl—O– –
O—Cl==O –
O—Cl==O
Hybridisation of Xe = sp3d
sp3d-hybridisation corresponds to trigonal bipyramidal geometry.
Hypochlorite Chlorate Perchlorate
Also, in trigonal bipyramidal geometry, lone pairs remain ion ion ion
present on equatorial positions in order to give less electronic
(c) In the disproportionation reaction, chlorate ion Cl(+ 5 ) is
repulsion.
oxidised to perchlorate, Cl(+ 7) and reduced to chloride,
F F
O O Cl (−1).
Remove 4ClO −3 → 3ClO 4− + Cl −
Xe Xe
lone-pair
While in hypochlorite ion, chlorite ion Cl(+ 1) is oxidised to
O O chlorate, Cl(+ 5) and reduced to chloride, Cl(− 1) ion.
F F
See-saw shape 3ClO− → ClO3− + 2 Cl −
NOTE According to Bent’s rule, the more electronegative atoms must Thus, statement (c) is incorrect.
be present on axial position. Hence, F are kept on axial (d) The hypochlorite ion oxidises the sulphite ion to
positions. sulphate ion, because HOCl is the strongest oxidising
15. Sodium thiosulphate, Na 2S2O3 gets oxidised by chlorine water Cl oxyacids,
as Na 2S2O3 + 4Cl 2 + 5H2O → 2NaHSO4 + 8HCl ClO − + SO 23− → SO 24− + Cl −
FeCl 3 oxidises Na 2S2O3 to Na 2S4O6. Thus, statement (d) is correct.
16. I– is oxidised by MnO–4 in alkaline medium to form IO–3 25. (a) ClO−4 is more stable than ClO− .
2KMnO4 + KI + H2O → 2KOH + 2MnO2 + KIO3
(b) Incorrect : Cl 2 + H2O → HCl + HOCl
17. Amongst oxyacids of a given halogen, higher the oxidation (c) O
number of halogen, stronger the acid. Hence,
HOCl < HClO2 < HClO3 < HClO4. HO Cl O HO Cl

18. Pseudo halides must contain atleast one nitrogen atom. sp3 sp3
O
19. Among oxyacids of halogens, if there are same number of (d) HClO4 is stronger acid than H2 O.
oxygens bonded to central atom, higher the electronegativity of
26. Colour of halogen arises due to transition from HOMO to LUMO
halogen, stronger the acid. Hence,
in the visible region. On moving down a group, the difference in
IOH < BrOH < ClOH energy between HOMO and LUMO decreases electronic
20. All others has at least one S-S linkage. transition occur more easily and colour intensity increases.
KF + HF → K+ + HF2− 27.
21. Among halogens, oxidising power decreases from top to bottom. Compounds Hydridisation Structures
Lone pair on
Hence, the upper halogen oxidises lower halides from aqueous central atom
solution. Chlorine will oxidise bromide into bromine. F F
22. Moist chlorine gives nascent oxygen, act as oxidising agent : BrF5 sp3d 2 Br 1
Cl 2 + H2 O → HCl + HOCl F F
F
HOCl → HCl + [O] F
nascent oxygen
(bleaching action) F
ClF3 sp3d Cl 2
23. Fluorine, being the most electronegative, its size is very small.
Therefore, it does not have a tendency to loose electrons. Hence, F
HF does not act as a reducing agent.
F F
24. (a) Order of acid strength different oxyacids of chlorine are : XeF4 sp3d 2 Xe 2
(+1) (+5) (+7)
HClO < HClO3 < HClO4 F F
(Hypochlorous acid) (Chloric acid) (Perchloric acid) SF4 sp3d 1
p-Block Elements-II 247

12
28. H  O  Cl H  O  Cl == O ⇒ m mol of OCl − = = 6 m mol. Remaining part is solved in
2
(i) (ii)
the same manner.
O == O

==
32. Bleaching powder is Ca(OCl)Cl. Therefore, the oxoacid whose
H  O  Cl == O H  O  Cl == O salt is present in bleaching powder is HOCl. Anhydride of HOCl

==
O is Cl 2O as
(iii) (iv) 2 HOCl → Cl 2O + H2O
− NOTE The oxidation number of element in anhydride and oxoacid
(a) Number of Cl == O bonds in (ii) and
(iii) together is three. Hence, wrong. remains the same.
(b) Number of Lone Pair on Cl in (ii) and (iii) together is three. Passage 3 Q.Nos. (33 to 35)
Hence, correct.
33. Ar, being inert, provide inert atmosphere in arc welding, and
(c) In (iv), Cl is sp3-hybridised. Hence, correct. prevent from undesired oxidation.
(d) Amongst (i) to (iv), the strongest acid is (iv). Hence, wrong. ••
34. O == Xe == O
Passage 1 Q. Nos. (29-30)

Cold O
2NaOH + Cl 2 → NaCl + NaOCl + H2O
P Xe is sp3-hybridised with one lone pair. Hence, molecule of
hot
6NaOH + 3Cl 2 → 5NaCl + NaClO3 + 3H2O XeO3 has pyramidal shape.
Q
35. Both XeF4 and XeF6 are strong oxidising agent.
NaOH
HOCl → NaOCl
hypochlorous P 36. (CH3 )2 SiCl 2 + H2O → (CH3 )2 Si(OH)2 + 2HCl
acid
CH3 CH3 Cl
NaOH
HClO3 → NaClO3 Polymerisation
chloric acid Q —Si—O—Si—O—Si—O
Cl 2 + SO2 → SO2Cl 2
CH3 CH3 CH3
R silicone
10 SO2Cl 2 + P4 → 4PCl 5 + 10 SO2
3
R S 3XeF4 + 6H2O → XeO3 + 2Xe + 12HF + O2
PCl 5 + 4H2O → H3PO4 + 5 HCl 2
T 1
Cl 2 + H2O → HCl + HOCl → HCl + O2
Passage 2 Q.Nos. (31-32) 2
VCl 5 + H2O → VOCl 3 + 2HCl
31. The involved redox reactions are :
2H+ + OCl − + 2I− → Cl − + I2 + H2O …(i) 37. KI + I2 → KI3

I2 + 2S2O2−
3 → 2I + S4O62− …(ii) 38. Among HX, acidic strength increases from HF to HI.
Also the n-factor of S2O32− is one as 39. XeF4 reacts with O 2F2 to form XeF6 ⋅ O 2F2 is fluoronating
2S2O32− → S4O62− + 2e− reagent.
143 K
[one ‘e’ is produced per unit of S2O32− ] XeF4 + O 2F2 → XeF6 + O 2
(Y)
⇒ Molarity of Na 2S2O3 = 0.25 N × 1 = 0.25 M
⇒ m mol of Na 2S2O3 used up = 0.25 × 48 = 12 The structore of XeF6 is
Now from stoichiometry of reaction (ii) F
12 m mol of S2O2−
3 would have reduced 6 m mol of I2.
F F
Xe
From stoichiometry of reaction (i)
F F
m mol of OCl − reduced = m mol in I2 produced = 6
F
6
⇒ Molarity of household bleach solution = = 0.24 M Y compound (XeF6 ) has 3 lone pair in each fluorine and one lone
25
pair in xenon.
Shortcut Method Hence, total number of lone pairs electrons is 19.
Milliequivalent of Na 2S2O3 =milliequivalent of OCl −
40. Br2 is disproportionated in basic medium as
= 0.25 × 48 =12
3Br2 + 3Na 2CO3 → 5NaBr + NaBrO3 + 3CO2
Also n-factor of OCl − = 2 [Cl + → Cl − , gain of 2e− ]
41. 2XeF4 + 3H2O → Xe + XeO3 + F2 + 6HF
248 p-Block Elements-II

42. F F 49. 2IO−3 + 5HSO−3 → I2 + H2O + 3HSO−4 + 2SO24−


F F O
50. Cl 2 + FeBr2 → FeCl 3 + Br2
Xe Xe Xe
51. ClO−3 + 6I− + 6H2SO4 → Cl − + 6HSO−4 + 3I2 + 3H2O
F F
O 52. (i) Bond strength is inversely related to bond length. Hence,
F F bond energy : HI < HBr < HCl < HF
linear square planar see-saw shaped
(ii) HI(– 1) < I2 (0) < ICl(+1) < HIO4 (+7)
43. Halogen above in the group oxidises halide below to it from
their aqueous solution, e.g.
53. F2 itself, is the strongest oxidising agent. Therefore, chemical
Cl 2 + 2I− (aq) → 2Cl − + I2 reagent cannot oxidise fluoride to fluorine.

40 ° C 54. Complete and balance the following reactions


44. Ca(OH)2 + Cl 2 → CaOCl 2 + H2O
Cl 2 + 2OH− → Cl − + ClO− + H2O
45. (i) 2KI + Cl 2 → 2KCl + I2
55. The bleaching action of bleaching powder is due to presence of
(ii) 2KClO3 + I2 → 2KIO3 + Cl 2 available chlorine, but in contact of moisture, it releases chlorine
46. (i) Due to small size and high electron density of fluorine atom, decreasing the amount of available chlorine. Hence, bleaching
there exist a significant repulsions between fluorine atoms property decreases gradually as bleaching powder is kept in
in F2 , they have greater tendency to get apart. Hence, bond open container for long time.
energy of F2 is less than that of Cl 2 . This is against to 56. (i) HBr is a stronger reducing agent, reduces cencentrated
bond-length bond-energy relationship,. H2SO4 to SO2. Hence, HBr cannot be prepared by heating
(ii) Sulphur dioxide is a more powerful reducing agent in bromide salts with concentrated H2SO4.
alkaline medium because nascent hydrogen is produced in
(ii) Hypochlorous acid is acidic in nature, therefore it turns blue
the presence of moisture
litmus paper into red. However, HOCl is also an oxidising
i.e. SO2 + 2H2O → H2SO4 + 2H acid (bleaching), it bleaches red colour to finally colourless.
And alkaline solution neutralises the acid i.e. H2SO4 and 40 ° C
shift the equilibrium in the forward direction producing 57. (i) Ca(OH)2 + Cl 2 → CaOCl 2 + H2 O
more nascent hydrogen. But in acidic medium the
equilibrium will suppressed resulting in a lesser amount of (ii) 3Cu + 8HNO3 (dil) → 3Cu(NO3 )2 + 4H2 O + 2NO
nascent hydrogen. (iii) 2NaCl + 2H2 SO4 + MnO2 → Na 2 SO4 + MnSO4
47. NaBrO3 + 3F2 → 3F2O + NaBr + 2H2O + Cl 2

48. HOCl < HOClO < HOClO2 < HOClO3 ∆


(iv) Al 2 O3 + 3C + 3Cl 2 → 2AlCl 3 + 3CO
17
Transition and
Inner-Transition Elements
Objective Questions I (Only one correct option) 8. The maximum number of possible oxidation states of

1. The shape/structure of [ XeF5 ] and XeO 3F2, respectively, are actinoides are shown by (2019 Main, 9 April II)

(2020 Main, 2 Sep II) (a) berkelium, (Bk) and californium (Cf)
(a) pentagonal planar and trigonal bipyramidal (b) nobelium (No) and lawrencium (Lr)
(b) octahedral and square pyramidal (c) actinium (Ac) and thorium (Th)
(c) trigonal bipyramidal and pentagonal planar (d) neptunium (Np) and plutonium (Pu)
(d) trigonal bipyramidal and trigonal bipyramidal 9. The lanthanide ion that would show colour is
2. The incorrect statement(s) among (1)-(3) is (are) (2019 Main, 8 April I)
(2020 Main, 4 Sep II) (a) Gd3+ (b) Sm 3+ (c) La 3+ (d) Lu3+
1. W(VI) is more stable than Cr(VI). 10. The correct order of atomic radii is (2019 Main, 12 Jan II)
2. in the presence of HCl, permanganate titrations provide (a) Ho > N > Eu > Ce (b) N > Ce > Eu > Ho
satisfactory results. (c) Eu > Ce > Ho > N (d) Ce > Eu > Ho > N
3. some lanthanoid oxides can be used as phosphors. 2
11. A →
4 KOH, O
2B + 2H 2O
(a) 2 and 3 only (b) 2 only (Green)
(c) 1 only (d) 1 and 2 only
4 HCl
3. Thermal decomposition of a Mn compound (X ) at 513 K 3B → 2C + MnO2 + 2H 2O
(Purple)
results in compound (Y ), MnO2 and a gaseous product. MnO2
H 2O, KI
reacts with NaCl and concentrated H2 SO4 to give a pungent 2C → 2 A + 2KOH + D
gas Z. X , Y and Z, respectively, are (2019 Main, 12 April II)
(a) K 3MnO4 , K 2MnO4 and Cl 2 (b) K 2MnO4 , KMnO4 and SO2
In the above sequence of reactions, A and D, respectively, are
(2019 Main, 11 Jan II)
(c) KMnO4 , K 2MnO4 and Cl 2 (d) K 2MnO4 , KMnO4 and Cl 2
(a) KI and KMnO4 (b) MnO2 and KIO3
4. The pair that has similar atomic radii is (c) KI and K2MnO4 (d) KIO3 and MnO2
(a) Mn and Re (b) Ti and Hf
(c) Sc and Ni (d) Mo and W 12. The element that usually does not show variable oxidation
states is (2019 Main, 11 Jan I)
5. The correct order of the first ionisation enthalpies is
(a) Sc (b) Cu (c) Ti (d) V
(2019 Main, 10 April II)
st
(a) Mn < Ti < Zn < Ni (b) Ti < Mn < Zn < Ni 13. The 71 electron of an element X with an atomic number of 71
(c) Zn < Ni < Mn < Ti (d) Ti < Mn < Ni < Zn enters into the orbital (2019 Main, 10 Jan II)
(a) 4 f (b) 6p (c) 5d (d) 6s
6. The highest possible oxidation states of uranium and
plutonium, respectively, are (2019 Main, 10 April II) 14. The effect of lanthanoid contraction in the lanthanoid series
(a) 7 and 6 (b) 6 and 7 of elements by and large means (2019 Main, 10 Jan I)
(c) 6 and 4 (d) 4 and 6 (a) increase in atomic radii and decrease in ionic radii
(b) decrease in both atomic and ionic radii
7. Consider the hydrated ions of Ti 2 + , V2 + , Ti 3 + and Sc3+ . The (c) increase in both atomic and ionic radii
correct order of their spin-only magnetic moment is (d) decrease in atomic radii and increase in ionic radii
(2019 Main, 10 April I)
15. The transition element having least enthalpy of atomisation is
3+ 3+ 2+ 2+
(a) Sc < Ti < Ti < V (b) Sc < Ti 3+ < V2+ < Ti 2+
3+
(2019 Main, 9 Jan II)
(c) Ti 3+ < Ti 2+ < Sc3+ < V2+ (d) V2+ < Ti 2+ < Ti 3+ < Sc3+ (a) Zn (b) V (c) Fe (d) Cu
250 Transition and Inner-Transition Elements

16. In the following reactions, ZnO is respectively acting as a/an 25. Consider the following reaction, (2013 Main)
(2017 Main) z
xMnO−4 + y C2 O42 − + zH+ → xMn 2+ + 2 y CO2 + H2 O
(i) ZnO + Na2O → Na2 ZnO2 2
(ii) ZnO + CO2 → ZnCO3 The values of x, y and z in the reaction are, respectively
(a) base and acid (b) base and base (a) 5, 2 and 16 (b) 2, 5 and 8
(c) acid and acid (d) acid and base (c) 2, 5 and 16 (d) 5, 2 and 8

17. Sodium salt of an organic acid ‘X ’ produces effervescence 26. Which of the following arrangements does not represent the
with conc. H2SO4 . ‘X ’ reacts with the acidified aqueous correct order of the property stated against it? (2013 Main)
CaCl2 solution to give a white precipitate which decolourises (a) V2+ < Cr 2+ < Mn 2+ < Fe2+ : paramagnetic behaviour
acidic solution of KMnO4 . ‘X ’ is (2017 Main) (b) Ni 2+ < Co2+ < Fe2+ < Mn 2+ : ionic size
(a) C6H5COONa (b) HCOONa (c) Co3+ < Fe3+ < Cr 3+ < Sc3+ : stability in aqueous solution
(c) CH3COONa (d) Na 2C2O4 (d) Sc < Ti < Cr < Mn : number of oxidation states

18. Which of the following combination will produce H 2 gas? 27. The colour of light absorbed by an aqueous solution of
CuSO4 is (2012)
(2017 Adv.)
(a) orange-red (b) blue-green (c) yellow (d) violet
(a) Fe metal and conc. HNO3
(b) Cu metal and conc. HNO3 28. Which of the following will not be oxidised by O3 ? (2005)
(c) Au metal and NaCN (aq) in the presence of air (a) KI (b) FeSO4 (c) KMnO4 (d) K 2MnO4
(d) Zn metal and NaOH (aq) 29. Which of the following pair is expected to exhibit same
19. Which of the following compounds is metallic and colour in solution? (2005, 1M)
ferromagnetic? (2016 Main) (a) VOCl 2 ; FeCl 2 (b) CuCl 2 ; VOCl 2
(a) CrO2 (b) VO2 (c) MnCl 2 ; FeCl 2 (d) FeCl 2 ; CuCl 2
(c) MnO2 (d) TiO2 30. When I − is oxidised by MnO−4 in alkaline medium,
20. The reaction of zinc with dilute and concentrated nitric acid, I − converts into (2004)
respectively, produce (2016 Main) (a) IO−3 (b) I2 (c) IO−4 (d) IO−
(a) NO2 and NO (b) NO and N2O
(c) NO2 and N2O (d) N2O and NO2
31. The pair of compounds having metals in their highest
oxidation state is (2004, 1M)
21. The geometries of the ammonia complexes of Ni 2 + , Pt 2 + and (a) MnO2 , FeCl 3 (b) [MnO4 ]− , CrO2 Cl 2
Zn 2+ , respectively, are (2016 Main) (c) [Fe(CN)6]3− , [Co(CN)3] (d) [NiCl 4 ]2 − , [CoCl 4 ]−
(a) octahedral, square planar and tetrahedral
(b) square planar, octahedral and tetrahedral 32. (NH4 ) 2 Cr2 O7 on heating gives a gas which is also given by
(c) tetrahedral, square planar and octahedral (2004, 1M)
(d) octahedral, tetrahedral and square planar (a) heating NH4NO2 (b) heating NH4NO3
(c) Mg3N2 + H2O (d) Na(comp.) + H2O2
22. Which of the following compounds is not yellow coloured?
(2015 Main) 33. When MnO2 is fused with KOH, a coloured compound is
(a) Zn 2 [ Fe (CN)6 ] (b) K 3 [ Co (NO2 )6 ] formed, the product and its colour is (2003, 1M)
(c) (NH4 )3 [ As (Mo3O10 )4 ] (d) BaCrO 4 (a) K 2MnO4, purple green (b) KMnO4, purple
(c) Mn 2O3, brown (d) Mn 3O4, black
23. Which series of reactions correctly represents chemical
relations related to iron and its compound? (2014 Main) 34. Amongst the following, identify the species with an atom in +
H SO ,O
6 oxidation state (2000, 1M)
Dil. H SO 2 4 2 Heat
(a) Fe 2 4
→ FeSO4 → Fe2 (SO4 ) 3 → Fe (a) MnO−4 (b) Cr(CN)63− (c) NiF62 − (d) CrO2Cl 2
O2 , Heat Dil. H 2 SO4 Heat
(b) Fe → FeO → FeSO4 → Fe 35. On heating ammonium dichromate, the gas evolved is
Cl , Heat Heat, air (a) oxygen (b) ammonia (1999, 2M)
(c) Fe →
2
FeCl 3 → FeCl 2 Zn
→ Fe
(c) nitrous oxide (d) nitrogen
O , Heat CO, 600°C CO, 700° C 36. In the dichromate dianion
(d) Fe →
2
Fe3O4 → FeO → Fe (1999, 2M)
(a) 4 Cr—O bonds are equivalent
24. Four successive members of the first row transition elements (b) 6 Cr—O bonds are equivalent
listed below with atomic numbers. Which one of them is (c) all Cr—O bonds are equivalent
expected to have the highest E °M 3+ / M 2+ value? (2013 Main) (d) all Cr—O bonds are non-equivalent

(a) Cr (Z = 24) (b) Mn (Z = 25) 37. Which of the following compounds is expected to be
(c) Fe (Z = 26) (d) Co (Z = 27) coloured? (1997, 1M)
(a) Ag 2 SO4 (b) CuF2 (c) MgF2 (d) CuCl
Transition and Inner-Transition Elements 251

38. Ammonium dichromate is used in some fireworks. The green (a) Both Y and Z are coloured and have tetrahedral shape
coloured powder blown in the air is (1997, 1M) (b) Y is diamagnetic in nature while Z is paramagnetic
(a) CrO3 (b) Cr2 O3 (c) Cr (d) CO (c) In both Y and Z, π-bonding occurs between p-orbitals of
oxygen and d-orbitals of manganese
39. The reaction which proceed in the forward direction is
(d) In aqueous acidic solution, Y undergoes disproportionation
(a) Fe2 O3 + 6HCl → 2FeCl 3 + 3H 2 O (1991, 1M)
reaction to give Z and MnO2
(b) NH 3 + H 2 O + NaCl → NH 4 Cl + NaOH
(c) SnCl 4 + Hg 2 Cl 2 → SnCl 2 + 2HgCl 2 46. Consider the following reactions (unbalanced).
(d) 2CuI + I2 + 4H + → 2Cu 2 + + 4KI Zn + Hot conc. H 2SO 4 → G + R + X
Zn + conc. NaOH → T + Q
40. Zinc-copper couple that can be used as a reducing agent is
G + H 2S + NH 4OH → Z (a precipitate) + X + Y
obtained by (1984, 1M)
Choose the correct option(s). (2019 Adv.)
(a) mixing of zinc dust and copper gauge
(b) zinc coated with copper (a) The oxidation state of Zn in T is +1
(c) copper coated with zinc (b) R is a V-shaped molecule
(d) zinc and copper wires welded together (c) Bond order of Q is 1 in its ground state
(d) Z is dirty white in colour
41. How many unpaired electrons are present in Ni 2 + ?
(a) 0 (b) 2 (1981, 1M) 47. With reference to aqua-regia, choose the correct option(s).
(c) 4 (d) 8 (2019 Adv.)
(a) Aqua-regia is prepared by mixing conc. HCl and conc. HNO3
42. One of the constituent of German silver is (1980, 1M)
in 3 : 1 (v / v) ratio
(a) Ag (b) Cu (c) Mg (d) Al
(b) The yellow colour of aqua-regia is due to the presence of
43. Which of the following dissolves in concentrated NaOH NOCl and Cl 2
solution? (1980, 1M) (c) Reaction of gold with aqua-regia produces an anion having Au
(a) Fe (b) Zn (c) Cu (d) Ag in +3 oxidation state
(d) Reaction of gold with aqua regia produces NO2 in the absence
Objective Questions II of air
(One or more than one correct option) 48. The correct statement(s) about Cr 2+ and Mn 3+ is/are [atomic
44. In an experiment, mgrams of a compound X (gas/liquid/solid) number of Cr = 24 and Mn = 25] (2015 Adv.)
taken in a container is loaded in a balance as shown in figure I (a) Cr 2+ is a reducing agent
below. (b) Mn 3+ is an oxidising agent
(I) (II) (III) (c) Both Cr 2+ and Mn 3+ exhibit d 4 electronic configuration
Balanced; Upward deflection; Downward deflection; (d) when Cr 2+ is used as a reducing agent, the chromium ion
Magnetic field absent Magnetic field present Magnetic field present
attains d 5 electronic configuration
49. Fe3+ is reduced to Fe2+ by using (2015 Adv.)
(a) H2O2 in presence of NaOH (b) Na 2O2 in water
(c) H2O2 in presence of H2SO4 (d) Na 2O2 in presence of H2SO4

m 50. Which of the following halides react(s) with AgNO3 ( aq ) to


X
N S N S give a precipitate that dissolves in Na 2 S2 O3 ( aq ) ?
(a) HCl (b) HF (c) HBr (d) HI

Magnet 51. Reduction of the metal centre in aqueous permanganate ion


involves (2011)
In the presence of a magnetic field, the pan with X is either
(a) three electrons in neutral medium
deflected upwards (figure II), or deflected downwards (figure (b) five electrons in neutral medium
III), depending on the compound X. Identify the correct (c) three electrons in alkaline medium
statement(s). (2020 Adv.)
(d) five electrons in acidic medium
(a) If X is H2O(l ) , deflection of the pan is upwards.
(b) If X is K 4 [Fe(CN)6 ](s), deflection of the pan is upwards. 52. Which of the following statement (s) is/are correct? (1998)
(c) If X is O 2 (g ), deflection of the pan is downwards. (a) The electronic configuration of Cr is [Ar] 3d 5 4s1 (Atomic
(d) If X is C 6H6 (l ), deflection of the pan is downwards. number of Cr = 24)
45. Fusion of MnO 2 with KOH in presence of O 2 produces a salt (b) The magnetic quantum number may have a negative value
W . Alkaline solution of W upon electrolytic oxidation yields (c) In silver atom, 23 electrons have a spin of one type and 24 of
another salt X . The manganese containing ions present in W the opposite type (Atomic number of Ag = 47)
and X , respectively, areY and Z. Correct statement(s) is (are) (d) The oxidation state of nitrogen in HN 3 is – 3
(2019 Adv.)
252 Transition and Inner-Transition Elements

53. Which of the following statement(s) is/are correct when a 63. Mn 2 + can be oxidised to MnO−4 by ………
mixture of NaCl and K 2 Cr2 O7 is gently warmed with (SnO2 , PbO2 , BaO2 ) (1981, 1M)
conc. H2 SO4 ? (1998, 2M)
(a) A deep red vapours is formed True/False
(b) Vapours when passed into NaOH solution gives a yellow
solution of Na 2 CrO4
64. Dipositive zinc exhibit paramagnetism due to loss of two
electrons from 3d-orbitals of neutral atom. (1987, 1M)
(c) Chlorine gas is evolved
2+
(d) Chromyl chloride is formed 65. Copper metal reduces Fe in an acidic medium. (1982, 1M)
54. Which of the following alloys contains Cu and Zn?
(a) Bronze (b) Brass (1993, 1M)
Numerical Answer Type Questions
(c) Gun metal (d) Type metal 66. In the chemical reaction between stoichiometric quantities
55. The aqueous solution of the following salts will be coloured of KMnO4 and KI in weakly basic solution, what is the
in case of (1990, 1M) number of moles of I2 released for 4 moles of KMnO4
(a) Zn(NO3 )2 (b) LiNO3 (c) Co(NO3 )2 (d) CrCl 3 consumed? (2020 Adv.)
(e) potash alum 67. An acidified solution of potassium chromate was layered
56. Potassium manganate (K 2 MnO4 ) is formed when (1988, 2M) with an equal volume of amyl alcohol. When it was shaken
(a) chlorine is passed into aqueous KMnO4 solution after the addition of 1 mL of 3% H2 O2 , a blue alcohol layer
(b) manganese dioxide is fused with KOH in air was obtained. The blue colour is due to the formation of a
(c) formaldehyde reacts with potassium permanganate in the chromium (VI) compound ‘X’. What is the number of
presence of strong alkali oxygen atoms bonded to chromium through only single
(d) potassium permanganate reacts with conc. H 2 SO4 bond in a molecule of X ? (2020 Adv.)

68. In neutral or faintly alkaline solution, 8 moles of


Assertion and Reason permanganate anion quantitative oxidise thiosulphate
Read the following questions and answer as per the direction anions to produce X moles of a sulphur containing product.
given below : The magnitude of X is ... . (2016 Adv.)
(a) Statement I is true; Statement II is true; Statement II is the
correct explanation of Statement I. 69. In dilute aqueous H2 SO4 the complex
(b) Statement I is true; Statement II is true; Statement II is not diaquadioxalatoferrate (II) is oxidised by MnO–4 . For this
the correct explanation of Statement I. +
reaction, the ratio of the rate of change of [H ] to the rate of
(c) Statement I is true; Statement II is false.
change of [MnO−4 ] is (2015 Adv.)
(d) Statement I is false; Statement II is true.
2+
57. Statement I Zn is diamagnetic. 70. Consider the following list of reagents, acidified K 2 Cr2 O7 ,
Statement II The electrons are lost from 4s orbital to alkaline KMnO4 , CuSO4 , H2 O2 , Cl 2 , O3 , FeCl 3 , HNO3 and
form Zn 2 + . (1998, 2M) Na 2 S2 O3 . The total number of reagents that can oxidise
58. Statement I To a solution of potassium chromate if a aqueous iodide to iodine is (2014 Adv.)
strong acid is added, it changes its colour from yellow to
orange. Subjective Questions
Statement II The colour change is due to the change in Moist air Zn
oxidation state of potassium chromate. (1988, 2M)
71. (B ) ← MCl 4 → (A)
W hite fumes ( M = Transition (purple colour)
with pungent smell element colourless)

Fill in the Blanks Identify the metal M and hence MCl 4 . Explain the difference
59. The compound Y Ba 2 Cu 3 O7 which show super conductivity in colours of MCl 4 and A. (2005)
has copper in oxidation state …… assuming that the rare earth
72. Give reasons : CrO3 is an acid anhydride. (1999, 2M )
element Yttrium in its usual +3 oxidation state. (1994, 1M)
60. The outermost electronic configuration of Cr is …… 73. A compound of vanadium has a magnetic moment of
(1994, 1M) 1.73 BM. Work out the electronic configuration of the
61. Fehling’s solution A consists of an aqueous solution of vanadium ion of the compound. (1997)
copper sulphate while Fehling’s solution B consists of an 74. Write balanced equations for the following
alkaline solution of …… (1990, 1M)
(i) Oxidation of hydrogen peroxide with potassium
62. The salts ……… and …… are isostructural. (FeSO4 ⋅7H2 O, permanganate in acidic medium.
CuSO4 ⋅ 5H2 O,MnSO4 ⋅ 4H2 O,ZnSO4 ⋅ 7H2 O (1990, 1M) (ii) Reaction of zinc with dilute nitric acid. (1997, 2M)
Transition and Inner-Transition Elements 253

75. Complete and balance the following reactions 79. Give reason in one or two sentences
2− − “Most transition metal compounds are coloured.”(1986, 1M)
(i) [MnO4 ] +H +
→ ..... + [MnO4 ] + H2 O
(ii) SO2 ( aq) + Cr2 O72 − + 2H+ → … + … + (1994, 2M) 80. Show with balanced equations for the reactions when
(i) potassium permanganate interacts with manganese
76. Complete and balance the following reaction. dioxide in the presence of potassium hydroxide.
(NH4 ) 2 S2 O8 + H2 O + MnSO4 → …… + …… + …… (ii) potassium ferricyanide is heated with concentrated
(1993, 1M) sulphuric acid. (1985, 2M)

77. Write the balanced chemical equations for the following 81. State the conditions under which the following
reactions. preparations are carried out. Give necessary equations
(i) A mixture of potassium dichromate and sodium chloride is which need not be balanced.
heated with concentrated H2 SO4 . “Potassium permanganate from manganese dioxide”
(ii) Potassium permanganate is added to a hot solution of (1983, 1M)
manganous sulphate. (1990, 2M)
82. Complete and balance the following reactions (1983, 2M)
78. Complete and balance the following reactions. (i) Zn + NO−3 → Zn 2+ + NH+4
(i) Mn 2 + + PbO2 → MnO4− + H2 O
(ii) Cr2 O27 − + C2 H4 O → C2 H4 O2 + Cr 3+
(ii) Ag + + AsH3 → H3AsO3 + H+ (1987, 2M

Answers
1. (a) 2. (b) 3. (c) 4. (d) 37. (b) 38. (b) 39. (a) 40. (b)
5. (d) 6. (b) 7. (a) 8. (d) 41. (b) 42. (b) 43. (b) 44. (a,b,c)
9. (b) 10. (c) 11. (b) 12. (a) 45. (a,c,d) 46. (b,c,d) 47. (a,b,c) 48. (a, b, c)
13. (c) 14. (b) 15. (a) 16. (d) 49. (a, b) 50. (a, c, d) 51. (a, c, d) 52. (a, b, c, d)
17. (d) 18. (d) 19. (a) 20. (d) 53. (a, b, d) 54. (b, c) 55. (c, d) 56. (b, c)
21. (a) 22. (a) 23. (d) 24. (d) 57. (b) 58. (c) 59. x = +7 / 3 60. 3d 5 4s 1
25. (c) 26. (a) 27. (a) 28. (c) 61. Rochelle salt 62. FeSO 4 ⋅ 7 H 2O and ZnSO 4 ⋅ 7H 2O
29. (b) 30. (a) 31. (b) 32. (a) 63. PbO 2 64. F 65. F 66. (6)
33. (a) 34. (d) 35. (d) 36. (b) 67. (4) 68. (6) 69. (8) 70. (7)

Hints & Solutions


1. [XeF5 ] 3. Thermal decomposition of Mn compound (X), i.e. KMnO4 at
513 K results in compound Y(i.e. K 2MnO4), MnO2 and a
F
Xe : sp3d3-hybridised
gaseous product. MnO2 reacts with NaCl and concentrated
F H2SO4 to give a pungent gas Z(i.e. Cl2). The reactions
Geometry = Pentagonal
Xe
s
F bipyramidal involved are as follows :
F Shape/Structure = Pentagonal 2KMnO4 513
 K
→ K2MnO4 + MnO2 + O2 (g )
planar ( X) ∆ (Y )
F
MnO2 + 4NaCl + 4 H2SO4 →
XeO3F2 MnCl 2 + 4NaHSO4 + 2H2O + Cl 2 (g )
O F (Z )
Xe: sp3d-hybridised Pungent gas.
Geometry = Shape/ structure
Xe O = Trigonal bipyramidal
4. The pair that has similar atomic radii is Mo and W. It is due to
lanthanoid contraction. The factor responsible for lanthanoid
[because, Xe in XeO3F2 does not
O contraction is the imperfect shielding of one electron by
F have pair of electrons] another in the same set of orbitals. Shielding of 4 f is very less
2. Only statement 2 is incorrect, whereas 1 and 3 are correct. due to its diffused shape. As a result, nuclear charge increases.
Hence, Mo and W have similar atomic radii.
KMnO4 will not give satisfactory result when it is titrated by HCl,
because same amount of KMnO4 is consumed to oxidise HCl into Cl 2.
254 Transition and Inner-Transition Elements

5. The 3d-transition series is 10. The correct order of atomic radii is


Sc Ti V Cr Mn Fe Co Ni Cu Zn Europium (Eu) > Cerium (Ce) > Holmium (Ho) > Nitrogen (N)
Atomic number 21 22 23 24 25 26 27 28 29 30
199 pm 183 pm 176 pm 65 pm


Outermost 3d24s2 3d54s2 3d84s2 3d104s2 Note
Electronic (i) N being the member of p-block and second period, have the
Configuration
smallest radii.
In 1st ionisation, one electron will be removed from 4 s2 (ii) Rest of all the 3 members are lanthanides with Eu having
subshell/orbital. stable half-filled configuration thus with bigger size than rest
With increase in atomic number (Z ), i.e. with increase in number two.
of protons in the nucleus, effective nuclear charge (Z* ) also
(iii) Among Ce and Ho, Ce has larger size which can be explained
increases from Sc to Zn.
on the basis of “Lanthanoid contraction”.
IE ∝ Z*
11. When MnO2(A) is fused with alkali in presence of air then
So, IE order of the given elements will be, potassium manganate (B) is formed. Potassium manganate (B) is
Ti < Mn < Ni < Zn of green colour which disproportionate in a neutral or acidic
solution to produce potassium permanganate (C). Potassium
6. Actinoids show a variety of oxidation states due to comparable
permanganate (C) in presence of acidic medium oxidises iodide
energies of 5 f , 6d and 7s energy levels.
to iodate.
In the actinoids family (5 f -block), uranium (U) neptunium
The reaction can be shown as:
(Np), plutonium (Pu) and americium (Am) have highest possible 0
oxidation states of +6, + 7, + 7 and +6 respectively. + 4 4 KOH, O2 +6 −2 −2
(i) 2 Mn O2 → 2K 2 Mn O4 + 2H2O
7. The spin only magnetic moment (µ ) of each ion can be calculated (A ) ∆ (B )
as : Potassium manganate
(Green)
µ = n (n + 2) BM +6 + 7 +7
4 HCl
[Q n = No. of unpaired electron(s)] ⇒ µ ∝ n, i.e. higher the (ii) 3K2 Mn O4 → 2K Mn O4 + Mn O2 + 2H2O + 4KCl
(B ) (C )
number of unpaired electron, higher will be the value of µ. Potassium
permanganate
Metal ion Z n (for metal ion) M (BM) Nature (purple)
−1
2+
22 2
Paramagnetic + 7 K I , H2O + 4 +5
Ti 2 (3d ) 8
(iii) 2K MnO4  → 2 MnO2 + 2KOH + K IO3
(C ) (A ) (D)
V2+ 23 3 (3d 3 ) 15 Paramagnetic
Thus, A and D are MnO2 and KIO3 respectively.
Ti 3+ 22 1 (3d 1 ) 3 Paramagnetic 12. The most stable oxidation states in the compounds of the given
3+ 0
transition metals of 3d-series are,
Sc 21 0 (3d ) 0 Diamagnetic
Sc : + 3; Ti : + 3, + 4; V : +2, + 3, + 4 , + 5; Cu : + 1, + 2
Thus, the correct order of spin only magnetic moments of given The electronic configuration of Sc (Z = 21) is [Ar] 3d 1 , 4 s2.
hydrated ions will be Due to the presence of only one 3d-electron
Sc3+ < Ti 3+ < Ti 2+ < V2+ (no pairing energy) and two 4s-electrons, they easily ionise to
achieve most stable +3 oxidation state.
8. The maximum number of possible oxidation states of actinoids
are shown by neptunium (Np) and plutonium (Pu). These 13. In the lanthanoid series, atomic number of fourteen 4 f -block
actinoids exhibit oxidation states of +3, +4, +5 and +6. elements ranges from 58 (Ce) to 71 (Lu).
9. The lanthanide ion that would show colour is Sm . Colour of a 3+ Ytterbium, Yb(Z = 70) has electronic configuration :
compound depends on the number of electrons in 4 f -orbitals. [ Xe ] 4 f 14 6s2. So, the 71nth electron of lutetium, Lu (Z = 71)
Electronic configuration of given lanthanides are as follows: should enter into 5d orbital and its (here, Lu is ‘X ’) electronic
Gd 3+ = 4 f 7 configuration will be : [ Xe ]4 f 14 5d 1 6s2. It happens so, because
f -block elements have general electronic configuration,
Sm 3+ = 4 f 5
(n − 2) f 1 − 14 (n − 1)d 1− 10ns2. Therefore, option (c) is correct.
La 3+ = 4 f 0
14. Lanthanoid contraction in the lanthanoid series takes place due
Lu 3+ = 4 f 14 to the presence of electron(s) in the 4 f -orbitals. f -orbitals have
Gd 3+ have half-filled 4 f -orbitals. poor shielding effect. As a result, the effective nucleur charge
La 3+ have no electron in 4 f -orbitals. will be more experienced by the 5d and 6s- electrons and it will
cause contraction or decrease in both atomic and ionic radii.
Lu 3+ have fully-filled 4 f -orbitals.
15. For transition metals,
Only Sm 3+ contain 4 f 5. The electrons can easily undergoes °
∆H Atomisation ∝ Strength of metallic bonding
excitation. That result in a formation of colour.
∝ Number of unpaired electrons in the metal atom
Transition and Inner-Transition Elements 255

For the given 3d-transition metals, The correct reactions are as follows:
V Fe Cu Zn (a) Fe + dil. H2SO4 → FeSO4 + H2
3d 34 s2 3d64 s2 3d 104 s1 3d 104 s0 1
H2SO4 + 2FeSO4 + O2 → Fe2 (SO4 )3 + H2O
n=3 n=4 n=0 n=0 2

[Q n = no. of unpaired electrons] Fe2 (SO4 )3 → Fe2O3 (s) + 3SO3 ↑
∆° H Atomisation (kJ mol −1) = 515 418 339 130 The given reaction is incorrect in question
So, absence of unpaired d-electrons and larger size of Zn atoms, 24. SRP value normally increases from left to right in the period of
make the crystal lattice of Zn less closely packed. d-block elements. Some SRP value are exceptionally higher due
to stability of product ion. e.g.
16. Zinc oxide (ZnO) when react with Na 2O it act as acid while with
E° = + 1.57 V; E°
Mn 3+ / Mn 2+
= + 1.97 V
Co3+ / Co2 +
CO2 it act as base. Therefore, it is an amphoteric oxide.
ZnO + Na 2 O → Na 2 ZnO2 Thus, EM° 3+ / M2+ is highest for Co.
Acid Base Salt
25. The half equations of the reaction are
ZnO + CO2 → ZnCO3
Base Acid Salt
MnO −4 → Mn 2+
C2O42− → CO2
17. The reaction takes place as follows
Na 2C2O4 + H2SO4 → Na2 SO4 + H2O + CO ↑ + CO2 ↑ The balanced half equations are
(X) (Conc.) Effervescence MnO−4 + 8H+ + 5e− → Mn 2+ + 4H2O

Na 2C2O4 + CaCl 2 → CaC2O4 + 2NaCl C2O24− → 2CO2 + 2e−


(X) White ppt. On equating number of electrons, we get
5CaC2O4 + 2KMnO4 + 8H2SO4 → K2SO4 + 5CaSO4 2MnO−4 + 16H+ + 10e− → 2Mn 2+ + 8H2O
Purple
5C2O42− → 10CO2 + 10e−
+ 2MnSO4 + 10CO2 + 8H2O
Colourless On adding both the equations, we get
Hence, X is Na 2C2O4. 16
2MnO−4 + 5C2O−4 + 16H+ → 2Mn 2+ + 2 × 5CO2 + H2O
2
18. Zn + 2NaOH → Na 2ZnO2 + H2
Amphoteric Thus x , y and z are 2, 5 and 16 respectively.
19. Only three elements iron (Fe), cobalt (Co) and nickel (Ni) show 26. (a) V2+ = 3 unpaired electrons
ferromagnetism at room temperature. CrO2 is also a metallic and Cr 2+ = 4 unpaired electrons
ferromagnetic compound which is used to make magnetic tapes Mn 2+ = 5 unpaired electrons
for cassette recorders. Fe2+ = 4 unpaired electrons
20. Zn + 4HNO3 → Zn (NO3 )2 + 2H2O + 2NO2 Hence, the order of paramagnetic behaviour should be
(Conc.)
V2+ < Cr 2 + < Fe2 + < Mn 2 +
4Zn +10HNO3 → 4Zn (NO3 )2 + N2O + 5H2O
(Dil.) (b) Ionic size decreases from left to right in the same period.
(c) (As per data from NCERT)
21. [Ni(NH3 )6 ] 2+ sp3d 2 octahedral
Co3+ / Co2+ = 1.97;
[Pt(NH3 )4 ] 2+ dsp2 square planar
Fe3+ /Fe2+ = 0.77;
[Zn(NH3 )4 ] 2+ sp3 tetrahedral
Cr 3+ / Cr 2+ = − 0.41
22. Zn 2[ Fe(CN)6 ], K3[ Co(NO2 )6 ] and (NH4 )3As [ Mo3O10 ] 4 show 3+
colour due to d-d transition while BaCrO4 is coloured due to Sc is highly stable (It does not show + 2).
charge transfer phenomenon. (d) The oxidation states increases as we go from group 3 to
Further according to spectrochemical series the strong ligand group 7 in the same period.
possessing complex has higher energy and hence lower
27. The aqueous solution of CuSO4 consist of the complex
wavelength. Therefore, complexes containing NO2 , NH+4 , O2−
[Cu(H2O)4 ]2+ ion which absorbed in orange-red region and
etc., ligands show yellow colour while CN− forces the complex
to impart white colour. impart deep blue colouration to solution.
Spectrochemical series 28. KMnO4 is itself a very strong oxidising agent, O3 cannot oxidise it.
I− < Br − < S2− < SCN− < Cl − < NO3− < N3− < F− < OH− 29. In CuCl 2, Cu 2 + has d 9 configuration, exhibit d-d transition and
< C2O42− ≈ H2O < NCS− < CH3CN < py < NH3 < en
show colour. Similarly in VOCl 2, V 4 + has d 1 configuration, can
< bipy < Phen < NO−2 < PPh 3 < CN− ≈ CO
exhibit d-d transition and show colour.
23. PLAN Analyse each reaction given in the question and choose the
30. MnO−4 + I− + OH− → MnO42− + IO−3
correct answer on the basis of oxidation state and stability of
iron compounds. Use the concept of Ellingham diagram to
solve this problem.
256 Transition and Inner-Transition Elements

31. In MnO−4 , Mn 7+ is in highest oxidation state possible for Mn. In (c) X = O2 (g)
CrO2Cl 2, Cr 6+
is in highest oxidation state possible for Cr. [Here, O2 (g ) is paramagnetic due to two-unpaired electrons
present in π * (antibonding orbitals). Hence, statement (c) is
32. Ammonium dichromate on heating produces N2 (g). NH4NO2 correct.
also gives N2 on heating :

(d) X = C6H6(l)
(NH4 )2 Cr2O7 → N2 + Cr2O3 + 4H2O (Here, C6H6 is diamagnetic due to presence of 0 unpaired
∆ electrons). Hence, statement (d) is incorrect.
NH4NO2 → N2 + 2H2O
∆ 1
33. K2MnO4 (purple green) is formed which is the first step of 45. MnO 2 + 2KOH+ O 2 → K 2MnO 4 + H2O
2 (W ) potassium
preparation of KMnO4. manganate

2MnO2 + 4KOH + O2 → K2MnO4 + 2H2O K2MnO4 (aq) +


92K (aq) + MnO 4
2–
(aq)
Purple green (W) (Y)
O
34. In CrO2Cl 2 , Cr is in + 6 oxidation state because Cl is in (–1) and, 3

oxygen is in (–2) oxidation states. é Mn


sp- hybridisation,
tetrahedral (manganate ion)
é
é

Heat O O Green coloured
35. (NH4 )2 Cr2O7 → N2 + Cr2O3 + 4H2O O complex

36. The structure of dichromate ion is : MnO 2−4 ion has one unpaired electrons, therefore it gives
O O
d-d transition to form green colour. Y complex has paramagnetic
Cr Cr nature due to presence of one unpaired electron.
O O O In aqueous solution,
O
– O
– Electrolytic oxidation
K 2MnO 4 + H2O → H2 + KOH+ KMnO 4
Cr2O72– (W) (X)
Exhibit resonance phenomena. Except the bridged Cr—O—Cr,
O
all Cr—O bonds are equivalent. sp3, tetrahedral
2+ 9
37. Cu (3d ) undergo d-d transition, exhibit colour.
KMnO4(aq)
D +
K + MnO4 -
é Mn (purple coloured
é
é
(Z) complex ion)
O O
38. Ammonium dichromate [(NH4 )2 Cr2O7 ] on heating decomposes O
producing green powder of Cr2O3 and N2 (g ) is evolved.
MnO −4 ions gives charge transfer spectrum in which a fraction of
39. Fe2O3 is a basic oxide, neutralised by HCl spontaneously
electronic charge is transferred between the molecular entities.
forming FeCl 3 and water. Electrolytic
40. Zinc coated with copper is used as a reducing agent. Q MnO 24− → MnO 4− + e−
(Y) oxidation ( Z)
41. The valence shell electronic configuration of Ni 2+ is :
In acidic medium, Y undergoes disproportionation reaction.
[Ar] ; two unpaired 3MnO 24− (aq) + 4H+ → 2MnO 4− + MnO 2 + 2H2O
electrons (Y ) (Z )
3d 8 4s0 −
4 and MnO 4 both ions form π-bonding between p-orbitals
MnO 2−
42. German silver is an alloy of copper (56%), Zn (24%) and (Y) (Z)
Ni(20%). of oxygen and d-orbitals of manganese.
43. Zn being amphoteric, dissolves in both acid and base : Thus, options (a, c, d) are correct.
Zn + 2NaOH → Na 2ZnO2 + H2 46. When Zn react with hot conc. H2SO 4 then SO 2 is released and
44. Paramagnetism is a form of magnetism whereby some materials ZnSO 4 is obtained.
are attracted by an externally applied magnetic field, and form Zn + 2H2SO 4 → ZnSO 4 + SO 2 ↑ + 2H2O
internal, induced magnetic fields in the direction of the applied (Hot + Conc.) (G) (R) (X)
magnetic field. So, magnetic balance shows downward
R(SO 2 ) molecule is V-Shaped
deflection. While diamagnetic substance shows repulsion in
magnetic field and magnetic balance shows upward deflection.
(a) X = H2O (l) S
(Water has no unpaired electrons and is thus diamagnetic). O O
Hence, statement (a) is correct.
Thus, option (b) is correct.
(b) X = K4[Fe(CN)6 ](s)
When Zn is react with conc. NaOH then H2 gas is evolved and
(CN is a strong field ligand which forces the d-orbital Na 2ZnO 2 is obtained.
electrons to pair up (t2g6eg0 ) and making it diamagnetic).
Zn + 2 NaOH (conc.) → Na 2ZnO 2 + H2 ↑
Hence, statement (b) is correct (T ) (Q)
Transition and Inner-Transition Elements 257

In ground state, H—H (Q) (bond order = 1) Na 2S2O3 solution dissolve all three, AgCl, AgBr, AgI by forming
Thus, option (c) is correct. complex [Ag(S2O3 )2 ]3− as S2O2−3 is a stronger complexing agent
The oxidation state of Zn in T (Na2ZnO 2 ) is +2 than ammonia.
Thus, option (a) is incorrect. 51. In neutral medium
ZnSO 4 + H2S + NH4OH → ZnS↓ + 2H2O + (NH4 )2SO 4 MnO–4 → MnO2 (Mn 7 + + 3e– → Mn 4+ )
(G) (Z) (X) (Y)
In alkaline medium
ZnS (Z) compound is dirty white coloured. MnO–4 → MnO2 (Mn 7 + + 3e– → Mn 4+ )
Thus, option (d) is correct.
In acidic medium
47. The explanation of given statements are as follows: MnO–4 → Mn2+ (Mn7 + + 5e– → Mn2+ )
(a) Aqua-regia is prepared by mixing conc. HCl and conc. HNO 3
52. Cr : [Ar]3d 5 4 s1
in 3:1 (v/v) ratio and is used in oxidation of gold and platinum.
Hence, option (a) is correct. Magnetic quantum number : – l……0……+ l.
(b) Yellow colour of aqua-regia is due to its decomposition into Ag(4 d 10 5s1 ) All paired electrons have opposite spin. The last
NOCl (orange yellow) and Cl 2 (greenish yellow). one has unpaired spin.
Hence, option (b) is correct. 53. 4NaCl + K2Cr2O7 + 6H2SO4 → 2CrO2Cl 2
Chromyl chloride
(c) When gold reacts with aqua-regia then it produces AuCl −4 (red vapour)
anion complex in which Au has +3 oxidation state. + 4NaHSO4 + 2KHSO4 + 3H2O
0 +3
Au + HNO3 +4HCl → Au Cl –4 + H3O+ + NO + H2O CrO2 Cl 2 + 4NaOH → Na 2 CrO4 + 2NaCl + 2H2 O
yellow solution
Oxidation

Hence, option (c) is correct. 54. Brass = Cu and Zn Gun metal = Cu, Sn, Zn
Bronze = Cu and Sn Type metal = Pb, Sn, Sb
(d) Reaction of gold with aqua-regia produces NO gas in
absence of air. 55. Co2+ (3d 7 ) and Cr 3+ (3d 3 ) have allowed d-d transition, therefore
Hence, option (d) is incorrect. produces coloured aqueous solution.
2+ 4 ∆
48. In aqueous solution Cr (3d )acts as a reducing agent, oxidising 56. 2KOH + MnO2 + O2 → K2MnO4 + H2O
3+ 3
itself to Cr (3d ) that gives a completely half-field t2g level in
HCHO + KMnO4 + 2KOH → K2MnO4 + H2O + HCOOH
octahedral ligand field of H2O.
57. Both Statement I and Statement II are independently true but
(b) Mn 3+ (3d 4 ) is an oxidising agent as it is reduced to Statement II is not the correct explanation of Statement I.
Mn 2+ (3d 5 ) , a completely half-filled stable configuration. Diamagnetism is due to lack of unpaired electron in Zn 2+ (3d 10 ).
(c) Both Cr 2+ and Mn 3+ have d 4 configuration. 58. Statement I is true but Statement II is false :
(d) 3d 4 Cr 2+ (aq) R.
A
→ Cr 3+ (aq)+ e– K2CrO4 + H2SO4 → K2Cr2 O7 + K2SO4 + H2O
Hence (d) is wrong statement. Yellow Orange

49. H2O2 is alkaline medium acts as reducing agent, reduces Fe3+ to In both K2CrO4 and K2Cr2O7 , chromium is in +6 oxidation state.
2+
Fe . In acidic medium the same H2O2 oxidises Fe 2+
to Fe . 3+ 59. Y = +3, 2Ba = 2 × 2 = 4
7 ‘O’ = 7 × (−2) = − 14
50. Solubilities of silver halides in water decreases from fluoride 7
3 + 4 + (−14 ) + 3x = 0 ⇒ x = +
(AgF) to iodide (AgI). Silver fluoride is readialy soluble in 3
water, hence when AgNO3 solution is added to HF solution (HF
60. 3d 5 4 s1
being weak acid, its solution maintain very low concentration of
F − ) no precipitate of AgF is formed. 61. Rochelle salt.
HCl, HBr and HI being all strong acid, forms precipitates of 62. FeSO4 ⋅ 7H2O and ZnSO4 ⋅ 7H2O
AgCl, AgBr and AgI when AgNO3 solution is added to their 63. PbO2, a strong oxidising agent, oxidises Mn 2+ to MnO−4 .
aqueous solution.
64. Zn 2+ (3d 10 ) has no unpaired electron–diamagnetic.
HCl(aq) + AgNO3(aq) → AgCl(s)+ HNO3(aq)
Curdy white 65. Cu cannot reduce Fe2+
HBr (aq) + AgNO3(aq) → AgBr (s) + HNO3(aq) 66. In alkaline medium : Iodide is oxidised to iodate
Pale yellow 2MnO −4 + H2O + I− → 2MnO 2 + 2OH− + IO 3−
Hl (aq) + AgNO 3(aq) → AgI (s) + HNO 3(aq) But, in weakly basic solution :
Yellow +7 −1 +4 0
The solubilities decreases from AgCl to AgI, AgCl dissolves in K Mn O4 + KI → Mn O2 + I2
aqueous ammonia, AgBr dissolves only slightly in concentrated Eq. of KMnO4 = Eq. of I2
ammonia while AgI does not dissolve in ammonia solution.
4 ×3=n×2 ⇒ n=6
258 Transition and Inner-Transition Elements

67. When a solution of K2CrO4 is treated with amyl alcohol and 71. A = [Ti(H2O)6 ]3+ and M = Ti, B = TiO2, Ti(IV) has no electron
acidified H2O2, the layer of amyl alcohol turns blue because in 3d-orbital, no d-d transition is possible, therefore MCl 4 is
acidified H2O2 converts K2CrO4 to CrO5 to given the blue colourless. In A, there is one electron in 3d-orbital and its d-d
colouration, transition is responsible for colour.
CrO24− + 2H + 2H2O2 → CrO5 + 3H2O 72. CrO3 is anhydride of chromic acid :
(Blue coloured
compound) CrO3 + H2O → H2CrO4
Chromic acid
O
O O 73. µ = n (n + 2) BM where ‘n’ is number of unpaired electrons.
CrO5 ⇒ Cr
1.73 = n (n + 2) ⇒ n = 1; V 4+ = 3d 1
O O
Number of oxygen atom bonded with chromium with single 74. (i) 2KMnO4 + 5H2O2 + 3H2SO4 → K2SO4
bond is (4). + 2MnSO4 + 5O2 + 8H2O
68. In neutral or faintly alkaline solution, MnO−4 is reduced to MnO2 (ii) 4Zn + 10HNO3 → 4Zn(NO3 )2 + N2 O + 5H2 O
and S2O32− is oxidised to SO2−
4 . 75. (i) 3MnO24− + 4H+ → MnO2 + 2MnO4− + 2H2O
Change in ON = 4 units
2–
(ii) 3SO2 (aq) + Cr2O72− + 2H+ → 3SO24− + 2Cr 3+ + H2O
– 2– + MnO2
MnO4 + 1/2 S2 O3 SO 4
+7 +6 +4 76. (NH4 )2 S2O8 + 2H2O + MnSO4 → MnO2
+2
Change in ON = 3 units
+ 2H2SO4 + (NH4 )2 SO4

Thus, 4MnO4− +
3
S2O23− → 3SO24− + 4 MnO2
77. (i) K2Cr2O7 + 4NaCl + 6H2SO4 → 2CrO2Cl 2
2 + 4NaHSO4 + 3H2 O + 2KHSO4
or 8MnO−4 + 3S2O32− → 6SO24− + 8MnO2 (ii) 2KMnO4 + 3MnSO4 + 2H2 O → 5MnO2
Thus, moles of SO2−
4 formed by 8 moles of MnO−4 =6 + K 2 SO4 + 2H2 SO4
78. (i) 2Mn 2+ + 5PbO2 + 4H+ → 2MnO−4 + 2H2O + 5Pb2+
69. The balanced redox reaction is
MnO−4 + [Fe(H2O)2 (C2O4 )2 ]2 − + 8H+ → Mn 2+ + Fe3+ (ii) 6Ag + + AsH3 + 3H2 O → 6Ag + H3AsO3 + 6H+

+ 4CO2 + 6H2O 79. Most transition metals have partially filled d-orbitals which
r [ H+ ] 8 absorb in visible region and undergo d-d transition, which is
⇒ = =8 responsible for colour.
r [ MnO 4– ] 1
80. (i) 2KMnO4 + 4KOH + MnO2 → 3K2MnO4 + 2H2O
70. Acidified K2Cr2O7 , CuSO4 , H2O2, Cl 2, O3, FeCl 3 and HNO3
(ii) K 4 Fe(CN)6 + 6H2 SO4 + 6H2 O →
oxidise aq. iodide to iodine. Alkaline KMnO4 oxidise aq. iodide
2K 2 SO4 + FeSO4 + 3(NH4 )2 SO4 + 6CO
to IO−3 .
Na 2S2O3 is a strong reducing agent which on reaction with I 2 81. Potassium permanganate can be prepared from MnO2 under the
produces I − . following conditions :
Na 2S2O3 + I2 → 2I− + Na 2S4O6 Heat
MnO2 + KOH + O2 → K 2 MnO4 + H2 O
Therefore, no reaction takes place between Na 2S2O3 and iodide
K 2 MnO4 + Cl 2 → KMnO4 + KCl
ion.
Hence, correct integer is (7). 82. (i) 4Zn + NO3− + 10H+ → 4Zn 2+ + NH+4 + 3H2O
(ii) Cr2O27 − + 3C2H4O + 8H+ → 3C2H4O2 + 2Cr 3+ + 4H2O
18
Coordination Compounds
Topic 1 Nomenclature and Isomerism of Coordination Compounds
Objective Questions I (Only one correct option) (a) hexadentate (b) tetradentate
(c) bidentate (d) tridentate
1. If AB4 molecule is a polar molecule, a possible geometry of
AB4 is (2020 Main, 2 Sep I) 7. The total number of isomers for a square planar complex
(a) square pyramidal (b) square planar [M(F)(Cl)(SCN)(NO 2 )] is (2019 Main, 10 Jan I)
(c) rectangular planar (d) tetrahedral (a) 12 (b) 16 (c) 4 (d) 8
2. The coordination numbers of Co and Al in [CoCl(en)2 ]Cl and 8. The oxidation states of Cr, in[Cr(H2 O)6 ]Cl 3 ,[Cr(C6 H6 )2 ], and
K 3 [Al(C2 O4 )3 ], respectively, are (en = ethane-1, 2-diamine) K 2 [Cr(CN)2 (O)2 (O2 )(NH3 )] respectively are (2018 Main)
(2019 Main, 12 April II)
(a) +3, +4 and +6 (b) +3, +2 and +4
(a) 5 and 3 (b) 3 and 3 (c) 6 and 6 (d) 5 and 6
(c) +3, 0 and +6 (d) +3, 0 and +4
3. The species that can have a trans-isomer is (en = ethane -1,
2-diamine, ox = oxalate) (2019 Main, 10 April I)
9. Consider the following reaction and statements :
(a) [Pt(en)Cl 2 ] (b) [Cr(en)2 (ox)] + [Co(NH3 )4 Br2 ]+ + Br − → [Co(NH3 )3 Br3 ] + NH3
(c) [Pt(en)2 Cl 2 ]2+ (d) [Zn(en)Cl 2 ] I. Two isomers are produces if the reactant complex ion is
a cis-isomer.
4. The maximum possible denticities of a ligand given below II. Two isomers are produced if the reactant complex ion is
towards a common transition and inner-transition metal ion, a trans-isomer.
respectively, are (2019 Main, 9 April II)
III. Only one isomer is produced if the reactant complex ion
sOOC COOs is a trans-isomer.
N N N IV. Only one isomer is produced if the reactant complex ion
sOOC
COOs COOs is a cis-isomer.
(a) 8 and 8 (b) 8 and 6 (c) 6 and 6 (d) 6 and 8 The correct statements are (2018 Main)
(a) (I) and (II) (b) (I) and (III)
5. The one that will show optical activity is (en = ethane-1,
(c) (III) and (IV) (d) (II) and (IV)
2-diamine) (2019 Main, 9 April I)
A A 10. Which one of the following complexes shows optical
A B B B isomerism? (2016 Main)
(a) M (b) M (a) cis [Co(en)2 Cl 2 ]Cl (b) trans [Co(en)2 Cl 2 ]Cl
B A B B (c) [Co(NH3 )4 Cl 2 ]Cl (d) [Co(NH3 )3 Cl 3 ]
B A 11. The number of geometric isomers that can exist for square planar
A A [Pt(Cl)(py)(NH3 )(NH2 OH)]+ is (py = pyridine). (2015 Main)
A (a) 2 (b) 3 (c) 4 (d) 6
(c) M en (d) en M en
12. Which of the following complex species is not expected to
B
exhibit optical isomerism? (2013 Main)
B A (a) [Co(en)3 ]3+ (b) [Co(en)2 Cl 2 ]+
6. The following ligand is (c) [Co (NH3 )3Cl 3 ] (d) [Co(en)(NH3 )Cl 2 ]+
NEt2
13. As per IUPAC nomenclature, the name of the complex
N [Co (H2 O)4 (NH3 )2 ]Cl 3 is (2012)
(a) tetraaquadiaminecobalt (III) chloride
(b) tetraaquadiamminecobalt (III) chloride
O– –
O
(c) diaminetetraaquacobalt (III) chloride
(d) diamminetetraaquacobalt (III) chloride
(2019 Main, 8 April I)
260 Coordination Compounds

14. Geometrical shapes of the complexes formed by the reaction (c) Statement I is true; Statement II is false.
of Ni 2+ with Cl − , CN− and H2 O, respectively, are (2011)
(d) Statement I is false; Statement II is true.
(a) octahedral, tetrahedral and square planar 21. Statement I The geometrical isomers of the complex
(b) tetrahedral, square planar and octahedral [ M (NH3 )4 Cl 2 ] are optically inactive.
(c) square planar, tetrahedral and octahedral Statement II Both geometrical isomers of the complex
(d) octahedral, square planar and octahedral [ M (NH3 )4 Cl 2 ] possess axis of symmetry. (2008, 3M)

15. The correct structure of ethylenediaminetetraacetic acid Passage Based Question


(EDTA) is (2010)
HOOCCH2 CH2COOH Passage
(a)
HOOCCH2
N — CH == CH — N
CH2COOH The coordination number of Ni 2 + is 4.
HOOC COOH NiCl 2 + KCN (excess) → A (cyano complex)
(b) N — CH — CH — N
HOOC COOH NiCl 2 + conc. HCl (excess) → B (chloro complex)
HOOCCH2 CH2COOH 22. The IUPAC name of A and B are (2006, 3 × 4M =12M)
(c) N — CH2 — CH2 — N (a) potassium tetracyanonickelate (II), potassium
HOOCCH2 CH2COOH
COOH tetrachloronickelate (II)
H 2C (b) tetracyanopotassiumnickelate (II),
HOOC—H2C H tetrachloropotassiumnickelate (II)
(d) N—CH—CH—N
H CH2—COOH (c) tetracyanonickel (II), tetrachloronickel (II)
CH2 (d) potassium tetracyanonickel (II), potassium
HOOC
tetrachloronickel (II)
16. The ionisation isomer of [Cr(H2 O)4 Cl(NO2 )]Cl is (2010)
(a) [Cr(H2O)4 (O2N)]Cl 2 (b) [Cr(H2O)4 Cl 2 ](NO2 ) Fill in the Blank
(c) [Cr(H2O)4 Cl(ONO)]Cl (d) [Cr(H2O)4 Cl 2 (NO2 )] ⋅ H2O
23. The type of magnetism exhibited by [Mn(H2 O)6 ]2+ ion is …
17. The IUPAC name of [Ni(NH3 )4 ][NiCl 4 ] is (2008, 3M) (1994, 1M)
(a) Tetrachloronickel (II)-tetraamminenickel (II) Integer Answer Type Questions
(b) Tetraamminenickel (II)-tetrachloronickel (II)
(c) Tetraamminenickel (II)-tetrachloronickelate (II)
24. The possible number of geometrical isomers for the complex
(d) Tetrachloronickel (II)-tetraamminenickelate (0) [CoL2 Cl 2 ]− ( L= H2 NCH2 CH2 O− ) is (are) ... (2016 Adv.)

18. Which kind of isomerism is shown by Co(NH3 )4 Br2 Cl? 25. Among the complex ions,
(2005, 1M) [Co(NH2 CH2 CH2  NH2 )2 Cl 2 ]+ , [CrCl 2 (C2 O4 )2 ]3− ,
(a) Geometrical and ionisation (b) Optical and ionisation [Fe(H2 O)4 (OH)2 ]+ , [Fe(NH3 )2 (CN)4 ]− ,
(c) Geometrical and optical (d) Geometrical only
[Co(NH2  CH2  CH2  NH2 )2 (NH3 ) Cl]2+ and
Objective Questions II [Co(NH3 )4 (H2 O)Cl]2+ the number of complex ion(s)
(One or more than one correct option) that show(s) cis-trans isomerism is (2015 Adv.)

19. The pair(s) of coordination complexes/ions exhibiting the 26. The volume (in mL) of 0.1 M AgNO3 required for complete
same kind of isomerism is/are (2013 Adv.) precipitation of chloride ions present in 30 mL of 0.01 M
(a) [Cr(NH3 )5 Cl]Cl 2 and [Cr(NH3 )4 Cl 2 ]Cl solution of [Cr(H2 O)5 Cl]Cl 2 , as silver chloride is close to
(b) [Co(NH3 )4 Cl 2 ]+ and [Pt(NH3 )2 (H2O)Cl]+ (2011)
(c) [CoBr2Cl 2 ]2− and [PtBr2Cl 2 ]2− 27. Total number of geometrical isomers for the complex
(d) [Pt(NH3 )3 (NO3 )]Cl and [Pt(NH3 )3 Cl]Br [RhCl(CO)(PPh 3 )(NH3 )] is (2010)
20. The compound(s) that exhibit(s) geometrical isomerism
is/are (2009) Subjective Questions
(a) [Pt(en)Cl2] (b) [Pt(en)2]Cl2 28. Write the formulae of the following complexes :
(c) [Pt(en)2Cl2]Cl2 (d) [Pt(NH3)2]Cl2 (i) Pentamminechlorocobalt (III) ion
(ii) Lithium tetrahydridoaluminate (III) (1997, 2M)
Assertion and Reason
29. Write the IUPAC name for [Cr(NH3 )5 CO3 ]Cl. (1996, 1M)
Read the following questions and answer as per the direction
given below : 30. Write the IUPAC name of the following compounds :
(a) Statement I is true; Statement II is true; Statement II is the (i) [Co(NH3 )5 ONO] Cl 2
correct explanation of Statement I. (ii) K 3[Cr(CN)6 ] (1995, 2M)
(b) Statement I is true; Statement II is true; Statement II is not the
correct explanation of Statement I.
Topic 2 Bonding and Important Property of
Coordination Compounds
Objective Questions I (Only one correct option) 7. The incorrect statement is (2019 Main, 10 April II)
6
1. Consider that a d metal ion ( M 2+
) forms a complex with (a) the gemstone, ruby, has Cr 3 + ions occupying the octahedral
sites of beryl
aqua ligands and the spin only magnetic moment of the
(b) the color of [CoCl(NH3 )5 ]2 + is violet as it absorbs the yellow
complex is 4.90 BM. The geometry and the crystal field
light
stabilisation energy of the complex is (2020 Main, 2 Sep I)
(c) the spin only magnetic moments of Fe(H2O)6 ]2 + and
(a) tetrahedral and −1.6∆ t + 1 P
(b) octahedral and −2.4 ∆ 0 + 2 P [Cr(H2O)6 ]2 + are nearly similar
(c) octahedral and −1.6 ∆ 0 (d) the spin only magnetic moment of [Ni(NH3 )4 (H2O)2 ]2 + is 2.83
(d) tetrahedral and −0.6 ∆ t BM
2. The d-electron configuration of [ Ru(en)3 ]Cl2 and 8. Three complexes,
[Fe(H 2O)6 ]Cl2 , respectively are (2020 Main, 3 Sep II) [CoCl(NH3 )5 ]2+ (I), [Co(NH3 )5 H2 O]3+ (II) and
(a) t62g eg0 and t62geg0 (b) t62g eg0 and t24geg2 [Co(NH3 )6 ] 3+
(III)
(c) t24g eg2 and t62geg0 (d) t24g eg2 and t24geg2 absorb light in the visible region. The correct order of the
wavelength of light absorbed by them is (2019 Main, 10 April I)
3. The compound used in the treatment of lead poisoning is (a) II > I > III (b) I > II > III
(2019 Main, 12 April II)
(a) D-penicillamine (b) desferrioxime-B (c) III > I > II (d) III > II > I
(c) cis-platin (d) EDTA 9. The degenerate orbitals of [Cr(H2 O)6 ]3+ are
(2019 Main, 9 April I)
4. Complete removal of both the axial ligands (along the z-axis)
(a) d 2 and dxz (b) dxz and d yz
from an octahedral complex leads to which of the following z
(c) d 2 2 and dxy (d) d yz and d 2
splitting patterns? (relative orbital energies not on scale). x − y z
(2019 Main, 12 April I) 10. The calculated spin only magnetic moments (BM) of the
anionic and cationic species of [Fe(H2 O)6 ]2 and [Fe(CN)6 ],
dx2 – y2 dz2
respectively, are (2019 Main, 8 April II)
dxy dx2 – y2 (a) 0 and 4.9 (b) 2.84 and 5.92
(a) E (b) E (c) 0 and 5.92 (d) 4.9 and 0
dz2 dxz, dyz
11. The compound that inhibits the growth of tumors is
dxz, dyz dxy (2019 Main, 8 April II)
(a) trans-[Pt(Cl)2 (NH3 )2 ] (b) cis-[Pd(Cl)2 (NH3 )2 ]
dx2 – y2 dx2 – y2
(c) cis-[Pt(Cl)2 (NH3 )2 ] (d) trans-[Pd(Cl)2 (NH3 )2 ]
dz2 dz2
(c) E (d) E
12. The correct order of the spin only magnetic moment of metal
dxy dyz, dxz ions in the following low spin complexes, [V(CN)6 ]4− ,
dxz, dyz dxy [Fe(CN)6 ]4− , [Ru(NH3 )6 ]3+ , and [Cr(NH3 )6 ]2+ , is
(2019 Main, 8 April I)
5. The complex ion that will lose its crystal field stabilisation (a) Cr 2+ > Ru3+ > Fe2+ > V2+ (b) V2+ > Cr 2+ > Ru3+ > Fe2+
energy upon oxidation of its metal to +3 state is (c) V2+ > Ru3+ > Cr 2+ > Fe2+ (d) Cr 2+ > V2+ > Ru3+ > Fe2+
13. The magnetic moment of an octahedral homoleptic Mn(II)
complex is 5.9 BM. The suitable ligand for this complex is
(Phen = (a) CN− (b) ethylenediamine
N N (c) NCS− (d) CO

Ignore pairing energy (2019 Main, 12 April I) 14. The pair of metal ions that can given a spin-only magnetic
(a) [Co(phen)3 ] 2+
(b) [Ni(phen)3 ] 2+ moment of 3.9 BM for the complex [M (H2 O)6 ]Cl 2 , is
(2019 Main, 12 Jan I)
(c) [Zn(phen)3 ]2+ (d) [Fe(phen)3 ]2+
(a) Co2+ and Fe2+ (b) Cr 2+ and Mn 2+
6. The crystal field stabilisation energy (CFSE) of (c) V2+ and Co2+ (d) V2+ and Fe2+
[Fe(H2 O)6 ]Cl 2 and K 2 [NiCl 4 ], respectively, are 15. The metal d-orbitals that are directly facing the ligands in
(2019 Main, 10 April II) K 3 [Co(CN)6 ] are (2019 Main, 12 Jan I)
(a) − 0.4 ∆ o and − 12
. ∆t (b) − 0.4 ∆ o and − 0.8 ∆ t (a) dxz , d yz and d 2 (b) d 2 2 and d 2
(c) − 2.4 ∆ o and − 12
. ∆t (d) − 0.6 ∆ o and − 0.8 ∆ t z
(c) dxy , dxz and d yz
x − y z
(d) dxz and d 2 2
x − y
262 Coordination Compounds

16. Mn 2 (CO)10 is an organometallic compound due to the 26. Two complexes [Cr(H 2O)6 ]Cl3 (A) and [Cr(NH3 )6 ]Cl3 (B)
presence of (2019 Main, 12 Jan I) are violet and yellow coloured, respectively. The incorrect
(a) Mn  C bond (b) Mn  O bond statement regarding them is (2019 Main, 9 Jan I)
(c) C O bond (d) Mn  Mn (a) ∆ o value for (A) is less than that of (B)
17 The number of bridging CO ligand(s) and Co Co bond(s) in (b) both absorb energies corresponding to their complementary
colours
Co2 (CO )8 , respectively are (2019 Main, 11 Jan II)
(c) ∆ o values of (A) and (B) are calculated from the energies of
(a) 2 and 0 (b) 0 and 2 (c) 4 and 0 (d) 2 and 1
violet and yellow light, respectively
18. The coordination number of Th in K 4 [Th(C2 O4 )4 (OH2 )2 ] is ( (d) both are paramagnetic with three unpaired electrons
C2O24 − = Oxalato) (2019 Main, 11 Jan II) 27. The recommended concentration of fluoride ion in drinking
(a) 14 (b) 10 (c) 8 (d) 6 water is up to 1 ppm as fluoride ion is required to make teeth
enamel harder by converting [3Ca 3 (PO4 )2 ⋅ Ca(OH)2 ] to :
19. Match the metals (Column I) with the coordination
(2018 Main)
compound(s)/enzyme(s) (Column II). (2019 Main, 11 Jan I)
(a) [CaF2 ] (b) [3(CaF2 ) ⋅ Ca(OH)2 ]
Column I Column II (c) [3Ca 3 (PO4 )2 ⋅ CaF2 ] (d) [3{Ca 3 (PO4 )2} ⋅ CaF2 ]
(A) Co (i) Wilkinson catalyst 28. On treatment of 100 mL of 0.1 M solution of
(B) Zn (ii Chlorophyll CoCl3 .6H2O with excess of AgNO3 ; 1.2 × 1022 ions are
(C) Rh (iii) Vitamin B 12 precipitated. The complex is (2017 Main)
(D) Mg (iv) Carbonic anhydrase (a) [Co(H2O)4 Cl 2 ] Cl ⋅ 2H2O (b) [Co(H2O)3 Cl 3 ] ⋅ 3H2O
A B C D (c) [Co(H2O)6 ]Cl 3 (d) [Co(H2O)5 Cl] Cl 2 ⋅ H2O
(a) (i) (ii) (iii) (iv)
29. The pair having the same magnetic moment is
(b) (iv) (iii) (i) (ii)
[at. no. Cr = 24, Mn = 25, Fe = 26 and Co = 27]
(c) (iii) (iv) (i) (ii)
(a) [Cr(H2O)6 ]2+ and [Fe(H2O)6 ]2+ (2016 Main)
(d) (ii) (i) (iv) (iii)
(b) [Mn(H2O)6 ]2+ and [Cr(H2O)6 ]2+
20. The difference in the number of unpaired electrons of a metal
ion in its high-spin and low-spin octahedral complexes is two. (c) [CoCl 4 ]2− and [Fe(H2O)6 ]2+
The metal ion is (2019 Main, 10 Jan II) (d) [Cr(H2O)6 ]2+ and [CoCl 4 ]2−
2+ 2+ 2+ 2+
(a) Mn (b) Fe (c) Ni (d) Co
30. Among [Ni(CO)4 ], [ NiCl 4 ]2 − , [ Co(NH3 )4 Cl 2 ] Cl,
21. A reaction of cobalt (III) chloride and ethylene diamine in a Na 3 [ CoF6 ], Na 2 O2 and CsO2 , the total number of
1 : 2 mole ratio generates two isomeric products A (violet
coloured) and B (green coloured). A can show optical paramagnetic compounds is (2016 Adv.)
activity, but B is optically inactive. What type of isomers does (a) 2 (b) 3
A and B represent ? (c) 4 (d) 5
(2019 Main, 10 Jan II)
(a) Ionisation isomers (b) Coordination isomers
31. The colour of KMnO4 is due to (2015 Main)
(c) Geometrical isomers (d) Linkage isomers (a) M → L charge transfer transition
(b) d → d transition
22. Wilkinson catalyst is (2019 Main, 10 Jan I)
(c) L → M charge transfer transition
(a) [(Et 3P)3 RhCl] (b) [(Et 3P)3 IrCl](Et = C2H5)
(d) σ → σ∗ transition
(c) [(Ph 3P)3 RhCl] (d) [(Ph 3P)3 IrCl]
23. Homoleptic octahedral complexes of a metal ion ‘M 3 + ’ with 32. The equation which is balanced and represents the correct
three monodentate ligands L1 , L2 and L3 absorb wavelengths product(s) is (2014 Main)
in the region of green, blue and red respectively. The (a) Li 2O + 2KCl → 2LiCl + K 2O
increasing order of the ligand strength is (2019 Main, 9 Jan II) (b) [CoCl(NH3 )5 ]+ + 5H+ → Co2+ + 5NH4+ + Cl −
(a) L1 < L2 < L3 (b) L2 < L1 < L3 Excess NaOH
(c) L3 < L1 < L2 (d) L3 < L2 < L1 (c) [Mg(H2 O)6 ]2+ + (EDTA)4 − →
24. The complex that has highest crystal field splitting energy [Mg(EDTA)]2+ + 6H2 O
( ∆ ), is (d) CuSO4 + 4KCN → K 2 [Cu(CN)4 ] + K 2SO4
(2019 Main, 9 Jan II)
(a) [Co(NH3 )5 Cl] Cl 2 (b) [Co(NH3 )5 (H2O)]Cl 3
33. The octahedral complex of a metal ion M 3+ with four
(c) K 3[Co(CN)6 ] (d) K 2[CoCl 4 ] monodentate ligands L1 , L2 , L3 and L4 absorb wavelengths in
the region of red, green, yellow and blue, respectively. The
25. The highest value of the calculated spin only magnetic increasing order of ligand strength of the four ligands is
moment (in BM) among all the transition metal complexes is (2014 Main)
(2019 Main, 9 Jan I) (a) L4 < L3 , L2 < L1 (b) L1 < L3 < L2 < L4
(a) 5.92 (b) 3.87 (c) L3 < L2 < L4 < L1 (d) L1 < L2 < L4 < L 3
(c) 6.93 (d) 4.90
Coordination Compounds 263

34. Consider the following complex ions, P, Q and R. 45. Mixture X = 0.02 mole of [Co(NH3)5SO4]Br and 0.02 mole of
3− 2+ 2+ [Co(NH3)5Br]SO4 was prepared in 2 L of solution.
P = [ FeF6 ] , Q = [ V(H 2O )6 ] and R = [ Fe(H 2O )6 ]
1 L of mixture X + excess AgNO3 → Y
The correct order of the complex ions, according to their
spin-only magnetic moment values (in BM) is 1 L of mixture X + excess BaCl2 → Z
(2013 Adv.) Number of moles of Y and Z are (2003)
(a) R < Q < P (b) Q < R < P (a) 0.01, 0.01 (b) 0.02, 0.01 (c) 0.01, 0.02 (d) 0.02, 0.02
(c) R < P < Q (d) Q < P < R 46. The complex ion which has no ‘d’-electrons in the central
35. NiCl 2 {P(C2 H5 )2 (C6 H5 )}2 exhibits temperature dependent metal atom is (2001, 1M)
– 3+ 3–
magnetic behaviour (paramagnetic/diamagnetic) the (a) [MnO4 ] (b) [Co(NH3 )6 ] (c) [Fe(CN)6 ] (d) [Cr(H2O)6 ]3+
coordination geometries of Ni 2+ in the paramagnetic and
diamagnetic states respectively, are (2012)
47. The geometry of Ni(CO)4 and Ni(PPh 3 )2 Cl 2 are (1999, 2M)
(a) tetrahedral and tetrahedral (a) both square planar
(b) square planar and square planar (b) tetrahedral and square planar, respectively
(c) tetrahedral and square planar (c) both tetrahedral
(d) square planar and tetrahedral (d) square planar and tetrahedral, respectively

36. Among the following complexes (K-P), (2011) 48. Which of the following is formed when excess of KCN is
added to aqueous solution of copper sulphate? (1996, 1M)
K 3 [Fe(CN)6 ] (K), [Co(NH3 )6 ]Cl 3 (L), (a) Cu (CN)2 (b) K 2[Cu(CN)4 ]
Na 3 [Co (ox)3 ] (M), [Ni(H2 O)6 ] Cl 2 (N), (c) K[Cu(CN)2 ] (d) K 3[Cu(CN)4 ]
K 2 [Pt(CN)4 ](O), [Zn(H2 O)6 ](NO3 )2 (P) 49. Among the following ions, which one has the highest
the diamagnetic complexes are paramagnetism? (1993, 1M)
(a) K, L, M, N (b) K, M, O, P (a) [Cr(H2O)6 ]3+ (b) [Fe(H2O)6 ]2+
(c) L, M, O, P (d) L, M, N, O
(c) [Cu(H2O)6 ]2+ (d) [Zn(H2O)6 ]2+
37. The complex showing a spin only magnetic moment of
50. Amongst Ni(CO)4 , [Ni(CN)4 ]2– and NiCl 2–
4 (1991, 1M)
2.82 BM is (2010)
(a) Ni(CO)4 (b) [NiCl 4 ]2− (a) Ni(CO)4 and NiCl 2–
4 are diamagnetic and [Ni(CN)4 ]
2–
is
paramagnetic
(c) Ni(PPh 3 )4 (d) [Ni(CN)4 ]2−
(b) [NiCl 4 ]2− and [Ni(CN)4 ]2– are diamagnetic and Ni(CO)4 is
38. The spin only magnetic moment value (in Bohr magneton paramagnetic
units) of Cr(CO)6 is (2009) (c) Ni(CO)4 and [Ni(CN)4 ]2– are diamagnetic and [NiCl 4 ]2− is
(a) 0 (b) 2.84 (c) 4.90 (d) 5.92 paramagnetic
(d) Ni(CO)4 is diamagnetic and [NiCl 4 ]2− and [Ni(CN)4 ]2– are
39. Among the following, the coloured compound is (2008, 3M)
(a) CuCl (b) K 3[Cu(CN)4 ] paramagnetic
(c) CuF2 (d) [Cu(CH3CN)4 ]BF4 51. Amongst the following, the lowest degree of paramagnetism
per mole of the compound at 298 K will be shown by
40. Both [Ni(CO)4 ] and [Ni(CN)4 ]2− are diamagnetic. The (1988, 2M)
hybridisations of nickel in these complexes respectively, are (a) MnSO4 ⋅ 4H2O (b) CuSO4 ⋅ 5H2O
(a) sp3 , sp3 (b) sp3 , dsp2 (2008, 3M) (c) FeSO4 ⋅ 6H2O (d) NiSO4 ⋅ 6H2O
2 3
(c) dsp , sp (d) dsp2 , dsp2
Objective Question II
41. Among the following metal carbonyls, the C—O bond order
is lowest in (2007, 3M)
(One or more than one correct option)
(a) [Mn(CO)6 ]+ (b) [Fe(CO)5 ] 52. Choose the correct statement(s) among the following:
− (2020 Adv.)
(c) [Cr(CO)6 ] (d) [V(CO)6 ]
(a) [FeCl 4 ]− has tetrahedral geometry.
42. If the bond length of CO bond in carbon monoxide is 1.128 Å, (b) [Co(en)(NH3 )2 Cl 2 ]+ has 2 geometrical isomers.
then what is the value of CO bond length in Fe(CO)5? (2006) (c) [FeCl 4 ]− has higher spin-only magnetic moment than
(a) 1.15 Å (b) 1.128 Å (c) 1.72 Å (d) 1.118 Å [Co(en)(NH3 )2 Cl 2 ]+ .
43. Spin only magnetic moment of the compound Hg[Co(SCN)4 ] (d) The cobalt ion in [Co(en)(NH3 )2 Cl 2 ]+ has sp3d 2 hybridisation.
is (2004, 1M) 53. A tin chloride Q undergoes the following reactions (not
(a) 3 (b) 15 (c) 24 (d) 8 balanced)
44. The compound having tetrahedral geometry is (2004, 1M) Q + Cl − → X
(a) [Ni(CN)4 ] 2–
(b) [Pd(CN)4 ] 2–
Q + Me3N → Y
(c) [PdCl 4 ] 2–
(d) [NiCl 4 ]2– Q + CuCl 2 → Z + CuCl
264 Coordination Compounds

X is a monoanion having pyramidal geometry. Both Y and Z 58. Statement I [Fe(H2 O)5 NO]SO4 is paramagnetic.
are neutral compounds. (2019 Adv.) Statement II The Fe in [Fe(H2 O)5 NO]SO4 has three
Choose the correct option(s).
unpaired electrons. (2008, 3M)
(a) There is a coordinate bond in Y
(b) The central atom in Z has one lone pair of electrons
(c) The oxidation state of the central atom in Z is + 2 Passage Based Questions
(d) The central atom in X is sp3 hybridised The coordination number of Ni 2 + is 4.
NiCl 2 + KCN (excess) → A (cyano complex)
54. The correct statement (s) regarding the binary transition
metal carbonyl compounds is (are) (Atomic numbers : NiCl 2 + conc. HCl (excess) → B (chloro complex)
Fe = 26, Ni = 28) (2018 Adv.) 59. Predict the magnetic nature of A and B.
(a) Total number of valence shell electrons at metal centre in (a) Both are diamagnetic
Fe(CO)5 or Ni(CO) 4 is 16 (b) A is diamagnetic and B is paramagnetic with one unpaired
(b) These are predominantly low spin in nature electron
(c) Metal-carbon bond strengthens when the oxidation state of the (c) A is diamagnetic and B is paramagnetic with two unpaired
metal is lowered electrons
(d) The carbonyl C—O bond weakens when the oxidation state of (d) Both are paramagnetic
the metal is increased
60. The hybridisation of A and B are
55. The correct option(s) regarding the complex (a) dsp2 , sp3 (b) sp3 , sp3
[Co(en)(NH3 )3 (H2 O)]3+ (en = H2 NCH2 CH2 NH2 ) is (are)
(c) dsp2 , dsp2 (d) sp3d 2 , d 2sp3
(a) It has two geometrical isomers (2018 Adv.)
(b) It will have three geometrical isomers, if bidentate ‘en’ is
replaced by two cyanide ligands Match the Columns
(c) It is paramagnetic 61. Match each set of hybrid orbitals from List–I with complexes
(d) It absorbs light at longer wavelength as compared to given in List-II.
[Co(en)(NH3 )4 ]3+
List–I List–II
56. Addition of excess aqueous ammonia to a pink coloured 2 4−
aqueous solution of MCl 2 ⋅ 6H2 O( X ) and NH4 Cl gives an P. dsp 1. [FeF6 ]
octahedral complex Y in the presence of air. In aqueous Q. sp3 2. [Ti(H2O)3 Cl 3 ]
solution, complex Y behaves as 1 : 3 electrolyte. The reaction
3 2
of X with excess HCl at room temperature results in the R. sp d 3. [Cr(NH3 )6 ]3+
formation of a blue colured complex Z. The calculated spin
only magnetic moment of X and Z is 3.87 B.M., whersas it is
S. d 2sp3 4. [FeCl 4 ]2−
zero for complex Y . 5. [Ni(CO)4 ]
Among the following options, which statement(s) is (are)
6. [Ni(CN)4 ]2−
correct? (2017 Adv.)
(a) The hybridisation of the central metal ion in Y is d 2sp3 The correct option is (2018 Adv.)
(b) Addition of silver nitrate to Y given only two equivalents of (a) P → 5; Q → 4, 6; R → 2, 3; S → 1
silver chloride (b) P → 5,6; Q → 4; R → 3; S → 1,2
(c) When X and Y are in equilibrium at 0°C, the colour of the (c) P → 6; Q → 4, 5; R → 1; S → 2, 3
solution is pink (d) P → 4,6; Q → 5, 6; R → 1,2; S → 3
(d) Z is a tetrahedral complex
62. Match each coordination compound in Column I with an
Numerical Answer Type Question appropriate pair of characteristics from Column II and
57. Total number of cis N  Mn Cl bond angles (that is select the correct answer using the codes given below the
Mn N and Mn Cl bonds in cis positions) present in a Columns (en = H2 NCH2 CH2 NH2 ; atomic numbers : Ti = 22;
molecule of cis [Mn(en)2 Cl 2 ] complex is ……… Cr = 24; Co = 27; Pt = 78) (2014 Adv.)

(en = NH 2CH 2CH 2NH 2 ) (2019 Adv.) Column I Column II


(A) [Cr(NH3 )4 Cl 2 ]Cl 1. Paramagnetic and exhibits
Assertion and Reason ionisation isomerism
Read the following questions and answer as per the direction
(B) [Ti(H2O)5 Cl](NO3 )2 2. Diamagnetic and exhibits
given below :
cis-trans isomerism
(a) Statement I is true; Statement II is true; Statement II is the
correct explanation of Statement I. (C) [Pt(en)(NH3 )Cl]NO3 3. Paramagnetic and exhibits
(b) Statement I is true; Statement II is true; Statement II is not the cis-trans isomerism
correct explanation of Statement I. (D) [Co(NH3 )4 (NO3 )2 ]NO3 4. Diamagnetic and exhibits
(c) Statement I is true; Statement II is false. ionisation isomerism
(d) Statement I is false; Statement II is true.
Coordination Compounds 265

Codes (a) Draw its structure and show H-bonding


A B C D A B C D (b) Give oxidation state of Ni and its hybridisation
(a) 4 2 3 1 (b) 3 1 4 2 (c) Predict whether it is paramagnetic or diamagnetic (2004, 4M)
(c) 2 1 3 4 (d) 1 3 4 2
71. Write the IUPAC name of the compound
63. Match the complexes in Column I with their properties listed K 2 [Cr(NO)(CN)4 (NH3 )] . Spin magnetic moment of the
in Column II. (2007, 6M) complex µ = 1.73 BM. Give the structure of anion.
(2003, 4M)
Column I Column II
2–
(A) [Co(NH3 )4 (H2O)2 ]Cl 2 p. Geometrical isomers 72. Deduce the structures of [NiCl 4 ] and [Ni(CN)4 ]2–
(B) [Pt(NH3 )2 Cl 2 ] q. Paramagnetic considering the hybridisation of the metal ion. Calculate the
(C) [Co(H2O)5 Cl]Cl r. Diamagnetic
magnetic moment (spin only) of the species. (2002, 5M)

(D) [Ni(H2O)6 ]Cl 2 s. Metal ion with +2 73. A metal complex having composition Cr(NH3 )4 Cl 2 Br has
oxidation state been isolated in two forms A and B. The form A reacts with
AgNO3 to give a white precipitate readily soluble in dilute
Fill in the Blank aqueous ammonia, whereas B gives a pale yellow precipitate
64. The IUPAC name of [Co(NH3 )6 ] Cl 3 is …… soluble in concentrated ammonia.
(1994, 1M)
Write the formula of A and B and state the hybridisation of
True/False chromium in each. Calculate their magnetic moments
65. Both potassium ferrocyanide and potassium ferricyanide are (spin-only value). (2001, 5M)
diamagnetic. (1989, 1M) 74. Draw the structures of [Co (NH3 )6 ]3+ , [Ni(CN)4 ]2– and
66. The electron density in the xy plane in 3dx 2 − y2
orbital is zero. [Ni(CO)4 ] . Write the hybridisation of atomic orbitals of the
(1986, 1M)
transition metal in each case. (2000, 4M)
Integer Answer Type Questions 75. A, B and C are three complexes of chromium (III) with the
67. For the octahedral complexes of Fe3+ in SCN− empirical formula H12 O6 Cl 3 Cr. All the three complexes have

(thiocyanato-S) and in CN ligand environments, the water and chloride ion as ligands.
difference between the spin only magnetic moments in Bohr Complex A does not react with concentrated H2 SO4 , whereas
magnetons (when approximated to the nearest integer) is complexes B and C lose 6.75% and 13.5% of their original
[atomic number of Fe = 26 ] (2015 Adv.)
mass, respectively, on treatment with concentrated H2 SO4 .
68. In the complex acetylbromidodicarbonylbis (triethylphosphine) Identify A, B and C. (1999, 2M)
iron (II), the number of Fe  C bond (s) is (2015 Adv.)
76. Identify the complexes which are expected to be coloured.
4−
69. EDTA is ethylenediaminetetraacetate ion. The total Explain (1994, 2M)
number of N  Co  O bond angles in [Co(EDTA)] − (i) [Ti(NO3 )4 ] (ii) [Cu(NCCH3 )]+ BF4
complex ion is (2013 Adv.) (iii) [Cr(NH3 )6 ] Cl3 (iv) K3 [VF6 ]

Subjective Questions 77. Give reasons in two or three sentences only for the
following :
70. NiCl 2 in the presence of dimethyl glyoxime (DMG) gives a
complex which precipitates in the presence of NH4 OH, giving “The species[CuCl 4 ]2– exists, while[CuI4 ]2– does not exist.”
a bright red colour. (1992, 1M)

Answers
Topic 1 Topic 2
1. (a) 2. (d) 3. (c) 4. (d) 1. (d) 2. (b) 3. (d) 4. (a)
5. (c) 6. (b) 7. (a) 8. (c) 5. (d) 6. (b) 7. (a) 8. (b)
9. (b) 10. (a) 11. (b) 12. (c) 9. (b) 10. (a) 11. (c) 12. (b)
13. (d) 14. (b) 15. (c) 16. (b) 13. (c) 14. (c) 15. (b) 16. (a)
17. (c) 18. (a) 19. (b,d) 20. (c,d) 17. (d) 18. (b) 19. (c) 20. (d)
21. (b) 22. (a) 23. (paramagnetism) 21. (c) 22. (c) 23. (c) 24. (c)
24. (5) 25. (6) 26. (6) 27. (3) 25. (a) 26. (c) 27. (c) 28. (d)
29. (a) 30. (b) 31. (c) 32. (b)
266 Coordination Compounds

33. (b) 34. (b) 35. (c) 36. (c) 57. (6) 58. (a) 59. (c) 60. (a)
37. (b) 38. (a) 39. (c) 40. (b) 61. (c) 62. (b)
41. (b) 42. (a) 43. (b) 44. (d) 63. (A → p, q, s B → p, r, s C → q, s D → q, s)
45. (a) 46. (a) 47. (c) 48. (d) 64. (hexaammine cobalt (III) chloride) 65. (F)
49. (b) 50. (c) 51. (b) 52. (a,c) 66. (F) 67. (4) 68. (3) 69. (8)
53. (a, d) 54. (b, c) 55. (a, b, d) 56. (a, b, d)

Hints & Solutions


Topic 1 Nomenclature and Isomerism of In first complex, ‘en’ is a didentate ligand and ‘Cl’ is a unidentate
Coordination Compounds ligand.
1. If AB4 molecule is a polar moelcule, a possible geometry of AB4 [Co(Cl)(en)2]Cl, coordination number = 1 + 2 × 2 = 1 + 4 ⇒ 5
is square pyramidal. So, the coordination number is 5.
All possible structure of AB4 molecule are as follows: For K 3 [Al(C2O4 )3 ], ‘C2O24 − ’ is a didentate ligand.
(i) AB4 ⇒ L = 0 ⇒ A is sp3 (tetrahedral) or Coordination number = 3 × 2 = 6.
Ais dsp2 (square planar) Hence, coordination number is 6.
⇒ Dipole moment, µ = 0(Non-polar) Key Idea Square planar complexes of general formulae :
3.
(ii) AB4L2 ⇒ L = 2 ⇒ A is sp3d 2 (octahedral) [ M(a − a)b2 ] and [ M(a − a) (b − b) ] do not show geometrical
⇒µ = 0(Non-polar), because isomerism. Whereas, an octahedral complex of general
formula [ M (a − a)2 b2 ] can show geometrical (cis-trans)
isomerism.
B B
[ Pt(en)2 (Cl 2 )]2+ with formula [ M (a − a)2 b2 ] will show
A ⇒ µnet = 0 (Non-polar) geometrical isomerism as follows:
2+
2+ Cl
B B
en
(Assuming, B is more electronegative than A) Cl
en Pt en
(iii) AB4L ⇒ L = 1 ⇒ A is sp3d (square pyramidal or trigonal Pt
bipyramidal) Cl
⇒µ ≠ 0 (polar), because en
Cl
B cis- trans-
(optically active) (optically inactive)
B B B (I)

A A
4. The maximum possible denticities of given ligand towards a
common transition and inner transition metal ion, are 6 and 8
respectively.
B B B –OOC COO–
µplane = 0 B N N N
µaxis ≠ 0 µplane ≠ 0 –OOC
COO–
µnet ≠ 0 (polar) µaxis ≠ 0
µnet ≠ 0 (polar) COO–
[Square pyramidal]
[Trigonal pyramidal] The given ligand act as hexadentate ligand in transition metal ion
because the common oxidation state shown by them is +3.
So, it can be seen that when AB4 molecule is a polar molecule Whereas in case of inner transition metal ion, its denticity is 8
then possible geometry of AB4 is square pyramidal. because their common oxidation state is +4.

2. 5. Optical activity is the ability of a chiral molecule to rotate the


Key Idea The total number of ligands to which the metal is
plane of polarised light, measured by a polarimeter. A chiral
directly attached is called coordination number. molecule does not have any plane of symmetry. If a molecule
possess any plane of symmetry, then it is an achiral molecule.
The coordination numbers of Co and Al in [Co(Cl)(en)2]Cl and
Given options (a), (b) and (d) possess plane of symmetry.
K 3[ Al(C2O4 )3 ]are 5 and 6 respectively.
Coordination Compounds 267

or 2 + x − 2 − 4 − 2 = 0 or x − 6 = 0
A A hence x = + 6
B Thus, +3, 0 and +6 is the answer.
B 9. If the reactant is cis isomer than following reaction takes place.
en M en
M Br Br Br
B NH3 Br NH3 Br NH3 Br
B
Br–
+
A A
NH3 NH3 NH3 Br NH3 NH3
(b) (d)
B NH3 NH3 Br
A B Cis-isomer Facial Meridonial
i.e. two isomers are produced. If the reactant is trans isomer than
M
following reaction takes place.
B A Br Br
NH3 NH3 NH3 NH3
Br–
A
(a)
NH3 NH3 NH3 Br
Only molecule (c) does not possess any plane of symmetry.
Hence, it is a chiral molecule and shows optical activity. Br Br
Trans Meridonial
6. Key Idea Denticity of ligand is defined as donor sites or i.e. only 1 isomer is produced. Thus, statement (I) and (III) are
number of ligating groups. correct resulting to option (b) as the correct answer.
The given ligand is tetradentate. It contains four donor atoms. It 10. Cl Cl
can bind through two nitrogen and two oxygen atom to the Cl
central metal ion. en Co en Co en
Ligand bound to the central atom or ion through coordinate bond
in the coordination entity. It act as a Lewis base. The attacking en
Cl
site of the given ligand is given in bold. cis-[Co(en)2Cl2]Cl trans-[Co(en)2Cl2]Cl
(optically active) (optically inactive due
NEt2 to plane of symmetry)

N [Co(NH3 )4 Cl 2 ]Cl can exist in both cis and trans forms that are
given below:
Cl + NH3 +
O– –O
H3N NH3 H 3N Cl
Co Co

7. A square planar complex of general formula, M abcd gives three H3N NH3 H 3N Cl
geometrical isomers only. Cl NH3
Let, a = F− , b = Cl − , c = S CN− , d = NO−2 trans-[Co(NH3)4Cl2]Cl cis-[Co(NH3)4Cl2]Cl
SCN− and NO−2 are ambidentate ligands and they also show (optically inactive) (optically inactive)

linkage isomerism (structural). Considering both linkage and [Co(NH3 )3 Cl 3 ]exists in fac and mer-isomeric forms and both are
geometrical isomerism. optically inactive.
Total number of possible isomers given by the complex, NH3 NH3
= 3 × (2 + 2) = 12 Cl NH3 Cl NH3
8. Let the oxidation state of Cr in all cases is ‘ x’ Co Co
(i) Oxidation state of Cr in [Cr(H2O)6 ]Cl 3
Cl NH3 Cl Cl
x + (0 × 6) + (−1 × 3 ) = 0 Cl NH3
or x + 0 − 3 = 0 or x = + 3 fac-isomer mer-isomer
(ii) Oxidation state of Cr in [Cr(C6H6 )2 ] (optically inactive) (optically inactive)
x + (2 × 0) = 0 or x = 0
(iii) Oxidation state of Cr in
11. [Pt(Cl)(py)(NH3 )(NH2OH)]+ is square planar complex. The
K2[Cr(CN)2 (O)2 (O2 )(NH3 )] structures are formed by fixing a group and then arranging all the
1 × 2 + x + (−1 × 2 ) + (−2 × 2 ) + (−2 ) + 0 = 0 groups.
268 Coordination Compounds

Py NH3 Py NH3 16. Ionisation isomers are the complexes that produces different
ions in solution, i.e. they have ions interchanged inside and
Pt Pt outside the coordination sphere.
Cl [Cr(H2O)4 Cl(NO2 )]Cl and [Cr(H2O)4 Cl 2 ](NO2 ) have different
NH2OH HOH2N Cl ions inside and outside the coordinate sphere and they are
isomers.
Py Cl Therefore, they are ionisation isomers.
Pt 17. [Ni(NH3 )4 ]2 + = tetraamminenickel (II)
HOH2N NH3 [NiCl 4 ]2− = tetrachloronickelate (II)
Cationic part is named first, hence :
Hence, this complex shows three geometrical isomers. tetraamminenickel (II)-tetrachloronickelate(II)
12. Optical isomerism is exhibited by only those complexes which 18. [Co(NH3 )4 Br2 ]Cl and [Co(NH3 )4 BrCl]Br are ionisation isomers.
lacks elements of symmetry. [Co(NH3 )3 Cl 3] shows facial as well Br Br
as meridional isomerism. But both the forms contain plane of H 3N NH3
+
H 3N Br +
symmetry. Thus, this complex does not exhibit optical
isomerism. Co Co
13. First of all, the compound has complex positive part
‘‘[Co(H2O)4 (NH3 )2 ]3+ therefore, according to IUPAC H 3N NH3 H3N NH3
conventions, positive part will be named first. Secondly, in Br NH3
writing name of complex, ligands are named first in alphabetical Geometrical isomers
order, irrespective of its charge, hence “ammine” will be written
prior to “aqua”. Therefore, name of the complex is 19. PLAN Depending on the structure of the complex,different types of
isomerism are shown.
[Co(H2O)4 (NH3 )2 ]Cl 3.Diamminetetraaqua cobalt (III) chloride.
NOTE In alphabetical order, original name of ligands are Complex Isomerism
considered not the initials of prefixes. Also, special precaution A.
should be taken in spelling name of NH3 ligand as it is ammine. [Cr(NH3 )5 Cl]Cl 2 Neither of structural nor
[Cr(NH3 )4 Cl 2 ] Cl stereoisomerism
14. Ni 2+ + 4 CN– → [ Ni(CN)4 ]2–
B. [Co (NH3 )4 Cl 2 ]+ Cl
Here, Ni 2+ has d 8 -configuration with CN– as strong ligand. H3N NH3
3d 4s 4p Co
H3N NH3
:

:
:
:

Cl
trans w.r.t. Cl
dsp2
NH3
d 8 -configuration in strong ligand field gives dsp2-hybridisation, H3N Cl
hence square planar geometry. Co
H3N Cl
Ni 2+ + 4Cl – → [NiCl 4 ]2– NH3
Here, Ni 2+ has d 8 -configuration with Cl – as weak ligand. cis w.r.t. Cl
+
3d [Pt(H2O) ⋅ (NH3 )2 Cl] H2O NH3
Pt
:

:
:
:

Cl NH3
2 cis
sp
H2O NH3
d 8 -configuration in weak ligand field gives sp 3-hybridisation, Pt
hence tetrahedral geometry. Ni 2+ with H 2O forms H3N Cl
[Ni(H 2O)6] 2+ complex and H 2O is a weak ligand. trans

3d
C. [Co Br2Cl 2 ]2− sp3 tetrahedral
:

:
:
:

4s 4p 4d [PtBr2Cl 2 ]2− dsp2 square planar


:

:
:
:

:
:

D. [Pt (NH3 )3 (NO3 )]Cl [ Pt (NH3 )3 (NO3 )Cl ]


sp3d2 [ Pt (NH3 )3 Cl ] NO3 ]
ionisation
Therefore, [Ni(H 2O)6] 2+ has octahedral geometry.
[Pt(NH3 )3 Cl]Br [ Pt (NH3 )3 Cl ] Br
HOOCH2C CH2COOH [ Pt (NH3 )3 Br ]
15. N—CH2CH2—N
HOOCH2C CH2COOH ionisation
Coordination Compounds 269

20. Both [Pt(en)2 Cl 2 ]Cl 2 and [Pt(NH3 )2 Cl 2 ] are capable of showing 25. All six complex will show cis-trans isomerism
geometrical isomerism. Cl
en + +
Cl + Cl + Cl
Cl
Co3+ and en Co3+ en
en Pt en en Pt
Cl
en
Cl en cis Cl
trans
trans cis O
H3N Cl H3N Cl O
O 3–
Cl 3–
Pt Pt O Cl O O O
O
H 3N Cl Cl NH3 Cr3+
cis trans Cr3+ and

Square planar complex


Cl O O O
O O
O Cl
21. Both statements are true. However, axis of symmetry is not a trans
criteria of optical isomerism. Optical inactivity of the two O O
geometrical isomers of [ M (NH3 )4 Cl 2 ] is due to the presence of OH2 + OH + CN –
plane of symmetry. H2O OH H2O OH2 NC NH3
Cl NH3 Fe3+ and Fe3+ Fe3+ and
NC NH3
H 3N NH3 H3N Cl H 2O OH H2O OH2
OH2 OH CN
trans cis
M M cis
– 2+ 2+
NH3 Cl
H 3N NH3 en
H3N Cl
NC CN NH3
Cl NH3 3+ Co3+ and en Co3+ en
Fe
Trans Cis
NC CN Cl
Both have several planes of symmetry en
NH3 NH3
22. A = K2[Ni(CN)4 ] ; B = K2[NiCl 4 ] trans cis trans
2+ 2+
A : Potassium tetracyanonickelate (II) NH3 Cl
B : Potassium tetrachloronickelate (II) H 3N
H3N
NH3
Cl
2+
23. Paramagnetism : In [Mn(H2O)6 ] , Mn(II) has 3d configuration. 5 Co3+ and Co3+
H3N OH2 H 3N NH3
Since, H2O is a weak ligand, all five d-electrons are unpaired :
3d 4s 4p 4d NH3 OH2
cis trans
Mn(II) :
(weak ligand field)
26. mmol of complex = 30 ×0.01 = 0.3
Also, 1 mole of complex [Cr(H2O)5Cl]Cl2 gives only two moles
24. Ligand is CH2 — NH2
of chloride ion when dissolved in solution

CH2 O [Cr(H2O)5 Cl]Cl 2 → [Cr(H2O)5 Cl]2+ + 2Cl –
Geometrical isomers are ⇒ mmol of Cl – ion produced from its 0.3 mmol = 0.6
Hence, 0.6 mmol of Ag+ would be required for precipitation.
NH2
NH2 NH2
Cl Cl
3+ NH2 O ⇒ 0.60 mmol of Ag+ = 0.1M × V(in mL) ⇒ V = 6 mL
3+ O O
Co Co Co3+
O 27. Cl PPh3 Cl CO
Cl O Cl NH2 Cl
NH2 O
Cl Rh Rh
Cis Trans Cis
H 3N CO H 3N PPh3
Cl PPh3
Rh

OC H 3N
28. (i) [Co(NH3 )5 Cl]2+ (ii) Li[AlH4 ]
270 Coordination Compounds

29. [Cr(NH3 )5 CO3 ]Cl 3. The compound used in the treatment of lead poisoning is EDTA.
: pentaamminecarbonatochromium (III) chloride. Medication occurs through chelation therapy. Calcium disodium
ethylenediamine tetraacetic acid chelates divalent metal ion such
30. (i) [Co(NH3 )5 ONO]Cl 2
as Pb2 + from plasma and interstitial body fluids.
: pentaamminenitritocobalt (III) chloride.
(ii) K3[Cr(CN)6 ] : potassium hexacyanochromate (III) The metal displaces Ca and is chelated, mobilised and usually
excreted. Less then 5% CaNa 2EDTA is absorbed in the
Topic 2 Bonding and Important Property gastrointestinal tract and it possibly increases the absorption of
of Coordination Compounds Pb present in the tract. Therefore, it is not recommended for oral
use. It is usually given intravenously.
1. M 2+ + mH2O → [ M (H2O)m ]2+
(d 6 ) 4. Complete removal of both the axial ligands (along the z-axis)
Here, H2O is a weak field ligand and it should give a high spin from an octahedral complex leads to the following splitting pattern.
paramagnetic (µ = 4.90 BM) complex ion.
dx2–y 2
We know, µ = n ( n + 2 ) BM = 4.90 BM (n = 4)
[∴n = Number of unpaired electrons in the complex]
dxy

Energy
(n–1)d6 ns0 np0 eg

M 2+ =
dz2
(d6) t2g
n=4 H2O H2O H2O H2O

M2+ is sp3 hybridised, i.e. it


dxzdyz
is a tetrahedral complex ion
[M(H2O)4]2+ [∴ M = 4]
The single electron in the dx2 − y2 orbital is being repelled by four
(n −1) d -orbitals in tetrahedral field. ligands, while the electron in the dz2 orbital is only being repelled
t2 by two ligands. Thus, the energy of the dx2 − y2 increases relative
to that of dz2 . A more stable arrangement arises when both the eg
electrons pair up and occupy the lower energy dz2 orbital. This
+0.4 ∆ t
leaves the dx2 − y2 orbital empty.
Thus, four ligands can now approach along + x , − x, + y and − y
directions without any difficulty as dx2 − y2 orbital is empty.
–0.6 ∆ t However, ligands approaching along + z and −z directions meet
very strong repulsive forces from filled dz2 orbitals. Thus, only
e
four ligands succeed in bonding to the metal. A square planar
CFSE = [(− 0.6) × 3 + (0.4 ) × 3 ]∆ t = − 0.6 ∆ t complex is formed, the attempt to form an octahedral complex
being unsuccessful.
Number of Number of
electrons in e set. electrons in t2 set.
5. Key Idea Crystal field splitting occurs due to the presence
So, the geometry and the crystal field stabilisation energy of the of ligands in a definite geometry. In octahedral complexes
complex is tetrahedral and − 0.6 ∆ t respectively. the energy of two, eg orbitals will increase by (0.6) ∆ o and
2. The d-electron configuration [Ru(en) 3]Cl 2 and [ Fe(H2O)6 ]Cl 2 that of three t2g will decrease by (0.4) ∆ o .
respectively are t62geg0 and t24geg2. The complex ion that will lose its crystal field stabilisation
[Ru(en)3 ]Cl2 energy upon oxidation of its metal to +3 state is[ Fe(phen)3 ]2+ .
Ru = 4d series −e−
en = bidentate ligand (strong field ligand) [Fe(phen)3 ]2+ → [Fe(phen)3 ]3+
C.N. = 6 In [Fe(phen)3 ]2+ , electronic configuration of Fe2+ is 3d6 4 s0.
Oxidation number = +2 Phenanthrene is a strong field symmetrical bidentate ligand. The
Ru 2+ = [Kr] 4 d6 5s0 splitting of orbital in Fe2+ is as follows:
Fe2+ = eg

[Fe(H 2O) 6 ]Cl2 = +0.6 ∆o


eg
Fe2+
0.6∆0
–0.4 ∆o
∆0 t 4 , e 2 t2g
(n–1)d 2g g
0.4∆0
(3d)
CFSE = 6 × −0.4 ∆ o = −2.4 ∆ o .
t2g
Coordination Compounds 271

The splitting of orbital and arrangement of electrons in Fe3+ is as [Cr(H2O)6 ]2+ , µ 2 = 5(5 + 2)
follows : (n = 5), = 35 = 5.92 BM
eg So, µ 1 ≈ µ 2.Thus, statement (c) is correct.
(d) [Ni(NH3 )4 (H2O)2 ]2+ is also a high-spin octahedral complex
+0.6 ∆o
of Ni2+ (3d 8 , n = 2)
Fe3+
µ = 2(2 + 2) = 8 = 2.83 BM
–0.4 ∆o
Thus, statement (d) is correct.
t2g
CFSE = 5 × −0.4 ∆ o = −2. 0 ∆ o 8. Key Idea The wavelength (λ ) of light absorbed by the
2+
Fe upon oxidation of its metal to +3 state lose its CFSE from complexes is inversely proportional to its ∆ 0 CFSE
−2.4 ∆ o to −2.0∆ o . (magnitude). ∆ 0 (CFSE) ∝ 1 / λ

6. The complexes can be written as:


Key Idea Crystal field stabilisation energy (CFSE) for
octahedral complexes = (−0.4 x + 0.6 y)∆ o I. [CoCl(NH3 )5 ]2+ ≡ [Co(NH3 )5 (Cl)]2+ ]
where, x = number of electrons occupying t2g orbital. II. [Co[NH3 ]5 H2O]3+ ≡ [Co(NH3 )5 (H2O)]3+
y = number of electrons occupying eg orbital. III. [Co(NH3 )5 ]3+ ≡ [Co(NH3 )5 (NH3 )]3+
CFSE for tetrahedral complexes So, the differentiating ligands in the octahedral complexes of
= (−0.6x + 0.4 y)∆ t Co (III) in I, II and III are Cl s, H2O and NH3 respectively. In the
where, x = number of electrons occupying e orbital. spectrochemical series, the order of this power for crystal field
y = number of electrons occupying t orbital. splitting is Cl − < H2O < NH3.
So, the crystal field splitting energy (magnitude) order will be
In [Fe(H2O)6 ]Cl2, H2O is a weak field ligand, so it is a high spin
(outer orbital) octahedral complex of Fe2+ . ∆CFSE
0 (I) < ∆CFSE
0 (II) < ∆CFSE
0 (III)
and the order of wavelength (λ ) of light absorbed by the
eg complexes will be
Fe2+(3d6) =
t2g  1
λ (I) > λ (II) > λ (III) Q Energy (∆CFSE )∝
 λ 
0
∴ CFSE = (−0.4 x + 0.6 y)∆ o
9. The degenerate orbitals of [Cr(H2O)6 ]3+ are dxz and d yz.
= [ −0.4 × 4 + 0.6 × 2 ]∆ o = − 0.4 ∆o
In K2[NiCl 4 ], Cl − is a weak field ligand, so it is a high spin Electronic configuration of Cr3 + is 3d 5 4 s1. The five d-orbitals in
an isolated gaseous atom or ion have same energy, i.e. they are
tetrahedral complex of Ni2+ . degenerate. This degeneracy has been removed due to the ligand
t
electron–metal electron repulsions in the octahedral complex to
yield three orbitals of lower energy, t2g set and two orbitals of
Ni2+(3d6) =
higher energy, eg set.
e
(e g )
∴CFSE = (−0.6 × 4 + 0.4 × 4 )∆ t = − 0.8∆ t
dx 2–y2 dz2
7. The explanation of given statements are as follows : ∆0
(a) Ruby, a pink or blood-red coloured gemstone belongs to Free metal ion (t2g)
corundum (Al 2O3 , alumina) system which has trigonal dxy dxz dyz
crystalline lattice containing the repeating unit of 10. [ Fe(H2O)6 ]2 ⇒ It will form 2 cationic species. i.e.
Al 2O3 – Cr 3+ . So, ruby does not belong to beryl lattice II
(Be3Al 2Si6O18 ). I. (i)As [ Fe(H2O)6 ]2+ ⇒ High spin octahedral complex of Fe2+ .
Thus, statement (a) is incorrect. Fe2+ : 3d6 , x = 4 (unpaired electrons)
(b) [Co(Cl)(NH3 )5 ]2+ is a low spin octahedral complex of Co3+ . µ = 4 (4 + 2) BM = 4.9 BM
It absorbs low energy yellow light and high energy III
complementary violet light will be shown off. Thus, or (ii) as[ Fe(H2O)6 ]3+ = High spin octahedral complex of Fe3+ .
statement (b) is correct. Fe3 + : 3d5 , x = 5, µ = 5 (5 + 2) = 5.92 BM
(c) [Fe(H2O)6 ]2+ and [Cr(H2O)6 ]2+ are the high-spin octahedral
[H2O is a neutral weak field ligand]
complexes of Fe2+ (3d6 , n = 4 ) and Cr 2+ (3d 5 , n = 5) ions
So, [ Fe(H2O)6 ]2+ will be the cationic specie, µ = 4.9 BM.
and weak field ligand, H2O respectively. So, spin-only
[ Fe(CN)6 ] will have two anionic complexes
magnetic moment = n(n + 2) of the complexes. II
II. (i) [ Fe(CN)6 ]4– ⇒ Low spin, octahedral complex of Fe2+ .
[Fe(H2O)6 ]2+ , µ 1 = 4 (4 + 2)
(n = 4 ), = 24 = 4.89 BM As CN− is a strong ligand it will pair up the electrons.
272 Coordination Compounds

d2sp3 The octahedral homoleptic complex suggests


Fe2+(3d6)
sp3d 2-hybridisation in the complex, i.e.
sp3d2-hybridisation
n=0, µ=0
III
Mn2+=
or, (ii) [ Fe(CN)6 ]3– ⇒ Low spin octahedral complex of Fe3+ . 3d 4s 4p 4d

Fe3+ ⇒ (3d5)
d2sp3 Thus, 5 unpaired electrons are present in the complex which
suggest the presence of a weak ligand like NCS − .
14. As H2O is a weak field ligand. It readily forms high spin
n=1, µ=√1(1+2)
=1.73 BM complexes. In[M (H2O)6 ]Cl2, M exist in +2 oxidation state. The
(Not in options) arrangement of electrons in the given metal ions are as follows:
[CN − is an anionic strong field ligand] Metal Configuration Number of Spin only
So,the anionic species is [Fe(CN)6 ]4− , µ = 0 ions unpaired Magnetic
Thus, the calculated spin only magnetic moments (BM) of the electrons moment (in
anionic and cationic species of [ Fe(H2O)6 ]2 and [ Fe(CN)6 ] BM) = n (n + 2)
respectively are 4.9 and 0. Co2+ (d 7 ) = t25geg2 3 3.9
11. cis-[Pt(Cl)2 (NH3 )2 ] is known as cis-platin. It is a σ-bonded
Fe2+ (d6 ) = t2g4eg2 4 4.9
organo-metallic compound and is used as an anti-tumor agent in
the treatment of cancer. Cr2+ (d 4 ) = t23geg1 4 4.9
H3 N Cl 5
II Mn 2+
(d ) = t23geg2 5 5.9
Pt
H3 N Cl V2+ (d 3 ) = t23geg0 3 3.9

12. Key Idea In presence of strong field ligands, ∆ 0 > p, for Therefore, Co 2+ and V 2+ contains same value of magnetic
fourth electron it is more energetically favourable to occupy moment (3.9 BM).
t2 g orbital with configuration t24geg0and form low spin complexes. 15. In K3 [Co(CN)6 ], Co have +3 oxidation state and electronic
The correct order of the spin only magnetic moment of metal ions configuration of Co3 + is [Ar ] 18 3d 6.
in the given low-spin complexes is V2+ > Cr 2+ > Ru 3+ > Fe2+ . 3d6 4s0 4p0
All the given complexes possess strong field ligands Co3+=
(CN, NH3 ). Hence, readily form low spin complexes.
Complex Oxidation Configuration Orbital No. of As, CN − is a strong field ligands so it pairs up the de− s
state splitting unpaired 3d6
electrons
∴ [Co(CN)6]3–= XX XX XX XX XX XX
eg Inner orbital
d2sp3 -hybridised
[V(CN)6]4– V
2+ 3
t2g eg
0
3 complex
(6e− pairs donated
t2g by 6 CN− ligands)
In an octahedral complex, the metal is at the centre of the
eg
octahedron and the ligands are at the six corners. The lobes of the
[Cr(NH3)6]2+ Cr2+ 4
t2g eg0 2
eg orbitals (dx2 − y2 and dz2 ) point along the axes x , y and z under
t2g
the influence of an octahedral field, the d- orbitals split as follow.
eg
dx2-y2, dz 2
[Ru(NH3)6]3+ Ru3+ t25g eg0 1
t2g

eg Average dxy, dyz, dxz


d-orbitals energy of
[Fe(CN)6]4– Fe2+ t26g eg0 0
in free ion the d-orbitals Splitting of d-orbitals
t2g in a spherical in an octahedral
crystal field crystal field
13. The magnetic moment of the magnitude 5.9 BM suggest the
presence of 5 unpaired electrons in Mn(II). This can be cross As the d-orbitals, i.e. dx2 − y2 and dz2 are vacant. Hence, these
verified by putting the value (5) of unpaired electrons in the both orbitals are directly facing the ligands in K3 [Co(CN)6 ].
formula, µ = n(n + 2) BM 16. Mn 2 (CO)10 is an organometallic compound due to the
Thus, the valence electronic configuration of Mn(II) in the presence of MnC bond. The metal-carbon bond in
complex is organometallic compounds possess both σ and-π character.
Mn2+= The MC σ bond is formed by the donation of lone pair of
electrons from the carbonyl carbon into a vacant orbital of the
3d 4s 4p 4d
metal. The MC π-bond is formed by the donation of pair of
Coordination Compounds 273

electrons from a filled d-orbital of metal into vacant Here, C2O24 − is a bidentate ligand,
antibonding π * orbital of CO. The M  L bonding creates a ¾

synergic effect which strengthens the bond between CO and C¾O ¾

¾
the metal. M (Metal)
C¾O ¾
The structure of Mn 2(CO)10 is shown below : O ¾
¾
CO CO
CO
and H2 O is a monodentate ligand, H2O M
CO
CO Mn Mn CO So, total number of sites offered by C2O24 −
and H2O ligands
CO CO CO around Th(IV) = Coordination number of Th (IV)
CO
= 4 × 2(by C2O24 −) + 2 × 1(by H2O) = 10
17. The structure of Co2 (CO)8 (a polynuclear metal carbonyl) can be
written as: 19. (A) Co is present in vitamin B12 (iii) having molecular formula,
O II
C63H88 CoN14O14P.
OC C CO (B) Zn is present in carbonic anhydrase (iv) in which three
OC——Co——–Co——CO histidine units and the —OH group coordinate with one Zn
(II) ion.
OC C CO
(C) Rh is present in Wilkinson catalyst (i) having molecular
O formula [(Ph 3P)3 RhCl] .
Total number of bridging CO ligands = 2 (D) Mg is present in chlorophyll (ii) having molecular formula
II
and the Co Co bond = 1 C55H70O6N4 Mg (chlorophyll-b).
18. Coordination number is defined as the total number of ligands to
which the metal is directly attached.

20. The difference in the number of unpaired electrons of different metal ions in their high spin and low spin octahedral complexes are given in
the table below :
Metal ion Number of e− in high spin complex ( n1 ) Number of e− in low spin complax ( n2 ) n2 − n1
Mn 2+
3d 4s 4p 3d 4s 4p 5 −1 = 4

4d 4d

n1=5 n2=1

Fe 2+
3d 4s 4p 3d 4s 4p 4−0 = 4

4d 4d

n1=4 n2=0
2+
Ni 3d 4s 4p

4d

n1=2
Ni 2+ does not form low spin octahedral complexes.
Co2+ 3d 4s 4p 3d 4s 4p 3 −1 = 2

4d 4d

∴ n1=3 ∴ n1=1

•• 
21. According to the situation given in question, reactions are as CH2 — N H2
follows:
III III
en = ethylene-1, 2-diamine,  
CoCl3 + 2en → [Co(en)2Cl2]Cl CH2 — NH2 
•• 
274 Coordination Compounds

Cl ∆(CFSE) is measured with help of wavelength of the colour


en
absorbed by the given coordination compound, as
Cl
c
III ∆ O = hν = h ×
Co Cl en Co en Cl λ
Both the complexes contain three unpaired electrons. Therefore,
Cl
en both are paramagnetic.
Cl
‘A’ Optically active ‘B’ Optically inactive 27. Fluoride ions help in making teeth enamel harder by converting
(cis-form) (trans-form) [3Ca 3 (PO4 )2 ⋅ Ca(OH)2 ] i.e. Hydroxy apatite to
(Violet) (Green) [3Ca 3 (PO4 )2 ⋅ CaF2 ] i.e., Fluorapatite (Harder teeth enamel) via
following reaction:
22. Wilkinson’s catalyst is a σ-bonded organometallic compound [3Ca 3 (PO4 ) 2 ⋅ Ca(OH) 2 ] + 2F − → [3Ca 3 (PO4 ) 2 ⋅ CaF2 ] + 2OH−
From
[(Ph 3P)3 RhCl] . It is commercially used for hydrogenation of drinking
water
alkenes and vegetable oils (unsaturated).
IUPAC name Chloridotris (triphenylphosphene) rhodium (I). Number of moles of solute
28. Molarity (M) =
Volume of solution (in L)
23. In homoleptic complexes, the metal atom/ion is linked to only
∴ Number of moles of complex
one type of ligand. Assuming, ligands are neutral, the octahedral
complexes of M 3+ can be, Molarity × volume (in mL) 01
. × 100
= = = 0.01 mole
[ M (L1 )6 ]3+ ,[ M (L2 )6 ]3+ and [ M (L3 )6 ]3+ 1000 1000

(I) (II) (III) . × 1022


12
Number of moles of ions precipitate = = 0.02 moles
λ Absorption Green Blue Red (wavelength) 6.02 × 1023
So, λ III > λLI1 > λLII2
L3
∴Number of Cl − present in ionisation sphere
1
∴∆°absorption : ∆LII2 > ∆LI1 > ∆LIII3 [QEnergy (∆, CFSE) ∝ ] Number of moles of ions precipitated 0.02
λ = = =2
Number of moles of complex 0.01
We know, ligand strength ∝ ∆°absorption
So, the increasing order of the ligand strength will be, ∴2 Cl − are present outside the square brackets, i.e. in ionisation
L3 < L1 < L2 sphere. Thus, the formula of complex is
[Co(H2 O) 5 Cl]Cl 2 ⋅H2 O.
24. All of the complex given are the octahedral complexes of Co (III) 29.
except K 2[CoCl 4], which is a tetrahedral complex of Co (II) Complex ion Electronic Number of unpaired
(sp3-hybridised). configuration of electrons (n)
 4  metal ion
We know, ∆t < ∆o Q ∆t = ∆o
 9  [Cr(H2O)6 ]2+ Cr 2+ ; [Ar] 3 d 4 ;4
So, the octahedral complexes (a, b, c) have higher ∆ o values than
that of tetrahedral, K2[CoCl 4 ]. [Fe(H2O)6 ]2+ Fe 2+ ; [Ar] 3 d6 ; 4
Now, for the complexes, a, b and c,
[Mn(H2O)6 ]2+ Mn2+ ; [Ar] 3 d 5 ;5
the magnitude of ∆ o ∝ ligand strength, which is based on their
positions in the spectrochemical series.
[CoCl 4 ]2− Co 2+ ; [Ar] 3 d 7 ;3
Cl − < H2O < NH3 < CN−
Hence, K3[Co(CN)6 ] will have the highest ∆ value. 30.
Compounds Hybridisation Unpaired Magnetic
25. The spin only magnetic moment (µ) (in BM) is given by
electron(s) character
µ(in BM) = n(n + 2) Ni(CO)4 sp3 No Diamagnetic
where, n = number of unpaired electrons [NiCl 4 ] 2−
sp 3
two Paramagnetic
The highest value of n in transition metal complex is 5 in its 3 2
[Co(NH3 )4 Cl 2 ]Cl sp d No Diamagnetic
d 5-configuration.
3 2
∴ µ = 5(5 + 2)BM = 5.916BM Na 3[CoF6 ] sp d three Paramagnetic
Na 2O2 — No Diamagnetic (O22 − )
26. ‘A’ absorbs yellow light of less energy and emits violet light of CsO2 — One Paramagnetic
high energy (complementary colour) because H2O is a weak
O−2 (superoxide
field ligand. But in case of ‘B’, due to presence of strong field
ligand (NH3 ), it absorbs high energy violet light and emits low ion is
energy complementary yellow colour. paramagnetic)
Coordination Compounds 275

31. KMnO4 → K + + MnO4−

number of

Unpaired
electrons
Complex

Atomic
∴ In MnO−4 , Mn has + 7 oxidation state having no electron in

Magnetic
O.N.

E.C.

moment
d-orbitals.
It is considered that higher the oxidation state of metal, greater is
the tendency to occur L →M charge transfer, because ligand is
P : [FeF6 ]3 − 26 +3 [Ar]3d 5 5 35 BM
able to donate the electron into the vacant d-orbital of metal.
weak ligand
Since, charge transfer is laporate as well as spin allowed,
therefore, it shows colour. Q : [V(H2O)6 ]2+ 23 +2 [Ar] 3 15 BM
Time saving Technique There is no need to check all the four weak ligand
options. Just find out the oxidation state of metal ion. If oxidation R :[Fe(H2O)6 ]2+ 26 +2 [Ar]3d6 4 24 BM
state is highest and ligand present there is of electron donating
nature, gives LMCT, which shows more intense colour. Thus, order of spin-only magnetic moment = Q < R < P
32. This problem is based on conceptual mixing of properties of 35. In the given complex, NiCl 2 {P (C2H5 )2 (C6H5 )}2 nickel is in
lithium oxide and preparation, properties of coordination + 2 oxidation state and the ground state electronic configuration
compounds. To answer this question, keep in mind that on of Ni 2+ ions in free gaseous state is
adding acid, ammine complexes get destroyed.
3d 8 4s0 4p0
(a) Li 2O + KCl → 2LiCl + K2O
2+
This is wrong equation, since a stronger base K2O cannot be Ni
generated by a weaker base Li 2O.
For the given four coordinated complex to be paramagnetic, it
(b) [CoCl(NH3 )5 ]+ + 5H+ → Co2+ ( aq) + 5 NH4+ + Cl − must possess unpaired electrons in the valence shell. To satisfy
This is correct. All ammine complexes can be destroyed by this condition, four lone pairs from the four ligands occupies the
adding H ⊕ . Hence, on adding acid to [CoCl(NH3 )5 ], it gets four sp3-hybrid orbitals as :
converted to Co2+ ( aq)+ NH+4 and Cl − . Ni2+

:
:
:
OH−
(c) [Mg (H2O)6 ]2+ + EDTA4− → [ Mg( EDTA)]2+ + 6H2O sp3
Excess
Therefore, geometry of paramagnetic complex must be
This is wrong, since the formula of complex must be
tetrahedral. On the otherhand, for complex to be diamagnetic,
[Mg(EDTA)]2+ as EDT.
there should not be any unpaired electrons in the valence shell.
(d) The 4th reaction is incorrect. It can be correctly
This condition can be fulfilled by pairing electrons of
represented as
3d-orbitals against Hund’s rule as
2CuSO4 + 10KCN → 2K3[Cu(CN)4 ]
+ 2K2SO4 + (CN)2 ↑ Ni2+
:

:
:
33. Arrange the complex formed by different ligands L 1 , L 2 , L 3 and 3d 8 dsp2
L4, according to wavelength of their absorbed light, then use of The above electronic arrangement gives dsp2-hybridisation and
the following relation to answer the question.
therefore, square planar geometry to the complex.
Ligand field strength ∝ Energy of light absorbed
1 36. For a diamagnetic complex, there should not be any unpaired
∝ electron in the valence shell of central metal.
Wavelength of light absorbed
In K3[Fe(CN)6], Fe (III) has d 5-configuration (odd electrons),
λ L1 L2 L3 L4 hence it is paramagnetic.
Absorbed light Red Green Yellow Blue
In [Co(NH3)6]Cl3, Co (III) has d 6-configuration in a strong
Wavelength of absorbed light decreases. ligand field, hence all the electrons are paired and the complex is
∴ Increasing order of energy of wavelengths absorbed reflect diamagnetic.
greater extent of crystal field splitting, hence, higher field In Na3[Co(ox)3], Co (III) has d 6-configuration and oxalate being
strength of the ligand. a chelating ligand, very strong ligand and all the six electrons
Energy blue (L4 ) > green (L 2 ) > yellow (L 3 ) > red (L 1 ) remains paired in lower t2g level, diamagnetic.
∴ L4 > L2 > L3 > L1 in field strength of ligands. In [Ni(H2O)6]Cl2, Ni (II) has 3d 8 -configuration and H2O is a
34. PLAN Spin only magnetic moment have the formula n( n + 2) BM, weak ligand, hence
where N is the number of unpaired electrons. In the presence of 3d
weak ligand (as H 2O, Cl − , F − ) there is no pairing of electrons,
and electrons donated by ligands are filled in outer vacant
orbitals. Paramagnetic
:

:
:
:

:
:

In the presence of strong ligand (as CN− , CO, NH3 , en) electrons
sp3d 2
are paired and electrons from ligands are filled in available inner
orbitals
276 Coordination Compounds

In K2[Pt(CN)4], Pt(II) has d 8 -configuration and CN– is a strong NiCl 2 (PPh 3 )2 , Ni 2+ has 3d 8 -configuration. Due to weak ligand
ligand, hence all the eight electrons are spin paired . Therefore, field, Ni is sp3-hybridised and complex is tetrahedral.
complex is diamagnetic.
In [Zn(H2O)6](NO3)2, Zn (II) has 3d 10configuration with all the 48. Cu 2+ + CN− → CuCN ↓
ten electrons spin paired, hence diamagnetic. CuCN + 3CN− → [Cu(CN)4 ]3 −

37. Magnetic moment = 2.83 BM indicates that there is two unpaired 49. Fe in [Fe(H2O)6 ]2+ has maximum (four) unpaired electrons, has
electrons. highest paramagnetism.
u = n(n + 2) BM = 8 BM = 2.82 BM 50. In Ni (CO)4, Ni has 3d 10-configuration, diamagnetic. In
Ni (CN)4 ]2− , Ni has 3d 8 -configuration but due to strong ligand
2− 8 − field, all the d-electrons are spin paired giving
In [NiCl 4 ] , Ni has d configuration and Cl is a weak ligand :
dsp2-hybridisation, diamagnetic.
3d
2+ In [NiCl 4 ]2− , Ni has 3d 8 -configuration and there is two unpaired
Ni
:

:
:
4s
: 4p electrons (weak chloride ligand do not pair up d - electrons),
3
hence paramagnetic.
sp -hybridisation
6
51. Salt with least number of unpaired electrons in d - orbital of
38. In Cr(CO)6 : 3d , has no unpaired electrons, zero magnetic central metal will show lowest degree of paramagnetism
moment.
Mn 2+ (3d 5, 5 unpaired electrons)
39. CuF2 : Cu 2+ has 3d 9 -configuration, allowed d-d transition, Cu 2+ (3d 9 , 1 unpaired electrons)
hence, coloured. Fe2+ (3d6, 4 unpaired electrons)
40. In Ni(CO)4, Ni is sp3-hybridised while in [Ni(CN)4 ] 2− , Ni 2+ is Ni 2+ (3d 8 , 2 unpaired electrons)
dsp2-hybridised. Hence, CuSO4 ⋅ 5H2O has lowest degree of paramagnetism.
52. (a) In [FeCl 4 ]− , oxidation number of Fe atom = + 3
41. Greater the extent of dπ - pπ back bonding, smaller will be the
Electronic configuration of Fe in ground state = 3d6 4 s2
bond order of CO bond in metal carbonyls. In Fe(CO)5, there is
maximum number of valence shell electrons (d-electrons), Electronic configuration of Fe3+ = 3d 5 4 s0 4 p0
greatest chances of pπ - dπ back bonding, lowest bond order of
CO bond.
42. In CO, bond order = 3. In metal carbonyls like Fe(CO)5, due to Cl Cl Cl Cl
sp 3-hybridisation
dπ - pπ back-bonding, bond order of CO decreases slightly
therefore, bond length increases slightly. Thus, [FeCl 4] − has tetrahedral geometry.
43. In Hg [Co(SCN)4 ], Co2+ has 3d 7 configuration. SCN − produces (b) [Co(en)(NH3 )2 Cl 2 ]+ have three geometrical isomers. Thus,
weak ligand field, no pairing of electrons in d-orbitals occurs statement (b) is incorrect.
against Hund’s rule, hence : Cl NH3 NH3
NH3 Cl Cl
Co2+ : µ = 3 (3 + 2) BM = 15 BM en Co3+ en Co3+ en Co3+
3d7 Cl NH3
NH3
44. [NiCl 4 ]2 − : Ni 2+ (3d 8 ) Cl NH3 Cl
Tetrahedral
sp3-hybridisation (c) Fe 3+ in [FeCl 4 ]− is sp3-hybridised with 5 unpaired electrons.
(higher spin-only magnetic moment
= n(n + 2 ) = 5.92 BM ). While Co3+ in
[Co(en)(NH3 )2 Cl 2 ]+ is d 2sp3-hybridised with zero unpaired
3d 8 4s
electrons (low spin-only magnetic moment
Under influence of weak ligand field
= n(n + 2 ) = 0 BM).
In all other complexes, hybridisation at central metal is dsp2 and Thus, the statement (c) is correct.
complexes have square planar geometries.
45. In 1 L solution, there will be 0.01 mole of each [Co(NH3 )5 SO4 ]
(d) Co3+ [Co(en)(NH3 )2 Cl 2 ]+
Br and [Co(NH3 )5 Br]SO4. Addition of excess of AgNO3 will
give 0.01 mole of AgBr. Addition of excess of BaCl 2 will give
0.01 mole of BaSO4. eg
46. In MnO−4 , Mn + 7 has 3d 0 configuration. Co3+ : [Ar]3d6 ∆0>P
10
47. In Ni (CO)4, Ni is in 3d state due to strong ligand field produced
t2g
by CO. Hence, Ni is sp3-hybridised and complex is tetrahedral. In
Coordination Compounds 277

Co3+ in [Co(en)(NH3 )2 Cl 2 ]+ is d 2sp3-hybridised and has 55. Statement wise explanation is


octahedral geometry with 0 unpaired electron. Thus, Statement (a) [Co(en)(NH3 )3 H2O ]3+ have following 2
statement (d) is incorrect. geometrical isomers.
53. NH3 NH3
Sn
– – NH3 H2O
SnCl2 + Cl SnCl3 Cl Cl
Tin chloride (X ) Cl en Co en Co
(Q) sp3 (pyramidal)
NH3 NH3
SnCl −3 has (3σ + 1lp) and exist in pyramidal structure. H2O NH3
Me Cl Fac Mer
SnCl2+Me3N SnCl2 × NMe3 Me N Sn [ M ( AA )b3 c] type complex
(3°amine) Me Cl
(Y ) Hence, this is correct statement.
(Q)
Statement (b) If bidentate ligand ‘en’ is replaced by two
Y complex has coordinate bond in between nitrogen and Sn metal.
cyanide ligands then [Co(NH3 )3 (H2O)(CN)2 ]+ is formed.
Reduction
It is [ Ma3 b2 c ] type complex which has following 3 geometrical
+2 +2 +4 +1
isomers.
SnCl2 + 2CuCl2 SnCl4 + 2CuCl NH3 NH3
(Q) (Z )
N C NH3 N C NH3
Oxidation
Co Co
Z is oxidised product and oxidation state of Sn is +4 in Z N C NH3 H2O NH3
compound. Structure of SnCl 4 (Z ) is H2O CN
Cl (Fac) Fac with respect to NH3 and
Mer with respect to —CN
Sn NH3
Cl
Cl Cl N C H2O
Co
Thus, options (a, d) are correct.
N C NH3
54. Statement wise explanation is NH3
(i) Statement (a) The total number of valence shell electrons at (Mer)
metal centre in Fe(CO)5 or Ni(CO)4 is 8 instead of 16 as Hence, this statement is also correct.
shown below
Statement (c) Co metal has [ Ar ]3d 7 4 s2 configuration while in
Valence shell having 8 electrons
in complex formation [Co(en)(NH3 )3 (H2O )]3+ it is in +3 oxidation state. Thus, Co 3+
has [Ar]3d6 configuration.
Fe(3d6,4s2) Ni(3d8,4s2)
3d 4s
3d 4s 4p 3d 4s 4p
Co3+ =

[Fe(CO)5] [Ni(CO)4]
As en is a strong ligand, so pairing will occur
3d 4s

Rearrangement Rearrangement
dsp3 sp3
Due to the presence of all paired electrons it show diamagnetic
behaviour rather than paramagnetic.
Hence, this statement is incorrect.
(ii) Statement (b) Carbonyl complexes are predominantly low Hence, this statement is incorrect.
spin complexes due to strong ligand fields. Hence, this Statement (d) According to CFT, absorption of light by
statement is correct. coordination complexes depends upon CFSE i.e., crystal field
(iii) Statement (c) For central metal lowering of oxidation state splitting energy (∆ 0 )as
results to increase in electron density on it. This in turn 1
∆0 ∝
results to increase in extent of synergic bonding. Thus, we λ
can say ‘‘metal carbonyl bond strengthens, when oxidation Among the complexes given [Co (en) (NH3)4 ]3+ has more ∆ 0
state of metal is lowered’’.
value as compared to complex [Co(en) (NH3)3(H2O) ]3+ . Thus,
Hence, it is a correct statement.
(iv) Statement (d) Increase in positive charge on metal (i.e., [Co (en) (NH3)3(H2O) ]3+ absorbs the light at longer wavelength
increase in oxidation state) results to decrease in synergic for d-d transition.
bonding strength. Hence, this statement is also correct.
This in turn makes C—O bond stronger instead of weaker. Note : For any complex, the value of ∆ 0 can be calculated via the
Hence this statement is also incorrect. difference or gap between eg and t2g values.
278 Coordination Compounds

Excess NH 4 OH / NH 4 Cl
56. [Co(H2O) 6 ] Cl 2 → Co(NH3 ) 6 ]Cl 3 61. For, P i.e. dsp2, It is seen in [Ni(CN)4 ]2 −
O 2 (Air)
Pink (X) Y Ni [Ar]3d 8 4 s2
2+ − 2−
[Co(H2 O) 6 ] + 4Cl → [CoCl 4 ] Ni2+ [Ar]3d 8
X (Excess) blue Z
3d 4s 4p
(a) Since NH3 is moderately strong ligand, hybridisation of
Ni2+
cobalt in Y is d 2sp3.
(b) Cobalt is sp3-hybridised in [CoCl 4 ]2− . as CN − is a strong ligand so when it approaches towards central
(c) [Co(NH 3 ) 6 ]Cl 3 + 3AgNO3 (aq) → 3AgCl ↓ metal pairing of unpaired electrons takes place.
Y Thus, in[Ni(CN)4 ]2 −
(d) [CoCl 4 ]2− + 6H2O q [Co(H2O)6 ] 2+ + 4Cl − ; ∆H < 0 dsp2-hybridisation
Blue Pink
3d 4s 4p
57. The structure of cis-[ Mn (en )2 Cl 2 ] complex is Ni2+
Cl (a) Cl (a)
Cl N1 Cl (b) CN– CN– CN– CN–
(b )
Mn Mn Structure : Square planar
en en N2 N4 So correct match for P is 6.
N
3 For Q i.e., sp3
Bond angles (Mn—N and Mn—Cl bond in cis positions) It is seen in [FeCl4 ]2 − and Ni(CO)4
Cl (a) —— Mn —— N(1) Fe − [Ar]3d6 4 s2
Cl (a) —— Mn —— N(2)
Fe 2+ − [Ar]3d6
Cl (a) —— Mn —— N(4)
Cl (b) —— Mn —— N1 3d 4s 4p
Fe2+
Cl (b) —— Mn —— N3
Cl (b) —— Mn —— N4 As Cl − is a weak ligand so when it approaches towards central
Number of cis Cl—Mn—N = 6 metal pairing of unpaired electrons does not take place.
58. In the complex [Fe(H2O)5 NO]SO4, Fe is in +1 oxidation state Thus, in[FeCl4 ]2 −
because NO is in +1 state. Also NO is a strong ligand, complex sp3 hybridisation
has 3d 7 -configuration at Fe(I) as :
3d 4s 4p
Fe2+
Cl– Cl– Cl– Cl–

Structure-Tetrahedral
3d7 Strong ligand field
Likewise in Ni(CO)4
Ni [Ar]3d 8 4 s2
3d 4s 4p
Three unpaired electrons
Ni

59. A is diamagnetic, square planar complex because of strong As CO is a strong ligand, hence when it approaches towards
ligand field of CN− . central metal atom pairing of unpaired electron of central atom
takes place.
Ni(CN)42 –(Ni2+) : Thus, in Ni(CO)4
dsp2 sp3 hybridisation
Diamagnetic 3d 4s 4p
B is paramagnetic, tetrahedral complex because of weak ligand Ni
field of Cl − . CO CO CO CO

NiCl42 –(Ni2+) : Structure Tetrahedral


3 So, for Q-4 and 5 are correct match.
sp
Paramagnetic For R i.e., sp3d 2
60. Described in 2, A has dsp2 hybridisation while B has It is seen in [FeF6]4−
sp3-hybridisation of Ni. Fe [Ar]3d6 4 s2
Fe 2+ [Ar]3d6
Coordination Compounds 279

3d 4s 4p 4d Coordination compounds of [MA4 B2 ] type show geometrical


Fe2+ isomerism. Molecular orbital electronic configuration (MOEC)
for various coordination compound can be drawn using VBT as
As F − is a weak field ligand hence, when it approaches towards A. MO EC for [Cr(NH3 )4 Cl 2 ]Cl is
central metal atom, pairing of its electrons does not take place.
3d 4s 4p
Thus, in[FeF6 ]4 − ×× ×× ×× ×× ×× ××
sp3d2 hybridisation NH3NH3 NH3 NH3Cl Cl
3d 4s 4p 4d Number of unpaired electrons (n) = 3
Fe2+
Magnetic properties = paramagnetic
F– F– F– F– F– F– Geometrical isomers of [Cr(NH3 )4 Cl 2 ]+ are
Structure : Octahedral Cl Cl
So, 1 is the correct match for R. + +
H 3N Cl H3 N NH3
For S i.e., d 2 sp 3
Cr and Cr
It is seen in [Ti(H2O)3 Cl3 ] and[Cr(NH3 )6 ]3+ H 3N NH3 H3N NH3
2 2
Ti [Ar]3d 4 s NH3 Cl
Ti3+ [Ar]3d 1 Cis Trans
3d 4s 4p B. n=1
Ti3+ Magnetic properties = paramagnetic
Ionisation isomers of [Ti(H2O)5 Cl](NO3 )2 are
Here, both H2O and Cl are weak ligands
[Ti(H2O)5 Cl](NO3 )2 and [Ti(H2O)5 (NO3 )]Cl(NO3 )
So, in[Ti(H2O)3 Cl3 ]
C. MOEC of [Pt(en)(NH3 )Cl]NO3 is
d2sp3 hybridisation
3d 4s 4p ×× ×× ×× ××
××
Ti3+ en en NH3 Cl
H2O H2O H2O Cl– Cl– Cl– n=0
Magnetic property = diamagnetic
Structure Octahedral
Ionisation isomers are [Pt(en)2 (NH3 )Cl]NO3
Likewise in [Cr(NH3)6]3+
and [Pt(en)2 NH3 (NO3 )]Cl
Cr [Ar]3d 5 4 s1
D. MOEC of [Co(NH3 )4 (NO3 )2 ]NO3
Cr 3+ [Ar]3d 3 4 s0
×× ×× ×× ×× ×× ××
3d 4s 4p
NH3NH3 NH3 NH3NO3NO3
Cr3+
n=0
Here, NH 3 is also a weak field ligand so due to its approach no Magnetic property = Diamagnetic
pairing takes place in Cr.
Geometrical isomers are
Thus, In[Cr(NH3 )6 ]3+
NO3 NH3
d2sp3 hybridisation
H3N NH3 NO3 NH3
3d 4s 4p CO and CO
Cr3+– H3N NH3 NO3 NH3

NH3 NH3 NH3 NH3 NH3 NH3 NO3 NH3


Trans Cis
So for, S-2 and 3 are the correct match.
Thus, magnetic property and isomerism in given coordination
62. PLAN This problem is based on concept of VBT and magnetic compound can be summarised as
properties of coordination compound. (P) [Cr(NH3 )4 Cl 2 ]Cl → Paramagnetic and exhibits cis-trans
Draw VBT for each coordination compound. isomerism (3)
If unpaired electron is present then coordination compound will (Q) [Ti(H2O)5 Cl](NO3 )2 → Paramagnetic and exhibits
be paramagnetic otherwise diamagnetic. ionisation isomerism (1)
280 Coordination Compounds

(R) [Pt(en)(NH3 )Cl]NO3 → Diamagnetic and exhibits 3d 4s 4p


ionisation isomerism (4)
(S) [Co(NH3 )4 (NO3 )2 ]NO3 → Diamagnetic and exhibits d2sp3
cis-trans isomerism (2)
∴ P → 3, Q → 1, R → 4, S → 2 Spin only magnetic moment (µ s ) = 1 (1 + 2) BM
Hence, (b) is the correct choice. = 3 BM
63. (A) [Co(NH3 )4 (H2O)2 ]Cl 2:Co2+ , 3d 7 Hence, difference in spin only magnetic moment
show geometrical isomerism, paramagnetic. = 35 − 3
(B) Pt(NH3 )2 Cl 2:Pt 2+ has d 8 -configuration with all paired ≈ 4 BM
electrons. Show geometrical isomerism, diamagnetic. 68. C O
(C) [Co(H2O)5 Cl]Cl : Co2+ , 3d 7 Br O

==
Cannot show geometrical isomerism, paramagnetic. (II)
(D) [Ni(H2O)6 ]Cl 2: Ni 2+ , 3d 8 , weak ligand, has two unpaired Et3P Fe C  CH 3
electrons. Paramagnetic but cannot show geometrical acetylbromidodicarbonylbis
isomerism. Et3P (triethylphosphine)iron (II)
C O
64. Complex part is cationic, named first : [Co(NH3 )6 ]Cl 3 :
hexaammine cobalt (III) chloride. 69. PLAN EDTA is a multidentate ligand as it can donate six pairs of

electrons – two pair from the two nitrogen atoms and four pair
65. False : Cyanide (CN ) is a strong ligand, brings about pairing of from the four terminal oxygens of the COO − groups.
3d electrons. –
OOCCH2 CH COO – 2
N—CH2CH2—N

OOCCH2 CH2COO –
Potassium ferrocyanide
The structure of a chelate of a divalent Co2+ with EDTA is
shown as
H2C H2C CH2
CH2
No unpaired electron, diamagnetic H 2C N N
CO CO

CH2
CO 1
4
Potassium ferrocyanide O Co O
2
3 CO
O O
Each N has four N  Co  O bonds thus total eight N  Co  O
bonds.
Has one unpaired electron, paramagnetic 70. H H
O O
66. False : Lobes of 3dx2 − y2 orbitals lies in X Y plane on the X and Y
H3C N N CH 3
coordinate axes, therefore electron density of dx2 − y2 orbital in C C
X Y plane is non-zero.
Ni
67. When S is donor atom of SCN−, it produces weak ligand field C C
H3C N N CH 3
and forms high spin complex as
3− 3+ 5
[ Fe(SCN)6 ] : Fe (3d ) = O O
H H
4s 4p 4d
Ni-DMG complex

3d 5(n-5) sp3d2 Oxidation state of Ni is +2 and hybridisation is dsp2.


Spin only magnetic moment (µ s ) = 5 (5 + 2) BM = 35 BM µ = 0 (no unpaired electron) hence, diamagnetic.
In case of CN− ligand, carbon is the donor atom , it produces 71. The spin-only magnetic moment (µ) of the complex is 1.73 BM.
strong ligand field and forms low spin complex as It indicates that nucleus of complex, chromium ion has one
3− 3+ 5 unpaired electron. So, the ligand NO is unit positively charged.
[Fe(CN)6 ] : Fe (3d )
Coordination Compounds 281

K2[Cr(NO) (CN)4 (NH3 )] In both A and B, hybridisation of chromium is d 2sp3 and


potassium amminetetracyanonitrosoniumchromate (I) magnetic moment : µ = n (n + 2) BM = 0
Cr+1 : (3d6, strong ligand, no unpaired electron)
3d 5 4s0 74. NH3 3+ 2–
Cr(I) “under influence of strong ligand field”. H3N NH3 NC CN

Co Ni

d 2sp3 H3N NH3 NC CN


Octahedral NH3
NO d 2sp3-octahedral dsp2-square planar
2–
NC CN
CO
Cr
Ni
CO
NC NH3
OC CO
CN sp3-tetrahedral
Octahedral geometry
75. A has no water molecules of crystallisation.
72. [NiCl 4 ] 2 − : Ni 2+ (3d 8 ), weak ligand field.
Hence, A is [Cr(H2O)6 ]Cl 3.
Ni2+(3d 8) : Both B and C loses weight with concentrated H2SO4, therefore,
both B and C have some water molecules of crystallisation.
sp3 Moreover, weight loss with C is just double of the same with B
µ = n (n + 2) ΒΜ = 8 ΒM indicates that number of water molecules of crystallisation of C
[Ni(CN)4 ]2 − : Ni 2+ (3d 8 ) , strong ligand field. is double of the same for B. Therefore, B has one and C has two
water molecules of crystallisation.
Ni2+(3d 8) : B = [Cr(H2O)5 Cl]Cl 2 ⋅ H2O, C = [Cr(H2O)4 Cl 2 ]Cl ⋅ 2H2O
dsp2 76. (i) [Ti(NO3 )4 ]: Ti 4+ (3d 0 ) No d-electron, no d-d transition
Square planar possible, colourless.
µ = 0 (no unpaired electron) (ii) [Cu(NCCH3 )]BF4 : Cu + (3d 10 ) All d-orbitals are completely
73. In complexes A and B, one halide (Cl − or Br − ) is outside filled, no d-d transition possible, colourless.
coordination sphere, i.e. complexes are : (iii) [Cr(NH3 )6 ]Cl 3 : Cr 3+ (3d 3 ) Complex has allowed
[Cr(NH3 )4 Br2 ]Cl and [Cr(NH3 )4 BrCl]Br d-d-transitions from t2g to eg level, hence coloured.
A gives white precipitate AgCl with excess of AgNO3 which (iv) K3[VF6 ]: V3+ (3d 2 ) Complex has allowed d-d-transitions
dissolve in excess ammonia. Therefore, A must be
from t2g to eg level, hence coloured.
[Cr(NH3 )4 Br2 ]Cl.
B gives a pale yellow precipitate with excess of AgNO3, which 77. I− is a strong reducing agent, reduces Cu 2+ to Cu + and
dissolve in concentrated ammonia solution. Therefore, precipitate out as stable CuI.
precipitate is AgBr and complex B is [Cr(NH3 )4 ClBr]Br.
19
Extraction of Metals
Objective Questions I (Only one correct option) 7. Match the refining methods Column I with metals Column II.
1. The processes of calcination and roasting in metallurgical Column I Column II
industries, respectively, can lead to (2020 Main, 4 Sep II) (Refining Methods) (Metals)
(a) global warming and acid rain
(b) global warming and photochemical smog I. Liquation (A) Zr
(c) photochemical smog and ozone layer depletion II. Zone refining (B) Ni
(d) photochemical smog and global warming
III. Mond process (C) Sn
2. An Ellingham diagram provides information about
(2020 Main, 5 Sep I) IV. van Arkel method (D) Ga
(a) the kinetics of the reduction process.
(b) the pressure dependence of the standard electrode potentials (a) I- (C) ; II-(D); III-(B) ; IV-(A) (2019 Main, 10 April I)
of reduction reactions involved in the extraction of metals. (b) I- (B) ; II-(C); III-(D) ; IV-(A)
(c) the temperature dependence of the standard Gibbs energies (c) I- (C) ; II-(A); III-(B) ; IV-(D)
of formation of some metal oxides. (d) I- (B) ; II-(D); III-(A) ; IV-(C)
(d) the conditions of pH and potential under which a species is 8. The one that is not a carbonate ore is (2019 Main, 9 April II)
thermodynamically stable. (a) siderite (b) calamine (c) malachite (d) bauxite
3. Calamine, malachite, magnetite and cryolite, respectively, 9. Assertion For the extraction of iron, haematite ore is used.
are (2019 Adv.) Reason Heamatite is a carbonate ore of iron.
(a) ZnCO3 , CuCO3 , Fe2 O3 , Na 3 AlF6 (2019 Main, 9 April II)
(a) Only the reason is correct.
(b) ZnSO4 , CuCO3 , Fe2 O3 , AlF3 (b) Both the assertion and reason are correct explanation for the
(c) ZnSO4 , Cu(OH)2 , Fe3 O4 , Na 3 AlF6 assertion.
(d) ZnCO3, CuCO3 ⋅ Cu(OH)2, Fe3O4, Na 3AlF6 (c) Both the assertion and reason are correct and the reason is the
4. The correct statement is correct explanation for the assertion.
(a) leaching of bauxite using concentrated NaOH solution (d) Only the assertion is correct.
gives sodium aluminate and sodium silicate. 10. The ore that contains the metal in the form of fluoride is
(2019 Main, 12 April II) (2019 Main, 9 April I)
(b) the hall-heroult process is used for the production of (a) magnetite (b) sphalerite (c) malachite (d) cryolite
aluminium and iron.
(c) pig iron is obtained from cast iron. 11. The Mond process is used for the (2019 Main, 8 April II)
(d) the blistered appearance of copper during the metallurgical (a) purification of Ni (b) extraction of Mo
process is due to the evolution of CO2. (c) purification of Zr and Ti (d) extraction of Zn
5. The idea of froth floatation method came from a person X 12. With respect to an ore, Ellingham diagram helps to predict the
and this method is related to the process Y of ores. X and Y , feasibility of its (2019 Main, 8 April I)
respectively, are (2019 Main, 12 April I) (a) electrolysis (b) zone refining
(a) fisher woman and concentration (c) vapour phase refining (d) thermal reduction
(b) washer woman and concentration
13. The pair that does not require calcination is
(c) fisher man and reduction (2019 Main, 12 Jan II)
(d) washer man and reduction (a) ZnO and MgO (b) ZnO and Fe2O3 ⋅ xH2O
6. The correct statement is (2019 Main, 10 April II) (c) ZnCO3 and CaO (d) Fe2O3 and CaCO3 ⋅ MgCO3
(a) zone refining process is used for the refining of titanium. 14. In the Hall-Heroult process, aluminium is formed at the cathode.
(b) zincite is a carbonate ore.
The cathode is made out of (2019 Main, 12 Jan I)
(c) sodium cyanide cannot be used in the metallurgy of silver.
(a) platinum (b) carbon
(d) aniline is a froth stabiliser.
(c) pure aluminium (d) copper
Extraction of Metals 283

15. The reaction that does not define calcination is 22. From the following statements regarding H2 O2 choose the
(a) Fe2O3 ⋅ XH2O → Fe2O3 + XH2O ∆ incorrect statement. (2015 Main)
(2019 Main, 11 Jan II)
(a) It can act only as an oxidising agent

(b) ZnCO3 → ZnO + CO2 (b) It decomposed on exposure to light
∆ (c) It has to be stored in plastic or wax lined glass bottles in dark
(c) CaCO3 ⋅ MgCO3 → CaO + MgO + 2CO2 (d) It has to be kept away from dust

(d) 2Cu2S + 3O2 → 2Cu2O + 2SO2 23. In the context of the Hall-Heroult process for the extraction of
16. Match the ores ( Column A ) with the metals (Column B). Al, which of the following statements is false?
(a) CO and CO2 are produced in this process (2015 Main)
Column A Column B (b) Al 2O3 is mixed with CaF2 which lowers the melting point of the
Ores Metals mixture and brings conductivity
(c) Al 3+ is reduced at the cathode to form Al
A. Siderite P. Zinc
(d) Na 3AlF6 serves as the electrolyte
B. Kaolinite Q. Copper
24. The metal that cannot be obtained by electrolysis of an
C. Malachite R. Iron
aqueous solution of its salts is (2014 Main)
D. Calamine S. Aluminium (a) Ag (b) Ca
(2019 Main, 11 Jan I) (c) Cu (d) Cr
(a) A - P; B- Q; C - R; D- S (b) A - R; B- S; C - P; D- Q
(c) A - Q; B- R; C - S; D- P (d) A - R; B- S; C - Q; D- P 25. Sulphide ores are common for the metals (2013 Adv.)
(a) Ag, Cu and Pb (b) Ag, Cu and Sn
17. The electrolytes usually used in the electroplating of gold and (c) Ag, Mg and Pb (d) Al, Cu and Pb
silver, respectively, are (2019 Main, 10 Jan II)
(a) [Au(OH) 4 ] − and [Ag(OH) 2 ] − (b) [Au(NH3 ) 2 ] + and [Ag(CN) 2 ] − 26. In the cyanide extraction process of silver from argentite ore,

(c) [Au(CN) 2 ] and [Ag(CN) 2 ] − −
(d) [Au(CN) 2 ] and [AgCl 2 ] − the oxidising and reducing agents used are (2012)
(a) O2 and CO respectively
18. Hall-Heroult’s process is given by (2019 Main, 10 Jan I) (b) O2 and Zn dust respectively
Coke, 1673 K (c) HNO3 and Zn dust respectively
(a) ZnO + C → Zn + CO
(d) HNO3 and CO respectively
(b) Cr2O3 + 2Al → Al 2O3 + 2Cr
(c) 2Al 2O3 + 3C → 4Al + 3CO2 27. Oxidation states of the metal in the minerals haematite and
magnetite, respectively, are (2011)
(d) Cu2+ (aq) + H2 (g ) → Cu(s) + 2H+ (aq)
(a) II, III in haematite and III in magnetite
19. The correct statement regarding the given Ellingham diagram (b) II, III in haematite and II in magnetite
is (2019 Main, 9 Jan II) (c) II in haematite and II, III in magnetite
(d) III in haematite and II, III in magnetite
O
→ 2Cu 2
+O 2 28. Native silver metal forms a water soluble complex with a
4Cu
dilute aqueous solution of NaCN in the presence of
–300 (a) nitrogen (b) oxygen (2008, 3M)
(c) carbon dioxide (d) argon
∆Gº (kJ/mol)

2C
+O
→ 2Z
nO 2 →2
CO
29. Extraction of zinc from zinc blende is achieved by
–600 O2
2Zn+ (a) electrolytic reduction (2007, 3M)
O3 (b) roasting followed by reduction with carbon
3 Al 2
→ 2/ (c) roasting followed by reduction with another metal
l+O 2
4/3 A (d) roasting followed by self-reduction
–1050
500ºC 800ºC 2000ºC 30. Which ore contains both iron and copper? (2005, 1M)
Temperature (ºC) (a) Cuprite (b) Chalcocite
(a) At 800°C, Cu can be used for the extraction of Zn from ZnO (c) Chalcopyrite (d) Malachite
(b) At 1400°C, Al can be used for the extraction of Zn from ZnO 31. The methods chiefly used for the extraction of lead and tin
(c) At 500°C, coke can be used for the extraction of Zn from ZnO from their ores are respectively (2004, 1M)
(d) Coke cannot be used for the extraction of Cu from Cu 2O (a) self-reduction and carbon reduction
20. The ore that contains both iron and copper is (b) self-reduction and electrolytic reduction
(2019 Main, 9 Jan I) (c) carbon reduction and self-reduction
(a) malachite (b) azurite (d) cyanide process and carbon reduction
(c) dolomite (d) copper pyrites
32. In the process of extraction of gold,
21. Which one of the following ores is best concentrated by froth
O2
floatation method? (2016 Main) Roasted gold ore + CN– + H2O → [ X ] + HO
(a) Siderite (b) Galena
[ X ] + Zn → [Y ] + Au
(c) Malachite (d) Magnetite
284 Extraction of Metals

Identify the complexes [X] and [Y]. (2003, 1M) 42. Type of bonds present in CuSO4 ⋅ 5H2O are only (1983)
(a) X = [Au(CN)2 ] − , Y = [Zn(CN)4 ] 2− (a) electrovalent and covalent
(b) electrovalent and coordinate covalent
(b) X = [Au(CN)4 ] 3 – ,Y = [Zn (CN)4 ] 2–
(c) electrovalent, covalent and coordinate covalent
(c) X = [Au(CN)2 ] − , Y = [Zn(CN)6 ] 4− (d) covalent and coordinate covalent
(d) X = [Au(CN)4 ] − , Y = [ Zn(CN)4 ] 2− 43. In metallurgy of iron, when limestone is added to the blast
33. Anhydrous ferric chloride is prepared by (2002) furnace, the calcium ion ends up in (1982)
(a) heating hydrated ferric chloride at a high temperature (a) slag (b) gangue
in a stream of air (c) metallic calcium (d) calcium carbonate
(b) heating metallic iron in a stream of dry chlorine gas 44. Iron is rendered passive by treatment with concentrated
(c) reaction of ferric oxide with hydrochloric acid (a) H2SO4 (b) H3PO4 (1982)
(d) reaction of metallic iron with hydrochloric acid (c) HCl (d) HNO3
34. Which of the following process is used in extractive
metallurgy of magnesium? (2002, 3M) Objective Questions II
(a) Fused salt electrolysis (One or more than one correct option)
(b) Self-reduction
(c) Aqueous solution electrolysis 45. Which among the following statement(s) is(are) true for the
(d) Thermite reduction extraction of aluminium from bauxite? (2020 Adv.)
(a) Hydrated Al 2O3 precipitates, when CO2 is bubbled through a
35. The chemical composition of ‘slag’ formed during the solution of sodium aluminate.
smelting process in the extraction of copper is (2001, 1M) (b) Addition of Na 3AlF6 lowers the melting point of alumina.
(a) Cu2O + FeS (b) FeSiO3 (c) CO2 is evolved at the anode during electrolysis.
(c) CuFeS2 (d) Cu2S + FeO (d) The cathode is a steel vessel with a lining of carbon.
36. Electrolytic reduction of alumina to aluminium by 46. Extraction of copper from copper pyrite (CuFeS 2 ) involves
Hall-Heroult process is carried out (2000, 1M) (2016 Adv.)
(a) in the presence of NaCl (a) crushing followed by concentration of the ore by froth-floatation
(b) in the presence of fluorite (b) removal of iron as slag
(c) in the presence of cryolite which forms a melt with lower melting (c) self reduction step to produce ‘blister copper’ following
temperature evolution of SO2
(d) in the presence of cryolite which forms a melt with higher (d) refining of ‘blister copper’ by carbon reduction
melting temperature 47. Copper is purified by electrolytic refining of blister copper.
37. The chemical process in the production of steel from The correct statement(s) about this process is/are (2015 Adv.)
haematite ore involve (2000, 1M) (a) impure Cu strip is used as cathode
(a) reduction (b) acidified aqueous CuSO4 is used as electrolyte
(b) oxidation (c) pure Cu deposits at cathode
(c) reduction followed by oxidation (d) impurities settle as anode-mud
(d) oxidation followed by reduction 48. Upon heating with Cu 2 S, the reagent(s) that give copper
38. In the commercial electrochemical process for aluminium metal is/are (2014 Adv.)
extraction, the electrolyte used is (1999, 2M) (a) CuFeS2 (b) CuO (c) Cu2O (d) CuSO4
(a) Al(OH)3 in NaOH solution
(b) an aqueous solution of Al 2 (SO4 )3 49. The carbon-based reduction method is not used for the
(c) a molten mixture of Al 2O3 and Na 3AlF6 extraction of (2013 Adv.)
(d) a molten mixture of AlO(OH) and Al(OH)3 (a) tin from SnO2
(b) iron from Fe2O3
39. The major role of fluorspar (CaF2 ) which is added in small (c) aluminium from Al 2O3
amount in the electrolytic reduction of alumina dissolved in (d) magnesium from MgCO3 , CaCO3
fused cryolite (Na 3 AlF6 ) is (1993, 1M)
(a) as a catalyst 50. Extraction of metal from the ore cassiterite involves (2011)
(b) to make the fused mixture very conducting (a) carbon reduction of an oxide ore
(c) to increase the temperature of the melt (b) self-reduction of a sulphide ore
(d) to decrease the rate of oxidation of carbon at the anode (c) removal of copper impurity
(d) removal of iron impurity
40. Hydrogen gas will not reduce (1985, 1M)
(a) heated cupric oxide (b) heated ferric oxide 51. Addition of high proportions of manganese makes steel
(c) heated stannic oxide (d) heated aluminium oxide useful in making rails (1998)
(a) gives hardness to steel
41. In the alumino-thermite process, aluminium acts as (b) helps the formation of oxides of iron
(a) an oxidising agent (b) a flux (1983, 1M)
(c) can remove oxygen and sulphur
(c) a reducing agent (d) a solder
(d) can show highest oxidation state of + 7
Extraction of Metals 285

52. Of the following, the metals that cannot be obtained by S ° [ C( s )] = 6.0 J K −1 mol −1
electrolysis of the aqueous solution of their salts are
S ° [ CO2 ( g )] = 210.0 JK −1 mol −1
(a) Ag (b) Mg (c) Cu (1990, 1M)
(d) Al (e) Cr Assume that, the enthalpies and the entropies are temperature
53. In the electrolysis of alumina, cryolite is added to (1986, 1M)
independent. (2020 Adv.)

(a) lower the melting point of alumina


(b) increase the electrical conductivity Match the Columns
(c) minimise the anode effect 59. Match the anionic species given in Column I that are present in
(d) remove impurities from alumina the ore (s) given in Column II. (2015 Adv.)

Assertion and Reason Column I Column II


Read the following questions and answer as per the direction A. Carbonate p. Siderite
given below : B. Sulphide q. Malachite
(a) Statement I is correct; Statement II is correct; Statement II
C. Hydroxide r. Bauxite
is the correct explanation of Statement I.
(b) Statement I is correct; Statement II is correct; Statement II D. Oxide s. Calamine
is not the correct explanation of Statement I. t. Argentite
(c) Statement I is correct; Statement II is incorrect.
(d) Statement I is incorrect; Statement II is true. 60. Match each of the reactions given in Column I with the
corresponding product(s) given in Column II. (2009)
54. Statement I Al(OH)3 is amphoteric in nature.
Statement II Al—O and O—H bonds can be broken with Column I Column II
equal ease in Al(OH)3 . (1998) A. Cu + dil. HNO3 p. NO
B. Cu + conc. HNO3 q. NO2
Passage Based Questions
C. Zn + dil. HNO3 r. N2 O
Passage D. Zn + conc. HNO3 s. Cu(NO3 )2
Copper is the most noble of the first row transition metals
t. Zn(NO3 )2
and occurs in small deposits in several countries. Ores of copper
include chalcanthite (CuSO4 ⋅ 5H2 O), atacamite (Cu 2 Cl(OH)3 ), 61. Match the conversions in Column I with the type(s) of
cuprite (Cu 2 O), copper glance (Cu 2 S) and malachite reaction(s) given in Column II. (2008, 6M)
(Cu 2 (OH)2 CO3 ). However, 80% of the world copper production
comes from the ore chalcopyrite (CuFeS2 ). The extraction of Column I Column II
copper from chalcopyrite involves partial roasting, removal of iron A. PbS → PbO p. Roasting
and self-reduction. (2010) B. CaCO3 → CaO q. Calcination
55. Partial roasting of chalcopyrite produces C. ZnS → Zn r. Carbon reduction
(a) Cu 2S and FeO (b) Cu 2O and FeO D. Cu 2 S → Cu s. Self-reduction
(c) CuS and Fe2O3 (d) Cu 2O and Fe2O3
62. Match the extraction processes listed in Column I with metals
56. Iron is removed from chalcopyrite as listed in Column II. (2006, 6M)
(a) FeO (b) FeS (c) Fe 2O3 (d) FeSiO3

57. In self-reduction, the reducing species is Column I Column II


(a) S (b) O 2−
(c) S 2−
(d) SO2 A. Self-reduction p. Lead
B. Carbon reduction q. Silver
Numerical Answer Type Questions C. Complex formation and r. Copper
58. Tin is obtained from cassiterite by reduction with coke. Use displacement by metal
the data given below to determine the minimum temperature D. Decomposition of iodide s. Boron
(in K) at which the reduction of cassiterite by coke would
take place. 63. Each entry in Column X is in some way related to the entries in
At 298 K : ∆ f H ° [ SnO2 ( s )] = − 5810
. kJ mol ,−1 Columns Y and Z. Match the appropriate entries. (1988, 3M)

∆ f H ° [(CO 2 )( g )] = − 394.0 kJ mol −1 Column X Column Y Column Z


A. Invar p. Co, Ni m. Cutlery
S ° [ SnO2 ( s )] = 56.0 J K −1 mol −1
B. Nichrome q. Fe, Ni n. Heating element
S ° [ Sn ( s )] = 52.0 JK −1 mol −1
C. Stainless steel r. Fe, Cr, Ni o. Watch spring
286 Extraction of Metals

64. Match the following choosing one item from Column X and 79. Give the number of water molecule (s) directly bonded to the
the appropriate item from Column Y. (1983, 2M) metal centre in CuSO4 ⋅ 5H2O. (2009, 2M)

Column X Column Y 80. Write balanced chemical equation for developing a black and
white photographic film. Also give reason, why the solution
A. Al p. Calamine of sodium thiosulphate on acidification turns milky white and
B. Cu q. Cryolite give balance equation of this reaction. (2005, 2M)
C. Mg r. Malachite 81. A1 and A2 are two ores of metal M . A1 on calcination gives
D. Zn s. Carnalite black precipitate, CO2 and water.
n Black solid + CO2 + H2O
65. Match the following metals listed in Column I with extraction atio
alcin
processes listed in Column II. (1979, 2M) C
Roasting
Dil. H ; A2 Metal + gas
A1 Cl
Column I Column II KI
A. Silver p. Fused salt electrolysis I2 + ppt. K 2Cr2O7
+ H2SO4
B. Calcium q. Carbon reduction
C. Zinc r. Carbon monoxide reduction Green colour
D. Iron s. Amalgamation Identify A1 and A2 . (2004, 4M)

E. Copper t. Self-reduction 82. Which of the two, anhydrous or hydrated AlCl3 is more
soluble in diethyl ether? Justify using the concepts of bonding
Fill in the Blanks in not more than 2 or 3 sentences. (2003)

66. Silver jewellery items tarnish slowly in the air due to their 83. Write the balanced chemical reactions involved in the
reaction with………… (1997)
extraction of lead from galena. Mention oxidation state of
lead in litharge. (2003, 2M)
67. In the extractive metallurgy of zinc, partial fusion of ZnO
with coke is called …… and reduction of the ore to the molten 84. Write the balanced chemical equation for developing
metal is called …… (smelting, calcining, roasting, sintering). photographic films. (2000)
(1988, 1M) 85. Write the chemical reactions involved in the extraction of
68. Silver chloride is sparingly soluble in water because its lattice silver from argentite. (2000, 2M)
energy greater than ……….. energy. (1987) 86. Work out the following using chemical equations.
69. Galvanisation of iron denote coating with …… (1983) In moist air, copper corrodes to produce a green layer on the
surface. (1998)
70. Cassiterite is an ore of …… (1980, 1M)
87. When the ore haematite is burnt in air with coke around
71. In the thermite process …… is used as a reducing agent. 2000°C along with lime, the process not only produces steel
(1980, 1M)
but also produces a silicate slag, that is useful in making
72. In the basic Bessemer process for the manufacture of steel, building materials such as cement. Discuss the same and
the lining of the converter is made up of … . The slag formed show through balanced chemical equations. (1998, 4M)
consists of …… (1980, 1M)
88. Give balance equation for the reaction of aluminium with
73. AgCl dissolve in excess of KCN solution to give ………… aqueous sodium hydroxide. (1997)
complex compound. (1980)
89. Write a balanced equation for the reaction of argentite with
KCN and name the products in the solution. (1996)
True/False
90. Give reasons for the following
74. Cu + disproportionate to Cu 2+ and elemental copper in “Although aluminium is above hydrogen in the
solution. electrochemical series, it is stable in air and water.”
(1991) (1994, 1M)
75. Silver chloride is more soluble in very concentrated sodium 91. Complete the following reaction :
chloride solution than in pure water. (1984, 1M) Sn + 2KOH + 4H2 O → ...... + ....... (1994)

76. Dilute HCl oxidises metallic Fe to Fe2+ . (1983, 1M) 92. Give briefly the isolation of magnesium from sea water by the
Dow’s process.
77. Silver fluoride is fairly soluble in water. (1982)
Give equations for the steps involved. (1993, 3M)

Subjective Questions 93. Complete and balance the following reaction :


Copper reacts with HNO3 to give NO and NO2 in the molar
78. Give the coordination number of Al in the crystalline state of
ratio of 2:1
AlCl3. (2009, 2M)
Cu + HNO3 → KK + NO + NO2 + KK (1992)
Extraction of Metals 287

94. Write balanced equation for the extraction of “Copper from 98. Each of the following statement is true, only under some
copper pyrites by self reduction.” (1990, 2M) specific conditions. Write the condition for each
95. Give balanced equations for the extraction of “Silver from subquestion in not more than 2 sentences.
silver glance by cyanide process.” (1988, 1M)
(i) Metals can be recovered from their ores by chemical methods
96. Answer the following questions briefly (ii) High purity metals can be obtained by zone refining method.
(1984, 2M)
(i) What is the actual reducing agent of haematite in blast furnace?
99. Give reason for the following in one or two sentences :
(ii) Give the equation for the recovery of lead from galena by air
“Silver bromide is used in photography.” (1983)
reduction.
(iii) Why is sodium chloride added during electrolysis of fused 100. State the conditions under which the preparation of
anhydrous magnesium chloride? alumina from aluminium is carried out. Give the necessary
(iv) Zinc, not copper is used for the recovery of metallic silver from equations which need not be balanced. (1983, 2M)
complex [Ag(CN)2 ]– , explain. 101. Write the chemical equations involved in the extraction of
(v) Why is chalcocite roasted and not calcinated during recovery of lead from galena by self reduction process. (1979, 2M)
copper? (1987, 5M)
102. Write balanced equation involved in the preparation of tin
97. Write balanced chemical equation for the following metal from cassiterite. (1979)
“Gold is dissolved in aqua regia.” (1987)

Answers
1. (a) 2. (c) 3. (d) 4. (a) 57. (c) 58. (935)
5. (b) 6. (d) 7. (a) 8. (d) 59. A → p, q, s; B → t; C → q; D → r
9. (d) 10. (d) 11. (a) 12. (d) 60. A → p, s; B → q, s; C → r, t; D → q, t
13. (a) 14. (b) 15. (d) 16. (d)
51. A → p; B → q; C → r, s; D → p, s
17. (c) 18. (c) 19. (b) 20. (d)
62. A → p, r; B → p; C → q; D → s
21. (b) 22. (a) 23. (d) 24. (b)
25. (a) 26. (b) 27. (d) 28. (b) 63. A → q, o; B → p, n; C → r, m
29. (b) 30. (c) 31. (a) 32. (a) 64. A → q; B → r; C → s; D → p
33. (b) 34. (a) 35. (b) 36. (c) 65. A → s; B → p; C → q; D → q, r; E → t
37. (a) 38. (c) 39. (b) 40. (d) 66. (H 2S) 67. (Sintering, Smelting)
41. (c) 42. (c) 43. (a) 44. (d) 68. (Hydration) 69. (Zn) 70. (Sn) 71. (Al)
45. (a,b,c,d) 46. (a,b,c) 47. (b,c,d) 48. (b,c,d) 72. (Lime, calcium phosphate) 73. K [Ag(CN) 2 ]
49. (c,d) 50. (a,d) 51. (a,c) 52. (b,d) 74. (T) 75. (T) 76. (T) 77. (T)
53. (a,b) 54. (b) 55. (b) 56. (d) 78. (6) 79. (4) 83. (2)

Hints & Solutions


1. Due to industrial process, CO2 release from calcination and SO2 It predicts the feasibility of thermal reduction of the ore. The
release from roasting respectively which is responsible for global criterion of feasibility is that at given temperature, Gibbs
warming and acid rain. energy of the reaction must be negative.
Calcination It involves heating where the volatile matter escapes 3. ZnCO3-Calamine (zinc ore)
leaving behind the metal oxide. In this process, the ore is heated below CuCO3 ⋅ Cu(OH)2-Malachite (copper ore)
its melting point in the absence of air or in the limited supply of air.
Fe3O4-Magnetite (iron ore)

e.g. CaCO3 (s) → CaO(s) + CO2 ↑ Na 3AlF6-Cryolite (aluminium ore)
(Carbon dioxide)
Responsible for Thus, option (d) is correct.
global warming
Roasting In this process the ore is heated in a regular supply of air
4. The correct statement is ‘‘leaching of bauxite using
concentrated NaOH solution gives sodium aluminate and
in a furnace at a temperature below the melting point of the metal.
sodium silicate’’. Bauxite usually contains SiO2, iron oxides
e.g. 2ZnS + 3O2 → 2ZnO + 2SO2 ↑ and titanium oxide (TiO2 ) as impurities. Concentration is
(Sulphur dioxide) carried out by digesting the powdered ore with a concentrated
Responsible for acid rain solution of NaOH at 473-523 K and 35-36 bar pressure. Al2O3
2. Ellingham diagram provides information about temperature is leached out as sodium aluminate (and SiO2 too as sodium
silicate) leaving the impurities behind.
dependence of the standard Gibbs energies of formation of some
metal oxides. Al 2O3 (s) + 2NaOH(aq) + 3H2O(l ) → 2Na[Al(OH)4 ](aq)
288 Extraction of Metals

The aluminate in solution is neutralised by passing CO2 gas and 8. Bauxite is not a carbonate ore. Its chemical formula is Al 2O3 or
hydrated Al 2O3 is precipitated. Here, the solution is seeded with AlOx(OH)3 − 2x , where 0 < x < I. Chemical formula of other ores
freshly prepared samples of hydrated Al 2O3 which induces given in options are as follows:
precipitation. Siderite-FeCO3
2Na[Al(OH)4 ](aq) + CO2 (g ) → Calamine-ZnCO3
Al 2O3 ⋅ xH2O(s) + 2NaHCO3 (aq) Malachite-CuCO3 ⋅ Cu(OH)2
The sodium silicate remains in the solution and hydrated alumina
is filtered, dried and heated to give back pure Al 2O3. 9. Only assertion is correct and reason is incorrect. Haematite is not
1470 K a carbonate ore. It is an oxide ore, i.e. Fe2O3. Cast iron is
Al 2O3 ⋅ xH2O(s)  → Al 2O3 (s) + xH2O(g ) extracted chiefly from its oxide ore (haematite) by heating in the
5. The idea of froth floatation method came from a person ‘washer presence of coke and limestone in a blast furnace.
woman’ ( X ) and this method is related to the process 10. Cryolite ore (Na 3AlF6, sodium hexafluoroaluminate) contain
concentration (Y ) of ores.
fluorine while other given options such as malachite
This method is based upon the preferential wetting properties (Cu 2 (CO)3 (OH)2 ), sphalerite ((Zn,Fe)S) and bauxite (Al 2O3 )
with the frothing agent (collector) and water. does not contain fluorine.
6. The explanation of given statements are as follows: 11. Mond process is used in the purification of Ni. It is a vapour
(a) Zone refining process is used for the refining of B, Ga, In, Si phase refining process.
and Ge.
It is based on the principle that Ni is heated in the presence of
Ti is refined by van Arkel method.
carbon monoxide to form nickel tetracarbonyl, which is a
Thus, statement (a) is incorrect.
volatile complex. This complex is then decomposed by
(b) Zincite (ZnO) is an oxide ore of Zn. subjecting it to a higher temperature (450-470 K) to obtain pure
Thus, statement (b) is incorrect. nickel metal.
(c) NaCN is used in the hydrometallurgy of silver. It is known as Crude nickel (s) + 4 CO (g)
Mc. Arthur Forrest process. (Impure) Recycled
The reactions occuring during the process are as follows:
330-350 K
Ag2S + 4NaCN → 2Na[Ag(CN)2 ] + Na 2S
4Na 2S + 2H2O + 5O2 → 2Na 2SO4 + 4NaOH + 2S [Ni (CO)4] (g) 450-470 K
Ni (s)
Pure + 4 CO (g)
2Na[Ag(CN)2 ] + Zn → Na 2[Zn(CN)4 ] + 2Ag (Volatile compound)
Thus, statement (c) is incorrect.
(d) Aniline and cresol help in stabilising the froth in froth 12. With respect to an ore, Ellingham diagram helps to predict the
floatation process. feasibility of its thermal reduction. It is a graph representation of
Thus, statement (d) is correct. Gibbs energy change versus absolute temperature.
0
7. Refining of crude metals results pure metals and its impurities –100
get separated out. 2Cu 2O
–200 4Cu+O 2
I. Liquation In this method low melting metals like Sn, Pb, Bi 2FeO
–300 2Fe+ O 2
C+O2 CO2
and Hg can be made to flow down through a sloping surface –400
∆Gº/kJ mol–1 of O2

2C+O
leaving behind the higher melting impurities on the hearth. 2CO 2 2
–500 +O 2 O 2CO
II. Zone refining The basic principle of the method is, 2CO 2Zn
–600 +O 2
impurities are more soluble in the molten metal than in the 2Zn
–700
solid state of the metal. This method is useful to produce
–800
semiconductors and ultra-pure metals like B, Ga, In,Si and Ge. l +O 3
–900 2/3 A 2
III. Mond process O2
Al+ O
–1000 4/3 2Mg
+O 2
Crude nickel (s) + 4CO(g)
330-350 K
Ni(CO)4(g)
450-470 K
–1100 2 Mg
(Impure) (Volatile compound)
–1200
Recycled Ni(s)
4CO(g) + 0°C 400°C 800°C 1200°C 1600°C 2000°C
(Pure)
273 K 673 K 1073 K 1473 K 1873 K 2273 K
IV. van Arkel method Temperature
500-600 K 1700-1800 K
Gibbs energy (∆Gº) versus T plots (schematic)
M (s) + 2I2 (g) MI4(g) for the formation of some oxides (Ellingham diagram)
(Crude (Volatile compound)
metal) Generally, the diagram consists of plots of ∆G ° versus T for the
Recycled M(s)
2I2(g) + formation of oxides of elements
(Ultra-pure)
2xM (s) + O2 (g ) → 2M x O(s)
Here, M = Zr, Hf, Ti Thermal reduction product

Hence, the correct matching is In this reaction, amount of gas decreases thus, randomness
I→ (C), II→(D), III →(B), IV → (A). decreases. Hence, ∆S becomes negative. Therefore, the value of
Extraction of Metals 289

free energy increases with increase in temperature. There is a 18. Hall-Heroult’s process is an electro-reduction process by which
point in a curve below which ∆G is negative. So, M xO is stable. pure alumina (Al 2O3 ) is reduced to crude Al.
Above this point, M xO will decompose on its own. In this process, electrolysis of a fused mixture of Al 2O3,
13. The hydroxide, hydrated oxides and carbonate ores, after Na 3[AlF6 ] (cryolite) and CaF2 (fluorspar) is carried out at carbon
concentration, are subjected to calcination. In the process, the cathode and graphite anode.
ore is heated below its melting point in the limited supply or The overall reaction is represented as:
absence of air. As the result, these are converted into their oxides.
2Al 2O3 + 3C → 4Al + 3CO2
So, among the given options, the options having either carbonates
(e.g. ZnCO3 and CaCO3 ⋅ MgCO3) or hydrated oxide (e.g. 19. From the Ellingham diagram, we can say that any oxide with
Fe2O3 ⋅ xH2O), require calcination while pair of option (a), i.e. lower value of ∆G ° is more stable than a oxide with higher value
ZnO and MgO does not require calcination. of ∆G °. We can also predict that the oxide placed higher in the
diagram can be reduced by the element involved in the formation
14. In the Hall - Heroult’s process, aluminium in formed at the
cathode. The cathode is made out of carbon. In this method, of its oxide placed lower at that temperature in the diagram.
Al 2O3 is melted with cryolite, Na 3[AlF6 ] and electrolysed in a It is happening in case of ZnO for its reduction by Al at 1400°C.
graphite lined steel tank, which serves as the cathode. The anode
is also made of graphite.
20. The formulae of the given ores are as follows:
Malachite : CuCO3 ⋅ Cu(OH)2
The cell runs continuously and at intervals molten aluminium is
drained from the bottom of the cell and more bauxite is added. Copper pyrites : CuFeS2
The electrolytic reactions are as follows: Dolomite : CaMg(CO3 )2
At cathode Al3+ + 3e− → Al Azurite : Cu 3 (CO)3 (OH)2
At anode C(s) + O 2− (melt) → CO(g)+ 2e− 21. Sulphide ores are concentrated by froth floatation method e.g.
2− − Galena (PbS)
C(s) + 2O (melt) → CO2(g)+ 4 e
22. H2O2 acts as an oxidising as well as reducing agent, because
15. Calcination is one of the pyrometallurgical process, like roasting
by which a concentrated ore gets converted into its oxide. oxidation number of oxygen in H2O2 is −1. So, it can be oxidised
to oxidation state 0 or reduced to oxidation state –2.
In calcination, a hydrated carbonate or bicarbonate ore or a
hydrated ore is heated at lower temperature (compared to roasting) H2O2 decomposes on exposure to light. So, it has to be stored in
in absence of air to give its oxide as in options (a), (b) and (c). plastic or wax linked glass bottles in dark for the prevention of
Here, volatile non-metallic oxides like H2O, CO2, are also exposure. It also has to be kept away from dust.
produced. 23. (a) In Hall-Heroult process for extraction Al, carbon anode is
Roasting is valid mainly for sulphide ores like option (d), where oxidised to CO and CO2.
SO2 gets liberated. In this reaction, calcination cannot be used. (b) When Al 2O3 is mixed with CaF2, it lowers the melting point
of the mixture and brings conductivity.
16. The correct match is: A → R; B → S; C → Q; D → P.
(A) Siderite is an ore of iron with molecular formula FeCO 3 (R). (c) Al 3+ is reduced at cathode to form Al.
(B) Kaolinite is an ore of aluminium with molecular formula (d) Here, Al 2O3 is an electrolyte, undergoing the redox process.
Al 2Si 2(OH)2O 5 (S). Na 3AlF6 although is an electrolyte but serves as a solvent, not
electrolyte.
(C) Malachite is an ore of copper with molecular formula
CuCO 3 ⋅ Cu(OH)2 (Q). 24. Higher the position of element in the electrochemical series more
(D) Calamine is an ore of zinc with molecular formulaZnCO 3 (P). difficult is the reduction of its cations.
17. Electroplating is a process of coating one metal or metal object If Ca 2+ (aq) is electrolysed, water is reduced in preference to it.
with a very thin layer of another metal typically applying a direct Hence, it cannot be reduced electrolytically from their aqueous
electric current. solution.
Electrolytes used in the electroplating of gold and silver are Ca 2+ ( aq) + H2O → Ca 2+ + OH− + H2 ↑
given in the table below:
25. Element Ores Name
Pure metal block
Article to acts an anode
Ag Ag 2 S Argentite
Process
be plated by which Electrolyte Cu CuFeS2 Copper pyrites
out acts as electroplating (aqueous solution)
cathode Pb PbS Galena
will be done
I Sn SnO2 Cassiterite
(a) Gold Article Au(s)
Na[A u(CN)2 ]
plating Mg MgCO3 ⋅ CaCO3 Dolomite
(Sodium
auro-cyanide) Al Al 2 O3 ⋅ xH2 O Bauxite
I
(b) Silver Article Ag(s) 26. The reactions involved in extraction of silver by cyanide process
Na[A g(CN)2 ]
plating are
(Sodium argento
cyanide) Ag2S + CN− + O2 → [Ag(CN)2 ]− + SO2 …(i)
290 Extraction of Metals

[Ag (CN)2 ]− + Zn → [Zn (CN)4 ]2− + Ag …(ii) (b) The electrolysis of alumina by Hall-Heroult’s process is
In reaction (i), sulphide is oxidised to SO2 by oxygen. In the carried by using a fused mixture of alumina and cryolite
(Na2AlF6 ) along with minor quantities of aluminium fluoride
reaction (ii), silver ion (Ag+ ) is reduced to Ag by Zn. Therefore,
and fluorspar. The addition of cryolite and fluorspar
O2 is oxidising agent and Zn is reducing agent. increases the electrical conductivity of alumina and lowers
27. Haematite is Fe2O3, in which oxidation number of iron is III. the fusion temperature. Hence, the statement (b) is true.
Magnetite is Fe3O4 which is infact a mixed oxide (FeO ⋅ Fe2O3), (c) At anode, alumina reacts with fluorine to liberate oxygen and
hence iron is present in both II and III oxidation state. evolved oxygen reacts with carbon to form carbon dioxide
28. A water soluble complex with silver and dilute aqueous solution and hence, statement (c) is true.
of NaCN is Na[Ag(CN)2 ]. In the cyanide process, the native (d) Steel cathode with carbon lining and graphite anode are used
silver is crushed and treated with aqueous NaCN solution and and hence,the statement (d) is true.
aerated. 46. CuFeS2 (copper pyrite) is converted into copper into following
4Ag + 8NaCN + 2H2 O + O2 → 4Na [Ag(CN)2 ] steps:
+ 4NaOH Step I Crushing (grinding ) followed by concentration by
29. Zinc blende contain ZnS which is first roasted partially and then froth-floatation process.
subjected to reduction with carbon Step II Roasting of ore in the presence of SiO2 which removes iron as
ZnS + O2 → ZnO + SO2 Roasting slag (FeSiO3).
∆ 2CuFeS2 + O2 → Cu 2S + 2FeS + SO2
ZnO + C → Zn + CO ↑ Carbon reduction
2FeS + 3O2 → 2SO2 + 2FeO
30. Chalcopyrite contain both iron and copper. FeO + SiO2 → FeSiO3 (Slag)
31. Lead is mainly extracted by self-reduction process while tin is Step III Self-reduction in Bessemer converter
extracted by carbon reduction method.
2Cu 2S + 3O2 → 2Cu 2O + 2SO2
32. Au + 2CN− → [Au(CN)2 ]− 2Cu 2O + Cu 2S → 6Cu + SO2
X Copper obtained is blister copper (98% pure).
2Au(CN)−2 + Zn → [Zn(CN)4 ]2 − + 2Au Step IV Refining of blister copper is done by electrolysis
Y
Impure copper—Anode
33. Heating iron in stream of dry chlorine gas gives FeCl 3 in Pure copper— Cathode
anhydrous form. In all other cases (a and c) hydrated FeCl 3 is
At anode : Cu → Cu 2+ + 2e−
obtained while in (d), FeCl 2 is formed.
At cathode : Cu 2+ + 2e− → Cu
34. Mg is extracted by electrolysis of molten MgCl 2 .
Carbon-reduction method is not used. Thus, (d) is incorrect.
35. Iron present in copper pyrite is removed by forming FeSiO3 as
slag. 47. (a) is wrong statement. Impure copper is set as anode where
36. Cryolite is added to alumina in order to lower the melting point. copper is oxidised to Cu 2+ and goes into electrolytic
37. Haematite ore contain Fe2 O3 which is reduced by CO in the blast solutions.
furnace as (b) CuSO4 is used as an electrolyte in purification process.
Fe2 O3 + CO → Fe + CO2 (c) Pure copper is deposited at cathode as:
38. Al 2 O3 mixed with cryolite Na 3[AlF6 ] is fused and electrolysed in Cu 2+ + 2e– → Cu : (At cathode)
the extraction of Al. (d) Less active metals like Ag, Au etc settle down as anode mud.
39. Fluorspar (CaF2 ) improve the electrical conductivity during 1100 °C
48. (b) 4CuO → 2Cu 2O + O2
electrolytic reduction of alumina.

40. Al itself is a very strong reducing agent. 2Cu 2O + Cu 2S → 6Cu + SO2

41. In thermite welding, Al acts as a reducing agent (c) Cu 2S + 2Cu 2O → 6Cu + SO2
2Al + Fe2 O3 → Al 2 O3 + 2Fe + Heat 720 ° C 1
(d) CuSO4 → CuO + SO2 + O2
42. The actual representation of CuSO4 ⋅ 5H2 O (blue vitriol) is 2
[Cu(H2 O)4 ]SO4 ⋅ H2 O and it has covalent, ionic and coordinate 1100 ° C
covalent bonds. 4CuO → 2Cu 2O + O2

43. Ca 2 + end up in CaSiO3 (slag). ∆


2Cu 2O + Cu 2S → 6Cu + SO2
44. Iron is rendered passive by concentrated HNO3 due to formation Reaction is believed to proceed as
of a thick protective layer of Fe3O4 . Cu 2S r 2Cu + + S2−
45. (a) 2Na[Al(OH)4 ](aq) + CO2 → Na 2CO3 2Cu 2O r 4 Cu + + 2O2−
+ H2O + 2Al(OH)3 ↓ or Al 2O3.2H2O (ppt.) S + 2O2− → SO2 + 6e−
2−

Hence, the statement (a) is true. 6Cu + + 6e− → 6Cu ; Ecell


o
= 0.52
Extraction of Metals 291

Here, copper sulphide is reduced to copper metal. Solidified 57. Cu 2S + 2Cu 2O → 6Cu + SO2
copper has blistered appearance due to evolution of SO2 and thus
obtained copper is known as blister copper. In Cu 2S, sulphur is S2− and in SO2, sulphur is in +4 state.
Hence, S2− is acting as reducing agent.
Other compounds which give Cu are
1100 °C 58. Tin is obtained from cassiterite by reduction with coke and the
(i) CuO as 4CuO → 2Cu 2O + O2 balanced chemical reaction is
∆ SnO2 (s) + C (s) → Sn (s) + CO2 (g )
2Cu 2O + Cu 2S → 6Cu + SO2
Standard enthalpy of reaction,
720 °C 1
(ii) CuSO4 as CuSO4 → CuO + SO2 + O2 ∆H R°n = (∆H °f CO2 (g )) − (∆H f° SnO2 (s))
2

4CuO → 2Cu 2O + O2 ∆H R°n = − 394 − (− 581) ⇒187 kJ

2Cu 2O + Cu 2S → 6Cu + SO2 °
Standard entropy of reaction, ∆S R°n = ∆S Products °
− ∆S Reactants
While CuFeS2 will not give Cu on heating. The heating in the ∆S R°n = [ S ° (Sn (s)) + S ° (CO2 (g ))] −
presence of O2 gives Cu 2S and FeS with the evolution of SO 2.
[ S ° (SnO2 (s)) + S ° (C(s))]
49. Al has greater affinity for oxygen, hence oxide is not reduced by
carbon. MgO and CaO (formed in the calcination from ∆S R°n = [ 52 + 210 ] − [ 56 + 6 ] ⇒ 200 JK −1 mol −1
carbonates) are stable species and not reduced by carbon. We know that, ∆H ° = T∆S °
1300 °C ∆H ° 187 × 1000
During Smelting SnO2 + C → Sn + CO ∴ T = = ⇒ 935 K
∆S ° 200
∆ 4Fe + 3 CO
2Fe2O3 + 3 C → For the reaction to be spontaneous, the temperature should be
2
greater than 935 K.
50. The important ore of tin is cassiterite (SnO2). Tin is extracted
from cassiterite ore by carbon reduction method in a blast furnace. 59. Siderite = FeCO3, Malachite = CuCO3 ⋅ Cu(OH)2
SnO2 + 2C  → Sn + 2CO Bauxite = Al 2O3 ⋅ 2H2O2 consisting some Al (OH)3
The product often contain traces of iron which is removed by Calamine = ZnCO3, Argentite = Ag2S
blowing air through the melt to oxidise to FeO which then floats
to the surface. 60. A. 3Cu + 8HNO3 → 3Cu(NO3 )2 + 2NO + 4H2 O
Dil.
2Fe + O2 
→ 2FeO
B. Cu + 4HNO3 → Cu(NO3 )2 + 2NO2 + 2H2 O
51. Addition of manganese to iron improve hardness of steel as well Conc.
as remove oxygen and sulphur. C. 4Zn + 10HNO3 → 4Zn(NO3 )2 + N2 O + 5H2 O
52. Magnesium and aluminium are both highly electropositive, more Dil.
electropositive than water cannot be obtained by electrolysis of D. Zn + 4HNO3 → Zn(NO3 )2 + 2NO2 + 2H2 O
aqueous solution of their salts. Conc.
O2
61. A. PbS → PbO + SO2 , roasting
53. Alumina (Al 2 O3 ) has very high melting point and it is poor
conductor of electricity. Both these factors posses difficulty in B. CaCO3 → CaO + CO2 ↑; calcination
electrolysis of molten alumina.
C. ZnS → Zn, can be done by carbon reduction
Cryolite, Na 3AlF6, when mixed with alumina, lowers melting or self reduction
point as well as improve electrical conductivity, hence helps in D. Cu 2 S → Cu, roasting followed by self reduction
electrolysis of Al 2 O3.
62. Extraction Metals
54. Al(OH)3 is amphoteric
methods extracted
Al(OH)3 + 3HCl → AlCl 3 + 3H2 O A. Self reduction r. Copper, (P) Lead
Base
Al(OH)3 + NaOH → Na[Al(OH)4 ]
B. Carbon reduction p. Lead
Acid C. Complex formation q. Silver : Ag 2 S + NaCN
High charge and small size of Al 3+ makes Al—O and O—H and displacement → Na[Ag(CN)2 ]
bonds equally ionisable. by metal Na[Ag(CN)2 ] + Zn
→ Na 2 [Zn(CN)4 ] + Ag
55. 2CuFeS2 + O2 → Cu 2S + 2FeS + SO2 ↑
D. Decomposition of s. Boron :
2Cu 2S + 3O2 → 2Cu 2O + 2SO2 ↑ iodide ∆ 3
2FeS + 3O2 → 2FeO + 2SO2 ↑
BI 3 → B + I2
2
56. FeO + SiO2 → FeSiO3 (Slag)
292 Extraction of Metals

63. Column X Column Y Column Z 76. True Iron is more electropositive than hydrogen
Invar Fe, Ni Watch spring Fe + 2HCl → FeCl 2 + H2 ↑

Nichrome Co, Ni Heating element 77. True : Solubility of silver halides decreases down in the group
Stainless steel Fe, Cr, Ni Cutlery Solubility : AgF > AgCl > AgBr > AgI
78. In crystalline state, AlCl 3 has rock-salt like structure with
64. Column X (Metals) Column Y coordination number of Al = 6.
(Ores)
A. Al q. Cryolite 79. Four, the complex has formula [Cu(H2 O)4 ] SO4 ⋅ H2 O
B. Cu r. Malachite 80. (a) 2AgBr + C6H4 (OH)2 → 2Ag + 2HBr + C6 H4 O2
Hydroquinone Quinone
C. Mg s. Carnalite (developer)
D. Zn p. Calamine
AgBr + 2Na 2 S2 O3 → Na 3[Ag(S2 O3 )2 ] + NaBr
65. A. Silver is extracted by amalgamation process (b) Na 2 S2 O3 + 2H+ → 2Na + + H2 SO3 + S ↓
Colloidal
Distillation
Ag + Hg → Ag(Hg) → Ag( s) + Hg( v ) ↑ sulphur

Amalgam 81. A1 is basic copper carbonate (Cu(OH)2 ⋅ CuCO3 ) while A2 is


B. Calcium is extracted by electrolysis of fused CaCl 2 . Cu 2S. The confirmatory reactions are :
C. Zinc is extracted by carbon reduction method ∆
ZnO + C → Zn + CO (i) CuCO3 ⋅ Cu(OH)2 → CuO + CO2 ↑ + H2O
Black
D. Iron is extracted by both carbon reduction method and CO HCl
reduction methods (ii) CuCO3 ⋅ Cu(OH)2 → CuCl 2 + CO2 ↑ + H2O
KI
Fe2 O3 + 3C → 2Fe + 3CO CuCl 2 → Cu 2 I2 ↓ + KCl + I2
Fe2 O3 + 3CO → 2Fe + 3CO2 D
Roasting
E. Copper is extracted by self reduction methods A2 → Cu 2 O + SO2 ↑
Cu 2 S + O2 → Cu 2 O + SO2
Cu 2 S + Cu 2 O → Cu + SO2 ↑
Cu 2 O + Cu 2 S → Cu + SO2
SO2 is a reducing gas that gives green colour with acidified
66. H 2S Ag 2 S (black) is formed on the surface. K 2 Cr2 O7 as
∆ 3SO2 + K 2 Cr2 O7 + H2 SO4 → K 2 SO4 + Cr2 (SO4 )3
67. ZnO + C → Zn + CO = Smelting Green
sintering + 4H2 O
68. Hydration energy Energy required to break the crystal lattice 82. Anhydrous AlCl 3 is more soluble in diethyl ether as the oxygen
during dissolving process comes from hydration. If lattice
atom of ether donate its lone-pair of electrons to the vacant
energy is very high and hydration energy is low, salt becomes
orbital of Al in electron deficient AlCl 3. In case of hydrated
sparingly soluble.
AlCl 3, Al is not electron deficient as oxygen of water molecule
69. Zn Galvanisation involves coating of iron with zinc metal in has already donated its lone-pair of electrons to compensate
order to prevent if from rusting. electron deficiency of Al.
70. Sn Cassiterite is an ore of tin. Cl Cl
 ••  ••
71. Al Aluminium reduces Fe2 O3 to Fe.
Cl — Al ← OH2 Cl — Al ← OEt 2
•• ••
72. Lime, calcium phosphate In basic Bessemer process, the  
Bessemer converter is lined with lime but in acid Bessemer Cl Cl
hydrated anhydrous
process, it is lined with silica. In basic Bessemer process,
phosphorus is slagged off as calcium phosphate : 83. The reactions involved in the extraction of lead from galena
P4 + 5O2 → P4 O10 (PbS) by self reduction are
6CaO + P4 O10 → 2Ca 3(PO4 )2 2PbS + 3O2 → 2PbO + 2SO2
Thomas slag PbS + 2PbO → 3Pb + SO2
73. K [Ag(CN)2 ] : AgCl + 2KCN → K[Ag(CN)2 ] + KCl PbS + 2O2 → PbSO4 (side reaction)
PbSO4 + PbS → 2Pb + 2SO2
74. True: Cu + is unstable
In litharge (PbO), the oxidation state of Pb is +2
H+
2Cu + → Cu 2 + + Cu 84. The common photographic film is coated with AgBr and during
75. True Complex (Na[AgCl 2 ]) formation increases solubility of developing of photographic film, the unreacted AgBr is removed
by Na 2S2O3 as
otherwise sparingly soluble AgCl.
AgBr + 2Na 2S2O3 → Na 3[Ag(S2O3 )2 ] + NaBr
Extraction of Metals 293

85. 4NaCN + Ag2S → 2Na[Ag(CN)2 ] + Na 2S 96. (i) Carbon monoxide :


2Na[Ag(CN)2 ] + Zn → Na 2[Zn(CN)4 ] + 2Ag C + O2 → CO
CO + Fe2O3 → CO2 + Fe
86. 2Cu + H2O + CO2 + O2 → Cu(OH)2 ⋅ CuCO3
Green (ii) 2PbS + 3O2 → 2PbO + 2SO2
(basic copper carbonate)
2PbO + PbS → 3Pb + SO2
2000 ° C
87. C + O2 → CO (iii) To improve electrical conductivity of melt.
3CO + Fe2O3 → 2Fe + 3CO2 (iv) A metal which is much more electropositive than Ag can only
replace Ag + completely from [Ag(CN)2 ] − as
CaCO3 → CaO + CO2
Zn + 2[Ag(CN)2 ]− → [Zn(CN)4 ]2− + 2Ag
CaO + SiO2 → CaSiO3
Slag (v) Chalcocite is a sulphide ore of copper, during roasting, SO2 is
H2O liberated, which is not possible in calcination.
88. Al + NaOH → NaAlO2+ 32 H2
97. 2Au + 3HNO3 + 11HCl → 2HAuCl 4 + 6H2O + 3NOCl
89. 4KCN + Ag2S → 2K[Ag(CN)2 ] + K2S
98. (i) If the metal is moderately electropositive, e.g. Fe, Sn, Pb or
Potassium
dicyanoargentate (I) Cu, they can be obtained from their ore by chemical
reduction methods.
90. Due to formation of protective, inert layer of Al 2O3 on surface. However, if the metal is highly electropositive, e.g. Al, Mg
91. Sn + 2KOH + 4H2O → K2[Sn(OH)6 ] + 2H2 etc., no reducing agent exist for reduction of their ions
(Al 3+ , Mg2+ ) and they are obtained by electrolytic
92. Sea water (contain MgCl 2) + Ca(OH)2 reduction of their molten salt.
→ Mg(OH)2 ↓ + CaCl 2 ( aq ) (ii) Metals like Ge is required in high purity, can be readily
melted and can easily crystallise out from the melt form.
(i) Mg(OH)2 + 2HCl → MgCl 2 + 2H2O
Heat to 99. AgBr is sensitive to visible light.
→ MgCl 2 ( s )
Dryness hν 1
AgBr → Ag + Br2
Fusion electrolysis 2
(ii) MgCl 2 (s) → Mg2+ + 2Cl − → Mg A photographic plate coated with AgBr, when exposed to
NaCl
(At cathode) light, gets blackened due to the above reaction.

93. 7Cu + 20HNO3 → 7Cu(NO3 )2 + 4NO + 2NO2 + 10H2O 100. Al + NaOH(aq) → NaAlO2 + H2
NaAlO2 + CO2 ( aq) → Na 2 CO3 + Al(OH)3 ↓
94. 2CuFeS2 + O2 → Cu 2S + 2FeS + SO2
Copper pyrite ∆
Al(OH)3 → Al 2 O3 + H2 O
2Cu 2S + 3O2 → 2Cu 2O + 2SO2
Roasting 101. In the first step, galena is heated in presence of O2
2FeS + 3O2 → 2FeO + 2SO2
(limited quantity) in a reverberatory furnace, where PbS is
FeS + SiO2 → FeSiO3
Flux Slag partially oxidised to PbO :
3
2Cu 2O + Cu 2S → 6Cu + SO2 Bessemerisation PbS + 2 O2 → PbO + SO2

95. Ag2S + 2NaCN → 2AgCN + Na 2S In the second step, more PbS is added and heated in absence
AgCN + NaCN → Na[Ag(CN)2 ]
of O2 , where the following self reduction takes place
PbS + 2PbO → 3Pb + SO2
2Na[Ag(CN)2 ] + Zn → Na 2[Zn(CN)4 ] + 2Ag
102. SnO2 + 2C → Sn + 2CO( g ), Carbon reduction method.
20
Qualitative Analysis
Objective Questions I (Only one correct option) 6. In the following reaction sequence in aqueous solution, the
species X, Y and Z, respectively, are (2016 Adv.)
1. A colourless aqueous solution contains nitrates of two
+ +
metals, X and Y. When it was added to an aqueous solution of Ag Ag With time
S2 O23 − → X → Y → Z
NaCl, a white precipitate was formed. This precipitate was (Clear solution) (White ppt.) (Black ppt.)
found to be partly soluble in hot water to give a residue P and
(a) [Ag(S2O3 )2 ]3− , Ag2S2O3 , Ag2S
a solution Q. The residue P was soluble in aqueous NH 3 and
also in excess sodium thiosulphate.The hot solution Q gave a (b) [Ag(S2O3 )3 ]5− , Ag2SO3 , Ag2S
yellow precipitate with KI. The metals X and Y , (c) [Ag(SO3 )2 ]3− , Ag2S2O3 , Ag
respectively, are (2020 Adv.) (d) [Ag(SO3 )3 ]3− , Ag2SO4 , Ag
(a) Ag and Pb (b) Ag and Cd
(c) Cd and Pb (d) Cd and Zn 7. In Carius method of estimation of halogens 250 mg of an
2. Reaction of an inorganic sulphite X with dilute H 2SO 4 organic compound gave 141 mg of AgBr. The percentage of
bromine in the compound is (atomic mass Ag = 108,
generates compound Y . Reaction of Y with NaOH gives X .
Br = 80)
Further, the reaction of X with Y and water affords (2015 Main)
compound Z. Y and Z respectively, are (a) 24 (b) 36 (c) 48 (d) 60
(2020 Main, 6 Sep II)
(a) SO2 and Na2 SO3 (b) SO3 and NaHSO3 8. Upon treatment with ammoniacal H 2S , the metal ion that
(c) SO2 and NaHSO3 (d) S and Na2 SO3 precipitates as a sulphide is (2013 Adv.)
(a) Fe (III) (b) Al (III)
3. Among (A) - (D), the complexes that can display
(c) Mg (II) (d) Zn(II)
geometrical isomerism are (2020 Main, 8 Jan II)

(A) [Pt(NH3 )3 Cl] +


(B) [Pt(NH3 )Cl5 ]−
9. Passing H2 S gas into a mixture of Mn 2+ , Ni 2+ , Cu 2+ and
Hg 2+ ions in an acidified aqueous solution precipitates
(C) [Pt(NH3 )2Cl(NO2 )] (D) [Pt(NH3 )4ClBr]2+
(a) CuS and HgS (b) MnS and CuS (2011)
(a) (D) and (A) (b) (C) and (D)
(c) MnS and NiS (d) NiS and HgS
(c) (A) and (B) (d) (B) and (C)
4. An organic compound X showing the following solubility 10. A solution of a metal ion when treated with KI gives a red
profile is (2019 Main, 8 April I) precipitate which dissolves in excess KI to give a colourless
solution. Moreover, the solution of metal ion on treatment
Water
Insoluble with a solution of cobalt (II) thiocyanate gives rise to a deep
5% HCl blue crystalline precipitate. The metal ion is (2007, 3M)
Insoluble
X (a) Pb2+ (b) Hg2+
10% NaOH
Soluble (c) Cu2+ (d) Co2+
10% NaHCO3
Insoluble 11. MgSO4 on reaction with NH4 OH and Na 2 HPO4 forms a
(a) o -toluidine (b) oleic acid white crystalline precipitate. What is its formula? (2006)
(c) m-cresol (d) benzamide (a) Mg(NH4 )PO4 (b) Mg3 (PO4 )2
5. When metal ‘M ’ is treated with NaOH, a white gelatinous (c) MgCl 2 ⋅ MgSO4 (d) MgSO4
precipitate ‘X ’ is obtained, which is soluble in excess of 12. CuSO4 decolourises on addition of KCN, the product is
NaOH. Compound ‘X ’ when heated strongly gives an oxide (a) [Cu(CN)4 ]2 − (2006, 3M)
which is used in chromatography as an adsorbent. The metal
‘M ’ is (2018 Main)
(b) Cu2 + get reduced to form [Cu(CN)4 ]3 −
(a) Zn (b) Ca (c) Al (d) Fe (c) Cu(CN)2
(d) CuCN
Qualitative Analysis 295

13. A solution when diluted with H2 O and boiled, it gives a 22. The brown ring complex compound is formulated as
white precipitate. On addition of excess NH4 Cl / NH4 OH, [Fe(H2 O)5 (NO)+ ] SO4 . The oxidation state of iron is
the volume of precipitate decreases leaving behind a white (1987, 1M)
gelatinous precipitate. Identify the precipitate which (a) 1 (b) 2 (c) 3 (d) 0
dissolves in NH4 OH / NH4 Cl. (2006, 3M)
(a) Zn(OH)2 (b) Al(OH)3
23. Which one amongst the following pairs of ions cannot be
separated by H2 S in dilute HCl? (1986, 1M)
(c) Mg(OH)2 (d) Ca(OH)2
(a) Bi 3+ , Sn 4+ (b) Al 3+ , Hg2+
14. A metal nitrate reacts with KI to give a black precipitate (c) Zn 2+ , Cu2+ (d) Ni 2+ , Cu2+
which on addition of excess of KI convert into orange colour
solution. The cation of metal nitrate is (2005, 1M) 24. The compound insoluble in acetic acid is (1986, 1M)
(a) Hg2+ (b) Bi 3+ (c) Sn 2+ (d) Pb2+ (a) calcium oxide (b) calcium carbonate
(c) calcium oxalate (d) calcium hydroxide
15. (NH4 )2 Cr2 O7 on heating gives a gas which is also given by
25. The ion that cannot be precipitated by both HCl and H2 S is
(2004, 1M)
(a) Heating NH4NO2 (b) Heating NH4NO3 (a) Pb2– (b) Cu+ (1982, 1M)
(c) Mg3N2 + H2O (d) Na(comp. ) + H2O2 (c) Ag+ (d) Sn 2+

16. A sodium salt of an unknown anion when treated with 26. For the equilibrium, 2H 2O - H3O+ + OH − , the value
MgCl 2 gives white precipitate only on boiling. The anion is of ∆Gº at 298 K is approximately (2019 Main 11 Jan II)
(2004, 1M) (a) − 80 kJ mol − 1 (b) 100 kJ mol − 1
(a) SO2– (b) HCO–3 (c) CO2− (d) NO−3
(c) 80 kJ mol − 1 (d) − 100 kJ mol − 1
4 3

17. [ X ] + H2 SO4 → [Y ] a colourless gas with irritating smell


[Y ] + K 2 Cr2 O7 + H2 SO4 → green solution Objective Questions II
[ X ] and [Y ] are (2003, 1M) (One or more than one correct option)
(a) SO23− , SO2 (b) Cl − , HCl 27. The correct option(s) to distinguish nitrate salts to Mn 2+ and
2−
(c) S , H2S (d) CO23− , CO2 Cu 2+ taken separately is (are) (2018 Adv.)
2+
18. A gas X is passed through water to form a saturated solution. (a) Mn shows the characteristic green colour in the flame
The aqueous solution on treatment with silver nitrate gives a test
white precipitate. The saturated aqueous solution also (b) Only Cu 2+ shows the formation of precipitate by passing
dissolves magnesium ribbon with evolution of a colourless H2 S in acidic medium
gas Y. Identify X and Y. (2002, 3M) (c) Only Mn 2+ shows the formation of precipitate by passing
(a) X = CO2, Y = Cl 2 (b) X = Cl 2 , Y = CO2 H2 S in faintly basic medium
(c) X = Cl 2 , Y = H2 (d) X = H2 , Y = Cl 2 (d) Cu 2+ / Cu has higher reduction potential than Mn 2+ / Mn
19. An aqueous solution of a substance gives a white precipitate (measured under similar conditions)
on treatment with dilute hydrochloric acid, which dissolves 28. The reagent(s) that can selectively precipitate S2 − from a
on heating. When hydrogen sulphide is passed through the mixture of S2 − and SO24 − in aqueous solution is (are)
hot acidic solution, a black precipitate is obtained. The (2016 Adv.)
substance is a (2000, 1M) (a) CuCl 2 (b) BaCl 2
(a) Hg2+
2 salt (b) Cr 2+ salt (c) Ag+ salt (d) Pb2+ salt (c) Pb(OOCCH3 )2 (d) Na 2[ Fe(CN)5 NO]
20. In nitroprusside ion the iron and NO exist as Fe (II) and NO+ 29. The pair(s) of ions where both the ions are precipitated upon
rather than Fe (III) and NO. These forms can be passing H2S gas in presence of dilute HCl, is (are)
differentiated by (1998, 2M) (2015 Adv.)
(a) estimating the concentration of iron (a) Ba 2+ , Zn 2+ (b) Bi 3+ , Fe3+
(b) measuring the concentration of CN
(c) Cu2+ , Pb2+ (d) Hg2+ , Bi 3+
(c) measuring the solid state magnetic moment
(d) thermally decomposing the compound 30. For the given aqueous reaction which of the statement(s) is
Dilute H2 SO4
21. An aqueous solution FeSO4 ⋅ Al 2 (SO4 )3 and chrome alum is (are) true? Excess KI + K 3 [Fe(CN)6 ] →
heated with excess of Na 2 O2 and filtered. The materials Brownish-yellow solution
obtained are (1996, 1M) ↓ ZnSO4
(a) a colourless filtrate and a green residue
(White precipitate + Brownish-yellow filtrate)
(b) a yellow filtrate and a green residue
↓ Na 2 S2 O3
(c) a yellow filtrate and a brown residue
(d) a green filtrate and brown residue Colourless solution (2012)
296 Qualitative Analysis

(a) The first reaction is a redox reaction M 2 always forms tetrahedral complexes with these reagents.
(b) White precipitate is Zn 3[ Fe(CN)6 ]2 Aqueous solution of M 2 on reaction with reagent S gives white
(c) Addition of filtrate to starch solution gives blue colour precipitate which dissolves in excess of S. The reactions are
(d) White precipitate is soluble in NaOH solution summarised in the scheme given below
Q R
31. A solution of colourless salt H on boiling with excess NaOH Tetrahedral excess M1 excess Square planar
produces a non-flammable gas. The gas evolution ceases
Q R
after sometime. Upon addition of Zn dust to the same Tetrahedral M2 excess Tetrahedral
excess
solution, the gas evolution restarts. The colourless salt(s) H
is (are) (2008, 4M) S, stoichiometric amount
(a) NH4NO3 (b) NH4NO2 (c) NH4Cl (d) (NH4 )2 SO4
S
White precipitate Precipitate
32. Which of the following statement(s) is(are) correct when a excess
dissolves
mixture of NaCl and K 2 Cr2 O7 is gently warmed with
conc. H2 SO4 ? (1998, 2M) 37. M1 , Q and R, respectively are
(a) A deep red vapour is evolved (a) Zn 2+ , KCN and HCl (b) Ni 2+ , HCl and KCN
(b) The vapour when passed into NaOH solution gives a (c) Cd 2+
, KCN and HCl (d) Co2+ , HCl and KCN
yellow solution of Na 2CrO4
(c) Chlorine gas is evolved
38. Reagent S is
(a) K 4[Fe(CN)6 ] (b) Na 2HPO4
(d) Chromyl chloride is formed
(c) K 2CrO4 (d) KOH
33. Which of the following statement(s) is (are) correct with
reference to the ferrous and ferric ions ? (1998, 2M) Passage 2
(a) Fe3+ gives brown colour with potassium ferricyanide An aqueous solution of a mixture of two inorganic salts,when
(b) Fe2+ gives blue precipitate with potassium ferricyanide treated with dilute HCl, gave a precipitate (P) and filtrate (Q). The
(c) Fe3+ gives red colour with potassium thiocyanate precipitate (P) was found to dissolve in hot water. The filtrate (Q)
remained unchanged, when treated with H2 S in a dilute mineral acid
(d) Fe2+ gives brown colour with ammonium thiocyanate
medium. However, it gave a precipitate (R) with H2 S in an
34. The reagents, NH4 Cl and aqueous NH3 will precipitate ammoniacal medium. The precipitate R gave a coloured solution
(1991, 1M) (S ), when treated with H2 O2 in an aqueous NaOH medium.
(a) Ca 2+ (b) Al 3+ (c) Bi 3+ (d) Mg2+
39. The precipitate P contains (2013 Adv.)
(e) Zn 2+ (a) Pb2+ (b) Hg2+ (c) Ag+ (d) Hg2+
2

Assertion and Reason 40. The coloured solution S contains


(a) Fe2 (SO4 )3 (b) CuSO4 (c) ZnSO4 (d) Na 2CrO4
Read the following questions and answer as per the direction
given below : Passage 3
(a) Statement I is correct Statement II is correct Statement II is When a metal rod M is dipped into an aqueous colourless
a correct; explanation of Statement I concentrated solution of compound N , the solution turns light blue.
(b) Statement I is correct; Statement II is correct Statement II is Addition of aqueous NaCl to the blue solution gives a white
not the correct explanation of Statement I. precipitate O. Addition of aqueous NH 3 dissolves O and gives an
(c) Statement I is correct; Statement II is incorrect. intense blue solution. (2011)
(d) Statement I is incorrect; Statement II is correct. 41. The metal rod M is
35. Statement I Sulphate is estimated as BaSO4 , not as MgSO4 . (a) Fe (b) Cu (c) Ni (d) Co

Statement II Ionic radius of Mg 2+ is smaller than that 42. The compound N is


2+
(a) AgNO3 (b) Zn(NO3 )2
of Ba . (1998, 2M) (c) Al(NO3 )3 (d) Pb(NO3 )2
36. Statement I A very dilute acidic solution of Cd 2+ and 43. The final solution contains
Ni 2+ gives yellow precipitate of CdS on passing H2 S. (a) [Pb(NH3 )4 ]2+ and [CoCl 4 ]2−
Statement II Solubility product of CdS is more than that (b) [Al(NH3 )4 ]3+ and [Cu(NH3 )4 ]2+
of NiS. (1989, 2M) (c) [Ag(NH3 )2 ]+ and [Cu(NH3 )4 ]2+
(d) [Ag(NH3 )2 ]+ and [Ni(NH3 )6 ]2+
Passage Based Questions
Passage 1 Passage 4
An aqueous solution of metal ion M 1 reacts separately with p-amino-N, N-dimethylaniline is added to a strongly acidic
reagents Q and R in excess to give tetrahedral and square planar solution of X. The resulting solution is treated with a few drops of
complexes, respectively. An aqueous solution of another metal ion aqueous solution of Y to yield blue colouration due to the formation
Qualitative Analysis 297

of methylene blue. Treatment of the aqueous solution of Y with the 54. A1 and A2 are two ores of metal M. A1 on calcination gives
reagent potassium hexacyanoferrate (II) leads to the formation of black precipitate, CO2 and water.
an intense blue precipitate. The precipitate dissolves on excess Calcination
addition of the reagent. Similarly, treatment of the solution of Y → Black solid + CO2 + OH2
with the solution of potassium hexacyanoferrate (III) leads to a A1 
brown colouration due to the formation of Z. dil. HCl
→ I2 + ppt
44. The compound X, is (2009, 1M) KI
(a) NaNO3 (b) NaCl Roasting
(c) Na 2SO4 (d) Na 2S
A2 → Metal + Gas

45. The compound Y, is (2009, 1M) K 2Cr2O7 + H2SO4


(a) MgCl 2 (b) FeCl 2 Green colour
(c) FeCl 3 (d) ZnCl 2
Identify A1 and A2 . (2004)
46. The compound Z, is (2009, 1M)
55. A salt mixture consists of a yellow solid (A) and a colourless
(a) Mg2[Fe(CN)6 ] (b) Fe[Fe(CN)6 ] solid (B). The aqueous solution of the mixture
(c) Fe4[Fe(CN)6 ]3 (d) K 2 Zn 3[Fe(CN)6 ]2
(i) On passing H2S, we get a black precipitate of (C), which
dissolves only in aqua-regia. On extraction and reaction
Fill in the Blanks with SnCl2 a greyish white precipitate is obtained.
47. The formula of the deep red liquid formed on warming (ii) On treatment with ammonium hydroxide a reddish brown
dichromate with KCl in concentrated sulphuric acid is... precipitate (D) is obtained.
(1993, 1M) The sodium extract of the solution gives the following tests:
48. If metal ions of group III are precipitated by NH4 Cl and (i) On reaction with AgNO3 it gives a yellow precipitate which
NH4 OH without prior oxidation by conc. HNO3 . …… is not is insoluble in NH3.
completely precipitated. (1984, 1M) (ii) On shaking with FeCl3 and CCl4 a violet colouration in CCl4
layer is obtained.
True/False Mixture on performing flame test gives lilac colour. Identify
49. From the solution containing copper (+2) and zinc (+2) ions, the compounds (A), (B), (C) and (D). (2003)
copper can be selectively precipitated using sodium 56. When a white crystalline compound X is heated with
sulphide. (1987, 1M) K 2 Cr2 O7 and concentrated H2 SO4 , a reddish brown gas A is
50. Addition of ammonium chloride to a solution containing evolved. On passing A into caustic soda solution, a yellow
ferric and magnesium ions is essential for selective coloured solution B is obtained. Neutralising the solution of
precipitation of ferric hydroxide by aqueous ammonia. B with acetic acid and on subsequent addition of lead acetate
(1985, 1/2M) a yellow precipitate C is obtained.
When X is heated with NaOH solution, colourless gas is
Integer Answer Type Question evolved and on passing this gas into K 2 HgI4 solution, a
51. Among PbS, CuS, HgS, MnS, Ag 2 S, NiS, CoS, Bi 2 S3 and reddish brown precipitate D is formed. Identify A, B, C, D and
SnS2 the total number of black coloured sulphides is X. Write the equations of reactions involved. (2002)
(2014 Adv.) 57. A white substance A reacts with dilute H2 SO4 to produce a
colourless gas B and a colourless solution C. The reaction
Subjective Questions between B and acidified K 2 Cr2 O7 solution produces a green
moist air Zn solution and a slightly coloured precipitate D. The substance
52. B ← MCl 4 → A D burns in air to produce a gas E which reacts with B to yield
(White fumes M = ( Transition (Purple
having smell) element colourless) colour) D and a colourless liquid. Anhydrous copper sulphate is
turned blue on addition of this colourless liquid. Addition of
Identify the metal M and hence MCl 4 . Explain the difference
aqueous NH3 or NaOH to C produces first a precipitate,
in colours of MCl 4 and A. (2005, 4M)
which dissolves in the excess of the respective reagent to
SCN− (excess) produce a clear solution in each case. Identify A, B, C, D and
53. Fe3+ → Blood red (A)
E. Write the equations of the reactions involved. (2001, 10M)
F − (excess)
→ Colourless ( B) 58. Write the chemical reactions associated with the ‘borax
bead test’ of cobalt (II) oxide. (2000, 3M)
Identify A and B.
(i) Write IUPAC name of A and B. 59. An aqueous blue coloured solution of a transition metal
(ii) Find out spin only magnetic moment of B. (2005)
sulphate reacts with H2 S in acidic medium to give a black
precipitate A, which is insoluble in warm aqueous solution
298 Qualitative Analysis

of KOH. The blue solution on treatment with KI in weakly 69. The acidic aqueous solution of ferrous ion forms a brown
acidic medium, turns yellow and produces a white complex in the presence of NO−3 , by the following two
precipitate B. Identify the transition metal ion. Write the steps :
chemical reactions involved in the formation of A and B.
(2000, 4M) [Fe(H2 O)6 ]2+ + NO–3 + H+ → K + [Fe(H2 O)6 ]3+
60. Write the chemical reactions associated with the ‘brown [Fe(H2 O)6 ]2+ + K → ...... + H2 O
ring test’. (2000, 1M)
Complete and balance the equation. (1993, 2M)
61. An aqueous solution containing one mole of HgI2 and two
70. A light bluish green crystalline compound responds to the
moles of NaI is orange in colour. On addition of excess NaI
following tests
the solution becomes colourless. The orange colour
reappears on subsequent addition of NaOCl. Explain with (i) Its aqueous solution gives a brown precipitate or
colouration with K2[HgI4 ].
equations. (1999, 3M)
(ii) Its aqueous solution gives a blue colour with K3[Fe(CN)6 ].
62. During the qualitative analysis of a mixture containing Cu 2+ (iii) Its solution in hydrochloric acid gives a white precipitate
and Zn 2+ ions, H2 S gas is passed through an acidified with BaCl 2.
solution containing these ions in order to test Cu 2+ alone. Identify the ions present and suggest the formula of the
Explain briefly. (1998, 2M) compound. (1992, 4M)
63. Aluminium sulphide gives a foul odour when it becomes 71. In the following reaction, identify the compounds/reaction
damp. Write a balanced chemical equation for the reaction. conditions represented by the alphabets A and B.
(1997) Heat in B
PbS → A + PbS → Pb + SO2
64. A soluble compound of a poisonous element M, when air (1991, 1M)
heated with Zn/H2SO4 gives a colourless and extremely
72. Give reason in one or two sentences for the following
poisonous gaseous compound N, which on passing through
“The hydroxides of aluminium and iron are insoluble in
a heated tube gives a silvery mirror of element M. Identify M
water. However, NaOH is used to separate one from other.
and N. (1997)
(1991, 2M)
65. A colourless inorganic salt (A) decomposes completely at 73. The gas liberated, on heating a mixture of two salts with
about 250° C to give only two products (B) and (C), leaving NaOH, gives a reddish brown precipitate with an alkaline
no residue. The oxide (C) is a liquid at room temperature and solution of K 2 HgI4 . The aqueous solution of the mixture on
neutral to moist litmus paper, while the gas (B) is a neutral treatment with BaCl 2 gives a white precipitate which is
oxide. sparingly soluble in conc. HCl.
White phosphorus burns in excess of (B) to produce a strong On heating the mixture with K 2 Cr2 O7 and conc. H2 SO4 , red
white dehydrating agent. Write balanced equations for the vapours A are produced. The aqueous solution of the
reactions involved in the above process. (1996, 3M) mixture gives a deep blue colouration B with potassium
66. Gradual addition of KI solution of Bi(NO3 )3 solution ferricyanide solution. Identify the radicals in the given
initially produces a dark brown precipitate which dissolves mixture and write the balanced equations for the formation
in excess of KI to give a clear yellow solution. Write of A and B. (1991, 4M)
chemical equations for the above reactions. (1996, 2M)
74. Write the balanced chemical equations for the following
67. A scarlet compound A is treated with conc. HNO3 to give a (i) Silver chloride is treated with aqueous sodium cyanide and
chocolate brown precipitate B. The precipitate is filtered the product thus formed is allowed to react with zinc in
and the filtrate is neutralised with NaOH. Addition of KI to alkaline medium.
the resulting solution gives a yellow ppt C. The brown ppt B (ii) Cobalt (II) solution reacts with KNO2 in acetic acid
on warming with conc. HNO3 in the presence of Mn(NO3 )2 medium. (1989, 2M)
produces a pink coloured solution due to the formation of D. 75. Give reasons for, “The colour of mercurous chloride,
Identify A, B, C and D. Write the reaction sequence. Hg 2 Cl 2 , changes from white to black when treated with
(1995, 4M) ammonia.” (1988, 1M)
68. An orange solid A on heating gave a green residue B, a 76. A mixture of two salts was treated as follows :
colourless gas C and water vapour. The dry gas C on passing
(i) The mixture was heated with maganese dioxide and
over heated Mg gave a white solid D. D on reaction with concentrated sulphuric acid, when yellowish green gas was
water gave a gas E which formed dense white fumes with liberated.
HCl. Identify A to E and give the reaction involved. (ii) The mixture on heating with sodium hydroxide solution
(1993, 3M)
gave a gas which turned red litmus blue.
Qualitative Analysis 299

(iii) Its solution in water gave blue precipitate with potassium and Y and write down the equation for the thermal
ferricyanide and red colouration with ammonium decomposition of X. (1984, 4M)
thiocyanate.
(iv) The mixture was boiled with potassium hydroxide and the 82. Compound A is a light green crystalline solid. It gives the
liberated gas was bubbled through an alkaline solution of following tests
K2HgI4 to give brown precipitate. (i) It dissolves in dilute sulphuric acid. No gas is produced.
Identify the two salts. Give ionic equations for reactions (ii) A drop of KMnO4 is added to the above solution. The pink
involved in the tests (i), (ii) and (iii). (1987, 5M) colour disappears.
77. Write balanced equation for the following “potassium (iii) Compound A is heated strongly. Gases B and C, with
permanganate is reacted with warm solution of oxalic acid pungent smell, come out. A brown residue D is left behind.
in the presence of sulphuric acid.” (1987, 1M) (iv) The gas mixture (B and C) is passed into a dichromate
solution. The solution turns green.
78. Mention the products formed in the following:
(v) The green solution from step (iv) gives a white precipitate E
(i) Zinc oxide is treated with excess of sodium hydroxide
with a solution of barium nitrate.
solution.
(ii) Iodine is added to a solution of stannous chloride.
(vi) Residue D from step (iii) is heated on charcoal in a reducing
(iii) Sulphur dioxide gas, water vapour and air are passed over
flame. It gives a magnetic substance.
heated sodium chloride. (1986, 3M)
Name the compound A, B, C, D and E. (1980, 4M)
79. What happen when
(i) hydrogen sulphide is bubbled through an aqueous solution
83. Explain the following in not more than two sentences.
of sulphur dioxide. A solution of FeCl 3 in water gives a brown precipitate on
standing. (1980, 1M)
(ii) aqueous ammonia is added dropwise to a solution of copper
sulphate till it is in excess. 84. The precipitation of second group sulphides in qualitative
(iii) tin is treated with concentrated nitric acid. analysis is carried out with hydrogen sulphide in the presence
(iv) CrCl 3 solution is treated with sodium hydroxide and then of hydrochloric acid but not in nitric acid. Explain. (1979, 2M)
with hydrogen peroxide. 85. A white amorphous powder A on heating yields a colourless,
(v) Pb3O4 is treated with nitric acid. (1985, 5M) non-combustible gas B and a solid C. The later compound
80. Write down the balanced equations for the reactions, when, assumes a yellow colour on heating and changes to white on
‘a mixture of potassium chlorate, oxalic acid and sulphuric cooling. C dissolves in dilute hydrochloric acid and the
acid is heated. (1985, 1M) resulting solution gives a white precipitate with K 4 Fe(CN)6
solution. A dissolves in dil. HCl with the evolution of gas,
81. When 16.8 g of white solid, X were heated, 4.4 g of acid gas
which is identical in all respect with B.
A, that turned lime water milky was driven off together with
1.8 g of a gas B which condensed to a colourless liquid. The gas B turns lime water milky, but the milkiness
disappears with the continuous passage of gas. The solution
The solid that remained, Y, dissolved in water to give an of A as obtained above, gives a white ppt E on addition of
alkaline solution, which with excess barium chloride NaOH solution, which dissolves on further addition of base.
solution gave a white precipitate Z. The precipitate Identify the compounds A, B, C, D and E. (1979, 4M)
effervesces with acid giving of carbon dioxide. Identify A, B

Answers
1. (a) 2. (c) 3. (b) 4. (c) 29. (c, d) 30. (a, c, d) 31. (a, b) 32. (a, b, c, d)
5. (c) 6. (a) 7. (a) 8. (d) 33. (b, c) 34. (b, c) 35. (b) 36. (c)
9. (a) 10. (b) 11. (a) 12. (b) 37. (b) 38. (d) 39. (d) 40. (d)
13. (a) 14. (b) 15. (a) 16. (b) 41. (b) 42. (a) 43. (c) 44. (d)
17. (a) 18. (c) 19. (d) 20. (c) 45. (c) 46. (b) 47. (CrO2Cl 2 ) 48. (Fe3+ )
21. (c) 22. (a) 23. (a) 24. (c) 49. (True) 50. (True) 51. (6 or 7) 53. (5.92 BM)
25. (c) 26. (c) 27. (b,d) 28. (a)
Hints & Solutions
1. X : Ag Y : Pb P : AgCl Q : PbCl 2 4. m-cresol is the organic compound that shows the following
NaCl solubility profile.
AgNO 3 + Pb(NO 3) 2 AgCl (↓) + PbCl2
Water
(P) ( Q) Insoluble
CH3
5% HCl
Heated and filtered Insoluble
10% NaOH
AgCl (P) PbCl2 (Q) Soluble
m-cresol
OH
(White ppt.) (Hot solution) 10% NaHCO3
KI Insoluble
PbI2
(Yellew ppt.) m-cresol on reaction with 10% NaOH forms 3-methyl
AgCl(s) + 2NH3 [Ag(NH3)2]+Cl– sodiumphenoxide ion.
(P) (Excess)
2N2S2O3 OH O–Na+

Na3[Ag(S2O3)2] + NaCl 10% NaOH


Soluble complex salt
CH3 CH3
Hence, the correct option is (a).
Dil.H2SO4 NaOH SO2 + H2 O It does not react with H2O, 5% HCl and 10% NaHCO3.
2. Na 2SO3 → SO2 → Na 2SO3 → NaHSO3 Oleic acid (C18 H34O2 ) is soluble in 10% NaOH and 10%
(X) (Y) (X) (Z)
NaHCO3 due to the presence of COOH group.
Here, X , Y , Z are O
X = Na 2SO3 , Y = SO2, Z = NaHSO3 
Na 2SO3 react with dil.H2SO4 form sulphur dioxide (SO2) gas. Benzamide (C6H5C NH2) is insoluble in 5% HCl, 10% NaOH
SO2 on reaction with NaOH again form Na 2SO3 . By the and 10% NaHCO3 due to the presence CONH2 group.
reaction of SO2 and H2O, Na 2SO3 form NaHSO3 . o-toluidine is soluble in 5%. HCl due to presence of basic group
Here, X , Y , Z are Na 2SO3 , SO2, NaHSO3 respectively. (  NH2 ) attached to ring.
3. (A) [Pt(NH3 )3 Cl]+ : No geometrical isomerism 5. Among the given metals Al forms white gelatinous ppt. with
− NaOH.
(B) [Pt(NH3 )Cl5 ] : No geometrical isomerism
Hence, the probable metal can be Al. This ppt. is dissolved in
(C) [Pt(NH3 )2Cl (NO2)] : Has 2 geometrical isomers excess of NaOH due to the formation of sodium metal
(D) [Pt(NH3 )4ClBr ]2+ : exhibit geometrical isomerism. Aluminate. Both the reactions are shown below.
All Pt2+ complexes with coordination number equals to 4 are Al 3+ NaOH
→ Al(OH)3 Excess
 of NaOH
→ NaAlO2
White gelatinous Sodium
square planar. ppt. (X) of metaaluminate)
soluble
∴ The two NH3 ligands could arrange in cis or trans positions. aluminium
hydroxide
∴ There are two geometrical isomers. Aluminium hydroxide on strong heating gives alumina (Al 2O3 )
NH3 NH3 NH3 NO2 which is used as an adsorbent in chromatography. This reaction
Pt Pt can be seen as :
Cl NO2 Cl NH3 ∆
2Al(OH)3 → Al 2O3 + 3H2O
cis trans
(D) ) [Pt(NH3 )4ClBr] : Two geometrical isomers Coordination Thus, metal M is Al.
number = 6 Ca, being below sodium in electrochemical reactivity series,
∴ Octahedral geometry cannot displaces Na from its aqueous solution.
Cl and Br can be cis or trans Zn reacts with NaOH to form sodium zincate which is a soluble
∴ 2 geometrical isomers compound.
Fe reacts with sodium hydroxide to form tetrahydroferrate (II)
Cl Cl sodium which is again a soluble complex.
H3N NH3 Br NH3
Pt Pt 6. 2S2O23− + Ag+ → [ Ag(S2O3 )2 ]3−
H3N NH3 NH3 NH3 (Clear solution)
Br NH3 3− +
[ Ag(S2O3 )2 ] + 3Ag → 2Ag2S2O3
trans (White ppt.)

Ag2S2O3 + H2O → Ag2S + H2SO4


(Black ppt.)
Qualitative Analysis 301

7. Given, Weight of organic compound = 250 mg 18. Cl 2 + H2O → HCl + HOCl


Weight of AgBr = 141mg X

∴ According to formula of % of bromine by Carius method HCl + AgNO3 → AgCl ↓ (white) + HNO3
Atomic weight of Br 2HCl + Mg → MgCl 2 + H2 (g ) ↑
% of Br =
Molecular weight of AgBr
19. PbCl 2 is soluble in hot water and PbS (black) is formed on
Weight of AgBr
× × 100 passing H2S(g ) through acidic solution.
Weight of organic bromide
80 141 1128000 20. Fe(II) and Fe(III) will have different values of magnetic moment
∴ % of Br = × × 100 = = 24% due to different number of unpaired electrons in their d-orbitals.
188 250 47000
8. PLAN Ksp (ZnS) is very high and Zn 2 + is precipitated as ZnS by high
21. Yellow filtrate contains CrO5 and brown residue contain Fe2O3.
concentration of S 2− formed when H 2S is passed in ammoniacal
solution.
22. The total positive valency is +2 (because the only anion is
SO2−
4 ) . Therefore, oxidation state of Fe must be +1.
H 2S a Zn + + S 2− (I)
H + + OH − a H 2O (II) 23. Both Bi 3+ and Sn 4+ belongs to same analytical group II.
Reaction (I) is favoured in forward side if H+ is removed
24. CaC2O4 is insoluble in acetic acid. This distinguishes Ca 2+ from
immediately by OH− (NH4OH) .
Ba 2+ ion.
Zn 2 + + S2 − → ZnS ↓
White ppt 25. Ag+ is precipitated by HCl only while all others are precipitated
Fe3+ and Al 3+ are precipitated as hydroxide. by passing H2S in the presence of HCl.
9. In acidic medium, H2S is very feebly ionised giving very small 26. We know that,
concentration of sulphide ion for precipitation. Therefore, the ∆G° = − 2.303 RT log K
most insoluble salts CuS and HgS are precipitated only.
Also, given equilibrium is
10. Hg2+ + 2I− → HgI2 (red) –
2H2O - H3O+ + O H
HgI2 + 2KI → K2[HgI4 ]
[H+ ][OH− ] = 10−14 or K = 10− 14
Soluble
Hg2+ + Co(SCN)2 → Co[Hg(SCN)4 ] ∴ ∆G °= − 2.303 × 8.314 JK–1mol−1 × 298K × log10−14
Blue
= 79881.8J mol − 1 = 79.8kJ ≈ 80 kJ mol − 1
11. MgSO4 + NH4OH + Na 2HPO4 → Mg(NH4 )PO4 ↓
White 27. Statement wise explanation is
+ Na 2 SO4 + H2 O Statement (a) Mn 2+ produces yellow-green colour in flame
test while Cu 2+ produces bluish-green colour in flame test.
12. CuSO4+ 2KCN → Cu(CN)2 + K2SO4
Thus, due to the presence of green colour in both the cases,
CN flame test is not the suitable method to distinguish between
2Cu(CN)2 → 2Cu(CN) +  (Cyanogen) nitrate salts ofCu 2+ and Mn 2+. Hence this statement is wrong.
Unstable CN
Statement (b) Cu 2+ belong to group II of cationic or basic
CuCN + 3KCN → 3K+ + [Cu(CN)4 ]3− radicals. It gives black ppt. of CuS if H2S is passed through it in
the presence of acid (e.g HCl). Mn 2+ does not show this
13. Zn 2+ + 2H2O → Zn(OH)2 ↓ + 2H+
White property hence this can be considered as a suitable method to
distinguish between Mn 2+ and Cu 2+.
I−
14. Bi 3+
+ 3I → BiI3 ↓ → [BiI4 ]−

Hence, this statement is correct.
excess
Black Orange
solution Statement (c) In faintly basic medium when H2S is passed both
Cu 2+ and Mn 2+ forms precipitates. Thus, it is not suitable
15. Both (NH4 )2 Cr2O7 and NH4NO2 on heating gives nitrogen gas. method to distinguish between them.
16. A sodium salt of an unknown anion when treated with MgCl Hence, this statement is incorrect
gives white precipitate (MgCO3 ) only on boiling. Hence, the Statement (d) The standard reduction potential of Cu 2+/Cu is
action must be HCO−3 ion. +0.34 V while that of Mn 2+/Mn is –1.18V. This can be used to
∆ distinguish between Cu 2+ and Mn 2+. In general less
MgCl 2 + 2NaHCO3 → MgCO3 + 2NaCl + H2O + CO2
electropositive metals have higher SRP.
17. SO23− + H2SO4 → SO2 ↑ + H2O + SO42− Hence, this statement is correct.
X Y
SO2 is a colourless gas with irritating odour. 28. (a) S2– + CuCl 2 → CuS ↓ (black ppt.)
3SO2 + K2Cr2O7 + H2SO4 → K2SO4 + Cr2 (SO4 )3 + H2O SO24− + CuCl 2 → Soluble, Thus
Y Green CuCl 2 selectively precipitates S2– .
solution
302 Qualitative Analysis

(b) S2– + BaCl 2 → BaS ↓ (soluble) 34. Both Al 3+ and Bi 3+ are precipitated as their hydroxides.
SO24− + BaCl 2 → BaSO4 ↓ (white ppt.) 35. As MgSO4 is soluble in water, so not used for estimation of SO2−
4
precipitates SO2−
4 and not S . 2−
ion.
2– 2+
(c) S + Pb → PbS ↓ (black ppt.) 36. Cation Cd 2+ belongs to group II while Ni 2+ belongs to group
SO24− + Pb 2+
→ PbSO4 ↓ (white ppt.) III of analytical group. Group II radicals are precipitated by
2−
passing H2S(g ) through acidic solution of salt but radicals of
S and SO2−
4 , both are precipitated. group III are precipitated by passing H2S(g ) in NH3 /NH4Cl
2− buffer solution of salt due to greater solubility products of later
(d) S + Na 2 [ Fe (CN)]5 NO → Na 4 [ Fe (CN)]5 NOS]
Sodium nitroprusside (Purple colour) salts.
But no colour with SO2−
4 . 37. PLAN This problem can be solved by using concept of chemical
reactions of transition metal ions (,) colour and structure of
29. Only radicals of I and II group of qualitative analysis get transition metal compounds.
precipitated with H2S in the presence of dilute HCl. Here, among given four option Ni 2+ and Zn 2+ has ability to form
+
(c) Cu 2+ + H2S H
→ CuS ↓ tetrahedral as well as square planar complex depending upon
Black types of reagent used.
+
Pb2+ + H2S H
→ PbS ↓ Ni 2+ on reaction with KCN forms square planar complex
Black
+ [Ni(CN)4 ]2− due to strong field strength of CN.
(d) Hg2+ + H2S H
→ HgS ↓
Black Ni 2+ + KCN → [Ni(CN)4 ]2−
+ Square planar
Bi 3+ + H2S H
→ Bi 2S3 ↓
Brown ppt While on reaction with HCl, Ni 2+ forms stable tetrahedral
2+
Ba , Ζn 2+
and Fe 3+
are not precipitated as sulphide. complex [Ni(Cl)4 ]2−.
+3 Zn 2+ , on the other hand, on reaction with KCN as well as HCl
30. K3 [Fe(CN)6 ] + KI (excess) → produces tetrahedral complex because of its d 10 electronic
+2
K4[Fe(CN)6 ] + KI3 (redox) configuration.
HCl
Brownish yellow [ZnCl4]2–
solution
KCN
Zn2+ [Zn(CN)4]2–
K4[Fe(CN)6 ] + ZnSO4 → K2Zn 3[Fe(CN)6 ]2
KOH
or K2 Zn[Fe(CN)6 ] [Zn(OH)4]2–
excess
White ppt White ppt
Complete reaction sequence can be shown as
I−3 + 2Na 2S2O3 → Na 2 S4O6 + 2NaI + I2
Brownish Clear (Turns starch HCl(Q) KCN(R)
yellow solution solution [NiCl4]2– excess Ni2+ excess [Ni(CN)4]2–
filtrate blue) Tetrahedral (M1) Square planar
HCl(Q) KCN(R) [Zn(CN)4]2–
K2Zn[Fe(CN)6 ] reacts with NaOH as [ZnCl4]2– Zn2+ excess
excess (M2)
Tetrahedral Tetrahedral
K2Zn[Fe(CN)6 ] + NaOH → [Zn(OH)4 ]2 − + [Fe(CN)6 ] 4− KOH (s)

31. NH4NO3 + NaOH → NaNO3 + NH3 + H2O Zn(OH)2 KOH [Zn(OH)4]2–


White ppt excess
Soluble
NH4NO2 + NaOH → NaNO2 + NH3 + H2O
32. When mixture of NaCl is heated with K2Cr2O7 in concentrated 38. Zn 2+ on treatment with excess of KOH produces [Zn(OH)4 ]2− .
H2SO4, red vapour of chromyl chloride (CrO2Cl 2 ) is produced. 39, 40. PLAN PbCl 2 is soluble in hot water.
Vapours of chromyl chloride when passed through NaOH,
solution turns yellow due to formation of Na 2CrO4. Some In ammoniacal medium, cations of group III and IV may be
chlorine gas is also evolved owing to the following side reaction: precipitated as hydroxide or sulphide and dissolved in H2O2 due
to oxidation.
6Cl − + Cr2O72− + 14H+ → 3Cl 2 (g ) ↑ + 2Cr 3+ + 7H2O (i) HCl
Mixture of two inorganic salts →
33. The blue precipitate of Fe2+ ion with potassium ferricyanide is (ii) Filter
due to formation of Turnbull’s blue KFe[Fe(CN)6 ]. 

H2 O2 NH4 OH/H2 S ↓
Fe2+ + K3[Fe(CN)6 ] → KFe[Fe(CN)6 ] + 2K+ CrO24 − ← Cr(OH)3 ← Filtrate ppt
3+ Yellow NaOH ppt
The red colour of Fe ion with potassium thiocyanate is due to
(S )
formation of [Fe(SCN)3 ].
Thus, Q. 38. P is Pb2+ . Q. 39. S is Na 2CrO4.
Fe3+ + 3KSCN → [Fe(SCN)3 ] + 3K +
Red coloured
Qualitative Analysis 303

41. Appearance of blue colour on addition of the metal rod M to the ⇒ M = Ti, A = [Ti(H2O)6 ]3+ ; B = TiO2
solution of N is an indication that metal may be Cu. Later Ti (IV) contains no d-electron, while d-d transition of single
formation of white precipitate on addition of NaCl and electron of Ti (III) will cause colour change.
dissolution of this precipitate in aqueous ammonia confirm that
while precipitate is of AgCl. This implies that the solution N is 53. (i) Fe3+ + 3SCN− → Fe(SCN)3 (blood red colouration)
Iron (III)
of AgNO3. This confirm that the metal M is only Cu. thiocyanate
A
42. The compound N is AgNO3, explained above.
Fe(SCN)3 + F− (excess) → [FeF6 ]3− + 3SCN−
43. Since, in the beginning, concentrated AgNO3 solution was (Hexafluoroferrate)
taken, some AgNO3 remain in solution which gives white B
precipitate of AgCl on addition of NaCl. The precipitate finally (ii) Magnetic moment (µ S ) = n (n + 2) BM
dissolve in ammonia and Cu2+ present in solution also forms
complex with ammonia as = 35 BM = 5.92 BM
54. A1 = Cu(OH)2 ⋅ CuCO3 and A2 = Cu 2S
Cu (NO3 )2 + 4NH3 (aq) → [ Cu(NH3 )4 ] 2+ + 2NO3−
Calcination
Blue
+ –
A1 → 2CuO + CO2 + H2O
AgCl(s) + 2NH3 (aq) → [Ag(NH3 )2 ] + Cl Black
Dil. HCl
The comprehension describing methylene-blue test. A1 → CuCl 2 + CO2 + H2O
NMe2
2CuCl 2 + 4KI → 2CuI + I2 + 4KCl
2Cu 2S ( A2 ) + 3O2 → 2Cu 2O + 2SO2
S2– + 2 + Fe(III) ∆
Cu 2S + 2Cu 2O → 6Cu + SO2
55. A + B → lilac colour in flame
NH2 123
Mixture
+
Me2N S NMe2 H2 S
(i) A + B → C ↓
123 Black
Mixture
N Soluble Evaporation
C → Soluble → Residue
Blue solution in aqua-regia
SnCl 2
Therefore, → Greyish black ↓ (D )
44. X is Na 2S. ⇒ C is HgS.
45. Y is FeCl 3. NH4 OH
(ii) A+ B → Brown ppt.
123
46. Compound Z is Fe[Fe(CN)6 ]. Solution of
mixture
47. Heating chloride salt with K2Cr2O7 in conc. H2SO4 gives off a AgNO NH
deep vapour of chromyl chloride (CrO2Cl 2 ) . Sodium extract →
3
Yellow ppt. →
3
Insoluble

48. Because Fe(OH)2 is soluble. CCl 4 /FeCl 3


Sodium extract of salt → Violet layer
49. ZnS is soluble in dil. HCl but CuS does not dissolve in dil. HCl. ⇒ A = KI and B = HgI2.
50. In absence of NH4Cl, both Fe(OH)3 and Mg(OH)2 will be 56. X = NH4Cl; A = CrO2Cl 2; B = Na 2CrO4; C = PbCrO4;
precipitated.
D = H2N(HgO)HgI.
51. From qualitative analysis of the different metal ions it is found
that PbS, CuS, HgS, Ag2S, NiS, CoS are black coloured. Reactions involved :

MnS — dirty pink/buff coloured, SnS2—yellow coloured. (i) 4NH4Cl ( X ) + K2Cr2O7 + 6H2SO4 → 2CrO2Cl 2 ( A )

Bi 2S3 — brown/black (brownish black) coloured. + 4NH4HSO4 + 2KHSO4 + 3H2O


(ii) CrO2Cl 2 ( A ) + 2NaOH → Na 2CrO4 ( B ) + 2HCl
Hence, correct integer is (6 or 7).
(iii) Na 2CrO4 ( B ) + (CH3COO)2 Pb → PbCrO4 ↓ (C )
Zn
52. MCl 4 → Purple coloured compound ( A ) + 2CH3COONa
Colourless
(iv) NH4Cl ( X ) + NaOH → NaCl + H2O + NH3
Moist
M → B (white fumes) (v) 2K2HgI4 + NH3 + 3KOH → H2N(HgO)HgI (D )
Transition metal Air
+ 7KI + 2H2O
304 Qualitative Analysis

57. Since, the white substance A gives a colourless gas B with 62. K sp (solubility product) of CuS is less than K sp of ZnS. On
dil. H2SO4, such gas may be H2S. So, the substance A may be a passing H2S(g ) in acidic solution, dissociation of H2S is
metal sulphide (Na/K/Zn, etc.) suppressed due to common ion effect and it provide small
When H2S gas reacts with acidified K2Cr2O7 , it gives green concentration of S2− , sufficient for precipitation of CuS but
coloured solution of Cr2 (SO4 )3 alongwith slightly yellow ppt of insufficient for precipitation of ZnS.
D as sulphur.
63. Al 2S3 + 6H2O → 2Al(OH)3 ↓ + 3H2S ↑
K2Cr2O7 + 4H2SO4 + 3H2S → K2SO4 + Cr2 (SO4 )3
Foul odour
Green
+ 3S + 7H2 64. The poisonous element M may be As. On the basis of given
D information :
S on burning in air gives SO2 (E). Substance E on reaction with Zn
B (H2S) produces D (s) : AsCl 3 + 6H → AsH3 + 3HCl
H2 SO4 N
2H2S + SO2 → 2H2O (l ) + 3S ↓

B E D 2AsH3 → 2As + 3H2 ↑
Anhydrous CuSO4 produces blue colour in water. M
Solution C produces ppt first with NH3 /NaOH which dissolve in 250 ° C
65. A → B + C
excess NH3 /NaOH. Hence, A must be ZnS. Inorganic salt Neutral Neutral oxide
ZnS + dil. H2SO4 → ZnSO4 (aq) + H2S(g ) oxide (liquid)
A C Oxide C is liquid and neutral to litmus, so it is H2O. White
ZnSO4 + 2NaOH → Zn(OH)2 ↓ + Na 2SO4 phosphorus burns in excess of B to give P4O10.
C White Therefore, B is N2O.
Zn(OH)2 + 2NaOH → [Zn(OH)4 ]2− + 2Na + ∆
NH4NO3 ( A ) → N2O + 2H2O (l )
∆ B
58. Na 2B4O7 → 2NaBO2 + B2O3
P4 + 10N2O → P4O10 + 10N2

CoO + B2O3 → Co(BO2 )2
Blue
66. On addition of KI solution to a Bi(NO3 )3 solution, firstly a dark
brown precipitate of BiI3 is formed that dissolve in excess of KI
59. The transition metal is Cu 2+ . The compound is CuSO4 ⋅ 5H2O. It forming a clear yellow solution of BiI−4 :
dissolves in water to give blue coloured solution due to presence
I−
of Cu 2+ . On passing H2S(g ) in acid medium of salt solution Bi 3+ + 3I− → BiI3 ↓ → BiI−4
black precipitate of CuS is obtained which is not soluble in Dark Clear yellow
brown solution
aqueous KOH solution.
H+ C⋅ HNO3 Filtered
CuSO4 + H2S → CuS ↓ + H2SO4 67. A (Pb3O4 ) → B (PbO2 ) → Pb(NO3 )2
black Scarlet Chocolate Filtrate

On adding KI solution to aqueous solution of CuSO4, yellow Filtrate is neutralised with NaOH and on reaction with KI, gives
solution of CuI2 is formed in the beginning which decompose yellow ppt of PbI2.
into white ppt of CuI. Pb(NO3 )2 + 2KI → PbI2 ↓ + 2KNO3
CuSO4 + 2KI → CuI2 + K2SO4 Yellow

2CuI2 → 2CuI ↓ + I2 PbO2 on warming with conc. HNO3 in presence of Mn(NO3 )2


White produced pink solution due to formation of Pb(MnO4 )2 (I).
5PbO2 + 2Mn(NO3 )2 + 4HNO3 → Pb(MnO4 )2
60. NaNO3 + H2SO4 → NaHSO4 + HNO3
+ 4Pb(NO3 )2 + 2H2O
2HNO3 + 6FeSO4 + 3H2SO4 → 3Fe2 (SO4 )3 + 2NO + 2H2O
⇒ A = Pb3O4, B = PbO2 , C = PbI2 and D = Pb(MnO4 )2.
FeSO4 + NO → [Fe(NO)SO4 ]
Brown ring ∆
(nitrosoferrous sulphate)
68. A → B ↓ + C↑ + H2O (v ) ↑
Orange solid Green Colourless
61. NaI on reaction with HgI2 gives a complex salt :
C + Mg → D (white solid)
2NaI + HgI2 r Na 2[HgI4 ]
HCl
The orange colour is due to residual HgI2. On addition of excess D + H2O → E (g ) → White fumes.
NaI, whole HgI2 is converted to complex Na 2[HgI4 ] as colour
disappear. The orange colour of HgI2 reappear due to conversion Hence, E must be ammonia gas so D must be Mg3N2 and C is
of Na 2[HgI4 ] into HgI2 on treatment with NaOCl. N2 (g ). This N2 is obtained on strong heating of (NH4 )2 Cr2O7
because (NH4 )2 Cr2O7 is orange solid and produces green Cr2O3
3Na 2[HgI4 ] + 2NaOCl + 2H2O → 3HgI2 + 2NaCl residue on heating.
+ 4 NaOH + 2NaI
Qualitative Analysis 305

∆ 75. Mercurous chloride changes from white to black when treated


(NH4 )2 Cr2O7 → Cr2O3 + N2 + 4 H2O
Orange (A) Green (B) C
with ammonia due to formation of metallic Hg.
∆ H2 O 76. (i) Mixture + MnO2 + H2SO4 → Yellowish green gas.
N2 + Mg → Mg3N2 → NH3 + Mg(OH)2 conc.
The above reaction suggest that the mixture contain Cl − .
D E

NH3 + HCl → NH4Cl (white fumes) 2Cl − + MnO2 + H2SO4 → MnSO4 + Cl 2 ↑ + H2O
E (ii) Mixture + NaOH → Gas turning red litmus blue.
69. 3[Fe(H2O)6 ]2+ + NO−3 + 4H+ → NO + 3[Fe(H2O)6 ]3+ The above reaction indicates that the gas is ammonia.
+ 2H2O NH+4 + NaOH → NH3 + H2O + Na +
[Fe(H2O)6 ]2+ + NO → [Fe(H2O)5 NO]2+ + H2O (iii) Solution of mixture + K3[Fe(CN)6 ] → blue ppt.
70. (i) Brown precipitate or colour with K2HgI4, indicates the The above reaction suggest that mixture contain a Fe(II) salt.
Fe(II) salt react with K3[Fe(CN)6 ] to give blue ppt of Prussian
presence of NH+4 ion.
blue complex
(ii) Aqueous Fe(II) solution gives a blue colour with
K3[Fe(CN)6 ] due to formation of Fe[Fe(CN)6 ]− ion : 3Fe2+ + 2Fe(CN)63− → Fe3[Fe(CN)6 ]2 ↓
Blue
Fe2+ + Fe(CN)63− → Fe[Fe(CN)6 ]− Red colouration with NH4SCN suggests that some Fe(III) is
Blue colouration also present. It is likely that a part of Fe(II) is oxidised to
(iii) Aqueous solution of SO2−
4 ion in HCl gives white ppt with
Fe(III) by air :
BaCl 2 solution. Fe3+ + 3NH4SCN → Fe(SCN)3 + 3NH+4
SO24− + BaCl 2 → BaSO4 ↓ + 2Cl − Red colouration
K 2 HgI 4
Heat in Heat in (iv) Mixture + KOH → gas ↑ → brown ppt.
71. PbS → PbO + PbS → Pb + SO2
air A absence of air The above reaction indicates that the gas is NH3, i.e. mixture
contain NH+4 ion. Hence, the mixture contains Fe2+ , NH+4 and
72. In excess of NaOH, Al(OH)3 dissolves forming NaAlO2 while
Cl − ions with some impurity of Fe3+ ion.
Fe(OH)3 remains insoluble.
The two salts are FeCl 2 and NH4Cl.
73. Formation of a reddish-brown precipitate with alkaline K2HgI4
solution indicates the presence of NH+4 ion and the gas
77. 2KMnO4 + 3H2SO4 + 5H2C2O4 → K2SO4 + 2MnSO4
liberated is ammonia : + 8H2 O + 10CO2

NH+4 + OH → H2O + NH3 (g ) 78. (i) ZnO + 2NaOH → Na 2ZnO2 + H2O
NH2 (ii) Sn 2+ + I2 → Sn 4+ + 2I−
Hg
2K2HgI4 + 3NaOH + NH+4 → ↓ 1
O (iii) SO2 + H2O + O2 + 2NaCl → Na 2SO4 + 2HCl
Hg 2
I
79. (i) SO2 + 2H2S → 3S + 2H2O
+ 4KI + 3NaI + 3OH − (ii) CuSO4 + 4NH3 → [Cu(NH3 )4 ]SO4

On treatment with BaCl 2, a white precipitate is formed which (iii) Sn + 3HNO3 → H2SnO3 + 4NO2 + H2O
Conc. Metastannic
indicates the presence of SO2−
4 anion : acid
SO24− + BaCl 2 → BaSO4 ↓ + 2Cl − (iv) H2O2 + CrCl 3 + NaOH → Na 2CrO4 + NaCl + H2O
White Yellow

With K2Cr2O7 and conc. H2SO4, red vapour of CrO2Cl 2 is (v) Pb3O4 → 2PbO + PbO2
evolved. This indicates presence of Cl − ion. 2PbO + PbO2 + 4HNO3 → 2Pb(NO3 )2 + 2H2O+ PbO2 ↓
On treatment with potassium ferricyanide, formation of deep 80. 3KClO3 + 3H2SO4 → 3KHSO4 + HClO4 + 2ClO2 + H2O
blue solution indicates presence of Fe (II) ion :
(COOH)2 + C ⋅ H2SO4 → CO + CO2 + H2O
Fe2+ + Fe(CN)63− → Fe4[Fe(CN)6 ]3
Blue 81. (X) is NaHCO3 (M = 84). The reactions involved are :
74. (i) AgCl + 2NaCN → NaCl + Na[Ag(CN)2 ] 2NaHCO3 → Na 2CO3 (s) + CO2 (g ) + H2O (g )
2Na[Ag(CN)2 ] + Zn → Na 2[Zn(CN)4 ] + 2Ag X Y A B
168 g 106 g 44 g 18 g
(ii) Co2+ + 3KNO3 + 4NO2− + 2H+ → K3[Co(NO2 )6 ]
⇒ 16.8 g 10.6 g 4.4 g 1.8 g
CO2 + Ca(OH)2 → CaCO3 ↓ + H2O
+ H2O + NO
306 Qualitative Analysis

H2O (g ) is condensed to liquid water. 84. Nitric acid is a strong oxidising agent, oxidises H2S to S.
Na 2CO3 + BaCl 2 → BaCO3 + 2NaCl
Y Z 85. (i) The compound C produced by heating A is white in colour
and changes to yellow on heating, thus compound C may be
BaCO3 + 2HCl → BaCl 2 + H2O + CO2
Z
ZnO. C with dil. HCl and K4[Fe(CN)6 ] gives white ppt. This
confirms that the compound C must be ZnO.
82. Compound A is a light green crystalline solid, so it may be
FeSO4. ∆
A → ZnO + B (gas)
(i) FeSO4 is a salt of strong acid and weak base, so it hydrolyses ‘C ’
in dil. H2SO4 but no gas is evolved. ZnO + 2HCl → ZnCl 2 + H2O
(ii) FeSO4 is a strong reducing agent, thus decolourises KMnO4 C
solution : 2ZnCl 2 + K4[Fe(CN)6 ] → 4KCl + Zn 2[Fe(CN)6 ] ↓
5Fe2+ + MnO4− + 8H+ → 5Fe3+ + Mn 2+ + 4H2O White ppt
Purple Colourless

(iii) FeSO4 on strong heating gives both SO2 (B) and SO3 (C ) (ii) The gas B turns lime water milky and milkiness disappear
gases alongwith a residue of Fe2O3 (D ). with continuous passage of gas. Hence, the gas is CO2 and
∆ compound A in ZnCO3.
2FeSO4 → Fe2O3 + SO2 + SO3
D B C
CO2 + Ca(OH)2 → H2O + CaCO3 ↓
B
(iv) The gaseous mixture reduced dichromate solution to green
solution and also gives H2SO4 in solution : CaCO3 + CO2 + H2O → Ca(HCO3 )2
Cr2O72− + 3SO2 + 2H+ → 3SO24− + H2O + 2Cr 3+ ∆
ZnCO3 → ZnO + CO2
Green
A C B
H2O + SO3 → H2SO4
(iii) The solution of A gives white ppt of ZnS D with NH4OH and
(v) The sulphuric acid (formed in previous step) gives white ppt.
excess of H2S.
with Ba(NO3 )2 due to formation of BaSO4 (E) :
ZnCO3 + HCl → CO2 ↑ + ZnCl 2
H2SO4 + Ba(NO3 )2 → BaSO4 ↓ + 2HNO3 B
White ( E )
NH4 OH
(vi) The residue D when heated on charcoal in a reducing flame ZnCl 2 + H2S → 2HCl + ZnS↓ (white)
reduces to iron (Fe) which is a magnetic substance. D

Hence, A = FeSO4, B = SO2, C = SO3, D = Fe2O3 and (iv) The solution of A also gives initially a white ppt E with
E = BaSO4. NaOH, which dissolve in excess of reagent.

83. FeCl 3 is a salt of strong acid and weak base. In water, it ZnCl 2 + 2NaOH → Zn(OH)2 ↓ + 2NaCl
E (white)
hydrolyses slowly producing brown ppt of Fe(OH)3.
Zn(OH)2 + 2NaOH → Na 2[Zn(OH)4 ]
FeCl 3 + 3H2O → 3HCl + Fe(OH)3 ↓ Soluble
Brown
21
Organic Chemistry Basics
Topic 1 Nomenclature and Isomerism
Objective Questions I (Only one correct option) 3. The IUPAC name for the following compound is
1. Newman projections P, Q, R and S are shown below : CH3
OH H3C CH
H3C CH3
CH3
CH3 H CH3
HO H
CH2
(2019 Main, 12 April II)
H3C C2H5 H C2H5 (a) 3-methyl-4-(3-methylprop-1-enyl)-1-heptyne
(b) 3, 5-dimethyl-4-propylhept-6-en-1-yne
CH3 CH3 (c) 3-methyl-4-(1-methylprop-2-ynyl)-1-heptene
(P) ( Q) (d) 3, 5-dimethyl-4-propylhept-1-en-6-yne
4. The correct IUPAC name of the following compound is
CH3 CH3
NO2
C2H5 H C2H5 CH(CH3)2

HO CH3 H H Cl
C2H5 OH CH3
(2019 Main, 9 April I)
( R) (S)
(a) 2-methyl-5-nitro-1-chlorobenzene
Which one of the following options represents identical (b) 3-chloro-1-methyl-1-nitrobenzene
molecules? (2020 Adv.) (c) 2-chloro-1-methyl 1-4-nitrobenzene
(a) P and Q (b) Q and S (d) 5-chloro-4-methyl 1-1-nitrobenzene
(c) Q and R (d) R and S 5. Which of the following compounds will show the maximum
2. Which one of the following structures has the IUPAC name ‘enol’ content? (2019 Main, 8 April II)
3-ethynyl-2-hydroxy-4-methylhex-3-en-5-ynoic acid? (a) CH3COCH3
(2020 Adv.)
(b) CH3COCH2COCH3
OH
(c) CH3COCH2COOC2H5
HO2C (d) CH3COCH2CONH2
(a) CO2H (b)
6. The IUPAC name of the following compound is
OH CH3 OH
OH OH  
H3C  CH  CH CH 2 COOH (2019 Main, 8 April I)
(c) CO2H (d) CO2H (a) 4,4 - dimethyl -3-hydroxybutanoic acid
(b) 2-methyl-3-hydroxypentan-5-oic acid
(c) 3- hydroxy -4- methylpentanoic acid
(d) 4-methyl-3-hydroxypentanoic acid
308 Organic Chemistry Basics

7. What is the IUPAC name of the following compound? 15. Which of the following compounds will exhibit geometrical
CH3 CH3 isomerism ? (2000, 1M)
(a) 1-phenyl-2-butene (b) 3-phenyl-1-butene
H
H (c) 2-phenyl-1-butene (d) 1,1-diphenyl-1-propene
Br
CH3 16. The optically active tartaric acid is named as D-(+)-tartaric
(2019 Main, 10 Jan II)
acid because it has a positive (1999, 2M)
(a) 3-bromo-3-methyl-1,2-dimethylprop-1-ene
(a) optical rotation and is derived from D-glucose
(b) 3-bromo-1,2-dimethylbut-1-ene
(c) 2-bromo-3-methylpent-3-ene (b) pH in organic solvent
(d) 4-bromo-3-methylpent-2-ene (c) optical rotation and is derived from D-(+)-glyceraldehydes
CO2H (d) optical rotation when substituted by deuterium
H OH 17. How many optically active stereoisomers are possible for
8. The absolute configuration of H Cl is (2016 Main)
butane-2, 3-diol ? (1997, 1M)
CH3 (a) 1 (b) 2 (c) 3 (d) 4

(a) (2S, 3R) (b) (2S, 3S) 18. Isomers which can be interconverted through rotation
(c) (2R, 3R) (d) (2R, 3S) around a single bond are (1992, 1M)
(a) conformers (b) diastereomers
9. The IUPAC name of the following compound is (c) enantiomers (d) positional isomers
OH 19. The enolic form of acetone contains (1990, 1M)
(a) 9 sigma bonds, 1 pi bond and 2 lone pairs
(b) 8 sigma bonds, 2 pi bonds and 2 lone pairs
CN (c) 10 sigma bonds, 1 pi bond and 1 lone pair
(d) 9 sigma bonds, 2 pi bonds and 1 lone pair
Br
(2009) 20. The number of isomers of C6 H14 is (1987, 2007, 3M)
(a) 4-bromo-3-cyanophenol (a) 4 (b) 5 (c) 6 (d) 7
(b) 2-bromo-5-hydroxybenzonitrile
21. If two compounds have the same empirical formula but
(c) 2-cyano-4-hydroxybromobenzene
different molecular formulae, they must have (1987, 1M)
(d) 6-bromo-3hydroxybenzonitrile
(a) different percentage composition
10. The number of stereoisomers obtained by bromination of (b) different molecular weight
trans-2-butene is (2007, 3M) (c) same velocity
(a) 1 (b) 2 (c) 3 (d) 4 (d) same vapour density
11. The IUPAC name of C 6 H5 COCl is (2006, 3M) 22. Which of the following will have least hindered rotation
(a) benzoyl chloride (b) benzene chloro ketone about carbon-carbon bond ? (1987, 1M)
(c) benzene carbonyl chloride (d) chloro phenyl ketone (a) Ethane (b) Ethylene
1 (c) Acetylene (d) Hexachloroethane
CH3
23. The IUPAC name of the compound
12. H H CH2 == CH  CH(CH3 )2 is (1987, 1M)
3
2 (a) 1,1-dimethyl-2-butene (b) 3-methyl-1-butene
H H (c) 2-vinyl propane (d) None of these

CH3 24. An isomer of ethanol is (1986, 1M)


(a) methanol (b) diethyl ether
C 2 is rotated anti-clockwise 120° about C 2 -C 3 bond. The (c) acetone (d) dimethyl ether
resulting conformer is (2004, 1M)
(a) partially eclipsed (b) eclipsed 25. The IUPAC name of the compound having the formula is
(c) gauche (d) staggered CH3

13. Which of the following compounds exhibits, H3 C  C  CH==CH2
stereoisomerism? (2002, 3M) 
(a) 2-methylbutene-1 (b) 3-methylbutyne-1 CH3 (1984, 1M)
(c) 3-methylbutanoic acid (d) 2-methylbutanoic acid (a) 3, 3, 3-trimethyl-1-propene
14. The number of isomers for the compound with molecular (b) 1, 1, 1-trimethyl-2-propene
formula C2 BrClFI is (2001, 1M) (c) 3, 3-dimethyl-1-butene
(a) 3 (b) 4 (c) 5 (d) 6 (d) 2, 2-dimethyl-3-butene
Organic Chemistry Basics 309

26. Which of the following compounds will exhibit cis-trans CH3 CH3
(geometrical) isomerism ? (1983, 1M) H OH HO H
(a) 2-butene (b) 2-butyne
(c) 2-butanol (d) butanal
HO H HO H
27. The compound which is not isomeric with diethyl ether is
(1981, 1M) Cl Cl
(a) n-propyl methyl ether P Q
(b) butane-1-ol
(a) M and N are non-mirror image stereoisomers
(c) 2-methyl propane-2-ol
(b) M and O are identical
(d) butanone
(c) M and P are enantiomers
(d) M and Q are identical
Objective Questions II
32. The correct statement(s) about the compound (2009)
(One or more than one correct option)
H3 C(HO) HC  CH == CH  CH(OH)CH3 ( X ) is/are
28. With respect to the compounds I-V. Choose the correct
(a) The total number of stereoisomers possible for X is 6
statement(s). (2020 Adv.)
(b) The total number of diastereomers possible for X is 3
H (c) If the stereochemistry about the double bond in X is trans, the
number of enantiomers possible for X is 4
H
H (d) If the stereochemistry about the double bond in X is cis, the
number of enantiomers possible for X is 2
H—CH3 H H
33. The correct statement(s) concerning the structures
I II III IV V E , F and G is/are
(a) The acidity of compound I is due to delocalisation in the H 3C O H3C OH H3C CH3
conjugate base.
(b) The conjugate base of compound IV is aromatic.
H 3C CH3 H3C CH3 H 3C OH
(c) Compound II becomes more acidic, when it has a —NO2
(E) (F) (G)
substituent.
(d) The acidity of compounds follows the order (a) E , F and G are resonance structures (2008, 4M)
I > IV > V > II > III. (b) E , F and E , G are tautomers
(c) F and G are geometrical isomers
29. The IUPAC name(s) of the following compound is (are) (d) F and G are diastereomers
(2017 Adv.)
34. The correct statement(s) about the compound given below
H 3C Cl is/are
Cl H

 

(a) 4-methylchlorobenzene
H3C   
CH3
(b) 4-chlorotoluene


H


(c) 1-chloro-4-methylbenzene Cl
(2008, 4M)
(d) 1-methyl-4-chlorobenzene
(a) the compound is optically active
30. The correct combination of names for isomeric alcohols with (b) the compound possesses centre of symmetry
molecular formula C4 H10 O is/are (2014 Adv.) (c) the compound possesses plane of symmetry
(a) tert-butanol and 2-methylpropan-2-ol (d) the compound possesses axis of symmetry
(b) tert-butanol and 1,1-dimethylethan-1-ol 35. Tautomerism is exhibited by (1998, 2M)
(c) n-butanol and butan-1-ol 
(d) iso-butyl alcohol and 2-methylpropan-1-ol (a)  CH== CH  OH (b) O 
 
O
31. Which of the given statement(s) about N, O, P and Q with 
respect to M is/are correct? (2012) 
(c) O
 (d) O

Cl
HO HO


H H H O O
Cl CH3 OH
HO H HO 36. Which of the following compounds will show geometrical
H OH H isomerism ? (1998, 2M)
(a) 2-butene (b) propene
CH3 CH3 CH3
(c) 1-phenyl propene (d) 2-methyl-2-butene
M N O
310 Organic Chemistry Basics

37. Which of the following have asymmetric carbon atom? Numerical Answer Type Questions
(1989, 1M)
Cl Br H Cl 41. The number of chiral carbons present in the molecule given
    below is …… . (2020 Main, 2 Sep I)
(a) H  C  C  H (b) H  C  C  Cl
   
H H H H
N
H Cl H H
    H O
(c) H  C  C  D (d) H  C  C  CH3
    C
H H Br OH H3C CH3
HO
38. Keto-enol tautomerism is observed in (1988, 2M) N
O O
  42. For the given compound X, the total number of optically
(a) C6H5  C  H (b) C6H5  C  CH3
active stereoisomers is ……… .
O O O
   HO
HO
(c) C6H5  C  C6H5 (d) C6H5  C  CH2  C  CH3

39. Only two isomeric monochloro derivatives are possible for


(1986, 1M)
(a) n-butane (b) 2, 4-dimethyl pentane
(c) benzene (d) 2-methyl propane HO
HO
Match the Columns This type of bond indicates that the configuration at the
specific carbon and the geometry of the double bond is fixed
40. The Fischer projection of D-erythrose is shown below : This type of bond indicates that the configuration at the specific
carbon and the geometry of the double bond is not fixed
CHO
(2018 Adv.)
H OH
H OH
Assertion and Reason
Read the following questions and answer as per the direction
CH2OH given below :
D-erythrose (a) Statement I is correct; Statement II is correct; Statement II is
D-erythrose and its isomers are listed as P, Q, R, and S in the correct explanation of Statement I
Column-I. Choose the correct relationship of P, Q, R, and S with (b) Statement I is correct; Statement II is correct; Statement II is
D-erythrose from Column II. (2020 Adv.) not the correct explanation of Statement I
Column I Column II (c) Statement I is correct; Statement II is incorrect
HO H (d) Statement I is incorrect; Statement II is correct
OHC
P. 1. Diastereomer 43. Statement I Molecules that are non-superimposable on
H
OH OH their mirror images are chiral.
OH Statement II All chiral molecules have chiral centres.
OHC (2007, 3M)
Q. OH H 2. Identical
H OH Fill in the Blank
H OH
OHC 44. Isomers which are ……… mirror images are known as
R. 3. Enantiomer ……… (superimposable, non-superimposable, enantiomers,
H
OH OH diastereomers, epimers) (1988, 1M)
OH
OHC True/False
S. HO OH
H H 45. 2, 3, 4-trichloropentane has three asymmetric carbon atoms.
(1990, 1M)
Codes
P Q R S 46. m-chlorobromobenzene is an isomer of m-bromochloro
(a) 2 3 2 2 benzene. 1
(1985, M)
2
(b) 3 1 1 2
(c) 2 1 1 3
(d) 2 3 3 1
Organic Chemistry Basics 311

Integer Answer Type Questions (i) Torsional strain


(ii) van der Waals’ strain
47. The total number of stereoisomers that can exist for M is
(2015 Adv.) (iii) Combination of the above two (2004, 5M)
H 3C CH3
52. Glycerine contain one ……… hydroxy group. (2004)

53. Identify the pairs of enantiomers and diastereomers from the


following (2000, 2M)

CH3 CH3 CH3

M
H OH HO H HO H
H3C O
HO H HO H H OH
48. The total number(s) of stable conformers with non-zero
dipole moment for the following compound is/are CH3 CH3 CH3
I II III
Cl
Br CH3 54. Write tautomeric forms of phenol. (1992, 1M)
Br Cl
55. Write the IUPAC name of the following compound:
CH3 (2014 Adv.)
CH3
Subjective Questions 
H3 C  N – C  C H2  CH3
49. Give the total number of cyclic structural as well as  
stereoisomers possible for a compound with the molecular CH3 C2H5 (1991, 1M)
formula C5 H10 . (2009) 56. Give the IUPAC name of the following compound :
50. µ obs = Σ µ i xi Me
where µ i is the dipole moment of stable conformer and xi is
the mole fraction of that conformer. Me Me
(Me = methyl)
(a) Write stable conformer for Z  CH2  CH2  Z in Newman’s
Me Me
projection.
(1990, 1M)
If µ solution = 1.0 D and mole fraction of anti form = 0.82, find
µ gauche
57. Write the IUPAC name of CH3 CH2 CH==CH  COOH .
(1986, 1M)
(b) Write most stable meso conformer of (CHDY )2. If
(i) Y = CH3 about C 2 -C 3 rotation and 58. Write the structure of all the possible isomers of
dichloroethene. Which of them will have zero dipole
(ii) Y = OH about C1 -C 2 rotation. (2005, 6M)
moment ? (1985, 2M)
51. (a) Draw Newman’s projection for the less stable staggered
59. Write structural formulae for the isomeric alcohols having
form of butane.
the molecular formula C4 H10 O. (1984, 2M)
(b) Relatively less stability of the staggered form is due to

Topic 2 General Organic Chemistry


Objective Questions I (Only one correct option) 2. 25 g of an unknown hydrocarbon upon burning produces
1. The increasing order of basicity for the following 88 g of CO2 and 9 g of H2 O. This unknown hydrocarbon
intermediates is (from weak to strong) (2020 Main, 9 Jan I) contains (2019 Main, 12 April II)
(a) 20 g of carbon and 5 g of hydrogen
CH3
(b) 22 g of carbon and 3 g of hydrogen
 s
H3C  C s H 2C == CH CH 2 (c) 24 g of carbon and 1 g of hydrogen
(d) 18 g of carbon and 7 g of hydrogen
 ( ii )
CH3 3. An organic compound A is oxidised with Na 2 O2 followed by
(i) boiling with HNO3 . The resultant solution is then treated with
s s s ammonium molybdate to yield a yellow precipitate.
HC ≡≡ C CH3 CN Based on above observation, the element present in the given
( iii ) ( iv ) (v ) compound is (2019 Main, 12 April I)
(a) (v) < (iii) < (ii) < (iv) < (i) (b) (iii) < (i) < (ii) < (iv) < (v) (a) nitrogen (b) phosphorus (c) fluorine (d) sulphur
(c) (v) < (i) < (iv) < (ii) < (iii) (d) (iii) < (iv) < (ii) < (i) < (v)
312 Organic Chemistry Basics

4. The increasing order of nucleophilicity of the following 11. Which amongst the following is the strongest acid?
nucleophiles is (2019 Main, 10 April II) (2019 Main, 9 Jan I)
È (a) CHBr3 (b) CHI3
(1) CH3 CO2È (2) H2O (3) CH3 SO3È (4) O H
(c) CHCl 3 (d) CH(CN)3
(a) (1) < (4) < (3) < (2) (b) (2) < (3) < (1) < (4)
(c) (4) < (1) < (3) < (2) (d) (2) < (3) < (4) < (1) 12. Which of the following compounds will be suitable for
Kjeldahl’s method for nitrogen estimation? (2018 Main)
5. In chromatography, which of the following statements is
NH2
incorrect for R f ? (2019 Main, 10 April II)
(a) R f value depends on the type of chromatography (a) (b)
(b) Higher R f value means higher adsorption N
(c) R f value is dependent on the mobile phase
(d) The value of R f can not be more than one NO2 N+2Cl–
6. The principle of column chromatography is (c) (d)
(2019 Main, 10 April I)
(a) differential absorption of the substances on the solid phase
(b) differential adsorption of the substances on the solid phase 13. Which of the following molecules is least resonance
(c) gravitational force stabilised? (2017 Main)
(d) capillary action
7. In the following compound, (a) (b)
a O
NH2
d N
Nb (c) (d)
N

e N N O
H c
14. The distillation technique most suited for separating glycerol
the favourable site/s for protonation is/are from spent lye in the soap industry is (2016 Main)
(2019 Main, 11 Jan II)
(a) fractional distillation
(a) (a) and (e) (b) (b), (c) and (d)
(b) steam distillation
(c) (a) and (d) (d) (a)
(c) distillation under reduced pressure
8. The correct match between items I and II is (d) simple distillation
Item - I Item II 15. The correct order of acidity for the following compounds is
(Mixture) (Separation method) (2016 Adv.)
A. H2O : Sugar P. Sublimation CO2H CO2H
B. H2O : Aniline Q. Recrystallisation
HO OH
C. H2O : Toluene R. Steam distillation I II
S. Differential extraction OH

(2019 Main, 11 Jan I)


(a) (A) → (Q); (B) → (R); (C) → (S) CO2H CO2H
(b) (A) → (Q); (B) → (R); (C) → (P)
OH
(c) (A) → (S); (B) → (R); (C) → (P)
(d) (A) → (R); (B) → (P); (C) → (S)
III IV
9. An organic compound is estimated through Dumas method
and was found to evolved 6 moles of CO2 , 4 moles of H2 O and OH
1 mole of nitrogen gas. The formula of the compound is (a) I > II > III > IV (b) III > I > II > IV
(2019 Main, 11 Jan I) (c) III > IV > II > I (d) I > III > IV > II
(a) C6H8 N (b) C12H8 N (c) C12H8 N2 (d) C6H8 N2
10. If dichloromethane (DCM) and water (H 2O) are used for 16. For the estimation of nitrogen, 1.4 g of an organic compound
differential extraction, which one of the following was digested by Kjeldahl's method and the evolved ammonia
statements is correct? (2019 Main, 10 Jan I) was absorbed in 60 mL of M /10 sulphuric acid. The
(a) DCM and H2O would stay as lower and upper layer unreacted acid required 20 mL of M /10 sodium hydroxide for
respectively in the S.F. complete neutralisation. The percentage of nitrogen in the
(b) DCM and H2O would stay as upper and lower layer compound is (2014 Main)
respectively in the separating funnel (S.F.) (a) 6% (b) 10%
(c) DCM and H2O will be miscible clearly (c) 3% (d) 5%
(d) DCM and H2O will make turbid/colloidal mixture
Organic Chemistry Basics 313

17. A gaseous hydrocarbon gives upon combustion 0.72 g of 25. The correct acidity order of the following is (2009)
water and 3.08 g of CO 2. The empirical formula of the OH OH COOH COOH
hydrocarbon is (2013 Main)
(a) C2H4 (b) C3H4
(c) C6H5 (d) C7 H8
18. The order of stability of the following carbocations
(I) (III)
Cl CH3
CH2 (II) (IV)
(a) (III) > (IV) > (II) > (I) (b) (IV) > (III) > (I) > (II)
CH2 CH CH2 ; CH3 CH2 CH2 ; is
(I) (II) (c) (III) > (II) > (I) > (IV) (d) (II) > (III) > (IV) > (I)
(III) (2013 Main) 26. Hyperconjugation involves overlap of the following orbitals
(a) III > II > I (b) II > III > I (2008, 3M)
(c) I > II > III (d) III > I > II (a) σ - σ (b) σ - p (c) p-p (d) π - π

19. Arrange the following compounds in the order of decreasing 27. The correct stability order for the following species is
acidity (2013 Main) ⊕ ⊕ 
OH OH OH OH
O
(I) (II)
; ; ;
⊕  
Cl CH3 NO2 OCH3 O ⊕
(I) (II) (III) (IV) (III) (IV)
(2008, 3M)
(a) II > IV > I > III (b) I > II > III > IV (a) (II) > (IV) > (I) > (III) (b) (I) > (II) > (III) > (IV)
(c) III > I > II > IV (d) IV > III > I > II (c) (II) > (I) > (IV) > (III) (d) (I) > (III) > (II) > (IV)
20. A solution of ( − l ) 1-chloro-1-phenylethane in toluene 28. Among the following, the least stable resonance structure is
racemises slowly in the presence of a small amount of SbCl 5, (2007, 3M)
due to the formation of (2013 Main) ⊕
⊕ O ⊕ O
(a) carbanion (b) carbene (a) N (b) ⊕ N
(c) carbocation (d) free radical  
O O
21. In allene (C3 H4 ), the type(s) of hybridisation of the carbon
atoms, is (are) (2012)
⊕ O ⊕ O
(a) sp and sp3 (b) sp and sp2 (d) N
(c) ⊕ N ⊕
(c) only sp3 (d) sp2 and sp3  
O O
22. Among the following compounds, the most acidic is (2011)
(a) p-nitrophenol (b) p-hydroxybenzoic acid 29. When benzene sulphonic acid and p-nitrophenol are treated
(c) o-hydroxybenzoic acid (d) p-toluic acid with NaHCO3, the gases released respectively are (2006)
(a) SO2, NO2 (b) SO2, NO
23. The correct stability order of the following resonance (c) SO2, CO2 (d) CO2, CO2
structure is + – + –
(I) H2 C == N == N (II) H2 C  N == N 30. Which of the following is obtained when 4-methylbenzene
– + – + sulphonic acid is hydrolysed with excess of sodium acetate?
(III) H2 C  N ≡≡ N (IV) H2 C  N == N (2009) (2005, 1M)
(a) (I) > (II) > (IV) > (III) (b) (I) > (III) > (II) > (IV)
(c) (II) > (I) > (III) > (IV) (d) (III) > (I) > (IV) > (II)
(a) H3C— —COONa

24. In the following carbocation; H/CH 3 that is most likely to


migrate to the positively charged carbon is (2009)
(b) H3C— + SO3
H H
1 +  5
H3 C C2  C3  C4  CH3 (c) H3C— —SO3Na + CH3COOH
  
OH H CH3
(a) CH3 at C-4 (b) H at C-4 (d) H3C— —SO2OCOCH3 + NaOH
(c) CH3 at C-2 (d) H at C-2
314 Organic Chemistry Basics

31. For 1-methoxy-1, 3-butadiene, which of the following HOOC


resonating structure is least stable? (2005, 1M) O–
s ⊕
(a) CH2  CH  CH ==CH  O  CH3
s ⊕
(b) CH2  CH== CH  CH ==O  CH3 (c)
⊕ s O 2N
(c) CH2 == CH  CH— CH  O  CH3
s ⊕ CH
(d) CH2 == CH  CH  CH== O  CH3
32. H3N+ +
O–
NH3
Z HOOC
Y
OH
COOH
X
Arrange in order of increasing acidic strength (2004, 1M) (d) O2 N
(a) X > Z > Y (b) Z < X > Y
(c) X > Y > Z (d) Z > X > Y C–
33. Among the following, the molecule with the highest dipole
moment is (2003, 1M) O–
(a) CH3Cl (b) CH2Cl 2 (c) CH2Cl 2 (d) CCl 4 36. Which of the following acids has the smallest dissociation
34. Which of the following represent the given mode of constant? (2002)
2 2 (a) CH3CHFCOOH
hybridisation sp − sp − sp − sp from left to right?
(b) FCH2CH2COOH
(2003, 1M)
(c) BrCH2CH2COOH
(a) H2C==CH  C ≡≡ N (b) HC ≡≡ C C ≡≡ CH
(d) CH3CHBrCOOH
(c) H2C==C==C==CH2 (d)



37. Identify the correct order of boiling points of the following


35. HOOC compounds:
CH3CH2CH2CH2OH (1) CH3CH2CH2CHO (2)
OH
CH3CH2CH2COOH (3) (2002)
(a) 1 > 2 > 3 (b) 3 > 1 > 2
(c) 1 > 3 > 2 (d) 3 > 2 > 1
O2N 2 moles of NaNH2
38. Which of the following hydrocarbons has the lowest dipole
CH moment? (2002)
(a) cis-2-butene (b) 2-butyne
OH (c) 1-butyne (d) H2C==CH—C≡≡CH
The product A will be (2003)
39. The correct order of basicities of the following compounds is

OOC NH
H3C C CH3CH2NH2
OH NH2 (2 )
(1) (2001)
(CH3 )2 NH CH3CONH2
(a) O2N (3 ) (4 )
(a) 2 > 1 > 3 > 4 (b) 1 > 3 > 2 > 4
CH
(c) 3 > 1 > 2 > 4 (d) 1 > 2 > 3 > 4
40. Among the following, the strongest base is (2000)
O–

OOC (a) C6H5NH2
(b) p-NO2C6H4NH2
OH
(c) m-NO2—C6H4NH2
(d) C6H5CH2NH2
(b) O2N 41. Which of the following, has the most acidic hydrogen?
– (2000)
C (a) 3-hexanone (b) 2, 4-hexanedione
(c) 2, 5-hexanedione (d) 2, 3-hexanedione
OH
Organic Chemistry Basics 315

42. The most unlikely representation of resonance structures of 48. The number of sigma and pi-bonds in 1-butene 3-yne are
p-nitrophenoxide ion is (1999) (1989, 1M)
(a) 5 sigma and 5 pi (b) 7 sigma and 3 pi
O– O– O O
+
(c) 8 sigma and 2 pi (d) 6 sigma and 4 pi
N N
49. The compound which gives the most stable carbonium ion
on dehydration is (1989, 1M)
(a) (b) CH3

(a) CH3  CH  CH2OH (b) CH3  C OH
– –
 
O O CH3 CH3
O – O O –
O– OH
N + + 
N (c) CH3CH2  CH2  CH2OH (d) CH3  CH  CH2CH3
50. Polarisation of electrons in acrolein may be written as
(c) (d) (1988, 1M)
δ+
O O
– δ−  δ− 
O O (a) H2C== CH  C  H (b) H2C== CH  C  H
δ+
43. Among the following compounds, the strongest acid is δ−
(1998) O O
(a) HC≡≡CH (b) C6H6 δ− δ+  δ+ 
(c) H2C== CH  C  H (d) H2C== CH  C  H
(c) C2H6 (d) CH3OH
51. The bond between carbon atom (1) and carbon atom (2) in
44. In the following compounds (1997)
compound
OH OH OH OH N≡≡ C  CH== CH2
1 2
involves the hybridisation as (1987, 1M)
(a) sp2 and sp2 (b) sp3 and sp (c) sp and sp2 (d) sp and sp
NO 2
Objective Questions II
CH 3 NO 2 (One or more than one correct option)
(I) (II) (III) (IV ) 52. Among P, Q, R and S, the aromatic compounds(s) is/are
The order of acidity is Cl
(a) III > IV > I > II
AlCl3 NaH
(b) I > IV > III > II P Q
(c) II > I > III > IV
(d) IV > III > I > II O
(NH4)2CO3 HCl
45. What is the decreasing order of strength of the bases? R S
100-115°C

OH , NH2− , H  C≡≡ C and −
CH3  CH2− (1997)
O O
(2013 Adv.)
(a) CH3  CH2− > NH2− > H  C≡≡ C > OH − −
(a) P (b) Q (c) R (d) S

(b) H  C≡≡ C − > CH3  CH2− > NH2− > OH − 53. The hyperconjugative stabilities of tert-butyl cation and
2-butene, respectively, are due to (2013 Adv.)
(c) OH − > NH2− > H  C≡≡ C − > CH3  CH2− (a) σ → p (empty) and σ → π* electron delocalisations
(d) NH2− > H  C≡≡ C − > OH − > CH3  CH2− (b) σ → σ* and σ → π electron delocalisations
(c) σ → p (filled) and σ → π electron delocalisations
46. The hybridisation of carbon atoms in C  C single bond (d) p (filled) → σ* and σ → π* electrons delocalisations
H  C ≡≡ C  CH==CH2 is (1991, 1M)
(a) sp3 − sp3 (b) sp2 − sp3 54. Amongst the given options, the compound(s) in which all the
atoms are in one plane in all the possible conformations
(c) sp − sp2 (d) sp3 − sp (if any), is/are (2011)

47. Amongst the following, the most basic compound is H H H


(a) C C (b) H  C ≡≡ C C
(a) benzylamine (b) aniline (1990, 1M) H2C CH2 CH2
(c) acetanilide (d) p-nitroaniline (c) H2C == C == O (d) H2C == C == CH2
316 Organic Chemistry Basics

55. In the Newman’s projection for 2,2-dimethylbutane 58. Phenol is less acidic than (1986)
X (a) acetic acid (b) p-methoxy phenol
H3C CH3
(c) p-nitrophenol (d) ethanol

H H Assertion and Reason


Y Read the following questions and answer as per the direction
X and Y can respectively be (2010) given below :
(a) H and H (a) Statement I is correct; Statement II is correct; Statement II is
(b) H and C2H5 the correct explanation of Statement I
(c) C2H5 and H (b) Statement I is correct; Statement II is correct; Statement II is
(d) CH3 and CH3 not the correct explanation of Statement I
56. The molecules that will have dipole moment are (c) Statement I is correct; Statement II is incorrect
(1992, 1M)
(d) Statement I is incorrect; Statement II is correct
(a) 2, 2-dimethyl propane
(b) trans-2-pentene
59. Statement I p-hydroxybenzoic acid has a lower boiling
point than o-hydroxybenzoic acid.
(c) cis-3-hexene
Statement II o-hydroxybenzoic acid has intramolecular
(d) 2,2,3,3-tetramethyl butane
hydrogen bonding.
57. The compound in which C uses its sp 3 -hybrid orbitals for 60. Statement I p-nitrophenol is a stronger acid than
bond formation is (2000, 1M) o-nitrophenol.
(a) HCOOH (b) (H2N)2 CO
Statement II Intramolecular hydrogen bonding make the
(c) (CH3 )3 COH (d) CH3CHO o-isomer weaker acid than p-isomer.

Match the Columns


61. Match the reactions in Column I with appropriate types of steps/reactive intermediate involved in these reactions as given in
Column II. (2011)

Column I Column II

H3C O O
O
A. p.
aq NaOH
Nucleophilic substitution

18
O O
B.  18
CH2CH2CH2OH
H2SO4 q. Electrophilic substitution

O O

C. CH2CH2CH2Cl CH3MgI r. Dehydration
CH3

CH2CH2CH2C(CH3)2
D. H2SO4 s. Nucleophilic addition
OH

H 3C CH3 t. Carbanion
Organic Chemistry Basics 317

62. Match the reaction in Column I with appropriate options in Column II. (2010)

Column I Column II
p. Racemic mixture
—N2Cl + —OH
A.
0°C NaOH/H2O

—N N— —OH

OH OH O q. Addition reaction
  H 2SO 4
B. H3C — C — — C — CH3 → CH3
C
  H 3 C C—CH 3
CH3 CH3 CH3

O r. Substitution reaction
1. LiAlH4
—–C
2. H3O +
CH3
C.
OH
—–HC
CH3

s. Coupling reaction
SH— —Cl Base S
D.
t. Carbocation intermediate

Fill in the Blanks Integer Answer Type Questions


63. The kind of delocalisation involving sigma bond orbitals is
74. The total number of contributing structures showing
called…… (1994, 1M)
hyperconjugation (involving C — H bonds) for the
64. The bond dissociation energy needed to form the benzyl following carbocation is
radical from toluene is ……… than the formation of the (2011)
methyl radical from methane. (1994, 1M) H 3C ⊕ CH2CH3

65. The structure of the enol form of 

CH3  CO  CH2  CO  CH3


with intermolecular hydrogen bonding is ……… (1993, 1M)
75. Amongst the following, the total number of compounds
66. The IUPAC name of succinic acid is ……… (1990, 1M)
soluble in aqueous NaOH is (2010)
67. The shape of (CH3 )+ is ……… (1990, 1M) H3C CH3 OH
68. A…… diol has two hydroxyl groups on ……carbon atoms. N COOH NO2
(1985, 1M)
69. The terminal carbon atom in butane is …… hybridised.
(1985, 1M)
70. ………ring is most strained. (cyclopropane, cyclobutane, N
H3C CH3
cyclopentane) (1981, 1M)
OCH2CH3 OH CH2CH3
71. The compound having both sp and sp 2 -hybridised carbon
CH2OH CH2CH3
atoms is ……. (propane, propene, propadiene). (1981, 1M)

72. In acidic medium, ……….. behaves as the strongest base. COOH


(nitrobenzene, aniline, phenol) (1981, 1M)

73. Among the given cations, ……… is most stable.


(sec-butyl carbonium ion, tert-butyl carbonium ion, n-butyl
carbonium ion) (1981, 1M)
318 Organic Chemistry Basics

Subjective Questions p-methylbenzoic acid 36.2 × 10 −5


p-methoxybenzoic acid 10.2 × 10 −5 (2003)
76. Which of the following is more acidic and why?
+ +
80. Give reasons for the following:
NH3 NH3 CH2 ==CH − is more basic than HC≡≡ C − . (2000)

81. Explain, why o-hydroxybenzaldehyde is a liquid at room


or temperature while p-hydroxybenzaldehyde is a high melting
solid? (1999)

82. Discuss the hybridisation of carbon atoms in allene (C3 H4 )


F OH (2003)
and show the π-orbital overlaps. (1999, 3M)
77. Draw the resonating structures of . (2003)
83. Give reasons for the following in one or two sentences.
78. You have an ether solution containing 4-hydroxybenzoic The central carbon-carbon bond in 1, 3-butadiene is shorter
acid and 4-aminobenzoic acid. Explain, how will you than that of n-butane. (1998)
separate the two in not more than 3 steps? Give confirmatory
84. Although phenoxide ion has more number of resonating
tests with reagents and conditions for functional groups of
structures than benzoate ion, benzoic acid is a stronger acid
each. (2003)
than phenol. Why? (1997)
79. Match the following with their K a values
85. Arrange the following in the order of their increasing
Benzoic acid 4.2 × 10 −5 basicity.
p-nitrobenzoic acid 3.3 × 10 −5 p-toluidine, N,N-dimethyl-p-toluidine, p-nitroaniline,
p-chlorobenzoic acid 6.4 × 10 −5 aniline. (1985, 1M)

Answers
Topic 1 13. (d) 14. (c) 15. (a) 16. (b)
1. (c) 2. (d) 3. (d) 4. (c) 17. (d) 18. (d) 19. (c) 20. (c)
5. (b) 6. (c) 7. (d) 8. (a) 21. (b) 22. (c) 23. (b) 24. (d)
9. (b) 10. (a) 11. (c) 12. (c) 25. (a) 26. (b) 27. (d) 28. (a)
13. (d) 14. (d) 15. (a) 16. (c) 29. (d) 30. (c) 31. (c) 32. (a)
17. (b) 18. (a) 19. (a) 20. (b) 33. (a) 34. (a) 35. (a) 36. (c)
21. (b) 22. (a) 23. (b) 24. (d) 37. (b) 38. (b) 39. (b) 40. (d)
25. (c) 26. (a) 27. (d) 28. (a,b,c) 41. (b) 42. (b) 43. (d) 44. (d)
29. (b,c) 30. (a,c,d) 31. (a,b,c) 32. (a,d) 45. (a) 46. (d) 47. (a) 48. (b)
33. (b,c,d) 34. (a,d) 35. (a,c,d) 36. (a,c) 49. (b) 50. (d) 51. (c) 52. (a,b,c,d)
37. (c,d) 38. (b,d) 39. (d) 40. (c) 53. (a) 54. (b,c) 55. (b,d) 56. (b,c)
41. (5) 42. (7) 43. (c) 57. (c,d) 58. (a,c) 59. (d) 60. (a)
44. (Non-superimposable, Enantiomers) 45. (False) 61. (A → r, s, t; B → p, s; C → r, s; D → q, r)
46. (False) 47. (2) 48. (3) 49. (7) 62. (A → r, s; B → t; C → p, q; D → r)
63. (hyperconjugation)
Topic 2
64. (less) 65. (cyclic) 66. (butanedioic acid)
1. (a) 2. (c) 3. (b) 4. (c)
67. (triangular planar) 68. (geminal, same)
5. (b) 6. (b) 7. (b) 8. (a)
69. (sp3 ) 70. (cylopropane) 71. (propene) 72. (aniline)
9. (d) 10. (a) 11. (d) 12. (b)
73. (tert-butyl carbonium ion), 74. (6) 75. (4)
Hints & Solutions
Topic 1 Nomenclature and Isomerism CH3
H3C 7 CH
1. IUPAC name of P, Q, R and S are : Longest possible
5
CH3 4 6 chain containing
CH3 OH 2
double and
CH3 3
HO CH3 triple bond
  1
⇒ C 2H5 C  C  CH3 CH2
H3C C2H5  
CH3 CH3 If both double and triple bonds are present in the compound, the
CH3
(2,3,3-trimethylpentan-2-ol) endings like-en-yne, a (numeral) dien-(numeral)-yne etc., are
(P) used. Numbers as low as possible are given to double and triple
bonds as a set.
OH 4. The IUPAC name of the given compound is
H3C CH3
CH3 NO2
H  4

Incorrect
H

Correct
⇒ C 2H5 C H  C  OH 3

way

way
  2
H C 2H5 C H2 CH3 1 Cl
CH3 
CH3
(Q) CH3
2- chloro-1-methy l- 4-nitrobenzene
(3-ethyl-2-methylpentan-2-ol)
Here, the given compound contains two or more functional
CH3 groups. So, the numbering is done in such a way that the sum of
CH3
C2H5 H  the locants is the lowest.
⇒ C 2H5 C H  C  CH3 5. O OH

HO  
CH3
C 2H5 OH Acetone No additional stability
C2H5 (3-ethyl-2-methylpentan-2-ol) with the ‘enol’ form
(R) H
O O
CH3 O O
CH2CH3 Acetylacetone
C2H5 CH(CH3)2
 This enol-form is highly stable
⇒ C 2H5 —C—CH— CH3 because of intramolecular
H-bonding (6-membered stability)
H H   and due to extended conjugation.
OH
OH CH3
(S) (3-ethyl-2-methylpentan-3-ol) O O (–R)
O O
IUPAC name of compounds Q and R are same, thus the correct 1 2 O
Ethyl acetoacetate 1 2 NH2
option is (c).
(EAA)
2. According to the IUPAC rules, first of all select the longest In both of the compounds, C -2 of C == Ogroup is a part of the acid
carbon chain (6-carbon atoms) then numbering should be start O O
from most prior functional group, which is carboxylic acid in the  
given compound, thus derivative (ester,  C  OEt and acid amide,  C  NH2). So,
OH C2 O does not take part in enolisation, because it is already in
HC≡≡C 3
2
1 resonance (–R) with the derivative group itself.
COOH
6. The IUPAC name of the given compound is
HC≡≡C 4
3-hydroxy-4-methylpentanoic acid.
6 5
3-ethynyl-2-hydroxy-4-methylhex-3-en-5-ynoic acid CH3 OH
5  4 3 2 1
3. The IUPAC name for the given compound is
H3 C  C H  C H  C H2  C OOH
3, 5-dimethyl-4-propylhept-1-en-6-yne.
Principal chain
320 Organic Chemistry Basics

While naming the compound, the longest chain that have H


principal functional group COOH is choosen and numbered in *
such a manner that the principal functional group gets the lowest 13. CH3 — CH2 — C  COOH : Has a chiral carbon, optically active.
possible number. OH act as substituent and used as prefix in 
nomenclature. CH3
(2-methylbutanoic acid)
7. While naming the compound, alkene gets priority over
functional group ( Br ) and numbering starts from alkene side. F Cl F I
Hence, IUPAC name: 4-bromo-3-methyl pent-2-ene 14. C == C + C == C Geometrical isomers
1 Br I Br Cl
CH3 CH3
F Br F I
2 H
3 C == C + C == C Geometrical isomers
4
H Cl I Cl Br
CH3 Br
5 F Cl F Br
8. 1 C == C + C == C Geometrical isomers
COOH
2 I Br I Cl
H OH
3
H Cl H3C CH2 — Ph H CH2 — Ph
4
CH3 15. C == C C == C
For C-2, order of priority of substituents is H H H3C H
OH > CH(Cl) (CH3 ) > COOH cis trans
For C-3, order of priority of substituents is 1-phenyl-2-butene
Cl > CH(OH)COOH > CH3 16. The ‘D’ term in name is derived from D-glyceraldehyde.
Hence, according to CIP rules, 17. M
3 CH3 CH3 CH3
COOH
1 H OH HO H H OH
H OH 2S
2 1
H Cl 3R HO H H OH H OH
3
CH3 CH3 CH3 CH3
I II III
OH
5
I and II are optically active while III is optically inactive
(meso form).
9. CN has the highest priority. 18. Conformers can be interconverted through rotation about C—C
1
bond. H
2 CN • •
•O •
Br 
(2-bromo-5-hydroxybenzonitrile) 19. H— C == C  CH3 :
Br 
H H
H CH3 C Enol of acetone
Br2 H 3C It has 9 σ (6 with H, two C—C and one C  O), one π-bond and
10. C==C C
CCl4 H two lone-pairs.
H 3C H CH3
Br 20.
CH3
H Br I II
III
H Br
CH3
(meso)
O IV V

11. C6H5  C  Cl : Benzene carbonyl chloride 21. Compounds with same empirical formula but different
molecular formula have same percentage composition of
1
CH3 H elements but different molecular weight.
H H H
H H H
120° 3  
3
12. 2 2 22. H— C — C — H ; Ethane has the smallest sized group (H)
Anti-clockwise
H H
1 H  
H 3C H H
CH3 CH3 bonded to carbons, hence there will be least hindered rotation
(gauche) about C—C bond.
Organic Chemistry Basics 321

1 2 3 4
The combination of names for possible isomeric alcohols with
23. CH2 == CH— CH — CH3 : 3-methyl-1-butene
molecular formula C4H10O is/are

CH3 Formula Names
24. Ethers and alcohols (saturated, acyclic) with same number of CH3CH2CH2CH2OH n-butyl alcohol / n-butanol
carbons are always isomeric. / butan-1-ol
CH3 — O— CH3 and CH3 — CH2 — OH are functional isomers. CH3  CH  CH2  OH Iso-butyl alcohol / 2-methyl
25. Double bond has preference over alkyl group hence :  propan-1-ol
CH3
CH 3
4 3 2 1 CH3  CH2  CH  OH Secondary butyl alcohol /
H3 C— C  CH == CH2 : 3,3-dimethyl-l-butene  butan-2-ol
 CH3
CH3
CH3 Tertiary butyl alcohol /
H3 C CH3 H CH3  tert butanol / 2-methyl
26. C== C C== C CH3  C  OH propan-2-ol

H H H3 C H CH3
cis-2- butene trans-2- butene
Hence, choices (a), (c) and (d) are correct.
27. Diethyl ether (C2H5OC2H5 ) will be isomeric with all 4-carbon
saturated alcohols. Butanone (CH3CH2COCH3 ) is unsaturated,
31. Converting all of them into Fischer projection.
HO H Cl
has two hydrogen less than the diethylether.
28. Triphenylmethane (I) is acidic because its conjugate base is Cl HO H
HO
stabilised by resonance. H HO H
(Ph)3CH → (Ph)3C + H+
Triphenylmethyl CH3 CH3
carbanion M M
Cyclopentadiene (IV) is acidic because its conjugate base is HO H CH3
aromatic.
CH3 HO H
+ H+ H
OH H OH
H H H Cl Cl
pKa=16 Aromatic N N
Nitrobenzene is more acidic than benzene because nitro group is Since, M and N have  OH on same side and opposite side
electron withdrawing. It will stabilise the conjugate base of respectively, they cannot be miror image, they are diastereomers.
Cl
benzene by –R and – I effect. HO H Cl
The acidic strength order on the basis of pK a data is H
IV > V > I > II > III. OH HO H
Cl
HO HO
Hence, the correct options are (a), (b) and (c) only. H H HO H
29. Since, there is no principal functional groups, numbering of CH3 CH3 CH3
disubstituted benzene is done in alphabetical order as O O O
Cl M and O are identical.
1 NOTE Fischer projection represents eclipse form of Sawhorse
6 2 projection.
For comparision purpose, similar types of eclipse conformers
5 3 must be drawn i.e. both vertically up or both vertically down.
4
CH3 H OH
CH3 H OH
Hence IUPAC name of this compound is 1-chloro-4-methyl H3C Cl
benzene.
HO H HO
Also, toluene is an acceptable name in IUPAC, hence this H
Cl
compound can also be named as 4-chloro toluene.
P H OH Cl
30. PLAN This problem is based on structure and nomenclature of organic
compound. Cl H OH
Draw structure of each compound and write IUPAC name of the H
OH H OH
given compound.
Match the molecular formula of given compound with molecular CH3 CH3
formula of compound given in choices. P
322 Organic Chemistry Basics

M and P are non-superimposable mirror images, hence,


enantiomers. (d) ==O r —OH
CH3 HO H
HO H O O
H3C Cl H3C CH3 H3C H
HO H HO 36. (a) C == C + C == C
H
Cl H H H CH3
Q HO H cis-2- butene trans-2- butene
Cl
H H Ph H
HO Cl HO H (c) C == C + C == C
H OH H OH Ph CH3 H CH3
CH3 cis-1-phenylpropene trans-1- phenylpropene
CH3
Q 37. A carbon bonded to four different atoms or groups is called
asymmetric carbon :
M and Q are non-identical they are distereomers.
Cl H
32. CH3 CH3 CH3 CH3  *
(c) CH3 — C*— D (d) BrCH2 — C— CH3
HO OH  
C C H C C OH
H OH
C==C C==C
H H HO H 38. Carbonyl compounds containing α-H show keto-enol
H H H H
tautomerism.
(I) (II + III)
(meso) (enantiomeric) O OH
 
CH3 CH3 (b) C6H5  C  CH3 r C6H5 — C == CH2
HO HO O O
C H C H
 
C==C C==C (d) C6H5  C  CH2  C  CH3 r
H OH H H O OH
H C H C
 
C6H5  C  CH== C— CH3
CH3 H CH3 OH
(IV + V) (VI) CH3 CH3 CH3
(enantiomeric) (meso)  Cl 2  
Total six isomers. In both cis and trans forms, there are two
39. CH3 — C — CH3 → CH3 — C — CH 2Cl + CH3 — C — CH3
 hν  
enantiomers each. H H Cl
33. E, F and G are not resonance structures because movement of
40. (c) First of all, we need to convert wedge-dash formula into
hydrogen between E and F are involved.
Fischer projection formula.
E, F and E, G are tautomers in which E is keto form and both F CHO
and G are enol form of the same E. HO H
F and G are geometrical isomers. OHC H OH
P. ⇒
F and G are distereomers as they are stereo isomers but not H H OH
HO OH
related by mirror image relationship. CH2OH
34. The compound is optically active as well as it possesses a
CHO
two-fold axis of symmetry. OH
OHC OH H
O Q. HO ⇒
H H OH
H OH
35. (a) —CH==CH—OH r —CH2 ==C—H CH2OH
CHO
H OH
OHC H OH
(c) ==O r —OH R. ⇒
H HO H
OH OH
CH2OH
O HO
Organic Chemistry Basics 323

CHO True / False


OH
OHC HO H 45. It has only two asymmetric carbon, carbon no.3 is not
S. ⇒
HO OH HO H asymmetric.
H H
CH2OH 46. They are identical.
Now we can clearly see, that compound P is same as given 47. Although the compound has two chiral carbons (indicated by
compound, thus compound P is identical to D-erythrose. stars), it does not has four optically active isomers as expected. It
Compound Q and D-erythrose are not mirror images of one is due to its existence in cis-form only.
another and are non-superimposable on one another. Thus, they CH3
CH3
CH3
are diastereomers. H 3C
Compound R and D - erythrose are also not mirror images of one ∗
another and are non-superimposable on one another. Thus, they
are diastereomers. ∗
Compound S and D-erythrose are chiral molecules. They are H 3C CH3
mirror images of one another. Furthermore, the molecules are O O
cis-form trans-form
non-superimosable on one another. This means that the
(only hypothetical)
molecules cannot be placed on top of one another and give the
The above shown transformation does not exist due to restricted
same molecule. Chiral molecules with one or more stereocenters
rotation about the bridge head carbons, hence only cis-form and
can be enantiomers.
its mirror image exist.
Hence, the correct option is (c).
48. PLAN This problem can be solved by using concept of conformational
41. * analysis of given organic compound. To solve the question draw
*
N the stable conformational structures of organic compound and
determine the net resultant dipole moment.
H * O
* The conformations of the given compound are as follows
C * Cl
H3C CH3 Br Cl
HO Cl CH3 Br
Br CH3 Cl Br
= =
The molecule contains five chiral carbon (C*) atoms. Br Cl Br Cl
42. (7) As given in the question 3 stereocentres are visible, i.e. CH3 CH3
H3C CH3
OH OH
H H H
* 1
   5
*
C H3  C*2  C*3  C*4  CH3
*   
HO Cl Cl Cl
OH
Cl CH3 Br
Hence, the total number of stereoisomers = 23 = 8
Br Cl Br Cl Br Cl
But out of these the following one is optically inactive due to
symmetry
OH OH CH3 Br Br Cl Cl CH3
CH3 CH3 CH3
(Me-Me)gauche (Br-Me)gauche (Cl-Me)gauche

These three have non-zero dipole moment due to non-cancellation


of all dipole moment created by C—Cl and C—Br bond.
HO
OH
49. CH2CH3 CH3 CH3
Hence, total number of optically active stereoisomers=7 CH3
43. Molecules that are non-superimposable on its mirror image are
optically active and known as chiral molecule. However, for
chirality of molecule, presence of chiral centre is not essential,
e.g. CH3 CH3 H CH3 Mirror CH3 H
H3C CH3
C==C==C H H H CH3
CH3 H
H H meso
pair of enantiomers
Molecule is chiral but does not possesses any chiral carbon.
⇒ Total seven isomers
44. Non-superimposable, Enantiomers.
324 Organic Chemistry Basics

50. (a) 4 Me 2
Z Z 5 1
Me 3 Me
H H H Z 56. 8 10
Me 6
Me
7 9
H H H H 5,6-diethyl-3-methyl decane
5 4 3 2 1
Z H 57. CH3 CH2 — CH== CH— COOH
anti gauche 2-pentenoic acid or pent-2-enoic acid
Mole fraction of anti form = 0.82
H Cl Cl Cl Cl H
Mole fraction of gauche form = 0.18
58. C== C C== C C== C
µ obs = 1 D ⇒ 1 = µ anti × 0.82 + µ gauche × 0.18
H Cl H H H Cl
Q µ anti = 0 ⇒ 1 = µ gauche × 0.18
I II III
1
⇒ µ gauche = = 5.55 D III is non-polar, has zero dipole moment.
018
.
H 59. CH3 — CH2 — CH2 — CH2OH CH3 — CH2 — CH — CH3
CH3 O 
OH
(b) D H D O enantiomeric ( ± )
H OH

H D D H CH3 — C  CH3 ; CH3 — CH — CH2 OH
 
CH3 H CH3 CH3
(i) (ii) Total number of isomers (including stereoisomers) = 5
Structure (ii) is more stable than its anti conformer because of
intramolecular H-bonding.
Topic 2 General Organic Chemistry
CH3 1. Basicity order for the following in intermediates is
CH3
H CH3
|
51. (a) CN− <CH == CÈ < H2C == CH CH2 <CH2 <CH3  CÈ
È È

(v) (iii) (ii) (iv)


H H |
CH3
H (i)
less stable staggered form of butane which is also the opposite of the order of stability of carbanions.
(b) The less stability of above mentioned conformer is due to van CH3
der Waals’ repulsion between the adjacent methyl groups. C N > CH C > CH2 CH—CH2 > CH3 > C CH3
52. (sp-carbon) (resonance) CH3
HO OH stabilised
(less stable due
OH to +I effect of
glycerine CH3 groups)

It contains a secondary (2°) hydroxy group.


2. Hydrocarbon containing C and H upon burning produces CO2
53. I and III are mirror images of one another as well as they are
and water vapour respectively. The equation is represented as
non-superimposable while II is a meso form.
⇒ (I + III) = Enantiomers CxH y + (x + y / 4 )O2 → xCO2 + ( y / 2)H2O
I + II and II + III = Diastereomers 12
Mass of carbon = × mass of CO2
OH O 44
54.
12
= × 88 g = 24 g
44
2 2
Mass of hydrogen = × mass of H2O = × 9 =1g
18 18
enol keto So, the unknown hydrocarbon contains 24 g of carbon and 1g of
CH3 hydrogen.
 2 1
55. H3C— N — C— CH2 — CH3 3. Organic compound ‘A’ contain phosphorus as it gives positive
3
  5 test with ammonium molybdate. Phosphorus present in organic
CH3 CH2CH3 compound ‘A’ get oxidised with Na 2O2 and form Na 3PO4.
4
3-(N, N-dimethyl)-3-methyl pentanamine 2P + 5Na 2O2 → 2Na 3PO4 + 2Na 2O
or 3-methyl-3-(N, N-dimethyl) pentanamine Compound Sodium phosphate
Organic Chemistry Basics 325

Na 3PO4 in presence of HNO3 form H3PO4 and NaNO3. 8. The correct option is :
Na 3PO4 + 3HNO3 → H3PO4 + 3NaNO3 (A) → (Q); (B) → (R); (C) → (S)
Upon cooling, a few drops of ammonium molybdate solution are (A) H2O and sugar mixture They do not react chemically. On
added. A yellow ppt. confirms the presence of phosphorus in the heating, solubility of sugar in H2O increases and on rapid
organic compound. cooling of saturated solution, sugar recrystallises (Q).
H3PO4 +12 (NH4 )2 MoO4 + 21HNO3 → (B) H2O and aniline mixture Aniline is steam volatile but
(NH4 )3 PO4 ⋅ 12MoO3 ↓ +21NH4NO3+12H2O insoluble in H2O. So, steam distillation (R) is employed for
Yellow ppt. their separation.
4. Higher the basicity of a base, stronger will be its nucleophilic (C) H2O and toluene mixture Toluene is steam non-volatile and
power. also insoluble in H2O. So, differential extraction method (S)
Again we know, a weaker acid produces a stronger base can be used to separate them.
(conjugate), i.e. a stronger nucleophile.
− H⊕ 9. In Dumas method, organic compound is heated with dry cupric
Acid → Conjugate base oxide in a combustion tube in the atmosphere of CO2. Upon
H3O+ H2O heating, C and H present are oxidised to CO2 and water vapours
CH3SO3H CH3SO3È while N 2 is set free. Let, the molecular formula of the organic
compound (1 mol) be C xH yN z. In Dumas method,
CH3CO2H CH3COÈ2
Cx H y N z +  2x +  CuO → xCO2 + H 2 O + N 2 +  2x +  Cu
y y z y
H2O È
OH 1 mol
 2  2 2  2 
Acid Base strength y z
x mol mol mol
strength ≡ Nucleophilicity 2 2
Thus, the correct order of nucleophilicity of given nucleophiles y z
Now, x = 6, = 4 ⇒ y = 8 and = 1⇒ z = 2
are as follows : 2 2
− ∴ Molecular formula of the compound is C6H8 N2.
H2O(2) < CH3SO3− (3) < CH3CO2− (1) < O H(4)
5. In chromatography, the expression of retention factor (R f ) is 10. Dichloromethane, DCM (CH2Cl 2 ) is heavier (density
Distance travelled by the compound from origin = 13266
. g cm−3) than water (density = 1 g cm −3 ). So, DCM and
Rf = <1 H2O will stay as lower and upper layer respectively in the
Distance travelled by the solvent from origin
separating funnel (SF).
The value of R f signifies the relative ratio of migration of each
component of the mixture with respect to the developing solvent 11. We know, a stronger acid produces its stable or weaker conjugate
used. R f value depends on the type of adsorption base. Here, CH(CN)3 produces the most stable conjugate base
chromatography like TLC (Thin-Layer Chromatography), paper (NC)3 C− . Stronger −R and −I effects of the CN − group, make the
chromatography etc. The R f value is also the characteristic of a carbanion (conjugate base) very stable. The resonance hybrid
compound (sample) for a given developing solvent at a given structure of [(NC)3 C]− is as follows:
temperature. CN
When the compound in the sample (usually less polar) is weakly δ– δ+ –
(NC) C H NC C y +H ⊕ (NC)2C C N
adsorbed the spot will travel a shorter distance from the origin 3
(–I)
and hence the R f value will be decreased.
C N (–R)
6. In column chromatography, separation of mixture of compounds
(adsorbate) takes place over a column of solid adsorbent (silica y
gel and Al 2O3 ) packed in a glass tube. N
When an appropriate eluant (liquid) is allowed to flow down the C
column, the compounds present in the mixture get adsorbed to N C C
different extent on the adsorbent column and thus complete C
separation takes place. N
Thus, column chromatography is based on the differential Resonance hybrid structure of [NC)3 C]–
adsorption of the substance on the solid phase.
However, halogen (X = Cl, Br, I) show −I effect but + R effect
7. All sites (a, b, c, d, e) of the given molecule have lone pair on of halogens, destabilises the carbanion, X 3C – (conjugate base of
N-atoms. Higher the ease of donation of lp of electrons of N, the haloform, HCX 3 ).
more favourable will be the site for protonation. Ease of donation s
of lp of e s, i.e. Lewis basicity inversely depend on the percentage X2C X (+R,–I )
of s-character in hybridisation of ‘N’ which will decide the
electronegativity of ‘N’. Thus, CH(CN)3 is the strongest acid among the given options.
At ‘a’ and ‘e’, N-atoms are sp3 (s% = 25) hybridised, whereas at ‘ 12. Estimation of nitrogen through Kjeldahl’s method is not suitable
b’, ‘c’ and ‘d’, N-atoms are sp2 (s% = 33) hybridised. So, ‘b’, ‘c’ for organic compounds containing nitrogen in ring or nitrogen in
and ‘d’ are the favourable sites for protonation nitro or azo groups. It is because of the fact that nitrogen of these
compounds does not show conversion to Ammonium sulphate
(H+ is a Lewis acid, i.e. electrons acceptor).
326 Organic Chemistry Basics

((NH4 )2 SO4 ) during the process. Hence, among the given (c) At last, calculate % of N present in organic compound using
compounds only aniline can be used suitably for estimation of formula
nitrogen by Kjeldahl’s method. Mass of N × 100
% of N =
13. Aromatic compounds are stable due to resonance while Mass of organic compound
non-aromatics are not. According to Huckel’s rule (or 4 n + 2 Mass of organic compound = 1.4 g
rule), “For a planar, cyclic compound to be aromatic, its π cloud
must contain (4 n + 2)π electrons, where, n is any whole Let it contain x m mole of N atom.
number.” Thus, Organic compound → NH3
x m mole
and 2NH3 + H2SO4 → (NH4 )2 SO4 …(i)
,
O N 6 m mole
initially taken
(6πe– system) (4πe– + 2e– system) (6πe– system)
H2SO4 + 2 NaOH → Na 2SO4 + 2 H2O ...(ii)
are aromatic and stabilised by resonance.
They follow Huckel’s rule. 2 m mole NaOH reacted.
Hence, m moles of H2SO4 reacted in Eq. (ii) = 1
is non-aromatic, hence, ⇒ m moles of H2SO4 reacted from Eq. (i) = 6 − 1= 5 m moles
least stabilised by resonance.
⇒ m moles of NH3 in Eq. (i) = 2 × 5 = 10 m moles
O ⇒ m moles of N atom in the organic compound = 10 m moles
14. Glycerol with high boiling point (290°C) can be separated from ⇒ Mass of N = 10 × 10−3 × 14 = 0.14 g
spent lye by distillation under reduced pressure. This process is Mass of N present in organic compound
% of N = × 100
used to purify liquids having very high boiling points. By this Mass of organic compound
process, liquid is made to boil at lower temperature than its 0 .14
boiling point by lowering the pressure on its surface. ⇒ % of N = × 100 = 10%
1.4
15.  OH group displays both kinds of effect; an electron
withdrawing acid-strengthening inductive effect from the 17. 18 g H2 O contains 2g H
meta-position and an electron-releasing acid weakening ∴ 0.72 g H2O contains 0.08 g H
resonance effect from the para-position (at this position,
44 g CO2 contains 12 g C
resonance effect overweighs the inductive effect).
Thus, III > IV. ∴ 3.08 g CO2 contains 0.84 g C
o-hydroxybenzoic acid (II) is far stronger than the corresponding 0.84 0.08
∴ C:H = : = 0.07 : 0.08 = 7 : 8
meta and para isomers as the carboxylate ion is stabilised by 12 1
intramolecular H-bonding.
∴ Empirical formula = C7 H8
2,6-dihydroxybenzoic acid (I) forms carboxylate ion which is
further stabilised by intramolecular H-bonding, Thus, correct 18. The order of stability of carbocation will be
order is CH2+
I > II > III > IV
δ−
O O δ− δ−
δ+ > CH2 CH CH2 > CH3 CH2 CH2
C δ+ O O δ+
H H C H
O O O Benzyl Allyl Propyl
(resonance (resonance (stabilisedby
stabilisedwithmore stabilised) +I effect)
canonicalforms)
(III) (I) (II)
II I
(Most stable) OH OH OH OH
16. THINKING PROCESS This problem is based on the estimation of
percentage of N in organic compound using Kjeldahl’s method. Use > > >
the concept of stoichiometry and follow the steps given below to solve 19.
the problem.
(a) Write the balanced chemical reaction for the conversion of N NO2 Cl CH3 OCH3
present in organic compound to ammonia, ammonia to (–M,–I ) (–I ) (+I, (+M)
ammonium sulphate and ammonium sulphate to sodium (hyperconjugation)
sulphate. (III) (I) (II) (IV)
(b) Calculate millimoles (m moles) of N present in organic
Electron releasing group decreases while electron withdrawing
compound followed by mass of N present in organic
group increases acidic strength by destabilising and stabilising
compound using the concept of stoichiometry. the phenoxide ion formed respectively.
Organic Chemistry Basics 327

SbCl 5 ⊕
20. Cl  CH  CH3 → Ph  CH  CH3 + SbCl6− —OH < Cl— —OH <
 Toluene (Carbocation) 25.
(planar)
Ph
(− ) I II
→ Ph  CH  CH3 + SbCl 5
 H3C— —COOH < —COOH
Cl
( d and l ) mixture IV III
+ I effect
21. Allene is the name given to propdiene, H2C == C == CH2 . decreases acid strength
Allene
Hybridisation of an atom is determined by determining the Cl has overall electron withdrawing effect, increases acid
number of hybrid orbitals at that atom which is equal to the strength.
number of sigma (σ ) bonds plus number of lone pairs at the H π H
concerned atom.

Pi(π ) bonds are not formed by hybrid orbitals, therefore, not 26. H—C—CH—CH2 H—C==CH—CH2
counted for hybridisation.
+
H H
H σ H 27. The σ-electron of C  H bond is delocalised with p-orbitals of
σ σ
C C C σ π-bond.
σ
H σ H
+ 3°, has resonance
O O stability
+
Here, the terminal carbons have only three sigma bonds associated I
with them, therefore, hybridisation of terminal carbons is sp3. The +
central carbon has only two sigma bonds associated, hence has no resonance stability but 2°
hybridisation at central carbon is sp. II
22. A mono-substituted benzoic acid is stronger than a +
2°, has resonance
mono-substituted phenol as former being a carboxylic acid. O O stability
+
Among the given substituted benzoic acid, ortho-hydroxy acid is III
strongest acid although —OH causes electron donation by
resonance effect which tends to decrease acid strength. 1°, has no resonance stability
+
It is due to a very high stabilisation of conjugate base by
IV
intramolecular H-bond which outweigh the electron donating
resonance effect of —OH. Therefore, overall stability order is : I > III > II > IV
O 28. The following structure has like charge on adjacent atoms,
COOH – therefore, least stable
O
H+ + +
+ O
H N
OH O –

The overall order of acid-strength of given four acids is O–


ortho-hydroxybenzoic acid (pKa = 2.98) > Toluic acid
pKa = 4.37) > p-hydroxybenzoic acid (pKa = 4.58) > 29.
—SO3H + NaHCO3 —SO3Na + H2O
p-nitrophenol (pKa = 7.15)
+ CO2
23. I is most stable because it has more covalent bonds and negative
charge on electronegative nitrogen. III is more stable than II and
O2N— —OH + NaHCO3 O2N— —ONa
IV due to greater number of covalent bonds. Between II and IV,
II is more stable since, it has negative charge on electronegative
atom and positive charge on electropositive atom. Hence, overall + H2O + CO2
stability order is 30. A spontaneous neutralisation will occur between strong acid and
I > III > II > IV strong base as
24. H at C 2 will migrate giving resonance stabilised carbocation.
H H H3C— —SO3H + CH3COONa
strong base
 +  H−
+

H3C  C  C  C  CH3 → H3C  C  CH2CH(CH3 )2 strong acid


shift
    – +
OH H CH 3 OH H3C— —SO3Na + CH3COOH
←→ H3C  C  CH2  CH(CH3 )2 weak acid
 weak base
+
OH
328 Organic Chemistry Basics
+ −
IV is amide, least basic.
31. H2C== CH  CH  CH  O  CH3 −
O O
Lone pair of oxygen is not the part of this mode of delocalisation.  • •  +
32. Carboxylic acid is stronger acid than ammonium ion, hence CH3  C  N H2 ←→ CH3  C == NH2
+ + lone pair is part of delocalisation
—COOH(X) is most acidic. Z (NH3 ) is more acidic than Y (NH3 )
Also, among alkyl amines, 2° is more basic than 1° amine.
due to – I effect of —COOH on Z. Hence, overall acid strength
order is Hence, overall order of basic strength is
X > Z > Y 1>3>2>4

33. CH3Cl has highest dipole moment. 40.


—CH2—NH2
34. H2C ==CH  C ≡≡ N
↑ ↑2 ↑ ↑ Lone pair is not taking part in resonance, most basic. In other
sp2 sp sp sp cases, lone pair of nitrogen is part of delocalisation which
decreases Lewis base strength.
35. In general, the order of acid strength is 41. A methylene (  CH2  ) with carbonyl on both side is highly
 C ≡≡ CH < ROH < H2O < PhOH < R  COOH acidic.
Therefore, during stepwise neutralisation of given acid, O O
—COOH will be neutralised first.  
In the second step, the phenolic —OH, assisted by – I effect of CH3  C  CH2  C  CH2  CH3
*
NO2 at meta position will be neutralised. ↑
has very acidic H.
HOOC O

OH 42. O— ==N
O
O 2N 2 moles of octet of nitrogen is violated.
CH NaNH2 43. Although alcohols are weaker acid than water, it is stronger than
ammonia and terminal alkynes.

OOC 44. Nitro group from para position exert electron withdrawing
OH OH resonance effect, increases acidity of phenol the most. This is
followed by meta nitrophenol in which nitro group exert electron
+ 2NH3 withdrawing effect on acidity. CH3— is an electron donating
O 2N group, decreases acid strength. Hence, the overall order is
CH IV > III > I > II
45. CH3CH−2 > NH2− > H  C ≡≡ C − > HO−
O– It is because the order of acid-strength of their conjugate acid is
CH3CH3 < NH3 < H  C ≡≡ C  H < H2O : Acid strength.
36. Weakest acid BrCH2CH2COOH has smallest dissociation
46. H  C≡≡ C  CH == CH2
constant. ↑ ↑
37. Butanoic acid forms more exhaustive H-bonds than butanol and sp sp3
butanal do not form intermolecular H-bonds. Hence, boiling
point order will be 3 > 1 > 2. 47. —CH2—NH2
38. CH3  C ≡≡ C  CH3
symmetric benzylamine
Lone pair is not involved in resonance, most basic. In all other
39. I is most basic due to formation of resonance stabilised conjugate cases, lone pair of nitrogen is involved in resonance, less basic.
acid . H
NH
+ H 
NH2
H + 48. C == C  C ≡≡ C  H : It has 7 sigma and 3 pi bonds.
H3C  C → H3C  C
H
NH2 N H2 1- butene-3-yne
••
NH2
CH3 CH3
←→ H3C  C  
+
H
NH2 49. CH3  C — OH → CH3  C+
+ − H2 O 
resonance stabilised 
conjugate acid CH3 CH3
(3°, most stable alkyl carbocation)
Organic Chemistry Basics 329


O O Oδ − 54. In both (b) and (c), all the atoms are present in one single plane
 +  δ+  sp2
50. CH 2 == CH  CH ←→ CH 2CH == CH ←→ CH 2 == CH  C  H H sp2 H
H—C C—C C C O
51. N ≡≡ C  CH == CH2 straight
H straight
1 2 C H

H
sp sp2
In (a) 1, 3-butadiene, conformational change is possible between
52. PLAN A species is said to have aromatic character if C2 — C3 bond in which atoms will be present in more than one
(a) ring is planar single plane.
(b) their is complete delocalisation of π-electrons In (d) allene, the terminals H—C—H planes are perpendicular to
(c) Huckel rule i.e. ( 4 n + 2) rule is followed. one another.
where, n is the number of rings CH3


( 4 n + 2) = π electron delocalised.
Cl 55. H3C—CH2—C—CH3
4 3 2 1


+
CH3

+ AlCl3 + AlCl4 On C2 — C3 axis, X = CH3 , Y = CH3
On C1 — C2 axis, X = H, Y = C2H5
Aromatic
P CH3
– 
56. H3C  C — CH3 µ=0
+ NaH + H2 
CH3
Aromatic H3C H
Q C == C µ≠0
O OH
H C2H5
unsymmetrical
r
O (NH4)2CO3 HCl H5C2 C2H5
O ∆, 100–115°C ; C == C µ≠0
N
Aromatic H H
H S
Aromatic CH3 CH3
R  
CH3  C — C — CH3 µ=0
n (4 n + 2) π electrons  
CH3 CH3
P – 0 2 2 symmetrical
Q – 1 6 6 (including lone pair)
O O
R – 1 6 6 (including lone pair on N)  
S – 1 6 6 57. H  C  OH H 2N  C  NH2
In all cases there is complete delocalisation of π-electrons. ↑ ↑
sp2 sp2
53. PLAN Spreading out charge by the overlap of an empty p-orbital with
an adjacent σ-bond is called hyperconjugation. This overlap CH3 O
(the hyperconjugation) delocalises the positive charge on the  
carbocation, spreading it over a larger volume, and this CH3  C — O  H CH3  C  H
stabilises the carbocation. 
Vacant sp 3 CH3 sp 3
p-orbital
r
CH3—C—CH2—H CH3—C CH2
r
H 58. Phenol is less acidic than a carboxylic acid (acetic acid). Nitro
group from para position exert electron withdrawing resonance
CH3 CH3 effect, increases acid strength. Therefore, phenol is less acidic
tertiary butyl carbocation has one vacant p-orbital, hence, it is than p-nitro phenol.
stabilised by σ-p (empty) hyperconjugation. On the other hand, methoxy group from para position, donate
⊕ – electrons by resonance effect, decreases acid strength of phenol.
H—CH
σ 2—CH==CH—CH
π 3 H CH2 ==CH—CH—CH 3 Also ethanol is weaker acid than phenol due to resonance
In 2-butene, stabilisation is due to hyperconjugation between stabilisation in phenoxide ion.
σ - π * electron delocalisation. Hence,
ethanol < p-methoxyphenol < phenol < p-nitrophenol < acetic
acid
increasing acid strength →
330 Organic Chemistry Basics

59. Statement I is incorrect; Statement II is correct. Intramolecular Pianacol-pinacolone rearrangement, occur through carbocation
H-bonding in ortho-hydroxy benzoic acid lowers the boiling point. intermediate.

60. Statement I is correct; Statement II is correct; Statement II is the Nucleophilic addition occur at sp2 (planar) carbon, generating a
correct explanation of Statement I. chiral centre, hence product will be a racemic mixture.
Intramolecular H-bonding discourage release of H+ to some S
extent, hence weaker acid than its para isomer. HS— —Cl Base
—S– —Cl + Cl
H Nucleophilic
O O– substitution
+
N 63. Hyperconjugation.
64. Less, stable free radical is formed.
O
65. Cyclic
61.
O O H
O O
A. Ph—C—CH2—CH2—C—CH3 + OH–
C
– CH3
:

O CH2 H 3C
OH OH
2O
O –H2O O
Ph Ph Ph O O
carbanion
Dehydration  
O– 66. HO  C  CH2  CH2  C  OH
O butanedioic acid
CH3MgBr
Cl Ph
B. Ph CH 67. Triangular planar; carbon is sp2-hybridised
Cl
Ph O 68. geminal, same
SN 2
H 3C 69. sp3-hybridised
+ 70. Cyclopropane: here the C—C—C bond angle is 60° while the
O 18 +
OH Ph
H –H+ requirement is 109°.
OH
C. Ph Ph HO O
:OH 71. Propene : CH2 == CH  CH3
Nucleophilic addition –H2 O+
Ph ↑ ↑
sp2 sp3
O
18
Dehydration 72. Aniline
+ Friedel-
H 73. Tert-butyl carbonium ion because the three methyl group
D. Ph –H2O Craft alkylation stabilises carbocation by + I effect.
OH +
Electrophilic 74. These are total 6α-H to sp2 carbon and they all can participate in
substitution
hyperconjugation.
+
62. + + H
—N2 + —OH + H
H+
—N==N— —OH + +
Three structures Two structures
this is an example of electrophilic substitution at para position of
75. Aromatic alcohols and carboxylic acids forms salt with NaOH,
phenol, giving a coupling product.
will dissolve in aqueous NaOH :
O H O–
COOH OH OH COOH
Ph—C—CH3 + H—Al– —H Ph—C—CH3

H H
Nucleophilic addition
OH
N
Ph—C—CH3 H3C CH3
H
Organic Chemistry Basics 331

+
NH3 Ka
p-methoxy benzoic acid 3.3 × 10− 5
p-methyl benzoic acid 4.2 × 10− 5
76. is more acidic due to – I effect of F. benzoic acid 6.4 × 10− 5
p-chlorobenzoic acid 10. 2 × 10− 5
p-nitrobenzoic acid 36.2 × 10− 5
F
80. H  C ≡≡ C  H is more acidic than CH2 == CH2.
+
OH O—H 81. Intramolecular H-bonding in ortho-hydroxybenzaldehyde
decreases its melting point as well as boiling point.
+
77. – O—H Molecules of p-hydroxybenzaldehyde is symmetrical,
associated together by intermolecular H-bonds, has higher
boiling point and melting point.

O
H
78. OH OH
OH NH2 O
Distillation
H
and Intramolecular H-bonding

COOH COOH H H H H
COOH COOH (ether layer) 82. C—C—C ≡≡ C==C==C
+
ether H H
NH3Cl– NH2 H H
solution
Allene sp sp2
HCl NaOH(aq)
83. –
+

≡≡
+
COOH COOH
(aqueous layer)
The above shown resonance introduces some double bond
character to central C—C bond. Therefore, the central C—C
bond in 1, 3-butadiene is shorter (stronger) than C—C bond in
Tests of functional groups HOOC— —OH butane.
FeCl 3 84. In case of acetate ion, both the resonance structures are
→ violet colouration confirm phenolic group equivalent and negative charge always remains on
NaHCO3 lime electronegative oxygen. These factors makes acetate ion more
→ CO2 ↑ → milky → — COOH stable than phenoxide ion in which negative charge also moves
water
on carbon atoms.
NH2 NC

O O
 −

+ CHCl3 + KOH – NH2 group confirmed. CH3  C  O ←→ CH3  C == O


—O ==O – ==O
COOH COOH
characteristic
foul smell of isocyanide
==O
79. p-methoxy benzoic acid is the weakest and p-nitrobenzoic acid is –
the strongest acid among these acids. Chloro group has overall
electron withdrawing effect on ring, therefore, increases acid 85. p-nitroaniline < aniline < p-toluidine < N,N-dimethyl-p-toluidine.
strength of benzoic acid. Methyl group decreases acid strength of Nitro group, by electron withdrawing resonance effect decreases
benzoic acid by + I effect. Therefore, the basic strength. Methyl group by electron donating inductive
effect, increases basic strength.
22
Hydrocarbons
CH3 Cl , hv
Topic 1 Saturated Hydrocarbons 5. →
2
N (isomeric products) C5 H11 Cl

Objective Questions I (Only one correct option) H 3C


1. Which of the following reactions produce(s) propane as a CH3
major product? (2019 Adv.)
Fractional distillation
(a) H3C Cl Zn, dil. HCl → M (isomeric products)
Br What are N and M ? (2006, 5M)
(a) 6, 6 (b) 6, 4
(b) H3C Br Zn
(c) 4, 4 (d) 3, 3

(c) H3C COONa NaOH, CaO, D


6. 1-bromo-3-chlorocyclobutane when treated with two
equivalents of Na, in the presence of ether which of the
(d) H3C COONa + H2O Electrolysis
following will be formed? (2005, 1M)
Br Cl
2. In the following skew conformation of ethane,
H ′ C C H ′′ dihedral angle is (2019 Main, 12 April II) (a) (b)
H
H H′ 29°
H (c) (d)

H′′ H 7. How many chiral compounds are possible on mono


chlorination of 2-methyl butane ? (2004, 1M)
(a) 58° (b) 149° (c) 151° (d) 120° (a) 2 (b) 4 (c) 6 (d) 8
3. Which of these factors does not govern the stability of a 8. Consider the following reaction
conformation in acyclic compounds? (2019 Main, 10 April II) •
(a) Electrostatic forces of interaction H3 C  CH  CH  CH3 + Br → X + HBr
(b) Torsional strain  
(c) Angle strain D CH3
(d) Steric interactions
Identify the structure of the major product X (2002, 3M)
4. Isomers of hexane, based on their branching, can be divided • •
(a) H3C CH CH  CH2 (b) H3C CH C CH3
into three distinct classes as shown in the figure. (2014 Adv.)    
D CH3 D CH3
and • •
I. and II. (c) H3C C  CH  CH3 (d) H3C CH  CH  CH3
  
D CH3 CH3
III.
9. Benzyl chloride (C6H5CH2Cl) can be prepared from toluene
by chlorination with (1998)
The correct order of their boiling point is (a) SO2Cl2 (b) SOCl2
(a) I > II > III (c) Cl2 (d) NaOCl
(b) III > II > I
(c) II > III > I 10. (CH3)3CMgCl on reaction with D2O produces (1997)
(d) III > I > II (a) (CH3)3CD (b) (CH3)3OD
(c) (CD3)3CD (d) (CD3)3OD
Hydrocarbons 333

11. When cyclohexane is poured on water, it floats because 15. Which of the following compounds does not dissolve in
(a) cyclohexane is in ‘boat’ form (1997, 1M) conc. H2 SO4 even on warming ?
(b) cyclohexane is in ‘chair’ form (1983, 1M)
(c) cyclohexane is in ‘crown’ form (a) Ethylene (b) Benzene
(d) cyclohexane is less dense than water (c) Hexane (d) Aniline
12. The C  H bond distance is the longest in (1989, 1M) 16. The compound with highest boiling point is (1982, 1M)
(a) C2H2 (b) C2H4 (a) 2-methyl butane (b) n -pentane
(c) C2H6 (d) C2H2Br2 (c) 2, 2-dimethyl propane (d) n-hexane
13. The compound which has one isopropyl group, is (1989, 1M) 17. Marsh gas mainly contains
(a) 2,2,3,3-tetramethyl pentane (b) 2,2-dimethyl pentane (1980, 1M)
(c) 2,2,3-trimethyl pentane (d) 2-methyl pentane (a) C2H2 (b) CH4 (c) H2S (d) CO
14. The highest boiling point is expected for
(1986, 1M)
Integer Answer Type Question
(a) iso-butane (b) n-octane 18. The maximum number of isomers (including stereoisomers)
(c) 2, 2, 3, 3-tetramethyl butane (d) n-butane that are possible on mono-chlorination of the following
compound, is CH3 CH2 CH( CH3 ) CH2 CH2 . (2011)

Topic 2 Unsaturated Hydrocarbons


Objective Questions I (Only one correct option) (c) CH3—C—CH2CH2CH3
1. Consider the following reactions: (2020 Main, 6 Sep I)
C
Ozonolysis
( A ) → ( B ) + (C ) H3 C CH3
(C7 H 14)

(I2 + NaOH) (d) CH2 == C  CH2CH2 CH3


‘B’ ∆
yellow ppt. 
CH(CH3 )2
Ag2O
Silver mirror 3. Consider the following reactions,

Ag2O
(I2 + NaOH) ppt.
no yellow ppt. ∆

(C ) A
Hg2+/H+ NaBH4 ZnCl2
LiAlH4 Anhydrous ZnCl2 B C Turbidity within
(D) Gives white Conc. HCl 5 minutes
∆ and conc. HCl turbidity within
5 minutes A is (2019 Main, 12 April II)
(a) CH ≡≡ CH (b) CH3  C ≡≡ C  CH3
(A) is
(c) CH3  C ≡≡ CH (d) CH2 == CH2
(a) (b) 4. But-2-ene on reaction with alkaline KMnO4 at elevated
temperature followed by acidification will give
(2019 Main, 12 April I)
(c) (d)
(a) CH3  CH CH  CH3
 
2. The major product [B] in the following sequence of reactions OH OH
is (2020 Main, 8 Jan II) (b) one molecule of CH3CHO and one molecule of CH3COOH
(c) 2 molecules of CH3COOH
(i) B2H6
CH 2 C == CH  CH 2CH3 → È
[A ] (d) 2 molecules of CH3CHO
 (ii) H2O2 , OH 5. The major product of the following addition reaction is
CH(CH3 )2 Cl 2 / H 2O
Dil. H2SO4
→ [B ] H3C CH == CH 2 →

(2019 Main, 12 April I)
(a) CH3 C CH == CH CH3 (a) CH3  CH  CH2 (b) CH3  CH  CH2
    
CH(CH3 )2 Cl OH OH Cl
O O
(b) CH3 C == CH  CH 2CH3
 (c) H3C (d)
CH(CH3 )2 H3C CH3
334 Hydrocarbons

6. The major product of the following reaction is 14. The number of optically active products obtained from the
(i) DCl (l equiv.)
CH 3C ≡≡ CH     → complete ozonolysis of the given compound, is (2012)
(ii) DI (2019 Main, 9 April I) CH3 H
(a) CH3CD(Cl)CHD(I) (b) CH3CD2CH(Cl)(I)
(c) CH3CD(I)CHD(Cl) (d) CH3C(I)(Cl)CHD2 CH3 CH CH C CH CH C CH CH CH2
7. Which one of the following alkenes when treated with HCl
yields majorly an anti Markownikov product? H CH3
(2019 Main, 8 April II) (a) 0 (b) 1 (c) 2 (d) 4
(a) Cl  CH == CH2 (b) H2N  CH == CH2 15. The synthesis of 3-octyne is achieved by adding a
(c) CH3O  CH == CH2 (d) F3C CH == CH2 bromoalkane into a mixture of sodium amide and an alkyne.
8. The correct order for acid strength of compounds The bromoalkane and alkyne respectively are (2010)
(a) BrCH2CH2CH2CH2CH3 and CH3CH2C ≡≡ CH
CH ≡≡ CH, CH3  C ≡≡ CH and CH 2 == CH 2 is as follows :
(2019 Main, 12 Jan I) (b) BrCH2CH2CH3 and CH3CH2CH2C ≡≡ CH
(a) CH3  C≡≡ CH > CH 2 == CH 2 > HC≡≡ CH (c) BrCH2CH2CH2CH2CH3 and CH3C ≡≡ CH
(b) CH3 −− C ≡≡ CH > CH ≡≡ CH > CH 2 == CH 2 (d) BrCH2CH2CH2CH3 and CH3CH2C ≡≡ CH
(c) HC ≡≡ CH > CH3 −− C ≡≡ CH > CH 2 == CH 2 16. The number of stereoisomers obtained by bromination of
(d) CH≡≡ C H > CH2 == CH2 > CH3 −− C≡≡ CH trans-2-butene is (2007)
9. The trans-alkenes are formed by the reduction of alkynes (a) 1 (b) 2 (c) 3 (d) 4
with (2018 Main) 17. The reagent(s) for the following conversion,
(a) H2-Pd/C, BaSO4 (b) NaBH4 Br ?
(c) Na/liq. NH3 (d) Sn-HCl H H
Br
10. The reaction of propene with HOCl (Cl2 + H2O) proceeds is/are (2007, 3M)
through the intermediate (2016 Main) (a) alcoholic KOH
+ (b) alcoholic KOH followed by NaNH2
(a) CH3  CH  CH2  Cl
(c) aqueous KOH followed by NaNH2
+
(b) CH3  CH(OH)  CH2 (d) Zn/CH3OH

(c) CH3  CHCl  CH2


+ 18. CH3 — CH == CH2 + NOCl → P; Identify the adduct.
+
(2006, 3M)
(d) CH3  CH  CH2  OH (a) CH3 —CH—CH2 (b) CH3— CH—CH2
   
11. Which of the following compounds will exhibit geometrical Cl NO NO Cl
isomerism? (2015 Main) Cl
(a) 1-phenyl-2-butene 
(c) CH3—CH2  CH (d) CH2 —CH2—CH2
(b) 3-phenyl-1-butene   
(c) 2-phenyl-1-butene NO NO Cl
(d) 1, 1-diphenyl-1-propane
19. Cyclohexene is best prepared from cyclohexanol by which of
12. Which compound would give 5-keto-2-methyl hexanal upon the following? (2005)
ozonolysis? (2015 Main) (a) conc. H3PO4 (b) conc. HCl / ZnCl2
CH3 CH3 (c) conc. HCl (d) conc. HBr
CH3 20. 2-hexyne gives trans-2-hexene on treatment with (2004, 1M)
(a) (b) (a) Li/NH3 (b) Pd/BaSO4 (c) LiAlH4 (d) Pt/H2
21. 2-phenyl propene on acidic hydration, gives (2004, 1M)
CH3
(a) 2-phenyl-2-propanol (b) 2-phenyl-1-propanol
CH3 CH3 (c) 3-phenyl-1-propanol (d) 1-phenyl-2-propanol
Hg 2+
(c) (d) 22. Ph  C ≡≡ C  CH3 → A; A is
H+ (2003, 3M)
O
Ph Ph
CH3 (a) (b) ==O
H 3C H 3C
13. The major organic compound formed by the reaction of
OH
1,1,1-trichloroethane with silver powder is (2014 Main) Ph Ph
(a) acetylene (b) ethene (c) (d) —OH
(c) 2-butyne (d) 2-butene H 3C H3C
Hydrocarbons 335

23. Identify a reagent from the following list which can easily (b) CH3CHBrCH2 Br
distinguish between 1-butyne and 2-butyne. (2002, 3M)
(a) bromine, CCl 4 (b) H2 , Lindlar catalyst
(c) dilute H2SO4 , HgSO4 (d) ammoniacal CuCl 2 solution (c) CH3CH2CHBr OH
24. In the presence of peroxide, hydrogen chloride and hydrogen
iodide do not give anti-Markownikoff’s addition to alkenes (d) CH3CH2CHBr Br
because (2001, 1M)
(a) both are highly ionic 32. Which one of the following has the smallest heat of
(b) one is oxidising and the other is reducing hydrogenation per mole ? (1993, 1M)
(c) one of the steps is endothermic in both the cases (a) 1-butene (b) trans-2-butene
(d) all the steps are exothermic in both the cases (c) cis-2-butene (d) 1, 3-butadiene
25. The reaction of propene with HOCl proceeds via the addition 33. The number of structural and configurational isomers of a
of (2001) bromo compound, C5 H9 Br, formed by the addition of HBr to
(a) H+ in the first step (b) Cl+ in the first step 2-pentyne respectively, are (1988, 1M)
(c) OH– in the first step (d) Cl+ and OH– single step (a) 1 and 2 (b) 2 and 4 (c) 4 and 2 (d) 2 and 1
26. Hydrogenation of the adjoining compound in the presence of 34. Acidic hydrogen is present in (1985, 1M)
poisoned palladium catalyst gives (a) ethyne (b) ethene (c) benzene (d) ethane
H3C H 35. Baeyer’s reagent is (1984, 1M)
CH3
(a) alkaline permanganate solution
(b) acidified permanganate solution
H 3C H
H (2001, 1M) (c) neutral permanganate solution
(a) an optically active compound (d) aqueous bromine solution
(b) an optically inactive compound 36. When propyne is treated with aqueous H2 SO4 in the presence
(c) a racemic mixture of HgSO4 , the major product is (1983, 1M)
(d) a diastereomeric mixture (a) propanal (b) propyl hydrogen sulphate
27. Propyne and propene can be distinguished by (2000) (c) acetone (d) propanol
(a) conc. H2SO4 (b) Br2 in CCl4 37. The compound 1, 2-butadiene has (1983, 1M)
(c) dil. KMnO4 (d) AgNO 3 in ammonia (a) only sp-hybridised carbon atoms
28. Which one of the following alkenes will react fastest with H2 (b) only sp2-hybridised carbon atoms
(c) both sp and sp2-hybridised carbon atoms
under catalytic hydrogenation condition ? (2000, 1M)
(d) sp, sp2 and sp3-hybridised carbon atoms
R R R H
(a) (b) 38. Which of the following will decolourise alkaline KMnO4
H H R H solution? (1980, 1M)
R R R R (a) C3H8 (b) CH4 (c) CCl 4 (d) C2H4
(c) (d)
R H R R Objective Questions II
29. The product(s) obtained via oxymercuration (One or more than one correct option)
(HgSO4 + H2 SO4 ) of 1-butyne would be (1999, 2M) 39. The correct statement(s) for the following addition reactions
O is (are) (2017 Adv.)

(a) CH3  CH2  C  CH3 H 3C H
(b) CH3  CH2  CH2  CHO (i)
Br2/CHCl3
M and N
(c) CH3  CH2  CHO + HCHO
(d) CH3  CH2  COOH + HCOOH H CH3

30. In the compound, H3C H


H , the C2-C3 bond is of the Br2/CHCl3
type (1999, 2M) (ii) O and P
2 3 3 3 2 3
(a) sp − sp (b) sp − sp (c) sp − sp (d) sp − sp H CH3

31. The reaction of CH3 CH==CH OH with HBr (a) (M and O) and (N and P) are two pairs of enantiomers
gives (1998, 2M) (b) Bromination proceeds through trans-addition in both the
reactions
(a) CH3CHBrCH2 OH (c) O and P are identical molecules
(d) (M and O) and (N and P) two pairs of diastereomers
336 Hydrocarbons

Assertion and Reason Passage 2


Read the following questions and answer as per the direction An acyclic hydrocarbon P, having molecular formula C 6H10, gave
given below : acetone as the only organic product through the following sequence
(a) Statement I is correct Statement II is correct Statement II is a of reactions, in which Q is an intermediate organic compound.
correct explanation of Statement I. (i) dil. H2SO4/HgSO4
P
(b) Statement I is correct Statement II is correct Statement II is (ii) NaBH4/ethanol
(C6H10) (iii) dil. acid
not the correct explanation of Statement I.
(c) Statement I is correct Statement II is incorrect. (i) dil. H2SO4(catalytic
O
amount) (–H2O)
(d) Statement I is incorrect Statement II is correct. Q 2 C
(ii) O3 /ethanol
40. Statement I Addition of bromine to trans-2-butene yields (iii) Zn/H2O H3C CH3 (2011)
meso-2, 3-dibromo butane.
Statement II Bromine addition to an alkene is an
electrophilic addition. (2001, 1M) 46. The structure of the compound Q is
41. Statement I Dimethyl sulphide is commonly used for the H3C OH H3C OH
 
reduction of an ozonide of an alkene to get the carbonyl (a) H  C C  CH2CH3 (b) H3C C C CH3
compound.  
H3C
H H3C H
Statement II It reduces the ozonide giving water soluble
H3C OH OH
dimethyl sulphoxide and excess of it evaporates. (2001, 1M)  
(c) H C — CH2 CH CH3 (d) CH3CH2CH2 CH CH2CH3
42. Statement I 1-butene on reaction with HBr in the presence
of a peroxide produces 1-bromobutane. H3C
Statement II It involves the formation of a primary radical. 47. The structure of compound P is
(2000, 1M)
(a) CH3CH2CH2CH2  C ≡≡ C H
43. Statement I Addition of Br2 to 1-butene gives two optical
(b) H3CH2C C ≡≡ C CH2CH3
isomers.
H3C
Statement II The product contains one asymmetric carbon.
(c) H C C ≡≡ C CH3
(1998 , 1M)
H3C

Passage Based Questions H3C


Passage 1 (d) H3C C C ≡≡ C H
H3C
Pd-BaSO4 (i) B2H6
C8H6 C8H8 X
H2 (ii) H2O2,NaOH, H2O Passage 3
H2O
HgSO4,H2SO4 Schemes 1 and 2 describe sequential transformation of alkynes M
C8H8
(i) EtMgBr, H2O and N. Consider only the major products formed in each step for
Y
(ii) H+, Heat (2015 Adv.) both schemes.
44. Compound X is 48. The correct statement with respect to product Y is
O OH (a) it gives a positive Tollen’s test and is a functional isomer of X
(b) it gives a positive Tollen’s test and is a geometrical isomer of X
(a) CH3 (b) CH3
(c) it gives a positive iodoform test and is a functional isomer of X
(d) it gives a positive iodoform test and is a geometrical isomer of X
OH
CHO 49. The product X is
(c) (d)
H3CO H

45. The major compound Y is (a) (b)

H H H
CH3 CH3 H3CO
(a) (b)
CH3 H
CH3 CH3CH2O
CH3
CH3 (c) (d)
(c) (d) H
CH3CH2O
H H
Hydrocarbons 337

Fill in the Blanks 61. (a) Identify (A), (B), (C), (D) and (E) in the following schemes and
write their structures :
50. 1,3-butadiene with bromine in molar ratio of 1 : 1 generate
predominantly …… . (1997, 1M) Br2/Cl4

51. Addition of water to acetylene compounds is catalysed by


…… and …… (1993, 1M) NaNH2 HgSO4/H2SO4
A B C
52. Kolbe’s electrolysis of potassium succinate gives CO2 and
H2 NHNCONH2 NaOD/D O (excess)
…… . (1993, 1M) C → D →
2
E
53. The terminal carbon atom in 2-butene is …… hybridised. (b) Identify (X), (Y) and (Z) in the following synthetic scheme and
(1985, 1M) write their structures. Explain the formation of labelled
formaldehyde (H2C*O) as one of the products when compound
54. Acetylene is treated with excess sodium in liquid ammonia. (Z) is treated with HBr and subsequently ozonolysed. Mark the
The product is reacted with excess methyl iodide. The final C* carbon in the entire scheme. BaC*O3 + H2SO4 → (X) gas
product is …… . (1983, 1M) [C* denotes C14]
(i) Mg/ether LiAlH
55. …… is most acidic. (Ethane, Ethene, Ethyne) (1981, 1M) H2C == CH  Br → (Y ) →
4
(Z )
(ii) X
(iii) H3 O+ (2001)
True/False 62. An alkene (A) C16H16 on ozonolysis gives only one product
56. Moist ethylene can be dried by passing it through (B) C8H8O. Compound (B) on reaction with NaOH/I2 yields
concentrated sulphuric acid. (1981) sodium benzoate. Compound (B) reacts with KOH/NH2NH2
yielding a hydrocarbon (C) C8H10. Write the structures of
Integer Answer Type Question compounds (B) and (C). Based on this information two
isomeric structures can be proposed for alkene (A). Write
57. The total number of cyclic isomers possible for a their structures and identify the isomer which on catalytic
hydrocarbon with the molecular formula C4 H6 is (2010) hydrogenation (H2/Pd - C) gives a racemic mixture. (2001)

Subjective Questions 63. What would be the major product in the following reaction?
58. Monomer A of a polymer on ozonolysis yields two moles of H2
HCHO and one mole of CH3 COCHO. CH3 Lindlar’s catalyst
(a) Deduce the structure of A. (2000, 1M)
(b) Write the structure of all cis form of polymer of compound A.
64. Complete the following reactions with appropriate reagents :
(2005, 2M)
59. A biologically active compound, Bombykol (C16H30O) is (a)
obtained from a natural source. The structure of the ==
compound is determine by the following reactions.
(a) On hydrogenation, Bombykol gives a compound D D OH CH3
A, C16H34O, which reacts with acetic anhydride to (b) CH3 C==C
give an ester. H3C D—C—C—H
H3C CH3
(b) Bombykol also reacts with acetic anhydride to H 3C D
CH3 CH3
give another ester, which on oxidative ozonolysis (1999)
(O3 /H 2O2 ) gives a mixture of butanoic acid, oxalic
65. Complete the following reactions with appropriate structures
acid and 10-acetoxy decanoic acid.
of products/reagents :
Determine the number of double bonds in
Br2 (i) NaNH (3 equivalent)
Bombykol. Write the structures of compound A C6H5CH == CH2 → [ A ] →
2
[B]
and Bombykol. How many geometrical isomers (ii) CH3I (1998, 2M)
are possible for Bombykol? 66. Write the intermediate steps for each of the following
60. Identify X, Y and Z in the following synthetic scheme and reactions :
write their structures. H 3O +
(i) NaNH
(i) C6 H5 CH(OH)C ≡≡ CH → C6 H5  CH == CH  CHO
CH3 CH2  C ≡≡ C  H →
2
(ii) CH3CH2Br H+
H2 /Pd BaSO4 Alkaline KMnO4 (ii)
X → Y→ Z
OH O CH3
Is the compound Z optically active? Justify your answer.
(2002)
338 Hydrocarbons

67. The hydrocarbon A, adds one mole of hydrogen in the 76. Give a chemical test and the reagents used to distinguish
presence of a platinum catalyst to form n-hexane. When A is between cyclohexane and cyclohexene. (1991, 1M)
oxidised vigorously with KMnO4 , a single carboxylic acid,
77. A white precipitate was formed slowly when silver nitrate
containing three carbon atoms, is isolated. Give the structure
was added to compound A with molecular formula C6 H13 Cl.
of A and explain. (1997, 2M)
Compound A on treatment with hot alcoholic potassium
68. Complete the following, giving the structures of the principal hydroxide gave a mixture of two isomeric alkenes B and C,
organic products : (1997, 1M) having formula C6 H12 . The mixture of B and C, on
ozonolysis, furnished four compounds
Ph H
(i) CH3 CHO (ii) C2 H5 CHO
A
(i) + KNH2 (iii) CH3 COCH3 (iv) CH3  CH  CHO
|
Ph Br CH3
R R
What are the structures of A, B and C ? (1986, 4M)
B
(ii) + HClO4 78. How would you convert acetylene to acetone? (1985, 1M)
[R = nPr]
79. Give the chemical test to distinguish between 2-butyne and
O
1-butyne. (1985, 1M)
C 80. Following statements are true, only under some specific
(iii) + CHBr3 + t-BuOK
conditions. Write the condition for each subquestion in not
69. An alkyl halide, X, of formula C6 H13 Cl on treatment with more than two sentences :
potassium tertiary butoxide gives two isomeric alkenes Y and (i) 2-methyl propene can be converted into isobutyl
Z (C6 H12 ) . Both alkenes on hydrogenation gives bromide by hydrogen bromide.
2, 3-dimethyl butane. Predict the structures of X, Y and Z. (ii) Ethyne and its derivatives will give white precipitate
(1996, 3M)
with ammoniacal silver nitrate solution.
70. Give the structure of the major organic products obtained (1984,1M × 2 = 2M)
from 3-ethyl-2-pentene under each of the following reaction 81. Give reasons for the following in one or two sentences :
conditions : (1996)
(i) Methane does not react with chlorine in the dark.
(a) HBr in the presence of peroxide
(ii) Propene reacts with HBr to give isopropyl bromide but
(b) Br2 / H2O
does not give n-propyl bromide. (1983,1M × 2 = 2M)
(c) Hg(OAc)2 / H2O, NaBH4
82. State with balanced equation, what happens when “propene is
71. Write down the structure of the stereoisomers formed when bubbled through a hot aqueous solution of potassium
cis-2-butene is reacted with bromine. (1995) permanganate.”? (1982, 1M)

72. An organic compound E (C5 H8 ) on hydrogenation gives 83. One mole of a hydrocarbon A reacts with one mole of
compound F (C5 H12 ). Compound E on ozonolysis gives bromine giving a dibromo compound, C5 H10 Br2 . Compound
formaldehyde and 2-keto propanal. Deduce the structure of A on treatment with cold dilute alkaline potassium
compound E. (1995) permanganate solution forms a compound, C5 H12 O2 . On
ozonolysis A gives equimolar quantities of propanone and
73. When gas A is passed through dry KOH at low temperature, a
ethanal. Deduce the structural formula of A. (1981, 1M)
deep red coloured compound B and a gas C are obtained. The
gas A, on reaction with but-2-ene, followed by treatment with 84. Write the structural formula of the major product in each of
Zn / H2 O yields acetaldehyde. Identify A, B and C. (1994, 3M) the following cases
(i) Ethene mixed with air is passed under pressure over a
74. Give the structures of A, B and C (explanation are not required)
silver catalyst.
(i) A (C4H8 ) which adds on HBr in the presence and in the absence
(ii) The compound obtained by hydration of ethyne is
of peroxide to give same product. treated with dilute alkali. (1981 , 2 × 1 / 2 M = 1M)
(ii) B (C4H8 ) which when treated with H2O / H2SO4 gives C4H10O
which cannot be resolved into optical isomers.
85. Outline the reaction sequence for the conversion of ethene to
ethyne (the number of steps should not be more than two).
(iii) C (C6H12 ), an optically active hydrocarbon which on catalytic
hydrogenation gives an optically inactive compound C6H14. (1981, 1M)
(1993,1M × 3 = 3M) 86. Give one characteristic test which would distinguish CH4
75. Write the balanced chemical equation for the following from C2 H2 . (1979, 1M)
“Ethylene glycol is obtained by the reaction of ethylene with
potassium permanganate.” (1991, 1M)
Answers
Topic 1 17. (b) 18. (a) 19. (a) 20. (a)
1. (a,c) 2. (b) 3. (c) 4. (b) 21. (a) 22. (a) 23. (d) 24. (c)
5. (b) 6. (d) 7. (c) 8. (b) 25. (b) 26. (b) 27. (d) 28. (a)
9. (c) 10. (a) 11. (d) 12. (c) 29. (a) 30. (d) 31. (c) 32. (b)
13. (d) 14. (b) 15. (c) 16. (d) 33. (b) 34. (a) 35. (a) 36. (c)
37. (d) 38. (d) 39. (a,b,d) 40. (b)
17. (b) 18. (8)
41. (a) 42. (c) 43. (a) 44. (c)
Topic 2 45. (d) 46. (b) 47. (d) 48. (c)
1. (d) 2. (c) 3. (c) 4. (c) 49. (a) 50. (3,4-dibromo-1-butene)
5. (b) 6. (d) 7. (d) 8. (c) 51. H 2SO 4 , HgSO 4 52. (ethene) 53. (sp 3) 54.
9. (c) 10. (a) 11. (a) 12. (b) (2-butyne)
13. (c) 14. (a) 15. (d) 16. (a) 55. Terminal alkyne (ethyne) 56. (False) 57. (5)

Hints & Solutions


Topic 1 Saturated Hydrocarbons
1. The given reactions takes place as follows: responsible for the stability or energy barriers of conformers. In
cyclic compounds, all types of strains may be present.
Cl Zn, dil. HCl
CH3 CH2 CH3
(a) CH3 (Propane) Compound Type of strains/forces
a+ c
Br
Br Zn
CH3 CH CH2 + ZnBr2 a+ b+ c
(b) CH ∆
3
(Elimination (Propene)
reaction) OH F a+ b+ c+ d

CH3 NaOH, CaO, D


CH3 CH2 CH3 + CO2
Electrolysis
(c) COONa (Propane)
In a cyclic or open-chain compounds, angle strain (c) is absent. e.g.
(d) CH3 Electrolysis
CH3 (CH2)4 CH3
Compound Types of strains/forces
COONa n-hexane
H a
Thus options (a,c) are correct. H

2. A dihedral angle is the angle between twoC  H bonds projected H


on a plane orthogonal to the C C bond. In the given skew H H
conformation, having Newman’s projection the dihedral angle is H
H′ C C  H′′ CH3 a+ b
= (H′ C C  Ha ) + (Ha C C  H′′ ) CH3
H
= 29°+120° = 149°
H H H
H
H H′
29º (ethane, C2H6) OH a+ b+ d
Ha
H OH
120º
H′′ H
b H H
H
3. The four types of strains viz (a) electrostatic force of attraction,
(b) torsional strain, (c) angle strain, (d) steric stain, are 4. PLAN This problem is based on boiling point of isomeric alkanes.
As we know more the branching in an alkane, lesser will be its
surface area and lesser will be the boiling point
340 Hydrocarbons

9. Toluene on treatment with Cl 2 in the presence of heat or light


undergo free-radical chlorination at benzylic position, giving
and benzyl chloride
CH3 CH2Cl
III II
hv
+ Cl2 + HCl
and or heat
Toluene Benzyl chloride
I
CH3
On moving left to right (III to I) 
• branching increases
10. (CH3)3 CMgCl + D2O → CH3  C — D + Mg(OD)Cl

• surface area decreases CH3
• boiling point decreases
11. Alkanes are all less dense than water, floats over water.
Hence, correct choice is (b).
12. C—H bond with sp3-C will be longest in C2H6.
5. CH3 CH3
CH3 Cl2 CH2Cl CH3 CH3 CH3
hν +   
H 3C CH3 13. CH3  C — C — CH2  CH3 CH3  C — CH2CH2CH3
I   
CH3 CH3 CH3
CH3 CH3 CH2Cl 2, 2, 3, 3-tetramethyl pentane 2, 2-dimethyl pentane
CH3 + CH3 + CH3 (no isopropyl group) (no isopropyl group)
H3C H 3C Cl CH3
Cl CH3 CH3 CH3
(II + III) IV (V + VI)   
chiral chiral CH3  C — CH— CH2CH3 CH3  CH — CH2CH2CH3
Since, fractional distillation cannot separate enantiomers 
CH3 Isopropyl group
(II + III and V + VI), M = 4 and N = 6. 2, 2, 3-trimethyl pentane 2-methyl pentane
(no isopropyl group)
Cl
Na / ∆ 14. Boiling point of alkane increases with molar mass. Among
6. Wurtz’s reaction isomeric alkanes, branching decreases boiling point. Therefore,
Br n-octane has highest boiling point, higher than
1-bromo-3-chlorocyclobutane 2, 2, 3, 3-tetramethyl-butane (an isomer of n-octane).
CH3 15. Ethylene absorb H2SO4 forming CH3  CH2OSO3H and dissolve.
CH3 hν Benzene, with warm H2SO4, undergo sulphonation and dissolve.
7. + Cl2
H 3C I Cl Aniline, with H2SO4, forms anilinium sulphate salt and dissolve.
2-methyl butane Hexane, a hydrophobic molecule, does not react with H2SO4,
Cl remains insoluble.
16. Among alkanes, boiling point increases with molar mass.
+ + + Among isomeric alkanes, branching decreases boiling point.
Cl Therefore, n-hexane has highest boiling point among these.
Cl
II III IV
17. Methane is produced due to the decay of vegetables or animal
Out of the four products formed above, II and IV are chiral, organisms present in swamps and marsh, by the action of bacteria.
produced in pairs, giving total of six mono-chlorination Due to this method of formation, methane is also known as marsh
products. gas.
18. Cl +
8. Bromination is highly selective, occur at the carbon, where the *

most stable free radical is formed : I


Cl
CH3  CHD  CH — CH3 + Br• → Cl
 * * + +
CH3
• II III IV Cl
CH3  CHD  C— CH3 + HBr I has one chiral carbon = two isomers
 II has two chiral carbons and no symmetry = four isomers.
CH3
(a tertiary free radical)
III and IV have no chiral carbon, no stereoisomers.
Hydrocarbons 341

Step 2 Hg2+
Topic 2 Unsaturated Hydrocarbons CH3 C CH dil. H SO
2 4
1. O O
OH Tauto-
Ozonolysis O
+ merisation
CH3 C CH2 CH3 C CH3
(B )
(A ) (C ) (Propan-2-one)
(B)
Alkene (A) undergoes ozonolysis and gives an aldehyde and NaBH4
ketone.
CH3 CH CH3
I2+NaOH – + OH
(B) ∆
H—C—ONa + CHI3 (Yellow ppt.)
(C) (2° alcohol)
Ag2O

CH3COO– + Ag (↓) (Silver mirror) Conc. HCl
+ ZnCl2
I2+NaOH
(C) No yellow ppt. CH3 CH CH3

OH
Anhydrous
LiAIH4
C2H5—CH—C2H5
ZnCl2 Cl
∆ Conc. HCl Turbidity within 5 minutes
(D ) (Insoluble in Lucas reagent)
C2H5—CH—C2H5 In step-1, prop-1-yne reacts with Ag2O to form
Cl CH3  C ≡≡ C  Ag, that forms white precipitates.
(White turbidity) In step 2, prop-1-yne in presence of mercuric sulphate and
dil ⋅ H2SO4 produces carbonyl compound (CH3 )2 C == O which
Methyl ketones react with I2 + NaOH , so option (a) and (c) are
produces (CH3 )2CH  OH in presence of NaBH4. 2ºalcohol on
incorrect, while option (b) only gives aldehydes, thus (b) also
reaction with Lucas reagent produces turbidity in about 5 min.
incorrect. Hence, the correct option is (d).
4. But-2-ene on reaction with alkaline KMnO4 at elevated
2.
temperature followed by acidification will give acetic acid
OH
(CH3COOH). Hot alkaline solution of potassium permanganate
(i) B2H6
CH3—C == CH—CH2CH3 – CH3—CH—CH—CH2CH3 followed by acidification oxidatively cleaved alkenes. The
(ii) H2O2/OH
reaction proceed as follows :
CH Anti-Markownikoff CH
hydration Alk. KMnO4 , heat
CH3 CH3 CH3 CH3 CH3 CH == CH CH3 → 2CH3COOH
But-2 -ene H3O+ Acetic acid
(A)

+
5. The major product of the given addition reaction is
(+H+) ∆
(A) + dil. H2SO4 CH3—CH—CH—CH2—CH3 H3C  CH  CH2.
–H2O  
(Protonation of OH CH (2° carbocation)
OH Cl
group and removal of H2O)
CH3 CH3 In this reaction, H2O is used as a solvent and the major product of
Carbocation the reaction will be a vicinal halohydrin. A halohydrin is an
rearrangement organic molecule that contains both OH group and a halogen. In
2º– to 3º
by H shift a vicinal halohydrin, the OH and halogen are bonded to adjacent
carbons.
β β
+ Cl /H O
CH3—C—CH2—CH2—CH3 + CH3—C——CH2—CH2CH3 H3 C  CH == CH2 →
2 2
H3C  CH  CH2 +
–H
(3° carbocation)  
C From b-position with CH OH
least number of Cl
CH3 CH3 H-atoms forms major CH3 CH3 A chlorohydrin (major product)
product (Saytzeff's rule)
[B] CH3  CH  CH2
 
3. According to the given conditions, the compound should be Cl Cl
alkyne with triple bond present at the terminal. The chemical
(Minor product)
reactions involved are as follows:
The reaction proceeds through following mechanism :
Step 1 +
Ag2O Cl
CH3 C CH CH3 C C Ag Slow H2 O
CH3—CH=
=CH2 + Cl—Cl CH3—CH—CH2 Fast
Prop-1-yne (Precipitates)
(A) Chloronium –
+ Cl
ion
342 Hydrocarbons

H2O + 8. Ethene (H2C == CH2 ) is sp 2-hybridised and ethyne


CH3—CHCH2—Cl Fast
CH3CH—CH2—Cl + H3O (HC ≡≡ CH) is sp-hybridised. In ethyne, the sp-hybridised
+
| |
OH OH carbon atom possesses maximum s-character and hence,
| Chlorohydrin
maximum electronegativity. Due to which, it attracts the shared
H electron pair of CH bond to a greater extent and makes the
removal of proton easier. Hence, alkyne is much more acidic than
6. The major product obtained in the given reaction is alkene.
CH3C (I) (Cl) CHD2. Presence of electron donating group in alkyne (H3C C ≡≡CH)
1. DCl (1equiv.) decreases the acidic strength of compound. Hence, the correct
CH3C ≡≡ CH → CH3C (I)(Cl)CHD2 order of acidic strength is:
2. DI

Addition in unsymmetrical alkynes takes place according to HC ≡≡ CH > H3C  C ≡ CH > CH2 == CH2
Markovnikov’s rule. 9. Sodium metal in liquid ammonia reduces alkynes with anti
Reaction proceed as follows : stereochemistry to give trans alkenes. The reduction is
Cl selectively anti since the vinyl radical formed during reduction is
DCl (1 equiv.)  DI more stable in trans configuration.
CH3 C ≡≡ CH → CH3 C == CHD 
Prop-1-yne Mechanism
Cl R R H
 R C C R′ C C
H–NH2
C C
CH3 C CHD2 –NH2
Na Na+ R′ R′
 Vinylic
I radical
Product Sodium atom donates an electron to
alkyne which after H-abstraction from
7. Attachment of electron donating group (+ R or + I) with NH3 forms vinylic radical. Transfer of
another electron gives a vinylic anion,
sp2-carbon of an unsymmetrical alkene supports Markownikov’s which is more stable in trans form. This Na Na+
addition rule through electrophilic-addition-pathway. in turn gives trans-alkene after
But, attachment of electron-withdrawing group (− R or − I) for H-abstraction from NH3.
the same will follow anti-Markownikov’s pathway (even in
absence of organic peroxide which favours free radical addition) R H R H
H–NH2
through electrophilic addition pathway. C C –NH2 – C C
The product formed by given alkenes when treated with HCl. H R′ R′
Vinylic
Cl anion
+R δ+ δ–
r
Cl–
Cl — CH — CH2 H—Cl Cl — CH— CH3 Cl— CH — CH3 δ–
Slow fast 10. δ+
HO—Cl +
— 2
CH3—CH—CH CH3—CH—CH2—Cl
Similarly, (Electrophilic
(Intermediate)
addition)
δ+ δ– –
·· H—Cl + OH
H2N — CH —
— CH2 NH2 — CH — CH3
(+R) Cl CH3—CH—CH2—Cl

Cl
NH2 — CH — CH3 OH
δ+ δ– Fast
H—Cl +
CH3O — CH —
— CH2 CH3O — CH — CH3 11. PhCH2 CH3 PhCH2 H
(+R) – C C C C
Cl H H H CH3
CH3O — CH — CH3
Fast cis form trans form
Cl CH3 CH3
δ– δ+
12.
Cl — H
O3 O
HCl r
Zn/H2O CHO
F3C — — CH2
< CH — F3C — CH2 — CH2
Slow
(– I) – Cl
s
CH3 CH3
s
Cl
F3C — CH2CH2Cl 5-keto-2-methyl hexanal
Slow
13. The reaction is
6Ag
2CH3  CCl 3  → CH3  C ≡≡ C  CH3 + 6 AgCl
∆ But-2-yne
Hydrocarbons 343

14. Ozonolysis of the given triene occur as follows : 21. Reaction proceeds through carbocation intermediate:
+
CH3 CH3 H+ H O
CH3  C== CH2 → CH3  C  CH3 →
2

 
H3C CH CH C CH CH C CH CH CH3 Ph Ph
2-phenyl propene 3 ° , resonance stabilised
H H OH
CH3

O3 CH3  C — CH3
2 CH3 CHO + 2 OHC C CHO 
Zn/H2O
Ph
H 2-phenyl-2-propanol
Since, none of the above dial is chiral, no optically active product 22. Reaction proceeds through carbocation intermediate :
is obtained. + H2 O
Ph  C ≡≡ C  CH3 + H+ → Ph  C == CH  CH3 →
15. CH3CH2C ≡≡ CH + Br — CH2CH2CH2CH3 Resonance stabilised
→ CH3CH2 — C ≡≡ C— CH2CH2CH2CH3 OH O
— HBr 3-octyne  
16. Br2 undergo anti-addition on C == C bonds as:
Br
Ph  C == CH  CH3
Unstable enol
e
(Tautomerisation)
Ph  C  CH2  CH3

23. Ammoniacal CuCl 2 forms red precipitate with terminal alkynes,


CH3 can be used to distinguish terminal alkynes from internal alkynes:
H3C H C
CH3 NH ( aq )
C==C + Br H CH3  CH2  C ≡≡ C  H + CuCl 2 →
3

CH3 C H
H
CH3  CH2  C ≡≡ C − Cu + ↓
Br red ppt.
(meso isomer) 24. In addition of HBr to an alkene, in the presence of peroxide, both
Br Alc. KOH NaNH2 the propagation steps are exothermic :
17. CH2 ==CHBr HBr + HO• → H2O + Br •
Br
H—C ≡≡C—H Propagation
18. NOCl undergo electrophilic addition on alkene as:  •

NOCl → +
N==O + Cl −  CH3 • CH == CH2 + Br → CH3  CH  CH2Br; ∆H < 0

CH3  CH  CH2Br + HBr → CH3  CH2  CH2Br
+
Cl −  + Br •; ∆H < 0
CH3  CH==CH2 + + NO → CH3  CH  CH2 → 

NO In case of addition of HCl and HI, one of the propagation step is
endothermic, reaction fail to occur.
CH3  CH  CH2
25. HOCl → HO− + Cl +
  Cl +
Cl NO
CH3  CH==CH2 + Cl + → CH3  CH  CH2
19. Cyclohexanol on treatment with concentrated H3PO4 undergo
acid catalysed dehydration giving cyclohexene. Cl
HO− 
OH → CH3  CH — CH2

conc.H3PO4 OH
i.e. reaction is initiated by Cl + (chloronium ion electrophile)
cyclohexanol cyclohexene 26. Hydrogenation with poisoned palladium brings about cis
hydrogenation of alkyne and does not affect double bonds :
20. Alkynes on treatment with alkali metals in liquid ammonia gives
trans hydrogenation product: CH3 H
CH3 CH3 H
Li/NH3 H2/Pd H3C CH3
CH3  C ≡≡ C  CH2  CH2  CH3 →
2-hexyne
H 3C H H H
H3 C HH H
H
C==C meso-compound
H CH2 CH2 CH3 (optically inactive)
trans-2-hexene
344 Hydrocarbons

27. Terminal alkynes forms silver salt with Tollen’s reagent while 34. Terminal alkynes are slightly acidic, forms salt with very strong
alkene does not react with Tollen’s reagent. base like Na, NaNH2 etc.
NH3 ( aq ) ∆ 1
CH3  C ≡≡ C  H + AgNO3 → CH3  C ≡≡ CAg ↓ H  C ≡≡ C  H + Na → H  C ≡≡ C − Na + + H2 ↑
white ppt. ethyne 2
Therefore, Tollen’s reagent can be used to distinguish a terminal 35. Baeyer’s reagent is cold, dilute, alkaline permanganate solution,
alkyne like propyne from alkene as well as from internal alkynes. used to detect presence of olefinic bonds.
28. Ease of catalytic hydrogenation depends upon the size of groups 36. Alkynes undergo Markownikoff’s addition of water in the
present at the doubly bonded carbon. Larger the size of groups, presence of H2SO4 / HgSO4 :
difficult the hydrogenation. Therefore, in the given situation,
disubstituted reacts at faster rate than tri and tetra substituted 
 OH
alkenes. Among disubstituted, the stability order is :  
HgSO4   

cis < geminal < trans. CH3  C ≡≡ C  H + H2SO4 →  CH3  C == CH2 

29. Oxymercuration-demercuration brings about Markownikoff’s Unstable enol
addition of water as : O
HgSO4 
CH3  CH2  C ≡≡ C  H + H2 SO4 → e CH3  C  CH3
(Tautomerisation) Acetone
 
 OH 
O 37. Structural formula of 1, 2-butadiene is :
   

CH3  CH2  C== CH2  ←→ CH3  CH2  C  CH3 H2C == C == CH CH3

unstable enol butanone ↑ ↑ ↑ ↑
sp2 sp sp2 sp3
30. According to the IUPAC conventions, compound can be
numbered as: 38. Unsaturated compounds which contain C == C or C ≡≡ C,
1 2 3 4 5 6
H2 C== CH  CH2  CH2  C ≡≡ C  H decolourises the purple colour of alkaline KMnO4 solution.
2 3 CH2  OH
Here, C-2 is sp and C-3 is sp -hybridised. HO−
CH2 == CH2 + KMnO4 →  + MnO2 ↓
31. Electrophilic addition on C == C is governed by stability of purple CH2  OH
carbocation: coloured

H+ 39. Addition of halogen at double bond occur in antiorientation via


CH3—CH==CH— —OH
cyclic halonium ion intermediate.
+ (i)
CH3—CH2—CH— —OH
r Br
CH3 H Br CH3 H
Br2 CH3 Br –
Br C C CH3
H H
– + H CH3 H3C H Br
Br CH3CH2CH== ==OH
CH3CH2—CH— — OH trans Meso (M and N)

(resonance stabilised carbocation) (ii) r


H H Br
32. Among alkenes-heat of hydrogenation depends on : Br2 CH3
C C
(a) The number of double bonds-greater, greater the amount of H
CH3 CH3 H
heat evolved in hydrogenation. cis CH3
Hence, 1, 3-butadiene has highest heat of hydrogenation Br
H CH3
among these. H CH3
Br
Br –
(b) Relative stability of alkenes-greater the stability, smaller the +
CH3 Br
heat evolved in hydrogenation. trans-2-butene is most stable H H
Br CH3
among three given butenes, has least heat of hydrogenation. O P

 ⌠

33. CH3  C ≡≡ C  CH2CH3 + HBr →


Enantiomers
H3C C2H5 H3C Br Here, (M + O) and (N + P) are pair of diastereomers.
C == C + C == C
+
H Br H C2H5 Br
144444444 424444444443 H3C
geometrical isomers H C
H3C C2H5 H3C 40. C==C + Br2 CH3
H C
C == C + C == C H CH3 H
trans H
Br H Br C2H5
144444444244444444 3
geometrical isomers CH3
Therefore, two structural and four configurational isomers.
Hydrocarbons 345

Statement I is correct. Statement II is also correct. Meso form of Passage 2


the product is due to anti addition of Br − on cyclic bromonium
The final ozonolysis product indicates that the alkene before
ion intermediate, hence Statement II is not correct explanation of
ozonolysis is
Statement I.
O
Br H3C CH3 O3 C
CH3 C C 2
Zn–H2O
C H3C CH3 H 3C CH3
Br– CH3
H C
P
H Also P(C6H10) has two degree of unsaturation and
Br oxymercuration demercuration hydration indicates that it is an
(meso)
alkyne. As alkyne, on hydration, gives a carbonyl compound
which on reduction with NaBH4 gives a 2º alcohol.
41. O O OH
C==C + O3 C C CH3—S—CH3
(i) NaBH4
—C C— + H2O —C—CH2— + — C—CH2—
(ii) H
O O
ozonide H
2° alcohol
C ==O + O == C + (CH3 )2 SO The secondary alcohol that can give above shown alkene on acid
Both Statement I and Statement II are correct and Statement II is catalysed dehydration is
correct explanation of Statement I. CH3 OH
42. CH3  CH2  CH== CH2 + Br • → H
+ + Me-shift

CH3 — C — CH — CH3 –H2O CH3 — C — CH— CH3
CH3  CH2  CH  CH2 Br
CH3
a secondary radical
Q 2° carbocation
Therefore, Statement I is correct but Statement II is incorrect. CH3 CH3
+
43. CH3  CH2  CH == CH2 + Br2 → +
CH3 —C—CH—CH
–H CH3 — C C — CH3
3
CH3  CH2  CH  CH2 Br
CH3 CH3

Br 46. Explained in the beginning.
has one chiral carbon
CH3 CH3 O
Both Statement I and Statement II are correct and Statement II is
HgSO4 (i)NaBH4
correct explanation of Statement I. CH3—C— C—CH3 Q
47. CH3—C—C CH H2SO4 (ii) H +

Passage 1 CH3 CH3


(P)
The reaction condition indicates that starting compound is phenyl
acetylene.
Passage 3
44. 48. PLAN This problem can be solved by using the concept of iodoform
CH 
 CH2
C≡≡C—H OH test and functional isomerism.
Pd/BaSO4 (i) B2H6
H2 (ii) H2O2, NaOH, H2O
Iodoform test The compound containing — COCH3 or
(2-phenyl ethanol)
CH(OH) group will undergo iodoform test
(X) Br OH
NaNH2 Cs
Hydroboration oxidation brings about anti-Markonikoff’s H
Iequivalent NaNH2(1eq)
hydration of alkene. N
O
45. s
C ≡≡ CH H2Pd/C
OH O
H2O, HgSO4 CH3 H3O s

H2SO4 mild
OH Me
CH3 CrO3 O
CH3
Y
H+/D CH3
C2H5 MgBr, H2O CH3 • Thus, X and Y are functional isomers of each other and Y gives
OH iodoform test due to the presence of CH 3CO group as Indicated.
Hence, correct choice is (c).
346 Hydrocarbons

49. PLAN This problem can be solved by using the concept of nucleophilic 54. 2-butyne :
substitution reaction, oxidation reaction and reduction reaction + − CH I
including strength of nucleophile and regioselectivity. H  C ≡≡ C  H + Na → Na C ≡≡ C − Na + →
3
excess excess
Reaction of Scheme 1 can be completed as
CH3  C ≡≡ C  CH3
1. NaNH2
H 55. Terminal alkyne (ethyne) is most acidic among these.
II
HO M O 56. Sulphuric acid undergo addition to alkene.
I

Among two nacked nucleophilic group I and II, II is more


nucleophilica and then will react selectively as follows 57.
, , , ,
1. CH3CH2—I
CH3 O
 O3 
O O 58. (a) CH2 == C — CH == CH2 → 2HCHO + CH3 C CHO
CH3I
(A ) Zn -H2 O
H2 Lindlar’s
catalyst (produces
cis alkene) H3C H H3C H
CH3O
H (b) C == C C == C
H
H2C CH2  H2C CH2
natural rubber
CH3O
59. From oxidation products, structure of starting compound can be
Hence, using the concept of regioselectivity we come on the deduced as :
conclusion that final product is correctly represented by C3 H8  COOH + HOOC  COOH
structure (a). butanoic acid oxalic acid O
50. 3, 4-dibromo-1-butene : 
+ HOOC (CH2 )8  CH2 O  C  CH3
+ Br2 10-acetoxy decanoic acid
Br
O 3 / H 2O 2
Br
CH3 CH2 CH2 CH==CHCH==CH(CH2 )8  CH2 OCOCH3
HgSO
51. CH ≡≡ CH + H2SO4 →
4
CH3CHO Therefore, Bombykol is :

CH2  COOK CH3CH2CH2  CH== CH  CH== CH (CH2 )8 CH2OH


Electrolysis
52.  → CH2 == CH2 + 2CO2 Bombykol
CH2  COOK ethene
H2
sp 3 CH3 −
( CH2 )14  CH2OH
53. CH3  CH== CH  CH3 (A )
2- butene

C2H5
H H alkaline KMnO4 H OH
(i) NaNH Pd/BaSO
60. CH3CH2  C ≡≡ C  H →
2
CH3CH2  C ≡≡ C  CH2CH3 →
4
C== C H OH
(ii) CH3 CH2 Br X H2 C2H5 C2H5 C2H5
Y (meso diol)
‘Z’
O
Br
Br2 NaNH2 HgSO4
61. (a) —C ≡≡CH B —C—CH3
CCl4 H2SO4
A Br B C
O
CH3 O C
H2NHNCONH2 NaOD
—C==N—NH—C—NH2 D CD3
D2O
E D
D
Hydrocarbons 347

* * +
(b) Ba CO3 + H2 SO4 → CO2 + BaSO4 OH OH2
X  + 
H
O 66. (i) C6H5  CH  C ≡≡ CH → C6H5  CH  C ≡≡ CH

(i) Mg / ether – H2 O +
H+
CH2 == CH  Br → CH2 ==CH  C  OH → C6H5  CH  C ≡≡ CH →
(ii) X * H2 O
(iii) H3O+ Y
LiAlH4
→ CH2 == CH  CH2  OH
*
C6H5  CH == CH  CHO e
(Tautomerisation)
C6H5  CH == C == CH

OH
Z unstable enol
62. B + NaOH + I2 → C6H5  COONa (Iodoform reaction)
+
O H
(ii)
 +
∴ B is C6H5  C  CH3 OH O CH3

B + KOH Hydrocarbon (C ) “Wolff-Kishner


N2H4 → H
C8 H10
Reduction” H+
∴C is C6H5  CH2  CH3
O CH3
Hence, A can be one of the following :
H5 C6 C6 H5 H5 C6 CH3 67. Oxidation product indicates that alkene is symmetrical :
C==C or C==C
H+
H3 C CH3 H3 C C6 H5 + KMnO4 2CH3CH2COOH
I II 3-hexene
‘A’
I on catalytic hydrogenation, would give meso compound while
II on catalytic hydrogenation, would produce racemic mixtures. Ph H
68. (i) C==C + KNH2 Ph—C C—Ph
CH3 Ph Br A
63. Lindlar’s
CH3 catalyst H R R
H
(ii) + HClO4 2R—COOH
cis hydrogenation
B
O OH O
Br
Zn-H2O LiAlH4
64. (a) + O3
(iii) + CHBr3 + t-BuOK

conc. H2SO4 C
∆ CH3 CH3
Cl (CH3)3COK
CH3 CH2
OH 69. H C H3C
D D D 3
H2O2
CH3
(b) CH3 C==C + BHR2 – CH3 C—C ‘X’ CH3 ‘Y’ CH3
HO
CH3 H
H 3C H3C
CH3 D CH3 CH3
CH3
CH3 H2/Pt CH3
R = (CHl 3 )2 CHCH2 + H3C H3C
Br ‘Z’ CH3 CH3

65. C6H5  CH == CH2 + Br2 → C6H5  CH  CH2 CH2CH3 C2H5
  
HBr
Br 70. CH3CH == C  CH2 CH3 → CH3  CH  CH  C2H5
A 3-ethyl-2-pentene peroxide 
NaNH2 Br
→ C6H5  C ≡≡ C  CH3 [A]
CH3 I B
348 Hydrocarbons

CH2 CH3 75. CH2 == CH2 + KMnO4 + H2O → CH2  CH2 + MnO2
Br 2 /H2O   
→ CH3  CH  C — CH2  CH3 OH OH
  Ethylene glycol
Br OH
[B] 76. Baeyer’s reagent (cold, dilute, alkaline permanganate) can be used
to distinguish between alkanes and alkenes. Alkenes decolourises
C2H5 purple colour of Baeyer’s reagent while alkanes do not.
Hg(OAc) 2 /H2 O 
→ CH3  CH2  C — CH2 CH3 77. The alkenes are :
NaBH4
 CH3  CH == O + O == CH  CH(CH3 )2
OH
[C ] CH3 CH3
H3C CH3  
71. C==C + Br2 CH3  CH == CH  CH — CH3 ; CH3CH2CH ==O + O == C  CH3
H H I (B )
cis
CH3
Br H CH3 
H CH3  CH2  CH== C — CH3
Br
C II (C )
C
CH3 C
H 3C C + Since, both alkenes I and II are obtained by β-elimination of
H Br same halides, the halides must be :
Br CH3
H CH3
Pair of enantiomers

O CH2 CH3  CH2  CH  CH — CH3
  
72. CH3  C  CHO + O== C  H ← CH3 C  CH == CH2 Cl
2-ketopropanal  E (A )
H
Formaldehyde Heat ( ∆ )
3 CH I
78. H  C ≡≡ C  H + Na → → CH3  C ≡≡ CH
Gas A O
73. CH3  CH == CH  CH3 → 2CH3  CHO
Zn − H2 O H2 SO4 
∴ Gas A is ozone (O3 ) → CH3  C  CH3
HgSO4 Acetone
Also; Gas A + KOH (dry) → B + Gas C
deep red coloured 79. 1-butyne (terminal) can be distinguished from 2-butyne
∴ 4O3 + 4KOH → 4KO3 + 2H2O + O2 (internal) by either Tollen’s test or through Fehling’s test.
B C AgNO3
NH3(aq)
CH3—CH2—C ≡≡ CAg↓
74. (i) A must be a symmetrical alkene : White ppt.
Br CH3—CH2—C ≡≡C—H
 CuCl2
CH3—CH2—C ≡≡ CCu↓
CH3 CH==CHCH3 + HBr → CH3 CH2  CH  CH3 NH3(aq)
Red ppt.

CH3 CH3 Peroxide


  80. (i) CH3 C == CH2 + HBr → CH3  CH  CH2Br
(ii) CH3  C == CH2 + H2SO4 + H2O → CH3  C — CH3  
 CH3 CH3
OH 2-methyl propene Isobutyl bromide
Achiral
In the absence of peroxide, HBr would be added giving
CH3 tertiary butyl bromide.
 Pt
(iii) CH3  CH2  C — CH == CH 2 + H2 → (ii) Tertiary alkynes are slightly acidic, forms silver salt with
 ∆ ammoniacal solution of silver nitrate :
H
NH ( aq )
has one chiral carbon R  C ≡≡ C  H + AgNO3 →
3
R  C ≡≡ CAg ↓
White ppt.
CH3
 81. (i) Free radical chlorination of alkane require energy which is
CH3CH2  C — CH 2CH 3
supplied either in the form of heat or radiation.

H
Achiral
Hydrocarbons 349

(ii) Addition of HBr proceeds through carbocation intermediates.


+ −
CH3  CH == CH2 + H+ → CH3  CH  CH3 →Br
CH3  CH  CH3
2 ° carbocation 
Br
Isopropyl bromide

∆ CH  CH  CH + MnO
82. CH3CH == CH2 + KMnO4 (aq) → 3 2 2
 
OH OH

83. Ozonolysis products are the key of identification :


CH3 CH3
 O3 
CH3  C== O + O == CH  CH3 ← CH3  C== CH CH3
Propanone Ethanal Zn -H2 O A

Other products are:


CH3 CH3 Br
 dil. KMnO4 Br 2  
CH3  C — CH  CH3 ← A → CH3  C — CH — CH3
  Cold, OH− 
OH OH Br
O
1 Ag
84. (i) CH2 == CH2 + O2 → H2C  CH2
2 heat Ethylene oxide
(oxirane)
OH
HgSO4 dil. OH
 +
(ii) CH ≡≡ CH + H2SO4 → CH3  CHO → CH3  CH CH2  CHO
(Aldol)

2 NaNH
85. CH2 == CH2 + Br2 → CH2  CH2 → H  C ≡≡ C  H
 
Br Br
86. Acetylene can be distinguished from methane using Tollen’s reagent :
NH (aq)
C2H2 + AgNO3 →
3
H  C ≡≡ CAg ↓
White ppt.

No such reaction occur with methane.


23
Alkyl Halides
1. The mechanism of SN 1reaction is given as : 4. The major product Y in the following reaction is
(2019 Main, 10 April II)
(2020 Main, 3 Sep I)

È Cl
R  X → R ⊕ X È → R ⊕||X È →
Y EtONa HBr
R Y + X È Heat
X Y
Ion pair Solvent
Separated ion
pair
Br
A student writes general characteristics based on the given (a) Br (b)
mechanism as :
(A) The reaction is favoured by weak nucleophiles.
(B) R ⊕ would be easily formed if the substituents are HO
(c) (d)
bulky.
(C) The reaction is accompanied by recemisation. Br
(D) The reaction is favoured by non-polar solvents.
6. The major product of the following reaction is
Which observations are correct? CH3
(a) (A) and (B) (b) (A) and (C)

(c) (A), (B) and (C) (d) (B) and (D) CH OH
CH3  C  CHCH3  3
→
2. 1-methyl ethylene oxide when treated with an excess of HBr  
produces : (2020 Main, 7 Jan I) H Br (2019 Main, 10 April I)

Br Br CH3 CH3
(a) Br (b)  
Br (a) CH3  C == CHCH3 (b) CH3  C  CH == CH 2
CH3 
H
Br
CH3
(c) Br (d) CH3 CH3
CH3  
(c) CH3  C  CH 2 CH3 (d) CH3  C  CHCH3
3. Heating of 2-chloro-1-phenyl butane with EtOK/EtOH gives   
OCH3 H OCH3
X as the major product. Reaction of X with
Hg(OAc)2 / H 2O followed by NaBH 4 gives Y as the major 7. Which of the following potential energy (PE) diagrams
product. Y is (2019 Main, 12 April II) represents the SN 1reaction? (2019 Main, 9 April II)
OH
Ph
(a) (b) Ph (a) PE (b) PE
OH
OH
Progress of reaction Progress of reaction
(c) Ph (d) Ph

4. Which one of the following is likely to give a precipitate with (c) (d) PE
PE
AgNO3 solution? (2019 Main, 12 April II)
(a) CH2 == CH  Cl (b) CCl 4
(c) CHCl 3 (d) (CH3 )3 CCl Progress of reaction Progress of reaction
Alkyl Halides 351

8. The major product of the following reaction is Cl


CH2CH3
CH3O CH3O
NaOEt (a) (b)
H 3C C Cl ∆ Cl
Cl
COOCH2CH3 CH3O
(c) (d) CH3O
(2019 Main, 12 Jan II)
(a) CH3 CH2 C == CH2 Cl

CO2CH2CH3 12. Which hydrogen in compound (E) is easily replaceable
(b) CO2CH2CH3 during bromination reaction in presence of light?
 CH3 CH 2 CH == CH 2 (2019 Main, 10 Jan I)
CH3 C == CHCH3
δ γ β α
CH2CH3 (E)
(a) β-hydrogen (b) δ-hydrogen
(c) H3C C OCH2CH3 (c) γ-hydrogen (d) α-hydrogen
13. The major product of the following reaction is
COOCH2CH3 Br
KOH alc. (excess)
OCH2CH3 ∆
Ph
Br
(d) H3C H2C C CO2CH2CH3 (2019 Main, 10 Jan I)

CH3
(a) Ph (b) Ph
9. The major product in the following conversion is
HBr (excess)
CH3O CH CH CH3 Heat ? (c) Ph (d) Ph

(2019 Main, 12 Jan II) 14. The major product of the following reaction is
(a) CH3O— —CH—CH2—CH3 CH3O
CH2Cl
Br (i) AlCl3 (anhyd.)
(ii) H2O
(b) HO— —CH2—CH—CH3
(2019 Main, 10 Jan I)
Br

(c) CH3O— —CH2—CH—CH3 CH3O CH3O


(a) (b)
Br
CH3
(d) HO— —CH—CH2—CH3 OCH3
Br CH3 CH3O
(c) (d)
10. The major product of the following reaction is
(i) KOH alc.
CH3 CH2 CH  CH2  →
  (ii) NaNH2 15. The major product of the following reaction is
Br Br in liq. NH3
Br
(2019 Main, 12 Jan II)
NaOMe
(a) CH3CH2 CH  CH2 (b) CH3CH == CHCH2NH2 MeOH
  (2018 Main)
NH2 NH2
OMe
(c) CH3CH == C == CH2 (d) CH3CH2C ≡≡ CH
(a) (b)
11. The major product of the following reaction is
(2019 Main, 12 Jan I)
CH3O OMe
(i) Cl2/CCl4
(c) (d)
(ii) AlCl3 (anhyd.)
352 Alkyl Halides

16. The increasing order of reactivity of the following halides for 22. The synthesis of alkyl fluorides is best accomplished by
the S N 1 reaction is (2017 Main) (2015 Main)
I. CH3CH(Cl)CH2CH3 (a) free radical fluorination
(b) Sandmeyer’s reaction
II. CH3CH2CH2Cl
(c) Finkelstein reaction
III. p-H3CO  C6H4  CH2Cl (d) Swarts reaction
(a) (III) < (II) < (I) (b) (II) < (I) < (III)
23. In SN 2 reactions, the correct order of reactivity for the
(c) (I) < (III) < (II) (d) (II) < (III) < (I) following compounds CH3 Cl,CH3 CH2 Cl, (CH3 )2 CHCl and
17. Which of the following, upon treatment with tert-BuONa (CH3 )3 CCl is (2014 Main)
followed by addition of bromine water, fails to decolourise (a) CH3Cl > (CH3 )2 CHCl > CH3CH2Cl > (CH3 )3 CCl
the colour of bromine? (2017 Main)
(b) CH3Cl > CH3CH2 Cl > (CH3 )2 CHCl > (CH3 )3 CCl
O C6H5
(c) CH3CH2Cl > CH3Cl > (CH3 )2 CHCl > (CH3 )3 CCl
(a) (b)
(d) (CH3 )2 CHCl > CH3CH2Cl > CH3Cl > (CH3 )3CCl
Br Br
O 24. KI in acetone, undergoes S N 2 reaction with each P, Q, R and
O S. The rates of the reaction vary as (2013 Adv.)
(c) (d)
Br Br
O
Cl
18. 3-methyl-pent-2-ene on reaction with HBr in presence of H3C—Cl —Cl
peroxide forms an addition product. The number of possible Cl
stereoisomers for the product is (2017 Main) P Q R S
(a) six (b) zero (c) two (d) four
(a) P > Q > R > S (b) S > P > R > Q
19. The major product obtained in the following reaction is (c) P > R > Q > S (d) R > P > S > Q
Br
H tBuOK
25. The major product of the following reaction is (2008, 3M)
C6 H 5 ∆ H 3C Br
C6H5
(2017 Main)
(a) (± ) C6 H5CH(Ot Bu)CH2C6 H5
F
PhSNa
(b) C6 H5CH == CHC6 H5 dimethyl formamide
(c) (+ ) C6H5CH(Ot Bu)CH2C6 H5
(d) (− )C6 H5CH(Ot Bu)CH2C6 H5 NO2
20. 2-chloro-2-methylpentane on reaction with sodium H 3C SPh H 3C SPh
methoxide in methanol yields (2016 Main)
F F
CH3
 (a) (b)
I. C2 H5 CH2 C  OCH3 II. C2 H5 CH2 C == CH2
 
CH3 CH3 NO2 NO2

III. C2 H5 CH == C  CH3 H 3C Br H 3C SPh



CH3 SPh SPh
(a) Both I and III (b) Only III (c) (d)
(c) Both I and II (d) All of these
21. The product of the reaction given below is (2015 Adv.)
NO2 NO2
(i) NBS/hν
X
(ii) H2O/K2CO3 26. The following compound on hydrolysis in aqueous acetone
will give (2005, 1M)
CH3 H CH3
OH O CO2H
(a) (b) (c) (d) MeO— —NO2

H Cl CH3
Alkyl Halides 353

CH3 H CH3 33. A solution of (+)-2-chloro-2-phenylethane in toluene


racemises slowly in the presence of small amount of SbCl5,
(K) MeO— —NO2 due to the formation of (1999)
(a) carbanion (b) carbene
H OH CH3 (c) free-radical (d) carbocation
CH3 H CH3 34. Which of the following is an organometallic compound?
(L) MeO— —NO2 (a) Lithium methoxide (1997)
(b) Lithium acetate
OH H CH3 (c) Lithium dimethylamide
(d) Methyl lithium
CH3 H CH3
35. (CH3 )3 CMgCl on treatment with D2 O produces (1997, 1M)
(M) MeO— —NO2 (a) (CH3 )3 CD (b) (CH3 )3 COD
(c) (CD)3 CD (d) (CD)3 COD
H CH3 OH
36. 1-chlorobutane on reaction with alcoholic potash gives
It mainly gives
(a) 1-butene (b) 1-butanol (1991, 1M)
(a) K and L (b) only K (c) L and M (d) only M
(c) 2-butene (d) 2-butanol
27. The product of following reaction is
OH
37. n-propyl bromide on treatment with ethanolic potassium
hydroxide produces (1987, 1M)
C2H5O– (a) propane (b) propene
+ C2H5I (c) propyne (d) propanol
anhy. C2H5OH
(2003, 1M)
(a) C6H5OC2H5 (b) C2H5OC2H5 38. The reaction condition leading to the best yield of C2 H5 Cl are
(c) C6H5OC6H5 (d) C6H5I (1986, 1M)
UV light
28. Identify the set of reagents/reaction conditions X and Y in the (a) C2H6 (excess) + Cl 2 →
following set of transformations– (2002, 3M) dark
(b) C2H6 + Cl 2 (excess) →
X room temp.
CH3  CH2  CH2 Br → Product →
Y CH  CH  CH
3 3
 (c) C2H6 + Cl 2 (excess) →
UV light
Br
UV light
(a) X = dilute aqueous NaOH, 20°C,Y = HBr/acetic acid, 20°C (d) C2H6 + Cl 2 →
(b) X = concentrated alcoholic NaOH, 80°C,
Y = HBr/acetic acid, 20°C Objective Questions II
(c) X = dilute aqueous NaOH, 20°C, Y = Br2 / CHCl 3 , 0°C (One or more than one correct option)
(d) X = concentrated aqueous NaOH, 80°C,
Y = Br2/CHCl 3 , 0°C
39. For the following compounds, the correct statement(s) with
respect to nucleophilic substitution reaction is(are)
29. The compound that will react most readily with NaOH to (2017 Adv.)
form methanol is (2001, 1M)
CH3 CH3
(a) (CH3 )4 N + I – (b) CH3OCH3
(c) (CH3 )3 S+ I – (d) (CH3 )3 Cl H3C C Br
Br Br Br
30. An SN 2 reaction at an asymmetric carbon of a compound CH3
always gives (2001) I II III IV
(a) an enantiomer of the substrate (a) Compound IV undergoes inversion of configuration
(b) a product with opposite optical rotation (b) The order of reactivity for I, III and IV is : IV > I > III
(c) a mixture of diastereomers (c) I and III follow S N 1 mechanism
(d) a single stereoisomer (d) I and II follow S N 1 mechanism
31. The order of reactivities of the following alkyl halides for a 40. Compound(s) that on hydrogenation produce(s) optically
SN 2 reaction is– (2000, 1M)
inactive compound (s) is/are (2015 Adv.)
(a) RF > RCl > RBr > RI (b) RF > RBr > RCl > RI
H Br H Br
(c) RCl > RBr > RF > RI (d) RI > RBr > RCl > RF
(a) (b) H2C CH3
32. Which of the following has the highest nucleophilicity? H 3C CH3
(2000)
H Br Br H
(a) F − (b) OH −
(c) (d) H2C
(c) CH3− (d) NH2− H2C CH3
CH3
CH3
354 Alkyl Halides

41. In the following reaction, the major product is (2015 Adv.) 45. Match the chemical conversion in Column I with the
CH3 appropriate reagents in Column II and select the correct
CH2 1 equivalent HBr
answer using the code given below the lists. (2013 Adv.)
H2C
CH3 CH3 Column I Column II
CH3
(a)
H2C
(b)
H3C 1. (i) Hg(OAc)2 ;
Br Cl
P. (ii) NaBH4
Br
CH3 CH3
(c) (d) ONa OEt
H2C Br H3C Br Q. 2. NaOEt

42. The compounds used as refrigerant are (1990, 1M)


(a) NH3 (b) CCl 4 (c) CF4 (d) CF2Cl 2 OH
(e) CH2F2 R. 3. Et-Br

Assertion and Reason


4. (i) BH3 ;
43. An Assertion and a Reason are given below. Choose the S.
correct answer from the following options. (ii)H2 O2 / NaOH
OH
Assertion (A) Vinyl halides do not undergo nucleophilic
substitution easily. Codes
Reason (R) Even though the intermediate carbocation is P Q R S
stabilised by loosely held π-electrons, the cleavage is (a) 2 3 1 4
difficult because of strong bonding. (2019 Main, 12 April II) (b) 3 2 1 4
(a) Both (A) and (R) are wrong statements. (c) 2 3 4 1
(b) Both (A) and (R) are correct statements and (R) is correct (d) 3 2 4 1
explanation of (A). 46. Match the following : (2006, 3M)
(c) Both (A) and (R) are correct statements but (R) is not the
correct explanation of (A). Column I Column II
(d) (A) is a correct statement but (R) is a wrong statement. A. CH3 CHBr CD3 on treatment p. E1 reaction
with alc. KOH gives
Match the Columns CH2 == CH CD3 as a major
44. List-I contains reactions and List-II contains major products. product.
B. Ph—CHBr—CH3 reacts faster q. E2 reaction
List-I List-II than Ph—CHBr—CD3
P. + 1. C. Ph—CH2 CH2Br on treatment r. E1CB
ONa Br OH with C2H5 OD /C2H5 O− gives reaction
Ph CD ==CH2 as the major
Q. + HBr
2. product.
OMe Br
D. PhCH2CH2Br and PhCD2CH2Br s. First order
R. 3. react with same rate. reaction
+ NaOMe
Br OMe
Fill in the Blanks
S. + MeBr 4. 47. Vinyl chloride on reaction with the dimethyl copper
ONa gives ………. (1997)

5. O 48. The starting material for the manufacture of polyvinyl


chloride is obtained by reacting HCl with …… (1983, 1M)
49. The halogen which is most reactive in the halogenation of
Match each reaction in List-I with one or more products in alkanes under sunlight is … (1981 , 1M)
List-II and choose the correct option. (2018 Adv.)
(a) P → 1, 5; Q → 2; R → 3; S → 4 True/False
(b) P → 1, 4; Q → 2; R → 4; S → 3 50. Photobromination of 2-methylpropane gives a mixture of
(c) P → 1, 4; Q → 1,2; R → 3,4; S → 4 1-bromo-2-methyl propane and 2-bromo-2-methyl propane
(d) P → 4, 5; Q → 4; R → 4; S → 3,4 in the ratio 9 : 1. (1993, 1M)
Alkyl Halides 355

51. During S Ν 1 reactions, the leaving group leaves the molecule 62. Predict the structure of the product in the following reaction
before the incoming group is attached to the molecule. Br
(1990, 2M)
H Ph NaI
52. The reaction of vinyl chloride with hydrogen iodide to give (1996, 1M)
MeO H acetone
1-chloro-1-iodoethane is an example of anti-Markownikoff’s
rule. (1989, 2M)
Ph
53. Iodide is better nucleophile than bromide. (1985)
63. Optically active 2-iodobutane on treatment with NaI in
54. Carbon tetrachloride is inflammable. (1985, 1/ 2M) acetone gives a product which does not show optical activity.
55. Carbon tetrachloride burns in air when lighted to give Explain briefly. (1995, 2M)
phosgene. (1983, 1M) 64. Draw the stereochemical structure of product in the
following reaction
Integer Answer Type Questions
CH3
56. In the following monobromination reaction, the number of NaOH
possible chiral product(s) is (are)... (2014) Br  H → (1994)
SN 2
CH 2 CH 2 CH 3
C2 H5
Br 2 (1.0 mole)
H Br
300°C 65. Aryl halides are less reactive than alkyl halides towards
CH3 nucleophilic reagents. Give reason. (1994, 2M)
(1.0 mole) (Enantiomerically pure) 66. Identify the major product in the following reaction.
57. The total number of alkenes possible by dehydrobromination H
of 3-bromo-3-cyclopentylhexane using alcoholic KOH is Alc. KOH HBr
(2011) C6 H5  CH2  CH3 → ? → ? (1993, 1M)

Subjective Questions Br
58. Identify X, Y and Z in the following synthetic scheme and
write their structures 67. Write the structures of the major organic product expected
from each of the following reactions :
(i) NaNH2 CH3
CH3 CH2 C ≡≡ CH → X
(ii) CH3 CH2 Br
Alc. KOH
(i) H3 C  CH2 CH3 →
H2 / Pd - BaSO4 alkaline
X → Y → Z
KMnO4 (2002, 5M) Cl
aq. alkali
59. What would be the major product in each of the following (ii) CH3 CH2 CHCl 2 → (1992, 2M)
boil
reactions?
CH3 68. Arrange the following in order of their
 (i) Increasing basicity
C2 H5 OH
(i) CH3  C  CH2 Br → (2000, 2M) H2O, OH− , CH3OH, CH3O–
 ∆
(ii) Increasing reactivity in nucleophilic substitution reactions
CH3
CH3 F, CH3 I, CH3 Br, CH3 Cl (1992, 2M)
60. Write the structures of the products :
69. Chloroform is stored in dark coloured bottles. Explain in not
Alc. KOH more than two sentences.
C6 H5 CH2 CHClC6 H5 → (1998, 2M)
(1980 , 1M)

70. Show by chemical equations only, how would you prepare


61. Which of the following is the correct method for synthesising the following from the indicated starting materials? Specify
methyl-t-butyl ether and why? the reagents in each step of the synthesis. (1979, 2M)
(CH3 )3 CBr + NaOMe → (i) Chloroform from carbon disulphide.
or CH3 Br + NaO -t -Bu → (1997, 2M) (ii) Hexachloroethane (C2Cl6) from calcium carbide.
Answers
1. (c) 2. (a) 3. (c) 4. (d) 33. (d) 34. (d) 35. (a) 36. (a)
5. (b) 6. (c) 7. (b) 8. (b) 37. (b) 38. (a) 39. (a, d) 40. (b,d)
9. (d) 10. (d) 11. (c) 12. (c) 41. (d) 42. (a,d) 43. (c) 44. (b)
13. (d) 14. (d) 15. (b) 16. (b) 45. (a) 46. A → q, B → q, C → r, D → p, s
17. (a) 18. (d) 19. (b) 20. (d) 47. (propene) 48. (ethyne) 49. (chlorine) 50. (False)
21. (a) 22. (d) 23. (b) 24. (b)
51. (True) 52. (True) 53. (True) 54. (False)
25. (a) 26. (a) 27. (a) 28. (b)
55. (False) 56. (5) 57. (5)
29. (a) 30. (d) 31. (d) 32. (c)

Hints & Solutions


⊕ ⊕ YÈ Mechanism
1. R  X → R X È → R|| X È → R Y + X È
Cl OAc
It indicates R ⊕ (carbocation) formation takes place and R ⊕ got s
OEt
stabilised by electronic factors and polar solvent molecules + Hg OAc
Ph dehydro-
because solvent separation of R ⊕ is possible. halogenation
Ph
H
So, statement (D) is not correct.
Oxymercuration
Here, X È (leaving group) of R  X is substituted by Y È,
(nucleophile) via the formation of carbocation (R ⊕ ) OAc
intermediate. So, it is an SN 1reaction.
Hg+
SN 1 mechanism supports weaker nucleophiles. So, statement Ph
H
(A) is correct. C H
SN 1 reaction is accompanied by inversion and retention in
configuration, i.e. racemisation provided ‘R’ of R  X is H H C2H5
chiral. So, statement (C) is correct.
H 2O
Attachment of bulkier substituents or 3°-nature of the carbon
atom of C X bond of R  X will favour formation of R ⊕ Ph CH CH Hg OAc
(carbocation). So, statement (D) is also correct.
OH C2 H5
2. The given reaction is completed as follows :
H+ H
CH3—CH—CH2 CH3—CH—CH2
(From HBr) +
s
O O OAc
1-methylethyleneoxide
H Ph CH CH HgOAc
Br Br
HBr Br–
OH C2H5
CH3—CH—CH2Br CH3—CH—CH2
Demercuration NaBH4
(Product)
OH
Ph CH CH2 + AcOs
3. Heating of 2-chloro-1-phenylbutane with EtOK/EtOH gives
1-phenyl but-1-ene(X). OH C2H5 (Y)

Reaction of X with Hg(OAc)2 / H2O followed by NaBH4 gives


1-phenyl butan-1-ol (Y).
4. (CH3 )3CCl gives a precipitate with AgNO3 solution because it
forms stable carbocation. (i.e. tertiary) that readily combines with
Reaction involved is as follows:
AgNO3 to give precipitates of AgCl.
Cl –+
EtOK CH3
EtOH  +
Ph Ph CH3 C  Cl + AgNO3 → (CH3 )3 C + AgCl
OH  (White ppts.)
1. Hg (OAc)2,H2O CH3
2. NaBH4
Ph CH2 == CH Cl forms unstable carbocation.
(Y )
Hence, it does not readily react with AgNO3.
Alkyl Halides 357

5. The given reaction takes place as follows : nucleophile reacts rapidly with the carbocation formed in the
first step.
Cl
EtONa/∆ In the above graph, the alkyl halide is the only species that
H participates in the rate limiting step. Here, the rate of reaction
β-H for β-elimination
Saytzeff elimination –HCl depends on the concentration of the alkyl halide and does not
CH2 depend on the concentration of nucleophile.
H β-H for
Hoffmann elimination
8. Presence of strong base (EtO− ) and heat indicates elimination.
Markownikov’s Thus, the compound undergo dehydrohalogenation and alkene
centre for Brs
+ is produced. As per the position of Cl in substrate, following 2
3-methyl but-2-ene 3-methyl-but-1-ene
alkenes are formed as product:
major (X) minor (i) CH3CH2 C == CH2
(Saytzeff product) (Hoffmann product)

Electrophilic addition COOCH2CH3
HBr
(Markownikov’s rule)
Br (ii) CH3 CH == C CH3

Br +
COOCH2CH3
2-bromo-3-methyl 2-bromo-2-methyl In accordance with Saytzeff rule
butane (minor) butane (major) (Y)
CH3 CH == C CH3 will be the major product
6. In the given question, the substrate is a 2º-halide (bromide) and 
the medium, CH3OH (as well as a poor nucleophile) is protic in COOCH2CH3
nature, So, the reaction will follow mainly S N1 pathways via the
formation of a carbocation intermediate (I).
9. Key Idea The excess of HBr and high temperature in given
Me Me r Me Me reaction serves for dual purpose:
–Br r CH3OH
rds
C CH r (i) Hydrolysis of ether via SN 2 mechanism, i.e. Zeisel’s
Me –H
H Me (2°) method.
Fast
H Br H (I) (ii) Markownikoff addition at double bond of the branch.
Me
CH—CH—Me The road map of complete reaction is as follows:
Me
OCH3 CH3 O CH CH CH3
(Minor)

The intermediate, I can be rearranged into the more stable form I′ H+ HBr excess/Heat
(3°) by α-hydride shift. I ′ will give the major product.
Me s Me +
r α-H shift r
CH3 O CH CH2 CH3
C CH—Me C—CH2—Me
Me Me
I′ CH3OH –Hr Br–
H (3° carbocation)
I Me
(2° carbocation)
C—CH2—Me CH3 O CH CH2 CH3
Me
OCH3 Br
(Major)
Br– H+
7. The potential energy (PE) diagram for S N1 reaction is H
Carbocation d– +
BF d BB d–
Br C O CH CH2 CH3
intermediate
BF
+
H H Hd Br
Transition state
PE R++X– SN2 Reaction

+Nu (Zeisel’s Method)

R X
R  Nu CH3Br + HO CH CH2 CH3
Progress of reaction
Br
S N1 reaction has two steps. In the first step, the carbon halogen Product
bond breaks heterolytically, with the halogen retaining the Here, BB = Bond breakage
previously shared pair of electron. In the second step, the BF = Bond formation
358 Alkyl Halides

10. Key Idea Both alc. KOH and NaNH2 in liquid NH3 are 14. In the given reaction, AlCl3 act as Lewis acid and helps in
dehydrohalogenating reagents. On comparative terms generation of carbocation. The resulting carbocation (1°)
NaNH2/liquid NH3 is stronger in action. rearranges itself to stable carbocation (3°).
The reaction proceeds as : MeO Cl
AlCl3(anhy.)
Alc. KOH
CH3 CH2 CH CH2 CH3 CH2 CH CH2
r s
MeO Cl AlCl3
Br Br Br MeO s
–AlCl4
NaNH2/liquid NH3 H r

CH3 CH2 C CH CH3CH2C CNa+ (1°)
H+
Thus, option (d) is the correct answer. MeO
a-Hs shift r
11. The given reactant in presence of Cl2/CCl4, given vicinal (H2O)
dihalide. Chlorine adds up to alkene via electrophilic addition
reaction involving cyclic chlorinium ion formation. 3° carbocation, more stable
Cl Protic medium, H2O supports the
rearrangement of carbcation
H3CO H3CO
Cl2/CCl4 Intramolecular ArSE2 pathway
r
Me O MeO
Cl –Hr
The vicinal dihalide in presence of anhyd. AlCl3 results in the
formation of carbocation that rearranges itself to form a cyclic H
compound.
Cl 15. Complete reaction can be represented as
H3CO Br
+ AlCl3 –
(Anhyd.) Na+OMe
Cl + NaBr + MeOH
Cl MeOH
H3CO H
+ –
r + AlCl
4 Thus, the given reaction is dehydrohalogenation which is a
β-elimination proceeding through E2 mechanism.
Mechanism The reaction proceeds through the formation of
CH3O CH3O following transition state with simultaneous removal of Br and
H atoms.
–H+
Cl Cl Na+ Na+Br–
H
BF (Product)
12. The compound (E ) has two allyl-hydrogen atoms (γ ). When E BB Br
reacts with Br2 / hν, it readily undergoes allylic free radical Here, BB = Bond breakage
substitution and forms 3, 3-dibromobut-1-ene BF
γ β
BF = Bond formation
δ α Br2 / hν
C H3 C H2 C H == C H2  
→ –
–HBr
(Product) BB H
BF
OMe
But-1-ene (Transition
Br State)
| MeOH (Product)
Br2 / hν
C H3 CH C H ==C H2  
→ 16. (i) The rate of S N 1 reaction depends only upon the concentration
δ γ β α –HBr
3-bromo-but-1-ene of the alkyl halide.
Br (ii) S N 1reaction proceeds through the formation of carbocation.
|
C H3 Cγ C H == C H2 The reactivity is decided by ease of dissociation of alkyl halide.
δ β α
| R  X q R⊕ + X È
Br
3,3- dibromobut-1-ene Higher the stability of R + (carbocation), higher would be the
reactivity towards S N1 reaction.
13. The reaction follows α, β-elimination mechanism to give a more
substituted stable alkene as a major product. As the substrate is a p - H3CO  C6H4  CH2⊕ is the most stable carbocation due
α, γ -dibromo (1, 3-) compound it gives a conjugated diene. to resonance and then CH3 CHCH2CH3 (2° carbocation) while
⊕ ⊕
CH3CH2 CH2(1°) is least stable.
Br Thus, the correct increasing order of the reactivity of the given
HOs H Alc. KOH halides towards the S N 1reaction is
D
Ph H
–2HBr CH3CH2CH2Cl < CH3 CHCH2CH3 < p-H3COC6H4CH2Cl
(It is very stable because |
Br sOH the conjugated diene system
present. It is further conjugated Cl
with the benzene ring.) (II) (I) (III)
Alkyl Halides 359

17. To show decolourisation, compound must be unsaturated. 21. NBS


O – + O hv
tert-BuONa

Br O—tBu
(Saturated) Br Br
(cannot decolourise Br2 water) HO OH H O/K CO
2 2 3 +
C6H5 – + C6H5 +
tert–BuONa

Br
(Unsaturated)
(decolourise Br2 water) 22. Alkyl fluorides can be prepared by action of mercurous fluoride
or antimony trifluorides (inorganic fluorides) on corresponding
O – + O
tert–BuONa alkyl halide. This reaction is known as Swarts reaction.

Br CH3Br + AgF → CH3F + AgBr


(Unsaturated) Methyl fluoride
(decolourise Br2 water)
O O But, when aciton of NaI/acetone takes place on alkyl chloride of
bromide, alkyl iodide forms. This reaction is called ‘Finkelstein
– +
tert–BuONa reaction’
Br C2H5Cl →
NaI
acetone
C2H5I + NaCl
(Unsaturated)
(decolourise Br2 water)
Free redical fluorination is highly explosive reaction, so not
18. The number of stereoisomers in molecules which are not preferred for the preparation of fluoride.
divisible into two equal halves and have n number of 23. Steric hindrance (crowding) is the basis of SN 2 reaction, by
asymmetric C-atoms = 2n. using which we can arrange the reactant in correct order of their
3-methyl-pent-2-ene on reaction with HBr in presence of peroxide reactivity towards SN 2 reaction.
forms an addition product i.e. 2-bromo-3-methyl pentane. It has
1
two chiral centres. Therefore, 4 stereoisomers are possible Rate of S N 2 ∝
Steric crowding of 'C'
Br
2 4 2 4 CH3
1 3 5 HBr 1 3 5
|
Peroxide CH3Cl > CH3CH2Cl > CH3  CH  Cl > CH3  C  Cl
(Less 1° | |
3-methylpent-2-ene Anti-Markownikoff's addition
crowded) CH3 CH3
Four stereoisomers are possible
(As molecule has two chiral 2° (More crowded)
centres and asymmetric).
As steric hinderance (crowding) increases, rate of S N 2 reaction
decreases.
19. An alkyl halide in presence of a bulkier base removes a proton
Note The order of reactivity towardsS N 2 reaction for alkyl halides is
from a carbon adjacent to the carbon bonded to the halogen. This
Primary > Secondary > Tertiary
reaction is called E2 ( β-elimination reaction). halides halides halides
Br (1° ) (2 ° ) (3 ° )
H
t-BuO–K+ C6H5 H
C == C
24. PLAN Acetone is an aprotic solvent and can dissolve both the
C6H5 C6H5 H nucleophile and the substrate and thus SN2 reaction is favoured.
C6H5
Also
H + tert-BuOH + Br– 1° 2° 3°
H SN 2 Alkyl halides
− SN 1
20. Key Idea Strong nucleophile ( OMe) in polar solvent (MeOH)
gives elimination products over substitution products but all S. O 1° alkyl halide but (C  Cl).
products are possible in different yields. Cl B.E. is decreased by electron
withdrawing [C 6H5CO ] group,
Cl OCH3 (a case of I-effect). Thus,
−+ maximum rate in SN 2 reaction
MeONa
CH3—C—CH2CH2CH3 CH3—C—CH2—CH2—CH3 + Q. 2° alkyl halide, rate is minimum
MeOH
Cl
CH3 CH3
(Less yield)
P. CH3 — Cl 1° alkyl halide
CH3 —C== CH—CH2—CH3+ CH2 ==C—CH2—CH2—CH3 R. 1° allylic halide but allylic 1°
Cl
carbocation is resonance
CH3 CH3
stabilised in SN 1reaction
(More yield)
Thus, reactivity order is S > P > R > Q
360 Alkyl Halides

25. Nucleophile PhS− substitute the Br − through SN 2 mechanism 31. If alkyl groups are same, the order of leaving ability of halides in
wih inversion of configuration at α-C. SN 2 reaction is
H3C Br RI > RBr > RCl > RF
H 3C SPh
32. CH−3 , being
the strongest base, has highest nucleophilicity.
F Cl
PhS
– F
 +
SN 2 33. C6H5  C  CH3 + SbCl 5 l C6H5  C  CH3 + SbCl6− The
 
H H
NO2 (I)
NO2
planar carbocation (I), when return back, forms racemic mixture
26. Reaction proceed through carbocation intermediate of the starting compounds.
CH3 H CH3
34. Compound in which metal is directly bonded to carbon, is known
CH3O— —NO2 – Cl+ are organometallic compound, e.g. CH3Li.
CH3 CH3
H Cl CH3  D O 
35. CH3  C − MgCl + → 2
CH3  C  D
CH3 H CH3  
CH3 CH3
+
CH3O— —NO2 H shift
+ C H OH
36. CH3  CH2  CH2  CH2  Cl + KOH →
2 5

H CH3 CH3CH2  CH == CH2


2°(I)
1- butene
CH3 H CH3
Ι
H2O
K
37. An alkyl halide containing at least one β-H, on treatment with
CH3O— —NO2 ethanolic KOH, undergoes dehydrohalogenation, giving alkene.
H2O
ΙΙ L +
Ethanol
CH3  CH2  CH2Br + KOH → CH3  CH == CH2
H CH3
3°(II) 38. During chlorination of alkane, if excess of alkane is treated with
stabilised by + R effect of – OCH3 Cl 2 (g ) in presence of light or heat, chance of mono-chlorination
OH OC2H5 predominate.
UV-light
C2H6 (excess) + Cl 2 → CH3CH2Cl + HCl
C 2 H 5O –
27. + C2H5Ι
anhy. C2H5ΟΗ 39. (a) Both I and II are 1° halide, undergos SN 2 reaction.
(d) III is a tertiary halide, undergoes SN 2 reaction. I is benzylic
SΝ2 reaction
bromide, it is very reactive in SN 1 also as if produces stable
Alcoholic NaOH benzylic carbonation.
28. CH3  CH2  CH2Br → CH3  CH == CH2
80 ° C
40. In both cases, hydrogenation of olefinic bond will render
HBr
→ CH3  CH  CH3 compound achiral as two identical ethyl group will come at the
 α-carbon which was earlier chiral carbon. However, in (a) and
Br (c), chirality will be retained even after hydrogenation.
CH3 H
H Br Br

29. H3C  N +  CH3I− → CH3OH + (CH3 )3 N+ I− H2
H3C CH3
 (a) H3C H2C Chiral
CH3
H Br
HO− H2
H Br
(b) H2C H 3C
Lack of β-H on quaternary ammonium iodide leads to SN 2 CH3
CH3
reaction otherwise E2 elimination usually takes place. CH3
CH3
30. SN 2 reaction at asymmetric carbon occur with inversion of Chiral
configuration and a single steroisomer is formed because the 41. Since, there is no mention of temperature, room temperature will
reactant and product are not enthtiomer. Therefore the sign of be considered and thermodynamically controlled product would
optical rotation may or may not change. be the major product as:
Alkyl Halides 361

CH3 CH3 45.


CH2 H+ CH2
CH2 Br– CH3 3. Column I Column II Explanation
3º-carbocation
P. NaOEt(2) O Et (strong
CH3 CH3 Cl
nucleophile) causes
CH2 dehydrohalogenation
CH3 CH3 Br of 3° alkyl halide
42. Both NH3 and CF2Cl 2 are used as refrigerant. Q. EtBr (3) 3° butoxide undergoes
ONa OEt
SN reaction with 1°
43. Vinyl halide (CH2 ==CH Cl) do not undergo nucleophilic alkyl halide
substitution reactions. This is because it forms highly unstable R. (i) Hg(OAC) 2 Mercuration-
⊕ OH
carbocation (CH2 == CH). It cannot delocalise its π-electron. In (ii) NaBH4 demercuration adds
vinyl halide C Cl bond possess double bond character also. (1) H2O by
Markownikoff’s rule
s r without rearrangement
CH2 CH Cl CH2 CH Cl
S. (i) BH3 Hydroboro-oxidation
44. For P, i.e. –+
(ii) H2O 2 /OH− adds H2O by
ONa + Br OH
anti-Markownikoff’s
(4) rule
For this reaction 1 and 4 are probable products.
Thus, P — (2), Q — (3), R — (1), S — (4)
Product 1 i.e., is formed due to substitution while Alc. KOH
OH 46. A. CH3  CHBr  CD3 → CH2 == CH  CD3
product 4 i.e., is formed due to elimination. A tertiary E2
E2 reaction is a single-step reaction in which both
carbocation i.e, + formed during the reaction. Remember deprotonation from β-C and loss of leaving group from α-C
for 3° carbocation ions elimination product predominates.
occur simultaneously in the rate-determining step.
For Q, i.e. + HBr C-D bond is stronger than C—H bond, C—H is preferably
OMe
broken in elimination.

Correctly matched product for this reaction is 2 i.e., . B. Ph  CHBr  CH3 reacts faster than Ph  CHBr  CD3 in
Br E2 reaction because in latter case, stronger C—D bond is to
The reaction proceeds as
be broken in the rate determining step.
+ HBr MeOH + C2 H5 OD
OMe Br C. Ph  CH2  CH2Br →

Ph  CD == CH2
C2 H5 O
For R i.e., + NaOMe
Deuterium incorporation in the product indicates E1CB
Br mechanism
C2 H5 O− −
Correctly matched product is 4 i.e., . It is a normal Ph  CH2  CH2Br l Ph  CH  CH2Br
elimination reaction and proceeds as carbanion
C2 H5 OD
+–
+ NaOMe l Ph  CHD  CH2Br
Br (Alkali in I
alcohol) D
C2 H5 O− 
3° alkyl halide preferes elimination. I → Ph  C  CH2  Br → Ph  C == CH2


For S i.e., + MeBr D
ONa
D. Both PhCH2CH2Br and PhCD2CH2Br will react at same
rate in E1 reaction because C—H bond is broken in fast non
The correct match is 3 i.e., OMe . The reaction proceeds rate determining step. Also E1 reaction follow first order
as kinetics.
47. Propene is produced
+ MeBr + NaBr
– +
ONa OMe (CH3 )2 Cu + CH2 == CHCl → CH3  CH== CH2
362 Alkyl Halides

48. Vinyl chloride is obtained by the reaction of HCl with ethyne. It has plane of symmetry thus, achiral.
Thus, chiral compounds are five. I, III A, III B, IV B and V.
49. Chlorine is most reactive.
50. 2-bromo-2-methylpropane is formed as major product. 57. The substrate has three different types of B—H, therefore, first
three structural isomers of alkenes are expected as :
51. In SN 1 reaction, leaving group is detached in the first step
H H
forming carbocation intermediate. Br
52. True H
KOH
53. Larger the size of donor atom, greater is its polarisability, + +
stronger is the nucleophile. 3 types of B-H
I II III
54. False
The last two alkenes II and III are also capable of showing
55. CCl 4 is fire retardent, used as fire-extinguisher.
geometrical isomerism, hence two geometrical isomers for each
56. Given compound undergoes free-radical bromination under of them will be counted giving a total of five isomers.
given conditions, replacing H by Br. (i) NaNH2
58. CH3CH2C ≡≡ CH → CH3CH2  C ≡≡ C  CH2CH3
C* is chiral carbon. (ii) CH3 CH2 Br X
CH2CH2CH3 CH2CH2CH3
C2H5
Br2 H H Alk. KMnO H
H Br H Br H2/Pd–BaSO4 4 OH
X C ==C
C2H5 H OH
CH3 CH2Br H5C2
Y C2H5
I. Chiral meso-diol
Br
CH2CH2CH3 59. Unimolecular reaction occur
CH—CH2CH3 CH3 CH3
Br Br  ∆  +
H Br CH3  C  CH2  Br l CH3  C  CH2
CH3  
CH3 CH3 CH3
primary carbocation
II. Achiral III. Chiral
CH3
Br 
methyl shift
CH2—CH2CH2CH3 → CH3  C  CH2 CH3
CH2— CH—CH3 +
I
H Br
H Br CH3
CH3 E1
CH3 CH3—C==CH—CH3

I CH3
IV. Chiral V. Chiral
S N1
(III) has two chiral centres and can have two structures. CH3—C—CH2CH3

CH2CH3 CH2CH3 OC2H5

H Br H Br Alc. KOH
60. C6H5CH2  CH  C6H5 → C6H5  CH == CH  C6H5
E2
H Br Br H 
Cl
CH3 CH3 CH3
(III) A (III) B  CH3
E2
61. CH3  C  Br + CH3 O− → CH2 == C
(IV) has also two chiral centres and can have two structures.  CH3
CH3 CH3 CH3
CH3 CH3
H Br H Br  
S 2
CH3  C  O− + CH3 Br → CH3  C  OCH3
N
H Br Br H
 
CH3 CH3 CH3 CH3
(IV)A (IV)B
Alkyl Halides 363

Br I Alc. KOH
62. 66. C6H5CH2  CH  CH3 → C6H5  CH == CH  CH3

H Ph NaI Ph H 
Br
MeO H acetone MeO H ↓ HBr
S N2 C6H5  CH  CH2  CH3
Ph Ph
inversion of configuration 
Br
occur only at α-carbon
CH3 CH 3
H 3C CH3 Alc. KOH 
63. 67. (i) H3C  CH2CH3 → CH3  C == CHCH3
– K=1 E2
C—I + I I—C Cl
H H
OH
H5C2 C2H5 
Aq. NaOH
(ii) CH3CH2CHCl 2 → [CH3CH2  CH  OH]
boil unstable
Above equilibrium is established which has equilibrium constant
equal to 1. Therefore, equilibrium mixture will have both the − H2 O
→ CH3CH2CHO
enantiomers in equal amount giving racemic mixture.
− −
68. (i) H2O < CH3OH < HO < CH3O
64. SN 2 reactions leads to inversion of configuration at α-C
(ii) CH3F < CH3Cl < CH3Br < CH3I
CH3 CH3
NaOH
69. Chloroform in presence of air and sunlight, oxidises slowly to
Br H H OH form a highly poisonous compound called phosgene
S N2
C2H5 C2H5 hν
2CHCl 3 + O2 → 2COCl 2 + 2HCl
product with inverted (Phosgene)
configuration
To prevent the above oxidation reaction, chloroform is kept in
+ dark bottles.
65. X X
70. (i) CS2 + Cl 2 → CCl 4 + S2Cl 2
– CS2 + 2S2Cl 2 → CCl 4 + 6S
Fe/H O
CCl 4 + 2 [H] →
2
CHCl 3 + HCl
Aryl halide (Chloroform)

Due to the above resonance phenomena, C—X bond acquire (ii) CaC2 + H2O → C2H2 + Ca(OH)2
partial double bond character and becomes difficult to break in Cl 2 (excess)
H2 /Ni
the rate determining step of SN 2 reaction. C2H2 → C2H6 → Cl 3C  CCl 3
hν (Hexachloroethane)
24
Alcohols and Ethers
Objective Questions I (Only one correct option) I OH
1. An organic compound ‘A’ (C 9H10O ) when treated with conc. HI (c) (d)
undergoes cleavage to yield compounds ‘B ’ and ‘C ’. ‘B’ gives
yellow precipitate with AgNO 3 whereas ‘C’ tautomerism to ‘ NC
NC I I
D’. ‘D’ gives positive iodoform test. ‘A’ could be
(2020 Main, 2 Sep II) 4. The synonym for water gas when used in the production of
methanol is (2019 Main, 10 April I)
(a) —O—CH2—CH==CH2
(a) natural gas (b) laughing gas
(c) syn gas (d) fuel gas
(b) —O—CH ==CH—CH3 5. The major product of the following reaction is
CH3 CH == CHCO2 CH3 LiAlH
 4→ (2019 Main, 9 April I)
(c) —CH2—O—CH ==CH2 (a) CH3CH == CHCH2OH (b) CH3CH2CH2CH2OH
(c) CH3CH2CH2CO2CH3 (d) CH3CH2CH2CHO

(d) H3C— —O—CH==CH2 6. The major product of the following reaction is


(2019 Main, 9 April I)
1. PBr3
2. The most suitable reagent for the given conversion is OH 2. KOH (alc.)
(2020 Main, 8 Jan I)
O
CH3
CONH2 CONH2
(a) (b) OH
C==O COCH3
?
HO O
HO2C HOH2C
(c) (d)
CN CN
(a) B2H6 (b) LiAlH4 O O
(c) NaBH4 (d) H2 / Pd
7. The major product of the following reaction is
3. The major product of the following reaction is (2019 Main, 8 April I)
(2019 Main, 10 April I) O
O Br
HI (excess) NaBH4

MeOH, 25°C

NC O OH OH
Br OMe
OH I (a) (b)
(a) (b)
OMe O
NC OH (c) (d)
NC OH
Alcohols and Ethers 365

8. The major product of the following reaction is O


(2019 Main, 8 April I)
(a) —O O
OCH3 n
Conc. HBr (excess)
O
Heat

OH
CH==CH2 O
(b) —O O—
Br OH n
(a) (b) O

OH
Br—CHCH3 Br—CHCH3 O
Br OH
(d) (c) O
(c) —O n
O

CH2CH2Br CH2CH2Br
OCH3
OH O
O—
 (d) —O
9. CH3CH 2 C  CH3 cannot be prepared by O
n

 OCOCH3
Ph (2019 Main, 12 Jan I)
(a) CH3CH2COCH3 + PhMgX 13. The major product formed in the following reaction is
(b) PhCOCH3 + CH3CH2MgX
(c) PhCOCH2CH3 + CH3MgX
O HI
(d) HCHO + PhCH(CH3 ) CH2MgX
Heat
10. The major product of the following reaction is O (2018 Main)
(2019 Main, 11 Jan II)
HO
(i) HCl
OH I
(ii) AlCl3 (Anhyd.) (a) (b)

OH I
HO Cl
(a) (b)
OH I
(c) (d)
I OH
HO Cl
(c) (d) 14. The acidic hydrolysis of ether (X) shown below is fastest
when (2014 Adv.)

11. Which is the most suitable reagent for the following


transformation? (2019 Main, 10 Jan II)
OH
Acid
 OR OH + ROH
CH3  CH ==CH  CH2  C H  CH3 →
CH3 CH ==CH CH2CO2H
(a) Tollen’s reagent (b) I2 / NaOH
(c) Alkaline KMnO4 (d) CrO2Cl 2 / CS2
[X ]
12. The major product of the following reaction is
(2019 Main, 10 Jan II) (a) one phenyl group is replaced by a methyl group
O CH3 (b) one phenyl group is replaced by a para-methoxyphenyl group
(i) Dil. HCl/∆
(c) two phenyl groups are replaced by two para-methoxyphenyl
O (ii) (COOH)2/ groups
CH3O OH Polymerisation
(d) no structural change is made to X
366 Alcohols and Ethers

15. An unknown alcohol is treated with the “Lucas reagent” to (c) oxidation by heating with copper followed by reaction with
determine whether the alcohol is primary, secondary or Fehling solution
tertiary. Which alcohol reacts fastest and by what mechanism? (d) oxidation with concentrated H2SO4 followed by reaction with
(2013 Main) Fehling solution
(a) Secondary alcohol by SN 1 (b) Tertiary alcohol by SN 1
(c) Secondary alcohol by SN 2 (d) Tertiary alcohol by SN 2 23. Which one of the following will most readily be dehydrated
in acidic condition? (2000, 1M)
16. The major product of the following reaction is
O OH OH
RCH2OH

H (anhydrous)
(a) (b)
O O O
(a) a hemiacetal (b) an acetal
(c) an ether (d) an ester (c) (d)
17. (I) 1, 2-dihydroxy benzene (II) 1, 3-dihydroxy benzene
OH OH
(III) 1, 4-dihydroxy benzene (IV) Hydroxy benzene
The increasing order of boiling points of above mentioned
24. The products of combustion of an aliphatic thiol (RSH) at 298
K are (1992)
alcohols is (2006, 3M)
(a) CO2 (g ), H2O(g ) and SO2 (g )
(a) I < II < III < IV (b) I < II < IV < III
(b) CO2 (g ), H2O(l ), and SO2 (g )
(c) IV < I < II < III (d) IV < II < I < III
(c) CO2 (l ), H2O(l ) and SO2 (g )
18. The best method to prepare cyclohexene from cyclohexanol (d) CO2 (g ), H2O(l ) and SO2 (l )
is by using (2005, 1M)
25. In CH3 CH2 OH, the bond that undergoes heterolytic cleavage
(a) conc. HCl + ZnCl 2 (b) conc. H3PO4
(c) HBr (d) conc. HCl most readily is (1988, 1M)
(a) C  C (b) C  O
19. When phenyl magnesium bromide reacts with tert butanol,
(c) C  H (d) O  H
which of the following is formed? (2005, 1M)
(a) Tert butyl methyl ether (b) Benzene 26. Hydrogen bonding is maximum in (1987, 1M)
(c) Tert butyl benzene (d) Phenol (a) ethanol (b) diethyl ether
CH3 (c) ethyl chloride (d) triethyl amine
H+ Br2/CCl4
20. [F ] C4H8Br2 27. HBr reacts fastest with (1986, 1M)
/ H2O
H3C 5 such products (a) 2-methyl propan-2-ol (b) propan-1-ol
OH
are possible (c) propan-2-ol (d) 2-methyl propan-1-ol

How many structures of F is possible? (2003, 1M) 28. An industrial method of preparation of methanol is
(1984, 1M)
(a) 2 (b) 5
(a) catalytic reduction of carbon monoxide in presence of ZnO-
(c) 6 (d) 3
Cr2O3
21. Compound ‘A’ (molecular formula C3H8O) is treated with (b) by reacting methane with steam at 900°C with nickel catalyst
acidified potassium dichromate to form a product ‘B’ (c) by reducing formaldehyde with LiAlH4
(molecular formula C3H6O) ‘B’ forms a shining silver mirror (d) by reacting formaldehyde with aqueous sodium hydroxide
on warming with ammoniacal silver nitrate. ‘B’ when treated solution
with an aqueous solution of H2NCONHNH2 and sodium 29. Diethyl ether on heating with conc. HI gives two moles of
acetate gives a product ‘C’. Identify the structure of ‘C’. (a) ethanol (b) iodoform (1983, 1M)
(a) CH3CH2CH == NNHCONH2 (2002, 3M) (c) ethyl iodide (d) methyl iodide
(b) H3C C == NNHCONH2
 30. The compound which reacts fastest with Lucas reagent at
CH3 room temperature is (1981 , 1M)

(c) H3C C == NCONHNH2 (a) butan-2-ol (b) butan-1-ol


 (c) 2-methyl propan-1-ol (d) 2-methyl propan-2-ol
CH3 31. Ethyl alcohol is heated with conc. H2 SO4 . The product
(d) CH3CH2OH + NCONHNH2 formed is
22. 1-propanol and 2-propanol can be best distinguished by (a) CH3COOC2H5 (b) C2H2 (1980 , 1M)
(a) oxidation with alkaline KMnO4 followed by reaction with (c) C2H4 (d) C2H6
Fehling solution (2001, 1M)
32. Which of the following is soluble in water? (1980, 1M)
(b) oxidation with acidic dichromate followed by reaction with
Fehling solution (a) CS2 (b) C2 H5 OH
(c) CCl 4 (d) CHCl 3
Alcohols and Ethers 367

Objective Questions II (b) Statement I is correct; Statement II is correct; Statement II is


not the correct explanation of Statement I.
(One or more than one correct option)
(c) Statement I is correct; Statement II is incorrect.
33. In the following reaction sequence, the correct structure (s)
(d) Statement I is incorrect; Statement II is correct.
of X is (are) (2018 Adv.)
Me N3 37. Statement I Solubility of n-alcohol in water decreases with
X
(1) PBr3, Et2O increase in molecular weight.
(2) NaI, Me2CO
(3) NaN3, HCONMe2
Statement II The relative proportion of the hydrocarbon part
Enantiomerically
pure in alcohols increases with increasing molecular weight which
permit enhanced hydrogen bonding with water. (1988 , 2M)
Me OH Me OH
(a) (b) Passage Based Questions
Passage 1
A tertiary alcohol H upon acid catalysed dehydration gives a
product I. Ozonolysis of I leads to compounds J and K.
(c) Me OH (d) Me OH
Compound J upon reaction with KOH gives benzyl alcohol
and a compound L, whereas K on reaction with KOH gives
34. The correct statement(s) about the following reaction only M.
sequence is (are)
O
(i) O2 CHCl 3 / NaOH H3C Ph
Cumene (C9 H12 ) →
+
P → M=
(ii) H 3O
NaOH Ph H
Q (major) + R (minor), Q → S (2008, 3 × 4M = 12M)
PhCH 2 Br (2016 Adv.)
38. The structures of compounds J , K and L respectively, are
(a) R is steam volatile
(b) Q gives dark violet colouration with 1% aqueous FeCl 3 (a) PhCOCH3 , PhCH2COCH3 and PhCH2COO− K+
solution (b) PhCHO, PhCH2CHO and PhCOO− K+
(c) S gives yellow precipitate with 2, 4-dinitrophenylhydrazine (c) PhCOCH3 , PhCH2CHO and CH3COO− K+
(d) S gives dark violet colouration with 1% aqueous FeCl 3 (d) PhCHO, PhCOCH3 and PhCOO− K+
solution
39. The structure of compound I is
35. The following ether, when treated with HI produces
(1999, 3M) Ph CH3 H3C Ph
(a) (b)
—O—CH2— + HI H Ph H Ph

Ph CH3 H3C CH3


(c) (d)
(a) CH2I (b) CH2OH H CH2Ph Ph H

40. Compound H is formed by the reaction of


O
(c) I (d) OH
(a) ; PhMgBr
Ph CH3
36. The products of reaction of alcoholic silver nitrate with
ethyl bromide are (1991, 1M) O
(a) ethane (b) ethene (b) ; PhCH2MgBr
(c) nitroethane (d) ethyl alcohol Ph CH3
(e) ethyl nitrite
O
(c) ; PhCH2MgBr
Assertion and Reason
Ph H
Read the following question and answer as per the direction
given below : O CH2
(a) Statement I is correct; Statement II is correct; Statement II (d) ;
is a correct explanation of Statement I. Ph H Ph MgBr
368 Alcohols and Ethers

Numerical Answer Type Question 52. Write the structures of the products :
HI (excess)
41. Total number of isomers considering both structural and (CH3 )2 CH  OCH3 →
heat (1998, 2M)
stereoisomers of cyclic ethers with the molecular formula
C4H 8O is ……… (2019 Adv.) 53. Give reasons for the following in one or two sentences.
“Acid catalysed dehydration of t-butanol is faster than that of
Fill in the Blanks n-butanol. (1998, 2M)
42. Glycerine contains one ……… hydroxyl group. (1997, 1M) 54. 2, 2-dimethyloxirane can be cleaved by acid (H+ ). Write
43. Aliphatic ethers are purified by shaking with a solution of mechanism. (1997, 2M)

ferrous salt to remove ……… which are formed on prolonged 55. A compound D(C8 H10 O) upon treatment with alkaline
standing in contact with water. (1992, 1M) solution of iodine gives a yellow precipitate. The filtrate on
44. A …… diol has two hydroxyl groups on …… carbon atoms. acidification gives a white solid E (C7 H6 O2 ). Write the
(1986, 1M) structures of D, E and explain the formation of E. (1996, 2M)
45. Ethanol vapour is passed over heated copper and the product 56. 3, 3-dimethylbutan-2-ol losses a molecule of water in the
is treated with aqueous NaOH. The final product is ……… presence of concentrated sulphuric acid to give
(1983, 1M)
tetramethylethylene as a major product. Suggest a suitable
True or False mechanism. (1996, 2M)
46. Sodium ethoxide is prepared by reacting ethanol with 57. When t-butanol and n-butanol are separately treated with a
aqueous sodium hydroxide. (1985, 1M)
few drops of dilute KMnO4 in one case only, the purple
47. The yield of a ketone when a secondary alcohol is oxidised is colour disappears and a brown precipitate is formed. Which
more than the yield of aldehyde when a primary alcohol is of the two alcohols gives the above reaction and what is the
oxidised. (1983, 1M) brown precipitate? (1994, 2M)

Subjective Questions 58. Compound X (molecular formula, C5 H8 O) does not react


48. O appreciably with Lucas reagent at room temperature but gives
—OH a precipitate with ammoniacal silver nitrate with excess of
MeMgBr, 0.42 g of X gives 224 mL of CH4 at STP. Treatment
H+/∆ (i) O3 NaOH of X with H2 in presence of Pt catalyst followed by boiling
X Y
(ii) Zn/CH3COOH with excess HI, gives n-pentane. Suggest structure for X and
write the equation involved. (1992, 5M)
Identify X and Y. (2005, 2M)
59. Arrange the following in increasing order of boiling point :
49. An organic compound P having the molecular formula
C5H10O when treated with dil H2SO4 gives two compounds, Q n-butane, n-butanol, n-butylchloride, iso-butane. (1988, 1M)

and R both gives positive iodoform test. The reaction of 60. How may be the following transformation be carried out (in
C5H10O with dil, H2SO4 gives reaction 1015 times faster than not more than six steps)?
ethylene. Identify organic compound of Q and R. Give the “Ethyl alcohol to vinyl acetate.” (1986, 3M)
reason for the extra stability of P. (2004)
61. Write down the main product of the following reaction :
50. Cyclobutylbromide on treatment with magnesium in dry
I /NaOH
ether forms an organometallic compound (A). The 2
Ethanol → (1985, 1M)
organometallic reacts with ethanal to give an alcohol (B) after
mild acidification. Prolonged treatment of alcohol (B) with an 62. Give a chemical test to distinguish between methanol and
equivalent amount of HBr gives 1-bromo-1- ethanol. (1985, 1M)
methylcyclopentane (C). Write the structures of (A), (B) and 63. Suggest a reason for the large difference between the boiling
explain how (C) is obtained from (B). (2001, 5M)
points of butanol and butanal, although they have almost the
51. Explain briefly the formation of products giving the same solubility in water. (1985, 2M)
structures of the intermediates.
HCl
64. An alcohol A, when heated with conc. H2 SO4 gives an alkene
(i) B. When B is bubbled through bromine water and the product
OH Cl
obtained is dehydrohalogenated with excess of sodamide, a
+ CH2Cl + etc. new compound C is obtained. The compound C gives D when
treated with warm dilute H2 SO4 in presence of HgSO4 . D can
CH3 CH3
also be obtained either by oxidising A with KMnO4 or from
HCl
(ii) acetic acid through its calcium salt. Identify A, B, C and D.
OH Cl (1983, 4M)
only (1999, 3M)
Alcohols and Ethers 369

65. State the conditions under which the following preparations 67. An organic liquid ‘A’ containing C, H and O with boiling
are carried out. Give necessary equations which need not be point 78°C, possessing a rather pleasant odour, on heating
balanced. with concentrated sulphuric acid gives a gaseous product ‘B’
(i) Ethanol from acetylene with the empirical formula, CH2 . ‘B’ decolourises bromine
(ii) Lead tetraethyl from sodium-lead alloy water as well as alkaline permanganate solution and takes up
(iii) Methyl chloride from aluminium carbide (1983, 3M) one mole of H2 (per mole of B) in presence of finely divided
66. A compound ‘X’ containing C, H and O is unreactive towards nickel at high temperature. Identify the substances A and B.
sodium. It does not add with bromine. It also does not react (1979 , 2M)
with Schiff’s reagent. On refluxing with an excess of HI, ‘X’
yields only one organic product ‘Y’. ‘Y’ on hydrolysis yields a Integer Answer Type Question
new compound ‘Z’ which can be converted into ‘Y’ by 68. The number of hydroxyl group(s) in Q is (2015 Adv.)
reaction with red phosphorus and iodine. The compound ‘Z’
Aqueous dilute
on oxidation with potassium permanganate gives a carboxylic H+ KMnO4 (excess)
H P Q
acid. The equivalent weight of acid is 60. What are the Heat 0°C
compounds ‘X’, ‘Y’ and ‘Z’ ? Write chemical equations HO
H 3C CH3
leading to the conversion of ‘X’ to ‘Y ’. (1981, 3M)

Answers
1. (c) 2. (a) 3. (d) 4. (c) 29. (c) 30. (d) 31. (c) 32. (b)
5. (a) 6. (c) 7. (d) 8. (b) 33. (b) 34. (b,c) 35. (a,d) 36. (c,e)
9. (d) 10. (a) 11. (b) 12. (c) 37. (c) 38. (d) 39. (a) 40. (b)
13. (d) 14. (c) 15. (b) 16. (b) 41. (10.0) 42. (secondary)43. (peroxides)
17. (c) 18. (b) 19. (b) 20. (d) 44. (vicinal; same) 45. (aldol) 46. (False)
21. (a) 22. (c) 23. (a) 24. (b) 47. (False) 68. (4)
25. (d) 26. (a) 27. (a) 28. (a)

Hints & Solutions


1. A (C9 H10O) has four isomeric ethers (options (a), (b), (c) and
(b) Aryl CH3
(d)), of each of which gets treated with conc. HI. linkage
O O
HI CH CH2
(a) O CH2 CH CH2 H3C CH CH Vinyl
–Is
(A) (A) linkage (A)
H

Ph O CH2 CH CH2 sI H I
r (c) CH2 O CH CH2
–Is
but SN2 attack of H
– sp2 (vinyl)
I with sp2 aromatic Ph
carbon will not take place CH2 Or CH CH2 sI

PhOH + I CH2 CH CH2 (it is allyl iodide sp3(benzyl)


(C ) (B) and responds to
AgNO3 test) H
but keto forms of
O O Ph CH2 I + O CH CH2
phenol
B C
keto-enol
or AgNO3 tautomerism
H
r s
do not respond to iodoform test. [PhCH2 ] NO3+AgI
Stable Yellow O C CH3 D
(b) and (d) will not respond to HI solvolysis, because in both of carbonation ppt.
Keto-methyl group
the ethers, the ether linkage (  O  ) is attached with sp2 carbon, is present. So, D
responds to
one is phenyl (aryl), while the other is vinyl. iodoform test.
370 Alcohols and Ethers

2. The most suitable reagent for the given conversion is B2H6. O


 LiAlH 4
CH3 CH3
CONH2 CONH2 CH3CH == CH C — OCH3 →
C C CH3 CH == CHCH2OH + CH3OH
B2H6
But -2-en -1-ol Methanol
Thus, the major product of the given reactant
HO2C HOH2C
CN CN
O
LiAlH4 or
NaBH4 H2/Pd (Given product) 
CH3CH == CHC OCH3 in presence of LiAlH4 is
CH3 CH3
CONH2 CH3NH2 CH3CH ==CH CH2OH and CH3OH. The reaction proceeds
CH CH through following mechanism.
OH OH Mechanism

HO2C HOH2C O Nucleophilic acyl


CN CH2NH2 – substitution reaction
CH3CH CH C OCH3 + H AlH3
s
3. The given reaction takes place as follows: O
Cleavage-I
| H+ |
+
CH3CH CH C OCH3
O O
HI (excess) H
Cleavage-II O H
Cleavage-I –
CH3CH CH C + CH3O
+ H An aldehyde
NC O H+ NC O
Cleavage-II H
cle H – A–
Nu
op lH
h
rea ilic a 3 s
II cti dd O
ge- on iti
ava on
cle
via SN1 pathway H+
[via cleavage I] CH3 CH CH CH2OH H 2O
CH3CH CH C H
Primary alcohol
I OH H
+ CH3OH + I
s
+ CH3 CH3I 6. Key Idea PBr3 reagent is used for the substitution of —Br group
+
+ CH (CH3)2CHI while alc. KOH reagent is used to carry out elimination
+ (CH3)2CHOH
reaction.
NC OH CH3 CH3 NC I
The given reaction proceed in following manner:
Product-1 Product-2
Step I In presence of PBr3, alcohols undergo substitution
Product-2 is formed because reactions to give halides. Reagent PBr3 is usually generated
(i) Cleavage-I will give more stable aryl carbocation. insitu by the reaction of red phosphorus with bromine.
(ii) Cleavage- I will give intermediate which is in conjugation PBr3
with ring. OH Br

4. The production of methanol from water gas is as follows: O O


ZnO + Cr2 O 3 3-bromohexanone
[CO + H2 ] + H2 → CH3OH
300 atm, 300º C
Water gas Methanol Step II 3-bromohexanone in presence of alc. KOH undergoes
or
syn gas elimination reaction and gives cyclohex-2-en-1-one.
It is an industrial process used for the production of methanol
Alc. KOH
where volume ratio of the reactant gases is maintained as: Br
syn gas : H2 = 2 : 1 and ZnO-Cr2O3 act as catalysts
Thus, water gas is also called syn gas because it is used for O O
Cyclohex-2-en-1-one
synthesis of methanol.

Key Idea LiAlH4 reagent is used for the reduction of CHO, 7. Major product obtained in the given reaction is
5.
O

— C OCH3. It does not reduce double bonds. O

The reaction of an ester with LiAlH4 produces two alcohols,


one corresponding to the acyl portion of the ester and one NaBH4 in the reaction is used for the reduction by addition of a
corresponding to the alkyl portion. hydride ion and a proton. Carbon-oxygen double bonds are easily
reduced by sodium borohydride. The actual reducing agent in
Alcohols and Ethers 371

these reductions is hydride ion (H− ). Hydride ion adds to the Step 3 As HBr is in excess, so, reaction will also take place at
carbonyl carbon and the alkoxide ion that is formed is alkene.
subsequently protonated by water. In other words, the OH OH
Markownikov’s rule, HBr
carbonyl group is reduced by adding an H− followed by an Addition reaction
H+ . The mechanism of the given reaction is as follows :
O O– CH == CH2 Br CH3
OH
C – C—CH2Br
CH2Br H from NaBH4 H 2O 
H
9. CH 3CH2 C  CH 3 cannot be prepared by HCHO and

OH
Ph
PhCH(CH3 )CH2MgX . This can be easily illustrated by
C—CH2 — Br following reaction.
EtOH, 25°C
O
O Intramolecular
H
cyclisation d– d+ d – d+
H C H + PhCH CH2 MgX Ph CHCH2 CH2 O MgX

CH3 CH3
8. Key Idea Ethers are least reactive functional groups. The
cleavage of C  O bond in ethers take place under drastic H 3O +

conditions with excess of HX .


Ph CH (CH2)2 OH
The major product obtained in the reaction is as follows :
CH3
OCH3 OH
Conc. HBr (excess)
The obtained product is not the required substance. While option
Heat
(a), (b) and (c) can readily prepare the required substance.
The reactions are as follows :
CH=
=CH2 Br—CH— CH3
d – d+
O O MgX
As conc. HBr is in excess. So, reaction will take place at both
d– d+
the substituents. Ph C CH2 CH3 + CH3MgX Ph C CH2CH3
Mechanism
Step 1 Protonation of ether to form oxonium ion. CH3

H OH
|r H3O+
O—CH3 O—CH3 + Brs Ph C CH2CH3
H—Br

CH3
d – d+
CH=
=CH2 CH ==CH2 O MgX
O
Oxonium ion
d– d+
Ph C CH3 + CH3CH2MgX Ph C CH3
Step 2 Attack of nucleophile at the protonated ether.

H s CH2CH3
|r H
|+ OH
O—CH3 SN2 O CH3
|

Br – + H 3O +
Slow Ph C CH3
Br

CH == CH2 CH2CH3
CH == CH2
d – d+
O O MgX
d – d+
OH CH3CH2 C CH3 + PhMgX CH3CH2 C CH3
CH3—Br +
Ph
CH == CH2 OH
H3 O +
CH3CH2 C CH3

Ph
372 Alcohols and Ethers

Hydrolysis of the ester group


10. In the given reaction, AlCl 3 helps in the generation of
H CH3
O OH
electrophile that further undergoes ArSE 2 reaction to give the
Dil. HCl/D
required product. Me
O – CH3COOH
O OH O O

{
AlCl3
HO HO Ether group
H—Cl remains unaffected
–AlCl s by dil. HCl (hot) OH OH C C OH
4

Me
Intramolecular O
ArSE2 reaction O H OH C C OH
H

–HCl O O O O
HO
–AlCl3 –H2O O
AlCls4 O
(COOH)2/
H Polymerisation O n
It does not show SN reaction
with HCl because of double bond
OCH3
nature of the O—C bond, (due
to +R-effect of the —OH group.)
13. Key idea The reaction given is a nucleophilic substitution
reaction in which cleavage at CO bond is visible. The product
11. The most suitable reagent to carry out given transformation is formation can be visualised with the help of following analysis.
I2/NaOH
OH These two atoms are directly
attached to benzene ring.
 Hence, development of
a O
C H3 C H== C H C H2 C H  C H3 Strong oxidation, [O]
→ any charge on these atoms is
6 5 4 3 2 1 (Oxidative cleavage) stabilised by the ring itself due
to conjugation (Resonance)
O O b

 If any one properly visualise the fact written with figure


C H3 C H== C H C H2  C  OH
6 5 4 3 2 above, than a conclusion can be made that CO bonds
+CHI3 ↓ + CO2 ↑ marked (a) and (b) in the figure will undergo heterolysis
during the reaction.
Here, the haloform reaction will give following reaction:
s s
The reaction can be represented as
2 4 6
1 5 I2/OH I2/OH
3 [O]
O O HI I
OH
Heat + I + CH3OH
CI3 s 2 4 OH + CHI3 O
OH 5 OH
1 3 (G)
(Yellow ppt.)
Mechanism
O O
Step I The reaction begins with the attack ofH + of HI on oxygen
(i) Tollen’s reagent (AgNO3 + NH4OH) is a mild oxidising to form oxonium ion as
agent. It does not react with C H CH3 group +
 O HI
O
OH ∆ H
(2°-alcohol). O O
Oxonium ion
(ii) Alkaline KMnO4 cannot perform the oxidative cleavage,
rather it will hydroxylate the C == C. Step II This oxonium ion undergoes lysis and addition of I− to
OH form two products as
 +
C H3 C H== C H C H2 C H  C H3 O
I– I + CH OH
6 5 4 3 2 1 H 3
OH OH OH O O
  
KMnO4
 → C H3 C H  C H  C H2 C H  CH3 Step III Similar pathway is followed at the other oxygen atom,
OH− 6 5 4 3 2 1 which can be visualised as
(iv) CrO2Cl2 / CS 2 will not react here.
I HI I I– I
12. In the given reaction, ester get cleaved in presence of dil. HCl ∆
+ CH3CH2I
+
and readily forms alcohol. This alcohol on reaction with oxalic O O O
acid undergoes polymerisation reaction.
H H
Note Mechanism of a reaction is always a logical sequencing of events
which may occur simultaneously as well.
Alcohols and Ethers 373

14. PLAN This problem can be solved by using the concept of stability of 21. A is an alcohol and its oxidation product gives Tollen’s test, i.e. B
carbocation and SN 1 reaction. must be an aldehyde (CH3CH2CHO).
When two phenyl groups are replaced by two para methoxy CH3CH2CHO + H2NHN  CO  NH2 →
group, carbocation formed will be more stable. As the semicarbazide
stability of carbocation formed increases, rate of acidic CH3CH2  CH== N  NH  CONH2
hydrolysis increases.
Cu/ ∆ Fehling
OCH3 22. 1-propanol → CH3  CH2  CHO → Cu 2O ↓
solution red
O
OCH3 group indicated Cu/ ∆  Fehling
by arrow increases 2-propanol → CH3  C  CH3 → No reaction
C C the stability of solution
carbocation O OH O O
electronically (by + M effect) 23. H+ + – H+
– H 2O
I conjugated
OCH3 O O O
More stable carbocation H+ – H+
– H 2O
Hence, (c) is the correct choice. +
II
15. The reaction of alcohol with Lucas reagent is mostly an SN 1 OH
reaction and the rate of reaction is directly proportional to the Although both reactions are giving the same product,
stability of carbocation formed in the reaction. Since 3° carbocation I is more stable than II.
R  OH forms 3° carbocation (most stable), hence it will
react fastest.
24. Thiol, (RSH), on combustion produces CO2 (g ), SO2 (g ) and
H2O(l ). At 298 K, H2O will be in liquid phase.
H+ RCH2OH
16.
+
25. OH → O− + H+ (has maximum electronegativity
: :

O O + –H O OCH2R
Acetal
difference)
Resonance
stabilisation 26. Ethanol is capable in forming intermolecular H-bonds :
H O—H O—H
O C2H5—O C2H5 C2H5
+

17. All dihydroxy benzene will have higher boiling points than 27. Reaction proceeds via carbocation intermediates :
monohydroxy benzene. Also, among dihydroxy benzenes, 1, CH3 CH3
2,-di-hydroxy benzene has lowest boiling point due to  − H2 O 
intramolecular H-bonding. CH3  C — OH + H+ → CH3  C +
 
O CH3 CH3
H (2-methylpropan -2-o1) (3 ° carbocation)

O CH3

Br −
H → CH3  C — Br
(intramolecular H-bonding in 1,2-dihydroxy benzene) .

CH3
OH
conc. H3PO4 ZnO-Cr O
18. 28. CO + H2 →
2 3
CH3OH
heat

Concentrated H3PO4 solution does not involve any substitution


29. CH3  CH2  O  CH2  CH3 + HI → 2C2H5I
product while with others, substitution products are also formed. 30. 2-methyl propan-2-ol is a tertiary alcohol, will react fastest with
19. C6 H5 MgBr + (CH3 )3 COH → C6 H6 + Mg[(CH3 )3 CO]Br Lucas reagent :
CH3 CH3
20. CH3  
+
H HCl
+ CH3  C — OH → CH3  C — Cl
H 2O  ZnCl 2 
OH trans
H3C CH3 CH3
H H SN1 reaction
+ C==C 31. On heating with concentrated H2SO4, ethanol would undergo
H3C CH3
cis
dehydration to produce ethene.
374 Alcohols and Ethers

32. Ethanol is soluble in water due to its ability to form OH O O H O H


intermolecular H-bonds with water :
H CHO
33. Key idea All the reactions involved in the problem are , NaOH
Nucleophilic substitution of second order i.e., SN 2 which have
the speciality of inversion of configuration at the carbon atom (Q) Major
(Q) Major
involved. CHO
(R) Minor
Of the reactions given
Reaction 1 in its generalised format is seen as
PBr3 O—Na Br CH2—Ph O—CH2Ph
ROH → RBr
In diethyl ether (Et 2O) CHO
Williamson's CHO
Reaction 2 is simple halogen exchange reaction called
synthesis
Finkelstein reaction. Its generalised format is
In acetone( Me2 CO ) (S)
RX + NaI → RI + NaX
(a) R is not steam volatile, but Q is steam volatile thus, incorrect.
where X =Cl or Br
Reaction 3 in its generalised format seen as (b) Q has enolic group thus, gives violet colour with 1% aqueous
HCONMe FeCl 3 solution thus, correct.
RI + NaN3 →
2
RN3 + NaI (c) S has carbonyl group hence, gives yellow precipitate with 2,
Now if the given product is 4-DNP thus, correct.
Me N3 (d) S does not give colour with FeCl 3 thus, incorrect.
*
35.
—O—CH2— + HI
and which is too enantiomerically pure i.e. 100% either
dextrorotatory or leavorotatory form, then the ‘X’ must be
Me OH —OH + —CH2I
*
Phenol does not react further with HI.
Note The configuration at carbon * atom in ‘X’ becomes inverted ••

due to SN 2 mechanism which is visible in the product as well.


36. CH3CH2Br + O == N  O− → CH3CH2NO2 + CH3CH2ONO
ambident nitroethane ethyl nitrite
Thus, the probable reactions will be nucleophile
Me OH Me Br
37. R—OH ← Hydrophilic
PBr3
Hydrophobic
Et2O
Increasing molecular weight increases hydrocarbon (R)
Me Br Me I proportion that lowers the solubility in water.
NaI
Me2CO
+ NaBr Passage 1
Compound J must be benzaldehyde because it on treatment
Me I Me N3 with KOH undergoing Cannizzaro’s reaction producing
NaN3 benzyl alcohol and pot-benzoate (L).
HCONMe2
KOH
C6H5  CHO → C6H5  CH2OH + C6H5COOK(L)
34. J benzyl alcohol
CH3
CH3 Also M is aldol condensation product formed from
CH CH3 O—O—CH acetophenone
CH3
O
O2 H3O +  KOH
Ph  C  CH3 →
acetophenone ( K )
O
OH CH3 O CH3
O 
—Ph
 − H2 O
Ph  C — CH2  C  Ph →
+ CH3—C—CH3
Acetone CHCl3/NaOH 
OH Ph M
Phenol ∆ , Reimer-Tiemann
reaction
(P)
Alcohols and Ethers 375

Ph 46. Ethanol is weaker acid than water, not neutralised with NaOH.
⇒I = C== CH  Ph
H3C
47. 2°-alcohol on oxidation yields ketone while 1°alcohol on
OH oxidation produces aldehyde which can further be oxidised to
 acid.
and H = Ph C — CH2  Ph
+
 —OH CH2 +
CH3
3 ° -alcohol H+ Ring – H+
48.
expansion
O
X
 − O
38. J = C6 H5 CHO, K = Ph  C  CH3 , L = PhCOOK +
H
O3
Ph [X ]
39. I = C== CH  Ph Zn/CH3COOH
H3 C O
Y
O O
 H 2O
40. Ph  C  CH3 + Ph  CH2 MgBr → NaOH
aldol
OH

Ph  C — CH2  Ph O
 dil. H2SO4/H +

CH3 49. CH3CH2OH + CH3—C—CH3


O
H P CH3 Q R

41. The structure of cyclic ether with molecular formula, C4 H8 O I2/NaOH


Both gives positive iodoform test.
are as follows:
Br OH
(chiral carbon)
Mg (i) CH3CHO
(1) O (2) * 50. —CH—CH3
R isomer ether (ii) H+/H2O ‘B’
= 2 isomer
, O S isomer
+ H–shift
H+ + Ring expansion
O —CH—CH3
(3) (4) R isomer = 2 isomer 2° Br
O , * S isomer
+ Br–
(1-chiral carbon)

O O R, R isomer 3° ‘C ’
(5) (6) R, S isomer = 3 isomer +
+
, H CH2
* * S, S isomer 51. (i)
(2-chiral carbon)
OH
Cl–
Total number of isomers of cyclic ether with molecular formula,
C 4H8O are 10.
Cl
42. Secondary : HO OH C–2(OH) is secondary +
CH2
Cl–
CH2Cl
OH
43. Peroxides : H2O2 + Fe2+ → H2O + Fe3+ H+ + Cl–
(ii)
44. Vicinal, same OH 2° Cl
OH
Cu NaOH  HI
45. C2H5OH → CH3CHO → CH3  CH  CH2  CHO 52. (CH3 )2 CH  O  CH3 → CH3  CH  CH3 + CH3I
∆ H 2O
aldol 
I
376 Alcohols and Ethers

53. Acid catalysed dehydration proceeds via carbocation Ag(NH3 )+2


⇒ H  C ≡≡ C  CH2  CH2  CH2OH →
intermediate. Also, greater the stability of reactive intermediate,
X
faster the reaction :
Ag  C ≡≡ C  CH2CH2CH2OH
CH3 CH3
  59. Isobutane < n-butane < n-butylchloride < n-butanol
H+
CH3  C — OH → CH3  C — CH3 Conc.H SO
 − H2 O 60. CH3CH2OH →
2 4
CH2 == CH2
+ ∆
CH3 (3 ° highly stable)
(t - butanol) Br 2 NaNH2
→ CH2  CH2 → H  C ≡≡ C  H
n-butanol forms less stable (1°) carbocation. 

H Br Br
Hg(CH3COO) 2
O O+ → CH2 ==CH  OCOCH3
54. H+ – H+ CH3COOH
H3C H3C vinyl acetate

H3C H3C 2 I /NaOH


OH2 61. C2H5OH → CHI3 + HCOONa
2,2-dimethyloxirane
CH3 OH iodoform
62. CH3  CH2 OH + I2 + NaOH → CHI3 ↓ (Iodoform test)
H3C—C——CH2
yellow
OH Iodoform test is not given by methanol.
55. E = C6H5COOH (benzoic acid) 63. Butanol forms intermolecular H-bonds, has higher boiling point
than butanal.
O
 I2 O
⇒ D = C6H5  C  CH3 → C6H5  COONa + CHI3  ∆
NaOH 64. (CH3COO)2 Ca → CH3  C  CH3 + CaCO3
CH3 OH CH3 D
  H+  + O
56. CH3  C — CH  CH3 → CH3  C — CH  CH3  H2 SO4
  CH3  C ≡≡ CH → CH3  C  CH3
CH3 CH3 C HgSO4 D

CH3 OH
Methyl  – H+ H C CH3 
→ CH3  C — CH — CH3 → 3 C == C Conc. Br 2
CH3  CH  CH3 → CH3  CH== CH2 →
shift  H3C CH3 H2 SO4 H2 O
+ B
CH3
Br
KMnO4 CHO + MnO2↓ 
57. brown CH3  CH  CH2
OH 
n-butanol
Br
X
Lucas reagent NaNH
58. Compound ‘X’ → No reaction at room X →
2
CH3  C ≡≡ CH
temperature. H 2O C
Ammoniacal Excess of HgSO Catalyst
C5H8 O → ppt, X → CH4; 65. (i) H  C ≡≡ C  H →
4
CH3  CHO →
AgNO3 CH3 MgBr H2 SO4 H2
H2 /Pt
X → n-pentane CH3CH2OH
HI excess
Heat
(ii) 4 C2H5Br + 4 (Na / Pb) → (C2H5 )4 Pb
Above information suggest that X has a terminal triple bond and sodium -lead tetraethyl lead
it contain primary —OH group. alloy
Alcohols and Ethers 377

H O CH2 == CH2 + Br2  H2O → CH2  CH2


(iii) Al 4C3 →
2
Al(OH)3 + CH4 (brown)  
Br Br

CH4 + Cl 2 → CH3Cl + HCl colourless

66. Compound X must be a symmetrical ether : CH2 == CH2 + alk. KMnO4 → CH2  CH2
purple  
HI Na OH OH
R  O R → 2R  I → No reaction, not an alcohol
colourless
X Y
Schiffs’ 68.
→ No reaction, not an aldehyde. H
+ Methyl shift
reagent H
Heat
H2 O [O] HO
R  I → R  OH (Z ) → Acid H3C CH3 H 3C CH3
EW = 60
 I2 / P

–H
R—I
Heat
⇒ Acid is CH3COOH and Z = CH3CH2OH CH3 H3C
⇒ X = C2H5  O  C2H5, Y = CH3  CH2  I CH3 CH3
‘P’
HI OH
CH3CH2  O  CH2CH3 → 2CH3CH2I
X Y KMnO4(dil)
0°C
HO
67. A is ethanol because B is an alkene (ethene). OH
H3C OH CH3
H2 /Ni
C2H5OH + Conc.H2SO4 → CH2 == CH2 → ‘Q’
CH3  CH3
25
Aldehydes and Ketones
Objective Questions I (Only one correct option) 5. The major product obtained in the following reaction is
(2019 Main, 8 April II)
1. The major product [C ] of the following reaction sequence CH3 O
will be (2020 Main, 4 Sep II) NaOH
OHC ∆
(i)NaBH 4
CH 2 == CH CHO → [ A ] CH3 H 3C
(ii) SOCl 2
(a) H (b) H

DBr O O
[B] [C] CH3 CH2
Anhy. AlCl3 CH3 CH3

(a) Br (b) D
D Br
(c) (d)
Br Br
O O
(c) D
(d) 6. The major product of the following reaction is
D (2019 Main, 8 April II)
O (1) t-BuOK
2. Which one of the following polymers is not obtained by
condensation polymerisation? (2) Conc. H2 SO4 /∆
(2020 Main, 5 Sep II) Cl
(a) Nylon 6,6 (b) Buna-N (c) Bakelite (d) Nylon 6
O O
3. The major product(s) obtained in the following reaction is/are (a) (b)
(2019 Main, 12 April I)

(i) KOtBu
(ii) O3/Me2S
O O
Br (c) (d)
CHO and OHC ¾ CHO
(a) OHC
7. An organic compound neither reacts with neutral ferric
(b) OHC chloride solution nor with Fehling solution. It however,
CHO reacts with Grignard reagent and gives positive iodoform
(c) OHC CHO test. The compound is (2019 Main, 8 April I)
O
OtBu
CH3 O
(d) OHC CH3
(a) (b)
CHO
H
OH
4. In the following reaction,
HCl O
Carbonyl compound + MeOH - acetal O OH
Rate of the reaction is the highest for: (2019 Main, 9 April II)
(a) Acetone as substrate and methanol in excess C2H5 CH3
(b) Propanal as substrate and methanol in stoichiometric amount (c) (d)
CH3 C2H5
(c) Acetone as substrate and methanol in stoichiometric amount O
(d) Propanal as substrate and methanol in excess O
Aldehydes and Ketones 379

8. In the following reactions, products A and B are Codes


(2019 Main, 12 Jan I) A B C D
O O (a) Q R S P
H
Dil. NaOH
[A] (b) R P Q S
H3C
H3C CH3 (c) Q P S R
[A]
H 3O +
[B]
(d) Q P R S

11. The major product ‘X’ formed in the following reaction is
O O (2019 Main, 10 Jan I)
CH3 CH3
O
(a) A = CH3 ; B = CH3 O
HO CH2 C OCH3 NaBH4
X
MeOH
O O
CH3 CH3 O
(b) A = CH3 ; B = CH3 OH OH
HO CH2 C OCH3
O CH2 CH2 OH
O (a) (b)
OH CH3
C
H H
H 3C ;B= O
(c) A = O OH
CH2 C H CH2 CH2 OH
H 3C H 3C
CH3 CH3 (c) (d)
O
O
OH
C H2C
H H 12. The correct sequence of reagents for the following
A = H 3C ;B= conversion will be (2017 Main)
(d)
H 3C H3C O HO CH3
CH3 CH3

9. The major product obtained in the following reaction is


(2019 Main, 11 Jan II)
O OH
CHO H3C OH
LiAlH4 CH3
(excess)
CH3
(a) [Ag(NH3 )2 ]+ OH− , H+ / CH3OH, CH3MgBr
NO2 O (b) CH3MgBr, H+ / CH3OH, [Ag(NH3 )2 ]+ OH−
OH (c) CH3MgBr, [Ag(NH3 )2 ]+ OH− , H+ / CH3OH
OH
(d) [Ag(NH3 )2 ]+ OH− , CH3MgBr, H+ / CH3OH

(a)
CH3
(b) 13. The major product of the following reaction sequence is
CH3 (2016 Adv.)
NO2 OH NH2 OH O

OH OH (i) HCHO (Excess)/NaOH, Heat


(ii) HCHO/H+ (Catalytic amount)

(c) (d)
CH3 CH3
O O O O OH
NO2 OH NH2 OH

10. The correct match between item ‘I’ and item ‘II’ is (a) (b)

Item ‘I’ Item ‘II’


(Compound) (Reagent) O OH
O O
(A) Lysine (P) 1-naphthol HO
(B) Furfural (Q) Ninhydrin
(c) (d)
(C) Benzyl alcohol (R) KMnO4
(D) Styrene (S) Ceric ammonium nitrate
(2019 Main, 10 Jan II) OH
380 Aldehydes and Ketones

14. Which compound would give 5-keto-2-methyl hexanal upon 18. The number of aldol reaction(s) that occurs in the given
ozonolysis? (2015 Main) transformation is (2012)
CH3 CH3 OH
  OH
Conc. aq. NaOH
CH3 CH3CHO + 4HCHO

(a) (b) HO
OH


CH3 (a) 1 (b) 2 (c) 3 (d) 4
CH3 CH3 19. Cyclohexene on ozonolysis followed by reaction with zinc
  dust and water gives compound E. Compound E on further
CH3

treatment with aqueous KOH yields compound F.


(c) (d)
Compound F is (2007, 3M)

CH3
(a) —CHO (b) —CHO
15. The major product in the following reaction is (2014 Adv.)
O
(i) CH3MgBr, dry ether, 0°C
Cl (c) —COOH (d)
(ii) Aqueous acid COOH
CH3
O 20. The smallest ketone and its next homologue are reacted with
NH2OH to form oxime (2006)
(a) H3C
CH3 (a) two different oximes are formed
OH (b) three different oximes are formed
(c) two oximes are optically active
(b) (d) all oximes are optically active
H 2C CH3
CH3 21. Butan-2-one can be converted to propanoic acid by which of
the following ? (2006)
(c) (a) NaOH, NaI/H+ (b) Fehling’s solution
CH2
O (c) NaOH, I2/H+ (d) Tollen’s reagent
CH3
CHO
(d) CH3COONa
O CH3 22. + (X )
MeO COOH
16. The most suitable reagent for the conversion of
R  CH2  OH → R  CHO is (2014 Main)
(a) KMnO4
(b) K 2Cr2O7 MeO
(c) CrO3
What is X ?
(d) PCC (pyridinium chlorochromate)
(a) CH3COOH (b) BrCH2COOH
17. The major product H in the given reaction sequence is (c) (CH3CO)2 O (d) HOC COOH
95% H2 SO4
CN
CH3  CH2  CO  CH3 → G → H
s 23. The order of reactivity of phenyl magnesium bromide with
the following compounds is (2004, 1M)
(a) CH3  CH == C COOH O O O

CH3   
(2012)
(b) CH3  CH == C CN H3 C CH3 H3 C H Ph Ph

CH3 (I) (II) (III)
OH (a) (II) > (III) > (I) (b) (I) > (III) > (II)
 (c) (II) > (I) > (III) (d) All of the above
(c) CH3  CH2  C COOH
 24. A mixture of benzaldehyde and formaldehyde on heating
CH3 with aqueous NaOH solution gives (2001, 1M)
(d) CH3  CH == C CO  NH2 (a) benzyl alcohol and sodium formate
 (b) sodium benzoate and methyl alcohol
CH3 (c) sodium benzoate and sodium formate
(d) benzyl alcohol and methyl alcohol
Aldehydes and Ketones 381

25. The appropriate reagent for the following transformation: Objective Questions II
O (One or more than one correct option)
CH2CH3 35. Reagent(s) which can be used to bring about the following
CH3
transformation is (are) (2016 Adv.)

HO HO
O O O O
(a) Zn (Hg), HCl (b) NH2NH2 , OH– C O C
(c) H2/Ni (d) NaBH4
O H O OH
26. Which of the following has the most acidic hydrogen?
(2000, 1M) COOH COOH
(a) 3-hexanone (b) 2, 4-hexanedione
(c) 2, 5-hexanedione (d) 2, 3-hexanedione (a) LiAlH 4 in (C2H5 )2 O (b) BH 3 in THF
(c) NaBH 4 in C2H5OH (d) Raney Ni/H 2 in THF
27. The enol form of acetone, after treatment with D2 O, gives
36. The major product of the following reaction is (2015 Adv.)
(1999, 2M)
O
OD O
(i) KOH, H2O
 
(a) H3C  C ==CH2 (b) D3C C  CD3 CH3
(ii) H+, Heat
OH OD O
 
(c) H2C==C CH2D (d) D2C==C CD3 CH3 CH3
O O
28. Which of the following will react with water? (1998, 2M) (a) (b)
(a) CHCl 3
(b) Cl 3CCHO O
(c) CCl 4 O CH3
(d) ClCH2CH2Cl
(c) CH3 (d)
29. Which of the following compounds is oxidised to prepare
methyl ethyl ketone? (1987, 1M)
(a) 2-propanol (b) 1-butanol 37. After completion of the reactions (I and II), the organic
(c) 2-butanol (d) t-butyl alcohol compound (s) in the reaction mixtures is/are (2013)
O
30. The compound that will not give iodoform on treatment with Br2 (1.0 mol)
alkali and iodine is (1985, 1M) H3C CH3 Aqueous/NaOH
Reaction I
(a) acetone (b) ethanol O
(c) diethyl ketone (d) isopropyl alcohol Br2 (1.0 mol)
31. The Cannizzaro’s reaction is not given by (1983, 1M) Reaction II H3C CH3 CH3COOH
(a) trimethyl acetaldehyde (1.0 mol)
(b) acetaldehyde O O O
(c) benzaldehyde
(d) formaldehyde H 3C CH2Br H3C CBr2 Br2C CBr2
P Q R
32. When acetaldehyde is treated with Fehling’s solution, it
gives a precipitate of O O
(1983, 1M)
(a) Cu (b) CuO CHBr3
BrH2C CH2Br H 3C ONa
(c) Cu2O (d) Cu + Cu2O + CuO S T U
33. A compound that gives a positive iodoform test is (1982, 1M) (a) reaction I : P and reaction II : P
(a) 1-pentanol (b) 3-pentanone (b) reaction I : U, acetone and reaction II : Q, acetone
(c) 2- pentanone (d) pentanal (c) reaction I : T , U , acetone and reaction II : P
(d) reaction I : R, acetone and reaction II : S , acetone
34. The reagent with which both acetaldehyde and acetone react
38. Tautomerism is exhibited by (1998)
easily is (1981, 1M)
(a) Tollen’s reagent
(a) —CH==CH—OH (b) O== ==O
(b) Schiff ’s reagent
(c) Grignard’s reagent
(d) Fehling’s reagent
(c) ==O (d) ==O
==

==

O O
382 Aldehydes and Ketones

39. A new carbon–carbon bond formation is possible in CH3


(1998, 2M) H H
(a) Cannizzaro’s reaction (b) Friedel-Crafts’ reaction CH H C
(b) H3C C and
(c) Clemmensen’s reduction (d) Reimer-Tiemann reaction
O
40. Which of the following will undergo aldol condensation? O
(1998, 2M)
(a) Acetaldehyde (b) Propanaldehyde H3C CH2 H H 3C H
CH C C
(c) Benzaldehyde (d) Trideutero acetaldehyde (c) and
41. Among the following compounds, which will react with CH3 O O

acetone to give a product containing C==N ?


H3C CH2 H H H
(a) C6H5NH2 (b) (CH3 )3 N (1998, 2M) CH C C
(c) C6H5NHC6H5 (d) C6H5NHNH2 (d) and
CH3 O O
42. Which of the following is an example of aldol condensation?
Dil.NaOH 47. The compound R is
(a) 2CH3CHO → CH3CH(OH)CH2CHO (1989, 1M) O O
OH
Dil.NaOH  H 3C C H 3C C
(b) 2CH3COCH3 → H3C C CH2COCH3 (a) C H (b) C H
 H 3C H3C
CH3 H2C CH
OH H 3C OH
Dil . NaOH
(c) 2HCHO → CH3OH + HCOONa
Dil . NaOH CH3 O CH3 O
(d) C6H5CHO + HCHO → C6H5CH2OH +HCOONa
CH C CH C
43. Which of the following compounds will react with ethanolic
(c) H3C CH H (d) H3C CH H
KCN? (1984, 1M)
(a) Ethyl chloride (b) Acetyl chloride H2C CH
(c) Chlorobenzene (d) Benzaldehyde OH H3C OH
44. Which of the following compounds will give a yellow 48. The compound S is
precipitate with iodine and alkali ? (1984, 1M)
(a) 2-hydroxy propane (b) Acetophenone CH3 O
O
(c) Methyl acetate (d) Acetamide
CH C
45. Base catalysed aldol condensation occurs with (1984, 1M) H 3C H H3C C
(a) CH (b) C H
(a) propionaldehyde H3C
(b) benzaldehyde H 2C H2C
CN CN
(c) 2-methyl propionaldehyde
(d) 2, 2-dimethyl propionaldehyde
CH3 CN CN
Passage Based Questions CH CH H3C CH
(c) H3C CH OH (d) C OH
Passage 1 H3C
Two aliphatic aldehydes P and Q react in the presence of aqueous H2C H2C
OH OH
K 2 CO3 to give compound R, which upon treatment with HCN
provides compound S . On acidification and heating, S gives the
product shown below : (2010) Passage 2
H 3C OH A carbonyl compound P, which gives positive iodoform test,
H3C undergoes reaction with MeMgBr followed by dehydration to give
O
an olefin Q. Ozonolysis of Q leads to a dicarbonyl compound R,
O which undergoes intramolecular aldol reaction to give
predominantly S. (2009)
46. The compounds P and Q respectively are
CH3 1. MeMgBr O3 /Zn -H2O OH–
H3 C H P → Q → R → S
CH C 2. H+ , H2O heat
H
(a) H3C C and 3. H2SO4 / heat
O
O
Aldehydes and Ketones 383

49. The structure of the carbonyl compound P, is 52. The structure of the product I is
CH3
(a) (b) (a) H3C Br
O CH3 O CH3 (b) H3C
O Br
(c) H3C Br
(c) (d)
(d) H3C Br
O C2H5 CH3
50. The structures of the products Q and R, respectively, are 53. The structures of compounds J and K , respectively, are
O
(a) H3C COOH and SOCl2
H
(a) CH3
COCH3
CH3 (b) H3C
H3C H3C CH3 COOH and SO2Cl2
O (c) H3C and SOCl2
COOH
H
(b)
CHO (d) H3C COOH
and CH3SO2Cl
CH3
H3C H 3C CH3
O 54. The structure of product L is
(a) H3C CHO
H
(c)
CHO
Et Et
Me Me (b) H3C CHO
O
Me (c) H3C
CH3 CHO
(d)
CHO
Me (d) H3C CHO
Me Et
51. The structure of the product S, is
O Match the Columns
55. Match each of the compounds given in Column I with the
(a) (b) reaction(s) that they can undergo, given in Column II.

O Column I Column II
Me Me A. p. Nucleophilic
Me Br
O O substitution

Me
O
(c) (d)
B. q. Elimination
OH
Me Me

Passage 3
C. CHO r. Nucleophilic addition
In the following sequence, product I , J and L are formed. K
represents a reagent.
(i) Mg/ether
OH
(i) NaBH4 (ii) CO2 K
Hex-3-ynal I J
(ii) PBr3 (iii) H3O+ D. Br s. Esterification with
H2 acetic anhydride
H 3C Cl L
Pd/BaSO4 quinoline
NO2
O (2008, 3 × 4M = 12M)
t. Dehydrogenation
384 Aldehydes and Ketones

56. Match the compounds/ions in Column I with their 64. An alkene (A) C16 H16 on ozonolysis gives only one product
properties/reactions in Column II. (2007, 6M) (B) C8 H8 O. Compound (B) on reaction with NaOH / I2 yields
sodium benzoate. Compound (B) reacts with KOH/NH2 NH2
Column I Column II yielding a hydrocarbon (C ) C8 H10 . Write the structures of
A. C6 H5 CHO p. gives precipitate with compounds (B) and (C ). Based on this information two
2, 4-dinitrophenylhydrazine isomeric structures can be proposed for alkene (A). Write
their structures and identify the isomer which on catalytic
B. CH3 C ≡≡ CH q. gives precipitate with AgNO3 hydrogenation (H2 / Pd – C) gives a racemic mixture.
(2001, 5M)
C. CN– r. is a nucleophile
65. Identify A, B and C, and give their structures.
D. I− s. is involved in cyanohydrin
formation O
CH3
CH3 Br2
Fill in the Blank A +B
NaOH
57. Fehling’s solution A consists of an aqueous solution of O H+
copper sulphate, while Fehling’s solution B consists of an heat
C(C7H12O) (2000)
alkaline solution …… (1990, 1M) 66. An organic compound A, C6 H10 O, on reaction with CH3 MgBr
followed by acid treatment gives compound B. The compound
True or False B on ozonolysis gives compound C, which in presence of a
58. The reaction of methyl magnesium iodide with acetone base gives 1-acetyl cyclopentane D. The compound B on
followed by hydrolysis gives secondary butanol. (1987, 1M) reaction with HBr gives compound E. Write the structures of
A, B, C and E. Show, how D is formed from C. (2000, 5M)
59. Benzaldehyde undergoes aldol condensation in an alkaline
medium. (1982 , 1M) 67. What would be the major product in the following reaction ?
Ph
Integer Answer Type Questions base
60. Consider all possible isomeric ketones including O
stereoisomers of MW = 100. All these isomers are Br
(2000, 1M)
independently reacted with NaBH4 . The total number of
ketones that gives a racemic product(s) is/are 68. (a) Compound A (C8 H8 O) on treatment with NH2OH. HCl
given B and C. B and C rearrange to give D and E,
NOTE (Stereoisomers are also reacted separately). (2014 Adv.)
respectively, on treatment with acid. B, C, D and E are all
61. In the scheme given below, the total number of isomers of molecular formula (C8 H9 NO). When D is boiled
intramolecular aldol condensation products formed fromY is with alcoholic KOH, an oil F (C6H7 N) separates out. F
reacts rapidly with CH3COCl to give back D. On the other
1. O3 1. NaOH(aq) hand, E on boiling with alkali followed by acidification
Y
2. Zn, H2O 2. heat gives a white solid G (C7 H6O2). Identify A-G.
(2010)
(b) Carry out the following transformation in not more than
three steps.
Subjective Questions 1-butyne → 2-pentanone (1999, 3M)
HCl 69. Write the intermediate steps for each of the following
62. (A), C6H12 → (B), C6H13Cl + (C ), C6H13Cl
reactions
Alcoholic KOH
(B) → (D), (an isomer of (A)) H3O+
(i) C6H5CH(OH)C ≡≡ CH → C6H5CH == CHCHO
O zonolysis
(D) → (E), (positive iodoform and negative
Fehling’s solution test) –
H
(ii)
Ozonolysis
(A) → (F) + (G), (positive Tollen's test for both) OH O CH3
Conc. NaOH
(F) + (G) → HCOONa + A primary alcohol 70. Complete the following reactions with appropriate structures
Identify the compounds (A) to (D). (2003) of products/reagents.
O
63. A compound C9H7O2Cl exists predominantly in enol form 
(A) and also in keto form (B). On oxidation with KMnO4 it O CHC6H6
gives m-chlorobenzoic acid as one of the products. Identify [C] (i) LiAIH4
the compounds (A) and (B). (2003) (ii) H +, heat
(1998)
Aldehydes and Ketones 385

71. An aldehyde A (C 11 H8 O), which does not undergo self aldol 80. A ketone A, which undergoes haloform reaction, gives
condensation, gives benzaldehyde and two moles of B on compound B on reduction. B on heating with sulphuric acid
ozonolysis. Compound B, on oxidation with silver ion gives gives compound C, which forms mono-ozonide D. D on
oxalic acid. Identify the compounds A and B. (1998, 2M) hydrolysis in the presence of zinc dust gives only
acetaldehyde. Identify A, B and C. Write down the reactions
72. Acetophenone on reaction with hydroxylamine hydrochloride involved. (1989, 4M)
can produce two isomeric oximes. Write structures of the
oxime. (1997, 2M) 81. Answer the followings with suitable equations wherever
necessary
73. Complete the following, giving the structures of the principal (i) suggest a reagent to distinguish acetaldehyde from acetone.
organic products,
O (ii) what happens when excess chlorine is passed through
boiling toluene in the presence of sunlight? (1987, 2M)

82. Complete the following with appropriate structures


(i) + Ph3P CH2 A
NaOH
? CH CH—CHO
(1986, 1M)
(ii) ClCH2CH2CH2COPh + KOH + MeOH → B
O 83. How may the following transformation be carried out (in not
more than six steps) “benzaldehyde to cyanobenzene”?
C6H5
H3O+ (1986, 2M)
(iii) H3C + NaOH C
O (1997, 2M) 84. Give reason in one or two sentences for the following:
74. Suggest appropriate structures for the missing compounds. “Hydrazones of aldehydes and ketones are not prepared in
(the number of carbon atoms remains the same throughout highly acidic medium”. (1986, 1M)
the reaction) 85. Write down product of the following reaction
CH3 NaOH
Propanal →
heat (1985, 1M)
Dil. KMnO4 HIO4 HO–
A B C
86. Arrange the following in order of their increasing reactivity
towards HCN :
CH3 (1996, 3M) CH3CHO, CH3COCH3 , HCHO, C2H5COCH3 (1985, 1M)

75. Complete the following reaction with appropriate structure : 87. Write down the reactions involved in the preparation of the
CH3 CH2 following using the reagents indicated against in parenthesis:
(i) KCN/H2SO4
C==O → “Acetoxime from acetaldehyde.”
(ii) LiAlH4
H (1996, 1M) [K2Cr2O7 / H+ , Ca(OH)2 and NH2OH, HCl] (1984, 2M)

76. Complete the following reaction with appropriate structure. 88. Show with balanced equation, what happens, when the
following are mixed :
NaOC 2 H5 in absolute
C6H5CHO + CH3COOC2H5 → A “Chloral is heated with aqueous hydroxide” (1984, 2M)
C2 H5 OH heat (1995, 1M)
89. An alkene A on ozonolysis yields acetone and an aldehyde.
77. Write the structure of the major organic product expected The aldehyde is easily oxidised to an acid B. When B is
from the following reaction. (1992, 1M) treated with bromine in presence of phosphorus yields a
compound C which on hydrolysis gives a hydroxyl acid D.
KOH
MeO CHO + HCHO This acid can also be obtained from acetone by the reaction
with hydrogen cyanide followed by hydrolysis. Identify the
78. Arrange the following in the increasing order of expected compounds A, B, C and D. (1982 , 2M)

enol content. 90. Outline the reaction sequence for the conversion of methanal
CH3 COCH2 CHO, CH3 COCH3 , CH3 CHO, to ethanol (the number of steps should not be more than
CH3 COCH2 COCH3 (1992, 1M) three). (1981, 2M)

79. Give reason in one or two sentences : 91. Write the structural formula of the main organic product
“Iodoform is obtained by the reaction of acetone with formed when methanal reacts with ammonia. (1981, 1/2M)
hypoiodite but not with iodide”. (1991)
386 Aldehydes and Ketones

Answers
1. (c) 2. (b) 3. (a) 4. (d) 37. (c) 38. (a,c,d) 39. (b,d) 40. (a,b,d)
5. (c) 6. (d) 7. (d) 8. (b) 41. (a,d) 42. (a,b) 43. (a,b,d) 44. (a,b)
9. (b) 10. (c) 11. (b) 12. (a) 45. (a,c) 46. (b) 47. (a) 48. (d)
13. (a) 14. (b) 15. (d) 16. (d) 49. (b) 50. (a) 51. (b) 52. (d)
17. (a) 18. (c) 19. (a) 20. (b) 53. (a) 54. (c)
21. (c) 22. (c) 23. (c) 24. (a) 55. (A → p, q, t B → p, s, t C → r, s D → p)
25. (b) 26. (b) 27. (b) 28. (b) 56. (A → p, q, s B → q, r C → q, r, s D → q, r)
29. (c) 30. (c) 31. (b) 32. (c) 57. (Sodium potassium tartarate)
33. (c) 34. (c) 35. (c) 36. (a) 58. (False) 59. (False) 60. (5) 61. (1)

Hints & Solutions


O
1.
O (i) NaBH4 reduce C group in —OH and then
(i) NaBH4 (ii) SOCl2 react with it
CH2 CH C H CH2 CH CH2Cl
(ii) SOCl2
(A )
+ Anhy. AlCl3

(Friedel-Crafts reaction)

Br D
r s
CH2 CH CH2
CH CH2 CH2 DBr
Br

D
Hence (C) will be
Step I
O 3. 3 Step II CHO
 Step I
2
O3/Me2S
and
Step I NaBH4 reduce  C  H group into alcohol. OHC
KOtBu Ozonolysis OHC CHO
1
Step II SOCl 2 react with this alcohol and form R — X . Br Cyclohex-1,
3-diene
Step III This R — X react with benzene in the presence of
In step-1 dehydrohalogenation reaction takes place. Here,
anhydrous AlCl 3 via Friedel-Crafts reaction and form product. hydrogen is eliminated from β-carbon and the halogen is lost
Step IV DBr react with this product and form rearranged from α-carbon atom. As a result diene is formed.
product.
2. Buna-N is an addition polymer. It is obtained by H –
copolymerisation of 1, 3-butadiene and acrylonitrile in the OtBu
presence of a peroxide catalyst. Br Cyclohex-1,3-diene

CH2 == CH CH == CH2 + CH2 == CH C ≡≡ N


Buta-1, 3 -diene Acrylonitrile Cyclohex -1, 3-diene on ozonolysis gives butane-1, 4- dial and
ethane- 1, 2- dial.
 Na CHO
↓ |
O3 CHO
  Me2S
| + (CH2)2
  CHO |
CH2 CH == CH2 C H CH2  CHO
  
 CN 
Nitrile rubber (Buna-N)
Aldehydes and Ketones 387

7. According to the given conditions, compound (d) neither reacts


4. Key Idea Aldehydes are more reactive than ketones in
nucleophilic addition reactions. with neutral ferric chloride solution nor with Fehling solution. It
however reacts with Grignard reagent and gives positive
For the reaction, iodoform test.
HCl
● As the compound does not contain any phenolic OH
Carbonyl compound + MeOH Acetal - group. Hence, it gives negative neutral FeCl 3 test.
Rate of reaction is the highest for propanal as substrate and OH
methanol in excess. Propanal is an aldehyde and more reactive
than ketones. When MeOH is taken in excess then reaction CH3 Neutral
moves in the forward direction that results in the formation of no reaction
FeCl3
(FeCl3 test for phenolic
acetal. Reaction involved is as follows : C—Et group)
Et Et OH Et OMe
H+/MeOH MeOH O
C O C C
Excess ● Compound gives reaction with RMgX as it contains
H H OMe H OMe
Acetal
C  Et.

5. It is an intramolecular aldol condensation reaction. O
OH OH
This attack will result 7-membering. | |
So, it will not proceed. CH3 CH3
RMgX
O O O Grignard R
s 4 reagent C2H5
OH 6 1 C—Et
OHC CH2 HC 2 CH2
–H2O 5 3 OH
H H 7 s O
H Compound with CH3CH  group undergoes iodoform test in
Acidic H-atom O
2 
OH– 5 4 1
O ==CH OH
– H 2O 3 s
presence of NaOH and I2.
Less acidic OH O
H s |
HO H 2O O CH3 NaOH+I2
O–Na+
∆ 5
4
3 CH3I ↓ +
Iodoform
–H2O – OH s 1 2
test Yellow Et
(option-c) C—Et ppts.
O Major O
O H More acidic O O
β-H for β-elimination
8. The reactant in presence of dil ⋅ NaOH undergoes intramolecular
6. –I
O O aldol condensation reaction.
H β-elimination
As a result of this, β-hydroxyketone (A) is obtained which on
Cl s r /∆
tBuOK hydrolysis followed by heating produces α , β-unsaturated
–HCl
Acidic hydrogen H ketone (B).
This benzylic –H activates O O O O
r
p-position by +ve H (Conc. H2SO4) dil.
for ArSO2 Electrophilic addition
H 3C H NaOH s H
H 3C CH3
H H
O O

r
2° O O O
r 1°
2°, but less stable carbocation H2O
1°, but more stable D
because of –I effect of O
carbocation s
–H2O HO O
ArSE 2 ArSE 2 (B) H (A ) H
H H
r O r O 9. LiAlH4 acts as a nucleophilic reducing agent that can reduce
COOH to CH2OH,  C == O into  CH  OH and
 
H H NO2 into NH2, but it cannot reduce isolated C==C
6- membered ring OH OH
∆ –Hr (more stable) ∆ –Hr O isolated C==C
H
O O LiAlH4 (excess)
CH3
Major product NO2 NH2 OH
(option-d) 5- membered ring Minor product
(less stable) (option-c) Compound
(b)
388 Aldehydes and Ketones

10. (A) → Q; B → (P) ; C → (S), D → (R) 13. O O


(A) Lysine (R=–(CH2)4–NH2)

OH
NH2 H
Ninhydrin test (B)
R CH Violet
COOH O
colouration H-atom at α carbon Carbanion
OH
(Ninhydrin), O
OH boil
O O
O
OH H—C—H H—C—H

(B)
CHO +
O O
O
(Furfural)
CH2SO4
D Violet colouration
Crossed – H 2O
Molisch Test (P) aldol CH2O CH2OH
(Molisch reagent)
α-carbon has no H-atom
CH2OH Ceric ammonium CHO hence, next reaction with HCHO is
nitrate (s) [CAN] crossed Cannizzaro reaction
(C)
50% CH3COOH HCHO, OH−

(Benzyl alcohol) (Benzaldehydel) OH


O O
OH
OH HCHO

dil. KMnO4/OH /Cold (R) Acetal CH2OH + HCOO–
(D) formation (By oxidation)
(Pink) H H
(By reduction)
H
(Styrene) CH3
+Mn2+(Colourless)
14. (a) 
CH3

11. NaBH4 is a selective reducing agent. It reduces carbonyl O3



C O Zn.H2O2
group into an alcohol but cannot reduce an isolated
C == C and an ester group too. O O
 
Carbonyl CH3  C  CH2  CH2  CH2  C  CH3
group O O Heptan -2, 6 dione
O OH
CH2 C OCH3 CH2 C OCH3 CH3
NaBH4

MeOH

Ester
group (b) O3
Isolated ∆
C C Zn.H2O2

CH3
12. O
O O O HO CH3 6 5 4 3 2 1
CH3  C  CH2  CH2  CH  CHO
[Ag (NH3)2]OH Esterification CH3MgBr 
Tollen’s reagent CH3OH, H+ (Excess) CH3
5-keto-2-methyl hexanal

CHO COOH COOCH3 OH CH3


Before final product is formed, intermediate is 

O3
HO CH3 (c) ∆
Zn.H2O2

CH3
O CH3
 
O CH3
CH3  C  CH2  CH  CH2  CHO
5-keto- 3-methyl hexanal
Aldehydes and Ketones 389

CH3 18. The given reaction is an example of repeated aldol condensation


 followed by Cannizzaro reaction.
(d) CH3


O3 −
∆ Step I CH3CHO + OH− → CH2  CHO + H2O
Zn.H2O2
O
O CH3  −
  H  C  H + CH2  CHO r
CH3  C  CH  CH2  CH2  CHO O− OH
5-keto-4- methyl hexanal
 H 2O

H  C  CH2  CHO → CH2  CH2  CHO
15. PLAN This problem includes concept of nucleophilic addition reaction

to carbonyl compound (ketone here) and intramolecular
nucleophilic substitution reaction.
H
Complete reaction sequence is as shown below: Step II HOCH2  CH2  CHO + HO− r

O HO  CH2  CH  CHO + H2O
Cl CH3MgBr, dry ether, 0°C O
CH3 Nucleophilic  −
addition reaction H  C  H + CH  CHO r

CH2OH
OMgX CH3 O− OH
aq. acid  H 2O

Cl intramolecular
CH3 nucleophilic H  C  CH  CHO → CH2  CH  CHO
O CH3  
CH3 substitution reaction 
+ Mg X Cl H CH2OH CH2OH

Step III HOCH2  CH  CHO + HO− r


PCC 
16. R — CH2OH  → R — CH == O CH2OH
Pyridinium chlorochromate is the mild oxidising agent which −
HOCH2  C  CHO + H2O
causes conversion of alcohol to aldehyde stage. While others

causes conversion of alcohol to acid. CH2OH
17. The first step is cyanohydrin reaction CH2OH
O
O  
 − H  C  H + – CH  CHO r
CH3  CH2  C  CH3 + CN → 
CH2OH
O− OH
 H2O  O− CH2OH CH2OH
CH3  CH2  C  CN → CH3  CH2  C  CN   
  H 2O
H  C  C  CHO → HOCH2  C  CHO
CH3 CH3   
(I) H CH2OH CH2OH
In the second step the  CN of intermediate (I) is first
hydrolysed and then dehydrated on heating in the presence of
CH2OH O O−
conc. H2SO4.   
Step IV HOCH2  C  C  H + H  C  OH
OH OH  
 H2SO4  CH2OH H
CH3  CH2  C  CN → CH3CH2  C  COOH
  OH
CH3 CH3 OH
Cannizzaro reaction
(I) + HCOO–
∆ HO
→ CH3  CH == C  COOH OH

CH3 In the last step, formaldehyde is oxidised and the other aldehyde
is reduced giving the desired products.
390 Aldehydes and Ketones

19. O OH O
 [O] 
H 29. CH3  CH  CH2  CH3 → CH3  C  CH2  CH3
O3 H
2- butanol catalyst Ethyl methyl ketone
Zn-H2O
O
E
30. Compounds that contain either CH3  CO or CH3  CH 
group gives iodoform test : 
KOH CHO OH
O
H2O 
Intramolecular aldol
CH3  C  CH3 CH3  CH H CH3  CH  CH3
condensation reaction Acetone  
OH OH
20. Three different oximes are formed out of which two are optically Ethanol Isopropyl alcohol
active i.e.exists as a pair of enantiomers while other is optically
Above three compounds has the desired group for iodoform test.
inactive.
Diethyl ketone does not has the required group for iodoform
21. CH3
NaOH test.
H 3C CHI3 + CH3CH2COONa
I2/H2O 31. Aldehydes lacking presence of α-H undergo Cannizzaro
O
H+ reaction. When treated with aqueous base CH3CHO does not
CH3CH2COOH undergo Cannizzaro reaction because it has α-H and in the
presence of aqueous base it undergoes aldol condensation.
22. X is (CH3CO)2 O and it is an example of Perkin’s reaction.
23. The reactivity of carbonyl compound towards nucleophilic 32. CH3  CHO + Fehling’s solution → Cu 2O ↓
Red
addition of Grignard’s reagent depends on extent of steric
hindrance at α-carbon. Greater the steric hindrance smaller the O
reactivity. Hence, reactivity order is 
33. For iodoform test, CH3  C  group is required
CH3CHO > CH3  CO  CH3 > Ph  CO  Ph
II I III O
O + NaOH + I2
NaOH H3C CH3
24. —CHO + H—C—H —CH2OH 2-pentanone COONa
H2O + CHI3↓
Yellow
+ H— COONa
This is an example of cross Cannizzaro reaction in which 34. Grignard’s reagent reacts with both aldehydes and ketones
formaldehyde is always oxidised. while other three reagents reacts only with aldehydes, not with
ketones.
25. O
35. Only  CHO group is to be reduced to CH2OH.
NH2–NH2 CH2CH3
CH3 It can be done using NaBH4 in C2H5OH.
HO–

HO HO
O IV O
(Wolff-Kishner reduction) C O
26. O O O O
O III H
+ H+ II
– COOH
Highly resonance I
H stabilised
OH O (a) LiAlH4 / (C2H5 )2 O reduces I, II and III into CH2OH, and
  IV into diol.
27. CH2 == C  CH3 + D2O r D3C  C  CD3
All α-H will be replaced by deuterium. (b) BH3 / THF show same properties as (a).

H (c) NaBH4 / C2H5OH reduces III into CH2OH.


28. O
Cl (d) Raney nickel, same as (a) and (b), thus (c) is correct
Cl3C—CHO + H2O Cl—C—–CH reagent.

Cl
H O
Highly stable hydrate
Aldehydes and Ketones 391

36. CH3 OH
O O O 
Dil. NaOH
42. 2CH3CHO → CH3  CH  CH2CHO
HO– Aldol

CH3 O CH3 O
Aldol type  Dil. NaOH
 
O Condensation 2CH3  C  CH3 → CH3  C — CH2  C  CH3
CH3 CH3 
O OH
O O Aldol
H+ Dil. NaOH
∆ 2HCHO → CH3OH + HCOONa
–H2O Cannizzaro reaction
α, β-unsaturated ketone Dil. NaOH
C6H5CHO + HCHO → C6H5CH2OH + HCOONa
37. Plan When acetone reacts with Br2 in basic medium, bromoform is Cannizzaro reaction
formed. S 2
N
Reaction I CH3COCH3 + 3Br2 + 4NaOH 43. CH3CH2Cl + KCN → CH3CH2CN + KCl
1 mol 3 mol O O
1
mol
1 mol  
3 CH3  C  Cl + KCN → CH3  C  CN + KCl
→ CH3COONa + CHBr3 + 3NaBr + 3H2O O O
(T ) (U )  
When CH3COCH3 and Br2 are in equimolar quantity, all the Br2 C6H5  C  H + KCN → C6H5  C  CH  C6H5
(limiting reactant) is converted into desired products and 
OH
2/3 mole of CH3COCH3 remains unreacted, being in excess. Benzoin
When acetone reacts with Br2 in acidic medium, there is O
monobromination of acetone. 
Reactions II 44. Aldehyde, ketones containing CH3  C  group gives iodoform
CH COOH test. Also alcohol containing CH3  CH  group gives iodoform
CH3COCH3 + Br2 →
3
CH3COCH2Br + HBr 
1 mol 1 mol (P ) OH
CH3COCH3 and Br2 react in 1 : 1 mole ratio and (P) is formed. OH O
In reaction I, (U ) and (T ) are formed and acetone (reactant)  
remains unreacted. In reaction II, (P) is formed. test. CH3  C — CH3 C6H5  C  CH3

38. All those carbonyl compounds containing α-H to sp2 carbon H
show keto-enol tautomerism. Both gives positive iodoform test
Esters and amides do not give iodoform test.
39. In both Friedel-Craft’s reaction and Reimer-Tiemann reaction
new carbon-carbon bond is formed. 45. For base catalysed aldol condensation, there must be at least one
CH3 α-H to carbonyl group.
AlCl3 CH3  CH2  CHO,
+ CH3Cl Propionaldehyde
(2 α -H)
Friedel-Crafts' alkylation CH3 CH3
OH OH  
CH3  CH  CHO , CH3  C — CHO
CHO 2-methyl propionaldehyde 
H+ (1 α -H) CH3
+ CHCl3 + NaOH 2, 2-dimethyl propionaldehyde
(no α -H)
Reimer-Tiemann’ reaction Passage 1 (For Q. Nos. 46-48)
40. All carbonyl compounds containing α-H or α-D undergo aldol The given product is an ester, obtained by condensation of a
condensation. In given example, benzaldehyde does not contain hydroxy acid obtained through hydrolysis of a cyanohydrin :
α-H to —CHO, hence does not undergo aldol condensation.
H3C H3C CH3
OH
41. C == O + C6H5NH2 → C == N  C6H5
H3C H3C CH3—C——CH H3C OH
H+
H3C CH2 C==O ∆
+ C6H5NHNH2 → C == N  NHC6H5 H3C
H3C HO
OH O O
392 Aldehydes and Ketones

Acid above is obtained by acid hydrolysis of cyanohydrin S as Column I Column II


55.
CH3 OH CH3 OH
H+ Br Undergo nucleophilic substitution of
H3C—C——CH—CN H2O
H3C—C——CH—COOH Br − . Undergo elimination of HBr.
A.
CH2OH CH2OH Does not undergo nucleophilic
S O addition. Does not esterify with
acetic anhydride, can be
S is obtained by nucleophile addition of HCN on R, hence R is dehydrogenated.
CH3 O CH3 OH Undergo nucleophilic substitution
OH
H3C—C——C—H + HCN H3C—C——CH—CN B. with SOCl 2 , PCl 5 etc.
Does not undergo elimination.
CH2OH CH2OH
Does not undergo nucleophilic
R S
addition.
R is obtained by treatment of P and Q with aqueous K2CO3 Undergo esterification with acetic
through aldol condensation reaction as anhydride.
CH3 O CH3 Undergo dehydrogenation to give
OH– C6 H5 CHO.
CH3—CH—CHO + H—C—H OHC—C—CH2OH
P+Q CH3 CHO Does not undergo nucleophilic
R C. substitution, there is no leaving
Passage 2 (For Q. Nos. 49-51) group.
OH
Does not undergo elimination.
CH3 CH3 Undergo nucleophilic addition at
CH3MgBr H+/H2O
CH3 carbonyl carbon of —CHO.
O CH3 Undergo esterification with acetic
CH3 anhydride.
P Q
(gives positive iodoform test) Does not undergo dehydrogenation.

CHO Br Undergo aromatic nucleophilic


O3 CH3 OH– D. substitution (SN Ar)
Q
Zn-H2O aldol NO2 Does not undergo elimination,
O O nucleophilic addition, esterification
H 3C CH3 or dehydrogenation.
R S
56.
Passage 3 (For Q. Nos. 52-54)
Column I Column II
O
A. C6H5CHO Gives phenyl hydrazone with 2,
1. NaBH4
4-dinitrophenyl hydrazine.
H 2. PBr3 Br Gives precipitate with AgNO3, Tollen’s
I
test forms cyanohydrin.
(i) Mg/ether
B. CH3  C ≡≡ CH Gives precipitate (CH3  C ≡≡ CAg)
(ii) CO2
+
COOH with AgNO3 .
(iii) H3O J
A nucleophile, undergo electrophilic attack.
SOCl2 C. CN− Forms AgCN with AgNO3.
J
K C—Cl A nucleophile is involved in cyanohydrin
formation.
O
D. I− Gives AgI precipitate with AgNO3 and it is
H H
H2 a nucleophile.
C==C
Pd/BaSO4
H5C2 CH2CH2CHO 57. Sodium potassium tartarate
L
Aldehydes and Ketones 393

O CH3 CH3 CH3 Cl


 H2 O    
58. CH3MgI + CH3  C  CH3 → CH3  C — OH 62. CH2 == CH  C — CH3 CH3  CH — C — CH3
  
CH3 CH3 CH3
Tertiary alcohol A B

59. Benzaldehyde, lacking α-H does not undergo aldol Cl CH3 CH3
condensation, rather it undergoes Cannizzaro reaction.   
CH3  CH — C — CH3 CH3  C == C — CH3
60. Molecular weight of the ketone is 100  
So, molecular formula = C6 H12O CH3 CH3
12 C D
Degree of unsaturation = (6 + 1) − =1
2 O CH3
According to question, compound contains ketone group. Since,  
the compound which contain chiral centre lead to the formation CH3  C  CH3 CH3  C — CHO HCHO
of diastereomer while other produces enantiomers. Various
E  G
CH3
isomers and their possible reduced product are as shown below. F
(1) n-butyl–C—CH3 n-butyl–CH—CH3 63. H
O O O
O OH
All are CHO
(2) Iso-butyl–C—CH3 Iso-butyl–CH—CH3 + racemic H
mixture
O NaBH4 OH
(3) 3° butyl–C—CH3 3° butyl–CH—CH3
Cl Cl
O OH A B
COOH
CH3 CH3 KMnO4

*
(4) CH3CH2—CH—C—CH3 CH3—CH2—CH—CH—CH3
* *
O OH Cl
R (2°butyl) 2-alcohols (R,R) and (R,S) 64. B + NaOH / I2 → C6H5COONa + CHI3
diastereomeric pair
CH3 NaBH4 O
CH3
(5) CH3CH2—CH—C—CH3 CH2CH3
* * C
CH3—CH2—CH—CH—CH 3 N2H4/OH–
O * ⇒ B= CH3
S (2°butyl)
OH
2-alcohols (S,S) and (S,R)
(Here, (*) represents chiral centre) diastereomeric pair Ph Ph Ph Ph
A= C == C + C == C
H3C CH3 H3C CH3
(6) n-propyl–C—Et n-butyl–CH—Et I II
(±) racemic H
O OH mixture
NaBH4 H
(7) Iso-propyl–C—Et Iso-propyl–CH—Et Ph
II + H2/Pd + Enantiomer
(±)racemic CH3
O = Racemic mixture
OH mixture Ph CH3
65. O O
While in case of (4) and (5) they do not produce enantiomer CH3
due to the presence of stereogenic centre on ketone. CH3
CH3 Br2
61. O O
NaOH
COONa + CHBr3
O ‘B’
O3 OH–
‘A’ O
Zn-H2O
CH3
O Only product H+
Y
Heat
+ CO2

C
394 Aldehydes and Ketones

66. O (b) CH3  CH2  C ≡≡ CH


O
OH– C 
B2 H6 (i) CH3 MgBr
C CH3 → CH3CH2CH2  C  H →
Aldol H2 O2 /OH− (ii) H3 O+
D OH
CHO O 
CH3 CH3 CH3CH2CH2  CH  CH3
⇒ C= B= O
[O] 
→ CH3CH2CH2  C  CH3
MeMgBr H+
A (C6H10O) → → B 2-pentanone.

O 69. (i) C6H5  CH  C ≡≡ C  H



OH
⇒ + CH3MgBr +
H+
→ C6H5  CH  C ≡≡ C  H
− H2 O
A HO CH3 CH3 ••
OH2
H+ O
−H+ 
→ C6H5  CH== CH  C  H
B
C6 H5  CH == C == C  OH
67. Ph
Base
Ph
e
tautomerisation 
H
_
O O (ii) H+
Br Br +
Enolisation
OH O
Ph
H
O
– H+

68. (a) G is benzoic acid C6H5  COOH , B and C are two O


stereomeric oximes which undergo Beckmann’s rearrangement O
on treatment with acid to give amides D and E. 70.
O O CH—Ph
(i) NaOH
 OH−
C6H5  C  NHCH3 → C6H5COOH + CH3NH2 (ii) C6H5CHO
E H2 O G
O CH—Ph
H5C6 OH
+ (i) LiAlH4
H
C==N C6H5—C—NHCH3
(ii) H+, heat
H 3C E
C 71. Aldehyde A does not has any α-H but undergo ozonolysis to
O give two moles of compound B and benzaldehyde. Compound B
H5C6 + on oxidation gives oxalic acid, so A is
H
C==N CH3—C—NHC6H5
O3
H3C OH
D C6H5  CH == CH C ≡≡ C  CHO →
A
B KOH
CH3COOH + C6H5NH2 C6H5  CHO + 2COOH
H2O 
F
CHO
F + CH3COCl → C6H5NHCOCH3 B
O D
 Ag+
B → COOH
⇒ A = C6H5  C  CH3 
COOH
Aldehydes and Ketones 395

72. O O
H5C6 OH
 C2 H5 ONa
C6H5—C—CH3 + H2NOH C==N 76. C6H5CHO + CH3  C  OC2H5 →
C2 H5 OH
H3C
O
H5C6 
C6H5  CH == CH  C  OC2H5
+ C==N
H3C 77. KOH
OH MeO— —CHO + HCHO
O CH2

MeO— —CH2OH + HCOOK


73. (i) + Ph3P==CH2 (Wittig reaction)
(Cannizzaro reaction)
A
In cross-Cannizzaro reaction, methanal is always oxidised.
O
 CH3O− O O H
(ii) Cl  CH2  CH2  CH2  C  Ph → O O
Cl Ph H
O B H3C H
CH2 II
CH2 Ph—C—
II is less stable than I because II is less substituted enol.
HC B
Acetone has greater enol content than ethanal
O OH
COPh
 
O O CH3  C  CH3 s CH2 == C  CH3 (more substituted)
(iii) C
C6H5
OH– – O OH
H 3C H2C C==O
 
O CH3  C  H s CH2 == C — H (less substituted)
Ph D
O O Therefore, overall enol-content order is D < C < B < A
H+/∆
OH 78. CH3  CO  CH2  COCH3 has highest enol content due to
—Ph
Ph C resonance and formation of six membered ring through
intramolecular H-bonding
74.
O O H
CH3 CH3 CH3 O O
Dil. KMnO4 OH HIO4 O H3C CH 3
OH O (A) H3C CH3
(I)
CH3 CH3 CH3
A B Also, enol content depends upon the number of substituents on
O double bond, greater the number of substituents, greater the
HO– stability, higher the enol content.
B Therefore, CH3COCH2CHO forms next most stable enol
aldol

C 79. Iodoform reaction is an oxidation reaction in which hypoiodite


OI− acts as oxidising agent :
75. KCN + H2SO4 → KHSO4 + HCN O
O OH 
  R  C  CH3 + OI− → R  COO− + CHI3
CH3CH2  C  H + HCN → CH3CH2  C — CN
 Iodide (I− ) is a reducing agent, does not give iodoform reaction.
OH H
 O
Zn CH CHO (only)
LiAlH4
→ CH CH  C — CH NH I 80. D →
3
→ 3
3 2 2 2 H2 O

H C = CH3  CH == CH  CH3,
II
I is formed as racemic mixture.
396 Aldehydes and Ketones

O 86. Steric hindrance at carbonyl carbon determine the reactivity


towards nucleophilic addition reaction. Greater the steric
D = CH3  CH CH  CH3 hindrance, smaller the reactivity.
 
O  O C2H5COCH3 < CH3COCH3 < CH3CHO < HCHO
OH O H + Ca(OH)
87. CH3CHO → CH3COOH → (CH3COO)2 Ca
2
  K 2 Cr 2 O7
B = CH3  CH  CH2CH3 and A = CH3  C  CH2CH3 O
81. (i) Tollen’s reagent gives grey precipitate of Ag, acetone heat 
 H2 NOH
does not. (CH3COO)2 Ca → CH3  C  CH3 →
CH3 CCl3 H3C
hν C == N  OH
(ii) + Cl2 + HCl H3C acetoxime
(excess)
88. Cl 3C  CHO + NaOH(aq) → Cl 3C  CH2OH
82. NaOH Cannizzaro reaction
C6H5CHO + CH3CHO —CH==CH—CHO
+ Cl 3C  COONa
Cross-aldol condensation
O OH
83.  HCN
 H+
CHO COOH 89. CH3  C  CH3 → CH3  C — CN →
[O] NaOH  H2 O
K2Cr2O7/H +
CaO/∆ CH3 OH

NO2 CH3  C — COOH
HNO3

H2SO4 CH3
Br D
CH3
NO2 NH2  
Fe/HCl NaNO2 ⇒ C = CH3  C — COOH , B = CH3  CH — COOH
HCl/0°C 
+ CH3
N2Cl– CN
KCN CH3 CH3
CuCN  
⇒ A = CH3 C== CH  CH  CH3
O O
 R 
84. R  C  R + PhNHNH2 → C ==N  NHPh E ther
90. H  C  H + CH3MgBr → CH3CH2OMgBr
R Hydrazone
H+
In acidic medium, hydrazine reacts to form salt and hydrazone is → CH3CH2OH
hydrolysed back to aldehyde/ketone. H2 O

91. N
OH−
85. CH3CH2CHO → CH3  CH2  CH == C  CHO
heat  6CH2O + 4NH3 ‘‘(CH2)6N4’’
N
CH3 N N
Aldol followed by
dehydration Hexamine
26
Carboxylic Acids and
Their Derivatives
Topic 1 Carboxylic Acids
Objective Questions I (Only one correct option) 5. The compound that undergoes decarboxylation most
1. The correct order of acid strength of the following carboxylic readily under mild condition is (2012)

acids is (2019 Adv.) COOH COOH


O
CH2COOH O
O (a) (b)
OH
I H II H
OH H COOH CH2COOH
COOH O
O
III MeO IV H3C OH (c) (d)
OH O
6. In the following reaction sequence, the correct structures
(a) III > II > I > IV of E , F and G are
(b) I > II > III > IV
O O
(c) II > I > IV > III
(d) I > III > II > IV Heat I2
Ph * OH [E] [F] + [G]
NaOH
2. In the reaction,
LiAlH 4 PCl 5 Alc. KOH (* implies 13 C labelled carbon) (2008, 3M)
CH3 COOH → A → B → C O O
The product C is (2014 Main)
(a) E =
*
F= *
⊕ G = CHI3
(a) acetaldehyde Ph CH3 Ph ONa
(b) acetylene
(c) ethylene O O
(d) acetyl chloride (b) E = F= ⊕ G = CHI3
*
Ph CH3 Ph ONa
3. The compound that does not liberate CO2 , on treatment with
aqueous sodium bicarbonate solution, is (2013 Adv.) O O
(a) benzoic acid *
(c) E = * F= ⊕ G = CHI3
(b) benzenesulphonic acid Ph CH3 Ph ONa
(c) salicylic acid
(d) carbolic acid (Phenol) O O
*
4. An organic compound A upon reacting with NH3 gives B. On (d) E = * F= ⊕ G = CH3I
Ph CH3 Ph ONa
heating, B gives C. C in the presence of KOH reacts with Br2
to give CH3 CH2 NH2 . A is (2013 Main)
7. When benzene sulphonic acid and p-nitrophenol are treated
(a) CH3COOH
(b) CH3CH2CH2COOH
with NaHCO3 , the gases released respectively, are
(a) SO2 , NO2 (2006, 3M)
(c) CH3  CH  COOH
 (b) SO2 , NO
CH3 (c) SO2 , CO2
(d) CH3CH2COOH (d) CO2 , CO2
398 Carboxylic Acids and Their Derivatives

8. An enantiomerically pure acid is treated with racemic List–I List–II


mixture of an alcohol having one chiral carbon.The ester
P. Ph 1. I2 , NaOH
formed will be (2003, S, 1M)
HO
(a) optically active mixture Me
(b) pure enantiomer Ph + H2SO4
OH
(c) meso compound
Me
(d) racemic mixture
9. Benzoyl chloride is prepared from benzoic acid by Q. Ph 2. [Ag(NH 3 )2 ] OH
(a) Cl 2, hν (b) SO2Cl 2 (2000, S, 1M) H 2N
H
(c) SOCl 2 (d) Cl 2 , H2O Ph + HNO2
OH
10. When propionic acid is treated with aqueous sodium Me
bicarbonate, CO2 is liberated. The C of CO2 comes from
(1999, 2M) R. Ph 3. Fehling
(a) methyl group HO solution
Ph
(b) carboxylic acid group Me + H2SO4
(c) methylene group OH
(d) bicarbonate group Me
11. Which of the following is basic ? (1980, 1M) S. Ph 4. HCHO, NaOH
(a) CH3CH2OH Br
(b) H2O2 H
Ph + AgNO3
(c) HOCH2CH2OH OH
(d) CH3COOH Me

5. NaOBr
Matching Type Questions
12. The desired product X can be prepared by reacting the major The correct option is
product of the reactions in List-I with one or more (a) P → 1; Q → 2, 3; R → 1, 4; S → 2,4
appropriate reagents in List-II. (b) P → 1, 5; Q → 3,4; R → 4, 5; S → 3
(given, order of migratory aptitude : aryl > alkyl > hydrogen) (c) P → 1, 5; Q → 3,4; R → 5; S → 2,4
O (d) P → 1, 5; Q → 2, 3; R → 1,5; S → 2,3
Ph
Me
OH Subjective Questions
Ph
13. How will you bring about the following conversion?
X (2018 Adv.) “Ethanal to 2-hydroxy-3-butenoic acid.’’ (1990, 2M)

Topic 2 Acid Derivatives


Objective Questions I 2. The major product of the following reaction is
(2019 Main, 8 April I)
(Only one correct option) O Cl
1. The major product of the following reaction is
(2019 Main, 9 April II) (i) AlCl3, heat
O+
OH (ii) H2O

CH2OH H2SO4(cat.) O
CO2Et CHCl3 O
O O CO2H Cl
(a) (b)
(a) (b)
Cl
CO2Et COOH O O
OH OH
O Cl O
(c) (d)
OEt O (c) (d)
O O COOH Cl
O
Carboxylic Acids and Their Derivatives 399

3. The increasing order of the reactivity of the following with 8. Different possible thermal decomposition pathways for
LiAlH 4 is (2019 Main, 12 Jan II) peroxyesters are shown below. Match each pathway from
O O Column t I with an appropriate structure from Column II and
(A ) (B) select the correct answer using the code given below the lists.
(2014 Adv.)
C2H5 NH2 C2H5 OCH3 P
R· + R′O·
O O O –CO2

(C) (D) O Q
C2H5 Cl C2H5 O C2H5 O R· + R′O· R·+ X ·
–CO2
R O R′ + Carbonyl compound
(a) (A ) < (B ) < (D ) < (C )
(b) (A ) < (B ) < (C ) < (D ) (Peroxyester) R
RCO·2+ R′O· R·+ X′·
–CO2
(c) (B ) < (A ) < (D ) < (C ) + Carbonyl
compound
(d) (B ) < (A ) < (C ) < (D )
S
4. The major product obtained in the following reaction is RCO2·+ R′O· R· + R′O·
–CO2
(2019 Main, 10 Jan II)
CO2Et
NaOEt/∆
Column I Column II
O
P. Pathway P 1. O
(a) (b) C6H5CH2 O CH3
CO2Et
O
CO2Et
Q. Pathway Q 2. O
C6H5 O CH3
(c) (d)
CO2Et O
O CH3
CO2Et R. Pathway R 3. C6H5CH2 O
CH3
5. The decreasing order of ease of alkaline hydrolysis for the CH2C6H5
following esters is (2019 Main, 10 Jan I)
O
COOC2H5 Cl COOC2H5 O CH3
S. Pathway S 4. C6H5 O
CH3
I II C6H5

O2N COOC2H5 CH3O COOC2H5 Codes


P Q R S
III IV
(a) 1 3 4 2
(a) III > II > IV > I (b) III > II > I > IV (b) 2 4 3 1
(c) II > III > I > IV (d) IV > II > III > I (c) 4 1 2 3
6. The compounds A and B in the following reaction are, (d) 3 2 1 4
respectively 9. A compound with molecular mass 180 is acylated with
HCHO+HCl AgCN
CH 3 COCl to get a compound with molecular mass 390. The
A B number of amino groups present per molecule of the former
(2019 Main, 9 Jan I) compound is (2013 Main)
(a) A = Benzyl alcohol, B = Benzyl isocyanide (a) 2 (b) 5
(b) A = Benzyl alcohol, B = Benzyl cyanide (c) 4 (d) 6
(c) A = Benzyl chloride, B = Benzyl isocyanide 10. Which of the following reactants on reaction with
(d) A = Benzyl chloride, B = Benzyl cyanide conc. NaOH followed by acidification gives the following
7. The major product of following reaction is lactone as the only product? (2006, 5M)
(i) AIH( i - Bu) 2 O
R  C ≡≡ N → ?
(ii) H 2O (2019 Main, 9 Jan I)
(a) RCHO (b) RCONH2 O
(c) RCOOH (d) RCH2NH2
400 Carboxylic Acids and Their Derivatives

COOCH3 COOH 17. With reference to the scheme given, which of the given
(a) (b) statement(s) about T, U, V and W is/are correct? (2012)

COOH CHO O

CHO COOH O
(c) (d) H3C
T
CHO COOH LiAlH4 (excess)
CrO3/H+ (CH3CO)2O
11. Benzamide on treatment with POCl 3 gives (2004, S, 1M) V U W
(a) aniline (b) benzonitrile (a) T is soluble in hot aqueous NaOH
(c) chlorobenzene (d) benzyl amine (b) U is optically active
CH3MgBr (c) Molecular formula of W is C10H18 O4
12. Ethyl ester → P, the product ‘P’ will be (d) V gives effervescence on treatment with aqueous NaHCO3
(excess) (2003, S, 1M)
H3C CH3 H3C C2 H 5 18. Identify the binary mixture(s) that can be separated into
(a) (b) individual compounds, by differential extraction, as shown in
H3C OH H5C2 OH the given scheme. (2012)
NaOH (aq)
H5C2 C2 H 5 H5C2 C2H5
(c) (d) Binary mixture containing
Compound 1 and Compound 2 NaHCO3 (aq)
H5C2 OH H7C3 OH
Compound 1 + Compound 2
13. The product of acid hydrolysis of P and Q can be
distinguished by (2003, S, 1M) Compound 1 + Compound 2
OCOCH H3C (a) C6H5OH and C6H5COOH
3
P : H2C== ; Q: (b) C6H5COOH and C6H5CH2OH
CH3 (c) C6H5CH2OH and C6H5OH
OCOCH3 (d) C6H5CH2OH and C6H5CH2COOH
(a) Lucas reagent (b) 2, 4-DNP
(c) Fehling’s solution (d) NaHSO 3 19. Reaction of RCONH2 with a mixture of Br2 and KOH gives
14. Hydrogenation of benzoyl chloride in the presence of Pd on R  NH2 as the main product. The intermediates involved in
BaSO4 gives (1992, 1M)
this reaction are (1992, 1M)
(a) benzyl alcohol (b) benzaldehyde (a) RCONHBr (b) RNHBr
(c) benzoic acid (d) phenol (c) R  N == C== O (d) RCONBr2
15. Acetamide is treated separately with the following reagents.
Which one of these would give methyl amine ? (1983, 1M)
Assertion and Reason
(a) PCl 5 (b) NaOH + Br2 Read the following questions and answer as per the direction
(c) Sodalime (d) Hot conc. H2SO4 given below :
(a) Statement I is true; Statement II is true; Statement II is a correct
Objective Questions I explanation of Statement I.
(One or more than one correct options) (b) Statement I is true; Statement II is true; Statement II is not the
correct explanation of Statement I.
16. An organic compound [A], molecular formula C10H 20O2 was (c) Statement I is true; Statement II is false.
hydrolysed with dilute sulphuric acid to give a carboxylic (d) Statement I is false; Statement II is true.
acid [B] and an alcohol [C]. Oxidation of [C] with
CrO3 H 2SO4 produced [B]. Which of the following 20. Statement I p-hydroxybenzoic acid has a lower boiling point
structures are not possible for [A]? (2020 Main, 3 Sep I) than o-hydroxybenzoic acid.
CH 3 Statement II o-hydroxybenzoic acid has intramolecular
 hydrogen bonding. (2007, 3M)
(a) CH3 CH2  CH  OCOCH2 CH  CH2CH3
 21. Statement I Acetic acid does not undergo haloform reaction.
CH 3 Statement II Acetic acid has no alpha hydrogen. (1998, 2M)
CH 3
 Passage Based Questions
(b) CH3 CH2  CH  COOCH2 CH  CH2CH3
 Passage 1
CH 3
(c) CH3 CH2CH2COOCH2CH2CH2CH3 The reaction of compound P with CH3 MgBr (excess) in (C2 H5 )2 O
(d) (CH3 )3  C  COOCH2C(CH3 )3 followed by addition of H2 O gives Q. The compound Q on treatment
with H2 SO4 at 0° C gives R. The reaction of R with CH3 COCl in the
Carboxylic Acids and Their Derivatives 401

presence of anhydrous AlCl 3 in CH2 Cl 2 followed by treatment with CH2OH


H2O produces compound S . [Et in compound P is ethyl group] (b) and
(H3C)3C CH2OH
CO2Et Q R S (V) (W)
O
P

22. The product S is (2017 Adv.) (c) O and


COCH3 H 3C CH3
(a) (H3C)3C CH3 (b) (H3C)3C O (W)
(V)

HOH2C
COCH3
(d) and
HO3S H3COC
CH2OH CH2OH
(c) (H3C)3C O CH3 (d) (H3C)3C H 3C CH3 (V) (W)

Passage 3
RCONH2 is converted into RNH 2 by means of Hofmann’s
COCH3
bromamide degradation.
23. The reactions, Q to R and R to S , are O O
(a) Aromatic sulfonation and Friedel-Crafts acylation Cl— Cl
(b) Friedel-Crafts alkylation and Friedel-Crafts acylation NH2 NHBr
(c) Friedel-Crafts alkylation, dehydration and Friedel-Crafts (i) (ii)
acylation O
(d) Dehydration and Friedel-Crafts acylation
C O
Passage 2 Cl
N Cl
P and Q are isomers of dicarboxylic acid C4 H4 O4 . Both decolourize –
N—Br
Br2 / H2 O. On heating, P forms the cyclic anhydride. (iv) (iii)
Upon treatment with dilute alkaline KMnO4 . P as well as Q could
produce one or more than one form S, T and U. (2013 Adv.) – +
COOH COOH COOH OM
O
H OH H OH HO H N Cl NH2 Cl
H OH HO H H OH H
(v) (vi)
COOH COOH COOH
S T U In this reaction, RCONHBr is formed from which this
reaction has derived its name. Electron donating group at
24. Compounds formed from P and Q are, respectively
phenyl activates the reaction. Hofmann’s degradation
(a) Optically active S and optically active pair (T, U)
(b) Optically inactive S and optically inactive pair (T, U)
reaction is an intramolecular reaction. (2006, 3 × 4M = 12M)
(c) Optically active pair (T, U) and optically active S 26. How can the conversion of (i) to (ii) be brought about?
(d) Optically inactive pair (T, U) and optically inactive S (a) KBr (b) KBr + CH 3ONa
25. In the following reaction sequences V and W are respectively (c) KBr + KOH (d) Br 2 + KOH

H 2 / Ni 27. Which is the rate determining step in Hofmann’s


Q → V bromamide degradation?

(a) Formation of (i) (b) Formation of (ii)
AlCl3 (anhydrous) (i) Zn—Hg/HCl (c) Formation of (iii) (d) Formation of (iv)
+V W
(ii) H3PO4
28. What are the constituent amines formed when the mixture of
O (1) and (2) undergoes Hofmann’s bromamide degradation?

15
(a) O and CONH2 CONH2

D
O O (1) (2)
(V) (W)
402 Carboxylic Acids and Their Derivatives

15 compound (Z) is treated with HBr and subsequently


(a) NH2, NH2, NH2,
ozonolysed. Mark the C* carbon in the entire scheme.
* *
D D
15
Ba CO3 + H2 SO4 → X ( C = C14 gas )
NH2 (i) Mg/ether LiAlH
4
15
CH2 == CH  Br → Y → Z
(ii) X
(b) NH2, NH2 (ii) H3 O+ (2001 Main, 5M)

D 37. Write the structures of the products A and B.


15 O
(c) NH2, NH2  18 H3O+
CH3  C  O C2H5 → A+ B (2000 Main, 3M)

(d)
15 38. Explain briefly the formation of the products giving the
NHD, NH2
structures of the intermediates
O
O
Fill in the Blank OC2H5
COOH
C OC2H5 (i) OH–
29. Formic acid when heated with conc. H2 SO4 produces H2C
NaOEt Br

………………… (ii) H+
(1983, 1M) C OC2H5 COOH

True/False O
(1999, 5M)
30. The boiling point of propanoic acid is less than that of n-butyl 39. Write the structures of the products :
alcohol, an alcohol of comparable molecular weight. (CH3CO) 2 O, heat
(1991, 1M)
CH3 CH2 NH2 → (1998)

31. Hydrolysis of an ester in the presence of a dilute acid is 40. An ester A (C4H8O2), on treatment with excess methyl
known as saponification. (1983, 1M)
magnesium chloride followed by acidification, gives an
alcohol B as the sole organic product. Alcohol B, on oxidation
Integer Answer Type Question with NaOCl followed by acidification, gives acetic acid.
Deduce the structures of A and B. Show the reactions
32. The total number of carboxylic acid groups in the product P is involved. (1998)
O O 41. Complete the following, giving the structures of the principal
(i) H3O+ , ∆
organic products
O (ii) O3
P Me
(iii) H2O2 (i) + (COOEt)2 + EtONa A
O O (2013 Adv.) NO2
conc. H2SO 4
Subjective Questions (ii) (COOH)2 + (CH2OH)2 → → B
H O+
CH2Cl (iii) H3CCOCOC6H5 + NaOH →
3
C (1997, 2M)
33. 42. A hydrocarbon A of the formula C8 H10 , on ozonolysis gives
+
KCN C2H5ONa/EtOH H3O /∆ compound B (C4 H6 O2 ) only. The compound B can also be
(A) (B) (C)
DMF C6H5CHO Heat obtained from the alkyl bromide C (C3 H5 Br) upon treatment
SOCl2
(D) with magnesium in dry ether, followed by carbon dioxide and
CH3NH2 acidification. Identify A, B and C and also give equations for
Identify A to D. (2004, M) the reactions. (1996, 3M)
34. Compound A of molecular formula C9 H7 O2 Cl exists in keto 43. Complete the following sequence of reactions with
form and predominantly in enolic form B. On oxidation with appropriate structures
KMnO4 , A gives m-chlorobenzoic acid. Identify A and B. Red-P A
(2003 Main, 2M) CH3  CH2  COOH →
Br 2
35. (±) 2-phenylpropanoic acid on treatment with (+) 2-butanol
gives (A) and (B). Deduce their structures and also establish (i) Alc. KOH (excess)
A →
+
B
stereochemical relation between them. (2003) (ii) H (1995, 2M)
36. Identify X and Y in the following synthetic scheme and write 44. Which of the following carboxylic acids undergoes
their structures. Explain the formation of labelled decarboxylation easily ? Explain briefly. (1995, 2M)

formaldehyde (H2 CO) as one of the products when (i) C6H5COCH2COOH (ii) C6H5COCOOH
(iii) C6H5CH(OH)COOH (iv) C6H5CH(NH2) COOH
Carboxylic Acids and Their Derivatives 403

45. Predict the major product in the following reaction : 53. A liquid X, having a molecular formula C6 H12 O2 is hydrolysed
(i) CH MgBr (excess) with water in the presence of an acid to give a carboxylic acid
3
C6H5  CH2COCH3 → (1994, 1M) Y and an alcohol Z. Oxidation of Z with chromic acid gives Y.
(ii) H+ What are the structures of X, Y and Z ? (1986, 3M)
46. In the following reactions, identify the compounds A, B, C 54. An ester A (C4H8O2) on treatment with excess of methyl
and D. magnesium chloride followed by acidification, gives an alcohol
(i) PCl5 + SO2 → A + B B as the sole organic product. Alcohol B, on oxidation with
(ii) A + CH3COOH → C + SO2 + HCl NaOCl followed by acidification, gives acetic acid. Deduce
(iii) 2C + (CH3 )2 Cd → 2D + CdCl2 (1994, 1M × 4 = 4M) structures of A and B. Show the reactions involved. (1998)

47. Complete the following sequence of the reactions with 55. Complete the following with appropriate structures :
appropriate structures C H OH
(CH3CO)2 O →
2 5
CH3COOH + ? (1986, 1M)
Fuming 1. NaOH (Fuse)
(i) —SO3H + 56. Arrange the following in order of their increasing ease of
H2SO4 2. H
hydrolysis : (1986, 1M)
+ CH3COOC2H5 , CH3COCl, (CH3CO)2 O, CH3CONH2
P 2O 5 H , H 2O
(ii) —CONH2
∆ 57. Give reasons in one or two sentences for the following :
(1992, 1M)
“Formic acid is a stronger acid than acetic acid.” (1985, 1M)
48. In the following identify the compounds/reaction
conditions represented by the alphabets A, B, and C : 58. Write down the reactions involved in the preparation of the
following using the reagents indicated against in parenthesis.
PCl 5 NH3
C6 H5 COOH → A → “Propionic anhydride from propionaldehyde”
P O H /Ni
[AgNO3 , NH4 OH, P2 O5 ] (1984, 2M)
2 5 2
B → C6 H5 CN → C (1991, 2M)
59. Give reasons for the following in one or two sentences.
49. Arrange the following as stated: ‘‘Acetic acid can be halogenated in the presence of P and Cl 2 ,
“Increasing order of acidic strength.” but formic acid cannot be halogenated in the same way.’’
ClCH2COOH, CH3CH2COOH, ClCH2CH2COOH, Why? (1983, 1M)
(CH3 )2 CHCOOH, CH3COOH (1991, 1M)
60. State with balanced equation, what happens when, “Acetic
50. How will you bring about the following conversion? anhydride reacts with phenol in presence of a base.” (1982, 1M)
“Ethanoic acid to a mixture of methanoic acid and diphenyl 61. Write the structural formula of main organic product formed
ketone.” (1990, 2M) when ethyl acetate is treated with double the molar quantity of
51. Give reasons for : methyl magnesium bromide and the reaction mixture is poured
“Carbon-oxygen bond lengths in formic acid are 1.23 Å into water. (1981, 1/2 M)
and 1.36 Å and both the carbon-oxygen bonds in sodium 62. Write the chemical equation to show what happens when,
formate have the same value, i.e. 1.27Å.” (1988, 2M) “Ethyl acetate is treated with sodium ethoxide in ethanol and
52. Write balanced equations for the following reaction : the reaction mixture is acidified”. (1981, 2 M)
“Acetamide is reacted with bromine in the presence of
potassium hydroxide.” (1987, 1M)

Answers
Topic 1 9. (b) 10. (c) 11. (b) 12. (a)
1. (b) 2. (c) 3. (d) 4. (d) 13. (c) 14. (b) 15. (b) 16. (a,c)
5. (b) 6. (c) 7. (d) 8. (d) 17. (a,c,d) 18. (b,d) 19. (a,c) 20. (d)
9. (c) 10. (d) 11. (a) 12. (d) 21. (c) 22. (a) 23. (c) 24. (b)
Topic 2 25. (a) 26. (d) 27. (d) 28. (b)
1. (d) 2. (d) 3. (a) 4. (b) 29. (H2O and CO gas) 30. (False) 31. (False)
32. (2)
5. (b) 6. (c) 7. (a) 8. (a)
Hints & Solutions
Topic 1 Carboxylic Acids 5. It is a β-keto acid which undergo decarboxylation in very mild
condtion, i.e. on simple heating. This occur through a six
1. Acidic nature depends upon nature of electron withdrawing member cyclic transition state as
group and electronegativity. Electronegativity further depends
on % s character. Higher the s-character, greater will be the H OH
electronegativity and hence tendency to loose H increases thus O O ∆ Tautomerism
–CO2
R C CH2
acidic character also increases. C C
R CH2 O O
O
O H Six membered cyclic R C CH3
(I) H OH transitionstate of a
(II) β-keto acid
OH H
sp- hybridisation sp 2 -hybridisation NOTE
(50% s character) (30-33% s character)
(pKa = 4.3)
l
Ordinary carboxylic acid require soda-lime catalyst for
(pKa = 1.86)
decarboxylation.
O OH l
Final step of decarboxylation in the above shown mechanism
(III) Me O (IV) H3C involve tautomerism, therefore, for decarboxylation of β-keto acid
OH O by above mechanism, the acid must contain an α-H].
sp2 -hybridisation O O O
sp3 -hybridisation
(Resonance effect) (25% s-character) Heat * I2
(pKa = 4.5) (pKa = 4.8) Ph—C—CH3
6. NaOH
Ph * OH E
Hence, acidic order I > II > III > IV.
*
Ph—COONa + CHI
II is more acidic than III since electron donating group (OCH3) 3
F G
is attached to benzene ring in III which decreases the acidic
character. SO3H SO3Na
On the other hand, pK a value also determined acidic nature,
7. + NaHCO3
lower pK a value gives maximum acidic character.
Hence, option (b) is correct.
+ H2O + CO2↑
2. This problem is based on successive reduction, chlorination and
elimination reaction. To solve such problem, use the function of OH ONa
the given reagents. + NaHCO3
(i) LiAlH 4 causes reduction
(ii) PCl 5 causes chlorination O 2N O2N
(iii) Alc. KOH causes elimination reaction + H2O + CO2↑
LiAlH4
CH3COOH  → CH3CH2OH O O−
(A )  H 
− H+
PCl 5 Alc.KOH
8. R  C  OH + O R′ → R  C — OH
••
→ CH3CH2Cl → CH2 == CH2 
(B ) − HCl (C ) OR′
Ethylene
Reaction occur at planar sp2 carbon giving racemic mixture of
3. PLAN NaHCO3 s Na+ + HCO3− product.
HCO−3 is decomposed by acid releasing CO2
9. C6H5COOH + SOCl 2 → C6H5  COCl
HCO−3 + H+ → H2O + CO2 *
If acid is stronger than HCO−3 then CO2 is released. Phenol is less
10. CH3  CH2  COOH + NaHCO3 → CH3CH2COONa
*
acidic and thus does not liberate CO2 with NaHCO3. + H2O + CO2
O 11. Ethanol is the weakest acid among these, hence it is most basic.
 NH3 Ph
4. CH3CH2  C  OH → 12. (d)
(A ) HO Me
For P, i.e.
O Ph OH + H2SO4
∆ 
CH3CH2COONH4 → CH3  CH2  C  NH2 Me
(B ) (C ) The correct match is 1 i.e., I2, NaOH and 5 i.e., NaOBr
Br 2 , KOH The reactions proceed as
→ CH3  CH2  NH2
Hoffmann’ s bromamide reaction
Carboxylic Acids and Their Derivatives 405

Ph Me Ph O Ph
Pinacol
+ H2SO4 For S, i.e. Br H
HO OH Pinacolone Me Me + AgNO3
Ph Me rearrangement Ph Ph OH
(Methyl shift)
Me
Both
The correct match is 2, 3.

H
aO
haloform The reaction proceed as

/N
reactions
I2
Ph
Br H
NaOBr Ph OH + AgNO3
Ph O Me
dibromination
rearrangement
Me OH
Ph Ph
‘ X’
O
Ph H Me
Ph H
For Q, i.e. + HNO2
NH2 OH
Ph Me Rest procedure is same as seen for Q above i.e., via oxidation.
The correct match is 2 i.e. [Ag(NH3)2]OH and 3 i.e., Fehling’s OH
solution.  ∆
NaOH
13. CH3CHO + HCHO → H  C — CH2  CHO →
The reactions proceed as  Aldol
Ph H Ph O H
Deamination
+ HNO2 OH
rearrangement Me
H2N
Ph Me OH (Methyl shift) Ph
H
HCN 
CH2 == CH  CHO → CH2 == CH  CH  CN
n
tio Cyanohydrin
lu
so
g's OH OH
lin
Fe
h Both  H+ 
oxidation
CH2 == CH  CH  CN → CH2 == CH  CH  COOH
Reactions H2 O
[Ag(NH3)2]OH
(Tollens's
Ph O reagent) Topic 2 Acid Derivatives
Me 1. Given reaction involves acidic hydrolysis of esters followed by
OH
Ph the intramolecular cyclisation. The chemical equation is as follows:
‘X’
OH OH
Ph Ph
For R, i.e. HO + H2SO4 CH2OH H2SO4(cat.) CH2OH
OH +C2H5OH
Me Me CHCl3

COEt C OH
The correct match is 1, 5 again.
O O
The reaction proceed as OH OH

Ph Ph Ph O CH2 OH –H2O
Pinacol O
+ H2SO4 Pinacolone
HO OH Me Me C OH
Me Me rearrangement Ph O
(Phenyl shift) O
2. The major product of the given reaction is (d).
H Both
aO haloform This reaction proceed via Friedel-Craft acylation. Here,  Cl
/N
I2 reactions group present on chlorobenzene is ortho- and para-directing. It can
be easily understood by resonating structures of chlorobenzene.
NaOBr r r r
Ph O Cl Cl Cl Cl Cl
Me
OH s s
Ph
X
s
406 Carboxylic Acids and Their Derivatives

The given reaction proceed as follows : Note The −I effect of  Cl and + m effects of
O O
O + • • 
C  N H2 O  C C2H5 and  OCH3 groups while
O + AlCl3 –
O----Al + Cl3 attached to  C  group to be reduced. Also add to the

O
O O
Cl
group leaving ability.
4. In presence of strong base, acidic H can easily be removed that
result in formation of anion. The resulting anion undergoes
O intramolecular nucleophilic addition which on hydrolysis
O
followed by heating gives the required product.
C
H2O More acidic
Cl C—O – + Cl H H
COOH —AlC
l3 CO2Et O
s r
O EtONa
O –EtOH s CO2Et
3. All the given compounds are acid derivatives, thus contain O O
carbonyl group in them. LiAlH4 reduces these compounds
(Nucleophilic
through nucleophilic substitution via addition elimination s addition)
(S NA E) reaction. The rate of reaction depends upon the following EtO
O –H
cidic b O
factors : H
more a H2 O
(i) Size of alkyl group. CO2Et s CO2Et
–OH
(ii) Steric hinderance around the > C == O group. OH s
O
(iii) (+ ) ve charge on the C-atom of > C == O group.
O
The alkyl groups are more or less same in the given compounds. a, b-eliminatioin
Thus, the reactivity order of given compounds depends upon –EtOH CO2Et
2nd and 3rd factor written above. The cumulative effect of –OHs
these two factors results to leaving group ability (LGA ) of the
substituents in the following order : 5. Alkaline hydrolysis of an ester (carboxylic acid derivative)
O follows acyl S N2 mechanism.
 − O
Cl− > O−  C CH3 >O CH3 > NH2 O s
OH
O
C + ROH
Good leaving Poor leaving C C + ROs
group group OR OH Os
This leaving group ability (weak conjugate
base) corresponds directly to the reactivity order. Rate of S N 2 mechanism depends on the polarity of C O
group of COOR group. Electron withdrawing group
Hence, the correct reactivity order is:
(− R > − I ) increases the rate of S N2 reaction whereas electron
O O
O donating group (+ R > + I ) decreases the rate of S N 2 reaction.
 Here, the nature of functional groups attached para to the
C2H5  C  Cl > C2H5 O C2H5 benzene ring are:
Most reactive
(D) — NO2 > —Cl > — OCH3
(C)
(− R ) (− I ) (+ R )
O O
  So, the order of hydrolysis will be,
> C2H5  C  OCH3 > CH3 C NH2
Least reactive III > II > I > IV
(B) (− R ) (− I ) (+ R )
(A)

6. The mechanism of the given reaction is as follows:


SN2
Å y
it is electrophilic substitution CH2 OH CH2Cl CH2CN CH2N—C
[CH2==OH]Cl , (Ar SE2): Haloformylation –H2O Ag CN
+
–HCl HCl (SN1) More covalent
[CH2==O + H Cl] (Ag—CººN )
(A) –AgCl (Minor) B (Major)
(Benzyl chloride) (Benzyl isocyanide)

Thus, both benzyl cyanide and benzyl isocyanide are the products of reaction but benzyl isocyanide being the major product gives the correct
option as (c).
Carboxylic Acids and Their Derivatives 407

7. Key Idea DIBAL-H is diisobutyl aluminium hydride, [(CH3 )2 CHCH2 ]2 AlH. It is a selective reducing agent. It reduces carboxylic
acids, carboxylic acid derivatives and nitriles into aldehydes. It is an electrophilic reducing agent.

The mechanism of the reaction is as follows:


H
(sp2) s
H2 O
Al (DIBAL-H) Al Al
(An electrophilie) R CH O
–NH3
[Aldehyde]
r –DIBAL-H
H N
R C N
R C N Hydride donor
(Reducing agent) CH (An imine)
(A nucleophile)
[Nitrile]
R
So, R  C H is the correct answer.

O
8. PLAN This problem can be solved by using the stability of radical obtained after fragmentation of peroxyester.
Allylic radical are more stable than alkyl radical, so when there is a possibility of formation of allyl radical, it will undergo fragmentation
through formation of allyl radical. i.e. fragmentation produces stable radical.
On the basis of stability of radical, fragmentation can be done as

Column I Column II Explanation


O
O
P. C6H5H2C CH3 1. C6H5 — CH2+ CO2+ CH3O
O
O
O CH3
O CH3 3. C6H5 — CH2+ CO2+ Ph—CH2 — C — CH3 Ph —CH2+ CH3—CO—CH3
Q. C6H5H2C O
CH2C6H5 CH3

O O
CH3
O CH3 Ph + CH3—CO—Ph + CH3 + CO2
R. C6H5 4. C6H5 — CO2+ CH3—C—CH3 –CO2
O
C6H5 C6H5
O
2. C6H5 —CO2+ CH3O
O
S. C6H5 O CH3 C6H5+ CO2

O O
 
9. R  NH2 + CH3  C  Cl → R  NH  C  CH3
(− HCl)

O

Since, each COCH3 group displace one H atom in the reaction of one mole of CH3  C  Cl with one NH2 group, the molecular mass
increases with 42 unit.
210
Since the mass increases by (390 − 180) = 210, hence the number of NH2 group is = 5.
42
O
CHO COO –
OH – H+
10. O
CHO CH2OH
O
 POCl 3
11. C6H5  C  NH2 → C6H5  CN
408 Carboxylic Acids and Their Derivatives

O OMgBr Note It is not having the molecular formula of A(C10H20O2 ). Its molecular
 CH3 MgBr
 formula (C8H16O2 ).
12. CH3  C  OC2H5 → CH3  C — CH3
excess O

CH3 (d) (CH3)3C C O CH2 C (CH3)3
CH3 5C (R+C) 5C (R′)
H2 O 
→ CH3  C — OH H3Or
(d) (CH3)3C COOH + HO CH2 C (CH3)3

CH3 (B) [O] (C)

OH O 17. (a) T undergoes an ester hydrolysis in hot aqueous alkali as


H+ 
13. P → CH3  C == CH2 r CH3  C  CH3 O
H2 O CH3O
NaOH O NaOH(aq) C ONa
→ CHI3 ↓ hot HO
I2 Yellow H 3C
T Soluble in aqueous NaOH
H+
Q → CH3  CH== CH  OH r CH3 CH2 OH (b) LiAlH4 reduces ester to alcohol as
H 2O
“U ” No chiral carbon optically inactive.
Fehling solution
→ Cu 2O ↓ (c) U on treatment with excess of acetic anhydride forms a
Red diester as
COCl CHO (d) U on treatment with CrO3 |H+ undergo oxidation to diacid
H2 which gives effervescence with NaHCO3.
14. Pd/BaSO4 ‘‘Rosenmund
O
reduction’’
H+ OH NaHCO3
O U + CrO3 CO2
COOH
 H 3C
15. CH3  C  NH2 + Br2 + NaOH → CH3NH2 “Hofmann’s ‘V ’
bromamide reaction”.
18. For separation by differential extraction one of the component
16. A can be written as : (CH3 )3C COOCH2C(CH3 )3 ] must form salt with the given base so that the salt will be
H2O
extracted in aqueous layer leaving other component in organic
C10H20O2 Carboxylic acid + Alcohol layer.
H2SO4
(B ) (C ) (a) Both phenol and benzoic acid forms salt with NaOH, hence
(A )
[O]
this mixture can’t be separated.
CrO3/H2SO4
(b) Benzoic acid forms salt with NaOH while benzyl alcohol
From the informations given, it is clear that A is an ester which does not, hence the mixture can be separated using NaOH.
contain 10 carbon atoms. Again, C on oxidation gives B. It Also benzoic acid forms salt with NaHCO3 but benzyl
indicates B and C contains equal number of carbon atom, i.e. alcohol does not, hence NaHCO3 can be used for separation.
each of B and C contains 5 carbon atom. (c) Neither benzyl alcohol nor phenol forms salt with NaHCO3,
O mixture cannot be separated using NaHCO3.
H2O
R C OR′ R COOH + R′ OH (d) C6H5CH2COOH forms salt with NaOH, C6H5CH2OH does
+
H not, hence mixture can be separated using NaOH.
10C 5C (R+C) 5C (R′)
C6H5CH2COOH forms salt with NaHCO3. C6H5CH2OH does
CH3 O CH3 not, hence mixture can be separated using NaHCO3.
O O
(a) CH3CH2 CH O C CH2 CH CH2CH3
 

4C(R′) 6C(R+C)
19. R  C  NH2 + OH− + Br2 → R  C  NHBr
CH3 O CH3 (A )

CH3CH2 CH C O CH2 CH CH2CH3


O O
(b)  −
 −
5C (R+C) 5C (R′) R  C  NHBr + OH → R  C  N  Br
O (B )
→ R  N == C == O + Br −
(c) CH3CH2CH2 C O CH2 CH2CH2CH3 (C )
4C (R+C) 4C (R′)
Carboxylic Acids and Their Derivatives 409

20. p-hydroxy benzoic acid has higher boiling point than o-hydroxy CHCOOH Br2 Water Br—CHCOOH CHCO

benzoic acid because former prefers intermolecular H-bonding O
while the latter prefer intramolecular H-bonding. CHCOOH Br—CHCOOH CHCO
H O
HOOC— —O C— —OH COOH COOH
H—O
intermolecular H-bonding H—C H 2O + O H—C—OH
OH
H—C dilute alkaline H—C—OH
KMnO4
C
O COOH COOH
(P) (S)
H
O (maleic acid) Optically inactive due to
intramolecular H-bonding internal compensation of
rotation (meso-isomer)
O

21. Compounds with CH3  C  or CH3  CH(OH)–group gives COOH COOH COOH
haloform reaction but this reaction is given only by aldehydes,
ketones and alcohols, so acetic acid does not give haloform H—C H 2O + O H—C—OH OH—C—H
+
reaction. However acetic acid has three α-H, therefore, statement
C—H dilute alkaline HO—C—H H—C—OH
I is true but statement II is false. KMnO4
Passage-1 COOH COOH COOH
(fumaric acid) (T) d (+) (U) l (–)
OH
(I) CH3MgBr(ex)
T and U (in 1 : 1 molar ratio) form optically inactive (racemic
OEt+ (ii) H2O
mixture) due to external compensation.
Q

O H+/0°C 25. PLAN Ni / H2 reduces( C == C ) bond.


Benzene undergoes Friedel-Crafts reaction Zn-Hg/HCl reduces
CH3COCl carbonyl group (Clemmensen reduction)
AlCl3
O
R(F.C. Alkylation)
CHCOOH CH2COOH CH2C
O Ni/H2 ∆ O
S(F.C. Acylation) HOOCHC CH2COOH CH2C

22. (a) 23. (c) O


succinic anhydride (V)
24. PLAN Alkenes decolouriseBr2 water
dil⋅KMnO 4
O
cis-isomer → Meso isomers by syn addition O
dil⋅KMnO 4 anhydrous C
trans-isomer → d( + ) and l( − ) isomers by syn CH2—C AlCl3 CH2
addition thus, racemic mixture. + O
Formation of anhydride from dicarboxylic acid indicates CH2—C CH2COOH
cis-isomer. O

P and Q are isomers of dicarboxylic acids.
Zn–Hg/HCl H3PO4
Br water
P , Q →
2
decolourised Clemmensen
HO
P and Q have (C == C) bond
O
∆ anhydride O
P → (W)
Thus, P is cis-isomer.
410 Carboxylic Acids and Their Derivatives

O CH2Cl CH2CN
26. Cl— —C—NH2 + KOH + Br2
KCN C2H5ONa/EtOH
O 33.
(i) DMF C6H5CHO
Cl— —C—NHBr (A)

OH
an intermediate (ii) C6H5 C6H5
H3O+/∆
27. Rearrangement of (iii) to (iv) is the rate determining step : C6H5—CH—CH—C6H5 C==C
O HOOC H
Slow Cl— CN (C )
Cl— —C – —N==C==O + Br– (B)
N—Br C6H5 C6H5
(iv) SOCl2
(iii) C==C
CH3NH2
28. The rate determining step of Hofmann’s bromamide reaction is CH3HN—C H
unimolecular rearrangement of bromamide anion (iii) and no (D)
cross-products are formed when mixture of amides are taken. O
15
KOH 34. Compound A of molecular formula C9 H7 O2Cl exist in keto and
—CONH2 + —CONH2
Br2
predominantly in enolic form B. Hence, A must be a carbonyl
compound which contain α-H. Enolic forms of B predominates
D (i) 15
because of presence of intramolecular H-bonding.
—NH2 + —NH2
Cl Cl Cl
[O] tautomerism
D

29. HCOOH + H2SO4 → H2O + CO ↑
conc. COOH O
O
30. Propanoic acid has higher boiling point than n-butanol because
of more exhaustive H-bonding in former case. CHO H H
O
31. Saponification is hydrolysis of ester in presence of dilute base
rather in presence of dilute acid. 35. The two stereoisomers of 2-phenyl propanoic acid in the racemic
mixture are :
32. PLAN Reactant is cyclic anhydride and changes to dicarboxylic acid on *
hydrolysis. COOH COO CH(CH3 )Et
Also there is decarboxylation on heating if there is keto group  *
CH3 C H(OH)Et 
w.r.t COOH group. Ozonolysis cleaves (C== C) bond and H  C — Ph → H  C — Ph
H2O2 oxidises CHO to COOH group.  
CH3 CH3
(A )
*
COOH COO CH(CH3 )Et
 *
CH3 C H(OH)Et

and Ph  C — H → Ph  C — H
 
Anhydride CH3 CH3
(B )
decarbo- Here A and B are diastereomers.
xylation
O

(i) Mg/ether
36. H2C == CH  Br → H2C == CH  C  OH
(ii) X *
(iii) H3O+ Y

LiAlH4 *
→ H2C == CH  CH2OH
Z
O O
 18 H3O+  18
37. CH3  C  OC2H5 → CH3  C  OH + C2H5  OH
(A ) (B )
Thus, number of COOH groups in P = 2.
Carboxylic Acids and Their Derivatives 411

O red-P
O 43. CH3CH2COOH → CH3  CH  COOH
Br 2 
C OC2H5 COOC2H5 OC2H5 A Br
NaOEt – (HVZ reaction)
38. CH2 CH
Br
C OC2H5 COOC2H5 (i) alc.KOH
→ CH2 == CH  COOH
(ii) H+ B
O
44. A β-keto acid undergo very fast decarboxylation
(i) OH– COOH
O CH3 COOC2H5 OH O
(ii) H+ H
O O – CO2
H5C2O—C—CH—CH—COOC2H5 C6H5—C==CH2 C6H5—C==CH3
COOH C enol keto
O
O C6H5
(CH3 CO) 2 O 
39. CH3CH2NH2 → CH3CH2NH  C  CH3 O OH
Heat
 (i) CH3 MgBr 
O CH3 45. C6H5CH2  C  CH3 → C6H 5  CH 2  C — CH3
(ii) H+ 
  MeMgBr /H2 O
40. H  C  O  CH — CH3 → CH3  CH  OH CH3
A excess 
CH3 46. (i) PCl 5 + SO2 → POCl 3 + SOCl 2
B B A

NaOCl
(ii) SOCl 2 + CH3COOH → CH3COCl + SO2 + HCl
→ CH3COOH + CHCl 3 (A )
O
– 
CH3 CH2 (iii) 2CH3COCl + Cd(CH3 )2 → 2CH3  C  CH3 + CdCl 2
EtO – (COOEt)2
41. (i)
SO3H SO3H OH
NO2 NO2

O fuming NaOH
47. (i)
H2SO4 fusion
CH2—C—COOEt SO3H OH
CONH2 CN COOH
NO2
A P2O5 H+
(ii)
HO H2O
O O
COOH CH2 H+
(ii) + O
COOH CH2 
O O PCl 5 NH3
HO 48. C6H5COOH → C6H5COCl → C6H5  C  NH2
B
A B
O O CH3
 P2 O 5 H2 /Ni
  NaOH → C6H5  CN → C6H5CH2NH2
(iii) CH3  C  C  C6H5 → C6H5  C  COOH C
H2 O 
OH 49. Electron withdrawing inductive effect increases acid strength
C while electron donating inductive effect decreases acid strength.
42. B is C3H5COOH and A is C3H5  C ≡≡ C  C3H5 + I-effect
Also A on ozonolysis gives B indicates that there is no olefinic H3C  CH  COOH < CH3CH2COOH < CH3COOH
bond in C3H5-unit of A and it is cyclopropyl group. 
CH3
⇒ A= —C==C— + I-effect + I-effect
+ I-effect
B= —COOH < C H2  CH2COOH < C H2  COOH
 
and C= —Br Cl Cl
− I-effect − I-effect
(greater distance)
412 Carboxylic Acids and Their Derivatives

O O
PCl 5  C6 H6 C2H5 OH 
50. CH3  COOH → CH3COCl → C6H5  C  CH3 55. (CH3CO)2 O → CH3COOH + CH3  C  OC2H5
AlCl 3

OH C6H5 56. Among acid derivatives, the reactivity towards nucleophilic acyl
  substitution is in the order of :
C6 H5 MgBr H +
→ C6H5  C — CH3 → C6H5  C==CH2 Amide < Ester < Anhydride < Acid chloride
H2 O ∆

C6H 5 Hydrolysis is an example of nucleophilic acyl substitution,
hence the reactivity towards hydrolysis is :
O CH3CONH2 < CH3COOC2H5 < (CH3CO)2 O < CH3COCl
O3 /H2 O2 
→ C6H5  C  C6H5 + H  COOH O O
 
51. Both formic acid and sodium formate exhibit the phenomenon of 57. CH3  C  OH < H  C OH
resonance as : +I -effect does not affect acidity.

O O decreases acid-strength
 • •  +
H  C  O  H ←→ H  C == O  H NH4 OH
•• •• 58. CH3CH2CHO + AgNO3 → CH3CH2COOH
O O− PO
  →
2 5
(CH3CH2CO)2 O

H  C  O− ←→ H  C == O
Red-P
In formic acid, the extent of delocalisation is less compared to 59. CH3  COOH + Cl 2 → CH 2  COOH (HVZ reaction)
sodium formate because of charge separation in the former case. 
Due to this reason, the bond length between carbon and sp3 Cl
For this reaction to occur, presence of a α-H is essential
oxygen in formic acid is slightly greater than the same between requirement. Formic acid does not has any α-H, fails in HVZ
carbon and sp2 oxygen. In formate ion, there is no separation of reaction.
charge and both the resonance structures are equivalent giving
OH O CH3
equal bond length of both carbon oxygen bonds. OH–
60. + (CH3CO)2O
NaOH
52. CH3CONH2 + Br2 → CH3NH2 + Na 2CO3 O
Hoffman bromamide
reaction O OMgBr
 
53. X is an ester and both its acid and alcohol fragments have same 61. CH3  C  OC2H5 + 2CH3MgBr → CH3  C  CH3
number of carbons. Hence, X is : 
+
CH3
H
CH3CH2COOCH2CH2CH3 → CH3CH2COOH OH
H2 O
X Y H2 O 
+ CH3CH2CH2OH → CH3  C — CH3
Z 
[O] CH3
Z → CH3CH2COOH
Y C H O− −
O CH3 62. CH3COOC2H5 →
2 5
CH2COOC2H5
  MeMgCl H+ /H2 O
54. H  C  O  CH — CH3 → → O
O
excess 
A 
CH3  C  OC2 H2
OH → CH3  C  CH2  COO2H5
 Claisen condensation
CH3  C — CH3 O
 + 
H H
→ C2H5OH + CH3  C  CH2COOH
B H2 O
B + NaOCl → CHCl 3 + CH3COOH
27
Aliphatic Compounds
Containing Nitrogen
Objective Questions I (Only one correct option) 5. The major products A and B for the following reactions are,
1. The final major product of the following reaction is respectively (2019 Main, 9 April II)
(2020 Main, 5 Sep II) O
Me I H2/Pd
(i) Ac2O/Pyridine KCN
[A ] [B ]
DMSO
(ii) Br2, FeCl3
(iii) OH–/∆
O OH
NH2
CN CH2NH2
Me Me Br
(a) ;
(a) (b)
Br HO CN HO CH2 NH2
NH2 NH2 I H
Br (b) ;

Me Me
O O
(c) (d) CN CH2NH2
Br
(c) ;
NH2 NH2
HO CN HO CH2 NH2
2. The major product of the following reaction is
I I
OH (d) ;
 Ethyl formate (1 equiv.)
CH3 CH CH2CH2NH2 →
Triethylamine
(2019 Main, 10 April I) 6. Which of the following amines can be prepared by Gabriel
OH phthalimide reaction? (2019 Main, 8 April I)

(a) CH3 CHCH2CH2NHCHO (b) CH3CH== CH  CH2NH2 (a) n - butylamine (b) triethylamine
(c) t-butylamine (d) neo -pentylamine
O
OH 7. In the following compounds, the decreasing order of basic
 strength will be (2019 Main, 8 April I)
(c) CH3  CH  CH == CH2 (d) O H
(a) C2H5NH2 > NH3 > (C2H5 )2 NH
CH3CHCH2CH2NH2 (b) (C2H5 )2 NH > NH3 > C2H5NH2
3. Ethylamine (C2H5 NH2) can be obtained from (c) (C2H5 )2 NH > C2H5NH2 > NH3
(d) NH3 > C2H5NH2 > (C2H5 )2 NH
N-ethylphthalimide on treatment with (2019 Main, 10 April I)
(a) NaBH4 (b) NH2NH2 8. The major product of the following reaction is
(c) H2O (d) CaH2 (2019 Main, 12 Jan II)
H3C O (i) NaNO2/H+
4. Hinsberg’s reagent is (2019 Main, 9 April II) NH2
(a) SOCl 2 (b) C6H5COCl O (ii) CrO3/H+
(c) C6H5SO2Cl (d) (COCl)2 (iii) H2SO4 (conc.), ∆
414 Aliphatic Compounds Containing Nitrogen

O O
12. The major product formed in the reaction given below will be
(a) CH3 O (b) HO NH2
NaNO2
Aq. HCl, 0-5°C
O
HO (2019 Main, 10 Jan I)
CH3 O
(c) (d)
O O (a) NO2 (b) OH
O

9. A compound ‘X’ on treatment with Br2 / NaOH, provided NO2 OH


C3 H9N, which gives positive carbylamine test. Compound ‘X’ is (c) (d)
(2019 Main, 11 Jan II)
(a) CH3COCH2NHCH3 (b) CH3CH2CH2CONH2
(c) CH3CON(CH3 )2 (d) CH3CH2COCH2NH2 13. The major product obtained in the following reaction is
10. The major product of the following reaction is OH
(2019 Main, 10 Jan II)
(CH3CO)2O/pyridine (1 eqv.)
O
NH2 room temperature
CH3N NaBH4
(2019 Main, 9 Jan II)
OH COCH3
(a) CH3N OH OH

(a) (b)
(b) CH3NH NH2 NHCOCH3
OH OCOCH3
(c) CH3NH
OH (c) (d)
NHCOCH3 NH2
(d) CH3N
14. Major product of the following reaction is
11. The correct structure of product ‘P’ in the following reaction is Cl
NEt 3 Cl NH2
Asn − Ser + (CH3CO)2 O → P + H 2N (i) Et3N
(Excess) (2019 Main, 10 Jan I)
O O (ii) Free radical
polymerisation
O
(2019 Main, 9 Jan I)
O NH2 O
H Cl O Cl O
N (a) n (b)
N NH2
(a) H3C OH
H O N n
O H O
OCOCH3
HN NH2
OCOCH3
O O
Ol Cl Cl
N (c) (d)
(b) H3C N H OH
H NHCOCH3
n n
O
HN O
O NH2 NH2
N
OCOCH3 O H
O O O
H
N
(c) H3C N OH 15. The increasing order of basicity of the following compounds is
H NH2
O NH2 NH
I. II.
O
O NH2
III. IV. NHCH3
O NHCOCH3
NH (2018 Main)
O
(d) H3C N NH (a) (I) < (II) < (III) < (IV) (b) (II) < (I) < (III) < (IV)
H OH (c) (II) < (I) < (IV) < (III) (d) (IV) < (II) < (I) < (III)
O OCOCH3
Aliphatic Compounds Containing Nitrogen 415

16. The order of basicity among the following compounds is 22. CH3NH2 + CHCl3 + KOH → Nitrogen containing compound +
(2017 Adv.) KCl + H2O. Nitrogen containing compound is (2006)
O NH2 (a) CH3CN (b) CH3NHCH3
− + + −
(c) CH3  N ≡≡ C (d) CH3  N ≡≡ C
H3C NH2 N NH HN N H2 N NH
23. Benzamide on treatment with POCl3 gives (2004)
(I) (II) (III) (IV) (a) aniline (b) benzonitrile
(a) II > I > IV > III (b) I > IV > III > I (c) chlorobenzene (d) benzyl amine
(c) IV > I > II > I (d) IV > I > II > III
24. The correct order of basicities of the following compounds is
17. In the Hofmann-bromamide degradation reaction, the number of NH
moles of NaOH and Br2 used per mole of amine produced are H3C  C , CH3  CH2  NH2 ,
(2016 Main) NH2 2
(a) four moles of NaOH and two moles of Br2 1
(b) two moles of NaOH and two moles of Br2 O
(c) four moles of NaOH and one mole of Br2 
(d) one mole of NaOH and one mole of Br2 (CH3 )2 NH, H3C  C  NH2
4 (2001, 1M)
18. Considering the basic strength of amines in aqueous solution, 3
(a) 2 > 1 > 3 > 4 (b) 1 > 3 > 2 > 4
which one has the smallest pK b value? (2014 Main)
(a) (CH3 )2 NH (b) CH3NH2 (c) (CH3 )3 N (d) C6H5NH2 (c) 3 > 1 > 2 > 4 (d) 1 > 2 > 3 > 4

19. On heating an aliphatic primary amine with chloroform and 25. A positive carbylamine test is given by (1999, 2M)
ethanolic potassium hydroxide, the organic compound formed is (a) N, N-dimethylaniline (b) 2, 4-dimethylaniline
(a) an alkanol (b) an alkanediol (2014 Main) (c) N-methyl-o-methylaniline (d) p-methylbenzylamine
(c) an alkyl cyanide (d) an alkyl isocyanide 26. p-chloroaniline and anilinium hydrochloride can be
20. The gas leaked from a storage tank of the Union Carbide plant in distinguished by (1998, 2M)
Bhopal gas tragedy was (2013 Main) (a) Sandmeyer reaction (b) NaHCO3
(a) methyl isocyanate (b) methylamine (c) AgNO3 (d) Carbylamine test
(c) ammonia (d) phosgene
27. Carbylamine test is performed in alc. KOH by heating a
21. The major product of the following reaction is (2011) mixture of (1984, 1M)
O (a) chloroform and silver powder
C (b) trihalogenated methane and a primary amine
(i) KOH
NH (c) an alkyl halide and a primary amine
(ii) Br— —CH2Cl (d) an alkyl cyanide and a primary amine
C
28. Acetamide is treated separately with the following reagents.
O
Which of these would give methylamine? (1983, 1M)
O
(a) PCl 5 (b) Sodalime
C
(c) NaOH + Br2 (d) Hot, conc. H2SO4
(a) N—CH2— —Br
C 29. The compound which on reaction with aqueous nitrous acid at
low temperature produces an oily nitrosamine, is
O (a) methylamine (b) ethylamine (1981, 1M)
O
(c) diethylamine (d) triethylamine
C
(b) N— —CH2Cl Objective Question II
C (Only one more than one correct option)
O
30. The major product of the reaction is (2015, Adv.)
O
H 3C CO2H
NaNO2, aq. HCl
C
0°C
(c) N CH3 NH2
C H3C NH2 H3C CO2H
O —CH2— —Br (a) (b)
O CH3 OH CH3 OH
C
(d) N H 3C CO2H H3C NH2
(c) (d)
C
O— —CH2Cl CH3 OH CH3 OH
416 Aliphatic Compounds Containing Nitrogen

31. A positive carbylamine test is given by (1999, 2M) After evolution of the gas was complete, the aqueous solution
(a) N, N-dimethyl aniline (b) 2, 4-dimethyl aniline was distilled to give an organic liquid which did not contain
(c) N-methyl-o-methyl aniline (d) p-methyl benzyl amine nitrogen and which on warming with alkali and iodine gave a
yellow precipitate. Identify the original substance assuming that
it contains one N-atom per molecule. (1993, 4M)
Fill in the Blank
39. Arrange the following in increasing order of basic strength:
32. (CH3OH+2 ) is …… acidic than (CH3NH3+ ). (1997 C, 1M) methylamine, dimethylamine, aniline, N-methylaniline.
(1988, 1M)
Match the Column 40. Give a chemical test and the reagent used to distinguish between
33. Match each of the compounds in Column I with its characteristic the following : “Ethylamine and diethylamine”. (1988, 1M)
reaction(s) in Column II. (2016, Adv.) 41. For nitromethane molecule, write structures
(i) showing significant resonance stabilisation
Column I Column II
(ii) indicating tautomerism (1986, 1M + 1M = 2M)
(A) CH3CH2CH2CN (p) Reduction with Pd - C/ H2
42. State the equation for the preparation of following compounds :
(B) CH3CH2OCOCH3 (q) Reduction with SnCl 2 / HCl n-propyl amine from ethyl chloride. (1982, 2 × 1M = 2M)

(C) CH3CH == CHCH2OH (r) Development of foul smell on


treatment with chloroform and Passage Based Questions
alcoholic KOH. Treatment of compound O with KMnO4 / H+ gave P, which on
(D) CH3CH 2 CH2CH2NH2 (s) Reduction with heating with ammonia gave Q. The compound Q on treatment with
diisobutylaluminium hydride Br2 / NaOH produced R. On strong heating, Q gave S, which on
(DIBAL-H) further treatment with ethyl 2-bromopropanoate in the presence of
(t) Alkaline hydrolysis KOH followed by acidification, gave a compound T. (2016 Adv.)

Subjective Questions
NaNO2 , HCl
34. C5H13N → Y(tertiary alcohol + other products)
– N2
optically active
X
Find X and Y. Is Y optically active ? Write the intermediate steps (O )
(2005, 4M) 43. The compound R is
35. Give reasons for the following in one or two sentences. O
Dimethylamine is a stronger base than trimethylamine. NH2
(1998, 2M) Br
(a) (b)
36. Following reaction gives two products. Write the structures of Br
NH2
the products.
O
(CH CO) O, heat
CH3CH2NH2 →
3 2
(1998, 2M) O O
37. Give the structure of A. ‘A (C3H9 N) reacts with benzenesulphonyl NHBr
chloride to give a solid, insoluble in alkali’. (1993, 1M) (c) (d) NBr
NHBr
38. A basic volatile nitrogen compound gave a foul smelling gas
when treated with chloroform and alcoholic KOH. A 0.295 g O O
sample of the substance dissolved in aq. HCl and treated with
NaNO2 solution at 0°C, liberated a colourless, odourless gas 44. The compound T is
whose volume corresponds to 112 mL at STP. (a) glycine (b) alanine (c) valine (d) serine

Answers
1. (d) 2. (a) 3. (b) 4. (c) 5. (c) 6. (a) 7. (c) 8. (c)
9. (b) 10. (c) 11. (a) 12. (*) 13. (b) 14. (d) 15. (c) 16. (d)
17. (c) 18. (a) 19. (d) 20. (a) 21. (a) 22. (d) 23. (b) 24. (b)
25. (d) 26. (c) 27. (b) 28. (c) 29. (c) 30. (c) 31. (b,d) 32. (more)
33. A → p, q, s, t; B → p, s, t; C → p; D → r 43. (a) 44. (b)
Hints & Solutions
1. CH3 Acetylation
CH3 This reaction is the second step of Gabriel phthalimide synthesis
for the preparation of aliphatic 1°-amines and amino acid. In this
CH3—C—O—C—CH3
step concentrated alkali can also be used in place of hydrazine.
NH2 O H—N—C—OCH3 4. Hinsberg’s reagent is C6H5SO2Cl (benzene sulphonyl chloride).
Bromination This reagent is used to distinguish between primary, secondary
m-toluidine
Br2/FeCl3
and tertiary amines.
Br Br 5. O O
CH3 CH3 I CN
Step I
+HOH KCN
CH3COOH DMSO

NH2 [A]
H—N—C—OCH3
Step II H2/Pd

O
The basic nature of m-toluidine is not greatly affected by the CH2NH2
CH3 group.
 NH  C  OCH3 is moderately activating group, its

O [B]

activating nature overcome weak activating CH3 group. Step I involves the nucleophilic substitution reaction in which I
Due to bulky nature of  NHC CH3 attack happen para to (Iodine) is substituted by
  CN group. In step II, H2/Pd reagent is used for reduction
O process. Here,  CN group reduces itself to CH2NH2.
NHCO–CH 3.
Here, NH 2 group has been protected by forming its aceyl
6. n-butylamine (CH3CH2CH2CH2NH2 ) can be prepared by
derivative and as such if does not interfere in the bromination. Gabriel phthalimide reaction. This method produces only
primary amines without the traces of secondary or tertiary
2. The mechanism of the given reaction is as follows: amines. In this method, phthalimide is treated with ethanolic
O KOH, it forms potassium salt of phthalimide which on heating
OH
H—C—OEt
with alkyl halide followed by alkaline hydrolysis forms
CH3—CH—CH2CH2—NH2 corresponding primary amines.
Et3N
[–NH2 is stronger —EtOs CO CO
nucleophile than
—OH] OH NH+ Alc. KOH –H O N–K+
2
r CO CO
CH3—CH—CH2CH2—NH—CH=
=O
nBu—X

H
COO–Na+ CO
OH EtOs NaOH(aq.)
+ n Bu—NH2 N—R

COO Na + CO
CH3—CH—CH2CH2—NH—CHO 1° amine
N-alkyl
The basic mechanism of the reaction is acyl S N 2 because the phthalimide
nucleophile, CH3 (OH)CH2CH2 NH2 attacks the sp2 carbon of
Triethylamine, t-butylamine and neo-pentylamine cannot be
the ester (H  CO2Et) and gets substituted. prepared by Gabriel phthalimide reaction.
3. The reaction for the production of ethylamine from 7. Basic strength can be compared by the reaction of an alkanamine
N -ethylphthalimide can be takes place as follows: and ammonia with proton.
O H HN H H
| |
R—N + H+ R—N+—H
N—C2H5 Hydrazinolysis | |
NH2—NH2 (Hydrazine) H H

O H H
O H HN | |
(N-ethylphthalimide) H—N + H+ H—N+—H
NH | |
| + C2H5—NH2 H H
NH
Ethylamine
(An aliphatic Basicity of an amine in aqueous solution depends upon the
O 1°-amine) solubility of ammonium cation formed by accepting proton from
418 Aliphatic Compounds Containing Nitrogen

water. The stability of ammonium cation depends upon the 9. The molecular formula, C3H9 N refers to four structural isomers
following factors. of amines
(a) + I effect (b) Steric effect
(a) CH3CH2CH2NH2 (b) CH3–CH–CH3
(c) Solvation effect
In first case, on increasing the size of alkyl group + I effect NH2
increases and the positive charge of ammonium cation gets Primary amine
dispersed more easily, Therefore, order of basicity is 2º amine >
CH3
1º amine > ammonia. In second case, substituted ammonium
cation is also stabilised by solvation with water molecules. (c) CH3–NH–CH2CH3 (d) CH3–N–CH3
Greater the size of ion, lesser will be the solvation and less
Secondary amine Tertiary amine
stabilised is the ion.
Here, only a and b (primary amines) can be prepared from
OH2
their respective amides by Hoffmann bromamide (Br2/NaOH)
H method and can give positive carbylamine test.
| C2H5 H----OH2
+
C2H2—N+—H----OH2 > N
| C2H5 H----OH2 Br2
H ⇒ CH3CH2CH2—C—NH2 CH3CH2CH2—NH2
2° amine NaOH
–Na2CO3
OH2
1° amine H2N—C NH2
Br2/NaOH
⇒ CH3—CH—CH3 CH3—CH—CH3
Considering both steric and solvation effect, it can be concluded –Na2CO3
that greater the number of H-atoms on the N-atom, greater will So, ‘X ’ can be CH3 CH2 CH2  CONH2 (a)
be the H-bonding and hence more stable is the ammonium or CH3 CH(CONH2 )CH3 (b).
cation. But in case of C2H5 group, + I effect predominates
Carbylamine test given by (a) and (b)
over H-bonding. Therefore, order is 2º amine > 1° amine > ∆
ammonia i.e. (C2H5 )2 NH > C2H5NH2 > NH3. CH3 CH2 CH2 NH2 + CHCl 3 + 3KOH  →
CH3 (CH2 )2 NC + 3KCl + 3H2
8. Key Idea The reaction involves:
NH2
A. Deamination in step (i)

B. Oxidation in step (ii) ∆
CH3  CH  CH3 + CHCl 3 + 3KOH  →
C. Hydroysis in step (iii)
NC
The complete pathway of reactions is as follows: 
H 3C O CH3  CH  CH3 + 3KCl + 3H2
(i) NH2
O As (b) is not among the given options So, it is ruled out and
compound (X) is CH3CH2CH2CONH2.
H3C O
NaNO2/H+ 10. NaBH4 is a selective reducing agent. It can reduce C==O
OH
Deamination O group into alcohol, N-methyl imine group, Me  N == CH 
into 2°-amine but cannot reduce an isolated C == C.
H 3C O O OH
(ii) OH NaBH4
O N NH
O H 3C H 3C
H3C O
CrO3/H+ N-methyl isolated Keto
OH imine C C group
Oxidation
O
11. Acetylation by Ac2O/Et3 N is possible with NH2 (1°-amine)
HO O and OH (alcohol) groups only, but not with ‘NH2’ part of
H
H3C O Conc. H2SO4/D O
(iii) Cyclisation 
O
H OH HO  C  NH2 (amide) groups, because the lp of nitrogen in amide
Hydrolysis
group is involved in resonance with C O part.
Intramolecular O Os
dehydration r
O C NH2 C NH2
Final product

Thus, option (c) is the correct answer.


Aliphatic Compounds Containing Nitrogen 419

O 14. The analysis of both the substrates :


H2N CH C NH CH COOH Vinylic centre
Cl
CH2 C NH2 CH2 OH (+R)

O
O Cl
Asn Ser Acyl SN2 centre
(Asparagine) (Serine)
Ac2O or (CH3CO)2O (2 moles) Acts as
Et3N H2N nucleophile
(1°-amine)
–2 AcOH
This N-atom can
O O O
not act as nucleophile
NH2 + R
CH3 C NH CH C NH CH COOH
Amide group

CH2 CONH2 CH2O C CH3 So, the reaction can take place as follows:
O Decrease in the delocalisation of π-bonds
because of which it undergoes free radical
12. No option is the correct answer.
addition (chain growth) polymerisation
Amines in presence of NaNO2 + dil. HCl undergoes
diazotisation. The diazotised product readily loses nitrogen gas Cl Cl Free Cl
with the formation of carbocation. The resulting carbocation radical
–Cl – polymerisation
rearranges itself to give the expanded ring. –H
n
absorbed O
r s by Et3N O C
NH2 NaNO2+dil. HCl N2Cl NH
0-5°C O Cl
(Diazotisation) NH2
(1°-amine) Aliphatic diazonium NH2 O
salt (unstable)
NH2
O
–N2
O
NH2
4
4 r NH2
3 5 5
3 6
6 Ring expansion
2 1
leads to give 2
r 15. Key Idea Among the given compounds the basic nature
–Hr H2O 1
thermodynamically depends upon their tendency to donate electron pair.
OH more stable carbocation
NH
Among the given compounds in , Nitrogen is
sp2-hybridised. This marginally increases the electronegativity
(Major) of nitrogen which in turn decreases the electron donation
tendency of nitrogen. Thus making compound least basic.
13. Rate of acetylation :  NH2 >  OH because N-bases are NH2
Among the rest NH is totally different from others as in
stronger than O-bases. Size of N-atom is larger than O-atom and
this compound lone pair of one nitrogen are in conjugation with
at the same time, N atom is less electronegative than O-atom. So,
•• π bond i.e. As a result of this conjugation the cation formed after
N-atom of the  N H2 group can donate its lone pair of electrons protonation becomes resonance stabilised
••
(Lewis basicity) more easily than that of O atom of the O H
Protonation
group. + +
H 2N NH2 H 2N NH2
OH OH HN NH2
s Equivalent resonance
CH3 CH3COO
H 2N C HN—COCH3
This equivalent resonance in cation makes HN NH2 most
O—COCH3 H
s basic among all.
O—C—CH3 NH2
Categorisation is very simple between rest two as
OH
N CH3
(primary amine) is less basic than (secondary
CH3COOH + H amine)
NHCOCH3 Hence, the correct order is
Absorbed
by pyridine (II) < (I) < (IV) < (III) i.e. option (c) is correct.
N
420 Aliphatic Compounds Containing Nitrogen

16. IV is most basic as conjugate acid is stabilised by resonance of The conjugate base formed above act as nucleophile in the
two  NH2. subsequent step of reaction. As shown above, the nucleophile
r exist in three resonating form, one may think of oxygen being the
NH NH2 NH2 donor atom in the nucleophilic attack. However, nitrogen act as
H+
r donor as it is better donor than oxygen.
H2N NH2 H2N NH2 H2N NH2 O
NH2
SN 2
r N–+ CH2 Br
H2N NH2
Cl
III is least basic as O O
H+ r
H N N H N N H N CH2 Br

Destablised by −I-effect of sp2-carbons. O


17. Hofmann-bromamide degradation reaction is given as: Bromine is not substituted in the above reaction as it is in
resonance with benzene ring giving partial double bond
RCONH2 + 4NaOH + Br2 → RNH2
(1° amine)
character to C—Br bond, hence difficult to break.

+ Na2 CO3 + 2NaBr + 2H2O +
ClCH2 Br ClCH2 Br
Hence, four moles of NaOH and one mole of Br2 are used.
+ −
18. This problem can be solved by using the concept of effect of 22. CH3NH2 + CHCl 3 + KOH → CH3  N ≡≡ C + KCl + H2O
steric hindrance, hydration and H-bonding in basic strength of isocyanide
amines. Order of basic strength of aliphatic amine in aqueous
O
solution is as follows (order of K b )
•• •• •• ••  POCl 3
(CH3 )2 NH > CH3 NH2 > (CH3 )3 N > C6H5 NH2 23. C6H5  C  NH2 → C6H5  C ≡≡ N + H2O
As we know, pK b = − log K b POCl 3 brings about dehydration of primary amide.
••
So, (CH3 )2 NH will have smallest pK b value. In case of phenyl 24. 4, (acetamide) is least basic because lone pair of nitrogen is
amine, N is attached to sp2-hybridised carbon, hence it has involved in delocalisation.
highest pK b and least basic strength. −
O O
19. This reaction is an example of carbylamine test which includes  • •  +
conversion of amine to isocyanide. CH3  C  NH2 ←→ CH3  C == NH2
•• C2 H 5OH + – ‘1’ is most basic due to formation of resonance stabilised
R  NH2 + CHCl 3 → R— N ≡≡ C •• conjugate acid.
1° amine KOH +
Alkyl isocyanide
NH NH2
NOTE The mechanism of this reaction included rearrangement of
nitrene in which migration of alkyl group from carbon to nitrogen H3C—C + H+ H3C—C
takes place. NH2 NH2
(1)
20. Methyl isocyanate CH3  N == C == O (MIC) gas was leaked NH2
from the storage tank of the union carbide plant in Bhopal gas H3C—C +
tragedy. NH2
21. It is the first step of Gabriel's phthalimide synthesis. The 3 (secondary amine) is stronger base than 2 (primary amine).
hydrogen bonded to nitrogen is sufficiently acidic due to two
Hence, overall order of basic strength is :
α-carbonyls.
1>3>2>4
O O
– KOH
NH + OH H 2O + N–
25. H3C— —CH2NH2 + CHCl3

p-methyl benzylamine
O O (a primary amine) H3C— —CH2—NC
O– O
(isocyanide)
N N
Carbylamine test is not given by secondary or tertiary amine.
O O–
Aliphatic Compounds Containing Nitrogen 421

+ +
26. C6H5 NH3Cl − + AgNO3 → C6H5 N H3NO3− + AgCl (s) ↓ 33. Column I Column II
anilinium (A) CH3 CH2 CH2 CN : Gives amine with Pd-C/H2
hydrochloride
Gives aldehyde with
No such precipitate of AgCl(s) would be formed with
SnCl 2 /HCl Gives amide with
p-chloroaniline.
diisobutyl- aluminium hydride.
KOH Gives carboxylic acid on
27. R  NH2 + CHCl 3 → R  NC ‘Carbylamine test’
isocyanide alkaline hydrolysis.
(foul smell)
(B) CH3 CH2 OCOCH3 : Reduced to alcohol with
The above test is characteristic of primary amine and used to ester Pd - C/H2 Reduced with
distinguish primary amine from other amines. diisobutylaluminium hydride
O into aldehyde. Undergo
 NaOH
alkaline hydrolysis.
28. CH3  C  NH2 + Br2 → CH3NH2
acetamide (C) CH3 CH == CHCH2 OH Reduced to butanol when
methylamine
: treated with Pd-C/H2.
(Hofmann’s bromamide reaction)
(D) CH3 CH 2 CH2 CH2 NH2 A primary amine, gives
29. A secondary amine, on treatment with aqueous nitrous acid at
carbylamine test.
low temperature produces oily nitrosamine.
(C2H5 )2 NH + HNO2 → (C2H5 )2 N  NO + H2O
nitrosamine
H C2H5
 
30. Reaction proceeds via diazonium salt with neighbouring group 34. CH3 *C  NH2 or CH3  CH  CH2NH2
*
participation. 
CH  CH3

R OH HO CH3
H
COOH (X )
CH3 HNO2 R  C O
O
H NN
H CH3
CH3 H NH2 NN r  
r * NaNO2
H CH3  C  NH2 → CH3  CH2  C  CH3
 HCl 
R
R COOH H COOH r Or CH  CH3 OH
–H
O  Y -optically inactive
H2 O H
H OH CH3
OH (I) R
Conformer of (I) Above reaction proceeds via carbocation intermediate, hence
rearrangement takes place.
31. Carbylamine test is given by primary amines only.
35. Conjugate acid of dimethylamine is more stable than conjugate
CH3 CH3 NH2 base of triethyl amine due to exhaustive H-bonding with water.
N CH3 O
(CH3 CO) 2 O 
36. CH3CH2NH2 → CH3CH2  NH  C  CH3
Heat
+ CH3COOH
CH3
N,N-dimethylaniline 2,4-dimethylaniline 37. A must be a secondary amine :
(3°-amine) (1°-amine)
CH3CH2NHCH3 + benzene sulphonylchloride →
NHCH3 CH3
 NaOH
CH3 CH3CH2  N  SO2  C6H5 → Insoluble
H3C CH2NH2
38. Starting compound is a primary amine.
N-methyl-o-methylaniline p-methyl benzylamine R  NH2 + CHCl 3 + KCl → R  NC
(2°-amine) (1°-amine) foul smell
Also, R  NH2 + HNO2 → R  OH + N2
32. More : CH3NH2 is stronger base than CH3OH.
422 Aliphatic Compounds Containing Nitrogen

112
Moles of N2 = = 5 × 10− 3 44. Explanation
22500 O
Q One mole of N2 is obtained from 1.0 mole of R NH2, mole
of R NH2 = 5 × 10− 3 –
MnO4 /H+
OH NH3
OH ∆
⇒ 5 × 10− 3 mol R NH2 weigh = 0.295 g ∆

0.295 OP
1 mole of R NH2 will weigh = = 59 g
5 × 10− 3
O
In R NH2, NH2 has molar mass = 16 NH2
⇒ R— has molar mass = 43 NH2 Br2/NaOH
NH2 ∆
Also, the alcohol R—OH gives iodoform test; it must has the NH2
following skeleton :
O R
R′  CH  CH3 Q O

OH ∆
⇒ R′— has molar mass = 15 (CH3 ) NH
KOH

CH3

⇒ Original compound is CH3  CH  NH2 O O
S
39. Aniline > N-methylaniline < methylamine < dimethylamine.
Br COOC2H5
40. Carbylamine test given only by primary amine. N—K
–KBr
KOH O
CH3CH2  NH2 + CHCl 3 → CH3  CH2  NC
primary amine foul smell O
S1 N
Nitrosamine test is given only by secondary amine.
(CH3CH2 )2 NH + HNO2 → (CH3CH2 )2 N  NO COOC2H5
oily nitrosamine O
O O– O S2
+ + +
41. (i) CH3—N CH3—N CH3—N – O
O– O O NH2
O H 3O + OH
+ + OH
(ii) CH3—N CH2==N : Tautomerism OH COOH
O– O–
O (T)
NaCN LiAlH4 (P)
42. CH3CH2Cl → CH3CH2CN → CH3CH2CH2NH2
SN 2

43. (a)
28
Benzene and Alkyl Benzene
Objective Questions I (Only one correct option) 5.
1. The number of sp 2 -hybrid orbitals in a molecule of benzene is The major product of the following reaction is
(2019 Main, 9 April I)
(2020 Main, 9 Jan II) CH2CH3
(a) 24 (b) 12 (c) 18 (d) 6
(i) Alkaline KMnO4
2. The major product obtained in the given reaction is (ii) H3 O+
(2019 Main, 10 April II)
AlCl3 COOH CH2CHO
CH3 O CH2 CH3 Product
CH2 CH

Cl (a) (b)
CH3
COCH3 CH2COOH
(a) CH3 O

(c) (d)
H3C O CH CH3
(b) CH2 CH 6. The major product of the following reaction is
(2019 Main, 9 April I)
H3C O
1. KOH (alc.)
(c)
Cl 2. Free radical
Cl polymerisation

CH3 CH3

O CH2 (a) (b)


H3C
(d) CH2 CH CH2
n n

3. The increasing order of the reactivity of the following


Cl
compounds towards electrophilic aromatic substitution
reaction is (2019 Main, 10 April I) CH3 Cl
Cl CH3 COCH3 (c) (d)

n n

(I) (II) (III)


(a) III < I < II (b) II < I < III OH OH
(c) III < II < I (d) I < III < II 7. The increasing order of reactivity of the following
4. Increasing order of reactivity of the following compounds for compounds towards aromatic electrophilic substitution
SN 1substitution is (2019 Main, 9 April II) reaction is (2019 Main, 9 April I)
CH3 Cl OMe Me CN
(A) CH2 Cl (B)
CH3 H3C Cl

Cl Cl
(C) (D )
H3CO A B C D
(a) (A) < (B) < (D) < (C) (b) (B) < (C) < (D) < (A) (a) A < B < C < D (b) B < C < A < D
(c) (B) < (A) < (D) < (C) (d) (B) < (C) < (A) < (D) (c) D < A < C < B (d) D < B < A < C
424 Benzene and Alkyl Benzene

8. Polysubstitutiion is a major drawback in (2019 Main, 8 April II)


(a) Friedel-Craft’s alkylation 10. Among the following four aromatic compounds, which one
(b) Reimer-Tiemann reaction will have the lowest melting point? (2019 Main, 12 Jan I)
(c) Friedel-Craft’s acylation O
(d) Acetylation of aniline
OH
9. The major product of the following reaction is (a) (b)
OH
(2019 Main, 8 April II)
CH3 O
CH3
(1) Cl2/hn OH
(2) H2O, D O
(c) O (d)

Cl CH3
CHCl2 CO2H
11. Which of the following compounds is not aromatic?
(2019 Main, 9 Jan II)
(a) (b)
(a) (b)
N
H
Cl Cl
(c) (d)
CH2 OH CHO
N

(c) (d) 12. In the following sequence of reactions (2015 Main)


KMnO 4 SOCl 2 H 2 / Pd
Toluene → A → B → C, the product C is
BaSO 4
Cl Cl (a) C6 H5 COOH (b) C6 H5 CH3
(c) C6 H5 CH3 OH (d) C6 H5 CHO

13. Match the four starting materials given in Column I with the corresponding reaction schemes provided in Column II and select the
correct answer using the code given below the lists.
Column I Column II
A. H H p. Scheme I
(i) KMnO4 , HO− , heat (ii) H + , H 2O
(iii) SOCl 2 (iv) NH 3
?  
→ C7 H6N2O3
B. OH q. Scheme II
(i) Sn / HCl (ii) CH 3COCl (iii) Conc. H 2SO4
(iv) HNO3 (v) Dil. H 2 SO4 , heat (vi) HO−
?  → C6H6N2O2

OH
C. NO2 r. Scheme III
(i) Red hot iron, 873 K (ii) fuming HNO3 , H 2SO4 , heat
(iii) H 2S ⋅ NH 3 (iv) NaNO2 , H 2SO4 (v) hydrolysis
?  → C6H5NO3

D. NO2 s. Scheme IV
(i) Conc. H 2SO4 ,60°C
(ii) Conc. HNO3 , conc. H 2SO4 (iii) Dil ⋅ H 2SO4 , heat
?  
→ C6H5NO4

CH3

Codes
A B C D A B C D
(a) p s q r (b) r p s q
(c) r s q p (d) s p r q (2014 Adv.)
Benzene and Alkyl Benzene 425

14. For which of the following molecule significant µ ≠ 0? COOH OCH3


Cl CN OH SH (d)
HO H3C NO2

NO2 O

C
O NO2
Cl CN OH SH
I II III IV

(a) Only I (b) I and II (2014 Main)


HBr
(c) Only III (d) III and IV 16. In the reaction, —OCH3 the products are,
(2010)
15. The compounds P, Q and S
COOH OCH3 (a) Br— —OCH3 and H2 (b) —Br and CH3Br

H3C H 3C (c) —Br and CH3OH (d) —OH and CH3Br


P Q O

C 17. In the following reaction,


O
O
H3C
S N Conc. HNO3
H X
Conc. H2SO4
were separately subjected to nitration using HNO3 / H2 SO4
mixture. The major product formed in each case The product X is (2007, 3M)
respectively, is (2010)
O
(a)
COOH OCH3 NO2
N
(a)
H
HO H3C
O O2N
NO2 NO2
O
C (b)
O N
H
NO2
O
(c)
COOH OCH3
N
(b) H
HO NO2 H 3C NO2
NO2 O
O (d)

C O2N N
O H

NO2 AlCl
18. + Cl — CH2 CH2 — CH3 →
3

COOH OCH3
(i) O / ∆
P →
2
Q + Phenol
(c) (ii) H 3 O +
HO H 3C NO2 P and Q are respectively (2006, 5M)

NO2 O NO2
(a) + CH3CH2CHO
C
O
(b) + CH3COCH3
426 Benzene and Alkyl Benzene

25. The compound that is most reactive towards electrophilic


substitution is (1985, 1M)
(c) + CH3COCH3
(a) toluene (b) benzene
(c) benzoic acid (d) nitrobenzene
26. Among the following, the compound that can be most readily
(d) + CH3CH2CHO sulphonated is (1982)
(a) benzene (b) nitrobenzene
19. O (c) toluene (d) chlorobenzene
HN
H3C CH3
Fe/Br2 Objective Questions II
(One or more than one correct option)
product on monobromination of this compound is 27. Consider the following transformations of a compound P.
(2004, 1M) R P (i) X (reagent)
Q
(i) NaNH2
(Optically (C9H12) (C8H12O6)
O O (ii) C H COCH (ii) KMnO4/H2SO4/∆
HN HN active) 6 5 3 (Optically
(iii) H3O+/ ∆ active
H3C CH3 H3C CH3 Pt/H2 acid)
(a) (b)
CH3

Br Br
O
HN Choose the correct option(s).
H3C CH3
(c)
Br
(a) P is (b) X is Pd-C/quinoline/H2
O
HN
H3C CH3
(d)
Br (c) P is (d) R is
20. Identify the correct order of reactivity in electrophilic
substitution reactions of the following compounds
Benzene (1), Toluene (2), Chlorobenzene (3) and 28. Choose the correct option(s) for the following set of
Nitrobenzene (4) (2002) reactions. (2019 Adv.)
(a) 1 > 2 > 3 > 4 (b) 4 > 3 > 2 > 1 (i) MeMgBr Conc. HCl
(c) 2 > 1 > 3 > 4 (d) 2 > 3 > 1 > 4 C6H10O
(ii) H2O
Q S
(major)
21. A solution of (+) -2-chloro-2-phenylethane in toluene 20 % H3PO4, 360 K
racemises slowly in the presence of small amount of SbCl 5 ,
due to the formation of (1999, 2M) (i) H2, Ni HBr, benzoyl peroxide
T R U
(a) carbanion (b) carbene (major) (ii) Br2, hν (major) ∆
(major)
(c) free-radical (d) carbocation
CH3 H C CH3
22. Benzyl chloride (C6 H5 CH2 Cl) can be prepared from toluene 3 Br H3C Br
by chlorination with (1998, 2M) Cl Cl
(a) (b)
(a) SO2Cl 2 (b) SOCl 2 (c) Cl 2 (d) NaOCl
23. Chlorination of toluene in the presence of light and heat S T U S
followed by treatment with aqueous NaOH gives (1990, 1M) CH3
CH3 H 3C Br
(a) o-cresol (b) p-cresol H 3C Cl
Br
(c) 2, 4-dihydroxy toluene (d) benzoic acid Br
(c) (d)
24. The reaction of toluene with chlorine in the presence of ferric
chloride (FeCl 3 ) gives predominantly (1986, 1M) U T
S U
(a) benzoyl chloride (b) m-chlorotoluene
(c) benzyl chloride (d) o- and p-chlorotoluene
Benzene and Alkyl Benzene 427

29. Choose the correct option(s) for the following reaction 32. The major product U in the following reaction is (2015 Adv.)
sequence (2019 Adv.) Radical
CH2==CH—CH 3,H + initiator, O2
CHO (i) Hg2+, dil. H2SO4 (i) SOCl2 T U
High pressure, Heat
(ii) AgNO3, NH4OH pyridine
Q
(iii) Zn-Hg, conc. HCl (ii) AlCl 3
MeO H
Zn-Hg
conc. HCl O
R S O H3C CH3
CH3 O
Consider Q, R and S as major products. O H
(a) (b)

H
(a) MeO MeO
R O S O
O CH2
CH2 O H
CO2H O
(b) (c) (d)
MeO MeO
O
Q R 33. Which of the following molecules, in pure form, is/are
unstable at room temperature? (2012)

(c) MeO MeO (a) (b)


O
R S O
O
OH

CO2H (c) (d)


(d)
MeO
MeO
Q S 34. An aromatic molecule will (1999, 3M)
(a) have 4nπ-electrons
30. The reaction(s) leading to the formation of
(b) have (4 n + 2)π-electrons
1,3,5-trimethylbenzene is (are) (2018 Adv.)
(c) be planar
O Conc. H2SO4 (d) be cyclic
(a)

35. Which compound(s) out of the following is/are not aromatic?
Heated iron tube (2019 Main, 11 Jan I)
(b) Me —— H
873 K
(a) (b)
O
(1) Br2, NaOH ⊕ ⊕
(2) H3O+
(c)
(3) Sodalime, ∆
(c) (d)
O O
CHO s
36. Toluene, when treated with Br2 /Fe, gives p-bromotoluene as
(d) Zn/Hg, HCl
the major product because the CH3 group (1999, 3M)

OHC CHO (a) is para-directing


(b) is meta-directing
31. Among the following reactions(s), which gives(give) (c) activates the ring by hyperconjugation
tert-butyl benzene as the major product? (2016 Adv.)
(d) deactivates the ring
Br Cl 37. A new carbon–carbon bond formation is possible in (1998)
(a) (b)
NaOC2H5 AlCl3 (a) Cannizzaro’s reaction
(b) Friedel-Craft’s alkylation
OH (c) Clemmensen reduction
(d) Reimer-Tiemann reaction
(c) (d)
H2SO4 BF3,OEt2
428 Benzene and Alkyl Benzene

Numerical Answer Type Questions 41. List-I includes starting materials and reagents of selected
chemical reactions. List-II gives structures of compounds that
38. In the following reaction, compound Q is obtained from may be formed as intermediate products and/or final products
compound P via an ionic intermediate. from the reactions of List-I.
CO2CH3
C6H5 C6H5 List-I List-II
Conc. H2SO4 CN (i) DIBAL-H
Q (A coloured compound) CHO
(I) (ii) Dil. HCl (P)
O (iii) NaBH4 CO2H
(iv) Conc. H2SO4
C6H5 O
(P)
(i) O3 OH
What is the degree of unsaturation of Q? (2020 Adv.) (II) (ii) Zn, H2O (Q)
CO2H (iii) NaBH4 OH
39. Total number of hydroxyl groups present in a molecule of the (iv) Conc. H2SO4
major product P is ……… (2019 Adv.)
Cl (i) KCN
(III) (ii) H3O+,∆ (R)
(iii) LiAIH4 O
(i) H2, Pd-BaSO4, quinoline CO2CH3 (iv) Conc. H2SO4
(ii) Dil. KMnO4, (excess), 273 K
P
CO2Me OH
(IV) (iii) LiAIH4 (S)
CO2Me (iv) Conc. H2SO4
CO2H

CO2H
Match the Columns (T)
40. List-I includes starting materials and reagents of selected CO2H
chemical reactions. List-II gives structures of compounds
that may be formed as intermediate products and/or final (U)
O
products from the reactions of List-I.
O
List-I List-II
CN (i) DIBAL-H CHO
(I) (ii) Dil. HCl (P)
Which of the following options has correct combination
O (iii) NaBH4 considering List-I and List-II? (2019 Adv.)
(iv) Conc. H2SO4 CO2H
O (a) (II), (P), (S), (U)
(b) (I), (Q), (T), (U)
(i) O3 OH
(II) (ii) Zn, H2O (Q) (c) (II), (P), (S), (T)
CO2H (iii) NaBH4 OH (d) (I), (S), (Q), (R)
(iv) Conc. H2SO4

(III)
Cl (i) KCN
(ii) H3O+,∆ (R)
True or False
(iii) LiAIH4 O 42. An electron donating substituent in benzene orients the
CO2CH3 (iv) Conc. H2SO4
incoming electrophilic group to the meta-position.
CO2Me OH (1987, 1M)
(iii) LiAIH4
(IV) (S)
CO2Me (iv) Conc. H2SO4 43. In benzene, carbon uses all the three p-orbitals for
CO2H hybridisation. (1987, 1M)
CO2H
(T)
Subjective Questions
CO2H
44. Explain the followings :
(U) CH3
O
Br C2H5OH (aq) acidic solution
O (a) (i) CH3
Which of the following options has correct combination CH3
considering List-I and List-II? (2019 Adv.) C2H5OH (aq)
(a) (III), (S), (R) (b) (IV), (Q), (R) (ii) Br neutral
CH3 solution
(c) (III), (T), (U) (d) (IV), (Q), (U)
Benzene and Alkyl Benzene 429

F 47. Give reasons for the following :


(b) (i) NaOH (aq) (i) tert-butylbenzene does not give benzoic acid on treatment
F– (liberated) with acidic KMnO4.
O 2N
(ii) Normally, benzene gives electrophilic substitution
CH3 reaction rather than electrophilic addition reaction
although it has double bond. (2000)
F 48. Show the steps to carry out the following transformations.
NaOH (aq)
(ii) F – (not liberated) (i) Ethylbenzene → benzene
H3C (ii) Ethylbenzene → 2-phenylpropionic acid
CH2NO2 49. Write the structures of the products.
Br 2 /Fe
NO NO NO CH3 CONHC6 H5 → (1998, 2M)
NO2 50. Give reasons for the following in one or two sentences
Conc. HNO3
(c) (i) + “Nitrobenzene does not undergo Friedel-Craft’s alkylation.”
Conc. H2SO4
(1998, 2M)

NO2 51. Complete the following, giving the structures of the principal
organic products. (1997)

NO2 NO2 Me
 CH3
Conc. HNO3  Anhy. AlCl3
(ii) + H3C—C—CH2Br
Conc. H2SO4
NO2 
H
52. Toluene reacts with bromine in the presence of the light to
give benzyl bromide, while in presence of FeBr3 it gives
p-bromotoluene. Give explanation for the above observation.
Pd/C (1996)
(d)
3.0 mol 53. Predict the structures of the intermediates/products in the
H2 following reaction sequences (1996)

OMe O

(i) AlCl3 Zn(Hg)/HCl


+ O A B
(ii) H3O+
O
O
MeO
H3PO4
not B

54. Predict the major product in the following reactions (1994)

H SO
(i) + (CH3 )2 CHCH2 OH →
2 4

(2005)
C2H5
45. 7-bromo-1, 3, 5-cycloheptatriene exists as ionic species in (i) Br , heat, light
2
aqueous solution while 5-bromo-1,3-cyclopentadiene (ii) →
(ii) NaCN
doesn't ionise even in presence of Ag + ( aq ). Explain. (2004)
55. Give reason for the following ‘In acylium ion the structure
46. What would be the major product in the following reaction? R  C ≡≡ O+ is more stable than R  C+ == O’ (1994, 1M)
(2000, Main, 1M)
O 56. Identify the major product in the following reactions :
HNO3/H2SO4
Br2/Fe(1eq) (i) —COO—
N— mononitration

(ii) C6H5COOH + CH3MgI ? + ? (1993, 2M)


430 Benzene and Alkyl Benzene

57. Write the structure of the major organic product expected 63. Write down the reaction involved in the preparation of
from the following reaction following using the reagents indicated against in parenthesis.
AlCl3 “Ethyl benzene from benzene.”
+ (CH3)2CHCH2Cl [C2 H5 OH, PCl 5 , anhyd. AlCl 3 ] (1984, 2M)
(1992, 1M)
64. Show with balanced equation what happens when the
58. Give reasons in two or three sentences only for the following:
‘p-xylene is reacted with concentrated sulphuric acid and the
Phenyl group is known to exert negative inductive effect, but resultant product is fused with KOH’. (1984, 1M)
each phenyl ring in biphenyl (C6 H5  C6 H5 ) is more
reactive than benzene towards electrophilic substitution. 65. Give reasons for the following in one or two sentences :
(1992, 1M) ‘Although benzene is highly unsaturated, normally it does
59. Arrange the following in increasing order of reactivity not undergo addition reaction.’ (1983, 1M)

towards sulphonation with fuming sulphuric acid.


Benzene, toluene, methoxy benzene, chlorobenzene.
Integer Type Question
(1988, 1M) 66. Among the following, the number of aromatic compound(s)
60. Answer the following with suitable equation wherever is
necessary s r
r

(i) How can you prepare benzene from lime ?


(ii) How will you convert toluene to m-nitrobenzoic acid ?
(1987, 2M)
61. Write down the main product of the following reaction r r
CH3CH2COCl/AlCl 3
Benzene → (1985, 1M)

62. How many sigma and pi-bonds are present in a benzene


molecule ? (1985, 1M)

Answers
1. (c) 2. (c) 3. (a) 4. (c) 25. (a) 26. (c) 27. (b,c) 28. (c,d)
5. (a) 6. (a) 7. (c) 8. (a) 29. (a,b) 30. (a,b,d) 31. (b,c,d) 32. (b)
9. (d) 10. (b) 11. (b) 12. (d) 33. (b,c) 34. (b,c,d) 35. (b,c,d) 36. (a,c)
13. (c) 14. (d) 15. (c) 16. (d) 37. (b,d) 38. (18) 39. (6) 40. (b)
17. (b) 18. (c) 19. (b) 20. (c) 41. (a) 42. False 43. False 62. (12σ, 3π)
21. (d) 22. (a) 23. (d) 24. (d) 66. (5)

Hints & Solutions


1. In benzene, every carbon atom is sp2-hybridised. 2. The given reaction takes place as follows :
CH3 O CH3 O
AlCl3
Cl – AlCl s
4
Each sp2-hybridised atom has three sp2 hybrid orbitals. r
Intramolecular
∴ Total number of sp2 hybrid orbitals in a molecule of benzene is ArSE 2-reaction
= 6 × 3 = 18
It will stabilise the carbocation by
(+ve) hyperconjugation

CH3 CH3 O
O
–HCl r
–AlCl3

H
CH3 –
AlCl4
Benzene and Alkyl Benzene 431

3. Key Idea In electrophilic aromatic substitution (Ar SE 2) 6. In presence of alc. KOH, the given halide undergoes
reaction, the aromatic (benzene nucleus here) compound
elimination reaction
(substrate) acts as a nucleophile which gets attacked by an
electrophile in the rate determining step (rds). Alc. KOH

Higher electron density on the nucleophile, i.e. benzene nucleus Cl


will fasten the rate of the reaction. Electron-donating groups Cl Cl
(EDG) will increase electron density in benzene nucleus by any The alkene produced undergoes polymerisation via free radical
or both of their +R/+M and hyperconjugative (HPC) effects. mechanism. This process involve three steps, i.e. initiation,
Now, let us consider the given substrate. propagation and termination.
Cl –I effect dominates the +R effect CH3 + HPC
Polymerisation n

(I) (II) Cl Cl
O
Polymer
C—CH3 –R effect
7. Aromatic electrophilic substitution reactions are fast in those
compounds in which the attacking site possess high electron
density. Electron withdrawing groups (EWGs ) reduces the
(III)
electron density in the benzene ring due to its strong –I effect
As –R effect is more powerful than –I effect, the order of their and makes further substitution difficult. Hence, called as
reactivity, towards an electrophile will be deactivating groups. While electron releasing groups (ERGs)
III I II
increases the electron density in the benzene due to resonance.
(–R) < (–I>+R) < (+ HPC)
Therefore, activates the benzene ring for the attack by an
4. Reactivity of substitution nucleophilic unimolecular (S N1) electrophile. Hence, called as activating groups.
reaction depends on the formation of carbocation. Greater the
••
stability of carbocation, greater will be its ease of formation of In given options, Activating groups are — CH3 < — OCH3
alkyl halide and faster will be the rate of reaction. So, the correct (+ I, + R)
••
Strong + R effect
order of (S N1) reactivity is
CH2Cl CH2—Cl Deactivating groups are — CN > — Cl
Strong – I effect (− I > − R)
So, the correct order is
> > > CH3 CH2Cl CN Cl Me OCH3

Cl
OCH3 < < <
(+R)
(C) (D) (A ) (B)
D A C B
In compound C, the carbocation formed is stabilised by activating
group (  OCH3 ). Compound D forms benzyl carbocation 8. Through aromatic electrophilic substitution mechanism
(C6H5 CH2+ ) that is stabilised by resonance. Compound A (ArS E2) when mono-alkylation (Friedel-Craft’s reaction) is
produces a primary carbocation that further rearranges itself to performed, we get mono-alkylated benzene. As, the alkyl group
secondary carbocation. is ring activating (towards ArS E 2) in nature, again o- and
p-substituted alkyl benzene will be obtained and so on. e.g.
Nus Rearrangement
r r CH2CH3
Cl 1° carbocation C2H5Cl (1 eqv.) C 2H 5 C 2H 5
2° carbocation

Compound B produces primary carbocation which is least stable AlCl3 (anhyd.) C2H5
C2H5Cl
–HCl +
among all the given options. C2H5 AlCl3

5. The major product of the given reaction is benzoic acid


C2H5Cl (excess) C2H5
(C6H5COOH). On vigorous oxidation of alkyl benzene with
acidic or alkaline KMnO4, aromatic acids are obtained. During AlCl3 (anhyd.)

oxidation of alkyl benzene, the aromatic nucleus remains intact


C 2H 5 C2H5
and the entire chain is oxidised to COOH group irrespective
C 2H C
5
of the length of carbon chain. H5C2 C2H5
2H5Cl
–+
CH2CH3 COOK COOH + AlCl3

(Minor)
(i) Alk. KMnO4 (ii) H3O+ C2H5
(Major)

Ethyl benzene Potassium benzoate Benzoic acid


432 Benzene and Alkyl Benzene
CH3
9. Compound (a) is a dicarboxylic acid and possess high melting
point due to intermolecular hydrogen bonding.
As a result, it exist as associated molecule. Compound (d) is an
alcohol and possess intermolecular H-bonding. No interactions
are present in hydrocarbon (naphthalene) compound (b).
Cl
Hence, melting point is lowest for naphthalene (− ~ 80°C), i.e.
hν/benzylic free radical
substitution with Cl2 compound (b).
Cl2(1 eqv.) Cl2(2 eqv.) Cl2(excess)
11. Aromaticity of a compound can be decided by Huckel’s rule. In
–HCl –2HCl –3HCl cyclopentadienyl cation (b), resonance takes place as follows:
CH2Cl CHCl2 CCl3

Cl Cl Cl
Resonance hybrid
s s
s s
–3H –3OH(aq) H
–Cl OH(Cl) –2HCl 2H2O/∆
OH
OH
C OH
Hence, is anti-aromatic does not follow
CH2OH CH
O—H
OH
Huckel’s rule as it has conjugated 4π-electron (4 nπ, n = 1)
Cl Cl Cl system. Rest of the species are aromatic as each of them belongs
(Option-c) Gem-diol to 6π-electron [(4 n + 2) π, n = 1] system.
–H2O
(unstable) OH CH3 COOH COCl CHO
12.
–H2O C O
KMnO4 SOCl2 H2/Pd
CH O BaSO4
Cl

as COO Toluene Benzoic acid Benzoyl Benzaldehyde
Cl chloride
(Option-d)
Cl 13. PLAN This problem can be solved by using the various concepts
in basic medium synthesis of benzene, electrophilic substitution reaction and
H3Or
directive influence of various substituents, including oxidation
and reduction.
COOH
⇒ —OH and —NH2 are o/p-directing groups.

Cl
⇒ N-acetylation is more favourable than C-acylation.
(Option-b)
⇒ N-sulphonation is more favourable than
So, considering the second reaction condition, Cl− , H2O, ∆ in C-sulphonation.
the statement of the question, the correct answer is option (d). ⇒ NO2 is a meta-directing group.
⇒ H2S⋅ NH3 reduces only one NO2 group selectively in the
10. Melting point of a compound depends on size and force of
attraction between the molecules. Compound (c) has largest size presence of two NO2 groups.
and also possess weak intermolecular association due to dipole - Using above concepts the correct sequence of reaction can be
dipole interactions. written as

NO2 NH2

NaNO2
Red hot Fuming HNO3 / H2SO4 H2S .NH3
A. 3CH CH H2SO4
iron 873K
∆ selective reduction
NO2 NO2 Here, N-sulphonation is
N+2HSO4– OH more favourable in
comparisons to
C-sulphonation.
Hydrolysis

NO2 NO2
OH OH OH OH

+
O 2N O2N
Conc. H2SO4 NO2 Dil.H2SO4
B.
60°C Here, nitration takes ∆
OH OH place at o-position of OH OH
less hindered side.
SO3H SO3H
Benzene and Alkyl Benzene 433

NO2 NH2 NHCOCH3 NHCOCH3 NHCOCH3


NO2
Sn/HCl CH3COCl Conc.H2SO4 HNO3
C.
Here, N-acetylation is
more favourable than
C-acetylation
SO3H SO3H
Dil.H2SO4 ∆

NH2 NHCOCH3

NO2 NO2
OH–

NO2 NO2 NO2 NO2 NO2

KMnO4 H 3O + SOCl2 NH3


D. – OH/∆

CH3 COO– COOH COCl CONH2


Above reaction proceeds through oxidation, chlorination and amide formation sequentially.
∴ A → r, B → s, C → q, D → p Hence, the correct choice is (c).
14. Draw the structure of organic compounds indicating net dipole moment which includes lone pair and bond angle also.
OH SH H H
O O
In the quinol and thioquinol
same as in thioquinol.
OH SH
— OH groups and —SH groups do not cancel their O O
dipole moment as they exist in different confirmation. H H

15. COOH COOH


HNO3
H2SO4
HO HO
P
NO2
— OH is activating while — COOH is deactivating group in SE Ar reaction. Therefore, electrophile attack to ortho of the activating
— OH group.
OCH3 OCH3
HNO3
H2SO4
H3C H3C NO2
Q

Both — OCH3 and — CH3 are activating ortho/para directing groups but — OCH3 is stronger activator, electrophile attack to ortho of
— OCH3.
O O NO2

C C
O HNO3 O
II H2SO4

I S
434 Benzene and Alkyl Benzene

Ring II is activated while ring I is deactivated in SE Ar reaction. 22. SO2Cl 2 brings about free-radical chlorination at —CH3 group.
Therefore, electrophile attack at para to ring-II, the less
hindered position. 23. Free radical chlorination occur at —CH3.

16.  OCH3 + HBr → CH3Br +  OH hv


—CH3 + Cl2 —CCl3
Heat

—OH of phenol is not further substituted by bromide due to


resonance with the ring. NaOH
—COOH
17. Ring attached to nitrogen is activated while ring attached to
C==O is deactivated. Also, electrophilic substitution occur benzoic acid
predominantly at para position of the activated ring due to
immense steric hindrance at ortho position. 24. The methyl group in toluene is ortho/para directing activating
group :
O
CH3 CH3 CH3
Conc. HNO3
N
Conc. H2SO4 Cl
FeCl3
H + Cl2 +
O2N
O

N Cl

H 25. Toluene is most reactive among these. Nitro and carboxylic groups
are deactivating in aromatic electrophilic substitution reaction.
18.
AlCl3
+ ClCH2CH2CH3 P 26. Toluene is most readily sulphonated among these because
Friedel-Craft’s methyl group is electron donating (+ I effect), activate benzene
reaction
ring for electrophilic aromatic substitution.
O2/heat
Q + Phenol,
H3O+ 27. Compound (P) on reaction with Lindlar catalyst (X) followed by
O oxidation in presence of acidified KMnO4, gives an optically
active compound (Q) 3-carboxy-heptane-1, 7-dioic acid.
P= , Q = CH3—C—CH3
Acetone 4 2
5 3 1
Cumene Pd–C/H2 CH==CH 2
19. Ring attached to nitrogen is activated by electron donating ( X)
6
resonance effect while ring attached to carbonyl group is (P) 7
deactivated by electron withdrawing resonance effect : 4 2
1
O O 5 * COOH
HN HN KMnO4/H2SO4 3
∆ , – CO 2 COOH
H 3C CH3 H3C CH3 6
Fe 7 COOH
Br2 (Q)

Compound (P) also react with strong base (NaNH2 ), gives


Br alkaline which one treatment with C6H5COCH3 / H3O+ / ∆,
20. Both chloro and nitro groups are deactivating in electrophilic gives an optically active compound (R).
aromatic substitution reaction. Also nitro group is stronger
deactivating group. Methyl group is activator in electrophilic (i) NaNH2 C==CNa
aromatic substitution.
Hence, overall order of reactivity is :
(P)
nitrobenzene < chlorobenzene < benzene < toluene
(ii) C6H5COCH3 * C==C—C—C6H5
H
 +
H +
(iii) H3O/∆
Cl − CH2
21. C6H5  C  Cl + SbCl 5 → SbCl6− + C6H5  C → – H2O
(R)
 CH3 Optically active
CH3 planar
(+)-2-chloro- carbocation
2-phenylethane
(±) -2-chloro-2-phenylethane (racemic mixture)
Benzene and Alkyl Benzene 435

28. The given road map problem is


H—OH
O CH3 CH3 OH CH3 Cl CH3
O MgBr
H 20% H3PO4,
CH3MgBr H2O Br Conc. HCl 360K
+ Mg + H2O
Hydrolysis –H2O Dehydration
OH
(C6H10O) (Q) Major of alcohol major
(S) (R)
CH3 Br CH3
(i) H2,Ni Br
(Reduction) HBr
(ii) Br2,hn Benzoyl peroxide
major (free radical (anti-Markovnikov's rule) major
(T ) substitution) (U)

Hence, options (c, d) are correct.


C C CH2 CHO C CH CH2 CHO
29. (i) Hg2+, dil. H2SO4
Hydration of Alkyne OH
CH3O CH3O
Keto–enol
Tautomerism
(ii) AgNO3+NH4OH
C CH2 CH2 C O– (Tollen's reagent)
C CH2 CH2 CHO

O O (¾CHO converts into COO–) O


CH3O CH3O
CH2 CH2
(iii) Zn – Hg + Conc. HCl CH2 (i) SOCl2+Pyridine CH2
(Clemmensen reduction) (ii) AlCl3
CH3O CH2 CH3O H
C ¾ reduce to ¾ CH2 Cl CH2
(Q) C
COOH
O
O
CH2 CH2
CH2 CH2
Zn-Hg/Conc. HCl Electrophilic
Clemmensen reduction CH2 substitution reaction
CH2 CH3O
CH3O CH2 C
(S)
O
(R)

30. Reaction shown in option (a) is aldol condensation in the presence of conc. H2SO 4 at high temperature.
In summerised way the formation of mesitylene through this can be visualised as
CH3
C H CH3
H2O O C Conc. H2SO4
H H ∆
HH H2O
O –3H2O
H—C
CH3 CH3
C C
H C Mesitylene
CH3 O CH3
H
H
H2O
Reaction given in option (b) is simple polymerisation (trimerisation) reaction of alkyne i.e.,
Me H or CH3 C CH when passed through heated iron tube at 873K then mesitylene is formed as
CH3
CH3
C
CH Heated iron tube
HC 873 K
C CH3 CH3
C HC CH3
CH3 Mesitylene
436 Benzene and Alkyl Benzene

This reaction is also called aromatisation. Cumene hydroperoxide formed above is an intermediate in the
(1) and (2) reactions of option (c) combined to give haloform synthesis of phenol.
reaction while (3) reaction given in this option is 33. According to Huckel rule, the compounds which have
decarboxylation reaction i.e., 4 n (n = 0, 1, 2, 3… ) delocalised π-electrons in a close-loop are

COCH3 COONa+ anti-aromatic and characteristically unstable. Compound B
(1) Br2/NaOH
satisfy the criteria of anti-aromaticity as :
(2) H3O+
+– – 4π
H3COC COCH3 NaOOC COONa+
+ 3CHBr3↓ Compound C is anti-aromatic in its resonance form :
Bromoform –
O O
The above product of haloform reaction on decarboxylation gives
benzene as

COONa+
+
(Sodalime)
NaOH/CaO anti-aromatic

+ – – –3Na2CO3 Compound A has 4π-electrons which are also delocalised but do


NaOOC COONa+ not constitute close loop, hence non-aromatic.
Benzene
The reaction given in option (d) is Clemmensen reduction i.e., Compound D is aromatic, characteristically stable.

− Hg –
C == O Zn
 → CH2 + H2O O O
HCl
Hence, the final product of this reaction is also mesitylene which
can be seen as +
CHO CH3 Tropyllium ion, aromatic

Order of stability Aromatic > Non-aromatic > Anti-aromatic


Zn-Hg/HCl
34. Aromatic molecule must.
OHC CHO H3C CH3 (b) have (4 n + 2)π electrons. (c) be planar. (d) be cyclic.
Mesitylene
35. A compound is considered to be aromatic, if it follows three
31. (a) NaOC2H5 rules:
Br
(a) Must be cyclic and planar.
C2H5O− (a strong nucleophile) causes E1 reaction to form (b) Must have conjugated system in it.
isobutene as the major product. (c) It must follow Huckel rule which states that number of
π-electrons = (4 n + 2)
Cl AlCl3 1, 2-hydride
(b) (A) ⇒ 2π-e− system [(4 n + 2)π , n = 0 ] ⇒ Aromatic
shift 3° carbocation
1° carbocation (Major)

(B) ⇒ 4π-e− system [ 4 nπ , n = 1] ⇒ Anti-aromatic

SE Tertiary butyl benzene(X)


(C) ⇒ 8π-e− system [ 4 nπ , n = 2 ] ⇒ Anti-aromatic

+
H
(c) (X)
(D) ⇒ The sp3-carbon does not support conjugation and
sp3 also devoids planarity.
BF3,Et2O
⇒ Non-aromatic
OH +
(d) 36. CH3 CH3

32. CH3 CH3 Fe


O + Br2
OH
+ O2 CH3
+ CH2 — CHCH3 H CH3
Toluene
Br
Cumene hydroperoxide
Cumene — CH3 in toluene is para directing group.It activates the ring by
both inductive and hyperconjugation effect.
Benzene and Alkyl Benzene 437

37. In both Friedel-Craft’s reaction and Reimer-Tiemann reaction, 40.


new carbon-carbon bond is formed :
R (III) Cl (i) KCN CN
AlCl3 Friedel-Craft’s Nucleophilic
+ R—Cl + HCl reaction substitution reaction
C OCH3 (Cl– replaces by CN– ion) C OCH3

OH OH O O
CHO
H+ CH3OH + COOH
+ CHCl3 + OH– (ii) H3O+
H2O ∆

LiAlH4
C OH (Complete hydrolysis
(T) of —CN give —COOH
Reimer-Tiemann and ester also get hydrolysed
reaction O into —COOH and alcohol )

38. Compound (P) on treatment with concentrated H2SO4, gives : CH2 CH2 OH
(iv) conc. H2SO4
OCH3 H2 O +
C C D O
(Q) CH2 OH
Ph Ph H+ Ph (R)
Conc. H2SO4
– CH3OH Hence, correct match of (III) are T , Q , R.
O
Ph Ph

This intermediate compound, on delocalisation, gives a C OMe


(i) LiAlH4
coloured compound Q. (IV) CH2OH
C OMe
CH2
(Q) CH2OH
C O

Ph –H Ph
(ii) conc. H2SO4
ArSE2
(Intramolecular) + H2O
Dehydration
O
(R)
Ph Ph
(Q)
Hence, correct match of IV is Q , R.
Number of rings = 4 + 1 = 5 Hence, correct matching from list-I and list II on the basis of
Number of π-bonds = 4 × 3 + 1 = 13 given option is (IV), Q, R.
Thus, the degree of unsaturation of Q is 18. 41. O

39. CH2 C H
CN (i) DIBAL–H
(i) H2, Pd–BaSO4 (I)
O (ii) Dil. HCl OH
Quinoline (reduction) CH
(Triple bond converts into
double bond) O OH
–H2O

CH2OH (iii) NaBH4 CH


CH2 CH2 (reduction) H
(ii) Dil. KMnO 4
C C (Q ) CH2 OH C
(excess) 273K
OH H H O
H H
(iv) Conc. H2SO4
(dehydration)
C C CH2 OH O
OH (R)
OH OH OH Hence, correct match of (I) are (Q, R)
(P) (Major product)

Compound (P) has total number of hydroxyl groups = 6


438 Benzene and Alkyl Benzene

O Hence, —NO is ortho/para directing although deactivating


group. Therefore, ortho/para nitro derivatives are formed on
(i) O3, (ii) Zn + H2O
CH2 C H treatment with mixed acid.
(II) (Ozonolysis) NO2 NO2
COOH COOH
(S) (P)
Conc. H2SO4
(iii) NaBH4
(ii) + Conc. HNO3
NO2
(iv) Conc. H2SO4 CH2 OH
–H2O
O Nitro group is meta directing as well as strongly deactivating.
C OH
O
(U) O
Hence, correct match of II is (P , S ,U ).
Hence, correct matching from list-I and list-II on the basis of Pd/C
given option is (II), P , S , U . (d)
H2
42. An electron donating substituent in benzene orient ortho/para
substitution.
43. In benzene, carbons are sp2-hybridised only, hence only two
p-orbitals are involved in hybridisation.
In the above hydrogenation reaction, the anti-aromatic character
CH3 CH3 of three cyclobutadiene rings are relieved that provide stability
to the hydrogenation product.
C—Br C—OC2H5
44. (a) (i) + C2H5OH —Br + Ag+(aq) AgBr↓ +
CH3 CH3 +
45.
+ HBr 7-bromo-1,3,5-cycloheptatriene Tropylium ion
Acid (aromatic, stable)

CH3
(ii) Br + C2H5OH No reaction
CH3
—Br + Ag+ + + AgBr
F OH
anti-aromatic
(b) (i) + NaOH + F– 5-bromo-1,3-cyclopentadiene (unstable)
O2N O2N
O
CH3 CH3
Nitro group from para position activate aromatic nucleophilic Br2
substitution. 46. N—
Fe
F deactivated activated
(ii) + NaOH No reaction O
H 3C
CH2NO2 N— —Br

—NO2 is not in resonance with benzene ring, does not activate 47. (i) Oxidation of side-chain alkyl group of benzene occur
the aromatic nucleophilic substitution reaction, hence fluoride is
through free radical mechanism and it initiates at α −C as.
not formed.
 
+ +
N==O N==O N==O Ph — C— H → Ph — C • + H •
 
– In tertiary butyl benzene, there is noα −Η, required to initiate
(c) (i) free radical oxidation reaction, hence not oxidised with
KMnO4.

Benzene and Alkyl Benzene 439

51. CH3 CH3 CH3


CH3
CH3 C—CH3
C—CH3 AlCl3
+ KMnO4 No oxidation + CH3—C—CH2Br CH3
CH3
takes place
H CH3
tert butyl benzene
(no α-H)

(ii) Pi-electrons of double bonds are involved in aromatic +


delocalisation (aromaticity), hence electrophilic addition do
not occur as it would destroy aromatic stability. However,
electrophilic substitution do not destroy aromaticity. CH3 C CH3

E H CH3
E+ Addition
H+ + + HBr 52. In presence of light, free radical reaction takes place at — CH3
Br group while in presence of FeBr3, electrophilic substitution
aromaticity retained electrophilic loss of aromaticity
(occur preferably) substitution (not occur) occur in the ring.
CH3 CH2Br
Br2/heat
CH2CH3 COOH or light
K2Cr2O7
48. (i)
Br2/FeBr3 CH3
CH3
CaO/NaOH
+
Heat
Br
CH2CH3 CH—CH3 Br
Br2 NaCN OCH3 O OCH3
(ii)
heat Br
CH3 53. (i) AlCl3 Zn(Hg)
+ O
(ii) H3O+ HCl
CH—CH3 C—COOH
H+/H2O
O
CN H O COOH
Friedel-Craft’s reaction
NHCOCH3 NHCOCH3 OCH3
Br2/Fe
49. H3PO4
Br Final product
NHCOCH3
+
Br COOH CH3
NHCOCH3 is ortho/para directing although deactivating group. C—CH3
Therefore, Ortho/pare bromoderivatives are fomed or H2SO4
54. (i) + (CH3)2CHCH2OH
treatment. CH3
50. Nitro group is a very strong deactivating group, the very slow H
Friedel-Craft’s reaction does not takes place in presence of nitro
group. CH2CH3 C—CH3
(i) Br2/heat
(ii)
(ii) NaCN CN
440 Benzene and Alkyl Benzene
+ +
55. R — C ≡≡ O ←→ R — C==O CaC2 + 2H2O → Ca(OH)2 + C2H2
I II Acetylene
I is more stable resonance structure of acylium ion than Cu
3C2H2 → C6H6
II on the following grounds. ∆ Benzene
It has more covalent bonds. (ii) —CH3 in toluene is ortho/para directing group, first
It has complete octet of both carbon and oxygen. oxidised to —COOH in order to make it meta directing and
O then nitrated.
HNO3/H2SO4
—C—O— CH3 COOH COOH
56. (i) mononitration

K2Cr2O7 Conc. HNO3


deactivated activated
Conc. H2SO4
ring ring
O NO2

—C—O— —NO2 O

C—CH2CH3
(ii) C6H5COOH + CH3MgI → CH4 + Mg(OOCC6H5 )I AlCl3
61. + CH3CH2COCl
CH3 CH3
57.
AlCl3 + Hydride Friedel-Craft’s acylation
CH3—C—CH2Cl CH3—C—CH2 624.
shift
H
H H
1° carbocation H H
CH3 It has 12 sigma and 3 pi-bonds.
CH3 H H
C—CH3
C6H6
CH3—C—CH3 H
+ CH3
3° carbocation
(more stable) 63. CH3CH2OH + PCl 5 → POCl 3 + HCl + CH3CH2Cl
+ – AlCl 3
C6H6 + CH3CH2Cl → C6H5— CH2CH3
58. — —

Friedel-Craft’s reaction
+
— –
64. H3C— —CH3 + Conc. H2SO4

p-xylene
The above resonance activate an aromatic ring for electrophilic SO3H OH
substitution reaction.
59. Both — CH3 and — OCH3 are activating groups but chloro is KOH
H3C— —CH3 H3C— —CH3
deactivating group in electrophilic aromatic substitution fusion
reaction.
Also — OCH3 activate more than — CH3 due to resonance
65. Pi-bonds of benzene are involved in aromaticity, not open for
effect by the former group. Therefore, the overall order of
electrophilic addition reaction. Rather, it undergo electrophilic
reactivity is:
substitution reaction.
Chlorobenzene < benzene < toluene < methoxy benzene
66. The aromatic systems are
60. (i) Lime is heated with coke at high temperature and then,
hydrolysed to form acetylene. This acetylene on passing red r s
hot tube polymerises to form benzene : r
2000 ° C
CaO + 3C → CaC2 + CO ↑ , , , ,
Lime Coke
29
Aromatic Compounds
Containing Nitrogen
3. The increasing order of the pK b of the following compound is
Objective Questions I (Only one correct option)
F
1. Three isomers A , B and C (molecular formula C8H11N) give the S
following results : (2020 Main, 3 Sep II) (A)
N N
Diazotisation
A andC → H H
CH3O
(i) Hydrolysis
R (product of A ) S
(B)
P + Q → +
(ii) Oxidation S (product of C )
+ N N
(KMnO4 + H )
C6 H5 SO2Cl O2N H H
R has lower boiling point than S . B → S
(C)
alkali-insoluble product A , B and C, respectively are
N N
NH2 CH2NHCH3 CH2CH3
(a) ; ; H H
H2N H3C
CH2CH3
S
(D)
NH2 CH2NH2 NH2
(b) ; ; N N

CH2CH3 H H (2019 Main, 12 April I)


CH3 CH2CH3 (a) (A) < (C) < (D) < (B) (b) (C) < (A) < (D) < (B)
CH2CH3 NHCH2CH3 NH2 (c) (B) < (D) < (A) < (C) (d) (B) < (D) < (C) < (A)
(c) ; ; 4. Which of the following is not a correct method of the preparation
H2N CH2CH3 of benzylamine from cyanobenzene?
CH2CH3 CH2NHCH3 NH2 (a) H2 / Ni (2019 Main, 10 April II)
(d) ; ; (b) (i) HCl / H2O (ii) NaBH4
CH2CH3 (c) (i) LiAlH4 (ii) H3 O+
NH2 (d) (i) SnCl 2 + HCl(gas) (ii) NaBH4

2. Benzene diazonium chloride on reaction with aniline in the 5. Aniline dissolved in dil. HCl is reacted with sodium nitrite at
presence of dilute hydrochloric acid gives (2019 Main, 12 April II) 0°C. This solution was added dropwise to a solution containing
equimolar mixture of aniline and phenol in dil.HCl. The
structure of the major product is (2019 Main, 9 April I)
(a) —NH2

(a) N N NH2
(b) —N==N —
(b) N N O
H2N

(c) —N==N— —NH2 (c) N N OH

(d) —N==N—NH— (d) N N NH


442 Aromatic Compounds Containing Nitrogen

6. The major product obtained in the following reaction is


NH2 OH
(i) CHCl3/KOH
(ii) Pd/C/H2 N
(c) (d)
N N
CN O (2019 Main, 8 April II) N

H H
NCH3 NCH3
(a) (b)
OH

CN O CN OH
9. The increasing order of reactivity of the following compounds
towards reaction with alkyl halides directly is
H H (2019 Main, 12 Jan I)
NCHCl2 NCH3 CN
O O
(c) (d) NH2 NH2
NH2
NH
CN OH H2N OH
O
(A) (B) (C) (D )
7. The major product in the following reaction is
(2019 Main, 8 April II) (a) (A) < (C) < (D) < (B)
NH2 (b) (B) < (A) < (C) < (D)
(c) (B) < (A) < (D) < (C)
N N
+ CH3I Base (d) (A) < (B) < (C) < (D)
N 10. The major product of the following reaction is
H N
(2019 Main, 11 Jan I)
NH2 NH2 O

N + N OEt (i) Ni/H2


NCH3 N
(a) (b) CN (ii) DIBAL- H
N N
H N N
NH N
CH3 (a) (b)
NHCH3 NH2
H
N N N N
(c) (d) O OH
N N + (c) (d)
CHO
H N N NH2
H 11. What will be the major product in the following mononitration
CH3
reaction ? (2019 Main, 10 Jan II)
8. Coupling of benzene diazonium chloride with 1-naphthol in
alkaline medium will give (2019 Main, 8 April I)
OH
N HNO3
H Conc.H2SO4
(a) (b)
O2N
NO2
(a) (b)
N N
N N
H H
N OH N

(c) (d)
O2N N N
H H
NO2
Aromatic Compounds Containing Nitrogen 443

12. An aromatic compound ‘A’ having molecular formula C7H6O2 16. Arrange the following amines in the decreasing order of basicity:
on treating with aqueous ammonia and heating forms (2019 Main, 9 Jan I)
compound ‘B’. The compound ‘B’ on reaction with molecular
bromine and potassium hydroxide provides compound ‘C’ N N N
having molecular formula C6H7N. The structure of ‘A’ is (I)
(2019 Main, 10 Jan II) H H
(II) (III)
OHC
CHO (a) I > II > III (b) III > II > I
(a) (b) (c) I > III > II (d) III > I > II
OH
OH
COOH
17. The correct decreasing order for acid strength is
(2019 Main, 9 Jan I)
(c) (d)
(a) FCH2COOH > NCCH2COOH > NO2CH2COOH > ClCH2COOH
CH==CHCHO
(b) CNCH2COOH > O2NCH2COOH > FCH2COOH > ClCH2COOH
13. The increasing basicity order of the following compounds is (c) NO2CH2COOH > NCCH2COOH > FCH2COOH > ClCH2COOH
CH2CH3 (d) NO2CH2COOH > FCH2COOH > CNCH2COOH > ClCH2COOH
 18. Which of the following compounds will give significant amount
(A) CH3 CH2NH2 (B) CH3 CH2NH of meta-product during mononitration reaction? (2017 Main)
CH3 CH3
OH OCOCH3
 
(C) H3 C  N  CH3 (D) Ph  N  H
(2019 Main, 9 Jan II) (a) (b)
(a) (D) < (C) < (B) < (A) (b) (A) < (B) < (C) < (D)
(c) (A) < (B) < (D) < (C) (d) (D) < (C) < (A) < (B)
NH2 NHCOCH3
14. The tests performed on compound X and their inferences are :
Test Inference (c) (d)
(a) 2, 4- DNP test Coloured precipitate
(b) Iodoform test Yellow precipitate 19. The major product of the following reaction is (2017 Adv.)
(c) Azo-dye test No dye formation OH
Compound ‘X ’ is (i) NaNO2/HCl/0°C
H3C CH3 (ii) (aq. NaOH)
N H 3C CH3
N
COCH3 NH2
(a) CHO
(b) – +
OH ONa
NH2 OH NH2
(a) (b)
CH3 CHO
(c) (d)
(2019 Main, 9 Jan II) N N NH2

15. The major product of the following reaction is N N OH OH

C (c) (d)
NH2 (i) Br2/hν
(ii) KOH (dil.)
CH2CH3 (2019 Main, 9 Jan II) Cl
O 20. In the reaction,
NH NH2
(a) (b) NH
CH3 NaNO2/HCl CuCN/KCN
0.5°C
D

E + N2
CH3
O
CH3
NH NH
(c) (d)
The product E is (2015 Main)
444 Aromatic Compounds Containing Nitrogen

COOH (CH3)2NH
24. F NO2 (A)
DMF,∆
(a) (b) H3C CH3 (i) Fe/HCl
(ii) NaNO2/HCl/0°C
(B), B is
(iii) H2/Ni
CH3
(2003, 1 M)
CN CH3
CH3 (a) H2N N
CH3
(c)
(d)
(b) H2N NH2
CH3
21. Amongst the compounds given, the one that would form a brilliant CH3
coloured dye on treatment with NaNO 2 in dil. HCl followed by (c) O2N N
addition to an alkaline solution of β-naphthol is CH3

N(CH3)2 NHCH3
(a) (b) (d) O2N NH2

NH2 CH2NH2 25. The most unlikely representation of resonance structures of


(c) (d) p-nitrophenoxide ion is (1999, 2M)
_ _ _
H 3C (2011) O + O O + O
N N
22. The species having pyramidal shape is (2010)
(a) SO3 (b) BrF3
(c) SiO2−
3 (d) OSF2 (a) (b)
23. In the following reaction,
O– O
O
O O O– + O
N N N
Conc. HNO3
H X
Conc. H2SO4
(c) (d)
The structure of the major product X is (2007, 3M) –

O O– O
(a)
NO2
N 26. Benzene diazonium chloride on reaction with phenol in
H weakly basic medium gives (1998, 2M)
(a) diphenyl ether (b) p-hydroxy azobenzene
(c) chlorobenzene (d) benzene
O2N
O 27. Examine the following two structures of anilinium ion and
(b)
choose the correct statement from the ones given below :
N (1993, 1M)
H +
NH3 NH3

O
(c)
N I II
H (a) II is not an acceptable canonical structure because
carbonium ions are less stable than ammonium ions
NO2
(b) II is not an acceptable canonical structure because it is
non-aromatic
O (c) II is not an acceptable canonical structure because the
(d)
nitrogen has 10 valence electrons
O2N N
H (d) II is an acceptable canonical structure
Aromatic Compounds Containing Nitrogen 445

28. Chlorobenzene can be prepared by reacting aniline with Br Br Br Br


(a) hydrochloric acid (1984, 1M) Br Br Br Br
(b) cuprous chloride
(c) chlorine in the presence of anhyd AlCl 3
Br
(d) nitrous acid followed by heating with cuprous chloride
Br Br
Objective Questions II (a) (b) (c) (d)
(One or more than one correct option) 32. In the following reactions, the major product W is (2015 Adv.)
OH
29. Consider the following four compounds, I, II, III, and IV. NH2
H3C CH3 NaNO2,HCl , NaOH
H3C CH2 V W
NH2 N NH2 N 0°C

O2N NO2 O2N NO2

OH
(a) (b)
I II N=
=N OH N=
=N
NO2 NO2
III IV

Choose the correct statement(s). (2020 Adv.)


HO
(a) The order of basicity is II > I > III > IV. OH
(b) The magnitude of pK b difference between I and II is more than
that between III and IV. (c) N=
=N (d)
(c) Resonance effect is more in III than in IV. N=
=N
(d) Steric effect makes compound IV more basic than III.

30. Aniline reacts with mixed acid (conc. HNO 3 and


33. In the reaction shown below, the major product(s) formed is/are
conc. H 2SO 4 ) at 288 K to give P (51%),Q (47%) and R (2%). (2014 Adv.)
The major product(s) of the following sequence is (are) NH2
(i) Ac2O, pyridine (i) Sn / HCl
(ii) Br2 / H 2O (excess)
R (ii)
Br2 , CH 3CO 2H
    → S → Acetic
(iii)H 3O + (iii) NaNO 2 , HCl / 273-278 K anhydride
(iv) H 3PO 2 NH2 Product(s)
(iv) NaNO 2 , HCl / 273-278 K
CH2Cl2
(v) EtOH, ∆
O
Major product(s) (2018 Adv.)
N
Br Br H CH3
Br Br O
(a) (b) (a)
Br NH2 + CH3COOH
Br
Br Br O
NH2
Br Br
(b) H
N CH3 + CH3COOH
(c) (d)
Br Br Br Br O O
Br H
N CH3
31. The product(s) of the following reaction sequence is (are)
(2017, Adv.) O
(c) H
NH2 N CH3 + H2O
(i) Acetic anhydride/Pyridine
(ii) KBrO3/Br O O
r s
(iii) H3O+, Heat NH3CH2COO
(iv) NaNO2/HCl, 273-278 K
(v) Cu/HBr
(d) H
N CH3

O O
446 Aromatic Compounds Containing Nitrogen

34. p-chloroaniline and anilinium hydrochloride can be Column I Column II


distinguished by (1998, 2M)
⊕ Gives white
(a) Sandmeyer reaction (b) NaHCO3 (C) HO NH3Cl precipitate with
(c) AgNO3 (d) Carbylamine test (r)
AgNO3
35. When nitrobenzene is treated with Br2 in the presence of FeBr3 , Reacts with aldehydes

the major product formed is m-bromonitrobenzene. Statements O2N NHNH3Br to form the
which are related to obtain the m-isomer, are (1992, 1M) (D) (s) corresponding
(a) the electron density on meta carbon is more than that on ortho NO2 hydrazone derivative
and para positions
(b) the intermediate carbonium ion formed after initial attack of
Numerical Answer Type Questions
Br + at the meta position is less destabilised
(c) loss of aromaticity when Br + attacks at the ortho and
39. Consider the reaction sequence from P to Q shown below.
para positions and not at meta position The overall yield of the major product Q from P is 75%. What
(d) easier loss of H+ to regain aromaticity from meta position than
is the amount in grams of Q obtained from 9.3 mL of P? (Use
from ortho and para positions
. g mL −1 ; Molar mass of C = 12.0, H = 10
density of P = 100 . ,O
= 16.0 and N = 14.0 g mol −1 ) (2020 Adv.)
Assertion and Reason
(i) NaNO2+HCl / 0-5°C
Read the following questions and answer as per the direction given Nh2 Q + NaOH
OH
below : (ii)
(a) Statement I is correct; Statement II is correct; Statement P
II is the correct explanation of Statement I. (iii) CH3CO2H/H2O
(b) Statement I is correct; Statement II is correct; Statement
II is not the correct explanation of Statement I. 40. Schemes 1 and 2 describe the conversion of P to Q and R to S ,
(c) Statement I is correct; Statement II is incorrect. respectively. Scheme 3 describes the synthesis of T from Q
(d) Statement I is incorrect; Statement II is correct. and S . The total number of Br atoms in a molecule of T is
36. Statement I Aniline on reaction with NaNO2/HCl at 0°C .............. (2019 Adv.)
followed by coupling with β-naphthol gives a dark blue coloured Scheme 1
precipitate.
NH2
Statement II The colour of the compound formed in the reaction (i) Br2(excess), H2O
(ii) NaNO2, HCl, 273 K
of aniline with NaNO2/HCl at 0°C followed by coupling with (iii) CuCN/KCN
β-naphthol is due to the extended conjugation (iv) H3O+, ∆ Q
(2008, 3M) (v) SOCl2, pyridine (major)
P
37. Statement I In strongly acidic solution, aniline becomes more
reactive towards electrophilic reagents. Scheme 2
Statement II The amino group being completely protonated in (i) Oleum
(ii) NaOH, ∆
strongly acidic solution, the lone pair of electrons on nitrogen is S
no longer available for resonance. (2001, 1M) (iii) H+
(iv) Br2, CS2, 273 K (major)
R
Match the Column Scheme 3
38. Match the compounds in Column I with their characteristic (i) NaOH, ∆
test(s)/reaction(s) given in Column II. (2008, 6M) S (ii) Q
T
(major)
Column I Column II
Sodium fusion extract Fill in the Blanks
⊕ s of the compound gives
(A) H2N  NH3 Cl (p) 41. The high melting point and insolubility in organic solvents of
Prussian blue colour
with FeSO4 sulphanilic acid are due to its…… structure. (1994, 1M)

42. In an acidic medium, ……… behaves as the strongest base.



O2N NHNH3Br (nitrobenzene, aniline, phenol) (1981, 1M)
(B) Gives positive FeCl 3
(q)
NO2 test
Aromatic Compounds Containing Nitrogen 447

Subjective Questions 50. Complete the following reactions with appropriate reagents
43. Convert O I
NO2 NO2
—NH—C—CH3 —NO2

to I (1994, 4M)
51. Following reaction gives two products. Write the structures of
OH the products.
in not more than four steps. Also mention the reaction conditions Br 2 /Fe
and temperature. (2004, 4M) CH3 CONHC6 H5 → (1998, 2M)
44. Which of the following is more acidic and why? 52. Write the structure of foul smelling compound obtained when
+ + aniline is treated with chloroform in the presence of KOH.
NH3 NH3 (1996, 1M)
53. Complete the following with appropriate structure:
NH2

I NO2 (i ) NaNO2 and HCl at 5° C


F → A
II (ii ) Anisole (1995, 1M)
(2004, 2M) NO2
45. Convert (in not more than 3 steps) : (2003, 2M)
COOH COOH 54. How will you bring about the following conversions?
“Benzamide from nitrobenzene” (1994, 2M)

55. How will you bring about the following conversions?


F
“4-nitrobenzaldehyde from benzene” (1994, 2M)
46. There is a solution of p-hydroxybenzoic acid and p-amino
benzoic acid. Discuss one method by which we can separate
56. Outline a synthesis of p-bromonitrobenzene from benzene in two
them and also write down the confirmatory test of the functional steps. (1993, 2M)
group present. (2003, 4M) 57. Write the structure of the major organic product expected from
47. Write structures of the products A, B, C, D and E in the following the following reaction.
scheme CH3
O
—N + HNO2
CH3
CH2CH2CH3 Cl2/FeCl3 Na-Hg/HCl (1992, 1M)
A B
58. How will you bring about the following conversion?
Cl CH2

CHCH2 ONa+ HNO3/H2SO4
D C “4-nitro aniline to 1, 2, 3-tribromobenzene.” (1990, 2M)
H 2O

H2 /Pd/C
59. Complete the following with appropriate structures: (1986, 1M)
E (2002, 5M)

48. What would be the major product in the following reaction? Base ?
—NH2 + —COCl
O
60. How would you convert aniline into chlorobenzene? (1985, 1M)
Br2/Fe
N— 61. State the conditions under which the following preparation is
(2000, Main, 1M) carried out. “Aniline from benzene” (1983, 1M)

49. How would you bring about the following conversion (in 3 steps)? 62. State the equation for the preparation of following compound.
Aniline → Benzylamine (2000, 3M) Chlorobenzene from aniline (in 2 steps). (1982, 1M)

Answers
1. (a) 2. (c) 3. (c) 4. (b) 25. (c) 26. (b) 27. (c) 28. (d)
5. (a) 6. (d) 7. (*) 8. (c) 29. (c,d) 30. (d) 31. (b) 32. (a)
9. (b) 10. (b) 11. (b) 12. (c) 33. (a) 34. (c,d) 35. (a,b) 36. (d)
13. (d) 14. (b) 15. (a) 16. (d) 37. (d) 38. (A → r, s B → p, q C → p, q, r D → p)
17. (c) 18. (c) 19. (a) 20. (c) 39. (18.60) 40. (4) 41. (zwitter ionic)
21. (c) 22. (d) 23. (b) 24. (a) 42. (Aniline)
Hints & Solutions
1. A , B and C respectively are C6H5SO2Cl act as electrophile then B must be amine.
Secondary amine act as a nucleophile.
NH2 CH2NHCH3
The C6H5SO2Cl act as electrophile and displace Cl.
, In the given option (b), secondary amine is present in the reaction
CH2CH3 takes place as follows :
CH2—NHCH3
CH2CH3 CH3
s –HCl
and + Ph SO+2Cl PhCH2—N
SO2Ph
H2N
Alkali insoluble amine
According to the question, A and C undergoes diazotisation to
give P and Q. So, amine must be primary aromatic amine. Primary and secondary amine forms poorly soluble ppt. on
+ – reaction with PhSO2Cl.
NH2 N ≡ NCl
2. Benzene diazonium chloride on reaction with aniline in the
diazotisation

presence of dilute hydrochloric acid undergoes coupling reaction


CH2CH3
CH2CH3 (P ) and produces p-amino azobenzene. In this reaction, benzene
(A ) diazonium chloride reacts with aniline in which aniline molecule
CH2CH3 CH2CH3 at its para-position is coupled with diazonium salt to give
p-amino azobenzene (yellow dye).
–+
H2N ClN ≡ N
(C ) + –
(Q) N NCl + H NH2
Which on hydrolysis followed by oxidation gives alcohol
Benzene Aniline
derivative. diazonium
salt H+
H N N NH2 + Cl– + H2O
O+ H Hydrolysis is done and X – p-amino azobenzene
Ortho attack the hydrogen and (yellow dye)
bond shift to oxygen
3. Key Idea pK b value is defined as the minus lagarithm of K b
(Para) OH
OH smallar the value of pK b stronger is the base and vice-versa.
CH2CH3 In the given options, +R effect is shown by —CH3 and —OCH3
+ group (  OCH3 > CH3 ). These group increases the electron
density at o and p-positions. Groups such as F and—NO2
(Ortho) CH2CH3 shows –R-effect (  NO2 >  F). These group decreases the
(Para) electron density at o and p- positions.
Increase in electron density at p-position makes the unshared
OH OH electron pair of ‘N ’ more available and decrease in electron
COOH density at p-position makes the unshared electron pair of ‘N ’ less
available. Compound containing —OCH3 group act as strongest
+
base and hence possess lowest value of pK b . So, the correct
Intramolecular
increasing order of pK b in the given compound is
H-bonding COOH O2N F
Intermolecular H-bonding S S
(R ) > >
(S )
N
H N
H N
H N
H
Given, R has low boiling point than S.
( C)
Intramolecular H-bond is weaker than intermolecular H-bond. (A)
It means intramolecular H-bonding show low boiling point and H3C H3CO
S S
here, R is ortho product and S is para product. >
Given, N
H N
H N
H N
H
C H SO Cl (D ) (B)
(B) →
6 5 2
alkali-insoluble product
Aromatic Compounds Containing Nitrogen 449

4. The preparation of benzylamine from cyanobenzene using given In step (i), 1° amine is converted into isocyanide, when reacts
reagents are as follows : with CHCl3 / KOH. This reaction is known as carbylamine
H2/Ni
reaction. This reaction is used to detect primary amine in a
Ph  C= N ∆ Ph  CH2  NH2 compound.
(Cyanobenzene) Benzylamine
In step (ii), isocyanide is reduced to secondary amine and
cyanide is reduced to primary amine, and carbonyl group to
alcoholic group when treated with Pd/C/H2.
(i) LiAlH4 (ii) NaBH4 δ− δ+
(ii) H Or
3 7. N is more electronegative than ‘C’. So, H of N  H bond is more
(i) SnCl2 r acidic than that of C  H bond in the ring.
[Ph  CH==NH2[2 SnCl 62-
(ii) HCl (g) bis (benzaldiminium ) Again, sp2-N is more electronegative (s% = 33.3) than sp3-N
hexachlorostannate (IV) (s% = 25) of the NH2 group.
HCl , H O NaBH sp3
PhCN →
2
PhCOOH →
4
PhCH2OH NH2 NH2
–NH4Cl Benzoic acid Benzyl alcohol
sp2
N sp2 N
Thus, option (b) is incorrect. N Base N

5. Major product of the reaction is N N N N


sp2 sp2 s
Non-aromatic
ring H Aromatic
N N NH2 Both aromatic
ring
s rings
B Apparently
It is obtained by coupling reaction. (Base) sp3
Reaction of aniline with dil. HCl and sodium nitrite at 0°C is δ+ δ– – BH
CH3–I –I
s
shown below :
+–
(SN2 attack)
NH2 N≡≡NCl
NH2

+ NaNO2 + Dil. HCl Note This C C′ N


0°C N
are missing in all
Diazonium salt the option given.
N N
Diazonium salt formed when added to equimolar mixture of
aniline and phenol in dil. HCl then aniline couples with CH3
diazonium salt. Reaction is as follows : (Option-b)

+
8. Coupling of benzene diazonium chloride with 1-naphthol in
+
N≡≡NCl + H2N alkaline medium will give the following coupling reaction.
OH OH
In acidic
H+ medium
PhN2+

—NH2 + HCl OH –
N=
=N

N==N
For an effective coupling, the solution must be so alkaline
that the concentration of diazonium ion is too low. It must not − +
In the presence of OH ion, Ar N 2 exists in equilibrium with an
be so acidic that the concentration of free amine or phenoxide ion
is too low. That is why amines couple fastest in mild unionised compound (Ar  N == N  OH) and salts
s
acidic solution and phenols couple fastest in mild alkaline [ ArN == N+ ]– OH derived from it, which do not couple.
solution. s NaOH
1° NH2 responds [ Ar  N ≡≡ N ]+ OH - Ar  N == N  OH
6. H+
NH2 N≡≡C Does not couple
CHCl3/KOH NaOH − +
(Carbylamine reaction) - Ar N == N O Na
H+ Does not couple

CN O N≡≡C O Following conditions are very useful for coupling reaction to


take place.
Pd/C/H2
(a) The solution must be so alkaline that the concentration of
diazonium ion is too low.
NH—CH3
(b) It must not be so acidic that the concentration of free phenoxide
ion is too low.
9. The reaction of alkyl halide with amine or amides is a
H2N OH nucleophilic substitution reaction. More nucleophilic nitrogen is
more reactive with alkyl halide. Compound (A) is benzamide
and lone pair of ‘N’ of it, is not available in this compound.
450 Aromatic Compounds Containing Nitrogen

s
O O 12. Using retro-synthesis, to get the required compounds, A , B andC.
+ NH 3 / ∆
Br
C NH2 C NH2 C6H7N ←
2  B ← A
(C ) KOH [An amide] [Carboxylic acid
[1°-amine] Hoffmann (No.of C =7) C 7 H 6 O2 ]
(A) (No. of C = 6 bromamide
1 carbon less) reaction
In compound (D), lone pair of ‘N’ are available but in compound (step down)
(C), the lone pair of ‘N’ are not readily available due to the
presence of electron withdrawing group ( CN) attach to So, the reactions can be shown as:
benzene ring. COOH CONH2
NH3/D NH2
In compound (B), i.e. pthalimide, ‘N’ is not nucleophilic due to Br2/KOH
–H2O
the presence of two polar groups attached to it that pulls lone pair –K2CO3
towards them. Hence, the correct order is: (A ) (B) –KBr
[C7H6O2] Benzenamide (C)
( B ) < ( A ) < (C ) < (D ). [C6H7N]
Benzoic
Aniline
10. Ni/H 2 can reduce —C ≡≡ N into CH2  NH2 (1°-amine) but acid

cannot reduce an ester group ( CO2Et) whereas DIBAL-H, 13. If we consider Lewis basicity (basicity in aprotic solvents or in
di-isobutylaluminium hydride, [(CH3 )2CH]2AlH reduces the vapour phase), the order of basicity will be.
ester group ( CO2Et) into —CHO (an aldehyde) and
C2H5 OH. D A B C
2° 1° 2° 3°

Ni/H2/D
But, this order does not match with the options given. So, it has
OEt OEt
C≡N been asked on basicity of the amines in aqueous solution. When
DIBAL–H
NH2 –EtOH no phase is given, then basicity of amine is considered in
aqueous solution as they are liquids. In aqueous solution,
1
2 CH basicity of 2°-amines (aliphatic) is maximum because, of the
N –H2O NH2
thermodynamic stability of its conjugate acid.
3 (Nucleophilic addition ⊕ È
4
elimination reaction)
Et 2 NH + H 2O c Et 2 N H 2 + OH
(B ) Conjugate acid
11. Here, in mononitration the electrophile produced from mixed ⊕
+
acid (HNO3 + conc.H2SO4 ) is N O2. Et 2 N H2 is a sterically symmetric tetrahedral ion as it contains
equal number (two) of bulkier Et-group and small size H-atoms.
O –R Here, two H-atoms give additional stability through hydrogen
A +R bonding with H2O (solvent) molecules.
N Et
B d+ d+
H H d

N d–
O H
Ring-(A) is activated, i.e. becomes more nucleophilic by the + R H
Et
• • H-bonding
effect of the  N H  group and it becomes o / p-directing
Aromatic amines (D) are always weaker bases than aliphatic
towards the electrophile, NO+2 in the ArS E 2 reaction. For amines, because of the conjugation of lp of electrons of N

mononitration, N O2 will preferably come at p-position, whereas (+ R-effect) with the benzene ring.
O So, the correct order is (D) < (C) < (A) < (B).
 14. It is a 3°-amino group (—NMe2) though
the ring-(B) gets deactivated by −R- effect of the  C  group, aromatic which will not respond to
r CH3
O2N N diazotisation (NaNO2 + dil. HCl/5°C).
O r CH3 So, formation of diazonium ion and its
NO2
further reaction with alkaline phenol or
C rds of ArSE2
NH β-napthol at 5-10°C to produce a
precipitate of azo-dye will not take place.
H (b) It is a keto group C==O and so it
O responds to 2, 4-DNP test with Brady’s
O2N CH3 —— C==O reagent (2, 4-dinitrophenyl hydrazine) to
r C
NH Fast,–Hr give a reddish orange precipitate.
O

O2N O It is a keto-methyl group —C—CH3 and


hence, it responds to iodoform test to
C give a yellow precipitate of iodoform
NH (CHI3).
Aromatic Compounds Containing Nitrogen 451

Compound (c) is an alcohol and does not give positive 2, 4-DNP 18. Aniline in presence of nitrating mixture
test. Hence, eliminated (a) and (d) does not have CH3CO (conc. HNO3 + conc. H2 SO4) gives significance amount
group and does not give idoform test. Hence, eliminated. (≈ 47%) of meta-product because in presence of H2SO4 its
15. In the given reaction, protonation takes place and anilinium ion is formed.
Responds to Hoffmann's NH2 NH3HSO4
bromamide degradation and
gets converted into H2SO4
C [NH2 +K2CO3] by Br2/KOH
NH2
Aniline Anilinium ion
CH2 Undergo benzylic free
radical substitution with Here, anilinium ion is strongly deactivating group and
(i) Br2/hν CH3 Br2/hν to give [CHCH3] meta-directing in nature. So, it gives meta-nitration product.
(ii) KOH (dil.)
Br NH2 NH3HSO4
H
N + H Conc.H2SO4
H –Br N
H +Conc.HNO3
Br NO2
CH3
+
Formation of –H 19. Diazo coupling occur at para-position of phenol.
2° carbocation
s OH OH OH
by removing Br NH
(i) NaNO2/HCl/0°C
CH3 (ii) NaOH

16. Key Idea NH2


+
N2Cl– N N
Basicity ∝ Ease of donation of lone pair of electrons
1 1 20. +
∝ ∝ NH2 N2 Cl
-
CN
% of s - character of N Electronegativity of N
NaNO2/HCl ∆
The % of s-character in the given amines are as follows:
0-5°C CuCN/KCN
diazotisation

N N N CH3 CH3
sp2-hybridised sp3-hybridised CH3
[s-character=33.3%] [s-character=25%] 4-methyl aniline D E
H H
sp2-hybridised 4-methyl diazonium chloride 4-methyl benzocyanide
[s-character=33.3%]
Pyridine Pyrrole Piperidine 21. As we know, benzenediazonium salt forms brilliant coloured dye
(I) (II) (III) with β-naphthol, the compound under consideration must be
p-toluidine (c) as it is a primary aromatic amine. Primary
Therefore, piperidine (III) having minimum aromatic amine, on treatment with NaNO2 in dil. HCl forms the
% s-character is most basic. Among the rest, pyridine (I) and corresponding diazonium chloride salt.
pyrrole (II) the lone pair of electrons of N in pyrrole (II) is
NH2 + –
involved in delocalisation and follows (4 n + 2) π aromatic N2 Cl
NaNO2
(n = 1) system. So, the N-atom of pyrrole (II) will show least
HCl
basicity. Thus, the order of basicity is as follows: H 3C H3C
> > OH
N N N

H H β-naphthol
(III) (I) (II) N N CH3

17. All the given compounds are α-monosubstituted acetic acid OH


derivatives and the α-substitutions have been made by strong–I
groups/atoms. More powerful the − I group, stronger will be the
acid. 22. F—S==O S
−I power of different groups is as follows :

F F O
NO2 > CN >  F >  Cl. F
S is sp3-hybridised Pyramidal
Thus, the correct decreasing order for acid strength is:
SO3 is planar (S is sp2-hybridised), BrF3 is T-shaped and SiO2−
3 is
NO2CH2COOH > NCCH2COOH > FCH2COOH 2
planar (Si is sp -hybridised).
> ClCH2COOH.
452 Aromatic Compounds Containing Nitrogen

O (II) is stronger than (I) since (III) is tertiary and (I) primary
23. aromatic amine.
N Conc. HNO3 So, option (a) is incorrect.
I (b) pK b different between I and II is 0.53 and that of III and IV is
Conc. H2SO4
H 4.6. So, option (b) is incorrect.
II O 2N (c) and (d) In 2, 4, 6-trinitro aniline (III) due to strong −R effect
O
of — NO2 groups, the lone pair of — NH2 is more involved
N with benzene ring hence it has least basic strength. Whereas
(IV) N, N-dimethyl 2, 4, 6-trinitro aniline, due to steric
H inhibition to resonance (SIR) effect; the lone pair of nitrogen
is not in the plane of benzene, hence makes it lone pair more
Ring-I is activated while ring-II is deactivated towards
electrophilic aromatic substitution reaction. free to protonate.
Conc. HNO3 +
H3C 30. Given, Aniline   → P + Q + R
(CH3)2NH Conc. H 2SO4
24. F NO2 N— NO2
(51%)
(47%) (2%)
DMF
H3C Then P, Q and R will be
(i) Fe/HCl H3C NH2 NH2 NH2 NH2
N— NH2
(ii) NaNO2 NO2
(iii) H2/Ni H 3C
Conc. HNO3+
+ +
Conc. H2SO4
O O
N NO2
NO2
Nitrogen has five bonds and 10 valence P (51%) Q (47%) R (2%)
25. electrons, not an acceptable resonance structure.
(i) Ac 2O, pyridine
(ii) Br2 , CH 3COOH
O
– Its given R  → S
(iii) H 3O+
+
26. C6H5N2Cl– + C6H5OH (iv) NaNO2 , HCl / 273-278 K
(v) EtOH, ∆
O O
—N==N— —OH
NH2 HN C CH3 HN C CH3
NO2 NO2 NO2
p-hydroxy azobenzene Ac2O Br2
Pyridine CH3COOH
27. In structure II, nitrogen is associated with five bonds and 10
valence electrons, hence not acceptable. (R)
Br
H
H3O +
–CH3COOH
H—N—H (Hydrolysis)

+ + –
N2Cl NH2
NO2 NO2 NO2
EtOH NaNO2/HCl
N-has five bonds ∆ 273-278K
and 10 valence electrons (Diazotisation)
CuCl Br Br Br
28. C6H5NH2 + HNO2 → C6H5N2OH → C6H5  Cl (S)
29. (a) The correct basic strength order is Now from S to major products its given.
(IV) > (II) > (I) > (III); (i) Sn / HCl
(IV) is strongest base due to steric inhibition to resonance (ii) Br 2 / H2O (Excess)
effect. S  
→ Major product
(iii) NaNO 2 , HCl / 273-278K
(III) is weakest base due to −M group of three nitro groups (iv) H3PO 2
present at ortho and para positions.
Aromatic Compounds Containing Nitrogen 453

So, Br CH2—NH2 O O
NO2 NH2 NH2 CH2Cl2
Sn/HCl Br2 + CH3—C—O—C—CH3
O
H2O (Excess)
Br Br C—NH2
CH2—NH—C—CH3
Br Br Br O
(S) NaNO2+HCl
273-278K
Br Br C—NH2

N2+Cl
H3PO2 O
 CH2  NH2 is more nucleophilic than  C  NH2.
Br Br Br Br ||
O
Br Br
Major product Hence, correct choice is (a).
+
1, 3, 4, 5-tetrabromobenzene
34. (C6H5 NH3Cl – ) + AgNO3 → AgCl ↓
Hence, only (d) is the correct answer. anilinium hydrochloride precipitate
NH 2 NHAc No such precipitate is formed with p-chloroaniline.
31.
Also, carbylamine test will not be given by anilinium
Ac2O/Pyridine
hydrochloride but p-chloroaniline give this test.
35. Nitro group withdraws electrons more from ortho/para position
than from meta position. Also the σ-complex formed from meta
KBrO3 + HBr → Br2 attack is less destabilised than from ortho/para positions.
Ac is CH3CO (acetyl), it protects NH2 group from being NO2 NO2 NO2
oxidised.
NHAc NHAc NH2 + +
+ Br+ H H
Br2 water H3O+
+ CH3COOH Br Br
Nitro group not destabilising the positive charge by resonance.

Br Br NO2 NO2 NO2


NH2 N2Cl Br +
+ Br+
NaNO2/HCl CuBr/HBr +
273-278 K (Sandmeyer
(Diazotisation) reaction) Br H Br H
Br Br Br Nitro group destabilises the positive charge directly by
+ resonance, less stable σ-complex.
32. NH2 N2Cl–
Similar phenomenon is observed with ortho attack.
NaNO2 –HCl β-naphthol
0°C NaOH 0°C
36. C6H5NH2 + NaNO2 / HCl → C6H5N+2 Cl –
V N==N––Ph OH
β-naphthol
OH
N==N—C6H5
coloured dye
W
37. In strongly acidic medium, aniline is fully protonated, becomes
33. PLAN This problem includes concept of acetylation reaction and deactivated for SE Ar reaction.
regioselectivity of chemical reaction. +
Regioselectivity means which group will react selectivity in the NH2 NH3
presence of two or more than two functional groups. Here, + H+
among two functional group NH2 and CONH2,
NH2 is more nucleophilic, hence NH 2 group will undergo Lone pair on nitrogen is not available for resonance. Positive
reaction faster than CONH2. charge makes the group strongly electron withdrawing.
454 Aromatic Compounds Containing Nitrogen

38. Sodium fusion extract gives Prussian blue colouration, nitrogen and carbon both present in the compound. Phenolic group and salt of
carboxylic acid gives FeCl 3 test.
Chloride salt gives white precipitate of AgCl on treatment with AgNO3. Hydrazone formation occur effectively at pH = 4.5. The reaction
proceeds in that condition only when H+ concentration is just sufficient to activate the following enolisation.
O OH
 H+ 
 C— l  C+

As H+ concentration rises sufficiently, a large number of molecules of hydrazine gets converted into hydrazinium ion which is not
nucleophilic and reaction becomes impossible. Further low concentration of H+ (in the case of 2,4-dinitrophenyl hydrazinium bromide) is
not effective to proceed elimination.
39. Aniline (P ) is treated with NaNO2 and HCl in cold condition to form benzene diazonium chloride. The process of conversion of primary
aromatic amines into its diazonium salt is called diazotisation.
40. Scheme -1
NH2 NH2 N+2 Cl–
Br Br Br Br
(i) Br2(excess) (ii) NaNO2 +HCl (iii) CuCN/KCN
¾¾¾¾¾¾® –
Cl is displace
H2O 273K
(diazotisation by CN–ion
Aniline
(P ) Br of aniline ) Br
2,4,6-tribromoaniline 2, 4, 6 tribromo
diazonium chloride salt
O

C ¾Cl COOH CN
(v) SOCl2,
Br Br Pyridine Br Br + Br Br
(iv) H3O

®
+ N2
(complete hydrolysis of
¾ CN group which
convert into ¾COOH)
Br Br Br
Major
(Q)

Scheme-2
SO3H SO3Na OH OH OH
+ Br
(i) Oleum (ii) NaOH, D (iii) H Hydrolysis Br2
¾¾¾¾® ¾¾¾¾® ¾¾¾¾¾¾¾® + NaHSO3 ¾¾¾¾® +
H2S2O7 CS2, 273K
(R) (H2SO4+ SO3) (Bromination)
minor
(Sulphonation)
2- bromophenol
Br
major (S)
4- bromophenol
Scheme-3
O
Br
C¾Cl O
Br Br O
OH O–Na+ C
Br Br
(i) NaOH, D (Q) Br
¾¾¾® + NaCl

Br Br Br
( S) (T)
(Major)
T compound has total number of Br atom =4

41. Zwitter ionic + –


H3N— —SO3

sulphanilic acid
Aromatic Compounds Containing Nitrogen 455

NH2 N2+Cl–

HCl + NaNO2
0 – 5°C

β-naphthol couples with phenyldiazonium electrophile to produce an intense orange-red dye (Q ) as major product.
N2+Cl–
OH
OH
0 – 5ºC
+ —N==N— + HCl

β–naphthol
(2-naphthol)
2-naphthol aniline dye
(Orange-red dye)
Given that, volume of aniline (P ) = 9.3 mL
(density of P = 100
. g/mL)
So, mass of aniline = 9.3 g
Molecular mass of aniline (C 6H7N ) = 93 g/mol
Therefore, moles of aniline = 9.3 / 93 = 01
. mol of P.
Molecular mass of 2 napthol aniline orange dye (Q ) = 248 g/mol
⇒ 0.1 mol of aniline (P ) will produce 0.1 mol of compound (Q ).
But, according to the question the major product Q from P is 75%.
Therefore, mass of ‘Q’ produced = (01 . × 248 × 0.75 ) g = 18.60 g
42. Aniline It is a stronger base than either phenol or nitrobenzene.
NO2 NO2 NO2 NO2 NO2

HNO3 NH4HS NaNO2 NaOH


43.
H2SO4 HCl/0°C + H2O
heat NO2 NH2 N2Cl– OH
44. II is more acidic due to − I effect of F.
COOH COOH COOH COOH

HNO3 Sn-HCl NaNO2


45. HBF4, heat
H2SO4
NO2 NH2 F

COOH COOH

46. +

OH NH2
dissolve in diethyl ether + –
HOOC— —NH3Cl(aq)
aq HCl

HO— —COOH
(ether layer)
456 Aromatic Compounds Containing Nitrogen

COOH

FeCl3(aq)
violet colouration

OH
NH2 NC

+ CHCl3 + KOH

COOH COOK
foul smell
O O

C C CH2CH2CH2CH3
CH2CH2CH3 Cl2 CH2CH2CH3A Na-Hg
47. FeCl3 HCl
Cl Cl Cl
Cl Cl
NO2 (A) (B)

CH2CH2CH2CH3 NO2 NO2


HNO3 H2C==CH—CH2ONa
H2SO4 H2/Pd/C
Cl
Cl Cl Cl
(C) O—CH2CH2CH3
O—CH2—CH==CH2
(D) (E)

O O

__ Br2
48. N __ __
Fe N Br

2 NaNO 4CuCN LiAlH


49. C6H5NH2 → C6H5N2+ Cl – → C6H5 — CN → C6H5 — CH2NH2
HCl/0 ° C
NO2
O
NaNO2(aq)
NO2
H+ NaNO2 + I2
50. —NH—C—CH3 —NH2 —N2Cl–
H2O HCl/0°C Heat Fe
I I
O O O

NH—C—CH3 NH—C—CH3 NH—C—CH3


Br2
51. +
Fe
Br Br
52. C6H5NH2 + CHCl 3 + KOH → C6H5NC
Isocyanide
(foul smell)
+
NH2 N2Cl–
NO2 NO2
(i) NaNO2/HCl Anisole
53. N N OCH3
0°C
NO2 NO2
O2N NO2
Aromatic Compounds Containing Nitrogen 457

NO2 Zn-HCl NH2 NaNO CuCN


CN H+/H O COOH NH CONH2
2 2 3
54.
HCl/0°C ∆

Benzamide

CH3 CH3 CHO

AlCl3 Conc. HNO3 CrO3 H+


55. + CH3Cl
Conc. H2SO4 (CH3CO)2O

NO2 NO2
Br Br

Fe HNO3
56. + Br2
H2SO4

NO2
57. No reaction. Tertiary amine does not react with nitrous acid.
NH2 NH2 N2Cl– Br
Br Br NaNO Br Br CuBr Br Br Zn—HCl
Fe 2
58. + Br2 (excess)
HCl/0°C

NO2 NO2 NO2 NO2


Br Br
Br Br NaNO2 H3PO2
Br Br

HCl/0°C

NH2
O
Base
59. NH2 + COCl NH—C—

NaNO2 /HCl CuCl


60. C6H5 — NH2 → C6H5N2+ Cl – → C6H5 — Cl
0°C Heat
Zn – HCl
61. C6H6 + Conc. HNO3 / Conc.H2SO4 → C6H5 — NO2 → C6H5 — NH2
Aniline
HCl CuCl
62. C6H5NH2 + NaNO2 → C6H5N2+ Cl – → C6H5 — Cl
0°C Heat
30
Aryl Halides and Phenols
Objective Questions I (Only one correct option) 3. What will be the major product when m-cresol is reacted with
1. The decreasing order of reactivity of the following compounds propargyl bromide (HC ≡≡ C CH 2Br ) in presence of
towards nucleophilic substitution (S N 2) is (2020 Main, 3 Sep II) K 2CO3 in acetone? (2019 Main, 12 April II)
O OH
CH2Cl CH2Cl
NO2 (a) (b)
(I) (II)
NO2 CH3 CH3
CH2Cl CH2Cl OH OH

(III) (IV) (c) (d)


NO2 O2N NO2 H 3C H3C

NO2

(a) (II) > (III) > (IV) > (I)


4. Increasing rate of S N1reaction in the following compounds is
I I
(b) (IV) > (II) > (III) > (I)
(c) (III) > (II) > (IV) > (I) MeO
(A) (B)
(d) (II) > (III) > (I) > (IV)
2. In the following reaction sequence, structures of A and B, I
respectively will be (2020 Main, 7 Jan II)
(C) (D) I
O
H3C H3CO
HBr Na
∆ A Ether (Intramolecular product) B (2019 Main, 10 April I)
(a) (A) < (B) < (C) < (D) (b) (B) < (A) < (C) < (D)
CH2Br (c) (A) < (B) < (D) < (C) (d) (B) < (A) < (D) < (C)

Br CH2Br 5. p-hydroxybenzophenone upon reaction with bromine in carbon


tetrachloride gives (2019 Main, 9 April II)
OH O
(a) and O
CH2Br O
Br
Br (b)
OH OH (a)
HO
HO
Br O Br O
(b) and
CH2Br Br
(c) (d)
Br Br HO HO

OH 6. The organic compound that gives following qualitative analysis


(c) and is (2019 Main, 9 April I)
O
CH2Br
Test Inference
OH OH (i) Dil. HCl Insoluble
(ii) NaOH solution Soluble
Br
(d) and
(iii) Br2 /water Decolourisation
CH2Br
Aryl Halides and Phenols 459

OH 11. The major product of the following reaction is


NH2 CH3
(a) (b)
OH
(i) aq. NaOH
OH (ii) CH3I
(2019 Main, 10 Jan II)
NH2 CH3 CH3
(c) (d)
OH OH
(a) (b)
7. Which of the following compounds reacts with ethyl CH3 CH3
magnesium bromide and also decolourises bromine water CH3 CH3
solution (2019 Main, 11 Jan II)
OCH3 OH
CN OCH3
(c) (d)
CH2—CO2CH3 CH
(a) (b) CH2 CH3
OH CN 12. The increasing order of the pK a values of the following
compounds is (2019 Main, 10 Jan I)
(c) (d) OH OH OH OH

8. Which of the following compounds will produce a precipitate NO2


with AgNO3 ? (2019 Main, 11 Jan I)
NO2 OMe
Br Br Br A B C D
Br
(a) D < A < C < B (b) B < C < A < D
(a) (b) (c) (d)
(c) C < B < A < D (d) B < C < D < A
N
13. The major product of the following reaction is
9. The major product of the following reaction is O HO
Cl CH3
(i) HBr AlCl3, ∆
O +
(ii) alc.KOH
(2019 Main, 9 Jan II)
O (2019 Main, 11 Jan I)
OH OH
OH Cl Cl
H 3C CH3
(a) (b) (c) (d) O
(a) (b)
O O O
OH
O
10. The major product of the following reaction is OH
OH OH
CH3
Br2 (excess) CH3
(c) (d)
O
O
SO3H (2019 Main, 11 Jan I)
OH OH
14. The products formed in the reaction of cumene with O2
Br
(a) (b) followed by treatment with dil. HCl are (2019 Main, 9 Jan II)
O CH3
Br Br OH
O
Br SO3H
(a) and CH3—OH (b) and H3C CH3
OH OH
Br Br Br Br
(c) (d) OH
OH

(c) and H3C CH3 (d) and H3C CH3


Br SO3H
460 Aryl Halides and Phenols

15. The major product of the following reaction is O O O O


(b) and
O O
(i) KOH (aq.)
Br
(ii) CrO3/H+
Br Br
(iii) H2SO4/∆
(2019 Main, 9 Jan I) O O O O
O O (c) and
O O
(a) (b) Br
OH OH
Br
HO (d) and
O OCH3 OCH3
O
(c) (d) O O
Br
HO
Br 19. For the identification of β-naphthol using dye test, it is necessary
to use (2014 Adv.)
16 The major product of the following reaction is (a) dichloromethane solution of β-naphthol
(i) Br2 (b) acidic solution of β-naphthol
(ii) EtOH (c) neutral solution of β -naphthol
(2019 Main, 9 Jan I)
(d) alkaline solution of β-naphthol
Br OEt
OEt 20. The major product of the following reaction is
(a) (b) H 3C Br

OEt OEt F ⊕
PhS Na
OEt Br
dimethyl formamide
(c) (d)

NO2 (2008, 3M)


17. Phenol on treatment with CO2 in the presence of NaOH followed
by acidification produces compound X as the major product. X H 3C SPh H 3C SPh
on treatment with (CH3CO)2 O in the presence of catalytic amount
of H2SO4 produces: (2018 Main) F F
O (a) (b)
O
O CH3
O CH3 NO2 NO2
(a) (b) H 3C Br H 3C SPh
CO2H
SPh
CO2H
(c) (d) SPh
O CO2H
O CH3
C
O NO2 NO2
(c) OH (d)
21. OH
CO2H
O CH3 –
OC2H5
+ C2H5I
O anhy. C2H5OH
(2003, 1M)
18. Phenol reacts with methyl chloroformate in the presence of (a) C6H5OC2H5 (b) C2H5OC2H5
NaOH to form product A. A reacts with Br 2 to form product (c) C6H5OC6H5 (d) C6H5I
B. A and B are respectively (2018 Main)
22. In the reaction of p-chlorotoluene with KNH2 in liq. NH3 , the
OH Br OH
major product is (1997, 1M)
(a) and
OCH3 OCH3 (a) o-toluidine (b) m-toluidine
(c) p-toluidine (d) p-chloroaniline
O O
Aryl Halides and Phenols 461

23. Phenol reacts with bromine in carbon disulphide at low 28. In the following reaction, the product (s) formed is/are
temperature to give (1988, 1M) OH
(a) m-bromophenol (b) o- and p-bromophenol
(c) p- bromophenol (d) 2, 4, 6-tribromophenol CHCl3
?
24. When phenol is treated with excess of bromine water, it gives OH–
(1984, 1M) (2013 Adv.)
(a) m-bromophenol (b) o-and p-bromophenol CH3
(c) 2, 4-dibromophenol (d) 2, 4, 6-tribromophenol
OH O
Objective Questions II OHC CHO
(One or more than one correct option)
25. Choose the correct option(s) that give(s) an aromatic compound H3C CHCl2
as the major product. (2019 Adv.) CH3 (Q)
(P)
Br
NaOEt
OH OH
(a)
CHO
UV, 500 K
(b) + Cl2 (excess)
H 3C CHCl2
(i) Alc. KOH (R) CH3
(c) H 3C Br (ii) NaNH2
(S)
(iii) Red hot iron tube, 873 K
Br (a) P (major) (b) Q (minor)
NaOMe (c) R (minor) (d) S (major)
(d)
OH

26. The reactivity of compound Z with different halogens under NaOH(aq)/Br2


appropriate conditions is given below 29. In the reaction,
monohalo substituted derivative
OH when X2 = I2 the intermediate(s) is/are (2010)
X2
dihalo substituted derivative s
O O
when X2 = Br2
trihalo substituted derivative Br
C(CH3)3
Z when X2 = Cl 2 (a) (b)

The observed pattern of electrophilic substitution can be explain by Br Br


(a) the steric effect on the halogen Br
(b) the steric effect of the tert-butyl group O
s
O
s
(c) the electronic effect of the phenolic group
(d) the electronic effect of the tert-butyl group
27. The major product(s) of the following reaction is/are (c) (d)
(2013 Adv.) Br
OH OH
Br
Br Br Br Br

Br 30. The ether O—CH2 ,when treated


OH
SO3H Br
Aqueous Br2 with HI produces (1999, 3M)
(3.0 equivalents) (P) (Q)
OH OH
(a) CH2I (b) CH2OH
Br
SO3H
Br Br Br Br
Br SO3H (c) I (d) OH
(R) (S)

(a) P (b) Q (c) R (d) S


462 Aryl Halides and Phenols

31. When phenol is reacted with CHCl 3 and NaOH followed Passage Based Problems
by acidification, salicylaldehyde is formed. Which of the
Reimer-Tiemann reaction introduces an aldehyde group, on to the aromatic
following species are involved in the above mentioned
ring of phenol, ortho to the hydroxyl group. This reaction involves
reaction as intermediates ? (1995, 2M)
electrophilic aromatic substitution. This is a general method for the synthesis
O OH of substituted salicylaldehydes as depicted below.
H CHCl2 OH ONa OH

(a) CCl2 (b)   CHO  CHO
[I ] aq. HCl

O O–
  
H CHCl2 CH3 CH3 CH3
(I) (III)
(c) CHCl (d) (II)

OH 37. The structure of the intermediate I is


ONa ONa
32. Aryl halides are less reactive towards nucleophilic
substitution reaction as compared to alkyl halide due to CH2Cl CHCl2
(1990, 1M)
(a) the formation of less stable carbonium ion (a) (b)
(b) resonance stabilisation
(c) longer carbon halogen bond
(d) sp2-hybridised carbon bonded to halogen
CH3 CH3
ONa ONa
Assertion and Reason CCl3 CH2OH
Read the following questions and answer as per the (c) (d)
direction given below :
(a) Statement I is correct; Statement II is correct
Statement II is the correct explanation of Statement I CH3 CH3
(b) Statement I is correct; Statement II is correct
38. The electrophile in this reaction is
Statement II is not the correct explanation of
(a) •• CHCl (b) + CHCl 2 (c) •• CCl 2 (d) • CCl 3
Statement I
(c) Statement I is correct; Statement II is incorrect 39. Which one of the following reagents is used in the above reaction ?
(d) Statement I is incorrect; Statement II is correct (a) aq NaOH + CH3Cl (b) aq NaOH + CH2Cl 2
(c) aq NaOH + CHCl 3 (d) aq NaOH + CCl 4
33. Statement I Bromobenzene, upon reaction with Br2/Fe
gives 1,4-dibromobenzene as the major product. Integer Answer Type Questions
Statement II In bromobenzene, the inductive effect of the 40. An organic compound (C8 H10O2 ) rotates plane-polarised light. It
bromo group is more dominant than the mesomeric effect produces pink color with neutral FeCl 3 solution. What is the total
in directing the incoming electrophile. (2008, 3M) number of all the possible isomers for this compound? (2020 Adv.)
34. Statement I Phenol is more reactive than benzene towards 41. The number of resonance structures for N is (2015, Adv.)
electrophilic substitution reaction. OH
Statement II In the case of phenol, the intermediate NaOH
N
carbocation is more resonance stabilised. (2000, M)

35. Statement I Benzonitrile is prepared by the reaction of Fill in the Blanks


chlorobenzene with potassium cyanide.
42. Amongst the three isomers of nitrophenol, the one that is least soluble
Statement II Cyanide (CN– ) is a strong nucleophile. in water is ……… (1992, 1M)
(1998, 2M)
43. Phenol is acidic due to resonance stabilisation of its conjugate base,
36. Statement I Aryl halides undergo nucleophilic namely …… (1990, 1M)
substitution with ease.
44. Formation of phenol from chlorobenzene is an example of ………
Statement II The carbon halogen bond in aryl halides has aromatic substitution. (1989, 1M)
partial double bond character. (1991, 2M)
45. The acidity of phenol is due to the ……… of its anion. (1984, 1M)
Aryl Halides and Phenols 463

Subjective Questions 50. Complete the following, giving the structures of the principal
organic products
46. Carry out the following conversions.
(i) Phenol to aspirin heat
Me I + Cu ----
(ii) Benzoic acid to meta-fluorobenzoic acid in not more than
three steps. (2003)

47. How would you synthesise 4-methoxyphenol from 51. How will you prepare m-bromoiodobenzene from benzene
bromobenzene in not more than five steps? State clearly the (in not more than 5-7 steps) ? (1996, 2M)
reagents used in each step and show the structures of the 52. Explain the following in one or two sentences only :
intermediate compounds in your synthetic scheme. (2001, 5M)
“Phenol is an acid, but it does not react with sodium
48. What would be the major product in the following reaction? bicarbonate.” (1987, 1M)

F 53. Complete the following with appropriate structures :


OH OH
NaOCH3
∆ CHO
?

NO2 (1986, 1M)

54. A compound of molecular formula C7 H8 O is insoluble in water


49. Explain briefly the formation of the products giving the
and dilute sodium bicarbonate but dissolve in dilute NaOH
structures of the intermediates.
solution and gives a characteristic colour with FeCl 3. On
treatment with bromine water, it readily gives a precipitate of
NaNH2 C7 H5OBr3. Write down the structure of the compound.
—OMe —OMe (1985, 2M)
NH3
55. Give reason in one or two sentences for the following :

Br “o-nitrophenol is steam volatile whereas p-nitrophenol is not.”


H 2N (1985, 1M)

Answers
1. (a) 2. (d) 3. (a) 4. (b) 29. (b,c) 30. (a, d) 31. (a,d) 32. (b,d)
5. (c) 6. (b) 7. (c) 8. (a) 33. (c) 34. (a) 35. (d) 36. (d)
9. (c) 10. (c) 11. (c) 12. (b) 37. (b) 38. (c) 39. (c) 40. (6)
13. (c) 14. (b) 15. (a) 16. (d) 41. (9) 42. (ortho-nitrophenol)
17. (a) 18. (c) 19. (d) 20. (a) 43. (phenoxide ion) 44. (nucleophilic) 45.(phenoxide
21. (a) 22. (b) 23. (c) 24. (d) ion)
25. (c,d) 26. (a,b,c) 27. (b) 28. (b,d)

Hints & Solutions


1 δ+ δ+
1. Rate of S N2 reaction ∝ CH2Cl CH2Cl
Stability of carbocation
1
Stability of carbocation ∝ (effective group)
−I / −M
δ+ δ+ NO2 NO2 NO2
CH2 Cl CH2Cl
NO2 –I-effect
NO2 –M-effect (IV)
–I-effect
(III)
NO2 So, the decreasing order of reactivity of the given compounds
–I-effect –M-effect
(I) –I-effect toward S N2 reaction is
(II) (II) > (III) > (IV) > (I)
(Distance dominating)
464 Aryl Halides and Phenols

2. The given reaction can be completed as follows : I CH3


r ⇒ Highly
O OH –I
s
Br stable
rds
HBr
CH3O CH3O
(D)
Additional
CH2Br CH2Br +R effect from p-position
OH
(ether + HBr reaction) (DN)

Na/ether 5. p-hydroxy benzophenone upon reaction with bromine in carbon


(intramolecular
Wurtz reaction) tetrachloride gives 3-bromo-4-hydroxy benzophenone.
O O
3. The major product when m-cresol reacts with propargyl bromide Br
Br2
(HC ≡≡ C CH2Br) in presence of K 2CO3 in acetone is given CCl4
in the following reaction: HO
HO
OH O–
Step 2
OH group attached on the benzene ring direct the incoming
Step 1 group at ortho and para-positions due to increase in electron
1. K2CO3 Br CH2 C CH
density at o and p-positions. OH group also exhibit –I group
–Brs
that reduces the electron density to some extent at o and
O CH2 C CH p-positions. But overall electron density increases at these
positions of the ring due to resonance. Hence, attack of —Br
occur at ortho position. Resonating structures are as follows:
OH +OH +OH

Major product
s
In step 1 K 2CO3 act as a base and abstract H-atom from OH
group. This leads to the formation of substituted phenoxide ion s
(highly stable).
+OH OH
In step 2 substituted phenoxide ion on reaction with
Br CH2 C ≡≡ CH gives the required product.
s
4. More stable the carbocation intermediate, higher will be the rate
of S N1 reaction.
The reaction involving carbocation intermediate formation for
the given compounds are as follows: 6. Phenol (ArOH) is insoluble in dil. HCl and readily soluble in
Three positive NaOH solution. It reacts with Br2/water to give 2, 4,
I CH3 hyperconjugation 6-tribromophenol. It readily decolourises the yellow colour of
r
2°-benzyl carbocation Br2 water. Reactions involved are as follows :
–I s
⇒ Stable OH
rds

(A) Dil. HCl


+R · no reaction
(AN)
I CH3 Phenol
(Acidic in nature)
MeO MeO r ⇒ Less – +
–I
s
stable OH ONa
rds

(B ) –I effect · NaOH
from m-position
(BN)
Phenol Sodium
I CH3 (Acidic in nature) phenoxide

r ⇒ More stable OH OH
s
–I
Br Br
rds Br2 Water
H3 C H3C
(C ) ·
Additional
three hyperconjugation Phenol
(C ′) Br
White ppts.
(2, 4, 6-tribromophenol)
Aryl Halides and Phenols 465

7. Ethyl magnesium bromide is a Grignard reagent (GR), it


⊕ 9. In presence of HBr, reactant containing C==C undergoes
constitutes C2H5– [ C2H5– M gBr in ether/aprotic medium] which electrophilic addition reaction and give substituted alkyl halide.
can act as nucleophile as well as strong base. Bromine water On further reaction with alc. KOH, α ,β-elimination takes place
(Br2/H2O, red) gets decolourised with phenol derivatives (option, that give corresponding diene. The diene undergoes enolisation
c), anisole derivatives (option, b) etc., as C==C is present to give stable product (phenol).
outside the ring (aliphatic, not aromatic).
Cl –I effect at a-position to C==C Cl H
δ+ δ–
C ≡≡ N Et group can react
HBr
—Br Acidic
β–H
δ+ δ–
(a) CH2—C—OMe Carbocation formation in the H
electrophilic addition of HBr
δ– –HCl Alc. KOH
–I-effect at b-position to C==C
(Br2/H2O does not react) –HBr (α, β–elimination)
OCH3 Br
Enolisation to H Conjugated
H attain aromaticity diene, stable
OCH3 Br2/H2O H
H
(excess) Br OH
[Very stable, phenol]
(b)
EtMgBr OH
No reaction
OH Br
10. In ipso-substitution takes place with the carbon bearing
Acidic 'H' H
Br2/H2O
H
(excess)
Br SO3H
OH
SO 3H group. After the attack of the electrophilic Br + in the
O Mg Br
rate determining step (rds) of the ArS E2 pathway desulphonation
(c)
Et MgBr ( SO 3 ) takes place with a faster rate.
+ C2H6
[Et acts as
a base] (+R) O—H O—H OH
δ+ δ– δ– (rds) (fast)
N ≡≡ C Br2/H2O –SO3
(Br ) –H
δ+
(d) Br—Br S Br
SO3H Br
—Br
Et group can react but (Br2/H2O) does not react H
− OH
8. Only ionic halides (X ) give precipitate of AgX with AgNO3 (i) 2 moles of Br2
Br Br
solution. So, an organic bromide able to produce R ⊕ (stable (ii) Attack at ortho- 1
carbocation) and Br − in aqueous solution will give precipitate of and ortho-2 by Br+
(iii) –2HBr
AgBr with AgNO3.
Br
(+ R-effect) A white ppt. of
(a) —Br + Br 2,4,6-tribromophenol

(Bond energy Unstable 11. Substituted phenols react with aq.NaOH to form sodium
also increases) (Aryl carbocation) phenoxides which on reaction with CH3 I undergoes S N 2
s reaction to give 2-methoxy-1-methyl benzene.
(b) —Br + Br
CH3 CH3 CH3
s r
Stable carbocation OH O Na d+ d– OCH3
(Aromatic, 6π system) NaOH (aq) CH3 I
s Acid-base reaction SN2 reaction
(c) —Br + Br –H2O
–NaI 2-methoxy-1-
(Unstable carbocation) methyl benzene
(+R-effect)
(d) —Br + Br
s 12. Acidic strength is inversely proportional to pK a value. The
N N acidity of phenols is due to greater resonance stabilisation of
Unstable phenoxide ion relative to phenol. Therefore, any substituent
(Aryl carbocation)
Br which stabilises the phenoxide ion more by dispersal of negative
charge will tend to increase the acidity of phenol. Electron
withdrawing groups ( NO2 ) increases the acidic strength of
So, only produces a precipitate of AgBr with AgNO3 solution.
phenol whereas electron donating group ( OCH3 ) decreases
the acidic strength of phenol. In case of NO2 group attached to
466 Aryl Halides and Phenols

phenol, the dispersal of negative charge is more pronounced at o- 15. Key Idea The reaction involves hydrolysis or nucleophilic
and p-position than at m-position. substitution in first step followed by oxidation and dehydration
Thus, order of acidic strength of nitrophenol is: in last step. The most important fact is that, the Br group
p-nitrophenol > o-nitrophenol and the correct order of the pK a attached directly to aromatic ring will not undergo substitution
values of give option is in step 1.
OH OH OH OH The road map of the given reaction is as follows:

< < < aq. KOH CrO3/H+


Br OH
SN2
NO2
NO2 OMe Br Br
(B) (C) (A) (D) It is aryl halide, remains unaffected
under ordinary SN2 reaction conditions

13. It is an aromatic electrophilic substitution reaction (ArS E 2). O


The reaction follows Ar S E2 (Aromatic electrophlic substitution O
H OH H2SO4/∆
pathway) as shown below :4 –H2O
(Intramolecular
it makes ring activating and p-directing Br dehydration) Br
It becomes ring activating
and m-directing 16. The road map of the given reaction is :
⊕s s⊕
Br Br
H—O HO—AlCl3 HO—AlCl3 Isolated C == C
*
Br2 /CCl4
H3C *
s ⊕ Anti-addition (electrophilic)
AlCl3 ⊕ –H /+H of Br2 with the isolated
Friedel-Craft's O (non-aromatic) C==C
EtOH (Polar protic
acetylation –
medium as well
O reaction SN1,–Br as nucleophile)
O 2°-benzyl
OEt carbocation (Stable)
O Stabilises the complex by ⊕
hyperconjugation * Br Br
EtOH
* ⊕
*
–H
⊕ s
s O—AlCl3 Racemised ± as well as
Os O inverted (+ and –)

−AlCl3 ⊕ OH
O 17. Followed by
O –H2O O + CO2 + NaOH acidification X
H (CH3CO)2O
conc. H2SO4
HO (Catalytic amount)
⊕ CH H2O
3 ?
OH O The very first reaction in the above road map looks like Kolbe’s
reaction which results to salicylic acid as
O O OH OH
(i) CO2, NaOH
H (ii) Acidification
COOH
Salicylic acid
(X)
14. The given process is cumene process (Hock process) to
synthesise phenol and acetone industrially. The salicylic acid with acetic anhydride [(CH3CO)2 O] in the
In Hock p rocess, Ph — group migrate and release H2O. presence of catalytic amount of conc. H2SO4 undergoes
acylation to produce aspirin as

+ H O
—O—O—H —O—O O
H
CH3 C O
O2 hν H+ Ph-group O
5 atm migration OH + CH3 C
OCCH3
Cumene Conc.
H 2O O H2SO4
+ CH3COOH
OH O COOH
COOH
OH Acetyl salicylic
Hydrolysis acid (Aspirin)
+
O
Aspirin is a non-narcotic analgesic (Pain killer).
Aryl Halides and Phenols 467

18. Given, Thus, formation of dye can be shown as


N NPh
OH OH OH
NaOH
+ Methyl A [Ph—N N]Cl–
chloroformate Br2 Alkaline solution

B
Thus, (d) is the correct choice.
In the above road map, first reaction appears as acid base
H3C Br H 3C SPh
reaction followed by SN AE (Nucleophilic substitution through
Addition and Elimination). Both the steps are shown below
(i) Acid base reaction 20. F F
PhS–

OH O DMF

OH
+ H2O
NO2 NO2
(ii) S N AE S N 2 reaction bring about inversion of configuration.
O
O 21. C6H5OH + C2H5O– → C6H5O–

O O C O CH3
Cl C OCH3 C2H5I
→ C6H5— OC2H5

CH3 CH3
In the product of SN AE the attached group is ortho and
para-directing due to following cross conjugation
liq. NH3
O 22. + KNH2 (Benzyne mechanism)
δ–
O1 C O 2 CH3
NH2
6
5 1
4 2 Cl
δ– 3 δ–
OH OH
Cross conjugation due to which lone pair of oxygen 1 will be CS2
23. + Br2
easily available to ring resulting to higher electron density at 2, 4, 1000 temp.
Br
6 position with respect to group. However from the stability
point of view ortho positions are not preferred by substituents as In carbon disulphide, no phenoxide ion exist, therefore only
group  O  C  O  CH3 is bulky. monobromination takes place.
|| OH OH
O
Hence, on further bromination of SN AE product para bromo Br Br
derivative will be the preferred product i.e. 24. + Br2–H2O
O C O CH3 O C O CH3
O O Br
+ Br2 + HBr precipitate
Br2 + H2O → HBrO + HBr
Br It is a reversible reaction, but equilibrium is significantly shifted
to left, also indicated as Br2 (aq).
19. PLAN This problem can be solved by using the concept of synthesis of
dye using electrophilic aromatic substitution reaction. 25. Key Idea An aromatic compound must be cyclic and planar. It
In basic (alkaline) solution naphthol exists as naphthoxide ion must follow (4 n + 2 )e− rule and have the conjugated system in it.
which is a strong o, p-directing group. Br OEt
NaOEt
OH O (a) +
KOH
Substitution product Elimination product

Dimerise
III
−δ −δ O (Non-aromatic)
−δ

−δ
468 Aryl Halides and Phenols

(b) Benzene react withCl 2 (excess) in presence of UV light and 500 28. PLAN Phenolic compounds in alkaline solution react with
K of temperature to form benzene hexachloride (non-aromatic). chloroform (CHCl 3) at a temperature lower than that of CHCl 3
Cl to form ortho-isomer as the major product (due to greater
stability resulting from intramolecular hydrogen bonding).
Cl Cl
– s –Cl –
UV HO + H—CCl a H O  CCl CCl2
+ Cl2 (excess) 500 K
3 2 3
dichloro
Cl Cl carbene

Cl Os O OH
(Non-aromatic)
H CHCl2
26. PLAN This problem includes concept of effect of steric and electronic s +
effect on reactivity of organic compounds.
+ : CCl2 CCl2 +H +
–H
Steric effect of halogens are as follows Cl 2 < Br2 < I2
Electronic effect of phenolic group directs the approaching CH3 CH3 CH3
electrophile towards ortho and para positions. Tertiary butyl
group has large size so it causes steric effect around aromatic H
OH O O
nucleus. On the basis of above factors the products of the given
reactions are as follows: CHO C
OH OH– H
I or
X2 = I2

C(CH3)3 CH3 CH3


(S)
OH OH
X2 = Br2
Br Major as stable due to intramolecular H-bonding.

Oσ O O O
C(CH3)3 C(CH3)3
Br :CCl2 H 2O
OH
X2 = Cl2 Cl Cl σ
σ
CH3 CH3 CH3 CCl2 CH3 CHCl2
C(CH3)3 Q (minor)
Cl
Thus, (b) and (d) are correct.
Hence, orientation in electrophilic substitution reaction is decided by
(a) The steric effect of the halogen 29. — OH in phenol is ortho/para directing group.
(b) The steric effect of the tert-butyl group
O– O– O
(c) The electronic effect of the phenolic group
So, (a), (b) and (c) are correct choices. Br2 s

27. PLAN OH group is activating group and is o- and p-directing.


Also, SO3H is a better leaving group and is knocked out by Br − .
Br Br
OH OH
O O
Br2 water Br Br
(3 equivalents) Br2 water
Br2


SO3H SO3H Br Br Br
ortho-attack
OH OH O O– O– O–
Br Br
Br Br Br
+ SO3 + +
rO

Br S —O—H Br Br Br
Br
O (major)
para-attack
Aryl Halides and Phenols 469

30. Compound (C8 H10O2 ) also rotate plane polarised light means
—O—CH2— + HI —OH + compound is an optically active and chiral carbon atom is
present.
So, the possible structures which are optically active and have
phenolic group are as followed:
—CH2I
OH
OH OH OH
31. – CH—CH3
O O
*
H *
+ CCl2 CCl CH—CH3
– 2 (d+l)=2 *CH
dichlorocarbene OH
(d+l)=2
O– HO CH3
(d+l)=2
CHCl2
Therefore, total optically active isomers will be 6.
41. OH sO s O
NaOH
32. +
X X
N (I) (II)

s O s O O O

Above resonance makes X- a poor leaving group. Also, the s


carbon bearing X is sp2- hybridised.
s
(VI) (V) (IV)
(III)
33. Bromo group is deactivating due to dominance of inductive s s
effect over resonance effect. However, orientation is determined O O O
by mesomeric effet of —Br.
34. + s
OH OH O—H VII IX (VIII)

+ H H
All the above shown nine resonance structures are different.
+ E + E E
42. Ortho-nitrophenol : Due to intramolecular H-bonding.
resonance stabilisation O
of intermediate carbocation H Intramolecular H-bonding in
ortho-nitrophenol
35. Statement I is incorrect, aryl halides do not undergo nucleophilic + O–
N
substitution reaction with ease. Cyanide ion (CN − ) is a strong
nucleophile. O
36. Statement I is incorrect, aryl halides do not usually undergo 43. Phenoxide ion 44. Nucleophilic
nucleophilic substitution with ease. Statement II is correct, 45. Phenoxide ion :
resonance introduces partial double bond character to C—X
bond. OH O–

O O ONa r H+ +
H CHCl2
37. – Resonance stabilised
+ CCl2 CCl2 conjugate base
OH O– OH
CH3 CH3 CH3 COO– COOH
Intermediate NaOH H+
46. (i)
38. Dichlorocarbene is the electrophile as shown above. CO2

39. CHCl 3 + NaOH → • CCl 2 (Dichlorocarbene) Kolbe’s reaction OCOCH3
electrophile
COOH
(CH3CO)2O
40. Phenolic (−OH) group gives positive test with neutral FeCl 3
solution, means organic compound (C8 H10O2 ) is a phenol Heat
derivative. Aspirin
470 Aryl Halides and Phenols

COOH COOH COOH


NO2
HNO3 Sn/HCl
(ii)
H2SO4 51. + Conc. HNO3/conc.H2SO4
NO2 NH2
COOH
NO2 NH2
NaNO2/HBF4
Br2 SnCl2 NaNO2

F Fe HCl HCl/0°C
meta-fluorobenzoic acid Br Br
Br Br OCH3
N2+Cl– I
HNO3 CH3ONa Zn/HCl
47. KI(aq)
H2SO4 Heat
CuI
Br Br
NO2 NO2 m-bromoiodo-benzene

OCH3 OCH3 OCH3 52. Phenol is weaker acid than carbonic acid.
OH OH
NaNO2/HCl H2O
CHO
0°C NaOH H+
53. + CHCl3 + NaOH
H2O
+
NH2 N2Cl– OH
4-methoxyphenol Reimer-Tiemann reaction

54. The compound must contain a hydroxy group on the ring with all
48. F OCH3
three ortho/para positions vacant :
CH3ONa OH OH
+ NaF Br Br
heat
+ Br2—H2O
NO2 NO2 CH3 CH3
Nucleophilic aromatic substitution occur which is assisted by Br
electron withdrawing —NO2 group from para position.
55. Intramolecular H-bonding in ortho-nitrophenol lowers its
49. OCH3 OCH3 OCH3 boiling point. No such intramolecular H-bonding is possible
Br with p-nitrophenol and rather it is associated together by
– NH3 NH–2 –
intermolecular H-bonding which increases the boiling point.
+ NH2
NH2 O
Benzyne more stable
H
carbanion is formed + O–
N
2Cu
50. 2 Me— —I
Heat O
ortho-nitrophenol

CH3— — —CH3 +2CuI

Ullmann’s reaction
31
Aromatic Aldehydes,
Ketones and Acids
Objective Questions I (Only one correct option) 4. The major product of the following reaction is
(2019 Main, 12 April I)
1. The correct electronic configuration and spin-only magnetic
moment (BM) of Gd 3+ ( Z = 64 ), respectively, are
(i) CrO3
(2020 Main, 5 Sep I) HO
7 7 (ii) SOCl2/∆
(a) [Xe] 4 f and 7.9 (b) [Xe] 5 f and 7.9 (iii) ∆
(c) [Xe] 5 f 7 and 8.9 (d) [Xe] 4 f 7 and 8.9 HO
2. The correct match between Item-I (starting material) and O
Item-II (reagent) for the preparation of benzaldehyde is
(2020 Main, 6 Sep II) O

(a) (b)
Item - I Item - II
I Benzene (P) HCl and SnCl 2 , H3 O+ HO
HO
II Benzonitrile (Q) H2 , Pd-BaSO4 , S and
quinoline O
III Benzoyl chloride (R) CO, HCl and AlCl3
O
(a) (I) - (Q), (II) - (R) and (III) - (P) (c) (d)
(b) (I) - (P), (II) - (Q) and (III) - (R)
(c) (I) - (R), (II) - (P) and (III) - (Q) Cl
(d) (I) - (R), (II) - (Q) and (III) - (P) Cl
3. The major products of the following reaction are 5. Compound A(C9 H10 O) shows positive iodoform test.
(2019 Main, 12 April I)
OH Oxidation of A with KMnO4 / KOH gives acid B (C8 H6 O4 ).
Anhydride of B is used for the preparation of
(1) CHCl3/aq. NaOH phenolphthalein. Compound A is (2019 Main, 10 April II)
(2) HCHO, NaOH (conc.) O
(3) H3O+
CH2 C H CH3
Cl
OH OH (a) (b)
COOH COOH CH3
(a) and methanol (b) and methanol
O CH3

Cl OH CH3 CH3

OH OH (c) (d)
CH3
O
OH OH
(c) (d) O
and formic acid and formic acid

OH Cl
472 Aromatic Aldehydes, Ketones and Acids

6. The major product Y in the following reaction is 9. The major product of the following reaction is
(2019 Main, 10 April II)
CN
Ph CH3
(i) SOCl2 (i) DIBAL-H
NaOCl
X (ii) Aniline
Y O (ii) H3O+
O
O O
(2019 Main, 12 Jan I)
Ph CHO CHO
N HN Ph (a) (b)
O O O
(a) (b)
O
CH NH CHO
NH2 NH2
O (c) (d)
(c) (d) OH OH
Ph OH
CHO

O Ph 10. The major product obtained in the following conversion is


(2019 Main, 11 Jan II)
7. Major products of the following reaction are CH3
(2019 Main, 10 April I)
CHO O Br2 (1 eqv.)
(i) 50% NaOH MeOH
+ HCHO
(ii) H3 O+

(a) CH3OH and HCO2H


O CH3 O
COOH CH3
O O OMe
(b) CH3OH and Br
(a) (b)
CH2OH OMe Br
(c) HCOOH and O O

CH3 O CH3 O
CH2OH COOH Br
(d) and O O

(c) (d)
Br
8. The major product of the following reaction is
(2019 Main, 12 Jan II) O O
O
11. The major product of the following reaction is
NaBH4 (2019 Main, 11 Jan I)
EtOH
COCH3
O (i) KMnO4/KOH, ∆
OH
(ii) H2SO4 (dil.)
CH3
(a) (b)
COOH COCH3
OH
(a) (b)
OH
HOOC HOOC
(c) (d) COOH COCOOH
(c) (d)
OEt
OHC HOOC
Aromatic Aldehydes, Ketones and Acids 473

12. The major product formed in the following reaction is OCOCH3 OH

O CH3 —COOH —COCH3


(a) (b)

H 3C H +
Dil. NaOH
COCH3
(2019 Main, 9 Jan II)
OH OCOCH3
O OH
—COOCH3
(c) (d)
(a) H3C —COOH

OH O 16. Compound ( A ) , C8 H9 Br gives a white precipitate when


warmed with alcoholic AgNO 3 . Oxidation of (A) gives an
(b) H3C acid (B), C8 H6 O4 . (B) easily forms anhydride on heating.
Identify the compound (A). (2013 Main)

OH O CH2Br C2H5
(a) (b)
(c) H3C H Br
O OH CH3
CH2Br
(d) H CH2Br
H 3C
(c) (d)
CH3
13. The major product obtained in the following reaction is CH3
O
O 17. Which of the following reactants on reaction with
DIBAL-H conc. NaOH followed by acidification gives following
lactone as the main product ? (2006, 5M)
COOH (2017 Main) O
OH OH
O
(a) CHO (b) CHO

COOH CHO
COOCH3 COOH
(a) (b)
(c) CHO (d) CHO
COOH CHO
COOH CHO CHO COOH
14. In the following sequence of reaction, (c) (d)
KMnO 4 SOCl 2 H 2 / Pd CHO COOH
Toluene → A → B → C
BaSO 4 (2015 Main) 18. 4-methyl benzene sulphonic acid reacts with sodium acetate
The product C is to give (2005, 1M)
(a) C6H5COOH
CH3 COONa
(b) C6H5CH3
(c) C6H5CH2OH
(a) ; CH3CO2H (b) ; SO3
(d) C6H5CHO
15. Sodium phenoxide when heated with CO2 under pressure at
SO3Na CH3
125°C yields a product which on acetylation produces C.
OCOCH3 SO3H
125 ° +
→ B H
ONa + CO2   → C
5 atm Ac2O
(c) ; SO3 (d) ; NaOH

The major product C would be (2014 Main)


CH3 CH3
474 Aromatic Aldehydes, Ketones and Acids

Hg 2+ / H+ 22. In the Cannizzaro’s reaction given below :


19. Ph  C ≡≡ C  CH3 → A , A is (2003, 1M) KOH
2Ph  CHO → Ph  CH2OH + PhCO–2
O
Ph Ph The slowest step is (1996, 1M)
(a) the attack of —OH at the carbonyl group
(a) (b) O
(b) the transfer of hydride to the carbonyl group
H3C H3C (c) the abstraction of proton from the carboxylic acid
(d) the deprotonation of Ph  CH2OH
O–
Ph
23. m-chlorobenzaldehyde on reaction with conc. KOH at room
Ph
temperature gives (1991, 1M)
(c) (d) OH (a) potassium m-chlorobenzoate and m-chlorobenzyl alcohol
H 3C H3C (b) m-hydroxy benzaldehyde and m-chlorobenzyl alcohol
(c) m-chlorobenzyl alcohol and m-hydroxy benzyl alcohol
CHO OHC (d) potassium m-chlorobenzoate and m-hydroxy benzaldehyde

20.
(i) NaOH/100°C Objective Questions II
(ii) H+/H2O (One or more than one correct option)
CHO OHC 24. In the reaction scheme shown below Q, R and S are the major
products. (2020 Adv.)
Major product (2003, 1M)
COOH HOOC
H3C CH3 H3C CH3
(a)
H3C (i) Zn-Hg/HCl
Q (ii) H R
COOH HOOC AlCl3 3PO4

P (i) CH3MgBr
CH2OH HOOC (ii) H3O+
(iii) H2SO4/∆
S
(b)
The correct structure of
COOH HOH2C
H3C CH3 H 3C
CH3
O H 3C
O

(a) S is
(c)
CH3
O
H3C CH3 H3C
O CH3
H3C
CH2OH HOH2C

(d) (b) Q is

HO2C O
CH2OH HOH2C
H3C CH3 H 3C
21. In Cannizzaro’s reaction, the intermediate which is the best CH3
hydride donor is (1997) H3C

H H
  (c) R is
(a) C6H5  C O− (b) C6H5  C O− O
 
OH O− H3C CH3 H3C
CH3
H H H3C
C — O– C — O– H 3C
(c) (d) (d) S is
O– O–
O2N MeO
Aromatic Aldehydes, Ketones and Acids 475

25. Compound P and R upon ozonolysis produce Q and S , 28. The compound Y is (2018 Adv.)
respectively. The molecular formula of Q and S is C8H 8O. Q OH
undergoes Cannizzaro reaction but not haloform reaction, COBr Br
whereas S undergoes haloform reaction but not Cannizzaro (a) (b)
reaction. (2017 Adv.)
HO O
(i) O3/CH2Cl 2 (i) O3/CH2Cl 2
(i) P → Q (ii) R → S Br
(ii) Zn/H2O (C 8H 8O) (ii) Zn/H2O (C 8H8O)
COBr
The option(s) with suitable combination of P and R, (d)
(c)
respectively, is(are)
Br
H 3C CH3
CH3 CH3
29. The compound Z is
(a) and
CH3 (a) (b)
CH3
O O
OH
(b) H3C and H3C
(c) (d)
CH3

O
(c) H3C and Passage 2
CH3 An organic acid P ( C11 H12 O2 ) can easily be oxidised to a dibasic acid
H3C which reacts with ethylene glycol to produce a polymer dacron. Upon
CH3 CH3
ozonolysis, P gives an aliphatic ketone as one of the products. P
(d) and undergoes the following reaction sequences to furnish R via Q. The
compound P also undergoes another set of reactions to produce S .
(2018 Adv.)
H3C CH3 (1) H2/Pd-C
(2) NH3/∆ (1) H2/Pd-C (1) HCl
26. Positive Tollen’s test is observed for (3) Br2/NaOH (2) SOCl2 (2) Mg/Et2O
S P Q R
OH (4) CHCl3/KOH, ∆ (3) MeMgBr,CdCl2 (3) CO2(dry ice)
(5) H2/Pd-C (4) NaBH4 (4) H3O+
H O CHO Ph
Ph O 30. The compound R is
H HO2C
H O
Ph Ph
Br (a) CO2H (b)
(a) (b) (c) (d)

27. The aldehydes which will not form Grignard product with
one equivalent Grignard reagents are (2019 Main 12 Jan II)
CO2H CO2H
CHO
(c) (d)
CHO
(a) (b)
HO2C

CHO CHO 31. The compound S is


(c) (d)
HO3CO HOH2C

Passage Based Questions (a) (b)

Passage 1
NH2 HN
Treatment of benzene with CO / HCl in the presence of anhydrous
AlCl 3 / CuCl followed by reaction with Ac2 O / NaOAc gives
compound X as the major product. Compound X upon reaction NH2 H
with Br2 / Na 2 CO3 followed by heating at 473 K with moist KOH (c) (d) N
furnishes Y as the major product. Reaction of X with H2 / Pd - C,
followed by H3 PO4 treatment gives Z as the major product.
476 Aromatic Aldehydes, Ketones and Acids

Passage 3 with hydrazine gives a cyclised compound D (C14H10N2).


In the following reactions sequence, the compound J is an Identify A, B, C and D. Explain the formation of D from C.
(2000, 5M)
intermediate.
(CH3 CO) 2 O (i) H2 , Pd/C 38. Explain, why o-hydroxy benzaldehyde is a liquid at room
I → J → K temperature while p-hydroxy benzaldehyde is a high melting
CH3 COONa (ii) SOCl 2
(iii) anhyd. AlCl 3
solid? (1999, 2M)

J (C9 H8 O2 ) gives effervescence on treatment with NaHCO3 Match the Columns


and positive Baeyer’s test. (2012)
Answer Q. 39, Q. 40 and Q. 41 by appropriately matching the
32. The compound K , is
information given in the three columns of the following table.
(a) (b) Column 1, 2 and 3 contain starting materials, reaction conditions,
O O and type of reactions, respectively. (2017 Adv.)

(c) (d) O Column 1 Column 2 Column 3


(I) Toluene (i) NaOH/ Br2 (P) Condensation
O
(II) Acetophenone (ii) Br2/hν (Q) Carboxylation
33. The compound I, is
H (III) Benzaldehyde (iii) (CH3CO)2 O/ (R) Substitution
O H OH O CH3 H CH3COOK
H

(a) (b) (c) (d) (IV) Phenol (iv) NaOH/CO 2 (S) Haloform

39. The only CORRECT combination in which the reaction


Fill in the Blank proceeds through radical mechanism is
(a) (IV) (i) (Q) (b) (III) (ii) (P)
34. The structure of the intermediate product formed by the
(c) (II) (iii) (R) (d) (I) (ii) (R)
oxidation of toluene with CrO3 and acetic anhydride, whose
hydrolysis gives benzaldehyde is …………… (1992, 2M) 40. For the synthesis of benzoic acid, the only CORRECT
combination is
True/False (a) (II) (i) (S) (b) (I) (iv) (Q)
35. Benzaldehyde undergoes aldol condensation in an alkaline (c) (IV) (ii) (P) (d) (III) (iv) (R)
medium. (1982 , 1M) 41. The only CORRECT combination that gives two different
carboxylic acids is
Subjective Questions (a) (IV) (iii) (Q) (b) (II) (iv) (R)
36. Five isomeric para-disubstituted aromatic compounds A to (c) (I) (i) (S) (d) (III) (iii) (P)
E with molecular formula C8 H8 O2 were given for
identification. Based on the following observations, give Integer Answer Type Question
structures of the compounds : (2002, Main, 5M)
42. Among the following the number of reaction(s) that
(i) Both A and B form a silver mirror with Tollen’s reagent; also, produce(s) benzaldehyde is (2015 Adv.)
B gives a positive test with FeCl 3 solution.
CHCl2
(ii) C gives positive iodoform test .
CO, HCl H2O
(iii) D is readily extracted in aqueous NaHCO3 solution. I.
Anhydrous AlCl3 / CuCl
II.
100°C
(iv) E on acid hydrolysis gives 1, 4-dihydroxy benzene.
COCl CO2Me
37. An organic compound A, C8H4O3, in dry benzene in the H2 DIBAL-H
presence of anhydrous AlCl3 gives compound B. The III. IV.
Pd-BaSO4 Toluene, -78°C
compound B on treatment with PCl5, followed by reaction H2 O
with H2/Pd/(BaSO4) gives compound C, which on reaction

Answers
1. (a) 2. (c) 3. (d) 4. (b) 25. (b,c) 26. (a,b,c) 27. (b,d) 28. (c)
5. (c) 6. (b) 7. (c) 8. (a) 29. (a) 30. (a) 31. (b) 32. (c)
9. (d) 10. (b) 11. (a) 12. (c) 33. (a) 34. C6H5  CH(OAc)2 35. False
13. (a) 14. (d) 15. (a) 16. (d)
39. (a) 40. (d) 41. (b) 42. (4)
17. (c) 18. (a) 19. (a) 20. (b)
21. (d) 22. (b) 23. (a) 24. (b,d)
Hints & Solutions
1. Electronic configuration of Gd3 + is OH OH
3+ 7
64Gd = [ Xe]4 f = [Xe] CHO
(1) CHCl3/aq. NaOH
3+
Gd having 7 unpaired electrons.
Magnetic moment (µ ) = n(n + 2) BM
Cl Cl
⇒ µ = 7(7 + 2) BM = 7.9 BM
where, n = number of unpaired electrons. The aldehyde obtained in above equation does not possess
α-hydrogen. In presence of formaldehyde and conc. NaOH it
2. (I) Given, reaction is Gattermann -Koch reaction involves the undergoes Cannizaro reaction. In this reaction, one molecule of
formation of an aryl aldehyde using CO and HCl as reactants. aldehyde is reduced to alcohol while another molecule is
The reaction catalysed by AlCl 3. oxidised to salt of carboxylic acid.
OH O–Na+
C—H CHO
Conc.
+ HCHO OH
CO, HCl NaOH + HCOO–Na+
AlCl3
Benzene Cl Cl
(II) Given, reaction is Stephen reduction reaction involves the Upon hydrolysis, following reaction takes place
preparation of aldehydes from nitriles using tin (II) chloride,
O–Na+ OH
hydrochloric acid and quenching the resulting minimum salt
with water. H 3O + OH
OH
+ HCOO–Na+ + HCOOH
CN CH==NH C—H
Cl Cl
SnCl2, HCl H3O+
4. The major product formed in the reaction is as follows :
H3O+
Cyanobenzene Imine
(Benzonitrile)
HO ==O
(1) CrO3
(III) Given, reaction is Rosenmund reduction is a hydrogenation (2) SOCl2/∆
process in which an acyl chloride is selectively reduced to an (3) ∆
HO HO
aldehyde.
Primary alcohol readily oxidised to corresponding carboxylic
Cl
C—H acid with oxidising agent, chromium trioxide (CrO3) in acidic
C
medium.
H2, Pd-BaSO4
S and Quinoline COOH
Benzoyl chloride
HO CrO3

Correct match of Item-I and Item-II will be


HO HO
(I) - (R), (II) - (P) and (III) - (Q).
OH group of carboxylic acid get substituted by Cl in
3. The major products of the given reaction are as follows: presence of SOCl2 (Thionyl chloride).
OH OH
COOH COCl
(1) CHCl3/aq. NaOH OH
(2) HCHO, conc. NaOH and HCOOH SOCl2/∆
(3) H3O+ + SO2↑ + HCl↑
Cl Cl HO HO
In step-I, substituted phenol undergoes Reimer-Tiemann
reaction in presence of CHCl3 / aq. NaOH Further, heating of product leads to intramolecular cyclisation.
478 Aromatic Aldehydes, Ketones and Acids

7. The given reaction is a crossed Cannizzaro reaction which is a


5. (i) C9H10O shows positive iodoform test thus,  C CH3 redox reaction too. Oxidation number of carbon atom of the
group is present. || CHO groups of PhCHO and HCHO are +1 and zero
O respectively. So, HCHO is the stronger reducing agent than
(ii) C9H10O on strong oxidation (KMnO4 / KOH), gives acid PhCHO. As a result, HCHO is oxidised to HCOONa (by
(C8H6O4 ), indicating it can be a dicarboxylic acid. So, ‘A’ donation of hydride, H− ) and PhCHO (H− acceptor) is reduced
contains COCH3 and one CH3 group which get to PhCH 2OH.
+1
oxidised into COOH and COOH respectively. 0 50% NaOH
Ph C HO + HC HO →
(iii) In the preparation of phenolphthalein from phenol, phthalic −1 +2 − +
anhydride is used. So, ‘B’ can be phthalic acid (benzene-1,2- Ph C H2OH + HC OONa
dicarboxylic acid) which readily forms anhydride.  +
↓ H3 O
Thus, the reaction sequence is as follows :
PhCH2OH + HCOOH
CH3 (Major products)
I2/NaOH The reaction proceed via following mechanism.
+ CHI3
CH3 (Iodoform test) – +
COONa Yellow ppts. OH OH

CH3 OH s Slowest
= O + Ph—CH2—Os
H—C ==
C O H2C=
=O Slower H—C—O
Ph—CH=
=O
δ+ δ– Fast
H

Ö
O C Os
More polar and less –
A(C9H10O) (i) [O] OH H shift
+ CO2 crowding for
H—C=
=O + PhCH2OH
KMnO4/KOH OH nucleophilic addition
(ii) H3O
r C

O 8. Reducing agents like LiAlH4, NaBH4, i.e. complex hydrides


Phthalic acid
B(C8H6O4) usually does not affect olifenic or π-bonds. Thus, if NaBH4 is
O
H
∆ –H2O

O
(2 mol)/
O applied to a compound like then its ( C == O bond will be
Conc. H2SO4 '
OH
C
O OH
O
–H2O C reduced only and we get as the
final product.
O Thus, option (a) is correct answer.
Phthalic anhydride
9. DIBAL-H (Diisobutylaluminium hydride) is a reducing agent
OH OH with formula [λ-Bu 2AlH ] . At ordinary temperatures, nitriles
(Phenolphthalein give imines which are readily converted in aldehydes by
an indicator)
hydrolysis whereas lactones are reduced directly to aldehydes.
6. NaOCl (sodium hypochlorite) is the reagent of haloform H
(chloroform formation) reaction.
Al (DIBAL–H) Al (DIBAL–H)
2NaOH + Cl2 → NaOCl + NaCl + H 2O Al
+ + H
The given reaction takes place as follows : N
CN N
C
O O C H
O
NaOCl s r
O
Ph—C—CH3 Ph—C—ONa + CHCl3
r O O
H3O (It is missing in the O
Keto-methyl statement of the question)
group O
O H2O
CHO CHO CHO
Ph—C—OH +H+ DIBAL–H
Benzoic acid (Hs)
(X) OH O O
O H
O O [Acyl SN2 O –O O
H
pathway]
SOCl2 –HCl —NH—C—Ph
Ph—C—OH Ph—C—Cl Benzanilide 10. In presence of Br2/EtOH, the reactant containing double bond
– SO2
(X) Benzoyl (Y)
– HCl H2N— undergoes electrophilic addition reaction via the formation of
chloride
Aniline
Aromatic Aldehydes, Ketones and Acids 479

•• OH (Cross Aldol condensation of


bromonium ion. On further attack of  O Me on bromonium ion CH3CHO and CH3COC6H5
••
(iii) CH3—C—C6H5 in which >C=O group of
gives the addition product. C6H5COCH3 is carbanion
CH2 acceptor).
H 3C COMe
O H—C
Br2(1 eqiv) O MeOH
O in MeOH
Br OH
(Protic solvent)
(Self Aldol condensation
(iv) C6H5—C—CH2—C—C6H5 product of C H COCH )
O 6 5 3
O CH3

O O H of base will prefer to attack on CH3 group of CH3CHO
for the formation of carbanion and as among the > C == O groups
O available, the > C == O group of CH3CHO is the best carbanion
OMe acceptor. Hence, self condensation product of CH3CHO will be
the major product.
Br
O
13. DIBAL-H (Di-isobutyl aluminium hydride) is a reducing agent
with formula. This is generally used for the preparation of
(Major)
aldehydes. Using DIBAL H, Lactones are reduced directly to
aldehydes.
11. In presence of alkaline KMnO4, vigorous oxidation of alkyl or O
acyl benzene takes place. During oxidation, aromatic nucleus OH
O
remains intact but the entire chain is oxidised to —COOH group DIBAL–H
irrespective of the length of carbon chain. CHO

Oxidative cleavage s⊕ COOH


in strong COOH
O==C—CH3 O ==C—O K
(MnO4– /OH–/D–)
+
H3O (dil. H2SO4)
14. Toluene undergoes oxidation with KMnO4, forms benzoic acid.
KMnO4/KOH/∆
Strong oxidation, [O]
In this conversion, alkyl part of toluene converts into carboxylic
group. Further, benzoic acid reacts with thionyl chloride (SOCl 2)
s⊕ to give benzoyl chloride which upon reduction with H2/Pd or
CH3 COOK
(also COOH BaSO4 forms benzaldehyde (Rosenmund reduction) The
oxidisable) conversion look like,
O Cl
CH3 COOH C
COOH KMnO4 SOCl2
+ SO2 + HCl
12. In aldol condensation, generally aldehydes react at a faster rate
than ketones towards base. In the given case CH3CHO will lose (Toluene) (Benzoic Acid) (Benzoyl chloride)
‘A’ ‘B’
O
H2/Pd BaSO4
C
CH3 due to one more O H
α-hydrogen faster than C

reason, i.e. conjugation between benzene ring and ‚ ƒC == O + HCl


group. Along with sterically less hindered nucleophile of
CH3CHO will also add to the major product formation. (Benzaldehyde)
‘C’
Following four products are possible in the reaction:
15. It is a Kolbe Schmidt reaction.
H + +
ONa COONa COOH O
(i) CH3—C—CH2—CHO (Self Aldol condensation
  
of CH3CHO) OH OC  CH3
OH H+/Ac2O
+ CO2 125°C
(Cross Aldol condensation of 5 atm
H
CH3CHO and CH3COC6H5 ‘B’ Aspirin
(ii) CH3—C—CH2—C—C6H5 in which >C=O group of (pain killer)
CH3CHO is carbanion ‘C’
OH acceptor).
The second step of the reaction is an example of acetylation
reaction.
480 Aromatic Aldehydes, Ketones and Acids

16. Compound A gives a precipitate with alcoholic AgNO3, so it 21. Dioxoanion is better hydride donor. Electron donating group at
must contains Br in side chain. On oxidation, it gives C8 H6O4, ortho/para position further promote H– transfer.
which shows the presence of two alkyl chains attached directly O


with the benzene nucleus. Since, compound B gives anhydride O
O
on heating, the two alkyl substituent must occupy adjacent (1, 2) C—H C
position. Thus, A must be + C—R
O– + O–
CH2 Br H3CO H3CO
H –
O
CH3
+ H— C —R
and the reactions are as follows :
CH2Br CH2 OR H
Alcoholic −
+ AgBr O O
AgNO3
CH3 CH3  

‘A’ 22. Ph  C  H + HO → Ph  C  H
Oxidation O 
OH
COOH I

O O− O
COOH   Slow
‘B’
O Ph  C  H + C  Ph →
Phthalic anhydride   hydride transfer
CHO COOH OH H
17. OH– I O O−
Intramolecular  
CHO Cannizzaro reaction CH2OH Ph  C  OH + Ph  C  H
O 
H
H+ 23. Cl
O Cl Cl

Ester formation
+ KOH +
18.
H3C— —SO3H + CH3COONa CHO COOK CH2OH
Base Cannizzaro’s
4-methyl benzene sulphonic reaction
acid
24. Compound (P) undergoes Friedel-Craft acylation reaction in
H3C— —SO3Na + CH3COOH presence of succinic anhydride and AlCl 3 to gives an acid (Q).

H+ +
19. Ph — C ≡≡ C— CH3 → Ph — C==CH— CH3
Stable carbocation AlCl3

O
H2O  (P) OH
→ Ph — C — CH2CH3
CHO OHC ( Q)

20. NaOH
Compound (Q) on treatment with Zn-Hg / HCl undergoes
Clemmensen reduction, where ketone group change into alkane
CHO OHC and on further reaction with dehydrating agent H3PO4 gives
COOH HOH2C compound (R).
+
H
H2 O
Zn-Hg H2O
CH2OH HOOC HCl H3PO4

The above reaction is an example of intramolecular Cannizzaro OH OH (R)


reaction.
(Q)
Aromatic Aldehydes, Ketones and Acids 481

Compound (R) on reaction with Grignard reagent and heated in In the above reaction, NH3 prefer to attack at aliphatic aldehyde
presence of acid gives dehydrating product (S). group than an less reactive aromatic aldehyde group.
O OH
H3C
H3C C N—H
(i) CH3MgBr HO H
Tautomerism
(ii) H3O+ Xs NH2
CH3
– H2O
(R ) O
H3C
(iii) H2 SO4 N
– H2 O ∆
‘S’
(S) Passage-1
28. Given, 1 2
Thus, the option (b) and (d) are correct. Ac2O
CO+HCl
Benzene X
25. anhy. AlCl3/CuCl NaOAc (Major)
O3
(b) CH3 CH CH2 CH3 CHO+H2CO Br2 +
Zn–H2O 3
(R) Na2CO3

C
5


Gives Cannizzaro

d
(P)

2 /P
but not haloform reaction

H
Moist KOH
O 6 4

4
O
473 K

3P
H
O3
Zn–H2O
C CH3+H2CO Y
(S)
Z (Major product)
(Q ) Gives haloform reaction
but not Cannizzaro reaction For this question we require only reaction 1 to 4 written above.
Let us explore them one by one.
O3
(c) CH CH CH3 Zn–H2O
CHO+CH3CHO Reaction 1 It is called formylation or Gatterman Koch reaction.
(R) A  CHO group is introduced to benzene ring through this
P
CH3 CH3
reaction as
CHO
Gives Cannizzaro
but not haloform reaction
− CO+HCl
OH
26. RCHO + Ag2O  → RCOOH + 2Ag In anhyd.
(Tollen' s reagent) Silver mirror AlCl3/CuCl
Benzene Benzaldehyde
Tollen’s test is given by all aldehydes and all reducing sugars as +
glucose, fructose and α-hydroxy ketones. The attacking electrophile is H C == O which is generated as
OH O (i) CO + HCl q H — C — Cl

— CH— C — O
α +
H O (ii) H — C — Cl + AlCl 3 q H — C + AlCl −4
 
H having an aldehyde group gives Tollen's test. O O
H Reaction 2 It is Perkin condensation which results in α, β
Br unsaturated acid as
O CHO CH CHCOOH
CHO (c)
CH C CH3COO–Na+
+ (CH3CO)2O + CH3COOH
180°C
OH
Aldehyde
α-hydroxy ketone Cinnamic acid
give positive Tollen’s test. Note Besides CH3COO− Na+, quinoline, pyridine, Na 2CO3,
triethylamine can also be used as bases in this reaction.
27. O
Reaction 3 It is simple addition of bromine to unsaturated acid
H 3C H 3C C—H formed through reaction 2.
(i) O3
CHO
(ii) Zn– H2O CH CHCOOH CHBr CHBr
R
H 3C Br2 COO–Na+
CHO
NH3 CH NH Na2CO3

(X) Cinnamic acid


482 Aromatic Aldehydes, Ketones and Acids

Na2CO 3 works as a base in the reaction to trap H+ to be released This indicates presence of benzene ring in P; as concluded from
OH the structure of dacron given below.
O
CH—CH—Br
O C C O CH2 CH2 O
COO–Na+
by — COOH of the reactant. is also formed O
Dicarboxylic Ethylene
in the reaction as the minor product. acid component glycol component
Reaction 4 It is decarboxylation and dehydrohalogenation of
Dacron
product produced by reaction 3 as
CHBr CHBr C CH Attachment of —COO group in dacron also confirm the para
position of branch with respect to —COOH group in P.
COO–Na+
Moist KOH
+ Na2CO3 + KBr (iii) P Ozonolysis
 → Aliphatic ketone + other oxidised products.
437 K This reaction confirms the presence of multiple bonded
C CH branch i.e., alkenyl group in P.
Thus P can be
Hence, Y is i.e., (c) is the correct answer. COOH

29. Reaction 5 The Perkin condensation product X is


CH CHCOOH

CH
i.e., cinnamic acid
C

This compound on hydrogenation with H2 in the presence of Pd CH3 CH3


activated with charcoal (Pd − C ) gives
IUPAC name : 4-(2-methyl) prop-l-enyl benzoic acid
CH CHCOOH CH2CH2COOH
Now look for the reactions
COOH COOH
Pd-C
+ H2
Oxidation
3-Phenyl propanoic acid

Reaction 6 The product of reaction 5 on heating with H3PO4


dehydrates to give COOH
Terephthalic
CH2CH2COOH CH2 CH2 (P)
acid

H3PO4 C COOH
∆ O CHO
COOH
O CHO
Z Ozonolysis
+ + C O + C O
Passage-2 CHO
Aliphatic Glyoxal CHO
30. Given, ketone
CHO
(i) C11H12O2  → Dibasic acid
Oxidation
(Acetone) other oxidised
(P) products
(An organic acid ‘P’)
This indicates the presence of alkyl or alkenyl branch in P 1. H 2 / Pd– C
along with −COOH group. 2. SOCl 2
Given, P → Q
(ii) Dibassic acid produced by oxidation of P 3. MeMgBr, CdCl 2
CH2 CH2 4. NaBH 4

OH OH O
(Ethylene glycol)
O C —C—O—(CH2)2—O—

O
Dacron
Aromatic Aldehydes, Ketones and Acids 483

So, So,
Cl COOH COOH CONH2

COOH COOH CO H2/Pd–C NH3/∆

H2/Pd––C SOCl2

MeMgBr,
CdCl2 (P)
CH3
(P)
Me O
NH N C NH2 Br2/NaOH
(Hofmann
H C OH C Me bromamide)
H2/Pd–C CHCl3/KOH
NaBH4 (Carbylamine)
[Here, Me = CH3]

( Q)
(S)
1. HCl
2. Mg/ Et 2O Passage 3
Further, Q → R
3. CO2 (Dry ice) Sol for (Q. Nos. 34 to 35) The first step of reaction is
4. H 3O+ Perkin’s condensation.
Me Me Me CHO
– CH3COONa
H C OH H C Cl H CMg
2+
Cl
– + (CH3CO)2O
C6H5 CH CH COOH
I
HCl Mg J
Et2O J being a carboxylic acid gives effervescence with NaHCO3 .
(Dry ether) Also, J has olefinic bond, it will decolourise Baeyer’s reagent.
In the second step, J on treatment with H2 / Pd / C undergo
(Q) hydrogenation at olefinic bond only as :
Grignard's reagent
CO2
J + H2 / Pd → C6 H5  CH2  CH2  COOH
(Carboxylation) The hydrogenated acid, on treatment with SOCl 2 gives acid
Me Me chloride.
C6 H5 — CH2 — CH2 — COOH + SOCl 2 →
H C COOH H C COOMgCl C6 H5 — CH2 — CH2 — COCl + HCl + SO2
H3O+ In the final step, acid chloride formed above undergo
Mg(OH)Cl + intramolecular Friedel-Craft acylation as:
(Hydrolysis)
AlCl3
C
Cl O O
( R) ‘K’
34. CH3 CH(OAc)2
31. Given, CrO3
1. H 2 / Pd– C
(CH3CO)2O
2. NH 3 / ∆
3. Br2 / NaOH Intermediate
P → S CHO
4. CHCl 3 / KOH, ∆
H2O
5. H 2 / Pd– C
H+
484 Aromatic Aldehydes, Ketones and Acids

35. For aldol condensation, presence of at least one α-H is essential, 38. Intramolecular H-bonding in ortho hydroxy benzaldehyde is
which is not available to benzaldehyde. responsible for decrease in melting and boiling points.
36. CHO O H
O
Gives positive Tollen’s test but does not
give FeCl3 test. H
Intramolecular
CH2OH H-bonding
A
p-hydroxy benzaldehyde molecules are associated by
OHC— —OH Gives positive Tollen’s test intermolecular H-bonding, has higher melting and boiling
and FeCl3 test. points.
B
Br
O 39. C6H5  CH3 →
2
C6H5  CH2Br + HBr
hv (Free radical bromination)

CH3—C— —OH Gives positive iodoform O


test. || NaOH / Br2
C 40. C6H5  C  CH3 → C6H 5COONa + CHBr3 (P )
(II) (i ) Bromoform

HOOC— —CH3 Can be extracted with O O


NaHCO3. || ||
41. C6H 5CHO+ CH3  C  O  C  CH3
HO— —OCH==CH2 On hydrolysis give →
3 CH COOK
C6H5  CH == CH  COOH
1,4-dihydroxy benzene. Perkin's condensation (Cinnamic acid)

Cinnamic acid shows cis-trans isomerism.


37. O
42. I. Gattermann-Koch reaction.
AlCl3 II. CHCl2 CH(OH)2 CHO
O + H2O
–H2O
100°C (S N1 )
gem-diol
O
A
III. Rosenmund’s reduction.
O O
IV. Acid chloride, anhydride and ester undergo controlled
reduction with di-iso-butylaluminium hydride (DIBAL-H) at
PCl5
− 78°C to give aldehydes.

HOOC Cl—C

B O

H2 N2H4
Pd/BaSO4 N
H—C N
O D
C
32
Biomolecules and
Chemistry in Everyday Life
Topic 1 Biomolecules
Objective Questions I (Only one correct option) 8. Maltose on treatment with dilute HCl gives
(2019 Main, 08 April I)
1. Which of the given statements is incorrect about glycogen?
(2019 Main, 12 April II)
(a) D-glucose and D-fructose (b) D-fructose
(a) It is straight chain polymer similar to amylose (c) D-galactose (d) D-glucose
(b) Only α-linkages are present in the molecule 9. The correct structure of histidine in a strongly acidic
(c) It is present in animal cells solution ( pH = 2 ) is (2019 Main, 12 Jan II)
(d) It is present in some yeast and fungi r s r
(a) H3N CH COO (b) H3N CH COOH
2. Which of the following statement is not true about RNA? r r
(2019 Main, 12 April I) NH2 NH2
(a) It controls the synthesis of protein
N N
(b) It has always double stranded α-helix structure
r s r
(c) It usually does not replicate (c) H3N CH COO (d) H3N CH COOH
(d) It is present in the nucleus of the cell +
NH NH
3. Number of stereo-centers present in linear and cyclic
NH
structures of glucose are respectively (2019 Main, 10 April II) Nr
(a) 4 and 5 (b) 4 and 4 H
(c) 5 and 4 (d) 5 and 5
10. The correct match between Item I and Item II is
4. Amylopectin is composed of (2019 Main, 10 April I)
Item I Item II
(a) β-D-glucose, C1-C4 and C2-C6 linkages
A. Ester test P. Tyr
(b) α-D-glucose, C1-C4 and C2-C6 linkages
B. Carbylamine test Q. Asp
(c) β-D-glucose, C1-C4 and C1-C6 linkages
C. Phthalein dye test R. Ser
(d) α-D-glucose, C1-C4 and C1 -C6linkages
S. Lys
5. The peptide that gives positive ceric ammonium nitrate and (2019 Main, 11 Jan II)
carbylamine tests is (2019 Main, 09 April II) (a) A → Q; B → S; C → R (b) A → R, B → Q; C → P
(a) Lys-Asp (b) Ser-Lys (c) A → R; B → S; C → Q (d) A → Q; B → S; C → P
(c) Gln-Asp (d) Asp-Gln
11 Among the following compounds, which one is found
6. Which of the following statement is not true about sucrose? in RNA? (2019 Main, 11 Jan I)
(2019 Main, 09 April I)
O O
(a) It is also named as invert sugar. CH3
(b) The glycosidic linkage is present between C1 of NH NH
(a) (b)
α-glucose and C1 of β-fructose N N
O O
(c) It is a non-reducing sugar H
(d) On hydrolysis, it produces glucose and fructose H
NH2 O
7. Fructose and glucose can be distinguished by
(2019 Main, 08 April II) N N Me
(c) (d)
(a) Fehling’s test (b) Barfoed’s test N O N O
(c) Benedict’s test (d) Seliwanoff’s test
H Me
486 Biomolecules and Chemistry in Everyday Life

12. Which of the following tests cannot be used for identifying 20. Which one of the following bases is not present in DNA?
amino acids ? (2019 Main, 10 Jan II) (a) Quinoline (b) Adenine (2014 Main)
(a) Barfoed test (b) Ninhydrin test (c) Cytosine (d) Thymine
(c) Xanthoproteic test (d) Biuret test 21. Synthesis of each molecule of glucose in photosynthesis
13. The correct sequence of amino acids present in the tripeptide involves (2013 Main)
given below is (2019 Main, 9 Jan II) (a) 18 molecules of ATP (b) 10 molecules of ATP
(c) 8 molecules of ATP (d) 6 molecules of ATP
Me Me Me OH
O
H 22. The following carbohydrate is
N OH
H2N N C H OH
O H
HO H O
OH O OH
HO H HO
(a) Thr - Ser - Leu (b) Leu - Ser - Thr H
H (2011)
(c) Val - Ser - Thr (d) Thr - Ser - Val (a) a ketohexose (b) an aldohexose
14 The increasing order of pK a of the following amino acids in] (c) an α-furanose (d) an α-pyranose
aqueous solution is Gly, Asp, Lys, Arg 23. The correct statement about the following disaccharide is
(2019 Main, 9 Jan I)
CH2OH
(a) Asp < Gly < Arg < Lys (b) Arg < Lys < Gly < Asp
H O H2COH O H
(c) Gly < Asp < Arg < Lys (d) Asp < Gly < Lys < Arg H
H
OH H H HO
15. Glucose on prolonged heating with HI gives (2018 Main) OCH2CH2O
HO CH2OH
(a) n-hexane (b) 1-hexene
(c) Hexanoic acid (d) 6-iodohexanal H OH OH H (2010)
(a) (b)
16. The predominant form of histamine present in human blood is (a) Ring (a) is pyranose with α-glycosidic link
(pK a , Histidine = 6.0 ) (2018 Main) (b) Ring (a) is furanose with α-glycosidic link
H H (c) Ring (b) is furanose with α-glycosidic link
N N r (d) Ring (b) is pyranose with β-glycosidic link
NH2 NH3
(a) (b) 24. Two forms of D-glucopyranose, are called (2005, 1M)
r
N N (a) enantiomers (b) anomers
H
H H (c) epimers (d) diastereomers
N NH2 N r
NH3 25. Which of the following pairs give positive Tollen’s test ?
(c) (d) (2004, 1M)
r
N N (a) Glucose, sucrose (b) Glucose, fructose
H
(c) Hexanal, acetophenone (d) Fructose, sucrose
17. Which of the following compounds will behave as a
reducing sugar in an aqueous KOH solution? (2017 Main) Objective Question II
O O
(One or more than one correct option)
HOH2C CH2OH HOH2C CH2OH
26. Which of the following statement(s) is(are) true? (2019 Adv.)
(a) HO (b) HO (a) The two six-membered cyclic hemiacetal forms of D-(+ )
OCOCH3 -glucose are called anomers
(b) Oxidation of glucose with bromine water gives
OH OH glutamic acid
HOH2C
(c) Monosaccharides cannot be hydrolysed to given
HOH2C O CH2OH O CH OCH polyhydroxy aldehydes and ketones
2 3
(d) Hydrolysis of sucrose gives dextrorotatory glucose and
(c) HO (d) OH laevorotatory fructose
OCH3 OH 27. The Fischer presentation of D-glucose is given below.
OH OH CHO

18. Thiol group is present in (2016 Main) H OH


(a) cystine (b) cysteine HO H
(c) methionine (d) cytosine H OH
19. Which of the vitamins given below is water soluble? H OH
(a) Vitamin C (b) Vitamin D (2015 Main)
CH2OH
(c) Vitamin E (d) Vitamin K D-glucose
Biomolecules and Chemistry in Everyday Life 487

The correct structure(s) of β-L-glucopyranose is (are) 30. Statement I Glucose gives a reddish-brown precipitate
(2018 Adv.) with Fehling’s solution.
H H
Statement II Reaction of glucose with Fehling’s solution
O O
HO OH HO OH gives CuO and gluconic acid. (2007, 3M)
(a) CH2OH (b) CH2OH

H
H HO
H H
H H
H
Integer Answer Type Questions
31. The structure of a peptide is given below.
OH H OH OH NH2
CH2OH H
HO
O O
H OH HO H O
H CH2OH H
(c) (d) N OH
OH HO H HO H2N N
HO H H OH H
O O
H H OH H CO2H

28. For ‘invert sugar’, the correct statement(s) is (are) If the absolute values of the net charge of the peptide at pH
(Given: specific rotations of ( + ) - sucrose, (+) - maltose, = 2, pH = 6, and pH = 11are | Z1 |, | Z2 |, and | Z3 |, respectively,
L-( − ) -glucose and L-( + ) -fructose in aqueous solution are then what is | Z1 | + | Z2 | + | Z3 |? (200 Adv.)
+ 66 °, +140 °, − 52 ° and 92° , respectively) (2016 Adv.) 32. The total number of distinct naturally occurring amino acids
(a) Invert sugar is prepared by acid catalysed hydrolysis obtained by complete acidic hydrolysis of the peptide shown
of maltose below is (2014 Adv.)
(b) Invert sugar is an equimolar mixture of D-(+) -glucose
O
and D-(−)- fructose
(c) Specific rotation of invert sugar is − 20 ° H
O O H O H O N
(d) On reaction with Br2 water, invert sugar forms
saccharic acid as one of the products N N N
N N N
29. The correct statement(s) about the following sugars X and Y
N CH2 O O H CH2 O
is/are: (2009) H
CH2OH H
H O H O H
O
HOH2C
H 33. A tetrapeptide has  COOH group on alanine. This produces
OH H H HO CH2OH
O
glycine (Gly), valine (Val), phenyl alanine (Phe) and alanine
OH
(Ala), on complete hydrolysis. For this tetrapeptide, the number
H OH OH H
X of possible sequences (primary structures) with  NH2 group
CH2OH attached to a chiral centre is (2013 Adv.)
H O H 34. The substituents R1 and R2 for nine peptides are listed in the
CH2OH H table given below. How many of these peptides are
OH H
H O O positively charged at pH = 7.0 ? (2012)
OH
H H3N  CH  CO  NH  CH  CO  NH  CH
OH H H OH
H H
HO R1 R2
H OH Y  CO  NH  CH  COO
(a) X is a reducing sugar and Y is a non-reducing sugar
(b) X is a non-reducing sugar and Y is a reducing sugar H
(c) The glucosidic linkages in X and Y are α and β, respectively Peptide R1 R2
(d) The glucosidic linkages in X and Y are β and α, respectively
I H H
II H CH 3
Assertion and Reason
III CH 2COOH H
Read the following questions and answer as per the direction
IV CH 2CONH 2 (CH 2 ) 4 NH 2
given below :
(a) Statement I is correct; Statement II is correct; V CH 2CONH 2 CH 2CONH 2
Statement II is a correct explanation of Statement I. VI (CH 2 ) 4 NH 2 (CH 2 ) 4 NH 2
(b) Statement I is correct; Statement II is correct; Statement VII CH 2COOH CH 2CONH 2
II is not the correct explanation of Statement I. VIII CH 2OH (CH 2 ) 4 NH 2
(c) Statement I is correct; Statement II is incorrect. IX (CH 2 ) 4 NH 2 CH 3
(d) Statement I is incorrect; Statement II is correct.
488 Biomolecules and Chemistry in Everyday Life

35. When the following aldohexose exists in its 39. The structure of D-glucose is as follows :
D-configuration, the total number of stereoisomers in its CHO
pyranose form, is (2012)
H OH
CHO
HO H
CH2
H OH
CHOH
H OH
CHOH OH
CHOH (i) Draw the structure of L-glucose.
CH2OH (ii) Give the reaction of L-glucose with Tollen’s reagent.
(2004, 2M)
36. A decapeptide (Molecular weight 796) on complete
hydrolysis gives glycine (Molecular weight 75), alanine and 40. Following two amino acids lysine and glutamine form
phenylalanine. Glycine contributes 47.0% to the total weight dipeptide linkage. What are two possible dipeptides?
(2003, 2M)
of the hydrolysed products. The number of glycine units
present in the decapeptide is (2011) H2 N  CH  COOH

37. The total number of basic groups in the following form of CH2 CH2 CH2 CH2 NH2
lysine is (2010)
⊕ H2 N  CH  COOH
H3N—CH2—CH2—CH2—H2C O

CH—C CH2 CH2 COOH

H2N O
s 41. Aspartame, an artificial sweetener, is a peptide and has the
following structure
Subjective Questions CH2 C6H5
38. Which of the following disaccharide will not reduce Tollen’s 
reagent? (2005, 2M) H2 N  CH  CONH  CH  COOCH3
CH2OH CH2OH 
CH2  COOH
HO O O OH
H H
(a)
(i) Identify the four functional groups.
H HO H HO (ii) Write the Zwitter ionic structure.
O
H H
(iii) Write the structures of the amino acids obtained from
OH H OH H the hydrolysis of aspartame.
CH2OH CH2OH
(iv) Which of the two amino acids is more hydrophobic?
OH O O OH (2001, 5M)
H H
(b) 42. Give the structures of the products in the following reaction
H HO OH H
O
H H H+
Sucrose → A + B (2000, 2M)
OH H H OH
43. Write the structure of alanine at pH = 2 and pH = 10.
(2000, 2M)

Topic 2 Chemistry in Everyday Life


Objective Questions I (Only one correct option)
1. The antifertility drug “Novestrol” can react with 2 The following molecule acts as an (2020 Main, 5 Sep II)
(2020 Main, 3 Sep I) N
(a) ZnCl2 / HCl;FeCl3 ; alcoholic HCN (CH2)2
N
(b) Br2 /water; ZnCl2 / HCl;FeCl3
(Brompheniramine)
(c) alcoholic HCN; NaOCl; ZnCl2 / HCl
(d) Br2 /water; ZnCl2 / HCl;NaOCl
Br
(a) antiseptic (b) anti-depressant
(c) anti-bacterial (d) anti-histamine
Biomolecules and Chemistry in Everyday Life 489

3. Which of the following is a thermosetting polymer? 11. The homopolymer formed from 4-hydroxybutanoic acid is
(2019 Main, 12 April I)
(a) Bakelite (b) Buna-N
(c) Nylon-6 (d) PVC (a) —OC(CH2)3—O—
n
(b) —C(CH2)2C—
n
4. The correct match between Item-I and Item-II is
Item-I Item-II
(c) —C(CH2)3—O— (d) —C(CH2)2C—O—n
A. High density polythene I. Peroxide catalyst n

B. Polyacrylonitrile II. Condensation at high (2019 Main, 11 Jan II)


temperature and pressure
12. The correct match between Item I and Item II is
C. Novolac III. Ziegler-Natta catalyst
Item I Item II
D. Nylon-6 IV. Acid or base catalyst
A. Allosteric effect P. Molecule binding to the active site
(2019 Main, 10 April II) of enzyme.
Codes B. Competitive Q. Molecule crucial for
A B C D inhibitor communication in the body.
(a) III I IV II C. Receptor R. Molecule binding to a site other
(b) IV II I III than the active site of enzyme.
(c) II IV I III D. Poison S. Molecule binding to the enzyme
(d) III I II IV covalently.
5. Which of the following is a condensation polymer? (a) A → P; B → R; C → S; D → Q
(2019 Main, 10 April I)
(b) A → P, B → R; C → Q; D → S
(a) Nylon-6, 6 (b) Neoprene (c) A → R; B → P; C → S; D → Q
(c) Teflon (d) Buna - S (d) A → R; B → P; C → Q; D → S (2019 Main, 11 Jan II)
6. Noradrenaline is a/an (2019 Main, 9 April II) 13. The polymer obtained from the following reaction is:
(a) antidepressant (b) antihistamine (2019 Main, 11 Jan I)
(c) neurotransmitter (d) antacid (i) NaNO2 /H3O
+

NH2
7. Which of the following compounds is a constituent of the HOOC (ii) Polymerisation

O
O O
 H
[ HN — C — NH — CH 2—
polymer — ]n ? (a) —C—(CH2)4—N—
n
(b) —O—(CH2)4—C—n
(2019 Main, 9 April II)
(a) N-methyl urea
H
(b) Methylamine (c) —OC(CH2)4O— (d) —HNC(CH2)4—C—N—
(c) Ammonia n n
(d) Formaldehyde 14. The correct match between item (I) and item (II) is
8. The structure of nylon-6 is Item - I Item - II
O H O H (A) Norethindrone (P) Antibiotic
|| | || |
(a) 
[ (CH2 )6  C  N 
]n (b) 
[ C  (CH2 )5  N 
]n (B) Ofloxacin (Q) Antifertility
(C) Equanil (R) Hypertension
O H O H
|| (S) Analgesics
|| | |
(c) 
[ (CH2 )4  C  N 
]n (d) 
[ C(CH2 )6  N 
]n (2019 Main, 11 Jan I)
(2019 Main, 8 April II) (a) (A) → (Q); (B) → (R); (C) → (S)
(b) (A) → (Q); (B) → (P); (C) → (R)
9. The two monomers for the synthesis of (c) (A) → (R); (B) → (P); (C) → (S)
nylon 6, 6 are (2019 Main, 12 Jan II) (d) (A) → (R); (B) → (P); (C) → (R)
(a) HOOC(CH2 )4 COOH, H2N(CH2 )4 NH2 15. The correct match between Item - I and Item - II is
(b) HOOC(CH2 )6 COOH, H2N(CH2 )4 NH2
Item I (Drug) Item II (Test)
(c) HOOC(CH2 )4 COOH, H2N(CH2 )6 NH2 A. Chloroxylenol P. Carbylamine test
(d) HOOC(CH2 )6 COOH, H2N(CH2 )6 NH2 B. Norethindrone Q. Sodium hydrogen carbonate test
10. Poly-β-hydroxybutyrate-Co-β-hydroxyvalerate (PHBV) is a C. Sulphapyridine R. Ferric chloride test
D. Penicillin S. Bayer’s test
copolymer of …… (2019 Main, 12 Jan I)
(2019 Main, 9 Jan I)
(a) 3-hydroxybutanoic acid and 2-hydroxypentanoic acid
(a) A→ R ; B→ P ; C→ S ; D→ Q
(b) 2-hydroxybutanoic acid and 3-hydroxypentanoic acid
(b) A→ R ; B→ S ; C→ P ; D→ Q
(c) 3-hydroxybutanoic acid and 4-hydroxypentanoic acid (c) A→ Q ; B→ P ; C→ S ; D→ R
(d) 3-hydroxybutanoic acid and 3-hydroxypentanoic acid (d) A→ Q ; B→ S ; C→ P ; D→ R
490 Biomolecules and Chemistry in Everyday Life

16. The formation of which of the following polymers involves 26. Cellulose upon acetylation with excess acetic anhydride/
hydrolysis reaction? (2017 Main) H2 SO4 (catalytic) gives cellulose triacetate whose structure
(a) Nylon-6 (b) Bakelite is (2008, 3M)
(c) Nylon-6, 6 (d) Terylene AcO
OO
17. Which of the following statements about low density AcO H H
polythene is false? (2016 Main) (a) O OAcH
H O H
(a) It is a poor conductor of electricity AcO H
(b) Its synthesis required dioxygen or a peroxide initiator OAcH H H OAc
HH O O
as a catalyst
OAc
(c) It is used in the manufacture of buckets, dustbins etc. O
OAcH H H
(d) Its synthesis requires high pressure
H OAc
18. Which of the following is an anionic detergent? (2016 Main) AcO
(a) Sodium lauryl sulphate OO
AcO H H
(b) Cetyltrimethyl ammonium bromide OH H
(b) O O H
(c) Glyceryl oleate AcO H H
(d) Sodium stearate OH H H H OH
HH O O
19. On complete hydrogenation, natural rubber produces
OH
(a) ethylene-propylene copolymer (2016 Adv.) O
OH H H H
(b) vulcanised rubber
H OH
(c) polypropylene AcO AcO AcO
(d) polybutylene OH H OH H OO
HH H H
20. Which polymer is used in the manufacture of paints and (c)
OAc H O OAc H O OAc H
lacquers? (2015 Main) O H
(a) Bakelite (b) Glyptal H OAc H OAc H OAc
(c) Polypropene (d) Polyvinyl chloride
21. Match the polymers in Column I with their main uses in AcO AcO AcO
Column II and choose the correct answer: HH OH H OH H OO
(d) H H
Column I Column II H H O H H O H H
O H
(A) Polystyrene 1. Paints and lacquers
OAc OAc OAc OAc OAc OAc
(B) Glyptal 2. Raincoats
(C) Polyvinyl chloride 3. Manufacture of toys
(D) Bakelite 4. Computer discs
Objective Question II
Codes
(One or more than one correct option)
A B C D A B C D 27. Choose the correct option(s) from the following. (2019 Adv.)
(a) 2 1 3 4 (b) 3 1 2 4 (a) Teflon is prepared by heating tetrafluoroethene in
(c) 2 4 3 1 (d) 3 4 2 1 presence of a persulphate catalyst at high pressure
(b) Natural rubber is polyisoprene containing trans alkene
OCOCH3
units
COOH is used as (c) Cellulose has only α-D-glucose units that are joined by
22. glycosidic linkages
(d) Nylon-6 has amide linkages

(a) Insecticide (b) Antihistamine


28. Under hydrolysis conditions, the compounds used for
(c) Analgesic (d) Antacid
preparation of linear polymer and for chain termination,
respectively are (2012)
23. Which of the following is not an antacid? (2015 Main)
(a) CH3SiCl 3 and Si(CH3 )4 (b) (CH3 )2 SiCl 2 and (CH3 )3 SiCl
(a) Aluminium hydroxide (b) Cimetidine
(c) (CH3 )SiCl 2 and CH3SiCl 3 (d) SiCl 4 and (CH3 )3 SiCl
(c) Phenelzine (d) Ranitidine
24. Which one is classified as a condensation polymer? 29. The correct functional group X and the reagent/reaction
(2014 Main) conditions Y in the following schemes are (2011)
(a) Dacron (b) Neoprene (i) Y
X—(CH2)4—X Condensation polymer
(c) Teflon (d) Acrylonitrile O O
(ii) C—(CH2)4—C
25. Among cellulose, poly (vinyl chloride), nylon and natural HO
heat OH
rubber, the polymer in which the intermolecular force of (a) X = COOCH3, Y = H2/Ni/heat
attraction is weakest is (2009) (b) X = CONH2, Y = H2/Ni/heat
(a) nylon (b) poly (vinyl chloride) (c) X = CONH2, Y = Br2/NaOH
(c) cellulose (d) natural rubber (d) X = CN, Y = H2/Ni/heat
Biomolecules and Chemistry in Everyday Life 491

Match the Columns (a) Deduce the structure of A.


30. Match the chemical substances in Column I with type of (b) Write the structure of ‘all cis’ – form of polymer of
polymers/type of bond in Column II. (2007, 6M)
compound A.
32. Name the heterogeneous catalyst used in the polymerisation
Column I Column II
of ethylene. (2003)
A. Cellulose p. Natural polymer
B. Nylon-66 q. Synthetic polymer 33. Give the structures of the products in the following reaction.
(2000, 2M)
C. Protein r. Amide linkage
NOH
D. Sucrose s. Glycoside linkage

H+ Polymerisation
Subjective Questions C *—D—n*
31. Monomer A of a polymer on ozonolysis yields two moles of
HCHO and one mole of CH3COCHO. (2005)

Answers
Topic 1 Topic 2
1. (a) 2. (b) 3. (a) 4. (d) 1. (b) 2. (d) 3. (a) 4. (a)
5. (b) 6. (b) 7. (d) 8. (d) 5. (a) 6. (c) 7. (d) 8. (b)
9. (a) 10. (d) 11. (a) 12. (a) 9. (c) 10. (d) 11. (c) 12. (d)
13. (c) 14. (d) 15. (a) 16. (d) 13. (b) 14. (b) 15. (b) 16. (a)
17. (a) 18. (b) 19. (a) 20. (a) 17. (c) 18. (a) 19. (a) 20. (b)
21. (a) 22. (b) 23. (a) 24. (b)
21. (b) 22. (c) 23. (c) 24. (a)
25. (b) 26. (a,c,d) 27. (d) 28. (b,c)
25. (d) 26. (a) 27. (d) 28. (b)
29. (b,c) 30. (c) 31. (5) 32. (1)
29. (a, b, c, d)
33. (4) 34. (4) 35. (8) 36. (6)
30. A → p, s; B → q, r; C → p, r; D → s
37. (2)

Hints & Solutions


Topic 1 Biomolecules
1. Statement (a) is incorrect. Glycogen is not a straight chain Fischer formula Number of stereo-centre (C* ) = 4.
polymer similar to amylose. It is highly branched structure Cyclic structure of glucose are as follows :
similar to amylopectin. It is known to be the storage material of
CH2OH CH2OH
animals. It is found in liver, muscles and brain. It breaks down to
glucose by the action of enzymes when body needs a glucose. It O O
H H H H H OH
is also found in yeast and fungi.
2. RNA does not have double stranded α-helix structure. Helixes HO OH H OH HO OH H H
present in RNA are single-stranded but sometimes they fold
back on themselves to form a double helix structure. RNA H OH H OH
usually does not replicate. α-D-glucose β-D-glucose
It is present in the nucleus of the cell. It controls the synthesis of
protein. RNA molecules are of three types, i.e. messenger’s Haworth formula Number of stereo-centre (C* ) in each anomer = 5.
RNA (m-RNA), ribosomal RNA (rRNA), transfer RNA (t-RNA). 4. Amylopectin is the water-soluble component of starch. It is a
3. Key Idea Chiral centre is also called stereo-centre or branched-chain polymer of α-D-glucose. The main chain
stereogenic center. consists of an α − 1, 4′- glycosidic linkages between α - D-
glucose units and the branches are connected to the main chain by
Linear structure of glucose is as follows : α - 1,6′- glycosidic linkages. Its structure can be represented as:
CHO
H OH
HO H
H OH
H OH
CH2OH
492 Biomolecules and Chemistry in Everyday Life

6 CH2OH CH2OH CH2OH


4 CH2OH O O O O
O 2 H H H H H H OH
5 H H Dil. HCl H
HO 1 2
3 OH α-1, 6′-glycosidic linkage (Branch) OH H O OH H OH H H
O OH OH OH
6 6
4 CH2 O 4 CH2OH
O H OH H OH
2 2 H
H OH
5 H 5 α-D-glucose
HO HO 1
3 OH 3 OH O
O 9. Histidine has following structure in
α-1, 4′-glycosidic linkage (Chain)
Structure of Amylopectin H 2N CH C OH

5. The peptide that gives positive cerric ammonium nitrate and CH2 O
carbylamine tests is ser - lys. The structures of serine and lysine Lone pairs of
are, both of these These lone pairs are
N not free for donation as
nitrogen are
HOCH2 C H  COOH, H2N(CH2)4 C H  COOH freely available N H these are delocalised
| | for donation with-p bonds.
NH2 NH2
Serine Lysine At highly acidic pH, i.e. 2 both the nitrogens with lone pairs will
6. Statement-(b) is not true for sucrose. It is linked through a accept one H+ each and  C  OH will not loose its H+ . Thus,
glycosidic linkage between C-1 of α-glucose and C-2 of 
β-fructose. Since, the reducing groups of glucose and fructose are O
involved in glycosidic bond formation, sucrose is a non-reducing the final structure of histidine at pH = 2 will be
sugar. +
H3 N CH C OH
CH2OH
O CH2 O
H H HOH2C O
H H
α β +
OH H H N
HO Glycosidic
linkage H HO CH2OH NH
H OH
Thus, option (d) is the correct answer.
α-D-glucose OH H
Note Amino acids have following generalise structure:
β-D-fructose

On hydrolysis with acids or enzyme, sucrose gives equimolar R C COOH


mixture of D-(+)-glucose and D-(–)-fructose.
NH2
HCl
C12H22O11 + H2O → C6H12O 6 + C6H12O 6
D-(+) -glucose D-(–)-fructose They have the tendency to loose H+ of their COOH group at
alkaline (higher) pH while the NH2 group present in them
7. Both fructose and glucose give following test positive. have the tendency to gain H+ at acidic (lower) pH.
(i) Fehling’s test (red ppt. of Cu 2O is obtained). 10. The correct match is :
(ii) Barfoed’s test (red ppt. of Cu 2O is obtained) A → (Q); (Β) → (S) (C) → (P)
(iii) Benedict’s test (red ppt. of Cu 2O is obtained)
Fehling’s solution : CuSO4 + Na, K-tartrate (Rochelle salt) R ⇒ CH2—COOH C H OH/H2SO4/∆
Asp [Aspartic acid] 2 5
Barfoed’s reagent (CH3COO )2 Cu + CH3COOH + H 2O (Q ) [Ester test] (A)
(7%) (1%) (92%) Sweet smell
of ester
Benedict’s solution : CuSO4 + Na-citrate + Na 2CO3 R ⇒ (CH2)4—NH2 CHCl /alc.KOH
3
Lys [Lysine]
Seliwanoff’s test is used to differentiate between ketose and (S) [Carbylamine test] (B)
OOC
aldose. The reagent is a solution of resorcinol in concentrated CH— R Foul smell
HCl. The reagent when heated along with a sugar will produce of isocyanide
H 3N
furfural or hydroxy-methylfurfural, which further reacts to give General
red color. Ketose (fructose) reacts more quickly than aldose formula of O, Conc. H2SO4/∆
(glucose). amino acid
R ⇒ ¾CH2 OH
8. Maltose on treatment with dil. HCl gives D-glucose. Hydrolysis [Phthalein dye test] (C)
Tyr [Tyrosine]
of maltose yields two moles of α- D-glucose. Thus, it is (P) Resultant colourless
composed of two α-D-glucose units in which C-1 of one glucose solution turns pink
unit (I) is linked to C-4 of another glucose unit (II). The free
aldehyde group can be produced at C-1 of second glucose in (A) Ester test confirms the presence of COOH group.
solution and it shows reducing properties. So, it is a reducing (B) Carbylamine test confirms the presence of NH2 group (1°).
sugar. (C) Phthalein dye test confirms the presence of phenolic OH
group.
Biomolecules and Chemistry in Everyday Life 493

11. RNA contains, adenine (A), guanine (G), cytosine (C) and uracil 14 Amino acid molecules can be represented as,
(U). In the given options, COOH
R CH
NH2
NH
is uracil (present in RNA only) Nature of the ‘R’ group will determine the basicity (hence, pK a )
of an amino acid.

Nature of
Nature
H 3C ‘R’ in the amino group of R the amino
NH acids
is thymine (present in DNA)
1. + Basic More basic (due
NH2 to the presence
 (CH2)3 NH  C
of acetamidine
NH2
NH2 group)
(Arginine : Arg)
N
is cytosine (present in both DNA and RNA) 2. O Acidic Acidic
CH2 C
O–
(Aspartic acid: Asp)
3. H (Glycine) : Gly Neutral Neutral
N¾Me +
4.  ( CH2 ) 4  NH3 Basic Basic
is not a pyrimidine base but a derivative of uracil.
(Lysine : Lys)
Me
12. (i) Barfoed test is used for detecting the presence of
monosaccharides like glucose, fructose etc. Barfoed reagents is
15. HI is a strong reducing agent. It reduces both primary and
secondary alcoholic groups of glucose along with the carbonyl
Cu (II) acetate solution.
group to produce n-hexane as
CHO COOH Carbonyl group
CHO CH3
H OH H OH Secondary HI
HO HO alcoholic (CH2)4
H H (CHOH)4 ∆
+ Cu2+ + 2H O + Cu2O +4H+ group
H OH (Cu (II)-acetale) 2 H OH (Prolonged)
(Red ppt.)
H OH H OH CH2OH Primary CH3
alcoholic
CH2OH CH2OH group
Glucose n-hexane
(D-glucose) (D-gluconic acid)
16. Our blood is slightly basic in nature with pH range from
(Ninhydrin Test)
(ii) Violet 7.35-7.4. The structure of histamine is given below :
Ninhydrin/ D
colouration Aliphatic
H amino group
(Xanthoprotic Test)
Amino acid Yellow
(Protein) Conc. HNO3 Imidazole N NH2
colouration ring
(Biuret Test) N
Violet
CuSO4 (aq)+NaOH
colouration
Basic nitrogen of imidazole ring
13. Formation of the tripeptide (Val-Ser-Thr) can be shown as: It is produced by decarboxylation of histidine having following
Me Me Me OH structure. It is clearly visible from the above structure that
CH O CH histamine has two basic centres namely aliphatic amino group
CH OH H HN C CH OH
and basic nitrogen of imidazole ring. The aliphatic amino group
H2N C CH OH H HN C has pK a around 9.4. In blood with pH around 7.4 the aliphatic
amino group of histamine become protonated to give a single
O CH2 O
OH charged cation as shown below
Valine (Val) Serine (Ser) Threonine (Thr) H
N +
Me Me –2H2O Me OH NH3
CH O CH
N
CH NH C CH OH
H2N C CH NH C
17. Sugars that have an aldehyde, a ketone, a hemiacetal or a
O CH2 O hemiketal group is able to reduce an oxidising agent. These
OH
sugars are classified as reducing sugars.
Val Ser Thr
494 Biomolecules and Chemistry in Everyday Life

HOH2C O HOH2C O 25. Both glucose and fructose are reducing sugars, reduces Tollen’s
CH2OH CH2OH
reagent to metallic silver.
aq·KOH
HO –+ OH 26. The explanation of given statements are as follows:
–CH3COOK
OCOCH3 O– (a) Two six membered cyclic hemiacetal form of
OH OH D-(+ )- glucose are called anomers.
Hemiacetal CH2OH CH2OH

H O OH H O H
O H H
HOH2C CH2OH OH H OH H
OH H OH OH
HO O
H OH H OH
a-D(+)-glucopyranose b-D(+)-glucopyranose
OH Both are anomers.
(a-hydroxyketone)
(b) Oxidation of glucose in presence of Br2 water gives
Hemiacetal can be easily reduced by oxidising agent such as gluconic acid.
Tollen’s reagent. CHO COOH
O H OH H OH
HOH2C CH2OH HO H Br2 HO H
Tollen’s
O positive silver H OH Water H OH
OH reagent
mirror test. H OH H OH
CH2OH CH2OH
OH D-(+)-glucose Gluconic acid
(Reducing sugar)
(c) Monosaccharides can not be hydrolysed into
18. NH2 polyhydroxy aldehydes and ketones.
Cystine HO S COOH (d) Hydrolysis of sucrose gives D-glucose and L-fructose.
C S Invertase
C12H22O11 + H2O → C6 H12O6 + C6H12O6
O NH2 D-glucose L-fructose or
or dextrorotatory laevorotatory
COOH
Cysteine Hence, options (a, c, d) are correct.
HS
NH2
NH2
O Cytosine 27. A pyranose ring is a 6 membered ring having 5 carbon atoms and
one oxygen atom. In glucose, it is formed by the reaction between
N
S C >C=O group at position 1 and —OH group at 5th carbon atom. In
Methionine
H 3C OH , N general reaction between >C=O group and —C—OH looks like
O
NH2 H
OH
Thiol group (SH) is present in cysteine. C O+ C OH C
O C
19. Vitamin B and C are water soluble while vitamin A,D,E and Carbonyl Alcoholic Hemiacetal or
K are fat soluble or water insoluble. group group Hemiketal

20. Quinoline is an alkaloid, it is not present in DNA. DNA has The product formed in called hemiacetal
four nitrogen bases in adenine, guanine, cytosine and thymine. (if>C=O group belongs to an aldehyde) or hemiketal (if >C=O
group belongs to a ketone). L- glucose has the mirror image
21. 18 ATPs are involved in the formation of 1 glucose molecule
configuration of D-glucose i.e.,
as shown in the reaction below :
6CO2 + 12NADPH + 18 ATP → H O H O
C1 C1
C6 H12 O6 + 12NADP + 18 ADP
H C2 OH HO C2 H
22. Here, the — OH of hemiacetal group is equatorial therefore, it
is a β-pyranose of an aldohexose. HO C3 H H C3 OH
23. The six-membered cyclic ether is known as pyranose while H C4 OH HO C4 H
the five membered cyclic ether is known as furanose. Hence,
ring (a) is a pyranose and it has ether linkage at α-position that H C5 OH HO C5 H
is known as α-glycosidic linkage in carbohydrate chemistry.
6CH2OH 6CH2OH
24. “α” and “β” cyclic hemiacetals of D-glucose having D-glucose L-glucose
difference in configuration at C-1 only are called anomers.
Biomolecules and Chemistry in Everyday Life 495

So, β-L glucopyranose is formed as COOH of glutamic acid has (− 1), COOH of lysine (− 1) and OH
H
of phenol (− 1).
H
5 O H 5 O Therefore, | Z3 | = |− 3| = 3 (All COOH and OH exist in the form of
HO 6 CH OH HO CH2OH H —COO− and — O− ).
4 2 1C O 4 6 1
H OH H H H OH OH ∴ | Z1 | + | Z2 | + | Z3 | = 2 + 0 + 3 = 5
H
3 2 3 2 32. PLAN This problem can be solved by performing hydrolysis of peptide
OH H OH H and deciding the nature of product.
β-L-glucopyranose Chemical reaction and product formed after hydrolysis of given
The α-L-glucopyranose has configurational change at C1 only peptide can be represented as
and looks like O
H
H H O H O N
O O
O
HO HO Configuration at N N
CH2OH N NH—C NH N
this carbon atom
1
H OH H is opposite in α CH2 O O H CH2 O
H N
and β-forms H
OH H O
α-L-glucopyranose
H3O+
CMe3
28. If there is inversion of specific rotation from (+ ) to (−), then + +

invert sugar is formed. H3N—CH2—CO2+ + O2C—CH—NH3
(A) (B)
(a ) C12H22O11 + H2O → Glucose +
(+) Maltose D(+)
52 ° H 3N CO2
140° HO—C—CHNH—CH2—C—OH
(b) C12H22O11 + H2O → Glucose + Fructose
(+) Sucrose D(+) L(−) CH2 + O CH2 O
+ 66 ° 52 ° −92 °
−40 ° for 2 moles mixture
−20 ° for 1 mole mixture
There is formation of invert sugar. Thus, correct.
(D )
(c) Specific rotation of invert sugar is −20 ° per mole. Thus, (C )

correct. (A) is glycine which is only naturally occurring amino acid. While
(d) Br2 water is a weak oxidising agent. It oxidises CHO to (B), (C) and (D) are not the naturally occurring amino acids.
COOH. CH2OH group is not affected. Hence, correct integer is (1).
COOH CHO COOH
33. PLAN A peptide linkage is hydrolysed to two free amino acids.
  Br2 water

HNO
(CHOH)4 ←3  (CHOH)4 → (CHOH)4 O
   ||
H2N  CH  C  NH  CH  COOH
COOH CH2OH CH2OH | |
Saccharic acid Gluconic acid R R′
(one of the products) ↑
HNO3 (a strong oxidising agent) oxidises invert sugar to Peptide
saccharic acid. Thus, incorrect. O
* || *
29. X is acetal, has no free hemiacetal, hence a non-reducing sugar H2N  CH  C  OH + H2N  CH  COOH
while Y has a free hemiacetal group, it is reducing sugar. Also, | |
glucosidic linkage of X is ‘α’ while that of Y is β-linkage. R R′

30. Statement I is correct Presence of — CHO group in glucose is C* is chiral carbon tetrapeptide has four amino acids joined by
tested by Fehling’s solution test where a reddish-brown three peptide linkage.
precipitate of Cu 2O is formed.  COOH group is on alanine part, thus it is at fixed C-terminal
Hence, Statement II is incorrect. position in each combination.
31. At pH = 2, Glycine is optically inactive thus it cannot be on the N
terminal side. Thus, possible combinations are
There are two — NH2 group, and + 1 charge on each group
because all amino groups exist in the form of — NH⊕ Phe-Gly-Val-Ala, Phe-Val-Gly-Ala,
3.
Val-Gly-Phe-Ala, Val-Phe-Gly-Ala
Therefore, | Z1 | = 2 .
Thus, in all four combinations are possible.
At pH = 6,
NH2 of lysine (+ 1) (pH = 9.47) and COOH (−1) of glutamic acid 34. The amino acid remain completely in Zwitter ionic form at its
(pH = 3.08), so because of dipolar ion exists, therefore | Z2 | = 0. isoelectric point. Amino acids with additional acidic group have
At pH =11, their isoelectric pH less than 7.0 and increasing pH above
isoelectric point makes them anionic.
496 Biomolecules and Chemistry in Everyday Life

On the other hand, amino acids with additional basic group have CHO COOH
their isoelectric pH greater than 7.0 and decreasing pH below OH H
H OH
isoelectric point (by adding acid solution) makes them cationic.
The given peptide with followings R1 and R2 are basic, will HO H +
Ag(NH3)2 HO H
remain protonated (cationic) at pH = 7.0. (ii)
H OH H OH
Peptide R1 R2
IV CH 2CONH 2 (CH 2 ) 4 NH 4 H OH H OH
VI (CH 2 ) 4 NH 2 (CH 2 ) 4 NH 4 CH2OH CH2OH
VIII CH 2OH (CH 2 ) 4 NH 4 gluconic acid
IX (CH 2 ) 4 NH 2 CH 3 40. The dipeptides are
Thus, 4 is the correct integer. HOOC  (CH2 )2  CH  CO  NH  CH  (CH2 )4  NH2
 
35. The D-form of given sugar is NH3
+
COO−
CHO and
CH2OH HOOC  (CH2 )2  CH  NH  CO  CH  (CH2 )4  NH2
CH2  
H O OH COO− NH3
+
CHOH Cyclisation H
*
H
* O CH2C6H5
CHOH  
OH H 41. H2N  CH  C  NH  CH  COOCH3
H OH * 
H CH2  COOH
CH2OH
D-Pyranose Aspartame
D- Sugar
(i) Aspartame has amine, acid, amide and ester groups.
Configurations at the three chiral carbons (starred) can be changed
O CH2C6H5
maintaining D-configuration. Hence, the total number of
+  
steroisomers of D-pyranose = 23 = 8 (ii) H3 N  CH  C  NH  CH  COOCH3
Thus, the correct integer is 8. 
CH2  COO −
36. A decapeptide has nine peptide (amide) linkage as H+
O O O O O (iii) Aspartame → H2N  CH  COOH +
H2 O
O O O O O 
CH2 COOH
I
—CONH— CH2C6H5
bond 
Therefore, on hydrolysis, it will absorb nine water molecules. H2N  CH  COOH + CH3OH
II
Hence, total mass of hydrolysis product = 796 +18 × 9 = 958
(iv) II is more hydrophobic due to the presence of phenyl group.
⇒ mass of glycine in hydrolysis product = 958 × 47 = 450 H+
100 42. Sucrose → D-glucose + D-fructose
⇒ number of glycine molecule in one H2 O
450 CHO
molecule of decapeptide = =6
75 CO2OH
− H OH
37. — OO and —NH2 are two basic groups in lysine.
C O
38. In structure (a), one ring has a free hemiacetal group, will HO H
hydrolyse into open chain in aqueous solution and therefore will HO H
reduce Tollen’s reagent. Structure (b) has only acetal groups, H OH
H OH
will not hydrolyse in aqueous solution into open chain, will not
reduce Tollen’s reagent H OH
H OH
39. (i) D-glucose and L-glucose are enantiomers, hence CH2OH
CH2OH
CHO Mirror CHO pH = 10
H OH HO H 43. H2N  CH  COO− → H2N  CH  COOH
 
HO H H OH CH3 CH3
H OH HO H Alanine
H OH HO H pH = 2 +
→ H3N  CH  COOH
CH2OH CH2OH 
D-glucose L-glucose CH3
Biomolecules and Chemistry in Everyday Life 497

Topic 2 Chemistry in Everyday Life 5. Nylon-6, 6 (an amide) is a condensation copolymer because it is
obtained by condensation between adipic acid and
1. The antifertility drug “novestrol” or ethinylestradiol has the hexamethylenediamine.
following structure :
O O
H
OH nHO—C—(CH2)4—C—OH + nN—(CH2)6—NHH
CH3 C CH
H Hexamethylene –n H 2 O
Adipic acid
diamine
H O O

——C—(CH2)4—C—NH—(CH2)6—NH——
1 mol n
H H 3°OH group responds nylon-6,6
to Lucas test
HO
(anhydrate ZnCl2 /CHCl) Neoprene, teflon and buna-S are addition polymers.
to and immediately
turbidity will be observed. 6. Noradrenaline is one of the example of neurotransmitters. It
Phenolic  OH group responds plays a major role in mood changes. If the level of noradrenaline
to Br2/H2O to give white ppt. is low for some reason, then signal-sending activity becomes low
and phenolic  OH group gives and the person suffers from depression.
violet coloration with O
neutral FeCl3 . ||
7. Monomer of [NH  C  NH CH2 ]n is formaldehyde.
So, the correct combination of reagents is in option (b). The polymer is also known as urea-formaldehyde resin. It is
i.e. Br 2/water, ZnCl 2/HCl and FeCl 3. made from urea (NH2CONH2) and formaldehyde (HCHO).
Polymerisation
2. Synthetic Brompheniramine is an anti-histamine that is used to NH 2CONH 2 + HCHO →
Formaldehyde
treat sneezing, runny nose and itchy or watery eyes caused by Urea
allergies, hay fever and the common cold. O
||
3. Bakelite is a thermosetting polymer. These polymers are (NH  C NH CH 2 )
n
cross-linked or heavily branched molecules which on heating Urea-formaldehyde resin
undergo extensive cross linking in moulds and become infusible.
Once they get set, they cannot be reshaped and reused. It is used for making unbreakable cups and laminated sheets.
[TiCl + (C H ) Al] 8. O
4. (A) n CH2 == CH2 →
4 2 5 3

[ CH2 
]n
Ziegler-Natta catalyst High density
Ethylene (III) Polymerisation
polythene (HDPE)
NH
CN CN
| O22 − /O2 
(B) nCH2 == CH →  [ CH2 C H ]
n Caprolactum
(Catalyst) Polyacrylonitrile
Acrylonitrile (I)
(PAN) O O
(C) OH OH
—C—NH—(CH2)5 —C—NH—(CH3)5—
CH2OH
sOH or
+ HCHO Polymerisation/
Acid or H3Or O
Formaldehyde
base catalyst
Phenol (IV)
—NH—(CH2)5—C—
n
OH OH Nylon-6

CH2 Nylon-6 is prepared by ring opening polymerisation of


caprolactum. It is heated about 533 K in an inert atmospheric
n nitrogen about 4-5 hrs. Nylon-6 fibres are tough, possessing high
Novolac tensile strength, as well as elasticity and lustre. They are wrinkle
proof and highly resistant to abrasion and chemicals such as
(D) O acids and alkalis.
NH
550 K
O 20 atm —C—NH—(CH2)5— 9. Nylon-6,6 has following structure:
–H2O Nylon-6 n  O O 
Caprolactum (II)    
Thus, the correct match is as follows :  N  (CH2)6  N  C  (CH2)4  C 
 
   
(A) → (III), (B) → (I), (C) → (IV), (D) → (II)  H 
 H n
6 C-atoms 6 C-atoms
498 Biomolecules and Chemistry in Everyday Life

As it is a condensation polymer hence, each of its monomeric form HOOC (CH2 )  OH that undergoes polymerisation to
unit must contain 6 carbon atoms in them. Hence, a combination give polymer.
of adipic acid and hexamethylene diamine is the correct answer. NaNO2/H3O+
Both of these units react as follows to form nylon-6, 6. HOOC—(CH2)4—NH2 HOOC—(CH2)4—N2
(Diazotisation)
O Aliphatic diazonium ion
(Unstable)
–N2
nH N (CH2)6 N H + OH C (CH2)4 C OH +
HOOC—(CH2)3—CH2
H H –n H 2 O H Adipic acid –H –H2O
Hexamethylene O Polymerisation
diamine C—(CH2)4—O H HOOC—(CH2)4—OH
N (CH2)6 N C (CH2)4 C OH OH
–H2O
H H O H O—(CH2)4—C == O
n
Nylon-6, 6

10. Poly-β-hydroxy butyrate Co-β-hydroxyvalerate (PHBV) is a -----O—(CH2)4—C—O—(CH2)4—C---- or —O—(CH2)4—C—


n
copolymer of 3-hydroxybutanoic acid and 3-hydroxypentanoic (An ester group) (Polymer)
acid. It is used in speciality packaging, orthopaedic devices and in
controlled release of drugs. PHBV undergoes bacterial degradation 14. The correct match is:
in the environment. The reaction involved is as follows : A → (Q) B → (P) C → (R)
OH OH (A) Norethindrone It is an antifertility drug(Q ) containing
  synthetic progesterone derivative. [Other similar drug, is
CH3 CH  CH2COOH + CH3CH2 CHCH2COOH → ethinylestradiol (novestrol)].
3-hydroxybutanoic acid 3-hydroxypentanoic acid (B) Ofloxacin It is an antibiotic (P ),
 O CH2CH 3  i.e produced wholly or partly by chemical synthesis with low
 
   
concentration of microorganism.[Some other similar drugs :
 O CH  C H2 C  O CH  CH2  C  Penicillin, chloramphenicol, salvarsan etc.]
   (C) Equanil (meprobamate) It is a mild tranquilizer for
 CH3 O n
 relieving hypertension. It relieve anxiety, stress, excitement by
inducing a sense of well being.
(PHBV)
(Other similar drug is chlordiazepoxide.)
11. On polymerisation, 4-hydroxy butanoic acid will produce a
condensation homopolymer by loss of H2O molecules. 15.
Phenolic
OH –OH group
FeCl3
HO—C—CH2CH2CH2—O H + HO —C—(CH2)3—O H (Ferric chloride)
(A) Violet colouration
4-hydroxy butanoic acid (R)
–H2O CH3
H 3C
Cl
Chloroxylenol (Dettol)
-----—C—(CH2)2CH2—O—C—(CH2)2CH2—O----
Ester linkage OH
H 3C C≡≡CH
The homopolymer obtained can also be represented as H Dil. KMnO4/5°C/OH –
H Pink colour of
O (Baeyer’s test)
 (B)
H H (S)
KMnO4 gets

[ C — (CH2)3 O —]n discharged
O
Norethindrone
12. (A) Molecule binding to a site other than the active site of
enzyme is called allosteric effect. C C
(Double bond)
(B) Molecule binding to the active site of enzyme is called
competitive inhibitor.
O
(C) Molecule crucial for communication in the body is called
receptor. (C) H2N S N
(D) Molecule binding to the enzyme covalently is called poison. H N
O
Thus, the correct match is : A → R, B → P, C → Q, D → S Primary Sulphapyridine
amine (–NH2)
+ CHCl3 + KOH (alc.)
13. Given amino acid on reaction with NaNO 2 / H3O gives Foul smell
(Carbylamine
diazotisation reaction which further evolves N 2 gas along test) (P) of isocyanide
with formation of carbocation. On further reaction with water, it
Biomolecules and Chemistry in Everyday Life 499

O
H H CH3 20. (a) Bakelite is used for making gears, protective coating and
S
(D) R C NH electrical fittings.
CH3
(b) Glyptal is used in the manufacture of paints and lacquers.
N COOH
O (c) PP is used in the manufacture of textile, packaging materials
H
Penicillin etc.
NaHCO3(aq)
Becomes soluble
(d) Polyvinyl chloride (PVC) is used in the manufacture of rain
(Sodium
with effervescence coats, hand bags, leather clothes etc.
hydrogen
carbonate) of CO2 gas 21. (a) Polystyreme- manufacturing toys (b) Glyptal- Paints and
(Q)
lacquers
Thus, the correct match is: A → R; B → S; C → P; D → Q (c) Polyvinyl chloride (PVC)- Raincoats (d) Bakelite- computer
discs
16. Nylon-6 or perlon is prepared by polymerisation of amino
Thus, the correct match is A → (1), B → (1), C → (2),
caproic acid at high temperature. Caprolactam is first hydrolysed
D → (4)
with water to form amino acid which on heating undergoes
polymerisation to give nylon-6. 22. The given structure is of aspirin which is used as analgesic.
O 23. Aluminium hydroxide Al(OH)3, cimetidine and ranitidine are
O
NH Hydrolysis antacids while phenelzine is not.
+ –
H3N—(CH2)5—C—O H
N N
Caprolactam D Polymerisation
O S C
[ HN— (CH2)5—C —
]n N N NHCN
Nylon-6 H
Cimetidine

17. High density polythene is used in the manufacture of buckets, CH(NO2)


dustbins etc.
Me2N S C
18. Sodium lauryl sulphate [(CH3 (CH2 )10 CH2OSO−3 Na + )] = N NHMe
Anionic detergent O
H
Ranitidine
Cetyltrimethyl ammonium bromide
+ Phenelzine is a tranquilizer, not an antacid.
 CH3 
   H
CH3 (CH2 )15  N  CH3  Br − = Cationic detergent N  ΝΗ2
  
 CH 
 3 
Glyceryl oleate [(C17 H32COO)3 C3H5 ] = Non-ionic detergent
Phenelzine is used as antidepressant drug.
Sodium stearate [C17 H35COO− Na + ] = Anionic soap
19. Natural rubber is formed by polymerisation of isoprene. 24. Dacron is a condensation polymer of ethylene glycol and methyl
terepthalate. Formation of dacron can be shown as

CH2 C—CH CH2 CH2—C CH—CH2 MeO — C C—OMe + nHO — CH2—CH2—OH


CH3 CH3 O O
Polymerise
Ethylene
Polymerisation
—O—C C—OCH2—CH2—
(n-1) MeOH
—CH2—CH— CH2—CH2— —CH2—C CH—CH2— O
Propylene

H2/Pt O n
Dacron
CH3 CH3
n n Here, elimination of MeOH occurs as a by product. So, this
Natural rubber reaction is known as condensation polymerisation.
Ethylene-propylene copolymer

25. Cellulose and nylons have H-bonding type of intermolecular


This co-polymer is formed from propylene and ethylene. attraction while poly (vinyl chloride) is polar. Natural rubber is
hydrocarbon and has the weakest intermolecular force of
n CH2 == CH + nCH2 == CH2 → —CH2—CH CH2—CH2— attraction, i.e. van der Waals’ force of attraction.

CH3 CH3
n
500 Biomolecules and Chemistry in Everyday Life

H OH CH2OH
CH2OAc CH2OAc CH2OAc
C C H H C O O O O
26. H O H O H O
OH H (CH3CO)2O H H H
C C C C
O OH H H
OAc H H OAc H H OAc H
H O
H C O C C
H OH H OAc H OAc H OAc
CHOH
Tri-acetylated cellulose Tri-acetylated cellulose

27. The explanation of given statements are as follows: (a) Teflon is prepared by heating tetrafluoroethene in presence of persulphate catalyst at
higher pressure.
Persulphate
nCF2 ==CF2 catalyst
CF2 CF2
Thermoplastic Polymer
(PTFE)

Thus, option (a) is correct.


(b) Natural rubber is polyisoprene containing cis alkene units.

CH2 CH2 n
nCH2 C CH CH2 C C
CH3 H
CH3 cis-ployisoprene unit
Isoprene (natural rubber)

Thus, option (b) is incorrect.


(c) Cellulose has only β-D-glucose units that are joined together by glycosidic linkages as shown in the following structure:

OH OH OH OH

O O O
O OH O
O OH 1 4 OH
, OH O
O

OH OH OH O
n OH
(cellulose)
(1, 4, b -linkage)
Thus, option (c) is incorrect.
(d) Nylon-6 has amide linkages.
O

C O
NH ∆, H2O – n H 2O
[H2N(CH2)5COOH] C (CH2)5 NH

Nylon-6 n
Caprolactam

Thus, option (d) is correct.


28. CH3 CH3 CH3 CH3
Polymerisations (CH3)3 XSiCl
(CH3)2SiCl2 + H2O HO—Si—OH HO—Si—O—Si—O —Si—OH —Si—O —Si— O—Si—O —Si—O —Si— CH3

CH3 CH3 CH3 CH3 n


CH3 CH3
n
Linear chain polymer Chain terminated linear chain silicone

29. (a) When X = COOCH3


H 2 / Ni HOOC—(CH 2 ) 4 —COOH
CH3OOC — (CH2 )4 — COOCH3  → HOCH2 — (CH2 )4 — CH2OH + 2CH3OH →
Heat Heat
O O

—O—(CH2)6—O—C—(CH2)4—C—
n
Ester, condensation polymer
Biomolecules and Chemistry in Everyday Life 501

(b) When X = CONH2


O O

H / Ni HOOC—(CH 2 ) 4 —COOH —HN—(CH2)6—NH—C—(CH2)4—C—


H2NOC — (CH2 )4 — CONH2  
2
→ H2N  (CH2 )6 — NH2 → n
Heat Heat Nylon, condensation polymer
O O
Br2 HOOC—(CH 2 ) 4 —COOH
(c) H2NOC — (CH2 )4 — CONH2 →
NaOH
H2N— (CH2 )4 — NH2 → —HN—(CH2)4—NH—C—(CH2)4—C—
Heat n
Hofmann's bromamide
reaction Nylon, condensation polymer

O O
H 2 / Ni HOOC—(CH 2 ) 4 —COOH
(d) When X = CN NC — (CH2 )4 — CN  → H2N(CH2 )6 NH2 → —HN—(CH2)6—NH—C—(CH2)4—C—
Heat Heat n
In author’s opinion (a) and (b) should also be the answer. Nylon, condensation polymer

O O O O
Heat
(a) HOCH2—(CH2)4—CH2OH + HO—C— (CH2)4—C—OH —O—(CH2)6 —O—C—(CH2)4—C—
n
Polyester, a condensation polymer

O O
H2/Ni
(b) H2N—C—(CH2)4—C—NH2 H2NCH2—(CH2)4—CH2NH2
Heat

O O O O
Heat
H2NCH2—(CH2)4—CH2NH2 + HO—C—(CH2)4—C—OH —NH—(CH2)6 —NH—C—(CH2)4—C—
n
Nylon, a condensation polymer

27. (A) Cellulose—a natural polymer of α-D-glucose, linked by glycoside linkage.


(B) Nylon-6, 6—a synthetic polymer of adipic acid and 1,6-diaminohexane. The diacid is linked with diamine through amide linkage.
(C) Protein—a natural polymer of α-amino acids where individual amino acid units are linked by amide linkage.
(D) Sucrose—has glycoside linkage, a disaccharide.
CH3 O
 O3 
28. (a) H2C == C  CH == CH2 → 2HCHO + CH3  C  CHO
‘ A ’ Isoprene Zn − H2 O
H2C CH2  CH2 CH2
(b) Isoprene → C == C C == C
H3C H H3C H
natural rubber
29. Zeigler-Natta catalyst, which is a mixture of triethylaluminium ‘(C2H5 )3 Al’ and TiCl 4, is used as heterogeneous catalyst in polymerisation of
ethylene.
30. N—OH
O O
Polymerisation
NH [–C—(CH2)5—NH–]
n
Nylon-6 (D)
Caprolactum
33
Environmental Chemistry
(a) 600 - 750 nm (b) 400 - 550 nm
Objective Questions I (Only one correct option) (c) 0.8 - 1.5 nm (d) 200 - 315 nm
1. The primary pollutant that leads to photochemical smog is 11. The compound that is not a common component of
(2019 Main, 12 April II) photochemical smog is (2019 Main, 12 Jan II)
(a) acrolein (b) nitrogen oxides (a) CF2Cl 2 (b) H3C  C  OONO 2
(c) ozone (d) sulphur dioxide 
2. The correct set of species responsible for the photochemical O
smog is (2019 Main, 12 April I) (c) CH2 == CHCHO (d) O 3
(a) N2 , NO2 and hydrocarbons 12. Water samples with BOD values of 4 ppm and 18 ppm,
(b) CO2 , NO2 , SO2 and hydrocarbons respectively, are (2019 Main, 12 Jan I)
(c) NO, NO2 , O3 and hydrocarbons (a) clean and clean
(d) N2 , O2 , O3 and hydrocarbons (b) highly polluted and clean
3. Air pollution that occurs in sunlight is (2019 Main, 10 April II) (c) highly polluted and highly polluted
(a) acid rain (b) oxidising smog (d) clean and highly polluted
(c) fog (d) reducing smog 13. The molecule that has minimum/no role in the formation of
4. The regions of the atmosphere, where clouds form and where photochemical smog, is (2019 Main, 12 Jan I)
we live, respectively, are (2019 Main, 10 April I) (a) N2 (b) CH2 == O (c) NO (d) O 3
(a) stratosphere and stratosphere (b) troposphere and troposphere 14. Taj Mahal is being slowly disfigured and discoloured. This is
(c) troposphere and stratosphere (d) stratosphere and troposphere primarily due to (2019 Main, 11 Jan II)
5. The layer of atmosphere between 10 km to 50 km above the sea (a) water pollution (b) soil pollution
level is called as (2019 Main, 9 April II) (c) global warming (d) acid rain
(a) stratosphere (b) mesosphere 15. The higher concentration of which gas in air can cause stiffness
(c) thermosphere (d) troposphere of flower buds? (2019 Main, 11 Jan II)
6. Excessive release of CO2 into the atmosphere results in (a) SO 2 (b) CO (c) CO 2 (d) NO 2
(2019 Main, 9 April I) 16. Peroxyacetyl nitrate (PAN), an eye irritant is produced by
(a) formation of smog (b) depletion of ozone (2019 Main, 11 Jan I)
(c) polar vortex (d) global warming (a) organic waste (b) acid rain
7. The maximum prescribed concentration of copper in drinking (c) classical smog (d) photochemical smog
water is (2019 Main, 8 April II) 17. The concentration of dissolved oxygen (DO) in cold water can
(a) 5 ppm (b) 0.5 ppm (c) 0.05 ppm (d) 3 ppm go upto (2019 Main, 11 Jan I)
8. Assertion (A) Ozone is destroyed by CFCs in the upper (a) 14 ppm (b) 10 ppm (c) 8 ppm (d) 16 ppm
stratosphere. 18. The reaction that is not involved in the ozone layer depletion
Reason (R) Ozone holes increase the amount of UV radiation mechanism in the stratosphere is (2019 Main, 10 Jan II)
reaching the earth. (2019 Main, 8 April I) (a) CH4 + 2O 3 → 3CH2 == O + 3H2O
(a) Assertion and Reason are incorrect. • •
(b) Assertion and Reason are both correct and the Reason is the correct (b) Cl O(g ) + O(g ) → C l (g ) + O2 (g )
hν • •
explanation for the Assertion. (c) HOCl(g ) → O H(g ) + Cl(g )
(c) Assertion and Reason are correct, but the Reason is not the • •

explanation for the Assertion. (d) CF2Cl 2 (g ) → Cl(g ) + CF2Cl(g )
(d) Assertion is false, but the Reason is correct.
19. Water filled in two glasses A and B have BOD values of 10 and
9. Which is wrong with respect to our responsibility as a human 20, respectively. The correct statement regarding them, is
being to protect our environment? (2019 Main, 8 April I) (2019 Main, 10 Jan I)
(a) Restricting the use of vehicles (a) A is more polluted than B
(b) Avoiding the use of floodlighted facilities (b) A is suitable for drinking, wherease B is not
(c) Setting up compost tin in gardens (c) Both A and B are suitable for drinking
(d) Using plastic bags (d) B is more polluted than A
10. The upper stratosphere consisting of the ozone layer protects us 20. The pH of rain water, is approximately (2019 Main, 9 Jan II)
from the sun’s radiation that falls in the wavelength region of (a) 7.5 (b) 6.5 (c) 5.6 (d) 7.0
(2019 Main, 12 Jan II)
Environmental Chemistry 503

21. The condition for methemoglobinemia by drinking water is 22. A water sample has ppm level concentration of the following
(2019 Main, 9 Jan II) metals: Fe = 0.2; Mn = 5.0 ; Cu = 3.0; Zn = 5.0. The metal that
(a) > 50 ppm nitrate (b) > 50 ppm chloride makes the water sample unsuitable for drinking is
(c) > 50 ppm lead (d) > 100 ppm sulphate (2019 Main, 9 Jan I)
(a) Cu (b) Fe (c) Mn (d) Zn

Answers
1. (b) 2. (c) 3. (b) 4. (b) 13. (a) 14. (d) 15. (a) 16. (d)
5. (a) 6. (d) 7. (d) 8. (c) 17. (b) 18. (a) 19. (d) 20. (c)
9. (d) 10. (d) 11. (a) 12. (d) 21. (a) 22. (c)

Hints & Solutions


1. The primary pollutant that leads to photochemical smog is nitrogen 4. The lowest region of atmosphere is troposphere which extends upto
oxides. Burning of fossil fuels such as petrol and diesel in the height of 10 km (approx) from sea level. We live in the
automobiles, reaction between nitrogen and oxygen and other such tropospheric region. It contains air, water vapour and dust which can
reactions result in a variety of pollutants, two main of which are form clouds with the help of strong air movement.
hydrocarbons (unburnt fuel) and nitric oxide (NO). Above the troposphere, stratospheric region extends upto 50 km
N2 (g ) + O2 (g ) diesel
 → 2NO(g )
In petrol and from sea level. It contains mainly N2 , O2 , O3 and little water vapour.
engines
O3 in the stratosphere absorbs 99.5% of the sun’s harmful UV
When the concentration of these pollutants is sufficiently high, a raditions and thus protects the lives on the earth.
chain reaction initiate because of the interaction of sunlight with 5. The atmosphere between the heights 10 to 50 km above the sea level
oxides of nitrogen. is stratosphere. Atmosphere is not of the same thickness at heights.
2NO(g ) + O2 (g ) Sunlight
 → 2NO2 (g ) 6. The effect of release of CO2 gas into atmosphere is global warming.
NO2 (g ) hν
→ NO(g ) + [ O] 7. According to W.H.O. and US environmental protection agency
Nascent oxygen guidelines, maximum allowable concentration of metals in drinking
O3 (g ) + NO(g ) → NO2 (g ) + O2 (g ) water are as follows :
Brown gas
Metal Maximum concentration
2. The correct set of species responsible for the photochemical smog is (ppm or mg dm−3)
NO, NO2, O3 and hydrocarbons. Photochemical smog appears in Cd 0.005
warm, dry and sunny climate which are obtained by the action of
sunlight on unsaturated hydrocarbons and nitrogen oxides. Mn 0.05 (option-c)
Following reactions are involved during the formation of Al 0.2
photochemical smog. Fe 0.2
(i) N2 (g ) + O2 (g ) → 2NO(g ) Cu 3.0 (option-d)
(Originates from
burning of fossil fuels)
Zn 5.0 (option-a)
Sunlight
(ii) 2NO(g ) + O2 (g ) → 2NO2 (g ) 8. Ozone is destroyed by CFCs in the upper stratosphere.
These compounds ultimately reach the stratosphere where they get

NO2 (g ) → NO(g ) + [ O ] broken down by powerful UV radiations and release chlorine free
Nascent oxygen radical. The chlorine free radicals react with ozone and cause its
(iii) O (g ) + O2 (g ) - O (g ) 3
depletion by converting it into chlorine monoxide radical and
Reacts rapidly with NO molecular oxygen.
hν •
O3 (g ) + NO(g ) → NO2(g ) + O2 (g ) CF2Cl 2 (g ) → Cl (g ) + CF2Cl(g )
Brown gas (in high hν •
concentration form haze) CFCl 3 (g ) → CFCl 2 (g ) + Cl(g )
• •
3CH (g ) + 2O3 (g ) → 3CH2 == O(g ) Cl(g ) + O3 (g ) → Cl O(g ) + O2 (g )
4
(Unburnt hydrocarbon) Formaldehyde Ozone holes increase the amount of UV radiation reaching the earth.
+ CH2 == CHCH == O + H2O These radiations can cause skin cancer, sunburns, ageing of skin.
Acrolein 9. Using plastic bags is wrong with respect to responsibility as a human
3. In sunlight oxidising smog or photochemical smog or Los-Angeles being to protect our environment. Plastic bags are non-biodegradable in
smog is formed. This smog is brown in colour. It occurs in warm, dry nature. It remains in the environment as such and does not degraded
and sunny climate. In presence of sunlight, NOx (N-oxides), O2 and by bacteria. If it is not disposed properly then it may lead serious
unburnt hydrocarbons of air combine to produce photochemical threat to the environment. The activities that can be used to protect
smog which mainly contains peroxyacetyl nitrate (PAN). our environment are as follows:
O l
Restricting the use of vehicles.
||
CH3  C  O  O  NO2 (PAN)
l
Avoiding the use of flood lighted facilities.
l
Setting up compost tin in gardens.
504 Environmental Chemistry
10. Sun emits UV-radiations, which according to following EM 16. Molecular formula of peroxyacetyl nitrate (PAN) is
categorisation have the wavelength range from 1 nm to 400 nm. O

Type Wavelength range CH3  C  O  O  NO2. It is a secondary pollutant. It is present
Radio wave > 0.1 m in photochemical smog (oxidising or Los Angeles smog). PAN is a
Microwave 0.1 m to 1 mm powerful lachrymator or tear producer and it also causes breathing
troubles.
Infrared wave 1 mm to 700 nm
Visible rays 700 nm to 400 nm 17. Dissolved oxygen (DO) is the oxygen dissolved in water either from
atmosphere or by photosynthesis. The lower the concentration of DO
Ultraviolet rays 400 nm to 1 nm
in a water sample, the more polluted is the water sample.
−3
X-rays 1 nm to 10 nm The concentration range of dissolved oxygen (DO) in cold water
−3
Gamma rays < 10 nm reaches upto 10 ppm, but that in normal water (at room temperature)
is within 5 ppm.
Thus, option (d) with 200-315 nm range is the correct option.
11. Freons or CFCs or chlorofluoro carbons, i.e. CF2Cl 2 is not the 18. CH4 is not present in the stratosphere and also it cannot diffuse or
common component of photochemical smog. This smog is produced escape into the stratosphere like freon-12 (CF2Cl 2 ) from the
as the result of tropospheric pollution while freons are the atmosphere.
components of stratospheric pollution. These are infact considered as In the stratosphere, ozone layer depletion take place mainly by
the major cause of ozone layer depletion. chlorofluorocarbons (CFCs) like CF2Cl 2 and the mechanism of
12. The amount of oxygen required by bacteria to break down the organic ozone layer depletion can be shown as:
hν • •
matter present in a certain value of a sample of water is called
(i) CF2Cl2(g ) → C l(g ) + C F2Cl(g ) [Option, (d)]
biochemical oxygen demand (BOD). The amount of BOD in the water

is a measure of the amount of organic material in the water, in terms (ii) C l(g) + O3 (g ) → ClO• (g ) + O2
of how much oxygen will be required to break it down biologically.
Clean water would have BOD value of less than 5ppm whereas (iii) ClO• (g ) + O(g ) → Cl• (g ) + O2 (g ) [Option (b)]
highly polluted water would have BOD value of 17 ppm or more. •
BOD value of clean water = 4 ppm (iv) C l(g ) + H2O(g ) → HOCl(g ) + H• (g )
BOD value of highly polluted water = 18 ppm [Present in the
stratosphere]
13. N2 molecule has minimum role in the formation of photochemical •

smog. While CH2 == O, O3 and NO has major role. When fossil (v) HOCl(g ) → OH(g ) + Cl• (g ) [Option (c)]
fuels are burnt, a variety of pollutants are emitted. Two of them are
hydrocarbons (unburnt) and NO. When these pollutants build upto ⇒ One Cl • can destroy or deplete 105 O3 molecules.
high levels, a chain reaction occurs from their interaction with As (i) reaction is involved in the formation of photochemical smog,
sunlight. The reactions involved in the formation of photochemical not in ozone layer depletion. So option (a) is correct.
smog are as follows:
19. BOD is defined as the amount of oxygen required by bacteria to
NO2( g )  hν
→ NO( g )+ O( g ) break down the organic matter present in a certain volume of a
O( g )+ O2( g ) d
O3( g )
NO( g ) + O3 ( g ) → NO2( g ) + O2( g )
sample of water. Clean water or drinking water has a BOD value < 5
ppm.
O 3 reats with unburnt hydrocarbons to produce chemicals such as So, water filled with A, BOD = 10 ppm is polluted and water filled
formaldehyde, acrolein and PAN. with B, BOD = 20 ppm, is also polluted. But, B is more polluted than
3CH4 + 2O3 → 3CH2 == O + 3H2O A.
+ CH2 == CCH == O + CH3 C OONO2 20. In clean air, rain water picks up some acidic oxides like CO2 and SO2
 (obtained from volcanic eruptions). These substance make the rain
O slightly acidic (pH = 5.6 − 6).
(PAN)

14. Acid rain (pH = 3.5 − 5.6) constitutes strong acids like HNO3, 21. According to EEC (European Environment Commission), excess of
H2SO4 and H2SO3 which slowly react with marble (CaCO3 ) of Taj NO−3 (> 50 ppm) in drinking water may lead to methemoglobinemia
Mahal and make it disfigured and discoloured. Here, CaCO3 (‘Blue baby syndrome’). It also may cause stomach-cancer.
(marble) gets dissolved in acids.
22. For drinking water, the maximum recommended levels of some
2 H ⊕ (aq) 2+ metals, set by European Environment Commission (EEC) is
CaCO3 (s) → Ca (aq) + H2O(l ) + CO2 (g ) ↑
(Acid rain)
15. Organic pigments (colourents) present in flower buds retain their Metal Max. concentration in ppm
colour in the oxidised form of the pigment as their nature is itself Zn 5
oxidising in nature. When they comes in contact with moist SO2
(acid rain) of higher concentration, they get decoloured and stiff. Mn 0.05
SO2 + H2O → H2SO4 Fe 0.2
H2SO4 → H+ + HSO4− Cu 3
HSO−4 → H + +
SO24 − As the concentration of Mn in the given water sample is more than
Due to the release of H+ ion (acid), the flower get decoloured and stiff. the recommended concentration. Thus, it makes water unsuitable for
As a result, flower eventually falls off from plants. drinking.
JEE ADVANCED
Solved Paper 2021
Paper 1
SECTION 1
l This section contains FOUR (04) questions.
l Each question has FOUR options (a), (b), (c) and (d). ONLY ONE of these four options is the correct answer.
l For each question, choose the option corresponding to the correct answer.
l Answer to each question will be evaluated according to the following marking scheme:
Full Marks : +3 If ONLY the correct option is chosen;
Zero Marks : 0 If none of the options is chosen (i.e. the question is unanswered);
Negative Marks : -1 In all other cases.

1. The major product formed in the following reaction is 3. For the given close packed structure of a salt made of
cation X and anion Y shown below (ions of only one
NaNH2
Na, face are shown for clarity), the packing fraction is
liq. NH3 approximately
packing efficiency
(a) (packing fraction = )
100

(b)
Y

(c)
Y
X
(d) Y

2. Among the following, the conformation that corresponds to Y


the most stable conformation of meso-butane-2,3-diol is
OH OH (a) 0.74 (b) 0.63
H Me H OH (c) 0.52 (d) 0.48
(a) (b) 4. The calculated spin only magnetic moments of
Me H H Me [ Cr( NH3 )6 ]3+ and [ CuF6 ]3- in BM, respectively, are
OH Me (Atomic numbers of Cr and Cu are 24 and 29,
respectively)
OH OH
(a) 3.87 and 2.84
Me OH H OH (b) 4.90 and 1.73
(c) (d) (c) 3.87 and 1.73
H Me (d) 4.90 and 2.84
Me H
H Me
2 JEE Advanced Solved Paper 2021

SECTION 2
l This section contains THREE (03) question stems.
l There are TWO (02) questions corresponding to each question stem.
l The answer to each question is a NUMERICAL VALUE.
l For each question, enter the correct numerical value corresponding to the answer in the designated place using the mouse and
the on-screen virtual numeric keypad.
l If the numerical value has more than two decimal places, truncate/round-off the value to TWO decimal places.
l Answer to each question will be evaluated according to the following marking scheme:
Full Marks : +2 If ONLY the correct numerical value is entered at the designated place;
Zero marks : 0 In all other cases.

Question Stem for Question Nos. 5 and 6 d (ln K ) DH °


(Given, =- , where the equilibrium
Question Stem æ 1ö R
dç ÷
èT ø
For the following reaction scheme, percentage yields are
p
given along the arrow: constant, K = Z and the gas constant,

NaNH2 Red hot iron tube
Mg2C3 H2O P Mel 873 K R
R = 8.314 JK - 1 mol - 1 )
Q
(4.0g) 75% 40% (x g)
7. The value of standard enthalpy, DH° (in kJ mol - 1 ) for
Hg2+/H+ 100%
333K
the given reaction is ……… .
Ba(OH)2 NaOCl (Decolourises
S T U 8. The value of, DS ° (in JK -1 mol - 1 ) for the given
Heat 80% (y g) Baeyer's
80% reagent) reaction, at 1000 K is ……… .
x g and y g are mass of R and U, respectively. Question Stem for Question Nos. 9 and 10
(Use Molar mass (in g mol - 1 ) of H, C and O as 1, 12 and 16, Question Stem
respectively) The boiling point of water in a 0.1 molal silver nitrate
5. The value of x is ……… . solution (solution A) is xºC. To this solution A, an equal
6. The value of y is ……… . volume of 0.1 molal aqueous barium chloride solution is
added to make a new solution B. The difference in the
Question Stem for Question Nos. 7 and 8 boiling points of water in the two solutions A and B is
Question Stem y ´ 10- 2 ºC.
For the reaction, X ( s ) -Y (s ) + Z (g ), the plot of ln pp° Z
(Assume: Densities of the solutions A and B are the
same as that of water and the soluble salts dissociate
104 completely).
versus is, given below (in solid line), where pZ is the
T Use Molal elevation constant (Ebullioscopic constant),
pressure (in bar) of the gas Z at temperature T and p° = 1bar.
K b = 0.5 K kg mol - 1 ; (Boiling point of pure water as
ln pz 100ºC.)

10 12 9. The value of x is ……… .
104/T (K–1)
10. The value of | y | is ……… .
–3

–7
JEE Advanced Solved Paper 2021 3

SECTION 3
l This section contains SIX (06) questions.
l Each question has FOUR options (a), (b), (c) and (d). ONE OR MORE THAN ONE of these four option(s) is (are) correct
answer(s).
l For each question, choose the option(s) corresponding to (all) the correct answer(s).
l Answer to each question will be evaluated according to the following marking scheme:
Full Marks : +4 If only (all) the correct option(s) is(are) chosen;
Partial Marks : +3 If all the four options are correct but ONLY three options are chosen;
Partial Marks : +2 If three or more options are correct but ONLY two options are chosen, both of which are correct;
Partial Marks : +1 If two or more options are correct but ONLY one option is chosen and it is a correct option;
Zero Marks : 0 If unanswered;
Negative Marks : -2 In all other cases.
l For example, in a question, if (a), (b) and (d) are the ONLY three options corresponding to correct answers, then
choosing ONLY (a), (b) and (d) will get +4 marks;
choosing ONLY (a) and (b) will get +2 marks;
choosing ONLY (a) and (d) will get +2 marks;
choosing ONLY (b) and (d) will get +2 marks;
choosing ONLY (a) will get +1 mark;
choosing ONLY (b) will get +1 mark;
choosing ONLY (d) will get +1 mark;
choosing no option(s) (i.e. the question is unanswered) will get 0 marks and
choosing any other option(s) will get -2 marks.

11. Given, 12. The reaction of Q with PhSNa yields an organic


CHO compound (major product) that gives positive Carius
test on treatment with Na 2 O2 followed by addition of
H OH BaCl 2 . The correct option(s) for Q is(are)
HO H HNO3
P O2N
H OH [α]D = +52.7º
H OH (a) O2N F (b) I

CH2OH O2N
NO2
D-glucose
MeS
The compound(s), which on reaction with HNO3 will give the
product having degree of rotation, [a ]D = - 52.7º is (are) (c) Br (d) O2N
Cl
O2N MeS
CHO CHO

HO H HO H 13. The correct statement(s) related to colloids is(are)


HO H HO H (a) the process of precipitating colloidal sol by an electrolyte
(a) (b)
H OH H OH is called peptisation.
H OH HO H (b) colloidal solution freezes at higher temperature, than the
true solution at the same concentration.
CH2OH CH2OH (c) surfactants form micelle above critical micelle
concentration (CMC). CMC depends on temperature.
CHO CHO (d) micelles are macromolecular colloids
HO H H OH
14. An ideal gas undergoes a reversible isothermal
H OH H OH expansion from state I to state II followed by a
(c) (d)
HO H HO H reversible adiabatic expansion from state II to state III.
HO H H OH The correct plot(s) representing the changes from state I
CH2OH CH2OH to state III is(are) ( p = pressure, V = volume, T =
temperature, H = enthalpy, S = entropy)
4 JEE Advanced Solved Paper 2021

(b) in the extraction process of copper from copper pyrites,


I silica is added to produce copper silicate.
I
(c) partial oxidation of sulphide ore of copper by roasting,
p p followed by self-reduction produces blister copper.
II
(a) (b) (d) in cyanide process, zinc powder is utilised to precipitate
II gold from Na[Au(CN)2 ]
III III
16. A mixture of two salts is used to prepare a solution S,
V T which gives the following results
Dilute NaOH( aq )
White precipitates (s) only ¬ ¾ ¾ ¾ ¾ ¾ ¾ S
Room temperature (aq solution
III I
II of the salts)
H T
Dilute HCl( aq )
(c) (d) ¾® White precipitate(s) only
¾¾¾¾¾
Room temperature
I III
II The correct option(s) for the salt mixture is (are)
(a) Pb(NO 3 )2 and Zn(NO 3 )2
S S
(b) Pb(NO 3 )2 and Bi(NO 3 )3
(c) AgNO 3 and Bi(NO 3 )3
15. The correct statement(s) related to the metal extraction (d) Pb(NO 3 )2 and Hg(NO 3 )2
processes is(are)
(a) a mixture of PbS and PbO undergoes self-reduction to produce
Pb and SO 2 .

SECTION 4
l
This section contains THREE (03) questions.
l
The answer to each question is a NON-NEGATIVE INTEGER.
l
For each question, enter the correct integer corresponding to the answer using the mouse and the on-screen virtual numeric
keypad in the place designated to enter the answer.
l
Answer to each question will be evaluated according to the following marking scheme:
Full Marks : +4 If ONLY the correct integer is entered;
Zero Marks : 0 In all other cases.

17. The maximum number of possible isomers (including 1. O3 (excess)


then Zn/H2O
stereoisomers) which may be formed on mono-bromination of (P)
2. NH2OH (excess)
1-methylcyclohex-1-ene using Br2 and UV light is …… .
18. In the reaction given below, the total number of atoms having
19. The total number of possible isomers for
sp 2 hybridisation in the major product P is ……… .
[Pt(NH3 )4 Cl 2 ] Br2 is ……… .
Paper 2

Section 1
l This section contains SIX (06) questions.
l Each question has FOUR options (a), (b), (c) and (d). ONE OR MORE THAN ONE of these four option(s) is (are) correct
answer(s).
l For each question, choose the option(s) corresponding to (all) the correct answer(s).
l Answer to each question will be evaluated according to the following marking scheme:
Full Marks : +4 If only (all) the correct option(s) is(are) chosen;
Partial Marks : +3 If all the four options are correct but ONLY three options are chosen;
Partial Marks : +2 If three or more options are correct but ONLY two options are chosen, both of which are correct;
Partial Marks : +1 If two or more options are correct but ONLY one option is chosen and it is a correct option;
Zero Marks : 0 If unanswered;
Negative Marks : - 2 In all other cases.
l For example, in a question, if (a), (b) and (d) are the ONLY three options corresponding to correct answers, then
choosing ONLY (a), (b) and (d) will get +4 marks;
choosing ONLY (a) and (b) will get +2 marks;
choosing ONLY (a) and (d) will get +2 marks;
choosing ONLY (b) and (d) will get +2 marks;
choosing ONLY (a) will get +1 mark;
choosing ONLY (b) will get +1 mark;
choosing ONLY (d) will get +1 mark;
choosing no option(s) (i.e. the question is unanswered) will get 0 marks and
choosing any other option(s) will get -2 marks.

1. The reaction sequence(s) that would lead to o-xylene as the ¾k ® P


3. For the following reaction, 2X + Y ¾
major product is(are) d[P ]
the rate of reaction is = k[ X ]. Two moles of X are
Me dt
NH2 1. NaNO2/HCl, 273 K mixed with one mole of Y to make 1.0 L of solution. At
2. CuCN
(a) 50 s, 0.5 mole of Y is left in the reaction mixture. The
3. DIBAL-H then H3O+
4. N2H4, KOH, Heat correct statement(s) about the reaction is(are)
Me 1. Mg, CO2, H3O+ (Use ln 2 = 0.693)
2. SOCl2
(b) (a) the rate constant k, of the reaction is 13.86 ´ 10- 4 s - 1.
3. H2, Pd-BaSO4
Br 4. Zn-Hg, HCl (b) half-life of X is 50 s.
d[ X ]
Me 1. BH3 (c) at 50 s, - = 13.86 ´ 10- 3 mol L- 1s - 1.
2. H2O2, NaOH dt
(c) d [Y ]
3. PBr3 (d) at 100 s, - = 3.46 ´ 10- 3 mol L- 1s - 1.
4. Zn, dil. HCl dt
1. O3, Zn/H2O 4. Some standard electrode potentials at 298 K are given
(d)
2. N2H4, KOH, Heat below :
Pb 2+ / Pb - 013
. V Ni 2+ / Ni - 0.24 V
2. Correct option(s) for the following sequence of reactions is(are) Cd 2+ / Cd - 0.40 V Fe2+ / Fe - 0.44 V
V
To a solution containing 0.001 M of X 2+ and 0.1 M of
Br2 1. Q CHCl3
PhCH3
Light P 2. H , Pd/C R KOH S Y 2+ , the metal rods X and Y are inserted (at 298 K) and
1. KMnO4 2 Foul smelling connected by a conducting wire. This resulted in
KOH, Heat
2. H3O+ W
dissolution of X . The correct combination(s) of X and
1. NH3
T
2. Heat
U Y , respectively, is(are)
(a) Q = KNO 2 , W = LiAlH4
(Given, Gas constant, R = 8.314 J K - 1 mol - 1 , Faraday
(b) R = benzenamine, V = KCN constant, F = 96500 C mol - 1 )
(c) Q = AgNO 2 , R = phenylmethanamine (a) Cd and Ni (b) Cd and Fe
(d) W = LiAlH4 , V = AgCN (c) Ni and Pb (d) Ni and Fe
6 JEE Advanced Solved Paper 2021

5. The pair(s) of complexes where in both exhibit tetrahedral 6. The correct statement(s) related to oxoacids of
geometry is(are) phosphorous is(are)
(Note py = pyridine) (a) upon heating, H3PO 3 undergoes disproportionation
reaction to produce H3PO 4 and PH3 .
(Given atomic numbers of Fe, Co, Ni and Cu are 26, 27, 28 (b) while H3PO 3 can act as reducing agent, H3PO 4 cannot.
and 29, respectively) (c) H3PO 3 is a monobasic acid.
(a) [FeCl 4 ]- and [Fe(CO)4 ]2 (b) [Co(CO)4 ]- and [CoCl 4 ]2 - (d) the H atom of P ¾ H bond in H3PO 3 is not ionisable in
2-
(c) [Ni(CO)4 ] and [Ni(CN)4 ] (d) [Cu(py)4 ]+ and [Cu(CN)4 ]3- water.

SECTION 2
l This section contains THREE (03) question stems.
l There are TWO (02) questions corresponding to each question stem.
l The answer to each question is a NUMERICAL VALUE.
l For each question, enter the correct numerical value corresponding to the answer in the designated place using the mouse and
the on-screen virtual numeric keypad.
l If the numerical value has more than two decimal places, truncate/round-off the value to TWO decimal places.
l Answer to each question will be evaluated according to the following marking scheme:
Full Marks : +2 If ONLY the correct numerical value is entered at the designated place;
Zero Marks : 0 In all other cases.

Question Stem for Question Nos. 7 and 8 (Use molar masses (in g mol - 1 ) of H, C, N, O, Cl and
Question Stem Sn as 1, 12, 14, 16, 35 and 119, respectively).
At 298 K, the limiting molar conductivity of a weak 9. The value of x is ……… .
monobasic acid is 4 ´ 102 S cm2 mol - 1 . At 298 K, for an 10. The value of y is ……… .
aqueous solution of the acid the degree of dissociation is a
Question Stem for Question Nos. 11 and 12
and the molar conductivity is y ´ 102 S cm2 mol - 1 . At 298 K,
Question Stem
upon 20 times dilution with water, the molar conductivity of
the solution becomes 3 y ´ 102 S cm2 mol - 1 . A sample (5.6 g) containing iron is completely dissolved
in cold dilute HCl to prepare a 250 mL of solution.
7. The value of a is ……… . Titration of 25.0 mL of this solution requires 12.5 mL of
8. The value of y is ……… . 0.03 M KMnO4 solution to reach the end point. Number
of moles of Fe2+ present in 250 mL solution is x ´ 10- 2
Question Stem for Question Nos. 9 and 10
(consider complete dissolution of FeCl 2 ). The amount of
Question Stem iron present in the sample is y % by weight.
Reaction of x g of Sn with HCl quantitatively produced a salt. (Assume: KMnO4 reacts only with Fe2+ in the solution
Entire amount of the salt reacted with y g of nitrobenzene in
the presence of required amount of HCl to produce 1.29 g of Use Molar mass of iron as 56 g mol - 1 )
an organic salt (quantitatively). 11. The value of x is ……… .
12. The value of y is ……… .

SECTION 3
l
This section contains TWO (02) paragraphs. Based on each paragraph, there are TWO (02) questions.
l
Each question has FOUR options (a), (b), (c) and (d). ONLY ONE of these four options is the correct answer.
l
For each question, choose the option corresponding to the correct answer.
l
Answer to each question will be evaluated according to the following marking scheme:
Full Marks : +3 If ONLY the correct option is chosen;
Zero Marks : 0 If none of the options is chosen (i.e. the question is unanswered);
Negative Marks : - 1 In all other cases.
JEE Advanced Solved Paper 2021 7

Paragraph 14. For the following reaction,


The amount of energy required to break a bond is same as the CH4 ( g ) + Cl 2 ( g ) ¾Light
¾¾® CH3 Cl ( g ) + HCl( g )
amount of energy released, when the same bond is formed. In
gaseous state, the energy required for homolytic cleavage of the correct statement is
a bond is called Bond Dissociation Energy (BDE) or Bond (a) initiation step is exothermic with DH° = - 58 kcal mol - 1.
Strength. BDE is affected by s-character of the bond and the (b) propagation step involving · CH3 formation is exothermic
stability of the radicals formed. Shorter bonds are typically with DH° = - 2 kcal mol - 1.
stronger bonds. BDEs for some bonds are given below (c) propagation step involving CH3Cl formation is
· ·
H3C––H(g) ¾® H3 C ( g ) + H ( g ) ; DH° = 105 kcal mol
-1 endothermic with DH° = + 27 kcal mol - 1.
(d) The reaction is exothermic with DH° = - 25 kcal mol - 1.
Cl ¾ Cl ( g ) ¾® Cl · ( g ) + Cl · ( g ) ; DH° = 58 kcal mol - 1
Paragraph
H3 C ¾ Cl ( g ) ¾® H3 C· ( g ) + Cl · ( g ) ; DH° = 85 kcal mol - 1
The reaction of K 3 [Fe(CN)6 ] with freshly prepared
H ¾ Cl ( g ) ¾® H· ( g ) + Cl · ( g ) ; DH° = 103 kcal mol - 1 FeSO4 solution produces a dark blue precipitate called
Turnbull’s blue. Reaction of K 4 [Fe(CN)6 ] with the
13. Correct match of the C¾ H bonds (shown in bold) in
FeSO4 solution in complete absence of air produces a
Column J with their BDE in Column K is
white precipitate X, which turns blue in air. Mixing the
Column J Column K FeSO4 solution with NaNO3 , followed by a slow addition
(Molecule) (BDE (kcal mol - 1)) of concentrated H2 SO4 through the side of the test tube
(P) H¾CH(CH 3 )2 (i) 132 produces a brown ring.
(Q) H¾CH 2 Ph (ii) 110 15. Precipitate X is
(R) H¾CH == CH 2 (iii) 95 (a) Fe4[Fe(CN)6 ]3 (b) Fe[Fe(CN)6 ]
(c) K 2Fe[Fe(CN)6 ] (d) KFe[Fe(CN)6 ]
(S) H¾ C ºº CH (iv) 88
16. Among the following, the brown ring is due to the
Codes formation of
P Q R S
(a) (iii) (iv) (ii) (i) (a) [Fe(NO)2 (SO 4 )2 ]2- (b) [Fe(NO)2 (H2O)4 ]3+
(b) (i) (ii) (iii) (iv) (c) [Fe(NO)4 (SO 4 )2 ] (d) [Fe(NO)(H2O)5 ]2+
(c) (iii) (ii) (i) (iv)
(d) (ii) (i) (iv) (iii)

SECTION 4
l
This section contains THREE (03) questions.
l
The answer to each question is a NON-NEGATIVE INTEGER.
l
For each question, enter the correct integer corresponding to the answer using the mouse and the on-screen virtual numeric
keypad in the place designated to enter the answer.
l
Answer to each question will be evaluated according to the following marking scheme:
Full Marks : +4 If ONLY the correct integer is entered;
Zero Marks : 0 In all other cases.

17. One mole of an ideal gas at 900 K, undergoes two reversible (Given, molar heat capacity at constant volume, CV , m of
processes, I followed by II, as shown below. If the work done 5
the gas is R)
by the gas in the two processes are same, the value of ln 2
V3 /V2 is ……… . 18. Consider a helium (He) atom that absorbs a photon of
-1
wavelength 330 nm. The change in the velocity (in cm/s )
2250 (p1, V1)
of He atom after the photon absorption is ……… .
I (Assume, momentum is conserved, when photon is
U/R (K)
absorbed.
450
II Use Planck’s constant = 6.6 ´ 10- 34 J s, Avogadro’s
(p3, V2) (p3, V3)
number = 6 ´ 1023 mol - 1 , Molar mass of He = 4 g mol - 1 )
S (J K–1 mol–1) 19. Ozonolysis of ClO2 produces an oxide of chlorine. The
(U = internal energy, S = entropy, average oxidation state of chlorine in this oxide is
p = pressure, V = volume, R = gas constant) ……… .
Answer with Explanations
Paper 1
1. (b) NaNH2 acts as a base and takes the acidic proton from As two anions are touching each other, so side of unit cell,
a = 2r−
terminal alkyne to form acetylide ion.
Volume of unit cell = a 3 = (2r−)3 = 8r−3
s
H NH2
s Volume of cations X = Number of cation × Volume of one
4 3
cation = 3 × πr+
Na/ liq. NH 3 is a reducing agent that reduces alkynes to 3
trans-alkene. Na-atom gives one electron to the alkyne. In above Volume of anions Y = Number of anion × Volume of one
step, an acetylide ion is formed, so reduction of terminal alkyne 4
anion = 1 × πr−3
does not take place because negative charge repulses the 3
incoming electron. Thus, the alkyne group is reduced to alkene. r
Radius ratio for close packed structure, + = 0.414
r−
Na
Packing efficiency
Volume of cation × Number of cations + Volume
H—NH2
of anion × Number of anions
Na
= × 100
Volume of unit cell
H H
Packing efficiency
Packing fraction =
H—NH2 100
4 3 4
H 1 × πr− + 3 × πr+3
H––NH2 H = 3 3
8r−3
or
4π  r−3 3r+3  π
H =  3 + 3  = [1 + 3(0.414) ]
3
3× 8  −
r r−  6
2. (b) 3.14 3.14
OH = (1 + 3 × 0.0710) = × 1.213 = 0.63
The structure of meso-butane-2,3-diol is 6 6
4. (a) [Cr(NH3)6 ]3+
OH Atomic number of Cr = 24
The most stable conformation of meso-butane-2,3-diol is the Electronic configuration of Cr = 1s 2 2s 2 2p 6 3s 2 3p 6 3d 5 4s1
one in which there is hydrogen-bonding between two alcohol
Electronic configuraiton of Cr 3+ = 1s 2 2s 2 2p 6 3s 2 3p 6 3d 3 4s 0
groups. The conformation is
δ+ NH 3 is a weak field ligand. The splitting of d-orbital occur.
OH
δ – eg0
3
H OH d
∆o

H Me
¼¼¼ t23g
Me
Number of unpaired electrons, n = 3
3. (b) Magnetic moment (spin only), µ s = n(n + 2) BM

Y = 3(3 + 2) = 3 × 5 = 15 = 3.87 BM
[CuF6 ]3− Atomic number of Cu = 29.
Y Electronic configuration of Cu = 1s 2 2s 2 2p 6 3s 2 3p 6 3d10 4s1
X
Y Electronic configuration of Cu 2+ = 1s 2 2s 2 2p 6 3s 2 3p 6 3d 8 4s 0
F − is a weak field ligand, so splitting of d-orbitals occur as
Y follows.
2
Radius of cation X = r+ eg
8
1 d
Number of cation X present = 6 × =3 ∆o
2
Radius of anion Y = r− 6
1 t2g
Number of anion Y present = × 8 =1
8 Number of unpaired electrons, n = 2
Side of unit cell = a
JEE Advanced Solved Paper 2021 9

Spin only magnetic moment, µ s = n(n + 2) BM So, 0.04 mole of T forms 0.8 × 0.04 = 0.032 mole of U
= 2 (2 + 2) = 2 × 4 = 2 2 = 2.84 BM Molar mass of U = 100 g mol −1
Mass of U formed or value of y = Molar mass × Number of
5. (1.62) Mg 2C3 reacts with water to give propyne. moles of U
Mg 2C3 + 4H 2O → 2Mg(OH)2 + CH 3 — C≡≡ CH = 100 × 0.032 = 3.2 g
(P)
Propyne 7. (166.28)
Sodium amide (NaNH 2) takes proton from alkyne to form ∆G° = − RT ln K …(i)
acetylide ion which, attacks on Me—I to give an alkyne with one Also, ∆G° = ∆H ° − T∆S ° …(ii)
more carbon.
Comparing Eqs. (i) and (ii),
CH3––C C––H
NH2
CH3––C C
Me–– I
CH3––C C––CH3
− RT ln K = ∆H ° − T∆S °
p
(P)
Propyne
Acetylide ion (Q) Given, K = Z and p° = 1 bar
But-2-yne p°
Mass of P formed = 4.0 g
− RT ln  Z  = ∆H ° − T∆S °
p
So,
Molar mass of P formed = 40 g mol −1 1 
− ∆H ° ∆S °
ln  Z  =
Mass of P(W) 4 p
Number of moles of P = = = 0.1 mol + …(iii)
Molar mass of P (M) 40 1  RT R
1 mole of P forms 75% i.e., 0.75 mole of Q. − ∆H °
In graph, slope =
So, 0.1 mole of P forms 0.75 × 0.1 = 0.075 mole of Q R
Polymerisation of alkyne (Butyne) occurs in presence of red hot  Y − Y1  4  −7 + 3
Slope = 104  2  = 10  
iron tube to form benzene derivative.  X 2 − X1   12 − 10 

= 104  −  = − 2 × 104
4
Red hot  2
CH3—C C—CH3 iron tube
∆H °
(Q ) 873 K Therefore, −2 × 104 = − ⇒ 2 × 104 × 8.314 = ∆H °
R
(R)
⇒ ∆H° = 166280 J mol −1 = 166.28 kJ mol −1
3 moles of alkyne forms 40% i.e., 0.40 mole of benzene derivative, R.
4
0.40
0.075 mole of Q forms × 0.075 = 0.01 mole of R. 8. (141.34) From graph, 10 = 10
3 T
Molar mass of R = 162g/mol Therefore, T = 103 K
Mass of R formed or value of x = Molar mass × Number of moles
Also, ln  2  = −3
p
= 162 × 0.01 = 1.62 g 1
6. (3.2) Alkyne, P oxidises to ketone on reacting with Hg 2+ / H + . Substituting the value in Eq. (iii),
∆H ° ∆S °
ln   = −
O p
+
2+
Hg / H +  1  RT R
CH 3 — C ≡≡ CH → CH 3 — C — CH 3
333 K 2 × 104 × R ∆S °
( P)
Propyne
( S)
Acetone −3 = − +
R × 103 R
As reaction efficiency is 100%, so 0.1 mole of P forms 0.1 mole of ∆S °
⇒ = 20 − 3 = 17
S. R
‘S’ in presence of base, Ba(OH)2 followed by heating leads to aldol ⇒ ∆S ° = + 17 × R = + 17 × 8.314 = 141.34 JK –1 mol −1
condensation reaction to formα , β-unsaturated carbonyl compound.
9. (100.10) Complete reaction as follows :
Ba(OH)2 CH3 AgNO 3 (aq) → Ag + (aq) + NO −3 (aq)
2 CH3—C—CH3 ∆ C==CH—C—CH3
(S) CH3 Concentration, m = 0.1 m
4-methylbut-3-en-2-one
(T) van’t Hoff factor, i = 2
2 moles of S forms 80% i.e. 0.8 mole of T. ∆Tb = iK b m;
0.8 where, ∆Tb is change in boiling point of H 2O.
So, 0.1 mole of S forms × 0.1 = 0.04 mole of T.
2 K b is ebullioscopic constant = 0.5K kg mol −1
Ketone oxidises to carboxylic acid in presence of NaOCl. m is concentration of solution
Complete reaction is as follows : ∆Tb = 2 × 0.5 × 0.1 = 0.1° C
CH3 Tb ° = boiling point of pure water
C CH—C—CH3
NaOCl
(CH3)2—C CH—C—OH + CHCl3 Tb = boiling point of solution
CH3 (U) Tb − Tb ° = 0.1 ⇒ Tb = 100 + 0.1 = 100.1° C
(T )
4-methylbut-3-enoic acid
Boiling of solution, x° C = 100.1° C
1 mole of T forms 80% i.e., 0.8 mole of U.
10 JEE Advanced Solved Paper 2021

10. (2.5) By adding equal volume of 0.1 m BaCl 2 , the volume of 12. (a, d) PhSNa replaces F from o- and p-dinitro- fluorobenzene
solution doubles and concentration becomes half. via nucleophilic aromatic substitution.
PhSNa replaces Cl from compound D via nucleophilic
AgNO 3(aq) → Ag + (aq) + NO −3 (aq)
bimolecular substitution reaction.
0.05 m 0.05 m
In case of compound B and C, nucleophilic substitution is not
BaCl 2(aq) → Ba 2+ (aq) + 2Cl − (aq) possible neither by SN Ar or SN 2. because, the electron
0.05 m 0.1 m withdrawing group is present at meta-position and halogen is
Ag + reacts with Cl − to form ppt. of AgCl. attached to sp 2-carbon.
Ag + (aq) + Cl − (aq) → AgCl(s) F SPh
t=0 0.05 0.1 m – NO2 NO2
t=∞ 0 0.05 m 0.05 m SNAr
+ PhSNa
Total species in solution are NO −3 , Cl − and Ba 2+ . So, van’t Hoff
factor, i = 3. NO2 NO2
Concentration of each species, m = 0.05 m (Q)
Elevation of boiling point 2, 4-dinitro fluorobenzene

∆Tb = iK bm Cl SPh
∆Tb = 3 × 0.5 × 0.05 = 0.075° C
SN 2
Tb ′ − 100 = 0.075 + PhSNa
Tb ′ = 100.075° C SMe SMe
Difference in boiling point, Tb − Tb ′ = 100.1 − 100.075= 0.025 NO2 NO2
(Q)
= 2.5 × 10−2 ° C
Positive Carius test is given by compound containing halogen,
So, y = 2.5
sulphur and phosphorus. So, only compound A and D reacts
11. (c, d) Monosaccharide oxidises on reacting with HNO 3 to with PhSNa to gives a positive Carius test on treatment with
form carboxylic acids. Primary alcohols also oxidise to Na 2O 2 followed by addition of BaCl 2.
carboxylic acid alongwith aldehyde. SPh
Complete reaction is as follows : NO2
CHO COOH

H OH H OH
HO H HO H NO2 Positive
Dil.HNO3 (1) Na2O2
SPh Carius
H OH H OH (2) BaCl2
test
H OH H OH

CH2OH COOH
(P) SMe
Glucose [α] =– 52.7º NO2
The enantiomer of product (P) has rotation −52.7°. The two
enantiomers (c and d) with their reactions are as follows :
13. (b, c) Statements (b) and (c) are correct whereas statements
(a) and (d) are incorrect.
CHO COOH (a) Process of precipitating colloidal sol is known as
HO H HO H coagulation.
H OH H OH (b) Concentration of colloidal solutions is small due to large
Dil.HNO3
molar mass and thus, their colligative properties are very
HO H HO H
small compared to true solutions. Hence, these solution
HO H HO H freezes at higher temperature.
CH2OH CH2OH (c) Micelles are formed at the critical micelle concentration
(c) [α]D =–52.7º (CMC) which depends to temperature.
Glucose (d) Micelles and macromolecular colloids are two different
types of colloid.
CHO COOH
14. (a, b and d)
H OH H OH From state I to II Reversible isothermal expansion takes
H OH Dil.HNO3 H OH place. So, following changes take place,
HO H HO H ● pressure decreases.
H OH H OH ● Volume increases.
● Temperature remains constant.
CH2OH CH2OH
● Enthalpy, H remains constant.
(d) [α](D) =–52.7º
Glucose ● Entropy, S for expansion increases.
So, all options follows the above mentioned conditions, so all
graphs are correct for state I and II.
JEE Advanced Solved Paper 2021 11

From state II to III Reversible adiabatic expansion takes place. Radicals formed are
So, following changes take place. CH2 CH2 CH3 CH3
● pressure decreases.
● Volume increases.
● Temperature decreases.
● Enthalpy, H decreases.
● Entropy, S remains constant.
● H increases instead of decreasing, so only option (c) is
incorrect.
All other options, i.e., (a), (b) and (d) follows the above
mentioned conditions. 18. (12) Alkene on treatment with ozone (O 3) followed by
Therefore, correct graphical representations are (a), (b) and (d). treatment with Zn/H 2O gives carbonyl compounds.

15. (a, c and d) 1. O3 (excess)


Zn/H2O
Statements (a), (c) and (d) are correct, whereas statement (b) is
incorrect. All statements are explained as follows :
(a) A mixture PbO and PbS reacts as follows:
Reduction Carbonyl compounds react with NH 2OH to give oximes.
2PbO + PbS → 3Pb + SO 2 HO OH
This reaction is known as self-reduction. N N
(b) Silica is added to remove impurities of Fe in the form of
2. NH2OH
slag, FeSiO 3.
(c) CuFeS2 are in partially oxidised first by roasting and then
self-reduction of Cu takes place to produce blister copper. N N
(d) Zn powder reacts with Na[Au(CN)2] to form gold. This HO OH
process is called cyanide process. (Structure I)
2Zn + 4Na[Au(CN)2] → 2Na 2[Zn(CN)4 ] + 4Au
16. (a, b) Pb(NO 3)2 , Zn(NO 3)2 and Bi(NO 3)3 give white precipitate OH OH
with dil. NaOH and dil. HCl. Complete reactions are as follows: N N
Dil. HCl Dil. NaOH
● PbCl 2 ←
Room temperature
Pb(NO 3)2 →
Room temperature
Pb(OH)2
White ppt. Leadnitrate White ppt.
Dil. HCl Dil. NaOH
● ZnCl 2 ←
Room temperature
Zn(NO 3)2 →
Room temperature
Zn(OH)2 N N
White ppt. Zinc nitrate White ppt.
Dil. HCl Dil. NaOH
HO OH
● BiCl 3 ←
Room temperature
Bi(NO 3)3 →
Room temperature
Bi(OH)3 (Structure II)
White ppt. Bismuth nitrate White ppt.

Whereas, AgNO 3 and Hg(NO 3)3 give ppt. of different colours. There are four sp 2-carbon atoms, four sp 2-nitrogen atoms and
● AgCl ←
Dil. HCl
AgNO 3 →
Dil. NaOH
Ag 2O four sp 2-oxygen atoms as shown in structures I and II.
Room temperature Room temperature
White ppt. Silver nitrate Brown ppt. Therefore, total number of atoms having sp 2-hybridisation are
Dil. HCl Dil. NaOH twelve (12).
● HgCl 2 ←
Room temperature
Hg(NO 3)2 →
Room temperature
HgO
White ppt. Mercury Yellow ppt.
19. (3) [Pt(NH 3)4 Cl 2]Br2 shows three ionisation isomers.
17. (13) Monobromination of 1-methyl cyclohex-1-ene in (i) [Pt(NH 3)4 Cl 2]Br2 (ii) [Pt(NH 3)4 ClBr]BrCl
presence of UV-light gives following products (1 achiral and (iii) [Pt(NH 3)4 Br2]Cl 2
6 optically active (*) compounds). So, total 13 products are Ionisation isomers are compounds having same molecular
formed (1 + 6 + 6 enantiomers) formula but have different counter ions.
CH3 CH2Br CH2 CH2 Each isomer shown above also possess two more geometrical
Br Br isomers. Geometrical isomers are the compounds having
+ Br2 hv + * + * + different arrangement of atoms in space but same molecular
formula.
1-methyl (1) (2,3) (4,5) The geometrical isomers are :
cyclohex-1-ene
CH3 CH3 H C CH3 [Pt(NH 3)4 Cl 2]Br2
* Br
3
Cl Cl
+ + + H3N NH3 H3N Cl
* Pt Br2 Pt Br2
Br Br *
(6,7) * (10,11) (12,13) H3N NH3 H3N NH3
Br
Cl NH3
(8,9)
trans cis
12 JEE Advanced Solved Paper 2021

[Pt(NH 3)4 ClBr]ClBr [Pt(NH 3)4 Br2]Cl 2


Cl Cl Br Br
H3N NH3 H3N Br H 3N NH3 H3N Br
Pt Cl Br Pt Cl Br Pt Cl2 Pt Br2
H3N NH3 H3N NH3 H 3N NH3 H3N NH3
Br NH3 Br NH3
trans cis trans cis

Paper 2
1. (a, b) Complete reaction sequences are as follows : Acid chlorides on treating with hydrogen gas in presence of
Pd-BaSO 4 catalyst forms aldehyde.
(a) o-toluidine reacts with NaNO 2/HCl at 273 K to give
diazonium salt.
CH3 CH3 C C
Cl H2 H
NH2 N2 Cl Pd–BaSO4
NaNO2/HCl
273 K Clemmensen reduction Carbonyl compounds gives alkanes
Diazonium salt reacts with CuCN to give benzonitrile product. on treating with Zn-Hg in presence of HCl.
CHO
N2 Cl CN Zn–Hg
Sandmeyer
+ CuCN reaction + N2 + CuCl HCl

2-methyl benzonitrile o-xylene

DIBAL-H is a reducing agent that reduces cyanide group to (c) Alkenes gives alcohol on treating with BH 3 followed by
aldehyde group. H 2O 2 , NaOH. The OH group attaches to the least
substituted carbon.
CN CHO OH
DIBAL-H BH3
+
H 3O H2O2, NaOH
2-methyl benzaldehyde Me Me
Alcohols reacts with PBr3 to give alkyl bromide. Bromine
Carbonyl compounds are reduced to alkane by treating, them replaces hydroxide group.
with N 2H 4 with KOH.
OH Br
PBr3
CHO
N2H4, KOH
∆ Me Me
(Wolff-Kishner reduction) o-xylene Alkyl halides on treating with Zn and dil. HCl reduce to alkanes.

(b) Treatment of 1-bromo-2-methyl benzene with Mg in dry Br


ether gives Grignard reagent which attacks on carbon Zn
Dil. HCl
dioxide to give carboxylate ion which give carboxylic acid Me Me
upon hydrolysis. 2-ethyl-1-methyl benzene
(o-xylene is not formed)

+ (d) Ozonolysis of alkenes give carbonyl compound. Double


Br C—O
Mg MgBr CO2 bond breaks to give two carbonyl groups.
dry ether

O3
H
COOH Zn/H2O
H+ H
H 2O
Wolff-Kishner reduction Carbonyl compounds reduce to
alkane on treating N 2H 4 in presence of KOH.
Carboxylic acids react with thionyl chloride (SOCl 2) to form
acid chlorides.
CH3
H N2H 4
KOH, ∆ CH3
C C H
OH SOCl2 Cl 2-ethyl-1-methyl benzene
(o-xylene is not formed)

Therefore, options (a) and (b) give only o-xylene products.


JEE Advanced Solved Paper 2021 13

dY 1
2. (c, d) − = 6.93 × 10−3 × = 3.46 × 10−3 mol L–1 s−1
dt 2
Sequence of reactions involved are
So, options (b), (c) and (d) are correct.
(V) AgCN
4. (a, b and c)
Br2 1. AgNO2(Q) CHCl3 Given,
PhCH3
Light PhCH2Br 2. H , Pd/C PhCH2NH2 KOH PhCH2NO2
i) KMnO4
(P) 2 (R) (S) At anode X(s) → X 2+ (aq) + 2e −
KOH, Heat (0.001 M)
ii) H3O+ (W ) At cathode Y 2+ (aq) + 2e − → Y (s)
1. NH3 LiAlH4 (0.1 M)
PhCOOH PhCONH2
2. Heat
(T) (U) Overall reaction X(s) + Y 2+ (aq) → X 2+ (aq) + Y (s)
(a) If KNO 2 (Q)-on reacting with PhCH 2Br(P) gives Nernst equation ,
PhCH 2ONO which on reduction with LiAlH 4 does not 0.06 [X 2+ ]
give PhCH 2NH 2. ° −
E cell = E cell log 2+
2 [Y ]
(b) If product R formed is benzylamine not benzene
0.001
amine. KCN reacts with PhCH 2Br to give PhCH 2CN E cell ° − 0.03 log
= E cell
which does not have foul smelling. 0.1
(c) If AgNO 2 (Q) reacts with PhCH 2Br to give PhCH 2NO 2 ° + 0.06
E cell = E cell
which on reduciton give PhCH 2NH 2 (Phenyl E cell should be positive for a reaction to be spontaneous.
methanamine or benzyl amine).
(d) If amide reacts with LiAlH 4 (W) to give amine. PhCH 2Br ° 2 + / Cd = − 0.40 V, E Ni
(a) E Cd ° 2 + / Ni = − 0.24 V
reacts with AgCN to give PhCH 2NO 2.
° = E cathode
E cell ° °
− E anode = − 0.24 − (−0.40)
3. (b, c and d)
° = 0.16
E cell
2X + Y →
k
P
° + 0.06 = 0.16 + 0.06 = 0.22 V
E cell = E cell
t=0 2 mol 1 mol −
t = 50 s (2 − 1) mol (1– 0.5) mol 0.5 mol Reaction is spontaneous.
= 1 mol = 0.5 mol (b) E ° = − 0.40 V E ° 2+ = − 0.44 V
Cd2+ / Cd Fe / Fe
As the concentration of reactant becomes half at t = 50 s.
° = − 0.44 + 0.40 = − 0.04 V
E cell
So, half-time of reaction is 50 s.
dP E cell = − 0.04 + 0.06 = 0.02 V
Given, = k[X]1
dt Reaction is spontaneous.

⇒ −
1 dX dP
= = k[X]1 (c) E ° = − 0.24 V
Ni 2+ / Ni
2 dt dt
dX E° = − 0.13 V
− = 2k[X]1 Pb 2+ / Pb
dt
° = − 0.13+ 0.24 = 0.11 V
E cell
− dX
= 2k dt E cell = 0.11 + 0.06 = 0.17 V
[X ]
X dX t Reaction is spontaneous.
−∫ = 2k ∫ dt
X0 X 0 (d) E ° = − 0.24 V
Ni 2+ / Ni
− [ln X]XX 0 = 2k [t]t0
E ° 2+ = − 0.44 V
Fe / Fe
− ln X + ln X 0 = 2kt
X ° = − 0.44 + 0.24 = − 0.20 V
E cell
ln 0 = 2kt °
X E cell = − 0.20 + 0.06 = − 0.14 V
X0
At t1 / 2, X = , Reaction is non-spontaneous.
2 Therefore, the correct combinations of X and Y are (a), (b) and (c).
2X 0
ln = 2kt1 / 2
X0 5. (a, b, and d)
ln 2 0.693 (a) [FeCl4 ]−
k= = = 6.93 × 10−3 s−1
2t1 / 2 100 Atomic number of Fe = 26.
dx Oxidation number of Fe = x + 4 (−1) = − 1
At, t = 50 s, − = 2k [X]1 ;[X] = 1 mol ⇒ x=3
dt
− dX Electronic configuration of Fe = 1s 2 2s 2 2p 6 3s 2 3p 6 3d 6 4s 2
So, = 2 × 6.93 × 10−3 × 1 = 13.86 × 10−3 mol L–1 s–1
dt Electronic configuration of Fe3+ = 1s 2 2s 2 2p 6 3s 2 3p 6 3d 5 4s 0
1 dX − dY − dY 1
At t = 100 s, − = ⇒ = k[Y ]1 ; [Y ] = Cl − is a weak field ligand, so, no pairing X of electrons occurs.
2 dt dt dt 2
14 JEE Advanced Solved Paper 2021

3d5 4s 4p CN is a strong field ligand, so pairing of electrons occur.


[FeCl4]– = 3d8 4s 4p
[Ni(CN)4]2– =
Cl Cl Cl Cl
Hybridisation is sp 3. CN CN CN CN
2
Geometry is tetrahedral. Hybridisation is dsp .
[Fe(CO)4 ]2+ Oxidation number of Fe = x + 4(0) = + 2 Geometry is square planar.
⇒x = + 2 (d) [Cu(py)4 ]+
Electronic configuration of Fe2+ = 1s 2 2s 2 2p 6 3s 2 3p 6 3d 8 4s 2 Oxidation number of Cu = x + 4(0) = + 1 ⇒ x = + 1
CO is a strong field ligand, so pairing of electrons occurs. Atomic number of Cu = 29
3d10 4s 4p Electronic configuration of Cu = 1s 2 2s 2 2p 6 3s 2 3p 6 3d10 4s1
2+
[Fe(CO)4] = Electronic configuration of Cu + = 1s 2 2s 2 2p 6 3s 2 3p 6 3d10 4s 0
CO CO CO CO 3d10 4s 4p
+
3 [Cu(py)4] =
Hybridisation is sp .
Geometry is tetrahedral. py py py py
(b) [Co(CO)4 ]− Atomic number of Co = 27. Hybridisation is sp . 3

Oxidation number of Co = x + 4(0) = − 1 ⇒ x = −1 Geometry is tetrahedral.


Electronic configuration of Co = 1s 2 2s 2 2p 6 3s 2 3p 6 3d 7 4s 2 [Cu(CN)4 ]3−
Electronic configuration of Co− = 1s 2 2s 2 2p 6 3s 2 3p 6 3d 8 4s 2 Oxidation number of Cu = x + 4(−1) = − 3 ⇒ x = + 1
CO is a strong field ligand, so pairing of electrons occurs. 3d10 4s 4p
10 3–
3d 4s 4p [Cu(CN)4] =
[Co(CO)4]– =
CN CN CN CN
CO CO CO CO Hybridisation is sp 3.
Hybridisation is sp 3. Geometry is tetrahedral.
Geometry is tetrahedral.
6. (a, b, and d)
[CoCl4 ]2− (a) H 3PO 3 gives H 3PO 4 and PH 3 on heating. This reaction is
Oxidation number of Co = x + 4 (−1) = − 2 known as disproportionation reaction.
⇒ x=+ 2 ∆
4H 3PO 3 → 3H 3PO 4 + PH 3
Electronic configuration of Co2+ = 1s 2 2s 2 2p 6 3s 2 3p 6 3d 7 4s 0
(b) P in H 3PO 4 is in its highest oxidation state i.e., + 5 hence,
Cl − is a weak field ligand, so pairing of electrons do not occur. it cannot act as reducing agent. But P in H 3PO 3 is in
3d7 4s 4p oxidation state, +3 hence, it can act as reducing agent.
[CoCl4]2– = (c) H 3PO 3 contains two  OH groups, hence it is a dibasic
acid.
Cl Cl Cl Cl
Hybridisation is sp 3.
P
Geometry is tetrahedral. H OH
(c) [Ni(CO)4 ] OH
Oxidation number of Ni = x + 4 (0) = 0 (d) Hydrogen attached to P, does not ionise in water.
Therefore, options (a), (b) and (d) are correct.
⇒x = 0
Atomic number of Ni = 28 7. (0.22)
Electronic configuration of Ni = 1s 2 2s 2 2p 6 3s 2 3p 6 3d 8 4s 2 Degree of dissociation = α
CO is a strong field ligand, so pairing of electrons occur. Limiting molar conductivity, Λ°m = 4 × 102 S cm2 mol −1
3d10 4s 4p Molar conductivity, Λm = y × 102 S cm2 mol −1
[Ni(CO)4]– =
Molar conductivity of dilution, Λ°m = 3y × 102 S cm2 mol −1
CO CO CO CO Concentration before dilution = C
Hybridisation is sp 3. C
Concentration after dilution =
Geometry is tetrahedral. 20
[Ni(CN)4 ]2− Using relation,
Λ
Oxidation number of Ni = x + 4(−1) = − 2 ⇒ x = + 2 α= m …(i)
Electronic configuration of Ni 2+ = 1s 2 2s 2 2p 6 3s 2 3p 6 3d 8 4s 0 Λ°m
JEE Advanced Solved Paper 2021 15

Cα 2 Moles of organic salt =


Mass of organic salt
Dissociation constant, K a =
(1 − α) Molar mass
1.29
Putting Eq. (i), = = 0.01 mol
CΛ2m CΛ2m 129
Ka = = From reaction 1 moles of salt is produced from 3 mole of Sn.

2  Λ  Λ°m (Λ°m − Λm)
Λ°m 1− m So, 0.01 mole of organic salt is produced by 0.03 mole Sn.
 ° 
Λm 
 Atomic mass of Sn = 119 g mol −1
Dissociation constant before dilution, Mass of Sn = x = mole of Sn × Molar mass
C(y × 102)2 x = 0.03 × 119 ⇒ x = 3.57 g
Ka = …(ii)
(4 × 10 )(4 × 102 − y × 102)
2
The value of x is 3.57
Dissociation constant after dilution,
10. (1.23)
C
(3y × 102)2 1 mole of organic salt is produced by 1 mole of nitrobenzene
Ka = 20 …(iii) 0.01 mole of organic salt is produced by 0.01 mole
(4 × 102)(4 × 102 − 3y × 102)
nitrobenzene.
Comparing Eqs. (ii) and (iii), Molar mass of nitrobenzene
C(y × 102)2 C(3y × 102)2 = 12 × 6 + 1 × 5 + 14 × 1 + 16 × 2
=
(4 × 10 ) (4 × 10 − y × 10 ) 20(4 × 10 ) (4 × 102 − 3y × 102)
2 2 2 2
= 72 + 5 + 14 + 32= 123 g mol −1
y 2 × 104 9 y 2 × 104 Mass of nitrobenzene required, y = moles of nitrobenzene
= 2
10 (4 − y) 10 × 20(4 − 3y)
2
× molar mass
1 9 = 0.01 × 123= 1.23 g
=
(4 − y) 20(4 − 3y) The value of y is 1.23.
80 − 60 y = 36 − 9 y 11. (1.88)
80 − 36 = 60 y − 9 y Fe + 2HCl → FeCl 2 + H2
44 = 51 y (x × 10−2 mol) (x × 10−2 mol)
44
⇒ y= …(iv) x × 10−2
51 Concentration of Fe2+ , M1 = × 1000
250
Putting in Eq. (i), 10 x x
= M= M
Λ y × 102
α = °m = 250 25
Λm 4 × 102 Volume of Fe2+ solution titrated, V1 = 25.0 mL
44 11
α= = Concentration of MnO −4 , M 2 = 0.03 M
51 × 4 51
Volume of MnO −4 used, V2 = 12.5 mL
α = 0.22
5Fe2+ + MnO –4 → Mn2+ + 5Fe3+
The value of α is 0.22.
Using relation,
8. (0.86) From equation (iv), M1 V1 MV
44 = 2 2
y= ⇒ y = 0.86 n1 n2
51
n2 M1 V1 = n1 M 2V2 [n1 and n2 are number of moles of Fe2+ and
The value of y is 0.86.
MnO −4 reacting]
9. (3.57) Reaction involved x
NO2 NH2 1× × 25 = 5 × 0.03 × 12.5 ⇒ x = 1.88
25
The volume of x is 1.88.
3Sn + 6HCl + + 3 SnCl2 + 2H2O
xg 12. (18.75) Mass of sample = 5.6 g
yg
HCl
Number of moles of Fe = 1.88 × 10−2 mol
NH3 Cl Molar mass of Fe = 56 g/mol
Mass of Fe = Moles of Fe × Molar mass
1.29 g = 1.88 × 10−2 × 56 = 1.05 g
Mass of Fe 1.05
Mass of organic salt produced (aniline) = 1.29 g % Fe = × 100 = × 100
Mass of sample 5.6
Molar mass of organic salt (aniline)
⇒ y = 18.75%
= 12 × 6 + 1 × 8 + 14 × 1 + 35 × 1
The value of y is 18.75.
= 72 + 8 + 14 + 35
= 129 g mol −1
16 JEE Advanced Solved Paper 2021

13. (a) P-iii, Q-iv, R-ii, S-i. 17. (10) For process 1, entropy is constant thus, q is constant
A more stable radical means lesser the bond dissociation ∴ WI = ∆U = nCV ,m ∆T
energy. = − (2250 − 450)R = − 1800 R …(i)
Radicals formed are ∆ U = n CV , m ∆ T
CH 3 5R
−1800R = 1 × × ∆T
•
• 2
P→ CH — CH 3 Q → Ph — CH 2

⇒ ∆T = − 720 K
R→ CH 2==CH S → HC ≡≡ C• T2 − T1 = − 720 K
Radical formed from Q (PhCH 2 — H) is most stable due to T2 = − 720 K + 900 K = 180 K
resonance. For process II,
Stability of free radical decreases with increase in % V 
WII = − nRT2 ln  3 
s-character.  V2 
P radical → sp 3 → % s-character 25%
V 
R radical → sp 2 → % s-character 33% = − 1 × R × 180 ln  3  …(ii)
 V2 
S radical → sp → % s-character 50%
As, both work done for process I and II are equal.
Thus, order of stability of free radical is
Therefore, WI = WII
Q> P> R> S
V 
Order of bond dissociation energy is −1800 R = − R × 180 × ln  3 
 V2 
S> R> P> Q
 V  1800 V 
Hence, option (a) P-iii, Q-iv, R-ii, S-i is correct. ln  3  = ⇒ ln  3  = 10
 V2  180  V2 
14. (d) The reaction is exothermic with ∆H° = − 25 kcal mol −1
hν 18. (30) Wavelength of photon absorbed, λ = 330 nm
CH 4 (g) + Cl 2(g) → CH 3Cl(g) + HCl(g)
= 330 × 10−9m
(a) Initiation step is breaking of Cl—Cl bond which requires
energy and hence, this step is endothermic. Planck’s constant, h = 6.6 × 10−34 J s
Cl—Cl hν 2Cl, ∆H°2 = +105 kcal/mol–1 Molar mass of He, M = 4 g mol −1 = 4 × 10−3 kg mol −1
• Avogadro number, N A = 6 × 1023 mol −1
(b) Propagation step involving formation of CH 3 is also
M
endothermic as bond between C—H is breaking. Mass of one atom of He, m =

NA
CH3—H CH3 + H, ∆H2° = +105 kcal/mol–1
4 × 10−3 2
(c) Propagation step involving formation of CH 3Cl is = = × 10−26 kg
6 × 1023 3
exothermic as bond is formed between C—Cl.
Velocity, = V cm/s.
CH3 + Cl CH3—Cl, ∆H°3 = –85 kcal/mol
Using de-Broglie equation,
• •
(d) H + Cl → HCl, ∆H4 ° = − 103 kcal/mol h
λ=
mv
Enthalpy of reaction, ∆H ° = ∆H1° + ∆H2° + ∆H3° + ∆H4° h 6.6 × 10−34
⇒ v= =
= 58 + 105 − 85 − 103 mλ 2 / 3 × 10−26 × 330 × 10−9
= − 25kcal mol −1
6.6 × 3 × 10−34 × 1035
= = 0.03 × 10 = 0.3m/s
Hence, the reaction is exothermic with ∆H° = − 25 2 × 330
kcal/mol
= 30 cm/s
15. (c) Precipitate X is K 2Fe[Fe(CN)6].
19. (6)
FeSO
K 4 [Fe(CN)6] →
4
K 2Fe[Fe(CN)6] 2ClO 2 + 2 O 3 → Cl 2O 6 + 2O 2
Absence of air White precipitate ‘ X ’ Chlorine Chloral
dioxide perchlorate

16. (d) Brown ring is due to the formation of [Fe(NO)(H 2O)5]2+ . Oxidation state of Cl in Cl 2O 6 = 2x + 6(−2) = 0
NaNO 2x − 12 = 0
FeSO 4 →
3
[Fe(NO)(H 2O)5]2+
Slow addition of Brown ring complex 2x = 12 ⇒ x = + 6
conc. H 2SO 4
Average oxidation state of Cl in Cl 2O 6 is 6.

You might also like